Skip to main content

Full text of "C. P. A. accounting; theory, questions, and problems"

See other formats


THE  LIBRARY 

OF 

THE  UNIVERSITY 

OF  CALIFORNIA 

LOS  ANGELES 


UNlVERSin'orcALlFORNiA, 

LIBRARY, 

lUDS  ANGELES.  CALlF. 


if 


Graduate  Schol  of  Business  Administration 

University  of  California 

Los  Angeles  24,  Calif oniia 


Digitized  by  the  Internet  Archive 

in  2007  with  funding  from 

IVIicrosoft  Corporation 


http://www.archive.org/details/cpaaccountingthe01newliala 


C.  p.  A.  ACCOUNTING 

THEORY,  QUESTIONS,  AND  PROBLEMS 


BY 

GEORGE  HILLIS  NEWLOVE,  Ph.D.,  C.P.A.  o..  .c) 

RESIDENT    AUDITOR,     INCOME     TAX     UNIT,     BUREAU    OP    INTERNAL    REVENUE; 
DEAN,  WASHINGTON  SCHOOL  OF  ACCOUNTANCY;   MEMBER,  NATIONAL 
Y.    M.    C.   A.    ACCOUNTANCY    STANDARDIZATION    COMMITTEE; 
MEMBER,   AMERICAN   ASSOCIATION  OP   UNIVERSITY 
INSTRUCTORS   OP   ACCOUNTING;    LIEU- 
TENANT (j.  G.),  PAY  CORPS,  U.  S" 
N.   R.   P.    (inactive). 


VOLUME  I 


V/ACA 


ASSOCIATION    PRESS 

New  York:  347  Madison  Avenue 
1921 

40064 


Copyright,    1921.   bv 

The  Ikternationai.  Committee  of 

YouNo  Men's  Christian-  Associations 

(Printed  in  the  United  States  of  America) 

All  Rights  Resened 


Bq8.  Admii^ 
Library 

56>C>   I 


H^6 


a 


FOREWORD 


^  The  pasi  school  year— 1920-1921— closed  the  first  decade  of  the  study 
*y  of  Accountancy  in  Y.  M.  C.  A.  Schools,  with  an  enrolment  of  approxi- 
•sSraately  10,000  students.  It  is,  therefore,  quite  in  line  with  the  progressive 
•policy  of  the  I'nited  Y.  M.  C.  A.  Schools  to  begin  the  second  decade 
^  of  Association  Accountancy  instruction  by  furnishing,  for  advanced  stu- 
^  dents,  these  two  volumes  on  "C.  P.  A.  Accounting — Theory,  Questions, 
^  AND  Problems,"  by  George  llillis  Newlove,  Ph.  D.,  C.  P.  A. 

For  two   years   tlie  standaid  Accountancy   Commission  of   the   United 

•Y.  M.  C.  A.  Schools  has  been  carefully  studying  the  Accountancy  courses 

'offered    by   the    local   Association    schools,    with    a   view    to    planning    a 

(*^ standard    four- year   curriculum    that   would   be   of   maximum   service   to 

the  student,  not  only  in  giving  him  sound  instruction,  but  in  equipping 

^lim  to  i^ass  his  state  examination. 

(  These  books  by  Dr.  Newlove  aie  admirably  planned  to  furnish  the 
^instruction  and  equipment  referred  to,  for  they  are  the  outgrowth  of 
^his  own  experiences  a.s  instructor  and  Dean  of  the   Washington,  D.   C, 

School  of  Accountancy. 
^     It  will   ])e  obsei-ved   that  there  are  moi-e  problems   in   the  books   than 
©can  be  solved   within  class  periods.     In  order   to   relieve   the  instructor 
/!and  to  aid  the  student,  model  solutions  for  all  })i'o])lems  have  been  pro- 
vided, in  mimeogi'aph  form.     These  may  be  secui-ed,  at  slight  cost,  from 
•  the   Educational  Director  of  the  local  Association,  by  resident  students, 

Jfter  they  have  solved  the  problems  required  by  the  instructor.     Those 
rho  are  using  these  books,  but  are  not  in  Association  classes,  may  secure 
he  model  problem  solutioTis  from  Association  Press,  347  Madison  Avenue, 
y^ew  York  City. 

The  Accounfancy  Commission,  to  which  full  ci'edit  is  due  for  its 
eai-nest  and  jii-actical  woi-k  in  formulating  curricula,  consists  of  D.  S. 
Sylvestei-,  B.  C.  S.,  LL.  B.,  Dean,  School  of  Commerce  and  Finance, 
Northeastern  College,  Boston,  Mass. ;  H.  C.  Daines,  A.  B.,  B.  C.  S.,  P^du- 
cational  Director,  Chicago;  Ralph  B.  Mayo,  President  of  R.  B.  Mayo. 
&  Co.,  Denver;  Gr.  H.  Newlove,  Ph.  D.,  C.  P.  A.,  Dean,  Washington  School 
of  Accountancy,  Washington,  D.  C. ;  F.  L.  Roth,  Dean,  School  of  Business 
Administration,  Cleveland;  and  C.  A.  Wesp,  Dean,  School  of  Commerce, 
Philadelphia. 


PREFACE 

This  volume  together  with  Volume  II  classifies  all  the  general  accounting 
theory  and  auditing  questions  given  in  335  C.P.A.  examinations.  When 
more  than  one  examination  has  used  identical  questions  all  references  are 
given,  and  where  questions  are  practically  identical  an  asterisk  has  been 
used  after  the  reference  which  used  the  question  in  a  slightly  different 
form. 

The  lectures,  which  attempt  to  answer  the  questions,  presuppose  at  least 
two  years'  study  of  accounting  or  its  equivalent,  in  a  resident  school.  The 
elimination  of  the  elementary  subject  matter  enables  the  lectures  to  con- 
tain in  a  comparatively  brief  compass,  a  digest  of  the  advanced  theory 
and  auditing  points. 

As  the  attempted  use  of  at  least  three  references  for  each  accounting 
point  rendered  the  giving  of  proper  credit  for  direct  quotations  impossible, 
direct  quotations  have  been  avoided.  The  references  practically  amount 
lo  indirect  quotations,  however,  and  no  special  changes  in  phraseology 
have  been  made  to  conceal  this  fact.  It  is  not  claimed  that  the  bibli- 
ography is  complete,  three  references  on  undisputed  points  and  as  many 
references  as  possible  on  disputed  points  being  the  desired  goal. 

The  purposes  of  this  work  are  (a)  to  show  the  C.P.A.  candidates  what 
will  be  expected  of  them  and  to  offer  them  an  opportunity  for  further 
study  on  the  particular  points  on  which  they  are  weak,  and  (b)  to  incite 
the  universities  to  offer  post-graduate  courses  in  accounting  literature  so 
that  there  will  not  be  as  much  diversity  of  thought  in  the  accounting  books 
of  tomorrow  as  is  shown  by  the  references  to  exist  in  the  accounting  books 
of  today.  This  volume  and  Volume  IT  have  been  arranged  so  that  the 
theory  and  auditing  questions  and  the  practical  problems  on  each  subject 
will  l)e  studied  concurrently  with  the  lecture  on  each  subject. 

The  undersigned  wishes  to  express  his  appreciation  to  Mr.  Thos.  W. 
Walton  and  Mr.  Arthur  L.  Ward,  the  former  and  present  educational  sec- 
retaries. Young  Men's  Christian  Association,  Washington  D.  C,  for  assist- 
ance in  developing  the  C.P.A.  preparatory  coui-ses  given  by  the  Wash- 
ington School  of  Accountancy,  from  16  two-hour  class  sessions  with  8 
students  in  November,  1918,  to  64  two-hour  class  sessions  with  142  stu- 
dents in  June,  1921.  This  teaching  experience  and  the  opportunity  for 
two  years  to  spend  practically  my  entire  time  on  the  material  is  responsible 
for  whatever  merit  there  may  be  in  this  work.  The  undersigned  is  further 
indebted  to  Professor  R.  B.  Kester,  C.P.A.,  Columbia  University,  for  his 
valual)le  suggestions;  to  Mr.  John  Berg,  C.P.A.,  Wasliington,  D.  C,  for 
his  help  on  the  detailed  references,  and  to  Miss  Margaret  E.  Batson,  Sec- 
retary, Washington  Scliool  of  Accountancy,  for  her  help  in  editing  the 
material. 

G.  H.  Newlove. 


moj^ 

05'^'-H00'*Q0t>.r-iO<-HTfG005C^^iOC^rf"i-it>.ccOPacC^Ht^^Ol 

0261 

>^/S^^,S,S,S     '^     '  &  &  ^     -030303     -aj     ■  a  s,  s,     -cross! 

6I6I 

/■S^     '  l.Si&&     '  ^        "^  :R  ^     -050303        e3        5!S     -n     -cats 
C^  1-H      -  rH  CM  CM  CM      -  C<l -H  1— 1  CM  r- (      .  ,-1  ,-1  ^  ^  ^  ^  ^  rt      .^      •  CM  CM  CM 

8I6I            j         <^^      :           ilN-HIN      -(N      •      .|M(M^«      .      .      .^(^^^      .      .      .(^(^N 

9I6I              J  ••"  -'(N   -•';;.■  .(N^   i,-^   .:   ;^^^   :::;;   :„ 

ei6i      1      :::::::::::^;:'-^;::-^'^;-^--;--<N 

i'lei 

■  •■•■■•  r^  •-•  ,-1  ..;.■.:„  CM  ■  r-i  :::;;  c» 

8I6I 

•  •   •   •   •  ^  •   •   •  •   -  rt   .   .   .  ^   ;   .  _  CM   -'  r-i   :   '.   :   :   :  c<i 

2I6I 

::::::::::  i"^  :;::  :  i-'^  :::::::" 

1 161 

■   ••-.■•.■.   .,-,...::   .\_   ;:;::::   icM 

0161             ::::;;:::;>::;::  :^^  :::::::'" 

6061 

;;■•;;•    I"-"--  —1 ^     .  _     .    ;    :    .     I    .  _<  cm 

8061             ;;;:^:::-.::c^::::;:;^:::::.^c^ 

i06l 

;;•;•••■  -^     -     -'  CM      •■'-'-'•••  rH     :;::':  r-i  CM 

V()6I 

;;■;'-';;;;•■  -I     cm     -••-••  -h  cm 

eo6i 

•■::::::::::-,::::::::::::;._. 

1-061 

:::  i'^  :::::  i*^  ::;:::;••■■•  •  •'^<^ 

}06l 

?.0(U 

'■'■'■■'■■■'.'■'■'■'■      -Q^i 

1061 

:   :  :   :   :   :  :  :  ;   icM 

0061 

.::::::::.:  CM 

6681 

'■'■'■'■'■'■'■'■'.'.'.'.'.::   \   ...'.:.....   -.^ 

8681 

::::::::::  icM 

i68I 

:;;:::;:::■■;•;•■■•-•■■•■      -CM 

9681 

^  :::::::::::::;:::::::::::  :^ 

a; 

Vj. 

■<  1  :::::::::::::::::;:::::::  ; 

F?ox 

l^(CO5^(M(N'-iTMNc£)»C^(Nt^<N0Cr^ 

CO 

CO 

I26T 

•   oj oj83c3c3     -03     ■     -cjcS 

CO 
CM 

0S6I 

o3cS^c3c3rtaJ     •     ■   ci 

^ (N  .-1  <M  (N -H -H  ^      •      -IN 

6I6I 

8I6I 

.03           .     .     .   cS     •      •      •                    o3 

CD 
CO 

ZT6T        1 

e3     •     ■     •     ■ 

CO 

-§ 

9I6I 

^  ^  ^   :  :  rt   ■.                   :  ,-H   '■ ,-( ^    t^ 

1 

ei6i 

.        ,  T-t 1-^        •        ■        ■        •>— Ir-t 

^ 

cS 

^161 

■    T-i    T-t           •           ■    T-< 1— ll-H 

CO 

l_] 

8I6I 

::  ^   '.:  ^  ■.:.:;'.■.:::  CM 

CO 

< 

SI6I 

fH 1—1        •        •»— 1 

t^ 

< 

II6T 

::::  i-^  ::::::  :^  :  :-" 

t^ 

S 

0T6I 

•       .1— 1 T-l       •       •  r-l 

00 

6061 

wi T-l 

Oi 

8061 

::-::-:;::::::::- 

^ 

Ph 

Z06I 

o 

rH  r-H                                                                            f— < 

55 
O 

9061 

•      •      ■      -      -(N      - 1-H 

o 

IZi 

H- 1 

S06T 

::::  :^   :::::::::  :^ 

■* 

W6l 

:::::-::::::::::: 

t^ 

S 

£061 

•      •■•      -(N      •■■•••■■•      -^ 

«o 

2061 

r-t 

CO 

T06I 

.— 1 

CO 

o 

0061 

C^      ■'■'■'■'.'■'■'■'.'.      '. 

■<*< 

o 

6681 

::::  :^  ::::::::::  : 

(N 

868T 

CM 

w 

1^ 

Z68I 

CM 

CQ 

-sJ 

H 

9681 

- 

5 

"3 

e2 

o  S 

^^ 
°°  tL 

<P  (1. 

-     §  a; 
1— I      "^  -tJ 

(H  03 

0)  -»^ 
ft  CO 

SJ.   - 

■•^  c 

S  § 


CONTENTS 


PAGE 

CHAPTER  I— Field  of  Accountancy 1 

Bookkeeping;  Accounting;  Installing  Systems;  Auditing;  Objects  of 
Auditing ;  Errors  of  Principle ;  Mechanical  Errors ;  Detection  of  Fraud ; 
Qualifications  of  Auditors;  Responsibility  of  Auditor;  Attitude  of 
Auditor;  Auditor  as  Witness;  Internal  Check. 


CHAPTER  II— Double  Entry  Bookkeeping 16 

Double  Entry  Bookkeeping;  Accounts;  Classes  of  Accounts;  Nominal 
Accounts;  Controlling  Accounts;  Suspense  Accounts;  Arrangements  of 
Accounts;  Numbered  Accounts;  Journalizing;  Verification  of  Footings 
and  Postings;  Trial  Balance;  Books;  Voucher  System;  Vouchers; 
Articulation  Statement. 


CHAPTER  III— Balance  Sheet 42 

Balance  Account;  Balance  Sheet;  Form  of  Balance  Sheet;  Model  Balance 
Sheet;  Valuation  in  Balance  Sheet;  Condensed  Balance  Sheets;  Com- 
parative Balance  Sheets ;  A  ssets ;  Kinds  of  Assets ;  Kinds  of  Liabilities ; 
Accrued  Items ;  Deferred  Items ;  Contingent  Items ;  Capital ;  Proprietor- 
ship Accounts;  Relation  of  Proprietor  to  Business. 


CHAPTER  IV— Profit  and  Loss  Statement 67 

Income;  Revenue;  Earnings;  Profits;  Revenue  and  Capital  Receipts; 
Revenue  and  Capital  Expenses;  Revenue  and  Capital  Expenditures; 
Fixed  Charges  aiul  Opei-ating  Expenses;  Summaries  of  Nominal  Ac- 
counts; Account  Foim  of  Profit  and  Loss  Statement;  Report  Form  of 
Profit  and  Loss;  Statements  of  Non-Trading  Concerns;  Comparative 
Profit  and  Loss  Statements ;  Use  of  Schedules  in  Profit  and  Loss  State- 
ments; Working  Sheet. 


X  CONTENTS 

CHAPTER    V— Partnership    at    Organization 86 

Partnei-ship  Defined;  Kinds  of  Partnerships;  Advantages  and  Disadvan- 
tages of  Partnership;  Liability  of  Partners;  Authority  of  Partners; 
Articles  of  Copartnership;  General  Partnership  Rules;  Partners'  Pro- 
prietorship Accounts;  Loans  of  Partners;  Opening  Entry  for  Partner- 
ship Books;  Guaranteed  Assets;  Buying  Interest  in  Business;  Buying 
Share  in  Profits;  Audit  of  Partnerships;  Joint  Stock  Companies. 

CHAPTER  VI— Partnership   During   Operation 101 

Division  of  Profits ;  Interest  on  Capital ;  Interest  on  Excess  or  Deficit  of 
Capital;  Interest  on  Drawings;  Partners'  Salaries;  Tabular  Form  of 
Partners'  Accounts. 

CHAPTER  VII— Partnership  at  Liquidation 113 

Causes  of  Dissolution;  Adjustment  upon  Partner's  Death;  Liquidating 
Partners;  Value  of  Assets  at  Dissolution;  Liquidation  of  Uncompleted 
Contracts;  Reserves  at  Dissolution;  Application  of  Assets  at  Dissolu- 
tion; Losses  and  Expenses  of  Liquidation;  Purchasing  Partner's  In- 
terest; Personal  Insolvency  of  One  Partner;  Illustrative  Problem; 
Capital  Deficit;  Goodwill  in  Liquidation;  Closing  Partnership  Books 
after  Sale;  Liquidation  by  Installments. 

CHAPTER  VIII— Corporation  Stock 126 

Corporation  Defined;  Classes  of  Corporations;  Advantages  and  Disad- 
vantages of  Incorpoiation;  Proeeduie  of  Incorporation;  Charter;  By- 
Laws;  Stockholders;  Directors;  Minute  Books;  Corporate  Officers; 
Capital  Stock ;  No  Par  Stock ;  Kinds  of  Stock  with  Par  Value ;  Capital 
Stock  on  Balance  Sheet;  Stock  Certificate  Book;  Stockholders'  Ledger; 
Opening  Entries  for  Corporation;  Subscriptions;  Subscription  Ledger; 
Transfer  Journal ;  Installments ;  Installment  Ledger ;  Installment  Scrip 
Book;  Forfeited  Stock;  Payment  for  Stock;  Premium  and  Discount  on 
Stock;  Donated  Stock;  Bonus  Stock;  Redemption  of  Stock;  Audit  of 
Capital  Stock;  Organization  Expense;  Investments  in  Stocks. 

ClIAl'TER   IX— Interest 102 

Interest  and  Discount;  Prepaid  Interest;  Accrued  Interest;  Intere.st  Ex- 
pense and  Income;  Calculation  of  Interest;  Interest  on  Partial  Pay- 


CONTENTS  xi 

merits;  Audit  of  Interest;  Interest  as  a  Cost  of  Manuf actiii'e ;  Compound 
Interest  Processes ;  Processes  Involving  Interest ;  Processes  Involving 
Discount;  Valuation  of  Serial  Bonds;  Valuation  of  Short  Terminal 
Bonds;  Amortization  of  Bond  Premium  and  Discount;  Interest  Bearing 
Debt  Paid  by  Equal  Annual  Installments;  Bond  Purchased  at  Inter- 
mediate Dates ;  Optional  Bond  Redemption ;  Computing  an  Unknown 
Rate ;  Computing  Elfective  Bond  Rates ;  Computing  an  Unknown  Time ; 
Amount  of  Annuities  with  Interim  Interest  Dates;  Present  Worth  of 
Annuities  Avith  Interim  Interest  Dates;  Present  Worth  of  Deferred 
Annuities;  Amount  of  Annuities  Due;  Present  Worth  of  Annuities  Due. 


CHAPTER  X— Reserves  and  Funds 180 

Classes  of  Reserves;  Asset  Valuation  Reserves;  Operating  Reserves; 
Appreciation  Reserves;  Contingency  Reserves;  Secret  Reserves;  Open 
Proprietorship  Reserves;  Funds;  Reserve  Funds;  Sinking  Fund;  En- 
tries for  Sinking  Fund ;  Reserve  for  Sinking  Fund. 


CHAPTER   XI— Bonds 202 

Classes  of  Bonds ;  Unissued  Bonds ;  Treasury  Bonds ;  Bonds  as  Collateral ; 
Issue  of  Bonds;  Bond  Interest;  Bond  Register;  Coupon  Register;  Audit 
of  Bonds  Payable;  Investments  in  Bonds. 


CHAPTER  XII— Dividends  and   Surplus 219 

Classes  of  Dividends;  Dividends  out  of  Capital;  Dividends  out  of  Profits; 
Declaration  of  Dividends;  Payment  of  Dividends;  Cumulative  Divi- 
dends; Dividend  Book;  Audit  of  Dividends;  Profit  Sharing;  Undivided 
Profits  Account;  Surplus  Defined;  Kinds  of  Surplus;  Statement  of 
Surplus;  Audit  of  Surplus;  Deficit. 


CHAPTER  XIII— Trading  Accounts 238 

Old  Merchandise  Account ;  Pui'chase  Records ;  Unrecorded  Purchase 
Orders;  Sales  Records;  Cash  Sales;  (LO.D.  Sales;  Installment  Sales; 
Sales  for  Future  Delivei-y;  Sales  to  Branches;  Consignment  Sales; 
Sales  of  Sci-ap  or  By-Products;  Sales  to  Proprietor;  Sales  on  Approval ; 
Sales  of  Fixed  Assets:  The  Turnover;  Merchandise  Expense;  Returns 
on  Sales  and  Purchases;  Rebates  and  Allowances;  Guaranteed  Sales; 
Sales  in  Transit;  Book  and  Physical  Inventories;  Verifying  Inventories; 
Trade  Discount;  Cash  Discount  on  Purchases;  Cash  Discount  on  Sales; 
Neglected  Discounts;  Reserve  for  Cash  Discounts. 


xii  CONTENTS 

CHAPTER  XIV — Insurance  and  General  Expense 26G 

Unexpired  Insurance;  Fire  Insurance  Register;  Reserve  for  Insurance; 
Audit  of  Fire  Insurance;  Coinsurance  Clause;  Fire  Loss;  Marine  In- 
surance; Life  Insurance;  Employers'  Liability  Insurance;  Burglary  In- 
surance; Accrued  and  Prepaid  Wages;  Payroll  Book;  Audit  of  Pay- 
roll; Insurance,  Rent,  and  Taxes  as  Costs;  Rent  as  Income;  Postage; 
Donations;  Adjustment  Entries. 

CHAPTER  XV— Depreciation    287 

Nature  of  Depreciation;  Causes  of  Depreciation;  Inadequacy;  Obso- 
lescence; Depletion;  Actual  and  Theoretical  Depreciation;  Appreciation 
and  Depreciation;  Booking  of  Depreciation;  Depreciation  Fund;  De- 
preciation Rates;  Plant  Ledger;  Calculation  of  Depreciation;  Straight 
Line  Method;  Working  Hours  Method;  Composite  Life  Method;  Service 
Output  Method;  Fixed  Percentage  on  Diminishing  Value  Method;  Sum 
of  Year  Digits  Method ;  Sinking  Fund  Method ;  Annuity  Method ;  Main- 
tenance Method;  Replacement  Method;  The  Fifty  Per  Cent  Method; 
Appraisal  Method;  Insurance  Method;  Gross  Earnings  Method. 

CHAPTER  XVI— Manufacturing  Accounts 309 

Nature  and  Function  of  Cost  Accounting;  Components  of  Cost  Data; 
Relation  of  Co^t  to  General  Accounting;  Comparison  of  Cost  and  Non- 
Cost  Systems ;  Kinds  of  Cost  Systems ;  Control  of  Material ;  Control  of 
Labor  Costs ;  Wage  Systems ;  Control  of  Manufacturing  Expenses ;  By- 
products; Inventories, 


CONTENTS  xiii 

APPENDIX 

KEY  TO  REFERENCES 

PAGK 

A  to  Q 339 

R  to  Z 340 

AA  to  MM 340 

NN  to  ZZ 341 

AAA  to  GGG 341 

HHH  to  ZZZ 342 

AAAA  to  WW 343 

WWWW  to  ZZZZ 344 

AAAAA  to  HHHHH 344 

REFERENCES  FOR  CHAPTERS 

Reperencks  For  Cfiapter  I    345 

References  For  Chapter  II    346 

References  For  Chapter  III    347 

References  For  Chapter  IV    349 

References  For  Chapter  \    350 

References  For  Chapter  VI    351 

References  For  Chapter  VII 351 

References  For  Chapter  VIII    352 

References  For  Chapter  IX   355 

References  For  Chapter  X    357 

References  For  Chapter  XI   358 

References  For  Chapter  XII    , 359 

References  For  Chapter  XIII 360 

References  For  Cjiapter  XIV  363 

References  For  Chapter  XV   364 

References  For  Chapter  XVI   366 


C.   p.   A.   ACCOUNTING 


CHAPTER  I 

FIELD  OF  ACCOUNTANCY 

Bookkeeping — Bookkeeping  may  be  defined  as  the  art  of  keeping  a 
systematic  record  of  business  transactions/ 

There  are  two  systems  of  bookkeeping,  namely:  (1)  Single  entiy  (some- 
times called  simple  entry),  the  system  wherein  entries  of  transactions  do 
not  require  e(iual  charges  and  credits  for  every  exchange,''  and  where  in  its 
simplest  form,  transactions  are  shown  only  in  the  relation  they  bear  to 
persons;'  and  (2)  double  entry,  the  system  wherein  two  equal  records  are 
made  for  each  transaction,  one  a  charge  and  the  other  a  credit  to  the  re- 
spective accounts  affected/ 

Accounting — Accounting  may  be  defined  as  the  science  which  treats  of 
the  systematic  record,  compilation,  and  presentation  in  a  comprehensive 
manner  for  administrative  purposes  of  the  financial  opeiations  of  a  busi- 
ness organization.''  While  accounting  includes  bookkeeping,  it  does  not 
stop  with  the  mechanical  recording  of  any  given  set  of  facts,  but  embraces 
the  designing  of  bookkeeping  forms  and  methods  and  the  presentation  of 
the  data  gathered  by  the  bookkeeper  in  the  form  of  statements  for  the 
infoi'mation  of  the  management." 

Installing  Systems — The  work  of  installing  a  system  for  an  industry 
requires  first,  a  knowledge  of  mechanics  of  the  industry;  second,  a  knowl- 
edge of  the  financial  and  legal  status  of  the  industry,  and  third,  a 
knowledge  of  the  men  who  will  operate  the  system.'  The  first  is  needed  to 
have  the  data  classified  according  to  the  operations  of  the  business,  the 
second  so  that  the  interests  of  the  firm  may  be  safeguarded  and  all  legal 
requirements  met,  and  the  third  in  order  that  the  system  may  be  operated 
by  the  men  available.  This  third  point  is  so  important  that  accountants 
specializing  in  system  work  usually  arrange  to  have  the  clients'  staff  in- 
structed in  the  operation  of  the  system. 

Auditing — Auditing  is  the  art  of  verifying  the  work  incident  to  the 
record,  compilation,  and  presentation  of  the  facts  concerning  business 
transactions.* 

Large  firms  have  "inside"  auditors  who  regularly  audit  their  books  and 
documents.  This  excellent  practice  has  the  effect  of  discouraging  fraudu- 
lent practices  and  of  increasing  the  care  with  which  the  records  are  kept. 
However,  even  if  "inside"  auditors  are  employed,  the  affairs  should  be 
regularly  investigated  by  a  professional  auditor,  who,  removed  from  local 
influences,  can  be  relied  upon  to  treat  matters  in  a  disinterested  manner, 
and  whose  wider  experience  makes  him  capable  of  advising  his  client  as 

'For  explanation  of  superior  figures  see  page  337. 

1 


■2  C.  P.  A.  ACCOUNTING 

to  the  probable  effect  of  practices  which  lie  condemns  and  those  which  he 
recomnu nds.*  Piofessional  auditors  f i-equently  are  able  from  their  experi- 
ence to  develop  a  percentage  table  based  upon  net  sales  which  shows  the 
normal  ratio  of  the  various  items  in  the  profit  and  loss  statement.  A 
(•umi')arison  of  the  actual  with  the  theoretical  percentages  frequently  leads 
to  discovery  of  leaks  and  over-expenditures. 

Objects  of  Auditing — The  three  objects  of  an  audit  usually  given  are  the 
discovery  of  errors  of  principle,  the  elimination  of  mechanical  errors,  and 
the  detection  of  fraud.'"  To  these  a  fourth  and  fifth  object  may  be  added, 
namely,  the  ascertaining  of  ways  of  improving  the  accounting  system  and 
office  organization  methods,"  and  the  preparation  of  analytical  and  un- 
biased statements  showing  the  business  conditions."  The  last  object  is 
the  most  important,  as  the  correct  statements  aid  (a)  in  the  securing  of 
short  time  credit,  (b)  in  floating  bond.s,  (c)  in  sale  of  the  business,  (d)  in 
adjustments  between  partners,  and  (e)  in  the  settlement  of  fire  loss  claims." 

Errors  of  Principle — As  mistakes  in  principle  are  usually  more  im- 
portant than  mechanical  errors  or  even  fraud,  the  auditor  must  be  con- 
stantly guarding  against  them."  As  errors  of  principle,  such  as  capitaliz- 
ing revenue  expenditures,'"''  omitting  a  proper  allowance  for  depreciation," 
etc.,  are  usually  caused  by  ignorance  and  because  the  errors  usually  make 
out  the  profits  to  be  higher  than  they  reallj'  are,  it  is  frequently  difficult  to 
convince  the  management  that  they  are  wrong.'' 

Mechanical  Errors — It  must  not  be  assumed  that  mechanical  eiTors  are 
due  to  fraud,'"  frequently  they  are  caused  by  the  ignorance  or  cai'elessne.ss 
of  the  bookkeeper. 

It  is  important  to  remember  that  incorrect  books  frequently  balance  as 
the  errors  otfset  one  another."'  This  fact  shows  the  danger  of  the  practice 
of  writing  off  a  small  error  in  the  trial  balance.""  However,  an  auditor 
lays  him.self  open  to  just  criticism  if  he  devotes  a  considerable  part  of  his 
time  searching  for  clerical  erroi's.^' 

Methods  for  locating  errors  in  a  ti-ial  balance  are  too  numerous  to 
mention,  especially  tis  they  ai-e  largely  guesswork.  Auditors  usually  try 
to  avoid  auditing  the  books  when  they  are  mechanically  incorrect,  as  they 
realize  that  their  clients  cannot  afford  to  pay  professional  fees  for  such 
mechanical  services."  However,  frequently  auditors  are  compelled  to 
audit  books  which  are  out  of  balance,  in  which  ca.><es  they  usually  depend 
upon  either  blocking  or  analvzing  the  ledger,  procedures  discussed  in 
Chapter  XIII,  Volume  II. 

Detection  of  Fraud — The  business  world  still  holds  the  discovery  of 
fraud  as  the  chief  aim  of  an  audit."'  Although  this  idea  is  incorrect,  it 
forces  the  auditor  to  pay  special  attention  to  this  feature,  for  even  though 
an  auditor  has  left  no  stone  unturned  in  his  search  for  fraud,  his  practice 
will  be  hurt  if  fraud  is  later  discovered,  even  though  he  is  not  legallv 
liable." 

There  is  one  feature  about  fraud,  however,  that  aids  the  auditor,  namely, 
Ihc  fact  that  each  case  of  fraud  is  usually  concealed  in  the  same  way,  due. 


FIELD  OF  ACCOUNTANCY  3 

no  doubt,  to  the  desire  of  the  culprit  to  simplify  the  continuation  of  the 
defalcation  and  the  fear  of  trying  a  new  method  of  concealment  when  one 
has  already  ])roved  so  far  successful."''  For  this  reason  the  auditor  is 
always  suspicious  of  any  unusual  vai'iations  in  the  accounts. 

Qualifications  of  Auditors — The  C.  P.  A.  boards  by  their  requirements 
show  the  legal  qualiiications  to  be  a  thorough  knowledge  in  accounting 
and  auditing  theory  and  practical  accounting  and  a  fair  knowledge  of 
commercial  law  coupled  with  a  high  school  education  and  several  years 
of  experience  as  a  junior  accountant.  There  are  two  additional  qualiiica- 
tions which  the  legal  requirements  cannot  reach— namely,  thorougimess 
and  imagination.'"'  Notliing  short  of  exact  correctness  can  be  accepted  by 
a  good  auditor.  The  faculty  for  imagination  comes  into  operation  when 
the  auditor  visualizes  tlie  accounting  system  in  search  for  possible  oppor- 
tunities for  fraud,  or  when  the  auditor  tries  to  work  out  in  his  imagina- 
tion the  operation  of  the  accounting  system  he  is  creating. 

Responsibility  of  Auditor — Legally,  an  auditor  does  not  insure  the 
accuracy  of  the  books  he  audits  or  the  statements  he  compiles,  but  is 
merely  liable  for  reasonable  care  in  his  investigation."  For  instance,  an 
auditor  is  not  liable  for  accepting  as  a  valid  voucher  a  document  not 
representing  value  actually  expemled,  if  the  true  situation  could  not  be 
ascertained  by  reasonable  carefulness.  Where  there  is  nothing  to  arouse 
suspicion,  a  fcAv  cases  at  haphazard  would  be  a  reasonable  check,  but,  when 
there  are  grounds  for  suspicion,  a  much  more  careful  scrutiny  must  be 
given."  Legally,  "reasonable"  care  and  skill  require  more  than  an  audit 
of  the  books,  the  auditor  being  held  for  "reasonable"  care  to  ascertain 
that  tlie  books  show  the  true  status  of  the  firm.'* 

The  moral  responsibility  of  auditors  greatly  exceeds  their  legal  liability.'* 
Sentiment  has  been  growing  that  investigations  should  be  so  thorough  that 
the  statements  will  show  the  entii-e  truth  about  the  business,  and  it  is  a 
matter  of  pride  to  the  profession  that  American  accountants  have  not 
accepted  the  limited,  though  legal,  views  of  their  responsibility." 

The  delegation  of  the  detailed  work  to  junior  accountants  does  not  re- 
lieve the  auditor  from  his  personal  responsibility.^'  This  is  I'eeognized 
by  the  courts,  who  will  pei'mit  a  senior  accountant  to  testify  in  regard  to 
facts  discovered  by  the  junior  accountants  working  under  his  personal 
supervision."'  As  he  cannot  excuse  himself  by  throwing  the  blame  on  his 
suboi'dinates,  a  careful  auditoi'  keeps  in  close  touch  with  the  detailed  work 
by  requiring  frequent  reports  showing  the  progress  made  in  conducting 
the  audit  according  to  the  auditing  program  which  he  has  carefully  de- 
vised.'* j  .\ 

The  auditor  is  responsible  foi-  the  reporting  of  all  illegal  acts  of  officers 
and  directors,  but  questions  of  the  wisdom  of  the  action  of  these  officials, 
imless  they  constitute  a  violation  of  accounting  principles,  are  outside  the 
scope  of  the  auditor's  responsibility."" 

Attitude  of  Auditor — In  preserving  his  complete  independence  the 
auditor  must  be  cai-eful  not  to  offend  his  client  by  too  much  assertiveness.** 
As  he  cannot  compel  the  adoption  of  his  improvements,  the  auditor  should 


4  C.  P.  A.  ACCOUNTING 

use  tact,  and,  l)y  avoiding  arguments  on  trivial  points  not  involving  prin- 
ciple, strengthen  his  ])osition  for  the  important  points  which  may  come 
under  consideration/' 

Cooperation  with  the  client's  statT  may  be  gained  by  avoiding  parades  of 
authority  and  by  showing  a  desire  lo  lessen  or  improve  the  quality  of  the 
work.'*  The  auditor  and  his  assistants  should  treat  the  clerks  with  due 
courtesy,™  but  no  intercourse  beyond  strictly  business  arrangements  should 
be  allowed/"     Tact  in  the  use  of  books  when  they  are  idle  is  advisable.*' 

Auditor  as  Witness — The  auditoj*,  as  a  witness,  must  tell  the  truth  and 
nothing  but  the  truth,  bvit  only  in  answer  to  direct  questions.*'  He  should 
not  volunteer  information  either  favorable  or  unfavorable  to  his  client.*' 
His  impartiality  on  the  witness  stand  sliould  not  deter  him  from  assisting 
his  client's  lawyer  to  determine  what  questions  should  be  a.sked  and  what 
subjects  should  be  avoided.**  It  is  therefore  necessary  that  auditors  have 
sufficient  knowledge  of  the  laws  of  evidence  to  know  what  answ'ers  are 
admissible.*'' 

Before  an  auditor  can  testify  as  an  expert  accountant  it  is  necessary  to 
have  his  qualifications  established  by  answers  to  his  client's  attorney.  The 
auditor,  by  concise,  modast  answers,  should  relate  his  experience  and 
reputation  as  a  public  accountant.*"  Before  entei-ing  the  court  room,  it 
is  important  that  all  the  statements  and  schedules  not  prepai'ed  by  the 
auditor  himself  should  be  ex.amined  with  the  assistant  who  did  the  work 
and  verified  by  reference  to  the  books  of  original  entry  and  supporting 
vouchers.*'  If  memoranda  and  references  in  his  own  handwriting  are  made, 
the  auditor  will  qualifj'  as  a  witness.** 

In  testifying,  the  auditor  should  prevent  his  being  placed  in  the  position 
of  advocate  for  his  client.**  On  the  stand,  the  auditor  is  in  much  the 
position  of  a  teacher,  for  he  must  biing  his  points  within  the  reach  of  the 
juj'y  who  are  untrained  in  accounting  principles.'""  He  must,  on  the  other 
hand,  avoid  in  his  effort  for  simplicity  covering  up  his  point  in  a  mass 
of  verbiage."  An  accountant  with  legal  knowledge  can  serve  his  client 
by  acting  as  an  interpi-eter  between  his  client's  lawyer  and  the  jury  of 
business  men,  for  lawyers,  from  their  legal  viewpoint,  frequently  ask 
questions  so  phrased  that  the  ordinary  jui'or  cannot  grasp  the  accounting 
point  involved. 

An  auditor  must  recognize  the  law  of  expert  testimony  which  requires 
that  the  testimony  must  be  the  best  available  for  each  item."  He  can,  if 
given  access  to  the  records,  give  an  opinion  as  to  the  value  of  goodwill 
based  upon  earning  power."  He,  however,  could  not  as  an  accountant  say 
that  the  plant  should  be  valued  at  cost  less  depreciation,  as  the  valuation 
of  fixed  assets  is  the  work  of  an  appraiser." 

Internal  Check — Internal  check  is  the  method  by  which  employees 
check  the  records  of  one  another,  and  the  entire  control  does  not  rest  in 
any  one  clerk."  Such  a  system  eliminates  the  probability  of  fraud'*  and 
accordingly  gi-eatly  lessens  the  amount  of  detailed  checking  which  must 
be  done."'     As  most  fraud  consists  either  in  the  misappropriation  of  cash 


FIELD  OF  ACCOUNTANCY  5 

or  mercliandise,  these  two  points  are  especially  guarded  by  the  system  of 
internal  check. 

The  receipt  of  cash  should  be  safeguarded  by  having  all  incoming  mail 
opened  and  listed  by  an  official  other  than  the  cashier  or  bookkeeper."^ 
This  practice,  if  cash  sales  are  made  and  recorded  by  parties  wdthout 
access  to  the  cashier's  depai'tment,  will  prevent  the  manipulation  of  cash 
receipts.  Cash  payments  should  be  made  by  check'"  and  supported  by 
a  duly  authorized  voucher.™  Small  payments  should  be  made  out  of  a 
petty  cash  fund  set  aside  for  the  purpose."^  The  cashier  should  not  have 
access  to  the  personal  accounts,'"  nor  to  bills  submitted  to  customers."'' 
Payrolls  should  be  changed  only  upon  written  authority"*  and  each  step 
in  their  preparation  should  be  checked  by  some  one  not  directly  in  charge 
of  the  work."  Payments  should  be  made  in  the  presence  of  a  witness,  who, 
together  with  the  paymaster,  should  sign  the  payroll."" 

The  purchase  of  materials  should  be  safeguarded  by  careful  records  of 
receipts"'  and  by  filing  of  filled  orders  to  prevent  passing  of  duplicate 
invoices."*  Invoices  should  be  checked  as  to  number  and  quality  of 
received  goods,  and  as  to  extensions,""  and  when  check  is  drawn  therefor, 
notation  should  be  made  on  invoice  to  prevent  duplicate  payment.'"  Sales 
invoices  should  be  cheeked  like  purchase  invoices"  and  also  against  the 
customer's  order.'^  Systematic  records  should  be  kept  of  all  orders  re- 
ceived and  shipments  made.'^  Records  of  returned  sales'*  and  returned 
purchases''  should  be  kept.  x\llowances  should  be  granted  only  upon 
written  approval  of  proper  authoi'ity.'"  Perpetual  inventories  should  be 
maintained"  which  should  be  verified  by  physical  inventories  from  time 
to  time." 


C.  p.  A.  ACCOUNTING 


QUESTIONS 


field  of  accountancy 

Bookkeeping 

1.  Define:  Bookkeeping.  (N.  Y.,  Jan.,  1900;  Wash.,  May,  1003; 
N.  J.,  1904-1909*;  Mich.,  July,  190G;  N.  Y.,  Oct.,  1907;  R.  I.,  Dec., 
1907.*) 

2.  Define:  Double-entry  bookkeejjing:.  (Pa.,  Nov.,  1S99* ;  N.  Y.,  Jan., 
1900*;  Pa.,  May,  1905;  Md.,  Jan.,  1909;  Ohio,  Nov.,  1913*;  S.  C,  Sept., 
1919.*) 

3.  State  various  kinds  of  bookkeeping  with  explanations.  (Mich., 
July,  1906.) 

4.  State  the  essential  piineiples  of  double-entry  bookkeeping.  (N.  Y., 
Dec,  1896;  N.  Y.,  Jan.,  1897;  Wash.,  May,  1903;  N.  Y.,  June,  1909;  Va., 
Nov.,  1910;  Wash.,  June,  1912*;  W.  Va.,  May,  1917.) 

5.  Under  what  conditions  does  double-entry  bookkeeping  become  an 
exact  science?     (N.  Y.,  June,  1917.) 

6.  Name  three  objects  of  bookkeeping.  (N.  Y.,  Oct.,  1907* ;  R.  I., 
Dec,  1907*;  Mich.,  June,  1912.) 

Accounting 

7.  Define:  Accounting.  (N.  Y.,  Jan.,  1900;  Wash.,  May.  1903;  N.  J., 
1904-1909*;  Ohio,  Nov.,  1913;  Ohio,  Nov.,  1915;  Ind.,  June,  1916*; 
N.  C,  Aug.,  1917.*) 

8.  What  is  a  certified  public  accountant?  (Wash.,  May,  1911;  Iowa, 
Dec,  1918.) 

9.  What  is  the  relation  of  the  accountant  to  the  bookkeeper?  (N.  Y., 
Jan.,  1900*;  Wash.,  May,  1903*;  Ind.,  June,  1916.) 

10.  State  the  fundamental  objects  of  accounting.  (Ohio,  Nov.,  1913*; 
Ohio,  Nov.,  1915*;  N.  C,  Aug.,  1917*;  Iowa,  Dec,  1918.) 

11.  As  a  certified  public  accountant  and  auditor,  what  would  be  your 
relation  with  your  client?     (N.  J.,  1904-1909.) 

12.  Ai*e  the  theory  of  accounting  and  the  theory  of  common  law 
based  in  any  respect  on  the  same  principle?  Which  is  the  more  reliable 
as  to  facts?     (N.  Y.,  June,  1917.) 

13.  (a)  Why  should  we  favor  the  stand jnlization  of  accounting 
terminology  and  forms?     (6)     What  means  would  you  suggest  as  best 


FIELD  OF  ACCOUNTANCY  7 

adapted  to  obtain  such  uniformity?  (c)  Give  two  examples  of  ac- 
counting terms  that  are  ambiguous.  Suggest  remedies.  (N.  Y.,  June, 
1915.) 

14.  If  you  were  employed  by  a  firm  or  corporation  to  install  a  system 
of  accounting,  what  would  be  your  procedure?  (Pa.,  Nov.,  1899*;  Ind., 
June,  1916;  N.  Y.,  Jan.,  1917.*) 

15.  Enumerate  the  fundamental  principles  to  be  followed  in  devising 
an  accounting  system  for  a  large  mercantile  company.  (N.  Y.,  Jan., 
1904*;  Ohio,  March,  1910*;  La.',  May,  1913*;  Mo.,  Dec,  1913;  S.  C, 
Sept.,  1919.*) 

16.  In  devising  a  system  of  accounts  for  a  business,  what  are  the 
main  subjects  for  consideration,  and  in  what  order  should  they  have 
attention?     (N.  Y.,  Dec,  1896.) 

Auditing 

17.  Define:  Auditing.  (N.  Y.,  Dec,  1898*;  N.  Y.,  June,  1899*;  Pa., 
Nov.,  1899*;  N.  Y.,  Jan.,  1900;  Pa.,  Nov.,  1900*;  Pa.,  Nov.,  1901*; 
Wash.,  May,  1903;  Pa.,  May,  1903*;  Md.,  Oct.,  1903;  N.  J.,  1904-1909*; 
Mich.,  Nov.,  1907*;  Mass.,  June,  1910*;  N.  Y.,  June,  1911.) 

18.  What  are  the  objects  to  be  attained  by  an  audit?  (N.  Y.,  June, 
1897;  N.  Y.,  Dec,  1898*;  Wash.,  May,  1903;*  Mass.,  June,  1910*;  Mich., 
June,  1910*;  N.  Y.,  Jan.,  1911*;  Cal.,  May.  1916*:  Cal.,  Nov.,  1916*; 
N.  D.,  July,  1916*;  W.  Va.,  May,  1917*;  W.  Va.,  May,  1919*;  S.  C, 
Sept.,  1919.*) 

19.  What  are  the  advantages  of  an  audit?  (Cal.,  May,  1916;  Cal., 
Nov.,  1916;  Cal.,  June,  1917.*) 

20.  Explain  the  value  of  an  outside  audit  for  a  business  having  a 
regularly  emi>loyed  auditor.     (Mich.,  June,  1913.) 

21.  State  what  are  the  advantages  of  the  certified  public  accountant 
(C.  P.  A.)  degree  to:  (a)  the  accountant,  (h)  the  client.  (Wash.,  Aug., 
1908*;  Wash.,' May,  1911.) 

22.  In  large  businesses  internal  auditors,  members  of  the  staff  of  the 
concern,  are  frequently  the  only  ones  employed.  Where  this  is  the  case, 
do  you  think  it  desirable  that  professional  certified  public  accountants 
should  be  engaged?  If  so,  give  reasons.  (TIL,  Nov..  1908;  Ind.,  June, 
1916.*) 

23.  What  meaning  attaches  to  the  term  "auditing"  other  than  the  re- 
view and  verification  of  accounts  of  past  transactions  /    (M.  Y.,  Jan.,  1907.) 

24.  State  generally  your  idea  of  your  duties  as  an  auditor  and  the 
points  to  be  covered  by  an  audit.     (Pa.,  Nov.,  1906.) 

25.  Is  criticism  the  chief  duty  of  *an  auditor?  What  additional  points 
should  be  covered  in  his  report?     (Minn.,  Oct.,  1916.) 

26.  Wherein  do  the  functions  of  accountants  and  auditors  differ? 
(Cal.,  June,  1904*;  W.  Va.,  May,  1917;  W.'Va.,  May;  1919.) 


8  C.  P.  A.  ACCOUNTING 

27.  Outline  the  duties  of  an  auditor.  (N.  Y.,  Dee.,  1896;  N.  Y.,  June, 
1898*;  N.  Y.,  Dec.  1898*;  Pa.,  Nov.,  1900*;  N.  Y.,  June,  1901*;  Pa., 
Nov.,  1901*;  Pa.,  May,  1902*;  Cal,  June,  1904*;  N.  Y.,  Jan.,  1907; 
N.  Y.,  June,  1909*;  Mass.,  June,  1910*;  N.  Y.,  June,  1910*;  Va.,  Nov., 
1910*;  Mich.,  June,  1912*;  La.,  May,  1913*;  Ga.,  May,  1919*;  N.  D., 
July,  1919.*) 

28.  What  qualifications  must  an  auditor  possess?     (N.  Y.,  Dec.,  1898 
Pa.,  Nov.,  1899 ;  N.  Y.,  June,  1900* ;  Pa.,  Nov.,  1900 ;  N.  Y.,  June,  1901 
Pa.,  Nov.,  1901;  Pa.,  May,  1903;  Wash.,  May,  1903;  Md.,  Oct.,  1903 
Cal.,  June,  1904;  Mich.,  Nov.,  1907;  Wash.,  Aug.,  1908;  Md.,  Jan.,  1909- 
Mich.,  June,  1912*;  La.,  May,  1913*;  N.  D.,  July,  1919.*) 

29.  State  the  training  necessary  to  fit  an  auditor  for  the  discharge  of 
his  duties.     (N.  Y.,  June,  1900;  Mich.,  June,  1912.*) 

30.  To  what  extent  and  for  what  purposes  should  an  auditor  be 
familiar  with :  (a)  law;  (b)  algebra;  (r)  economics;  {d)  financial  history? 
(Cal.,  Nov.,  1916.) 

31.  Is  it  an  auditor's  duty  to  concern  himself  to  any  extent  with  the 
validity  of  the  transactions  that  come  under  his  notice?  Explain.  (N.  Y., 
June,  1900;  Wash.,  May,  1903;  111.,  Nov.,  1903.*) 

32.  To  whom  is  an  auditor  responsible?    (S.  C,  Sept.,  1919.) 

33.  What  are  the  responsibilities  of  an  auditor?  (N.  Y.,  Dee.,  1896*; 
N.  Y.,  June,  1901*;  Pa.,  May,  1902;  Pa.,  May,  1903;  N.  J.,  1904-1909*; 
N.  Y.,  Jan.,  1907*;  Wash.,  Aug.,  1908;  Md.,  Jan.,  1909;  N.  Y.,  June, 
1909*;  Mass..  June,  1910;  Va.,  Nov.,  1910*;  Mich.,  June,  1912*;  La., 
May,  1913*;  Ga.,  May,  1919*;  N.  D.,  July,  1919.*) 

34.  To  what  extent  is  an  auditor  morally  responsible  for  his  certifi- 
cate?    (N.  Y.,  Dec.,  1898;  Wash.,  May,  1903.) 

35.  To  what  extent  is  an  auditor  legallv  responsible  for  his  certifi- 
cate?    (N.  Y.,  Dec,  1898;  Wash.,  May,  1903;  Wash.,  March,  1909.) 

36.  What  are  the  duties  and  responsibilities  of  an  auditor  in  the 
following  circumstances : 

(a)     Where  the  system  of  internal  check  is  defective? 

(&)  Where  the  transactions  for  the  year  under  review  are  correctly 
recorded  but  the  auditor  has  reason  to  believe  that  the  property  accounts, 
Goodwill  account,  and  Surplus  account  of  prior  years  (not  audited)  have 
been  manipulated  or  improperly  handled? 

(c)  Where  restrictions  are  placed  on  an  audit  by  one  of  the  prin- 
cipal officers? 

(111.,  Dec,  1918.) 

37.  To  what  extent  should  an  auditor  be  governed  by  instructions? 
(Pa.,  May.,  1902.) 

38.  Give  at  length  your  views  on  the  ethics  of  accountancy.  (N.  Y., 
June,  1912.) 

39.  Name  the  character  of  in-egularities,  the  evidence  of  which  your 
audit  would  disclose.     (N.  C,  June,  1920.) 


FIELD  OF  ACCOUNTANCY  9 

40.  Define  and  illustrate:  (a)  Technical  error;  (b)  error  of  principle; 
(c)  compensating  error.     (Kan.,  May,  1916.) 

41.  What  is  the  principal  effect  of  errors  of  principle?  In  what  part 
of  the  accounts  are  they  most  apt  to  be  found?     (Minn.,  Oct.,  1916.) 

42.  The  A  1  Manufacturing  Company  employs  a  staff  of  bookkeepers. 
The  head  bookkeeper  has  not  studied  the  theoiy  of  accounting,  neither  has 
his  experience  been  extensive.  He  understands  double-entry  bookkeeping, 
but  he  cannot  be  described  as  a  well  trained  accountant. 

Previously  the  A  1  Manufacturing  Company  has  been  satisfied  with 
annual  accounts  prepared  by  competent  auditors.  They  decide  to  have 
monthly  statements  of  accounts  in  future,  and  wilh  this  end  in  view  instruct 
the  head  bookkeeper  to  prepare  statements  on  the  same  lines  as  the  last 
annual  accounts  were  prepared. 

State  the  imaginary  errors  of  principle  that  might  not  unreasonably  be 
found  upon  an  expeii  examination  of  his  works. 

(111.,  May,  1916.) 

43.  In  auditing  a  client's  boolcs  you  are  requested  to  note  any  defects 
in  the  system  of  accounting  and  submit  suggestions  for  its  improvement. 
Make  a  brief  report  to  your  client  acsordingly.  For  this  purpose  suggest 
the  remedying  of  some  of  the  defects  which  you  have  most  frequently 
observed  in  your  own  experience.  (Wash.,  March,  1909;  Wash.,  May, 
1911.) 

44.  What  is  your  idea  of  the  proper  and  a  just  charge  or  compensation 
(a)  to  the  employer  and  the  employee,  for  your  services  as  a  certified 
public  accountant;  (b)  to  the  services  of  a  first  assistant;  (c)  to  other 
assistants.     (N.  J.,  1904-1909.) 

Internal  Check 

45.  Define:  Internal  check.  (Fla.,  April,  1908*;  Colo.,  Dec,  1913; 
Mich.,  Dec.,  1914;  Ohio,  Nov.,  1918;  111.,  Dec.,  1918;  Ohio,  Nov.,  1919.) 

46.  State  what  you  consider  the  best  system  of  bookkeeping  to  guard 
against  speculations  and  what  regulations  you  would  reconunend  for  the 
conduct  of  the  financial  affaire  of  the  concern.     (La.,  May,  1913.) 

47.  What  plan  would  you  suggest  to  minimize  the  risk  of  fraud  in  the 
following:  (1)  Payrolls;  (2)  accounting  of  cash  receipts;  (3)  invoices  of 
purchases.     (Colo.,"^Dec.,  1913;  111.,  Dec,  1918.*) 

48.  How  is  the  position  of  an  auditor  affected  if  the  system  of  the 
concern  under  audit  is  defective,  (a)  as  to  internal  check;  (&)  as  to  cost 
methods?     (A.  I.  of  A.,  May,  1918.) 

49.  Trace  the  various  operations  in  a  well  regulated  wholesale  mer- 
cantile office  from  the  time  an  order  is  given  for  the  purchase  of  material 
until  such  material  is  paid  for,  to  protect  the  company  from  any  possible 
loss  in  the  transaction.     (Colo.,  Dec,  1913.) 

50.  Name  the  ten  matters  of  special  importance  in  devising  any  system 
of  internal  cheek  in  the  handling  of  office  records.     (Mich.,  June,  1910.) 


10  C.  p.  A.  ACCOUNTING 

51.  Outline  a  system  of  internal  cheek  for  a  v>'holesale  grocery  concern 
doing  a  business  of  $3,000,000  a  year,  with  about  2,000  customers.  The 
system  should  coordinate  with  an  annual  audit  by  professional  account- 
ants.    (A.  I.  of  A.,  Nov.,  1919.) 

52.  A  clothing  manufacturer  has  in  an  adjacent  city  a  separate  retail 
establishment  for  the  sale  of  his  goods.  The  whole  of  the  clerical  work 
and  cash  transactions  are  in  the  hands  of  one  clerk.  Suggest  the  method  of 
bookkeeping  which,  in  your  opinion,  will  afford  the  best  check  from  the 
auditor's  point  of  view  and  at  the  same  time  be  thoroughly  practical.  (Pa., 
Nov.,  1906.) 

53.  You  are  asked  to  offer  suggestions  regarding  a  system  of  internal 
check  for  a  company  whose  business  is  that  of  wholesale  merchants.  De- 
scribe the  office  methods  you  would  introduce  to  safeguard  the  funds  and 
merchandise  of  the  company.     (111.,  Dec,  1910*;  ^yash.,  Nov.,  1913.) 

54.  The  accounting  department  of  a  jobbing  house,  consisting  of  5 
men,  keeps  the  books,  does  the  billing,  makes  city  collections,  handles  the 
general  and  petty  cash  and  pays  the  invoices  of  the  eompanj\  On  what 
general  lines  would  you  distribute  the  work  of  the  department  to  secure 
the  best  internal  check?  (Md.,  Jan.,  1909*;  Mass.,  Oct.,  1915;  Ind.,  Nov., 
1917.) 

55.  What  condition  of  office  organization,  above  all  others,  leads  to 
fraud  and  defalcation  by  bookkeeper  and  cashiers?  Support  your 
opinion.     (N.  Y.,  June,  1912.) 

56.  Where  would  irregularities  be  most  likely  to  be  discovered  in  a 
detailed  audit  of  a  men's  furnishing  store  which  employs  10  salesmen? 
In  your  answer  assume  that  the  books  are  kept  according  to  a  good  system 
of  accounting  and  that  the  bookkeeper  has  complete  charge  of  the  office 
with  only  the  occasional  supervision  of  the  owner.    (111.,  May,  1917.) 

57.  Outline  a  .system  for  handling  the  accounts  of  a  large  retail  store, 
explaining  specially  how  you  would  ari'ange  to  guard  against  possible 
dishonestv  of  such  emplovees  as  have  access  to  valuable  jewels  and  plate. 
(111.,  May,  1907.) 

58.  Under  what  circumstances  can  a  bookkeeper  cover  up  forced  foot- 
ings and  postings  during  the  course  of  an  audit,  so  that  oidinarily  you 
would  not  detect  the  same  unless  certain  precautions  are  taken?  State 
what  these  precautions  should  be.     (Colo.,  Dec.,  1913.) 

59.  Give  six  typical  examples  of  fraud,  of  which  only  four  involve  the 
abstraction  of  actual  money,  and  explain  shortly  what  means  you  would 
suggest  to  reduce  the  risk  of  loss  under  each  of  these  headings  to  a  mini- 
mum.   (111.,  May,  1909.) 

60.  The  General  Ledger  of  a  trading  concern  contains,  in  addition  to 
the  usual  Balance  Sheet  and  Profit  and  Loss  accounts,  pei-sonal  accounts 
of  officers  whose  salaries  are  credited  and  drawings  charged  thereto.  In 
addition  to  currency,  drawings  on  signed  and  numbered  vouchei-s,  they  are 
all  privileged  to  pay  private  bills  with  company  checks,  and  also  to  pur- 
chase company  merchandise  for  personal  use. 


FIELD  OF  ACCOUNTANCY  11 

The  General  Ledger  and  General  Journal  are  kei>t  by  and  in  full  charge 
of  one  of  the  officers.  This  officer  handles  no  funds  of  the  company.  The 
Ledger  is  in  balance;  there  are  no  errois  in  the  footings  of  the  Ledger 
accounts ;  the  cash  at  banks  and  in  hand  is  correct ;  and  this  officer's 
account  shows  a  small  credit  balance. 

You  find  that  periodical  statements  prepared  by  this  officer  are  accepted 
without  question;  and  that  the  several  amounts  stated  thereon  are  in 
agreement  with  balances  of  the  respective  accounts  in  the  General  Ledger. 

(a)  State  three  ways  by  which  this  officer,  without  collusion,  might 
defraud  the  company. 

(b)  State,  brittly,  the  way  you  would  adjust  any  differences  that 
might  be  disclosed;  and 

(c)  What  preparation  you,  as  a  })ublic  accountant,  would  make  to 
clearly  state  the  facts  and  prove  the  defalcation  in  event  you  are  re- 
quired to  give  expert  testimony  in  court  proceedings. 

(Wis.,  April,  1915.) 

61.  Name  five  common  methods  of  defalcation  and  explain  the  steps 
which  you  would  suggest  to  detect  each  of  them.     (M'ch.,  June,  1919.) 

62.  What  system  would  you  recommend  to  a  factory  for  the  proper 
checking  of  all  labor  emi)loyed : 

(a)  To  obtain  complete  record  of  each  emj^loyee  from  date  of  applica- 
tion for  employment  to  date  of  discharge? 

(h)     To  prove  that  all  labor  paid  for  was  actually  accounted  for? 

(c)  That  no  mistakes  had  been  made  in  figuring  the  time  or  in  the  pay- 
ing off? 

(d)  That  the  payroll  had  not  been  padded  in  any  manner? 

Mention  the  forms,  mechanical  appliances,  etc.,  required  for  such  system 
and  the  independent  internal  checks  necessary  to  prevent  collusion  between 
clerks  and  employees. 

(Mich.,  July,  1909.) 

63.  Discuss  fully  an  effective  method  of  controlling  raw  material  for  a 
mill  or  factory.  Select  any  types  of  mill  or  factory  you  desire.  Illustrate 
by  sketches  any  forms  not  clearly  described  by  their  titles.  (Pa,,  Nov., 
1919.) 

64.  Where  embezzlement  of  cash  by  usual  means  is  impossible,  how 
can  fraud  be  perpetuated  by  manipulation  of  accounts  and  records  result- 
ing ultimately  in  a  large  loss?     (N.  Y.,  Jan.,  1916.) 


12 


C.  P.  A.  ACCOUNTING 


PROBLEMS 


FIELD  OP  ACCOUNTANCY 


1.  From  the  following  accounts  appearing  on  the  trial  balance  prepare, 
without  using  figures,  statements  which  you  consider  best  calculated  to  set 
forth  the  operations  of  the  year  and  the  financial  position  at  December 
31,  1916,  assuming  that  you  are  preparing  these  statements  on  behalf  of 
a  bank  which  desired  paper  available  for  rediscount  with  the  Federal 
Reserve  Bank: 


Accounts  payable. 

Accounts  receivable. 

Advertising. 

Buildings. 

Capital  stock. 

Capital  stock  unsubscribed. 

Cash  on  deposit. 

Commissions  paid  salesmen. 

Depreciation,  buildings,  1916. 

Depreciation,  machinery,  1916. 

Discount  allowed  on  sales. 

Discount  received  on  purchases. 

Doubtful  accounts  receivable. 

Factory  expense. 

Finished  goods,  inventory,  December 
:U,  1915. 

Freight  and  cartage  inward. 

Freight  and  cartage  outward. 

Material  purchased. 

Mortgage  on  plant. 

Notes  payable. 

Notes  receivable. 

Office  expenses. 

Office  furniture  and  fixtures. 

Offi.ce  payroll. 

Organization  expenses  (to  be  distrib- 
uted over  three  years  from  Jan.  1, 
1916). 

Payroll  factory,  accrued. 

Petty  cash. 

Prepaid  taxes,  real  estate. 

Profit  and  loss,  1915  surplus. 


Eepairs,  buildings. 

Fuel. 

Goodwill. 

Insurance,  buildings  and  machinery. 

Insurance,  finished  goods. 

Insurance,  unexpired,  buildings  and 
machinery. 

Insurance,  unexpired,  finished  goods. 

Interest  accrued  on  investments. 

Interest  accrued  on  mortgage  payable. 

Interest  paid. 

Interest  received. 

Investments. 

Labor  factory  payroll. 

Land. 

Machinery 

Material  inventory,  December  31,  1915. 

Repairs   machinery. 

Reserve  for  bad  and  doubtful  ac- 
counts. 

Reserve    for    depreciation,   mnchinery. 

Reserve  for  depreciation,  buildings. 

Returns  and  allowance  on  sales. 

Salaries  of  general  officers. 

Sales. 

Salesmen 's  accounts,  advanced  on  sal- 
aries. 

Subscriptions  and   donations. 

Taxes,  income,  U.  S. 

Taxes,  real  estate. 

Work  in  progress  inventory,  Decem- 
ber 31,  1915. 


The  inventories  December  31.  1916,  not  on  the  books  were:     Finished 
goods, ;  mateiial,  work  in  progress, . 

(It  is  assumed  that  the  la.st  inventory  is  intended  to  mean  both  the 
inventory  of  material  and  that  of  work  in  process.)     (A.  I.  of  A  ,  Nov 
1917.) 

2.    Following  is  a  list  of  the  accounts  appearing  on  the  trial  balance 
cf  a  manufacturing  company  which  deals  in  finished  merchandise  pur- 


FIELD  OF  ACCOUNTANCY 


13 


chased,  as  well  as  in  its  own  products.  From  the  list,  and  without  using 
figures,  draw  up  plans  of  financial  statements  (balance  sheet,  manufac- 
turing account,  profit  and  loss  account,  etc.)  in  the  form  which  you  think 
most  suitable: 


Accounts  payable. 

Salaries,  management. 

Capital  stock. 

Notes  receivable. 

Cash. 

Notes  payable,  partly  secured  by  deed 

of  trust. 
Salaries,  office  and  store. 
Real  estate. 
Fuel. 

Insurance  plant. 
Light. 

Freight  on  merchandise  purchased. 
Machinery  and  tools. 
Freight  on  raw  materials. 
Printing  and  stationery. 
Accounts  receivable. 
Horse,  wagon,  and  harness. 
Stable  expense. 
Advertising. 

Purchases,  raw  material. 
Productive  labor,  factory. 
Machinery  repairs. 
Office  furniture. 
Reserve  uncollectable  accounts. 
Reserve  for  depreciation. 
Insurance  merchandise. 
Uncollectable  accounts. 


Buildings. 

Sales,  own  products. 

Inventory,  own  products. 

Inventory,  raw  materials. 

Inventory,  partly  manufactured  goods. 

Inventory,  merchandise  purchased. 

Inventory,  repair  supplies. 

Undivided  profits,  end  of  last  year. 

Purchases,  merchandise. 

Sales,  merchandise  purchased. 

Rent,  factory. 

Rent,  store  and  office. 

Discount  on  sales,  own  product. 

Interest  payable. 

Depreciation,  building,  machinery, 
wagons  and  harness. 

Sundry  factory  expenses. 

Sundry  office  expenses. 

Postage. 

Subscriptions  and  donations. 

Discount  on  purchases,  merchandise. 

Rents,  income. 

Insurance  paid  in  advance,  plant. 

Labor,  warehouse. 

Insurance  paid  in  advance,  merchan- 
dise. 

Management,  salary,  factory. 

Management,  salary,  office. 


Salesmen's  expenses  and  salaries. 

(Mich.,  Dec.,  1906*;  Mo.,  Dec.,  1913;  Ohio,  Nov.,  1915.) 

3.  You  are  given  a  trial  balance  of  a  wholesale  jobbing  corporation 
embodying  the  following  accounts,  and  ai*e  requested  to  prepare  a  Balance 
Sheet  and  Profit  and  Loss  statement.  Prepare  such  statement,  without  the 
use  of  figures,  showing  the  form  in  which  you  would  present  them. 

Real  Estate,  Capital  Stock,  Traveling  Expense,  Sinking  Fund,  Reserve 
for  Depreciation,  Taxes  (State  and  City),  Accounts  Receivable  (Cus- 
tomers'), Sales,  10-years  debenture  Insurance  Policy,  Insurance  (all  ex- 
pired), Purchuoes,  Depieciation,  Accrued  Interest  on  Bonds,  Notes  Re- 
ceivable (Customers'),  United  States  Income  and  Excess  Profits  Taxes 
Paid,  Furniture  and  Fixtures,  Surplus,  Liberty  Bonds,  Overdrafts  on 
Salary  Accounts  of  Officers,  Reserve  for  Bad  Debts,  Trade  Acceptances 
Receivable,  Bond  Interest,  Petty  Cash  Fund,  Auto  Trucks,  General  Ex- 
pense, Collections  on  Bad  Debts  Previously  Charged  Off,  Notes  Receivable 
Discounted,  Merchandise  Inventory  (at  beginning  of  period — the  amount 
of  inventory  at  the  date  of  trial  balance  is  also  given  you),  Returns  and 
Allowances  on  Sales,  Interest  Received  on  Liberty  Bonds,  Discounts  Re- 
ceived on  Purchases,  Dividends  Paid,  Trade  Acceptances  Payable,  Dis- 
counts Allowed  on  Sales,  Notes  Payable  Officers,  and  Treasury  Stock. 

(Va.,  Nov.,  1918.) 


u 


C.  p.  A.  ACCOUNTING 


4.  The  Spark  Plug  and  Auto  Supply,  Inc.,  is  the  manufacturer  of  a 
patented  spark  plug  and  is  also  dealer  in  automobile  supplies.  From 
the  following  trial  balance  (as  of  October  31,  1919)  and  information 
prepare  Balance  Sheet  and  Profit  and  Loss  statements  showing  cost  of 
manufacture  of  spark  plugs  and  gross  and  net  profit  on  sales : 


Advertising 

Accounts  Receivable 

Accounts  Payable 

Bills  Receivable 

Bills  Payable  Trade  Creditors 

Bills  Payable  First  National  Bank 

Bonds  5%  First  mortgage 

Building  Factory 

Bad  Debts  written  off 

Capital  Stock,  common  (authorized  $250,000)  Fully 

paid 

Capital  Stock,  6%  preferred,  authorized  and  issued — 

Dividend  preferred  stock 

DeUvery  Expenses 

DeUvery  Equipment  and  Trucks 

Directors'  Fees 

Discount  on  Sales 

Freight,  Raw  Materials 

Freight,  Automobile  SuppUea 

Finished  Goods 

First  National  Bank  Current  Account 

General  Expenses 

Goods  in  Process 

Heat,  Light  and  Power 

Interest  on  Bonds 

Insurance  and  Taxes,  Factory 

Labor,  Productive 

Labor,  Nonproductive 

Liberty  Bonds 

Loose  Tools 

Machinery  and  Plant 

Office  Furniture  and  Fixtures 

PajToll 

Patent  Rights 

Purchases,  Raw  Materials 

Purchases,  Automobile  Supphes 

Repairs 

Rent,  Warehouse 

Reserve  for  Depreciation,  Buildings 

Reserve  for  Depreciation,  Machinery 

Reserve  for  Bad  Debts 

Real  Estate,  Factory  Site 

Shop  Supplies  and  Expenses 

Surplus 

Sales,  Spark  Plugs 

Sales,  Automobile  Supplies 

Salaries,  Office  and  General 

Salaries,  Salesmen 

Travehng  Expenses 


$  26,450 
180,105 

35,000 


225,000 
7,850 


18,000 

7,140 

9,250 

2,500 

12,200 

12,050 

2,345 

34,320 

51,850 

14,770 

13,250 

22,200 

9,375 

17,400 

233,846 

99,444 

195,000 

15,270 

165,090 

1,200 

30,000 
450,960 
141,690 

14,050 
3,875 


150,000 
15,560 


14,£00 
34.600 
22,300 

52,288,440 


$2,288,440 


FIELD  OF  ACCOUNTANCY  15 

Inventories,  November  1,  1918:  Raw  materials,  $14,500;  automobile 
supplies,  $22,450. 

Inventories,  October  31,  1919:  Raw  materials,  $27,300;  automobile 
supplies,  $19,200;  finished  goods,  $50,400;  goods  in  process,  $17,205; 
loose  tools,  $10,500. 

Reserve  for  bad  debts  to  be  adjusted  to  5  per  cent  of  open  accounts. 

Depreciation  for  the  12  months  ended  October  31,  to  be  allowed  as 
follows :  Factory  buildings,  2  per  cent ;  machinery,  5  per  cent ;  delivery 
equipment,  10  per  cent ;  furniture  and  fixtures,  $200.  Disregard  fractional 
parts  of  a  dollar.  Patent  rights  expire  October  31,  1925;  advertising, 
$950,  applies  to  next  season;  taxes  on  factory  buildings  accrued,  $1,400. 
First  mortgage  5  per  cent  gold  bonds  are  a  first  charge  on  all  the  assets 
of  the  company.  Interest  payable  quarterly  on  the  first  of  February, 
May,  August  and  November.  [Calculate  depreciation  on  diminishing 
values.] 

(A.  I.  of  A.,  Nov.,  1919.) 


Note:  Model  solutions  of  the  problems  presented  in  this  book  may 
he  secui-ed  by  students  in  Y.  M.  C.  A.  classes,  on  application  to  the 
instructor.  Those  who  are  not  students  in  Association  classes  may  secure 
the  Model  Solutions  from  Association  Press,  347  Madison  Avenue,  New 
York. 


CHAPTER  II 
DOUBLE-ENTRY  BOOKKEEPING 

Double  Entry  Bookkeeping — As  the  name  implies,  double  entry  book- 
keeping is  based  on  the  principle  of  two  entries  for  each  business  trans- 
action, a  debit  and  a  credit/  It  presupposes  an  initial  equation  at  the 
inception  of  the  bu.siness  between  the  money  or  property  furnished  by  the 
proprietor  and  the  proprietor's  interest  in  the  business/  Starting  with 
this  initial  equilibrium  or  balance  of  both  sides  of  the  accounts,  each 
subsequent  transaction  affects  the  two  sides  of  the  equation  alike  necessi- 
tating for  each  debit  enti-y  a  credit  entrj'  of  equal  amount,  thus  evidenc- 
ing that  no  matter  what  transactions  occur,  if  they  are  recorded  by  a 
double  entry,  the  oi'iginal  equilibrium  will  not  be  destroyed/ 

Accounts — An  account  is  a  record  under  a  specific  heading  of  a  single  or 
group  of  either  similar  or  dissimilar  items  relating  to  the  same  person  or 
thing/  For  convenience  the  account  is  divided  into  two  parts  or  sides  in 
which  the  similar  items  are  giouped/  As  each  of  the  sides  of  the  account 
is  opposed  to  the  other,  the  rule  for  joui-nalizing,  or  entering  trans- 
actions into  accounts,  is  to  debit  or  enter  on  the  left  side  of  the  account 
all  increases  of  assets  or  decreases  of  liabilities  and  proprietorship,  and 
to  credit  or  enter  on  the  right  side  of  the  account  all  decreases  in  assets 
or  increases  in  liabilities  and  pi-oprietorship/  If  the  balance  of  an 
account,  the  difference  between  the  sum  of  the  debits  and  the  sum  of  the 
credits,  is  a  debit,  then  the  account  is  an  asset,  expense,  or  deficit,  while 
accounts  with  credit  balances  represent  liabilities,  income,  or  proprietor- 
ship/ 

Classes  of  Accounts — Classification  of  accounts  usually  divides  the 
accounts  into  the  following  classes : 

1.  Personal  or  accounts  showing  the  effect  of  the  transactions  on  the 
vaiious  people  or  corporations  with  whom  the  fii'm  deals/ 

2.  Impersonal  or  accounts  recording  profits,  losses,  receipts,  disburse- 
ments, and  non-personal  assets  and  liabilities/  The  limiting  of  the  term 
"impersonal  accounts"  to  real  accounts'"  seems  fallacious. 

3.  Real  or  accounts  recording  assets,  liabilities,  and  vested  proprietor- 
ship/' 

4.  Nominal  (sometimes  called  representative)"  or  accounts  recording 
income  and  expenses/' 

5.  Mixed  or  accounts  containing  both  real  and  nominal  elements/* 

6.  Major  (sometimes  called  primary)"  or  accounts  recording  all  of  the 
transactions  of  a  particular  class/" 

'For  explanation  of  superior  figures  see  page  337. 

16 


DOUBLE-ENTRY  BOOKKEEPING  17 

7.  Subsidiary  ov  aoeounts  auxiliary  to,  though  not  necessarily  rlepenfl- 
ent  upon,  a  major  account." 

8.  Summary  or  accounts  summing  up  the  data  contained  in  a  number 
of  other  accounts  of  different  classes  to  display  some  fact." 

9.  Specific  or  accounts  containing  items  of  only  an  exact,  particular 
nature." 

10.  Controlling  (sometimes  called  collective)'"'  or  accounts  containing 
the  totals  of  the  debits  and  credits  of  a  number  of  accounts  in  order  to 
show  at  any  time  the  balance  of  the  aggregate  of  these  accounts.^*  Con- 
trolling accounts  are  sometimes  called  summary  accounts,"^  but  summary 
accounts  combine  dissimilar  accounts,  and  controlling  accounts  do  not. 

The  purpose  of  classification  of  accounts  is  to  clarify  the  financial  state- 
ments by  grouping  similar  accounts  together.  Another  use  is  to  place  all 
the  subsidiary  accounts  in  schedules  and  place  only  the  major  accounts  in 
the  statements. 

Nominal  Accounts — If  nominal  accounts  were  not  used  an  entry  to 
vested  capital  would  be  necessary  with  every  income  or  expense  trans- 
action.'^'' This  is  impractical,  for  the  exact  profit  on  each  sale  is  seldom 
known,  at  the  time  of  the  sale,  and  it  is  further  inadvisable,  for  the 
administration  requires  statistics  showing  the  detailed  effect  of  the  various 
transactions  upon  the  business  and  not  just  the  net  result."  Accordingly 
items  of  income  and  expense  are  classified  as  minutely  as  the  administra- 
tion desires  and  then  these  nominal  accounts  are  closed  through  the  sum- 
mary accounts  into  the  vested  proprietorship  accounts.^" 

Controlling  Accounts — Controlling  accounts  are  used  because  they 
enable  daily  or  monthly  financial  statements  to  be  prepared  without  waiting 
for  the  balancing  of  all  the  subsidiary  accounts  and  greatly  reduce  the 
work  of  locating  errors  in  posting.^"  The  subsidiary  ledger  can  be  made 
self-balancing  by  an  "adjustment"  account,  which  is  an  exact  copy  of  the 
controlling  account  in  the  general  ledger  except  it  is  reversed." 

Special  columns  for  the  controlling  accounts  are  made  in  the  books  of 
original  entry  from  which  posting  is  done  to  the  subsidiary  accounts."* 
The  posting  to  the  subsidiary  accounts  is  done  as  usual  and  then  the  totals 
of  such  debits  and  credits  are  posted  to  the  controlling  account."  As  the 
controlling  accounts  and  not  the  subsidiary  accounts  are  entered  in  the 
trial  balance  of  the  general  ledger,  the  books  will  balance.'" 

All  posting  to  controlling  accoiints  should  be  checked  by  the  auditor.'* 
Then  the  subsidiary  ledger  balance  should  be  compared  with  the  balance 
of  the  controlling  account.'^  If  these  balances  agree  only  a  test  check  of 
the  cases  at  random  is  necessary  for  the  satisfactory  audit  of  the  sub- 
sidiary accounts.'' 

Suspense  Accounts — An  account  used  as  a  place  for  temporary  records 
of  items  pending  determination  of  their  final  allocation  is  a  suspense 
account.'*  The  usual  examples  of  suspense  accounts  are  unlocated  errors 
in  trial  balance,  cash  received  without  name  of  sender,  and  discrepancies 
between  the  bank's  records  and  the  firm's  cash  book.'"    It  is  important  that 


18  C.  P.  A.  ACCOUNTING 

each  suspense  item  be  placed  in  an  aeeonnt  under  a  suitable  title  instead 
of  general  suspense  accoimt.'" 

When  it  becomes  apparent  that  the  cause  of  the  suspended  items  cannot 
be  discovered,  such  item  should  be  closed  into  pi-oflt  and  loss.''  However, 
in  a  C.  P.  A.  problem  where  the  books  are  out  of  balance,  the  necessary 
adjustment  cannot  be  investigated  and  the  suspense  account  should  be 
carried  on  the  balance  sheet  instead  of  being  closed  into  profit  and  loss. 
This  is  especially  true  if  the  balance  is  a  credit. 

Some  bu.siness  men  carry  their  doubtful  accounts  and  notes  receivable 
in  a  suspense  ledger.'*  This  provides  a  convenient  way  of  keeping  track 
of  these  items,  especially  if  the  accounts  ai-e  kept  in  loose-leaf  binders.* 
However,  as  such  a  transfer  does  not  affect  the  lx>ok  valuation  of  the 
items,  it  must  not  be  used  as  an  argument  for  omitting  the  reserve  for 
doubtful  accounts.*' 

Arrangements  of  Accounts — The  accounts  should  be  arranged  in  such  a 
manner  so  as  to  facilitate  the  preparation  of  the  financial  statements." 
Such  an  arrangement  would  list  the  assets  in  the  order  of  their  liquidity, 
each  valuation  account  following  its  related  asset;  and  would  list  the 
liabilities  likewise  followed  in  order  by  the  vested  proprietorship  accounts, 
the  profit  and  loss  account,  and  the  expense  and  income  accounts  in  the 
order  in  which  they  appear  in  the  profit  and  loss  statement.^  If  eon- 
trolling  accounts  are  not  maintained,  the  customers'  and  creditors'  accounts 
would  be  placed  in  the  back  of  the  ledger."  A  trial  balance  from  a  ledger 
whose  accounts  are  arranged  as  mentioned  above  is  called  a  classified  trial 
balance."  Sometimes,  for  reference  purposes,  the  accounts  are  arranged 
alphabetically." 

Numbered  Accounts — In  some  large  businesses  the  accounts  are  so 
numerous  that  they  are  given  numbers  as  well  as  names.  This  is  especially 
important  where  the  accounts  are  used  for  detailed  statistical  purposes  by 
having  the  numbers  of  the  accounts  punched  on  the  cards,  together  with 
the  data. 

There  are  several  good  systems  of  numbering  accounts.  A  good  general 
idea  of  these  plans  can  be  had  from  the  study  of  one  of  the  simplest  which 
arranges  the  accounts  into  nine  fundamental  gi-oups,  as  follows  :*^ 

1.  Asset  Accounts.  6.     Valuation  Reserves. 

2.  Investments  and  Treasury  Securities.         7.     Capital  Surplus. 

3.  Prepaid  Items.  8.     Income  Accounts. 

4.  Liability  Accounts.  9.     Expense  Accounts. 

5.  Accrued  Items. 

These  general  groups  may  be  subdivided  into  more  special  groups  if 
desired.    An  example  of  this  subdivision  would  be : 
9.     Expense  Accounts. 
91   ^Manufacturing. 
911  Prime. 

9111  Wages. 

9112  Freight. 


DOUBLE-ENTRY   BOOKKEEPING  19 

912  Overhead. 

92  Selling  Expenses. 

93  Administrative  Expense. 
931  Salaries. 

9311  Clerks. 

9312  Stenographers. 

94  Non-opei-ating  Expenses. 

The  above  system  is  especially  good  as  it  emphasizes  the  major  divisions 
of  accounts.  A  bookkeeper  Avonld  hardly  post  a  debit  to  an  asset  to  an 
account  beginning  with  9.  Another  advantage  is  that  the  general  correct- 
ness of  the  system  could  be  checked  by  an  official  with  only  a  general 
knowledge  of  the  business,  for  a  cursory  survey  of  the  books  would  detect 
the  capitalization  of  an  expense. 

A  system  very  similar  to  the  above  is  one  that  has  only  three  grand 
divisions,  viz.,  property,  proprietorship,  and  revenue  accounts,  the  initial 
numbers  being  respectively  1,  2,  and  3,  and  the  liabilities  being  differenti- 
ated from  the  assets  by  having  the  second  number  2  instead  of  1,  as  in  the 
ease  of  assets.^' 

In  some  systems  key  letters  are  also  used  with  the  numbers  to  aid  in 
rapid  identification  and  to  facilitate  the  operation  of  the  system.**  In 
others  each  class  of  accounts  is  given  the  numbers  starting  with  a  certain, 
hundred,  viz.,  100  to  290  for  plant  and  equipment,  700  to  800  for  de- 
ferred charges,  and  1700  to  1800  for  factory  overhead,  etc.* 

Journalizing — Journalizing  is  the  systematic  classification  of  the  debits 
and  credits  of  a  business  transaction."  In  double  entry  bookkeeping 
there  are  two  axioms  for  journalizing,  namely,  that  the  sum  of  the  debits 
equals  the  sum  of  the  credits,  and  that  all  entries  are  journal  entries." 
Formerly  the  words  "to"  and  "by"  were  almost  universally  used."  A 
few  accountants  still  use  the  word  "to"  although  they  do  not  use  the  word 
"by.""  However,  the  majoiity  no  longer  advocate  the  use  of  either  of 
the  words." 

Another  recent  change  in  journalizing  is  the  dropping  of  the  practice 
of  making  the  entries  closing  the  nominal  accounts  at  the  end  of  the  fiscal 
period  with  red  ink  posting  directly  from  one  account  to  another,  the 
practice  generally  followed  now  being  to  have  the  entries  entered  in  the 
journal  and  posted  in  black  ink.'*' 

The  verification  of  journal  entries  is  very  important  as  the  journal  is 
sometimes  used  fraudulently  by  making  fictitious  or  irregular  entries  to 
personal  accounts  to  conceal  misappropriation  of  cash."  All  journal 
entries  to  personal  accounts  should  be  substantiated  by  vouchers  approved 
by  authorized  offteials."  If  they  have  not  been  so  approved,  the  auditor 
should  have  them  initialed." 

Verification  of  Footings  and  Postings — As  experience  has  shown  that 
the  percentage  of  frauds  which  have  been  concealed  in  false  posting  and 
incorrect  footings  is  small,  auditors  should  not  vei'ify  every  posting  and 
every  footing,  biit  should  depend  on  tests  to  locate  such  frauds." 


20  C.  P.  A.  ACCOUNTING 

Posting  to  the  ledgers  should  be  verified  by  working  back  from  the  ledger 
to  the  book  of  original  entry  and  not  rice  versa,  as  this  procedure  pre- 
vents anyone  fi-om  tampering  with  the  auditor's  check  marks,  and  because 
it  will  enable  the  aiulitor  to  detect  entries  made  in  the  accounts,  which 
were  not  made  on  the  books  of  original  entry."^ 

If  there  are  controlling  accounts,  it  will  not  be  necessary  to  verify  the 
detailed  posting  to  the  subsidiary  accounts."  If  there  are  no  controlling 
accounts,  the  auditor  should  create  them."''  Even  if  it  is  deemed  wise 
to  verify  the  postings  to  the  subsidiary  accounts,  all  of  the  accounts 
should  not  be  checked.''  In  periodical  audits  a  certain  number  of  the 
subsidiary  accounts  should  be  verified  each  audit  so  that  all  of  the  accounts 
would  be  verified  in  a  reasonable  time.*^ 

As  to  the  footings,  checking,  in  addition  to  the  last  page  of  each  month, 
about  every  tenth  page  in  the  purchase  reeords*^'  and  every  eighth  page 
in  the  sales  records  would  be  satisfactory  if  the  firm  is  large.'"  In  an 
audit  of  a  small  concern,  the  footing  of  every  sixth  page  of  the  purchase 
journals"'  and  every  fourth  page  of  the  sales  journals'^  should  be  verified. 
If  all  receipts  are  deposited  in  the  bank  and  all  disbursements  made  by 
check,  no  footing  need  be  done  in  the  general  cash  book,  provided  the 
disbursements  have  been  vouched  and  the  bank  account  reconciled.'*  If 
the  cash  book  footings  can  not  be  verified  by  the  bank  account,  the  net 
cash  and  discount  columns  on  both  sides  are  usually  added  for  the  entire 
period.  Some  auditors  only  add  every  third  or  fourth  page  of  the  cash 
disbursements,  if  checks  are  always  used,"*  but  add  all  receipt  columns,  if 
the  cashier  has  access  to  the  receipts."  In  verifying  additions  special  care 
should  be  taken  of  the  amounts  carried  forward." 

Trial  Balance — A  trial  balance  is  a  table  of  the  balances  shown  on  a 
ledger."  Its  purpose  is  to  prove  the  mathematical  accuracy  of  the  ledger, 
but  mathematical  equilibrium  is  not  in  itself  sufficient  proof  of  correct- 
ness, as  frequently  errors  offset  each  other.'*  While  a  trial  balance  does 
not  prove  the  correctness  of  the  ledger,  the  failure  to  balance  does  prove 
incorrectness."  This,  together  with  the  fact  that  the  trial  balance  places 
the  accounts  in  a  convenient  form  for  the  making  of  the  financial  state- 
ments, is  the  value  of  taking  a  trial  balance.'"  Before  closing,  the  trial 
balance  of  the  general  ledger  contains  both  nominal  and  real  accounts, 
but  after  closing,  it  really  constitutes  a  non-classified  balance  sheet." 

Books — The  desire  to  keep  some  of  the  facts  of  the  business  unknown 
to  the  clerks  has  cau.sed  the  particulars  relative  to  the  capital  of  the 
business,  the  profit  and  loss  accounts  and  other  matters  to  be  kept  in  a 
separate  ledger  called  "private  ledger,"  which  is  made  self-balancing  by 
using  an  "adjustment"  account  called  general  ledger  account."  The  other 
data  is  recorded  in  the  ledger  accessible  to  the  clerks,  called  the  "general 
ledger,"  which  is  also  self-balancing,  the  "adjustment"  account,  which  is 
called  the  private  ledger  account,  being  the  reverse  of  general  ledger  ac- 
count in  the  "private  ledger."™ 

While  bookkeeping  can  be  done  with  only  the  standard  "T"  ledger 
accounts,  many  variations  have  been  devised  whose  chief  point  of  interest 


DOUBLE-ENTRY   BOOKKEEPING  21 

is  the  insertion  of  a  "balance"  column.  Sometimes  the  money  columns 
are  placed  in  the  middle  of  the  page,  the  order  being  debit,  balance,  and 
credit.^"  Another  variation  is  for  the  money  columns  to  be  all  at  the 
extreme  right  of  the  page  in  the  order  debit,  credit,  and  balance."  Still 
another  variation,  called  the  Boston  or  tabulated  ledger,  has  very  wide 
sheets  containing  space  for  the  names  of  the  accounts  (written  one  to 
a  line  under  each  other)  and  a  group  of  three  money  columns  in  the 
order  debit,  ci-edit,  and  balance  for  each  day  or  month,  as  the  case  may 
be.'^  Used  in  a  bank,  the  total  of  balance  columns  gives  the  aggregate 
depositors'  balance,  and  the  totals  of  the  debit  and  credit  columns  give 
the  aggregate  of  the  deposits  received  and  checks  cashed,  respectively.*' 

Voucher  System — The  voucher  system  is  the  treatment  of  creditors'  ac- 
counts in  such  a  way  as  to  obviate  the  necessity  of  keeping  the  individual 
accounts  in  the  ledger.**  All  invoices,  which  are  held  up  until  the  goods 
are  received  and  accepted,  are  attached  to  voucher  jackets,  which,  num- 
bered consecutively,  show  the  creditor's  name  and  address,  the  material 
received  and  the  account  to  be  charged.'"  The  voucher  is  sent  for  signature, 
along  with  the  check,  but,  as  many  vouchers  are  not  returned,  the  system 
relies  largely  on  duplicate  copies.'*  To  avoid  this,  voucher  checks  have 
been  devised  which  show  upon  the  check  itself  what  is  being  paid  for." 

As  soon  as  the  voucher  jacket  is  made,  it  is  entered  on  a  voucher  register 
or  record  which  lists  these  voucher  jackets  by  date,  number  and  firm.™ 
Columns  are  provided  to  take  the  credit  to  the  vouchers  payable  account 
(another  name  for  accounts  paj^able),  for  the  allocation  of  the  charges 
to  the  usual  expense  accounts,  and  for  the  names  of  unusual  accounts 
with  a  money  column  for  such  items.™  When  a  voucher  is  paid,  it  is 
entered  in  the  cash  book  as  would  be  done  in  any  system.'*  However,  this 
payment  is  recorded  in  the  voucher  register  which  shows  which  vouchers 
are  still  unpaid,  the  aggregate  of  which  agrees  with  the  balance  of  the 
vouchers 'payable  account  in  the  general  ledger." 

The  use  of  voucher-checks  combines  the  advantage  of  having  evidence 
that  the  creditor  was  paid  with  that  of  having  the  creditor's  signature 
as  to  the  payment  for  a  certain  invoice.*'  Vouchers  are  numbered  con- 
secutively as  entered  in  the  voucher  register."  The  voucher-checks  may 
have  only  one  number,  in  which  case  the  number  of  the  voucher-check 
would  agree  with  the  number  of  the  voucher.  As  vouchers  are  seldom 
paid  in  the  order  in  which  they  are  recorded  in  the  voucher  register,  the 
voucher-checks  having  only  one  number  would  not  be  entered  in  con- 
secutive order  in  the  cash  book.  Sometimes  voucher-checks  are  given  two 
numbers,  the  first  being  the  number  of  the  voiicher  and  the  second  the 
treasurer's  number.**  This  method  of  numbering  enables  the  voucher- 
checks  to  be  entered  consecutively  in  the  cash  book."*  By  the  use  of  the 
first  number  the  entry  in  the  voucher  register  can  be  traced  and  by  the 
use  of  the  second  number  the  cash  book  entry  can  be  located. 

The  advantages  claimed  for  the  voucher  system  are  :*"  First,  gives  de- 
tailed analysis  of  all  purchases;  .second,  saves  labor  by  eliminating  the 
purchase  ledger;  third,  secures  an  up-to-date  entry  of  all  liabilities;  fourth, 


22  C.  P.  A.  ACCOUNTING 

localizes  responsibility  by  showing  authority  for  auditing,  payment,  and 
entry  of  the  items;  fifth,  secures  a  receipted  bill  for  all  payments  of 
cash;  and  sixth,  increases  the  effectiveness  and  decreases  the  cost  of  an 
audit.  The  chief  disadvantages  claimed  are:*'  First,  clumsy  provision 
for  returns  and  allowances,  partial  jiayment  of  bills,  and  notes  payable; 
second,  inadequate  showing  of  volume  of  business  with  each  creditor; 
third,  the  giving  out  of  information  about  the  business  which  should  be 
kept  private;  fourth,  the  expense  of  filing  and  recording  the  vouchers 
and  the  labor  of  having  them  vised  by  the  various  officials;  and  fifth,  no 
ledger  accounts  available  for  future  reference. 

The  first  of  these  objections  is  by  far  the  most  important  for  the  volume 
of  business  with  each  creditor  can,  if  desired,  be  readily  computed  on  an 
adding  machine,  and,  if  only  the  date  and  number  of  the  creditor's  invoice 
appear  on  the  voucher  check,  little  secret  information  will  be  disclosed. 
Purchase  returns  and  allowances  should  be  recorded  in  red  ink  in  both 
the  votichers  payable  column  and  the  distributive  columns  affected.''  Then 
the  red  ink  entries  may  be  deducted  from  the  regular  entries  or  both  red 
and  black  totals  may  be  used,  in  which  case  both  totals  should  be  posted, 
the  red  as  offsets  to  the  corresponding  black  postings.'*  "Where  partial 
payments  are  to  be  made,  the  original  voucher  may  be  canceled  in  full 
and  two  new  voucliers  issued  in  place  of  it.^*"  This  adjustment  may  be 
made  directly  on  the  face  of  the  voucher  register  by  cross-references  be- 
tween the  old  and  new  vouchers,  usually  shown  in  the  "manner  of  payment" 
column;  or  by  reversing  the  voucher  register  entry  for  the  old  voucher 
in  the  general  journal  and  recording  the  new  vouchers  as  usual  in  the 
voucher  register."" 

Vouchers — A  voucher  is  a  document  verifying  the  correctness  of  charges 
for  values  paid  out  or  of  credits  for  values  received.^*"  The  fact  that  one 
source  of  information  may  be  missing  does  not  prevent  the  verification 
of  entries.  For  instance,  sales  items  may  l)e  vouched  by  customers'  orders, 
shipping  clerk's  records,  carbons  of  invoices,  bills  of  lading,  customers' 
accounts  and  cash  collections.*"*  Vouchers  for  returned  sales  consist  of 
correspondence  carbons  of  credit  memos  and  invoices  showing  the  original 
charge,  customers'  settlement  statements,  and  cash  refund  vouchers.*"* 
Copies  of  deposit  slips,  stubs  of  receipt  books,  and  bank  pass  book  are 
the  cash  receipt  vouchers.*"*  The  vouchers  for  cash  payments  and  pur- 
chases comprise  copies  of  orders,  receiving  clerk's  records,  checked  invoices, 
receipts  for  payments,  canceled  checks,  petty  cash  vouchers,  pay  rolls, 
wages  and  salary  records,  and  the  minute  book.*"^  Returned  purchases 
may  be  verified  from  stock  records,  shipment  records,  credit  memos,  and 
specification  of  requirements.*"'  As  journal  entries  are  generally  of  an 
unusual  nature  they  are  frequently  supported  by  formal  vouchers  approved 
by  proper  authority.*"*  Correspondence,  and  the  minute  book  also,  are 
used  to  vouch  the  journal  entries. 

Articulation  Statement — The  articulation  statement  consists  of  a  sheet 
ruled  so  that  the  accounts  ai-e  listed  under  each  other,  one  account  to  a 
line,   and  debit  and  credit  money   columns  are  provided  for  each   class 


DOUBLE-ENTRY   BOOKKEEPING 


23 


of  transactions^"*  or  for  each  ledger  account.'^"  If  a  man  started  a  period 
with  cash  $2,000  and  merchandise  $3,000  and  then  purchased  $5,000  and 
sold  $7,500  worth  of  goods  and  incurred  expense  amounting  to  $1,000, 
the  articulation  statement  would  be  as  follows : 


Articulation  Statement 


Accounts 

Starting 
Balances 

Merchandise 

Expense 

Final 
Balances 

Cash 

$2,000 
3,000 

$1,000 

$1,000 
500 

Merchandise. . . 

$5,000 

$5,000 

$7,500 

Capital      .... 

$5 , 000 

Creditors. 

5,000 

5,000 

Debtors. 

7,500 

7,500 
1,000 

Expense 

$1,000 

$5,000 

$5,000 

$12,500 

$12,500 

$1,000 

$1,000 

$10,000 

$10,000 

It  is  possible  to  subdivide  the  headings  into  basic  elements,  e.  g.,  the 
merchandise  could  be  divided  into  sales  and  purchases.  Sometimes  two 
columns  are  allotted  to  cash,  on  account  of  the  large  number  of  such 
transactions."' 

The  articulation  statement  is  of  little  or  no  practical  value"'  but  the 
same  idea  is  given  a  practical  application  when  the  interim  entries  are 
classified  according  to  the  books  of  original  entry."" 


24  C.  P.  A.  ACCOUNTING 


QUESTIONS 

DOUBLE-ENlTiY    BOOKKEEPING 

Accounts 

I.  What  do  you  understand  by  the  term  "an  account"?  (N.  J.,  1904— 
1909;  Mo.,  Dec,  1914;  Iowa,  Dec.,  1918.) 

2.-    Define:     Account  stated.     (Mich.,  Dec,  1914;  Mo.,  Dec,  1914.) 

3.  What  various  meanings  may  an  entry  in  a  Ledger  account  have,  on 
(a)  the  debit  side,  (b)  the  credit  side?     (N.  Y.,  Jan.,  1897.) 

4.  State  fully  the  value  of  accounts  under  the  following  conditions : 
(a)  When  correctly  kept;  (&)  when  incorrectly  kept;  (c)  as  a  basis  of 
liquidation;  (d)  as  a  going  concern;  (e)  for  the  purpose  of  .sale  of  busi- 
ness.    (Pa.,  Nov.,  1906.) 

5.  Define  the  principal  or  main  accounts  of  a  corporation  of  your  own 
selection.     (Pa.,  May,  1905.) 

6.  State  the  two  piimary  results  to  be  sought  in  keeping  accounts.  In 
what  financial  statements  are  these  results  shown?  Point  out  the  relation 
between  these  statements.     (Mo.,  Dec,  1913.) 

7.  What  is  the  basis  of  accounts,  (a)  in  a  corporation,  (b)  in  a  part- 
nership?    (Pa.,  May,  1906.) 

8.  What  do  you  consider  the  most  important  account  in  a  set  of  account 
books?     Explain  fully.     (Pa.,  Nov.,  1899.) 

9.  Define:  Suspense  account.  (N.  Y.,  Dec,  1896;  N.  Y.,  Dec,  1897; 
N.  Y.,  June,  1898*;  N.  Y.,  Dec,  1899;  Pa.,  Nov.,  1900;  N.  Y.,  Jan.,  1902; 
Mich.,  June,  1908;  Wash.,  Aug.,  1908;  Ohio,  March,  1910;  Wash.,  Nov., 
1913;  W.  Va.,  May,  1917.) 

10.  Give  three  instances  of  items  carried  to  a  Suspense  account,  and 
state  how  they  may  be  removed  from  that  account.     (N.  Y.,  Jan.,  1902.) 

II.  State  where  and  how  the  Suspense  account  is  employed.  (N.  Y., 
Jan.,  1902.) 

12.  Explain  the  meaning  of  an  item  in  suspense.     (N.  Y.,  June,  1900.) 

13.  Show  proper  classification  of  the  Suspense  account  on  the  debit  and 
credit  sides  of  a  Balance  Sheet.    Give  examples.     (Mich.,  June,  1908.*) 

14.  Describe  the  use  and  purpose  of  a  Suspense  account.  (N.  Y.,  June, 
1898*;  Va.,  Oct.,  1912.) 


DOUBLE-ENTRY  BOOKKEEPING  2o 

15.  What  do  you  understand  to  be  the  difference  between  personal 
accounts,  impersonal  accounts,  real  accounts,  and  nominal  accounts'?  (111. 
May,  1913.) 

16.  Define:  Nominal  accounts.  (N.  Y.,  June,  1899;  N.  Y.,  June, 
1900;  Pa.,  Nov.,  1900;  N.  Y.,  Jan.,  1906;  Cal.,  May,  1908;  N.  Y.,  Jan., 
1911;  Mich.,  June,  1912;  La.,  May,  1913;  N.  Y.,  Jan.,  1914;  Mich.,  June, 
1914;  N.  D.,  June,  1914;  Kan.,  May,  1916;  Ind.,  June,  1916;  N.  D., 
July,  1916*;  Minn.,  Oct.,  1916;  Ohio,  Nov.,  1916;  N.  D.,  Aug.,  1917*; 
A.  I.  of  A.,  May,  1919.) 

17.  What  purpose  does  the  use  of  nominal  accounts  subserve?  (Ohio, 
Nov.,  1913.) 

18.  When  and  how  are  nominal  accounts  disposed  of?  (Ohio,  Nov., 
1913;  Ohio,  Nov.,  1915.) 

19.  Defi.i3.  Real  accounts.  (N.  J.,  1904-1909;  N,  Y.,  Jan.,  1906; 
Cal.,  May,  1908;  La.,  May,  1913;  Mich.,  June,  1914;  Ind.,  June,  1916; 
N.  D.,  July,  1916*;  Ohio,  Nov.,  1916;  N.  D.,  Aug.,  1917.*) 

20.  What  is  the  relation  of  nominal  accounts  to  real  accounts'?  How 
do  these  accounts  fulfill  the  purpose  for  which  they  are  created?  (N.  Y,, 
June,  1912;  Okla.,  Nov.,  1919.) 

21.  Classify  the  following  accounts  as  real  and  nominal :  Salary,  Notes 
Payable,  Fixtures,  Rent,  Interest  Earned,  Discount  on  Sales.  (N.  D., 
July,  1916.) 

22.  Define:  Personal  accounts,  (N.  Y.,  June,  1899;  Pa.,  Nov.,  1900; 
Md.,  Oct.,  1903;  N.  Y.,  Jan.,  1906;  Cal.,  May,  1908;  Mich.,  June,  1912; 
Ind.,  June,  1916;  Ohio,  Nov.,  1916;  N,  D.  Aug.,  1917.*) 

23.  Define:  Impersonal  accounts.  (N.  Y.,  Jan.,  1906;  Cal.,  May, 
1908;  Mich.,  June,  1912*;  Ind.,  June,  1916;  N.  D.,  Aug.,  1917.*) 

24.  Define:    Mixed  accounts.     (Cal.,  May,  1916*;  N.  Y.,  Jan.,  1918.) 

25.  What  are  the  objections  to  a  mixed  account?    (Cal.,  May,  1916.) 

26.  Illustrate  mixed  accounts.     (Cal.,  May,  1916*;  Mich.,  June,  1919,) 

27.  Define:    Major  accounts.     (N.  Y.,  Jan.,  1918.) 

28.  What  is  a  representative  account?     (N.  J.,  1904-1909.) 

29.  Define:    Subsidiary  accounts.    (N.  Y.,  Jan.,  1918.) 

30.  Define:  Current  account.  (N.  Y.,  Jan.,  1906;  Mich.,  June,  1912; 
La.,  May,  1913;  Wash.,  July,  1917.) 

31.  Define:     Collective  accounts.     (N.  Y.,  Jan.,  1918.) 

32.  Define:  Summary  accounts.  (N.  Y.,  Jan.,  1906*;  Mich.,  June, 
1912;  La.,  May,  1913;  N.  Y.,  Jan.,  1918.) 

33.  What  is  meant  by  the  term  "primary  accounts"?  (111.,  May,  1913* ; 
N.  D.,  Aug.,  1917.) 

34.  Define:  Specific  accounts.  (N.  Y.,  Jan.,  1906;  Mich.,  June,  1912*; 
La.,  May,  1913;  N.  Y.,  Jan.,  1918.) 


26  C.  P.  A.  ACCOUNTING 

35.  Define:  Controlling  accounts.  (N.  Y.,  Jan.,  1900;  N.  Y.,  June, 
1904* ;  N.  Y.,  Jan.,  1906 ;  N.  Y.,  Feb.,  1910 ;  Va.,  Nov.,  1910 ;  Mich.,  June, 
1912;  La.,  May,  1913;  Wash.,  Nov.,  1913;  Ohio,  Nov.,  1913*;  Ohio,  Nov., 
1915;  Kan.,  Dec.,  1915;  Mo.,  Dec.,  1915;  N.  D.,  July,  1916*;  Ohio,  Nov., 
1916;  Cal.,  Nov.,  1916;  W.  Va.,  May,  1917;  111.,  May,  1917;  N.  C,  Aug., 
1917*;  A.  I.  of  A.,  Nov.,  1917;  Ind.,  Nov.,  1918;  Iowa,  Dec,  1918.). 

36.  State  the  purposes  of  a  controlling  account.  (N.  Y.,  June,  1904; 
N.  J.,  1904-1909;  Va.,  Nov.,  1910;  Ohio,  Nov.,  1915;  Mo.,  Dec.,  1915; 
Kan.,  Dec,  1915;  N.  D.,  July,  1916*;  Okla.  Nov.,  1919.*) 

37.  Give  an  illustration  of  the  use  of  a  controlling  account.  (N.  Y., 
Jan.,  1900;  N.  Y.,  June,  1904*;  Va.,  Nov.,  1910*;  Mich.,  June,  1913*; 
Ohio,  Nov.,  1913*;  W.  Va.,  May,  1917.) 

38.  Give  the  names  of  several  controlling  accounts.  (N.  Y.,  June, 
1904*;  Cal.,  June,  1904*;  N.  Y.,  Feb.,  1910;  Wash.,  May,  1911*;  Ohio, 
Nov.,  1915;  Kan.,  Dec,  1915;  Mo,,  Dec,  1915;  Ohio,  Nov.,  1916*;  Okla., 
Nov.,  1919.*) 

39.  Name  the  advantages  of  controlling  accounts.  (Mich.,  Jime, 
1910*;  Mich.,  June,  1913*;  Ohio,  Nov.,  1913;  N.  D.,  July,  1916*;  Ohio, 
Nov.,  1916.) 

40.  How  do  you  reconcile  a  debit  to  a  controlling  account  and  another 
debit  for  the  same  amount  to  an  individual  creditor  with  the  fundamental 
principle  of  double-entry  bookkeeping?     (Ohio,  Nov.,  1915.) 

41.  In  making  the  audit  of  the  Springfield  Grocery'  Company  you  find 
that  the  controlling  account  in  the  general  ledger  exceeds  the  aggregate  of 
the  balances  in  the  Accounts  Receivable  Ledger  by  $25,210.66.  What  steps 
would  you  take  to  bring  this  ledger  into  agreement  with  the  controlling 
account  in  the  General  Ledger?     (111.,  May,  1916.) 

42.  In  designing  a  set  of  accounts  for  a  business,  how  might  provision 
be  made  for  a  constant  showing  of  the  aggi'egate  sum  owing  by  customers 
and  the  aggregate  sum  owing  to  cretlitors,  without  the  neees.sity  of  pre- 
paring a  schetlule  of  the  accounts  of  such  customers  and  creditors?  (N.  Y., 
Dec,  1896*;  N.  Y.,  June,  1901.) 

43.  Describe  a  method  of  keeping  accounts  so  that  an  independent 
balance  of  the  ledger,  containing  only  the  real,  nominal,  special  and  con- 
trolling accounts,  exclusive  of  the  individual  accounts  of  customers  and  of 
trade  crcc^itors,  may  be  taken.     (N.  Y.,  Dec,  1896.) 

44.  What  is  meant  by  "classification  of  accounts"?  (Md.,  Oct.,  1903; 
Wis.,  April,  1917;  Okla.,  Nov.,  1919.) 

45.  Into  what  general  classes  should  Ledger  accounts  be  divided?  State 
the  distinguishing  feature  of  each  class.  Mention  one  account  belonging  to 
each  class.  (N.  Y.,  Dec,  1898*;  N.  Y.,  Jan.,  1901;  N.  Y.,  Oct.,  1907*; 
R.  L,  Dec,  1907*;  Wis.,  April,  1914.*) 

46.  State  what  verification  you  would  make  of  the  classification  of 
expenses.    (N.  C,  Aug.,  1917;  N.  C,  Nov.,  1918.*) 


DOUBLE-ENTRY  BOOKKEEPING  27 

47.  A  wholesale  house  has  on  its  books  200  individual  accounts  with 
creditors,  500  with  city  customers,  and  1500  with  country  customers,  be- 
sides about  75  impersonal,  or  representative  accounts.  Owing  to  the 
methods  of'^bookkeeping  in  force,  it  is  necessary,  in  order  to  ascertain  the 
amount  of  Accounts  Receivable  or  Payable,  to  take  otf  a  complete  list  of 
the  accounts  in  question.  You  are  called  upon  to  advise  as  to  how  this 
difficulty  can  be  overcome,  and  also  as  to  whether  the  bookkeeping  work, 
on  the  Accounts  Payable  cannot  be  reduced,  having  regard  to  the  fact  that 
the  firm  discounts  all  its  bills. 

Embody  your  suggestions  in  a  brief  report.     (Wash.,  April,  1906.) 

48.  Draw  up  a  brief  but  effective  classification  of  General  Ledger  ac- 
counts suitable  for  a  company  with  a  total  capital  investment  of  say 
$750,000,  doing  an  annual  business  of  about  $1,000,000,  employing  about 
300  men,  and  with  500  customers'  accounts.  Assume  the  company  manufac- 
tures and  sells  to  dealers  bicycles  of  10  different  patterns  and  styles,  and 
does  a  small  business  in  the  sale  of  duplicate  parts,  the  accounts  to  be 
framed  to  permit  of  the  preparation  monthly  of  an  approximate  Balance 
Sheet  and  approximate  Profit  and  Loss  account  without  the  taking  of  a 
physical  inventory  except  at  the  annual  closing  on  September  30  of  each 
year. 

State  the  classification  in  the  order  in  which,  in  your  opinion,  the 
accounts  should  appear  in  the  Ledger,  giving  briefly  your  reasons  there- 
for, and  indicate  which  accounts,  if  any,  should  be  supplemented  with 
subsidiary  ledgers  of  other  sub-account  records.     (111.,  May,  1913.) 

49.  Classify  and  group  the  following  accounts  according  to  kind  of 
asset,  liability,  proprietary  interest,  income  and  expense: 

Interest  Collected  in  Advance.  Bonds  Payable. 

Sinking  Fund.  Interest  Earned  on  Liberty  Bonds. 

Notes  Receivable  Discounted.  Interest  Paid  on  Liberty  Bond  Install- 
Treasury  Stock.  ments. 

Work  in  Progress.  Reserve  for  Depreciation,  Buildings. 

Dividends  Unclaimed.  Dividends  Declared. 

Capital  Stock  Subscription.  Dividends  Payable. 

Suspense  Accounts  Receivable.  Investments  in  and  advances  to  corn- 
Discount  on  Bonds  Issued  Written  Off.  panics  for  purposes  of  control. 

Accrued  Property  Taxes.  Advances  to  Company  Officials. 

Accrued    Income    and    Excess   Profits  Insurance  Premiums  paid  by  company 
Taxes.  on   life   of   its  president    (in   which 

Interest  Accrued  on  Notes  Receivable.  the  company  is  beneficiary). 

Interest  Accrued  on  Bonds  Payable.  Installment    Payments    by    employees 
Reserve  for  Sinking  Funds.  on  Liberty  Bonds  bought  for  them 

Reserve  for  Bad  Debts.  by  company. 

Merchandise  Purchases.  Liberty  Bonds  bought  for  employees. 

Sales  Returns  and  Allowances.  Employees'    Liberty    Bond    Subscrip- 
Reserve  for  Building  Extensions.  tion. 

(Wis.,  Nov.,  1919.) 

50.     Classify  and  group  the  following  accounts  of  a  manufacturing  com- 
pany, according  to  kind  of  asset,  liability,  loss  and  gain : 

1.  Accounts  Payable.  3.  Accrued  Salaries  and  Wages. 

2.  Accounts  Receivable.  4.  Advertising. 


28 


C.  P.  A.  ACCOUNTING 


5.  Bad  Debts  Written  Off, 

6.  Bills  Payable. 

7.  Bills  Receivable. 

8.  Bond  Discount. 

9.  Bond   Premium. 

10.  Bond  Interest  Accrued. 

11.  Capital  Stock. 

12.  Cash. 

13.  Credit  Department  Expenses. 

14.  Depreciation    of    Buildings,    Ma- 

chinery and  Plant. 

15.  Depreciation   of    Workmen's  Cot- 

tages. 

16.  Directors'  Fees. 

17.  Discount  on  Purchases. 

18.  Discount  on  Sales. 

19.  Federal  Corporation  Tax. 

20.  First  Mortgage  Bonds. 

21.  Freight  and  Cartage  Inward. 

22.  Freight  and  Cartage  Outward. 

23.  General  OflSce  Expenses. 

24.  Goodwill. 
2-5.  Insurance. 

26.  Insurance  Premiums  Unexpired. 

27.  Interest  on  Bills  Payable. 

28.  Interest  on  Bonds. 

29.  Income  from  Investments. 

30.  Inventory,  Raw  Materials. 

31.  Inventory,  Goods  in  Process. 

32.  Inventory,  Manufactured  Goods. 

33.  Investments  (Outside). 

34.  Maintenance    of    Buildings,    Ma- 

chinery, and  Plant. 

35.  Maintenance  of  Workmen  's  Cot- 

tages. 

36.  Manufacturing  Power,   Heat,  and 

Light. 

(Wis 


37.  Miscellaneous  Factory  Expenses. 

38.  Miscellaneous  Selling  Expenses. 

39.  Nonproductive  Labor. 

40.  Office  Equipment. 

41.  Office   Salaries. 

42.  Officers'  Salaries  and  Expenses. 

43.  Organization  Expenses. 

44.  Patent  Rights. 

45.  Patterns  and  Drawings. 

46.  Plant  Site. 

47.  Plant  Buildings. 

48.  Plant,  Machinery,  and  Equipment. 

49.  Productive  Labor. 

50.  Purchasing  Department  Expenses. 

51.  Raw  Materials  Purchased. 

52.  Rent  of  Workmen's  Cottages. 

53.  Reserve  for  Depreciation  of  Build- 

ings, Machinery,  and  Plant. 

54.  Reserve  for  Depreciation  of  Work- 

men 's  Cottages. 

55.  Reserve  for  Doubtful  Accounts. 

56.  Reserve  for  Sinking  Fund. 

57.  Returns  and  Allowances  on  Pur- 

chases. 

58.  Returns  and  Allowances  on  Sales. 

59.  Sales  of  Manufactured  Goods. 

60.  Sales  of  Waste  Material. 

61.  Sales  Agents'  Commissions. 

62.  Salesmen  's   Salaries. 

63.  Salesmen's  Expenses. 

64.  Sinking  Fund  Investments. 

65.  Surplus. 

66.  Taxes   on    Plant   and    Equipment. 

67.  Taxes  Accrued. 

68.  Workmen 's  Cottages. 

,,  April,  1915;  S.  C,  Sept.,  1919.) 


51.  Assuming  that  these  are  the  principal  divisions  of  the  expense 
accounts  of  a  manufacturing  business  selling  the  product  through  traveling 
salesmen  to  the  retail  trade:  (a)  Manufacturing  Expenses  Division;  (b) 
Selling  Expenses  Division;  (c)  Administration  and  General  Expenses 
Division;  (rf)  Profit  Deduction  Expenses  Division,  designate  in  what 
division  you  would  classify  each  of  the  following  accounts,  by  giving  the 
number  of  account  and  the  letter  indicating  division  opposite: 


10. 


Materials  and  Supplies  Consumed. 
Interest  on  Loans  Paid. 
Interest  on  Bonds  Paid. 
Postage  for  Correspondence. 
Postage   for   Parcel   Post. 
Domestic   Taxes. 

Street  Assessments  for  Street  Im- 
provements. 
Federal   Taxes. 

Discounts  Received  on  Purchases. 
Discounts  Given  on  Sales. 


11.  Discounts  Allowed  for  Prompt  Pay- 

ment of  Accounts  Receivable. 

12.  Exchange  on  Checks. 

13.  Revenue  Stamps. 

14.  Bonus  to  Traveling  Salesmen. 

15.  Bonus  to  Office   Force. 

16.  Bonus  to  Factory  Operatives. 

17.  Bonus  to  Superintendent  of  Fac- 

tory. 

18.  Fire  Insurance  Premiums  on  Fac- 

tory. 


DOUBLE-ENTRY  BOOKKEEPING  29 

19.  Fire   Insurance  Premiums  on   Of-       31.  Traveling  Expenses  of  Buyer. 

fice.  32.  Expenses  of  Lawsuits  for  Colieet- 

20.  Fire      Insurance      Premiums      on  ing  Accounts. 

Warehouses.  33.  Lawsuit  Expenses  Defending  Suit 

2L  Liability  Insurance  Premiums.  brought  by  Employee  for  Dam- 

22.  Life  Insurance  i'rcmiums  on  Fac-  ages 

tory  Operatives'  Lives.  34  Expense  of  Welfare  Work. 

23.  Credit  Insurance  Premiums.  35  pj^^g    Assessed    for    Violation   of 

24.  Tornado  Insurance  Premiums  Child  Labor  Laws. 

25.  Freight   Prepaid   on  Shipments.  o^  tt    1   •       r,     ^      ^  t-. 

26.  Claims  Allowed  on  Sales  Made.  l^'  Hedging  Contract  Expenses. 

27.  Rent   of   Factorv.  ^^'  Donations  to  Employees. 

^8    Pent  of  Office     '  ^^'  Donations    to    Other    Than    Em- 

29."  (Omitted       on       Official       Docu-  ployees. 

ment.)  3^-  Freights  on   Purchases. 

30.  Traveling    Expenses    of    Superin-      40.  Claims    allowed    on    Merchandise 
tendent  to  Secure  Operatives,  Purchased. 

(N.  C,  Nov.,  1919.) 

52.  Show  by  chart  or  outline  the  various  divisions  and  subdivisions  of 
all  asset,  liability,  income  and  expenditure  accounts.     (Wis.,  April,  1914.) 

53.  Into  what  two  general  and  what  three  special  classes  are  accounts 
divided  in  double-entry  bookkeeping?     (Mich.,  June,  1912.) 

54.  What  are  the  purposes  of  elassifieations  of  accounts  in  a  business, 
and  how  should  they  be  grouped  to  show  the  proper  results?  (Pa.,  Nov., 
1906;  Okla.,  Nov.,'l919.*) 

55.  On  the  theory  that  accounts  may  fall  into  the  following  classifica- 
tions, (a)  accounts  with  individuals,  {h)  accounts  with  things,  (c)  ac- 
counts with  forces  or  ideas,  give  a  number  of  illustrations  or  names  of 
accounts  in  a  modern  accounting  system  under  each  of  these  heads.  (Ohio, 
March,  1910.) 

56.  Which  group  of  accounts,  if  eliminated  from  double-entry  bookkeep- 
ing, would  reduce  accounts  to  an  economic  history?     (N.  Y.,  Jan.,  1917.) 

57.  Which  class  or  classes  of  accounts  close  into  Loss  and  Gain  ac- 
count?    (N.  Y.,  Dec.,  1898.*) 

58.  What  is  the  nature  of  the  accounts  on  the  debit  or  left-hand  side 
of  the  Ledger?     (Ohio,  March,  1910.) 

59.  What  is  the  nature  of  the  accounts  on  the  credit  or  right-hand  side 
of  the  Ledger?     (Ohio,  March,  1910.) 

60.  What  are  the  advantages  obtained  through  numbering  accounts? 
Name  and  briefly  describe  two  methods  of  numbering  or  lettering  ac- 
counts.    (Wis.,  April,  1914;  Mich.,  Dec,  1916.*) 

61.  In  the  opening  of  a  Ledger,  what  principle  should  be  followed  as 
to  the  order  or  arrangement  of  the  accounts?  Show  the  advantages  of  the 
different  plans.  (N.  Y.,  Jan.,  1897*;  V.  Y.,  Dec,  1897;  N.^Y.,  Jan., 
1907*;  N.  Y.,  June,  1911*;  Cal.,  Nov.,  19  V6*;  N.  Y.,  Jan.,  1918.*) 

62.  What  is  the  chief  consideration  in  the  arrangement  of  Ledger  ac- 
counts?    (Ind.,  June,  1916.) 

63.  How  may  the  accounts  in  a  trial  balance  be  best  arranged  to  facili- 


30  C.  P.  A.  ACCOUNTING 

tate  the  preparation  of  a  business  and  financial  statement?     (S.  C,  Sept., 
1919.) 

64.  In  what  order  should  the  accounts  be  aiTanged  as  they  successively 
appear  in  (a)  a  Ledjrer  containing::  all  the  accounts  of  a  business,  (6)  a 
Ledger  containing  accounts  of  fixed  assets  and  fixed  liabilities,  as  well  as 
special,  nominal,  and  summary  accounts?     (N.  Y.,  Jan.,  1906.) 

65.  What  is  meant  by  theory  of  accounts?     (N,  Y.,  June,  1917.) 

Journal  Entries 

66.  What  is  a  Journal  entry?    When  used?     (Iowa,  Dec,  1918.) 

67.  Define:    Debit.     (N.  Y.,  Dec,  1898;  N.  J.,  1904-1909.) 

68.  Define:  Credit.     (N.  Y.,  Dec,  1898;  N.  J.,  1904-1909.) 

69.  Define  journalizing  in  its  broadest  sense.  (N.  Y.,  June,  1900; 
Wash.,  May,  1903.) 

70.  State  your  opinion  in  regard  to  the  technie  of  journalizing.  Show 
wherein  your  view  is  in  accord  with  the  evolution  of  the  books  of  account. 
(X.  Y.,  June,  1912.) 

71.  Describe  the  theory  of  double-entry  bookkeeping.  (N.  Y.,  June, 
1901;  N.  Y.,  June,  1902*;  Ohio,  March,  1910;  Mass.,  June,  1910;  III.,  May, 
1914;  Ohio,  Nov.,  1915;  Ind.,  June.  1916;  Iowa,  Dec,  1918;  N.  Y.,  Jan., 
1920.*) 

72.  In  a  recent  work  on  double-entry  bookkeeping  it  is  contended  that 
each  transaction  involves  two  entries  with  the  proprietor;  do  you  ag'ree 
with  the  author?    If  so,  why?    If  not,  why  not ?     (N.  Y.,  June,  1914.) 

73.  State  a  comprehensive  general  nde  for  journalizing.  (N.  Y.,  June, 
1899*;  N.  Y.,  June,  1900*;  N.  Y.,  Jan.,  1901;  N.  Y.,  Jime,  1902*;  Wash., 
May,  1903.*) 

74.  Why  is  the  word  "To"  used  in  connection  with  debit  entries,  and  is 
its  use  necessary?     (N.  J.,  1904-1909.) 

75.  Why  is  the  word  "By"  used  in  connection  with  credit  entries,  and 
is  its  use  necessary-?     (N.  J.,  1904-1909.) 

76.  State  how  you  would  verify  Journal  entries.     (Va.,  Nov.,  1910.) 

77.  Prepare  a  form  of  monthly  summary  Journal  entries  for  the  books 
of  original  entry.     (N.  Y.,  Jan.,  1907.) 

78.  State  the  general  law  growing  out  of  the  relationship  of  debtor  and 
creditor  that  governs  double-entiy  bookkeeping.     (N.  Y.,  Dec,  1898.) 

79.  In  double-entry  bookkeeping,  why  are  the  debits  on  the  left  and  the 
credits  on  the  right?     (N.  Y.,  June,  1900.) 

80.  What  is  the  result  of  a  debit  entry?  of  a  credit  entry?  Illustrate 
in  the  case  of  an  account  of  each  of  the  following  classes:  (a)  Personal, 
(&)  real,  (c)  nominal.     (N.  Y.,  Dec,  1898;  N.  Y.,  June,  1900.) 

81.  (o)  What  may  the  placing  of  an  item  on  the  debit  side  of  a  Ledger 
account  represent?  (b)  What  may  the  placing  of  an  item  on  the  credit 
side  of  a  Ledger  account  represent?     (Va.,  Oct.,  1911.) 


DOUBLE-ENTRY  BOOKKEEPING  31 

82.  Why  is  an  account  debited?  Why  is  an  account  credited!  (Iowa, 
Dec,  1918.) 

S3.  Stale  the  advantaf,'-es  of  double-entry  bookkeeping.  (N.  Y.,  June, 
1901;  N.  J.,  1904-1S09*;  Mass.,  June,  1910.) 

84.  What  is  the  best  plan  for  distributing  monthly  bills  among  500 
or  more  expense  and  construction  accounts?     (Mass.,  Oct.,  1914.) 

85.  In  making  detailed  audits  some  auditors  verify  all  postings  and 
footings  of  general  and  subsidiary  ledgers,  even  though  controlling  ac- 
counts are  kept.  State  reasons  for  and  against  such  procedure.  (A.  I.  of 
A.,  Nov.,  1919.) 

86.  Describe  the  entries  necessai-y  to  open  a  set  of  double-entry  books 
for  a  firm  just  starting  in  bushiess.  (N.  Y.,  Dec,  1898*;  N.  Y.,  June, 
1900;  Pa.,  Nov.,  1900*;  Wash.,  May,  1903.) 

87.  What  is  the  proper  course  of  procedure  in  taking  charge  of  the 
bookkeeping  of  a  tirm  that  has  either  no  books  of  account  or  very  im- 
perfect ones?     (N.  Y.,  June,  1898;  Pa.,  May,  1902.*) 

Trial  Balance 

88.  What  is  a  trial  balance?  (Pa.,  Nov.,  1899*;  Pa.,  Nov.,  1901;  Pa., 
May,  1902;  Pa.,  May,  1905;  N.  Y.,  Jan.,  1906;  N.  Y.,  Oct.,  1907;  111.,  Dec, 
1907;  R.  I.,  Dec,  1907;  Md.,  Jan.,  1909*;  N.  J.,  1904-1909*;  N.  Y.,  Feb., 
1910;  La.,  May,  1913*;  Colo.,  Dec,  1913*;  Wis.,  April,  1914;  Kan., 
May,  1916;  W.  Va.,  May,  1917;  Cal.,  June,  1917;  N.  C,  Aug.,  1917*;  Va., 
Nov.,  1918;  N.  C,  Nov.,  1918*;  Iowa,  Dec,  1918.) 

89.  What  is  the  purpose  of  a  trial  balance?  (N.  Y.,  Jan.,  1897* ;  N.  Y., 
Jiine,  1898;  N.  J.,  1904-1909* ;  N.  Y.,  June,  1909* ;  Ohio,  Nov.,  1913;  Ohio, 
Nov.,  1915;  Ohio,  Nov.,  1916;  Ohio,  Nov.,  1918;  Ind.,  Nov..  1918;  Md., 
Oct.,  1919.) 

90.  If  your  trial  balance  shows  an  account  having  a  debit  excess  what 
does  the  cireum.stanee  signify?  Does  the  debit  balance  shown  by  the  trial 
balance  indicate  the  true  status  of  the  account?  Explain.  (N.  Y.,  June, 
1904.) 

91.  What  are  the  principal  differences  between  a  trial  balance  taken 
before  the  books  are  closed,  and  one  taken  directly  after  they  are  closed? 
(N.  Y.,  Jan..  1897;  N.  Y.,  Dec,  1897*;  N.  Y.,  June,  1908;  Iowa,  Dec, 
1918.) 

92.  How  may  the  accounts  in  a  trial  balance  be  best  arranged  to  facili- 
tate the  preparation  of  a  business  and  financial  statement?  (N.  Y.,  Dec, 
1896.) 

93.  What  is  the  character  of  accounts  contained  in  a  trial  balance? 
(N.  Y.,  Jan.,  1897;  N.  Y.,  June,  1909.) 

94.  Describe  the  process  of  taking  a  trial  balance.  (N.  Y.,  June, 
1898.) 


3ii  C.  P.  A.  ACCOUNTING 

95.  State  the  scope  and  value  of  the  trial  balance.  (N.  Y.,  Jan., 
1902.) 

96.  Submit  trial  balance  taken  from  the  books  of  a  copartnei'ship  and 
also  a  similar  trial  balance,  as  it  would  apjiear  by  the  books  of  a  corpora- 
tion (assuming  that  in  each  '!ase  the  books  have  been  closed  at  the  end  of 
the  fiscal  period),  using  your  own  figures.     (Ohio,  Dec,  1908.) 

97.  When  accounts  are  in  equilibrium  what  may  be  said  as  to  their 
correctness?     (N.  Y.,  Jan.,  1917.) 

98.  Does  a  trial  balance  furnish  conclusive  evidence  of  the  absence  of 
errors?  Explain.  (Pa.,  May,  1905* ;  Ohio,  Nov.,  1913* ;  Ohio,  Nov.,  1915; 
N.  Y.,  June,  1918*;  Ind.,  Nov.,  1918;  Ohio,  Nov.,  1918;  Md.,  Oct.,  1919.) 

99.  How  should  one  proceed  to  detect  an  error  in  a  trial  balance? 
(N.  Y.,  Jan.,  1897;  N.  Y.,  Dec.,  1897*;  N.  Y.,  June.  1901*;  N.  Y.,  Jan., 
1902.) 

100.  Give  the  order  of  procedure  and  describe  tersely  in  enumerated 
paragi'aphs  the  steps  required  when  called  upon  to  adjust  an  incorrect 
trial  balance  which  the  bookkeeper  cannot  agree.     (111.,  May,  1913.) 

101.  In  balancing  a  set  of  books  consisting  of  Cash  Book,  Purchase 
Book,  Sales  Book,  Journal  and  Ledger,  the  debit  side  of  the  trial  balance 
is  found  to  be  $87.19  in  excess  of  the  credit  side;  how  should  the  error 
be  sought?     (N.  Y.,  Jan.,  1900.) 

102.  In  taking  off  a  trial  balance,  a  bookkeeper  finds  that  his  debit 
footings  exceed  the  credit  by  $131.56,  which  he  carries  to  a  Suspense 
account.  Later,  he  discovers  that  a  purchase  amounting  to  $417.50  has 
been  debited  to  a  creditor  as  $192.94;  that  $312.50  for  depreciation  of 
furniture  has  not  been  posted  to  depreciation  account ;  that  $500  with- 
drawn by  the  principal  has  been  charged  against  wages  account;  that  a 
discount  of  $76.13  allowed  to  a  customer  has  been  credited  to  him  as 
$71.13;  and  that  the  total  of  sales  returned  was  footed  $5  short.  Give 
detailed  entries  showing  how  you  would  remedy  these  errors,  and  starting 
with  the  original  difference  prepare  a  supplemental  trial  balance  showing 
whether  the  books  balance  or  not.     (111.,  Dec.,  1910.) 

103.  When  an  auditor  employed  to  adjust  the  accounts  of  a  finn  finds 
that  the  current  work  is  behind  and  that  no  trial  balance  has  been  made 
for  over  a  year,  what  course  should  he  pursue,  having  regard  for  his  own 
interests  as  well  as  for  those  of  the  firm?     (N.  Y.,  Jan.,  1900.) 

104.  What  deductions  may  be  safely  drawn  from  a  trial  balance  rep- 
resenting intrinsic  values  and  true  economic  history  of  the  transactions 
of  a  given  period?     (N.  Y.,  June,  1917.) 

Books 

105.  What  books  of  account  do  you  consider  necessary  for  the  con- 
duet  of  a  small  business?  Describe  their  form  and  use.  (N.  Y.,  Jan., 
1897;  N.  Y.,  Dec,  1898*;   Pa.,  Nov.,  1900*;  Pa.,  May,  1905*;  N.  Y., 


DOUBLE-ENTRY  BOOKKEEPING  33 

June,  190G*;  N.  Y.,  Feb.,  1908*;  Mich.,  July,  1909;  N.  Y,,  Feb.,  1910; 
Ohio,  March,  1910.*) 

106.  Describe  a  system  of  bookkeeping  by  which  the  errors  in  a  trial 
balance  may  be  localized.     (N.  Y.,  June,  1908.) 

107.  Suggest  a  plan  for  recording  and  posting,  with  the  least  possible 
loss  of  time,  the  remittances  received  from  customers  by  a  concern  whose 
Ledger  is  in  several  divisions,  with  a  bookkeeper  to  each  division.  (Wash., 
April,  1906.) 

108.  Describe  a  condition  in  which  the  use  of  a  few  books  would  not 
result  in  simplicity  of  system,  while  the  number  of  books  currently  oper- 
ated could  be  increased  with  advantage.     (N.  Y.,  Jan.,  1907.) 

109.  What  is  a  Ledger?     (N.  Y.,  Oct.,  1907;  R.  I.,  Dec,  1907.) 

110.  Do  you  approve  of  a  business  concern  keeping  more  than  one 
Ledger?  If  so,  why,  and  how  would  the  different  ledgers  be  character- 
ized ?     ( Mass.,  April,  191 1 . ) 

111.  Name  seven  kinds  of  ledgers  and  briefly  state  the  use  of  each 
kind.     (N.  Y.,  Dec.,  1897*;  N.  Y.,  June,  1908;  N.  Y.,  June,  1911.*) 

112.  Define:  Boston  ledger.     (Ill,  Dec,  1916.) 

113.  State  arguments  briefly  for  and  against  the  use  of  Boston  ledger. 
(Cal.,  Nov.,  1916.) 

114.  Define:  Subsidiary  ledger.     (S.  C,  Sept.,  1919.) 

115.  What  are  the  advantages  of  a  subsidiary  ledger?  (S.  C,  Sept., 
1919.) 

116.  Describe:  Self -balancing  ledger.  (R,  I.,  Dec,  1907;  N.  Y.,  June, 
1912*;  N.  Y.,  June,  1919.) 

117.  How  is  a  self-balancing  ledger  operated?  (R.  I.,  Dec,  1907*; 
N.  Y.,  June,  1912.) 

118.  In  a  large  dry  goods  business  it  is  considered  necessary  to  divide 
the  bookkeeping  in  such  a  way  that  each  ledger  shall  be  balanced  sepa- 
rately.    How  should  this  be  done?     (N.  Y.,  June,  1898.) 

119.  In  making  an  audit  would  you  consider  it  necessary  to  check  in 
detail  the  postings  of  subsidiary  ledgers?  Explain  fully.  (N.  Y.,  Feb., 
1908;  Mich.,  June,  1912.) 

120.  A  Milwaukee  corporation  has  twenty  Sales  Ledgers  and  desires 
to  adopt  a  system  of  sectional  balancing  so  that  each  ledger  can  be 
balanced  separately  each  month.  Outline  and  illustrate  the  use  of  the 
system.     (Wis.,  April,  1917.) 

121.  What  do  you  consider  the  best  subdivision  of  ledgers  when 
sales  amount  to  $6,000,000  to  $10,000,000  per  annum  in  a  manufacturing 
business,  say,  steel?  It  is  desired  to  ascertain  losses  and  gains  monthly. 
Describe  fully  your  plan  or  method  of  connecting  up  the  ledgers  em- 
ployed, also  the  method  of  treating  the  purchases  and  sales.  (Pa.,  Nov., 
1900.) 

122.  Define :  General  Ledger.     (Wash.,  Nov.,  1913.) 


34  C.  P.  A.  ACCOUNTING 

123.  State  your  opinion  briefly  as  to  the  merits  or  the  demerits  of 
General  Ledger.     (N.  J.,  1904-1909.) 

124.  Define:  Private  Ledger.  (N.  Y.,  June,  1899*;  N.  Y.,  Jan.,  1900; 
N.  Y.,  June,  1901*;  N.  Y.,  Jan.,  1902*;  Wash.,  May,  1903;  Pa.,  May, 
1903;  Cal.,  June,  1904*;  N.  J.,  1904-1909*;  N.  Y.,  Jan.,  1906*;  Mich., 
July,  1906* ;  111.,  Mav,  1907* ;  Wash.,  Aug.,  1908* ;  N.  Y.,  Feb.,  1910* ; 
Mass.,  June,  1910*;  Va.,  Oct.,  1912*;  Wash.,  Nov.,  1913;  N.  Y.,  Jan., 
1914*;  Mich.,  June,  1915*;  Cal.,  Nov.,  1916*;  111.,  Dee.,  1916;  Md., 
Oct.,  1919.*) 

125.  What  accounts  should  be  embodied  in  a  private  ledger  to  make 
it  a  complete  synopsis  of  the  business?  How  would  you  prove  the  cor- 
rectness of  these  accounts?     (N.  Y.,  June,  1918.) 

126.  When  would  it  be  advisable  to  use  a  Distribution  Ledger?  (Md., 
Jan.,  1909.) 

127.  Define:  Loose-leaf  system  of  bookkeeping.  (Va.,  Nov.,  1910; 
A.  L  of  A.,  May,  1918.) 

128.  Discuss  the  relative  merits  of  bound  books  and  loose-leaf  or 
card  records  for  various  accounting  purposes.  (N.  Y.,  Jan.,  1904* ;  Cal., 
June,  1904*;  N.  J.,  1904-1909*;  R.  L,  Dec.,  1907*;  Fla.,  April,  1908*; 
Wash.,  Mav,  1910*:  Ya.,  Nov.,  1910*;  Mass.,  April,  1911*;  Va.,  Oct., 
1912*;  La.,  May,  1913*;  Cal.,  Nov.,  1916*;  A.  I.  of  A.,  Nov.,  1917;  A.  L 
of  A.,  May,  1918* ;  Ind.,  Nov.,  1918* ;  Ga.,  May,  1919.) 

129.  State  your  view  of  the  possibility  of  removal,  destruction  or  sub- 
stitution of  loose  leaves  or  cards  to  the  extent  of  effectually  preventing  evi- 
dence in  cases  of  litigation.     (Wash.,  May,  1910.) 

130.  Give  at  least  two  methods  for  preventing  improper  abstracting 
of  leaves  from  loose-leaf  books.     (Fla.,  April,  1908.) 

131.  In  what  kind  of  business  would  loose-leaf  ledgers  be  most  use- 
ful? Under  what  circumstances  would  you  advise  the  use  of  card  ledgers? 
(Mass.,  April,  1911.) 

132.  Describe:  Tabular  books.  (Pa.,  Nov.,  1899;  Md.,  Oct.,  1903;  Pa., 
Nov.,  1906.) 

133.  What  is  the  advantage  or  disadvantage  in  the  use  of  columnar 
books?  (Pa.,  Nov.,  1899;  N.  Y.,  Jan.,  1900*;  N.  Y.,  Jan.,  1902*;  Wash., 
May,  1903*;  Pa.,  May,  1903;  N.  J.,  1904-1909;  Pa.,  Nov.,  1906;  Md., 
Jan.,  1909.*) 

134.  State  the  purposes  for  which  series  of  perpendicular  columns  are 
employed  in  books  of  original  entry  and  how  these  purposes  may  be  ac- 
complished relative  to  the  following  conditions:  (a)  Several  ledgers  com- 
prehended in  one  system  of  accounts;  (ft)  several  departments  com- 
prehended in  one  business;  (c)  several  accounts  comprehended  in  income 
and  expenditure.     (N.  Y.,  Dec.,  1896.) 

135.  Prepare  form  of  book  for  small  business  combining  General 
Ledger,  General  Journal  and  General  Cash  Book  in  one  binding  to  show 
transactions  involving  all  three  on  each  double  page.     (III.,  Nov.,  1904.) 


DOUBLE-ENTRY  BOOKKEEPING  35 

136.  Illustrate  a  Columnar  Cash  Book,  a  Columnar  Journal,  and  a 
Columnar  Sales  Book.  What  getieral  requirements  should  be  observed  in 
designing  such  books?     (N.  Y.,  Jan.,  1900*;  Wash.,  May,  1903.) 

137.  What  is  a  Journal?     (N.  Y.,  Of^t.,  1907;  R.  I.,  Dec,  1907.) 

138.  Explain  what  is  meant  by  "books  of  original  entry"  and  give 
two  illustrations  of  same.     (Md.,  Oct.,  1903*;  Ohio,  Nov.,  1916.) 

139.  Describe  various  uses  of  the  Journal.  (N.  Y.,  Jan.,  1897;  N.  Y., 
June,  1902*;  N.  Y.,  June,  1906.) 

140.  Write  a  short  article  upon  the  Journal:  (o)  its  form;  (b)  its 
principle,  and  (c)  how  the  principle  works  in  all  proper  bookkeeping, 
even  in  the  absence  of  a  Journal  as  a  book,     (Md.,  Oct.,  1903.) 

141.  State  your  opinion  briefly  as  to  the  merits  or  the  demerits  of  the 
Journal;  do  you  consider  its  use  necessary?     (N.  J.,  1904-1909.) 

142.  In  what  does  the  Journal  differ  from  the  Cash  Book?  (N.  Y., 
June,  1902.) 

143.  Describe  the  following  books  and  explain  the  nature  and  the 
objects  of  each  :  (a)  Summary  Journal ;  (6)  Con.sumption  Journal.  (N.  Y., 
Jan.,  1907.) 

144.  What  do  you  consider  a  complete  checking  of  the  General  Jour- 
nal?    (R.  L,  Dec,  1907* ;  Ohio,  Nov.,  1915.) 

145.  What  are  auxiliary  books  as  understood  in  accounting  terminol- 
ogy? Mention  three  books  of  this  class  and  explain  their  use.  (N.  Y., 
June,  1904.) 

146.  Messrs.  "C"  and  "D"  desire  that  their  Ledger  shall  show  all  the 
transactions  with  their  creditors,  i.e.,  purchases  and  payments  with  each 
creditor.  Naturally  the  bookkeeper  wishes  so  to  arrange  his  books  as  to 
accomplish  the  desired  end  with  the  minimum  of  labor. 

(a)  What  would  you  recommend?  (6)  What  would  you  name  your 
plan?  (c)  What  general  instruction  would  you  give  to  the  bookkeeper 
to  enable  him  properly  to  maintain  your  "plan"  after  its  installation  by 
you?  (N.  J.,  1904-1909.) 

147.  You  wish  so  to  keep  your  accounts  that  at  any  given  time  you 
may  readily  ascertain  the  aggregate  balances  due  from  your  customers 
and  the  aggregate  balances  due  to  your  creditors,  without  taking  the  time 
to  list  the  same  (customers  and  creditors)  and  at  the  same  time  you 
wish  to  maintain  your  Ledger  in  perfect  balance. 

(a)  How  would  you  arrange  your  Ledger  or  Ledgers?  (&)  How  would 
you  name  the  Ledger  or  Ledgers?  (c)  How  would  you  maintain  the 
balance  of  the  Ledgers  or  make  it  possible  to  balance  or  prove  one  Ledger 
independent  of  the  other  Ledgers?  (N.  J.,  1904-1909.) 

148.  Define  or  describe  what  is  meant  by  the  words  "a  book  of  original 
entry"  and  state  what  must  be  shown  with  respect  to  the  entries  therein 
upon  trial  of  any  cause,  before  such  entries  become  admissible  in  evi- 
dence.    (Mo.,  Dec,  1914.) 

149.    Does  any  advantage  attach  to  the  employment  of  more  than  one 


36  C.  P.  A.  ACCOUNTING 

volume  for  the  Ledger  of  a  business  requiring  only  one  bookkeeper?    Give 
reasons.     (N.  Y.,  June,  1897;  N.  Y.,  Jan.,  1907.) 

Voucher  System 

150.  Describe  the  voucher  system.  (N.  Y.,  June,  1899*;  Pa.,  May, 
1903*;  Pa.,  Nov.,  1903*;  111.,  Nov.,  1903*;  Md.,  Jan.,  1909*;  Wash.,  May, 
1910* ;  Va.,  Nov.,  1910* ;  Wash.,  June,  1915* ;  Mich.,  June,  1915* ;  Mass., 
Oct.,  1915*;  Cal.,  May,  1916*;  Cal.,  June,  1917*;  A.  I.  of  A.,  May,  1918*; 
111.,  Dec,  1918;  Mich.,  June.,  1919.*) 

151.  State  the  arguments  briefly  for  and  against  the  use  of  voucher 
system.  (N.  Y.,  June,  1899*;  Pa.,  May,  1903;  Pa.,  Nov.,  1903;  Cal., 
June,  1904*;  Pa.,  Nov.,  1906;  111.,  May,  1907*;  Md.,  Jan.,  1909;  Fla., 
Julv,  1909*;  Wash.,  May,  1910*;  Va.,  Nov.,  1910;  Mass.,  April,  1911*; 
111.,'^  May,  1912*;  Mass.,  Oct.,  1915;  Cal.,  Nov.,  1916;  111.,  Dec,  1916; 
Cal.,  June,  1917;  N.  D.,  Aug.,  1917*;  A.  I.  of  A.,  May,  1918;  N.  Y., 
Jan.,  1920.*) 

152.  For  what  classes  of  undertaking  is  the  voucher  system  peculiarly 
desirable?  For  what  classes  of  undertakiniis  is  it  undesirable?  Why? 
(111.,  Nov.,  1903*;  Wash.,  May,  1910*;  Cal.,  May,  1916.) 

153.  Describe  the  Voucher  Record.  (N.  Y.,  June,  1898;  N.  Y.,  June, 
1899;  N.  Y.,  June,  1912;  Wash.,  June,  1915*;  W.  Va.,  May,  1917;  Wash., 
July,  1917.) 

154.  What  is  the  relation  of  the  Voucher  Record  and  Purchase  Journal 
to  the  General  Ledger?     (111.,  Dec,  1916.) 

155.  Define:  Voucher  cheek.     (Wash.,  June,  1915.) 

156.  What  Ls  a  voucher?     (N.  Y.,  Jan.,  1906.) 

157.  Define:  Journal  voucher.     (Wash.,  July,  1917.) 

158.  What  is  your  understanding  of  the  purpose  of  a  voucher?  (N.  Y., 
Jan.,  1906.) 

159.  Sketch  forms  of  disbursement  voucher  and  journal  voucher.  (111., 
May,  1907.) 

160.  State  how  you  would  satisfy  yourself  of  the  correctness  or  regu- 
larity of  absent  vouchers.     (N.  C,  Nov.,  1918.) 

161.  A  finn  whose  accounts  have  been  audited  annually  for  a  number 
of  years,  loses,  through  fire,  all  its  receipted  expense  bills  for  the  past 
year,  which  have  not  been  audited.  All  the  other  records  were  saved. 
The  accountant  is  requested  to  make  the  annual  audit  as  usual.  How 
should  he  proceed  to  satisfy  himself  with  respect  to  the  expenses  for 
which  the  vouchers  are  missing?     (N.  D.,  July,  1919.) 

162.  Would  you  consider  it  necessary  to  place  your  initials  or  some 
other  distinctive  marks  on  each  voucher,  and  if  so,  why?  (La.,  May, 
1913.) 

163.  A  corporation,  in  paying  two  bills,  fastens  the  bills  together,  adds 
to  the  first  bill  of  the  two  the  amount  of  the  second  bill  and  sends  these 


DOUBLE-ENTRY  BOOKKEEPING  37 

with  a  cheek  for  receipt.  "When  returned,  only  the  top  bill  is  receipted. 
Do  you  consider  this  a  valid  voucher?  If  not,  how  could  you  satisfy 
yourself  at  the  office  of  the  corporation?     (Mass.,  June,  1913.) 

164.  How  would  an  auditor  protect  himself  from  duplicate  vouchers 
or  fraudulent  bank  books?  (N.  Y.,  Dec,  1896*;  N.  Y.,  June,  1898*;  Pa., 
Nov.,  1899;  Pa.,  Nov.,  1901*;  Wash.,  May,  1903*;  Pa.,  Nov.,  1903; 
Mich.,  Nov.,  1907;  Md.,  Jan.,  1909*;  Wash.,  May,  1911.*) 

165.  In  an  audit  of  the  books  of  a  corporation  using  the  voucher 
system,  what  means  should  be  adopted  to  prevent  the  reproduction  of 
vouchers  already  passed,  in  cases  where  the  auditor  is  not  permitted  to 
deface  the  vouchers  by  stamps  or  writing?     (N.  Y.,  Jan.,  1902.) 

166.  Give  a  broad  definition  of  vouchers  as  a  means  of  verifying  items 
entered  in  the  books  of  account.     (N.  Y.,  Jan.,  1907.) 

167.  In  an  audit  stipulating  for  the  examination  of  all  vouchers  of 
every  description,  what  would  be  proper  vouchers  for  the  following: 
purchases,  returned  purchases,  sales,  returned  sales,  cash  receipts,  cash 
payments,  journal  entries?  (N.  Y.,  Dec,  1896;  111.,  May,  1906;  Ohio, 
March,  1910;  Mich.,  June,  1912;  Wis.,  April,  1914*;  Wis,,  May,  1916*; 
Va.,  Nov.,  1918.) 

168.  State  your  method  and  procedure  in  an  audit,  in  determining 
that  you  have  seen  all  vouchers,  even  though  some  of  them  have  been 
credited  to  personal  accounts,  and  payments  have  been  made  to  apply 
on  account.     (Colo.,  Dec,  1913;  N.  D.,  Aug.,  1917.) 

169.  How  would  you  vouch  the  following  items  appearing  in  the  books 
of  a  company  you  are  auditing;  and  state  specifically  the  papers  or  docu- 
ments you  would  call  for  in  support  of  the  disbursement : 

1.  The  Rapid  Typewriter  Company — 

Typewriter  purchased  in  exchange  for  old  one.  .$       30.00 

2.  Alex.  Green — 

Real  estate  for  plant  site 7,500.00 

3.  Automatic  Sprinkler  Company — 

Installment  paid  on  sprinkler  system 1,000.00 

4.  John  Mace — 

Stumpage  purchased  for 625.00 

5.  Safety  Trust  Company — 

Par  value  $3,000  bonds 2,970.00 

6.  Machinery  constructed  and  erected  by  the  com- 

pany's staff 10,500.00 

7.  Thomas  Jones,  Salesman — 

Traveling  expenses  for  one  week 73.20 

8.  A  B  Company — 

Note  payable  discounted 987.50 

(111.,  May,  1914.) 

170.  What  is  the  purpose  of  examining  vouchers?  In  a  large  concern 
would  you  attempt  to  inspect  them  all?  If  not,  what  tests  would  you 
make?     (N.  Y.,  Jan.,  1916.) 


38 


C.  P.  A.  ACCOUNTING 


PROBLEMS 


DOUBLE-ENTRY    BOOKKEEPING 


1,    The  following  is  the  Balance  Sheet  of  the  A.  B.  Company,  January 
1915 : 


Cash 

Accounts  Receivable 

Inventories: 

Raw  Material 

Finished  Goods 

Office  Furniture  and  Fix 

tures 

Land 

Buildings 

Machinery 


$52,864 
197,425 

84,268 

31,597 

7,500 

180,000 
150,000 
250,000 


8953,654 


Accounts  Payable 

Dividends  Payable  Preferred 

Stock  Feb.  1,  1915 

Dividends  Payable  Common 

Stock  Feb.  1, 1915 

Mortgage  Bonds  20  Year  6% 

Dated  Jan.  1,  1915 

Premium  on  Bonds 

Capital  Stock  Preferred 

Capital  Stock  Common  .... 

Reserve  for  Bad  Debts 

Surplus 


$35,482 

7,500 

10,000 

100,000 

5.000 

250,000 

500,000 

4,718 

40,954 


$953,664 


The  transactions  for  the  year  ending  January  1,  1916,  Lave  been  as 
follows:  Cash  received  from  customers,  $793,501;  rent  received,  $600. 
There  has  been  purchased  1,232,000  pounds  raw  material  at  20  cents  per 
pound.  Sales  have  been  $823,334;  discount  and  allowance  on  sales,  $23,- 
519;  bad  debts  written  off,  $2,143. 

Disbursements  have  been  made  for:  Accounts  Payable,  $243,356;  Fac- 
tory Expense,  $7,489;  Factory  Labor,  $351,426;  Factory  Repairs,  $23,843; 
Office  Expense,  $1,927;  Selling  Expense,  $52,914;  Salaries,  $58,471;  Taxes, 
$7,853. 

Inventories  January  1,  1916:  Raw  material,  412,595  pounds,  having 
market  value  of  20  cents  per  pound  and  finished  goods,  $30,842. 

The  land  is  estimated  to  be  worth  $200,000.  Semiannual  dividends  of 
3  per  cent  on  preferred  and  2  per  cent  on  common,  declared  in  June  and 
December,  payable  August  1  and  February  1.  Reserves  for  depreciation 
of  building,  3  per  cent ;  machinery,  5  per  cent ;  office  fixtures^  10  per  cent. 
Bad  and  doubtful  debts  reserve  should  be  2  per  cent  of  Accounts  Receiv- 
able. 

Prepare  an  Operating  statement  and  Balance  Sheet  as  on  January  1, 
1916. 

(A.  I.  of  A.,  Nov.,  1917.) 

2.  The  trial  balance  of  the  Interstate  Manufacturing  Company,  on 
June  30,  1918,  after  closing  entries  have  been  made,  is  given  below : 


DOUBLE-ENTRY  BOOKKEEPING 


39 


Patents  and  Goodwill 

Office  Furniture 

Inventory,  June  30,  1918: 

Raw  Material 

Supplies 

Finished  Goods 

Petty  Cash 

Land 

Buildings 

Cash  subject  to  check 

Machinery' 

Accounts  Receivable 

Common  Capital  Stock 

Preferred  Capital  Stock 

Bonds  6%,  50  year  First  Mortgage,  issued  June  30, 

1918 

Premium  on  Bonds 

Preferred  Stock  Dividends,  payable  August,  1918  . 
Common  Stock  Dividends,  payable  August,  1918  .  . 

Reserve  for  Bad  and  Doubtful  Accounts 

Undivided  Surplus 

Accounts  Payable 


$250,000 

8,746 

83,247 

4,932 

42,761 

100 

270,000 

165,000 

69,433 

235,000 

273,842 


$1,403,061 


$500,000 
500,000 

200,000 
20,000 
17,500 
12,500 
8,294 
66,375 
78,392 


$1,403,061 


During  the  year  ending  June  30,  1919,  the  Company  purchased  29,047 
tons  of  raw  material  at  $22  per  ton,  which  was  delivered  before  the  books 
closed.  Of  the  amount  purchased,  payment  has  been  made  for  26,647 
tons. 

They  have  also  made  payments  for  the  following  accounts:  Accounts 
Payable,  $78,392;  Salaries,  $80,360;  Selling  Expense,  $86,017;  Labor, 
$468,932;  Shop  Expense,  $9,461;  Repairs  and  Maintenance,  $30,955; 
Taxes,  $7,842;  Office  Expense,  $2,478;  and  Supplies,  $37,637. 

Customers  have  paid  $1,502,927  in  cash  and  have  been  given  discounts 
amounting  to  $18,395.  Returns  and  allowances  amount  to  $8,474.  Bad 
debts  written  off,  $2,407;  rents  received,  $500;  and  sales,  $1,515,572. 
$50,000  was  borrowed  on  call  on  June  30,  1919,  the  market  value  of  the 
collateral  security  being  $72,100. 

The  inventory  on  June  30,  1919,  is  made  up  of  finished  goods,  $20,495 ; 
supplies,  $8,129;  and  2,163  tons  of  raw  material,  the  market  price  of 
which  is  $24  per  ton.     The  land  is  estimated  to  be  worth  $300,000. 

Semiannual  dividends  of  3^4  per  cent  on  the  preferred  stock  and  21/^ 
per  cent  on  the  common  stock  have  been  paid  from  the  earnings  of  the 
half  year  ending  December  31,  1918.  Dividends  at  the  same  rate  have 
been  declared  on  the  preferred  and  common  stock  for  the  last  half  of  the 
fiscal  year,  payable  in  August,  1919. 

You  are  asked  to  set  up  a  Balance  Sheet  dated  June  30,  1919,  and  ac- 
company it  with  a  statement  which  will  show  correctly  the  operation  of 
the  Company. 

The  following  annual  rates  of  depreciation  are  to  be  assumed :  Buildings, 
3  per  cent;  machinery,  ly^  per  cent;  office  furniture,  10  per  cent.     It  is 


40 


C.  P.  A.  ACCOUNTING 


also  assumed  that  there  should  be  a  reserve  for  bad  and  doubtful  accounts 
equal  to  3  per  cent  of  the  balance  of  accounts  receivable.  Calculate  these 
percentages  to  the  nearest  dollar. 

(Mo.,  Dec,  1914;  Md.,  Oct.,  1916;  Kan.,  May,  1918.*) 

3.  The  books  of  Robert  West,  real  estate  agent,  for  the  year  1900, 
disclose  the  following  opening  and  closing  balances  and  intervening  volume 
of  transactions: 


Titles  of  Accounts 

Balances  Dec.  30,  1905 

Transactions  in  1906 

Balances  Dec.  31.  1906 

Cash 

$9,760.08 
1,060.00 

$137,797.62 
34,656.00 
34,788.00 
34.610.00 
100.934.00 
4,841.40 

$135,893.70 

34,788.00 

34,656.00 

34,788.00 

102,070.00 

5.007.40 

125.00 

3.118.92 

180.00 

$11,664.00 
928.00 

$1,060.00 

2,500,00 

5,929.00 

444.00 

$928.00 

2,678.00 

Clients 

260.00 

104.00 

6.909.00 

610.00 

Fees 

125.00 

3,118.92 

180.00 

1.000.00 
2,000.00 

1,000.00 

2,000.00 

500.00 

500.00 

Capital 

1.647.08 

1,647.08 

$11,580.08 

$11,580.08 

$350,627.02 

$350,627.02 

$16,196.00 

$16,196.00 

An  analysis  of  the  books  afforded  further  information  as  follows: 
Tenants  were  allowed  $71  for  repairs  made  by  them,  which  sum  was 
applied  on  account  of  rent  and  charged  to  owners. 

Owners  were  charged  for  commissions  on  collections,  $869.70;  trade 
creditors'  bills  for  repairs,  $3,566;  and  insurance,  $52. 

Clients  were  charged,  insurance,  $668;  coal,  $906;  fees,  $125;  commis- 
sions on  sales,  $1,004. 

Trade  creditors  presented  bills  for  office  supplies,  $50;  insurance  writ- 
ten, $576;  coal,  $815.40;  they  were  allowed  $180  for  discount  on  settle- 
ments made. 

Commissions  on  sales,  collections,  insurance  written,  and  coal  orders 
were  closed  into  the  General  Commission  account  and  Supplies  account 
was  transferred  to  Expense. 

The  cash  tran?actions  were  as  follows: 

Receipts,  tenants,  $34,717;  clients,  $102,070;  commission  on  sales, 
$1,010.62.  Payments,  owners,  $30,051.30;  clients,  $98,231;  trade  credi- 
tors, $4,661.40;  personal  drawings,  $2,000;  expense,  $950. 

Prepare  an  articulation  statement  showing  in  each  account  the  several 
elements  of  debit  and  credit  and  giving  each  element'  the  title  of  the  articu- 
lation account  wherein  the  contra  credit  or  charge  appears. 

(N.  Y.,  Jan.,  1907.) 

4.  The  following  statements  comprise  the  trial  balances  of  a  business 
at  the  beginning  and  the  end  of  a  fiscal  period,  together  with  the  volume 
of  the  transactions  during  said  period : 


DOUBLE-ENTRY  BOOKKEEPING 


41 


Cash 

Merchandise 

Debtors 

Fixtures 

Creditors 

Loan 

Capital 

Interest  and  discount 

Rent 

Insurance 

Salaries 

Advertising 

Carting 

Expense 

Drawings,  proprietor. 


Trial  Balance 
January  1 


$1,115 
5,050 
3,110 
2,800 


$12,075 


$1,575 

500 

10,000 


$12,075 


Interim 
Transactions 


$16,583 

17,665 

25,135 

505 

18,922 


693 

900 

50 

1,820 

900 

1,705 

1,333 

2,000 


$88,211 


$16,338 
26,874 
24,229 

19,410 
1,000 

360 


$88,211 


Trial  Balance 
December  31 


$1,360 

4.016 
3,305 


333 

900 

50 

1,820 

900 

1,705 

1,333 

2.000 


$17,722 


$4,159 


2,063 

1,500 

10,000 


$17,722 


(a)  The  Sale  Book  shows  sales  posted  to  debtors  to  the  amount  of 
$25,135. 

(6)  The  Journal  shows  allowances  to  debtors  for  returns  of  merchan- 
dise sales,  $1,015,  and  claims  on  creditors  for  returns  of  mer- 
chandise purchases,  $230;  also  application  of  debtors'  balance 
to  settle  creditors'  account  in  the  amount  of  $9,500,  both  ac- 
counts being  in  the  name  of  the  same  correspondent. 

(c)  The  Ledger  shows  that  the  nominal  accounts  entitled  Rent,  In- 

surance and  Office  Salaries  contain  only  cash  charges,  while 
the  nominal  accounts  entitled  Advertising,  Cartage  and  Expense 
show  cash  charges  in  the  total  amounts  of  $100,  $200  and  $773, 
respectively,  all  other  charges  therein  being  by  invoice  duly 
posted  to  creditors'  accounts. 

(d)  The  Merchandise  account  shows  cash  charges  of  $610  and  cash 

credits  of  $1,509  for  cash  purchases  and  cash  sales,  respectively. 

(e)  The  Invoice  Books  show  invoices  posted  to  creditors'  accounts  to 

the  amount  of  $19,410. 
Prepare  an  articulation  statement  showing  in  each  account  the  several 
elements  of  debit  and  credit  and  giving  each  element  the  title  of  the  articu- 
lation account  wherein  the  contra  credit  or  charge  appears. 

(N.  Y.,  June,  1906.») 


CHAPTER  III 
BALANCE  SHEET 

Balance  Account — The  balance  sheet  is  an  outgrowth  of  the  old  practice, 
now  discarded,  of  closing  all  the  accounts  of  the  ledger  at  the  end  of  the 
fiscal  3'ear  by  closing  the  nominal  accounts  through  the  profit  and  loss 
account  into  the  vested  capital  accounts  and  the  real  accounts  into  a 
balance  account,  which  was  itself  closed  when  the  real  accounts  were 
reopened  at  the  start  of  the  new  period/ 

Balance  Sheet — The  balance  sheet  may  be  defined  as  a  statement  showing 
the  financial  position  of  a  business,  its  assets  and  liabilities,  the  capital 
employed  therein,  as  well  as  any  reserves,  surplus,  or  deficiency  there  may 
be  at  a  specific  date/  As  to  content,  the  balance  sheet  differs  from  the 
prior-closing  trial  balance  in  not  containing  the  nominal  accounts,'  but 
is  identical  with  a  post-closing  trial  balance.*  However,  the  purpose  of 
the  post-closing  trial  balance  is  merely  to  prove  that  the  ledger  is  in 
balance,*  while  the  balance  sheet  pui'ports  showing  the  financial  condition 
of  the  business  in  a  manner  intelligible  to  the  lay  reader." 

Form  of  Balance  Sheet — There  are  two  general  classes  of  balance  sheets, 
namely:  (a)  The  account  form;  and  (b)  the  report  form.'  The  account 
form  is  frequently  called  the  technical  foi'm  of  balance  sheet.* 

The  account  form  places  the  asset  accounts  on  one  side  and  the  liability 
and  capital  accoiuits  on  the  other.  If  the  assets  are  on  the  left  the  balance 
sheet  is  in  the  American  account  form;  if  on  the  right,  it  is  in  the  English 
account  form."  The  fact  that  deductions  are  not  used  in  accounts  does 
not  prevent  the  valuation  resei*ves,  etc.,  being  deducted  from  the  related 
asset  accounts  in  account  form  balance  sheets."" 

There  are  two  general  classes  of  report  form  balance  sheets,  namely: 
(a)  Same  as  the  account  form  if  the  liability  and  capital  accounts  w^ere 
placed  under  insfead  of  to  the  right  of  the  assets;"  and  (b)  a  form  in 
which  the  assets  are  listed  and  their  totals  extended,  and  under  them  the 
liabilities  listed  and  their  totals  extended  and  deducted  from  the  total  of 
the  assets,  the  difference  being  the  net  worth  of  the  business,  which  is  then 
itemized  and  total  extended." 

Classification  of  accounts  is  vitally  important  for  balance  sheet  pur- 
poses. However,  it  is  seldom  that  two  accountants  will  agi-ee  as  to  the 
degree  of  classification  necessary.  This  results  in  some  accountants 
dividing  the  assets  into  (a)  fixed  assets,  (b)  permanent  investments,  (c) 
investments  of  reserve,  (d)  working  assets,  (e)  current  assets,  and  (f) 
deferred  charges,"  while  othfers  merely  classify  them  as  to  (a)  current 
assets,   (b)  defen-ed  charges,  and   (c)  fixed  assets."     More  uniformity  is 


'For  explanation  of  superior  figures  see  page  337. 

42 


BALANCE  SHEET  43 

found  on  the  liability  and  capital  side  of  the  balance  sheet,  as  usually  the 
liabilities  are  classified  as  to  (a)  current  liabilities,  (b)  deferred  credits 
to  income,  and  (c)  fixed  liabilities,  while  the  net  worth  accounts  are 
separated  into  (a)  capital  stock  accounts  and  (b)  surplus  accounts." 

There  is  little  uniformity  as  to  the  order  in  which  the  classes  of  accounts 
appear  on  the  balance  sheet.  In  general,  hoAvever,  there  are  two  plans  of 
marshalling  the  balance  sheet  accounts.  One  lists  the  current  assets  and 
liabilities  first,'"  and  the  other  lists  the  fixed  assets  and  lialiilities  first." 
A  variation  from  the  latter  plan  is  to  list  the  capital  stock  accounts, 
followed  in  order  by  the  fixed  liabilities,  current  liabilities,  and  surplus." 
As  the  reader  is  usually  moi"e  interested  in  the  current  than  in  the  fixed 
accounts,  it  seems  preferable  to  list  the  current  items  first.  It  seems 
decidedly  objectionable  to  place  the  liabilities  between  the  net  worth 
accounts,  as  so  doing  obscures  both  the  amount  of  the  liabilities  and  the 
net  worth  of  the  business.  There  is  a  decided  difference  of  opinion  among 
accountants  as  to  the  place  to  be  assigned  to  deferred  charges  to  opera- 
tions and  deferred  credits  to  income.  Some  accountants  place  these  items 
after  the  current  and  fixed  assets  and  liabilities,  respectively,"  while  others 
place  them  between  the  current  and  fixed  accounts."  As  the  deferred 
items  have  a  much  shorter  life  than  the  fixed  items,  the  latter  arrangement 
seems  to  be  preferable. 

A  special  arrangement  of  the  accounts  in  the  account  form  of  balance 
sheet,  called  the  "double  account"  form,  divides  the  accounts  into  two 
distinct  tables.^*  In  the  first  table  the  fixed  assets  are  placed  against  the 
fixed  liabilities  and  net  worth  accounts."  The  credit  balance  of  the  first 
table  representing  the  amount  of  free  or  available  capital  is  brought  down 
into  the  second  table  in  which  the  current  assets  and  current  liabilities 
are  listed." 

Still  another  noteworthy  point  in  the  form  of  a  balance  sheet  is  the 
number  of  columns  used.  Usually  three  columns  are  found  to  be  suffi- 
cient. What  is  gained  in  classification  by  the  use  of  more  than  three 
columns  is  more  than  offset  by  the  increased  complexity  of  the  form,  the 
additional  difficulty  in  having  the  statement  typed,  and  what  is  important 
for  C.  P.  A.  candidates,  the  increased  time  it  takes  to  prepare  the 
statement. 

Model  Balance  Sheet — For  ordinary'  purposes,  it  is  comparatively  un- 
important which  of  the  recognized  forms  of  balance  sheet  is  used.  How- 
ever, it  is  vitally  important  that  all  student,s  limit  themselves  to  the  use 
of  only  one  form  after  they  have  become  familiar  with  all  the  forms. 
This  specialization  will  gi'eatly  enhance  the  speed  and  accuracy  of  the 
student,  the  two  requirements  which  the  C.  P.  A.  candidates  find  the  most 
difficult  to  acquire. 

Valuation  in  Balance  Sheet — The  accounts  shown  in  a  balance  sheet 
are  valued  at  book  value,  that  is,  the  fixed  assets  are  valued  at  cost  less 
depreciation,"^  the  working  assets  at  cost  or  market  whichever  is  the  lower" 
and  the  other  current  assets  at  the  estimated  cash  value.  As  there  is  no 
asset,  other  than  cash,  whose  actual  value  can  be  definitely  ascertained,  a 
balance  sheet  is  a  statement  of  opinion  and  not  a  statement  of  fact."* 


44 


C.  P.  A.  ACCOUNTING 


A.  B.  COMPANY 
BALANCE  SHEET 
Docomber  31,  1916 


Assets 
Current  Assets: 
Cash: 

Petty  Cash 

Cash  on  Deposit . 


Notes  Receivable 

Accounts  Receivable 

Less  Reserve  for  Bad  Accounts. 


Interest  Accrued  on  Investments . 

Investments 

Advances  to  Salesmen 

Inventories : 

Material 

Work  in  Process 

Finished  Goods 


Deferred  Charges: 
Unexpired  Insurance: 
Building  and  Machinery . 
Finished  Goods 


Prepaid  Taxes  Real  Estate . 


Fixed  Assets: 

Land 

Buildings 

Less  Reserve  for  Depreciation. 

Machinery 

Less  Reserve  for  Depreciation. 


Office  Furniture. 
Goodwill 


Liabilities  and  Capital 
Current  LiabiUties: 

Notes  Payable 

Accounts  Payable 

Interest  on  Mortgage .  .  . 
Accrued  Payrolls: 

Factory 

Office 


Fixed  LiabiUties: 

Mortgage  on  Plant 

Net  Worth: 

Capital  Stock 

Less  Unissued  Capital  Stock. 


Surplus : 

Start  of  Period 

Addition  During  Period . 


BALANCE  SHEET 


45 


Since  the  reserves  for  depreciation  and  doubtful  accounts  and  the  valua- 
tions pLnced  upon  the  inventories  are  merely  matters  of  opinion,  the  profit 
and  loss  statement  and  the  resultant  surplus  are  also  matters  of  opinion 
and  not  of  actually  ascertained  facts." 

The  auditor's  responsihility  as  to  the  balance  sheet  largely  centers  around 
the  valuation  of  the  accounts.  The  sources  of  data  as  to  values  are  (a) 
books  of  account,  (b)  original  purchase  invoices,  (c)  catalogs,  (d) 
experience  in  the  business,  (e)  appraisal  concerns,  and  (f)  capitalization 
(goodwill)   of  earnings.^* 

Condensed  Balance  Sheets — The  purposes  served  by  balance  sheets  are 
(a)  internal  use  by  executives,  (b)  formal  stockholders'  report,  (c)  report 
to  supervising  commissions,  (d)  annual  report  to  state,  (e)  basis  of  appli- 
cation for  credit,  and  (f)  basis  for  issuing  new  secui-ities."*  When  used 
for  some  of  these  pui-poses,  the  amount  of  detailed  information  in  the 
balance  sheet  itself  should  be  reduced  to  a  minimum.  This  is  accomplished 
by  grouping  similar  accounts  under  general  headings  with  the  detailed 
information  contained  in  the  schedules,  references  to  which  are  made  on 
the  condensed  or  general  balance  sheet.'"  Tlie  use  of  only  the  controlling 
accounts  for  the  accounts  and  notes  receivable  and  payable  in  the  balance 
sheet  with  lists  of  the  subsidiary  accounts  appended  is  the  most  common 
illustration  of  the  supporting  schedules."  The  same  idea  can  be  used  for 
the  plant  and  property,  investments,  inventories,  deferred  charges,  etc.'' 

Coynparative  Balance  Sheets — One  of  the  most  valuable  statements  an 
auditor  can  prepare  for  his  client  is  a  balance  sheet  which  will  show  the 
increases  and  decreases  Avhich  occurred  during  the  period  in  the  various 
accounts.  Such  a  statement  will  detect  any  tendency  to  tie  up  the  funds 
in  fixed  assets  or  inventories  of  merchandise  and  will  warn  the  client 
against  unusual  increases  in  current  liabilities. 

Practically  all  of  the  comparative  balance  sheets  prepared  by  accountants 
do  not  classify  the  accounts'^  thereby  partially  obscuring  the  changes  in 
the  classes  of  accounts.  The  model  comparative  balance  sheet  given  below, 
however,  classifies  the  accounts'*  and  shows  the  increases  and  decreases 
which  occurred  within  the  classes  of  accounts : 

A.  B.  COMPANY 
COMPARATIVE  BALANCE  SHEET 


Assets 

Jan.  1,  1907 

Jan.  1,  1908 

Increase 

Decrease 

Current  Assets: 

Cash 

S800.00 
5,400.00 
4,200.00 

$4,800.00 
1,000.00 
2,300.00 

$4,000.00 

Accounts  Receivable  .... 

$4,400.00 

Inventories 

1,900.00 

.1510,400.00 

$8,100.00 

$4,000.00 

$6,300.00 

Deferred  Charges: 

Insurance  Prepaid 

$400.00 

$200.00 

$200.00 

Fixed  Assets: 

Plant  and  Equipment.. .  . 

$17,000.00 

$15,000.00 

$2,000.00 

$27,800.00 

$23,300,00 

$4,000.00 

$8,500.00 

40 


C.  P.  A.  ACCOUNTING 

A.  B.  COMPANY 
COMPARATIVE  BALANCE  SHEET 


Liabilities  and  Capital 

Jan.  1,  1907 

Jan.  1,  1908 

Increase 

Decrease 

Current  Liabilities: 

Notes  Payable 

Accounts  Payable 

$5,000.00 
11,800.00 

$5,000  00 

$500.00 

11,800.00 

$16,800.00 

$500.00 

$16,300.00 

Valuation  Reserves: 

Reserve  for  Bad  Debts.. . 

$300.00 
700.00 

$300.00 
700.00 

Reserve  for  Depreciation. 

$7,000.00 
4,800.00 

$1,000.00 

$1,000.00 

Net  Worth: 

Capital  Stock 

$10,000.00 

$17,000.00 
4,800.00 

Net  Profits 

$10,000.00 

$21,800.00 

$11,800.00 

$27,800.00 

$23,300.00 

$11,800.00 

$16,300.00 

Summary 


Current  Liabilities: 
Decrease 

$16,300,00 

Current  Assets : 
Decrease 

$6,300.00 
4,000.00 

Increase 

$2,300.00 

200.00 
2,000.00 
7,000.00 

Deferred  Charges: 
Decrease 

Fixed  Assets: 

Decrease 

Capital  Stock : 

Increase 

11  500  00 

Net  Profits 

$4  800  00 

The  above  summary'"  is  not  the  only  recognized  method,  for  many  ac- 
countants would  draw  up  the  summary  as  follows  :™ 

Funds  Available: 

Decrease  in  Current  Assets $6 , 300. 00 

Decrease  in  Deferred  Charges 200. 00 

Decrease  in  Fixed  Assets 2,000. 00 

Increase  in  Net  Worth 11, 800 .  00 


$20,300.00 


BALANCE  SHEET 

A  p plied  as  Follows: 

Increase  in  Current  Assets $4,000.00 

Decrease  in  Current  Liabilities 16,300.00 


$20,300.00 


Assets — The  assets  of  a  going  eoncern  are  its  property  of  all  sorts  ap- 
plicable or  subject  to  the  payment  of  debts,  and  amounts  expended  for 
the  benefit  of  future  periods."  Assets  of  concerns  about  to  liquidate  in- 
clude only  such  items  as  are  applicable  to  the  payment  of  the  lialiilities'* 
and  do  not  include  the  items  exempted  by  law  or  the  amounts  expended 
for  the  benefit  of  future  payments.  The  term  "net  assets"  means  the 
j)roprietor's  interest™  or  the  difference  between  the  total  assets  and 
liabilities  of  a  business. 

Kinds  of  Assets — The  classes  into  which  assets  may  be  divided  are : 

(a)  Current  (sometimes  called  circulating,  active,  or  floating)  or  assets 
maturing  within  a  short  time  or  those  convertible  into  cash  by  the  ordinary 
routine  of  business.^" 

(b)  Working  or  trading  assets  or  a  sub-group  of  curi-ent  assets  including 
the  inventories  of  raw  material,  goods  in  process  of  manufacture,  and 
finished  goods." 

(c)  Quick  (sometimes  called  liquid)  or  a  sub-group  of  current  assets 
including  only  those  assets  very  readily  converted  into  cash.  An  advance 
to  a  salesman  is  a  current  asset,  but  it  is  not  a  quick  asset." 

(d)  Fixed  (sometimes  called  permanent,  passive,  or  capital)  or  assets 
necessary  for  the  conduct  of  the  business,  which  are  not  intended  for  sale.*" 

(e)  Secret  or  assets  representing  the  excess  of  the  appraised  value  of 
property  over  its  book  value." 

(f)  Wasting  (sometimes  called  diminishing)  or  fixed  assets  which  ai'e 
consumed  in  direct  ratio  with  operations,  or  merely  through  efflux  of 
time."  The  term  "wasting  assets"  includes  both  the  assets  subject  to 
depreciation  and  those  subject  to  depletion,*"  but  the  term  is  most  com- 
monly used  in  reference  to  natural  resources. 

(g)  Contingent  or  assets  that  are  subject  to  pi'eceding  events  or  situa- 
tions which  may  or  may  not  happen." 

(h)  Accrued  or  a  sub-group  of  ciirrent  assets  which  accumulate 
gi'adually  with  the  efHux  of  time.** 

(i)  Deferred  charges  to  operations  or  those  portions  of  expenses  paid 

in  one  period  and  allocated  to  future  periods.* 

Kinds  of  Liabilities — The  liabilities,  the  debts  or  obligations  due  by  a 
firm  to  its  ereditors,°*  may  be  classified  as  follows : 

(a)  Current  (sometimes  called  floating,  active,  circulating,  or  quick)  or 
liabilities  maturing  in  a  short  time  which  will  be  liquidated  in  the  normal 
course  of  business." 

(b)  Accrued  or  eui-rent  liabilities  which  accumulate  gradually  with  the 
efflux  of  time." 


48  C.  P.  A.  ACCOUNTING 

(c)  Deferred  credits  to  income  or  income  received  during  one  period 
applicable  to  future  periods." 

(d)  Contingent  oi-  liabilities  subject  to  preceding  events  or  situations 
which  may  or  may  not  happen." 

(e)  Fixed  (sometimes  called  capital)  or  liabilities  which  will  not  mature 
for  a  long  time.*^  The  assumption  that  fixed  liabilities  represent  that  part 
of  the  capital  income  which  has  been  invested  in  fixed  assets'"  is  unwar- 
ranted because  a  firm  may  have  large  fixed  assets  and  no  fixed  liabilities 
at  all. 

(f)  Funded  or  liabilities  for  which  definite  provision  for  payment  has 
been  made,  usually  secured  by  a  mortgage  or  other  lien." 

(g)  Unfunded  or  liabilities  for  which  no  definite  provision  has  been 
made.** 

(h)  Bonded  or  liabilities  evidenced  by  an  issue  of  bonds.^  Bonded  debt 
may  be  either  secured  or  unsecured  or  either  funded  or  unfunded. 

Accrued  Items — There  are  two  bases  of  accounting.  One,  the  cash  basis, 
is  to  record  the  expenses  and  incomes  only  when  they  affect  the  cash  or 
personal  accoimts."*  The  other  takes  into  records  all  of  the  expenses  and 
incomes  including  the  accruals  on  the  accounting  principle  that  the  primary 
connection  between  the  net  assets  and  the  resulting  net  income  is  the 
matter  of  earnings  and  expense  incurred,  and  not  one  of  income  received 
in  cash  and  expenses  paid  in  cash."" 

Accruals  should  be  placed  on  the  books  at  the  end  of  the  fiscal  period.*^ 
The  plea  that  they  are  practically  the  same  at  the  beginning  and  end  of 
the  period  should  be  disregarded  in  the  interest  of  accuracy.  Some  ac- 
countants do  not  differentiate  between  accrued  and  deferred  items,  but  it 
is  advisable  to  make  the  distinction." 

Deferred  Items — There  are  two  kinds  of  deferred  charges  to  operations, 
namely,  (a)  items  which  will  benefit  the  future  periods  such  as  the  unex- 
l)ired  insurance,'"  and  (b)  items  which  do  not  benefit  future  periods  such 
as  the  cost  of  equipment  suddenlj'^  made  obsolete  through  a  new  invention." 
Deferred  assets  may  be  classified  as  to  whether  or  not  they  have  realizable 
values,  viz.,  prepaid  insurance  has  a  realizable  value  while  organization 
expenses  have  not.  Prepaid  insurance,  rents,  and  interest  and  expense 
inventories  are  the  most  usual  illustrations  of  deferred  charges.*' 

The  deferred  credits  to  income  include  income  or  gains  received  but 
applicable  to  su])sequent  periods,  such  as  discounts  and  premiums  on 
securities,  unearned  fees,  and  similar  items."'  Aggregates  of  credits  meas- 
uring the  amount  of  cash  receipts  for  the  distribution  of  which  informa- 
tion is  lacking,  are  also  deferred  liabilities."*  The  contention  that  deferred 
credits  to  income  represent  liabilities  not  payable  in  cash  but  in  goods  or 
services,""  is  incorrect  because  items  such  as  sales  paid  for  in  advance 
include  two  elements,  namely,  cost  and  profit.  Even  the  profit  element  can 
not  be  taken,  as  prospective  profits  can  not  be  capitalized." 

Contingent  Items — When  a  note  receivable  is  endorsed  and  discounted 
the  note  is  changed  from  an  ordinary  to  a  contingent  asset,  and  a  contingent 
liability  is  created."    When  the  drawer  pays  the  note,  both  the  contingent 


BALANCE  SHEET  4f) 

asset  aiul  liability  are  canceled.'^  Other  illustrations  of  contingent  assets 
are  pending  lawsuits  tor  damages  against  another  firm  or  individual,  or 
a  conditional  bequest  in  a  will.'"  Notes  receivable  discounted,  pending  law- 
suits for  damages  against  firm^  guarantees,  long-term  leases,  purchases 
for  future  delivery,  surety  bonds,  and  accommodation  indorsements  are 
the  most  common  contingent  liabilities.'*  Except  for  the  notes  receivable 
discounted  which  shovdd  be  shown  as  assets  among  the  other  notes  and 
liabilities  under  the  caption  "notes  receivable  discounted,""  contingent 
assets  and  liabilities  are  not  ordinarily  shown  on  the  books.'" 

Contingent  items,  other  than  notes  receivable  discounted,  are  usually 
shown  in  foot  notes  on  the  balance  sheet,''  although  they  may  be  shown  in 
the  body  of  the  balance  sheet  in  the  appropriate  place,  indented  and  not 
carried  out  to  the  significant  money  column.'"  If  desired,  the  captions  "con- 
tingent assets"  and  "contingent  liabilities"  may  appear  on  the  balance 
sheet.'*  If  the  notes  receivable  account  is  credited  when  notes  are  dis- 
counted, the  contingent  liability  would  merely  be  mentioned  in  a  footnote 
to  the  balance  sheet.""  However,  when  the  notes  receivable  discounted  ac- 
count is  credited  when  notes  are  discounted,  the  contingent  liability  is 
shown  on  the  balance  sheet  by  deducting  the  notes  receivable  discounted 
from  the  ordinary  notes  receivable.'^ 

It  is  imperative  that  the  auditor  inquii'e  of  the  executives  as  to  whether 
any  guarantees  or  indorsements  exist  and  whether  any  security  is  available 
to  protect  tlie  business."^  If  the  officials  are  interested  in  other  concerns, 
the  investigations  should  be  very  searching."  The  minute  books  should 
be  scrutinized."*  Certificates  as  to  the  notes  receivable  discounted  at  the 
bank  should  be  obtained  from  the  banker.""  Certificates  should  also  be 
obtained  from  the  beneficiaries  under  a  guarantee,  as  some  guarantees  are 
more  of  a  real  liability  than  a  contingency.""  The  unfilled  purchase  orders 
should  also  be  examined  to  ascertain  whether  the  financial  condition  of 
the  business  has  been  jeopardized."' 

Capital — Professor  Charles  J.  Bullock,  in  "The  Elements  of  Economies," 
defines  capital  as  "all  the  intermediate  products  which  man  creates  for  the 
pui'pose  of  employing  them  in  the  production  of  finished  commodities." 
This  definition  of  capital  from  the  economic  viewpoint  makes  the  capital 
of  the  business  include  all  the  assets  of  a  business.  However,  from  an 
accounting  viewpoint  the  capital  of  a  business  is  the  excess  of  its  assets 
over  its  liabilities."* 

Working  capital  is  the  excess  of  current  assets  over  current  liabilities"* 
after  the  valuation  reserves  have  been  deducted  from  their  related  current 
assets.*"  The  term  is  also  used  occasionally  as  a  synonym  for  quick  assets.*' 
In  courts,  capital  is  classified  into  fixed  and  circulating  capital,  the  terms 
meaning  respectively  fixed  and  circulating  assets.*^  Capital  is  sometimes 
classified  as  owned  and  unowned,  the  former  meaning  that  contributed 
to  the  business  by  the  proprietor  or  accumulated  as  profits  and  the  latter 
meaning  that  borrowed  or  obtained  from  creditors.*' 

The  term  "deficiency"  signifies  the  insufficiency  of  the  assets  of  a  firm 
to  discharge  its  liabilities;   that  is,   deficiency  is  the   exact  opposite  of 


50 


C.  P.  A.  ACCOUNTING 


capital."     The  terai  "deficit"  has  two  meanings,  namely,   (a)   an  excess 
of  expenses  over  income,""  and  (b)  the  excess  of  liabilities  over  assets.'" 

Proprietorship  Accounts — In  general,  proprietorship  accounts  are  of 
two  classes,  namely,  accounts  with  the  original  investments,  and  accounts 
with  subsequent  changes  in  net  worth.  The  following  chart  summarizes 
the  original  investments  accoiuits  :*' 

ORIGINAL  CAPITAL  ACCOUNTS 

Proprietor's  Investment  Account 

Partner's  Individual  Investment  Ac- 
count 

Partner's  Individual  Investment  Ac- 
counts 

Membership,  Certificate  of  Indebted- 
ness and  Capital  Accounts 

Capital  Stock 

Capital  Stock 

None 

Capital  Stock 


TYPE   OF  ORGANIZATION 
Sole  Proprietors. 
Copartnerships 

Joint  Ventures 

Associations  and  Societies 

Joint  Stock  Associations 
Stock  Corporations 
Non-stock  Corporations 
Trusts  and  Holding  Companies 


A  similar  chart  for  the  accounts  recording  changes  in  net  worth  subse- 
quent to  original  investment  is  as  follows:** 

CAPITAL  ACCOUNTS  SHOWING 
CHANGES   IN  NET   WORTH   SUB- 
SEQUENT TO  INVESTMENTS 
Proprietor's  Personal  Accounts  and  Re- 
serves 
Partner's  Individual  Personal  Accounts 

and  Reserves 
Partner's  Individual  Personal  Accounts 
Surplus  and  Reserves 
Undivided  Profits,  Surplus  and  Reserves 
Undivided  Profits,  Surplus  and  Reserves 
Surplus  and  Reserves 
Undivided  Profits,  Surplus  and  Reserves 


TYPE  OF  ORGANIZATION 

Sole  Proprietors 

Copartnerships 

Joint  Ventures 
Associations  and  Societies 
Joint  Stock  Associations 
Stock  Corporations 
Non-stock  Corporations 
Trusts  and  Holding  Companies 


Relation  of  Proprietor  to  Business — Viewing  the  business  as  an  entity, 
it  has  been  claimed  that  the  proprietor  is  a  creditor  of  the  business,  whose 
claim  against  the  business  is  subject  to  the  prior  claims  of  the  creditor. 
The  exponents  of  this  opinion  show  the  right  side  of  the  balance  sheet 
under  the  heading  of  "liabilities."**  As  the  liabilities  have  contractual 
rights  in  the  assets,  while  the  proprietorship  has  merely  residual  rights, 
a  distinction  must  be  made  between  proprietorship  and  liabilities.^"*  The 
fact  that  losses  and  gains  affect  proprietoi'ship  and  do  not  affect  the 
liabilities  also  shows  that  there  is  a  distinction  between  net  worth  and 
liability  accounts.'*'  Holders  of  this  opinion  use  the  heading  "liabilities 
and  capital"  over  the  right  side  of  the  balance  sheet.'"' 

To  avoid  confusion  by  including  proprietorship  and  contractual  obliga- 
tions under  the  caption  "liabilities,"  and  to  lessen  the  danger  of  misunder- 
standing, the  use  of  the  term  "equities"  has  been  suggested  to  represent 


BALANCE  SHEET  51 

the  right-hand  membei-  of  the  balance  sheet  equation,  viz.,  assets  equal 
equities.""  This  term  covers  all  elements  of  ownership — control,  risk,  and 
income,  the  proprietorship  equities  having  more  of  the  control  and  risk 
and  residual  rights  to  the  income,  and  liabilities  having  a  small  amount 
of  the  control  and  risk,  and  contractual  rights  to  the  income.'"* 


C.  p.  A.  ACCOUNTING 


QUESTIONS 

balance  sheet 

Statement 

1.  Define:  Balance  Sheet.  (Pa.,  Nov.,  1899*;  Pa.,  Nov.,  1901*;  Pa., 
May,  1902*;  N.  Y.,  Jan.,  1906;  N.  Y.,  Oct.,  1907*;  111.,  Dec.,  1907;  R.  I., 
Dec.,  1907*;  Md.,  Jan.,  1909*;  N.  Y.,  Feb.,  1910;  Va.,  Oct.,  1911*;  Mich., 
June,  1912*;  La.,  May,  1913;  Colo.,  Dec,  1913;  Wis.,  April,  1914;  Kan., 
May,  1916;  Wis.,  May,  1916*;  W.  Va.,  May,  1917;  Cal.,  June,  1917;  N.  C, 
Aug.,  1917;  N.  C,  Nov.,  1918*;  Va.,  Nov.,  1918;  Iowa,  Dec,  1918*;  N.  C, 
June,  1919*;  N.  C,  Sept.  1919.*) 

2.  In  a  general  way,  what  is  the  difference  between  a  Financial  State- 
ment and  a  Balance  Sheet?     (Mich.,  June,  1910.) 

3.  Wherein  does  a  Trial  Balance  differ  from  a  Balance  Sheet?  (N.  Y., 
Jan.,  1897;  N.  Y.,  June,  1898*;  Pa.,  Nov.,  1899;  Pa.,  May,  1903;  Md., 
Oct.,  1903;  N.  J.,  1904-1909;  Pa.,  May,  1905;  Md.,  Jan.,  1909;  N.  Y., 
June,  1909;  Va.,  Nov.,  1910;  Ohio,  Nov.,  1913;  Ohio,  Nov.,  1915;  Cal., 
May,  1916;  Cal.,  Nov.,  1916;  Mich.,  Dec,  1916*;  N.  C,  Aug.,  1917*; 
Ohio,  Oct.,  1919.*) 

4.  State  how  the  following  differ:  a  trial  balance  after  closing  and  a 
Balance  Sheet.     (Md.,  Jan.,  1909.) 

5.  Show  the  relations  of  a  trial  balance  to  the  Balance  Sheet.  (N.  Y., 
June,  1898.) 

6.  Is  the  trial  balance  essential  to  the  making  of  the  Balance  Sheet, 
and  if  so,  why;  and  if  not,  why  not?     (N.  J.,  1904-1909.) 

7.  What  is  the  purpose  of  a  Balance  Sheet?  (Ohio,  Nov.,  1913;  Ohio, 
Nov.,  1915*;  Cal.,  May,  1916;  Ohio,  Nov.,  1916;  Ohio,  Nov.,  1917;  Ohio, 
Nov.,  1918*;  N.  C,  Nov.,  1918*;  Ohio,  Oct.,  1919.) 

8.  In  submitting  a  Balance  Sheet  at  the  end  of  the  fiscal  year,  are 
you  stating  facts  or  opinions?  Give  reasons.  (111.,  May,  1907*;  Mich., 
June,  1912*;  Ohio,  Nov.,  1915*;  Mass,  Oct.,  1916;  Ohio,  Nov.,  1918*; 
N.  Y.,  Jan.,  1920.*) 

9.  On  the  Balance  Sheet  prepared  by  you  from  the  books  of  a  client, 
state  which  items  are  matters  of  fact  and  which  matter  of  opinion.  (N.  Y., 
Jan.,  1920.) 

10.  State  the  general  theory  of  the  Balance  Sheet.  (N.  Y.,  June, 
1899.) 


BALANCE  SHEET 


53 


11.  Is  strict  accuracy  as  to  the  facts  possible  in  a  Balance  Sheet  f 
Why?     (Ohio,  Nov.,  1913*;  Ohio,  Nov.,  1917*;  Ohio,  Oct.,  1919.) 

12.  Assuming  that  the  amounts  are  correctly  stated,  point  out  several 
ways  in  which  a  Balance  Sheet  may  fail  of  its  purpose.  (Ohio,  Nov., 
1917.) 

13.  What  is  meant  by  a  Balance  account?  (N.  Y.,  Oct.,  1907;  B.  I., 
Dec,  1907;  N.  Y.,  Jan.,  1914.) 

14.  How  was  the  "Balance  account"  used  originally?  What  has  be- 
come of  it  in  modern  bookkeeping  practice?     (N.  Y.,  Jan.,  1914.) 

15.  How  would  you  determine  the  character  of  assets  the  surplus  of 
a  business  consists  of,  and  what  would  be  the  advantage  of  such  inforiiia- 
ion?     (N.  C,  June,  1916.) 

16.  Give  method  of  preparing  a  Balance  Sheet  where  ledgers  have 
not  been  closed.     (111.,  May,  1905.) 

17.  Why  must  the  revenue  account  be  completed  before  a  Balance 
Sheet  can  be  prepared?     (N.  Y.,  Oct.,  1907;  R.  I.,  Dec,  1907.) 

18.  What  is  your  interpretation  of  the  difference  between  the  total 
debit  balances  and  the  total  credit  balances  of  the  accounts  belonging  to 
the  capital  division  and  to  the  current  division  of  a  Balance  Sheet? 
(N.  C,  June,  1916.) 

19.  Draw  up  a  short  report  on  the  following  Balance  Sheet,  criticizing 
such  items  as  you  consider  abnormal : 


Buildings $^7;500 


Machinery 

Sundry  Stock 

Cash 

Bills  Receivable 

Customers 

Goodwill  and  Patents 


12,500 
90,000 
3,200 
6,800 
20,000 
30,000 


$2.50.000 


Capital  and  Surplus $155,000 

Current  Liabilities 87,500 

Suspense  Account 7,500 


$250,000 


(N.  Y.,  Jan.,  1901;  Wash.,  Sept.,  1917.) 

20.  What  are  the  most  important  things  an  auditor  has  to  certify  as 
regards  a  Balance  Sheet?  (N.  Y.,  Jan.,  1902*;  Md.,  Oct.,  1903;  Mich., 
July,  1906.*) 

21.  In  making  up  a  general  statement  of  assets  and  liabilities,  what 
groups  of  accoimts  constitute  assets  and  what  constitute  liabilities? 
(N.  Y.,  Dec,  1893.) 

22.  On  what  theory  does  the  English  form  of  Balance  Sheet  differ 
from  the  continental  and  American  form?  Give  an  argument  either  for 
or  against  the  English  fonii.     (N.  Y.,  June,  1899.) 

23.  The  nigli  Pressure  Valve  Manufacturing  Coiiniaiiy  submit  the 
following  trial  balance  taken  from  their  books  in  connection  with  the  facts 
enumerated;  (he  organization  is  highly  skilled  and  dependent  upon  co- 
operation. Give  your  reconunendations  as  to  what  should  be  done  to 
serve  the  interest  of  the  business : 


54 


C.  P.  A.  ACCOUXTING 


Plant,  $14,500;  Accounts  Receivable,  $4,000;  Cash,  $1,510;  Capital 
Stock,  $15,000;  Loans  Payable,  $3,000;  Accounts  Payable,  $4,000;  Notes 
Payable,  $3,500;  Sales,  $16,000;  Cost  of  Sales,  $10,000;  General  Expense, 
$2,000;  Selling  Expense,  $3,500;  Deductions  from  Income,  $4,000.  The 
valves  are  in  demand  and  in  general  use.     (N.  Y.,  June,  1919.) 

24.  Criticize  the  following  Balance  Sheet  from  both  the  auditor's 
standpoint  and  that  of  the  Company's  financial  condition : 

"X."  "Y."  "Z."  Co.  Balance  Sheet,  December  31,  1915. 


Assets 

Real  estate,   building,   plant,   machinery,   equipment 

and  goodwill 

Investments  in  stocks  and  bonds  at  cost  (market  value 

$100,000) 

Current  assets: 
Inventories: 

Raw  materials  (market  value) 

Finished  stock  at  selling  prices,  less  discount  at  5% 

Consignment  (selling  value) 

Supplies  (estimated) 


Accounts  and  bills  receivable  including  advances  to 

employees 

Stock  in  treasury  (unissued) 

Preferred $150,000 

Common 237,000 


Investments  in  subsidiary  companies . 
Cash  and  miscellaneous  items 


Liabilities 
Capital  stock: 

Preferred  stock 

Common  stock 

Bonds  and  bankers'  loans 

Reserves : 

For  depreciation 

Less  renewal  expenditures  written  off . 


Balance  (debit)  . .  .  . 

P'or  bad  debts 

Other  contingencies . 


Current  liabilities: 
Accounts  payable .  . 
Other  indebtedness . 
Accrued  items 


Surplus . 


$230,000 

200,000 

50,000 

200,000 


$680,000 


225,000 


387,000 

425,000 
50,000 


$110,000 
135,000 


$25,000 

5,000 

50,000 


$115,000 

231,000 

52,000 


$2,000,000 
150,000 


1,767,000 


$3,917,000 


$1,000,000 

1,800,000 

625,000 


30,000 

398,000 
64,000 


$3,917,000 


(Mass.,  Oct.,  1916.) 


BALANCf]  SHEET  55 

25.  In  the  case  of  a  company  which  publishes  an  annual  Balance  Sheet 
but  no  Profit  and  Loss  account,  state  whether  or  not  you  would  recom- 
mend to  your  client  that  the  profits  earned  during  the  year,  less  dividends 
paid,  be  shown  on  the  face  of  the  Balance  Sheet.  Give  your  reasons, 
(A.  L  of  A.,  May,  1920.) 

26.  How  should  you  arrange  the  items  on  a  Balance  Sheet  of  a  large 
corporation?  Of  a  partnership?  (N.  Y.,  Jan.,  1897* ;  N.  Y.,  June,  1899* ; 
N.  Y.,  Jan.,  1907*;  111.,  May,  1907*;  Ohio,  March,  1910*;  Mich.,  June, 
1912*;. Ohio,  Nov.,  1913*;  Mich.,  Dec.,  1913*;  Mich.,  Dec,  1914*;  111., 
May,  1915;  Mass.,  Oct..  1915*;  Cal.,  May,  1916*;  N.  C,  June,  1916*; 
Mass.,  Oct.,  1916*;  Ohio,  Nov.,  1917*;  Ohio,  Nov.,  1918*;  Iowa,  Dec, 
1918;  N.  Y.,  Jan.,  1919*;  N.  Y.,  Jan.,  1920.*) 

27.  Is  the  form  of  a  Balance  Sheet  a  matter  of  principle  or  conven- 
tion?    (Ohio,  Nov.,  1917*;  Ohio,  Oct.,  1919.) 

28.  What  is  the  mechanism  of  the  double  form  Balance  Sheet?  Ex- 
plain the  connection  between  its  sections,  stating  the  theory  of  its  organ- 
ism.    (N.  Y.,  Jan.,  1914;  Cal.,  May,  1916*;  Md.,  Oct.,  1919.) 

29.  In  drawing  up  a  Balance  Sheet,  is  it  desirable  to  show  the  assets 
and  liabilities  by  groups,  and  if  so,  into  what  groups  would  you  classify? 
Give  reasons  for  your  classification.     (N.  Y.,  Jan.,  1906;  Va.,  Nov.,  1918.*) 

30.  What  is  meant  by  marshaling  the  accounts  of  a  Balance  Sheet? 
(Cal.,  May,  1916.) 

31.  State  two  different  theories  in  relation  to  the  presentation  of  a 
Balance  Sheet  as  far  as  classification  is  concerned.  What  is  tlie  reasoning 
on  which  they  are  based?     (N.  Y.,  June,  1912;  Cal.,  Nov.,  1916.*) 

32.  Under  what  theory  or  theories  would  you  classify  the  Balance 
Sheet  accounts  into  three  cardinal  divisions?  (N.  C,  June,  1919;  N.  C, 
Sept.,  1919.) 

33.  Give  a  simple  equation  for  the  contents  of  a  Balance  Sheet.  (Ohio, 
Nov.,  1913;  Minn.,  Oct.,  1916.) 

34.  In  making  up  a  business  statement  or  a  Balance  Sheet,  why  are 
the  assets  placed  as  debits  and  the  liabilities  as  credits?  Are  there  any 
exceptions  to  this  rule?     (N.  Y.,  June,  1900;  N.  J.,  1904-1909.*) 

35.  What  special  points  in  the  Balance  Sheet  of  a  company  aside  from 
the  correctness  of  the  figures  require  careful  consideration  by  the  auditor? 
(111.,  May,  1908.) 

36.  What  is  the  extent  of  an  auditor's  responsibility  in  respect  to  the 
classification  of  assets  and  liabilities  in  a  balance  sheet  which  he  certifies? 
(A.  I.  of  A.,  May,  1918;  Ga.,  May,  1919.) 

37.  What  schedules  should  support  the  Balance  Sheet  in  a  thorough 
audit  of  a  manufacturing  concern  owning  its  entire  plant?  (Md.,  Dec, 
1917.) 

38.  Outline  the  forms  of  the  Profit  and  Loss  statement  and  of  the 
Balance  Sheet  as  submitted  by  the  Federal  Reserve  Board  in  their  pro- 


56  C.  P.  A.  ACCOUNTING 

posal  for  a  uniform  system  of  accounting  to  be  adopted  by  manufacturing 
and  merchandising  concerns.     (Wis.,  April^  1918.) 

39.  Suppose  you  had  certified  the  Balance  Sheet  of  a  manufacturing 
concern,  and  were  asked  by  a  stockholder  why  you  had  certified  it  as 
correct  when  some  of  the  assets  were  not  salable  at  the  figures  placed 
against  them,  what  reply  would  you  make?     (Fla.,  April,  1908.) 

40.  On  what  important  points  will  the  Balance  Sheet  of  a  trading  and 
non-trading  company  differ?     (N.  D.,  June,  1914.) 

41.  What  should  the  financial  statement  of  a  trading  company  show? 
(N.  D.,  June,  1914.) 

42.  Explain  the  difference  between  cost  and  book  value.  (La.,  May, 
1913.) 

43.  Explain  the  principles  that  underlie  the  mechanism  of  the  state- 
ment of  resources  and  show  their  application.     (N.  Y.,  June,  1913.) 

44.  Prepare  a  chart  of  a  Balance  Sheet,  showing  the  relationship  of 
all  factors  involved  in  its  construction  that  will  exhibit  a  view  of  Avhat, 
in  your  opinion,  is  the  meaning  of  a  Balance  Sheet.     (N.  C,  June,  1920.) 

Assets 

45.  Define:  Assets.     (N.  Y.,  Dec,  1898;  N.  J.,  1904-1909.) 

46.  Define:  Current  assets.  (N.  Y.,  June,  1901;  N.  Y.,  Jan.,  1902; 
N.  Y.,  Jan.,  1904;  Mich.,  July,  1906;  Ohio,  March,  1910*;  Va.,  Oct., 
1911*;  La.,  May,  1913*;  Colo.,"Dec.,  1913;  Mo.,  Dec,  1913*;  Kan.,  Dec, 
1915;  Mo.,  Dec,  1915;  Ohio,  Nov.,  1916;  III.,  May,  1917;  W.  Va.,  1917; 
Wash.,  July,  1917;  Va.,  Nov.,  1918*;  Iowa,  Dec.,'l918.) 

47.  What  are  quick  assets?  (N.  Y.,  Dec,  1898;  N.  Y.,  June,  1900*; 
Mich.,  Dec,  1906;  Md.,  Jan.,  1909*;  Va.,  Oct.,  1911*;  Mich.,  June,  1914*; 
Iowa,  Dec,  1918.) 

48.  Prepare  an  imaginary  statement  of  the  net  quick  assets.  (Okla., 
Nov.,  1919.) 

49.  What  items  would  you  designate  as  "quick  assets"?  (La.,  May, 
1913.) 

50.  Define:  Floating  assets.  (N.  Y.,  June,  1899;  Cal.,  July,  1904*; 
111.,  May,  1908;  Fla.,  July,  1909;  N.  D.,  July,  1916;  111.,  Dec,  1916.) 

51.  Define:  Capital  assets.  (N.  Y.,  June,  1902;  N.  Y.,  Jan.,  1904; 
La.  May,  1913*;  Wash.,  Nov.,  1913;  Mo.,  Dec,  1913*;  Kan.,  Dec,  1915; 
Mo.,  Dec,  1915;  Va.,  Nov.,  1918;  Iowa,  Dec,  1918.) 

52.  Define:  Active  assets.     (Va.,  Nov.,  1910.) 

53.  Define:  Secret  assets.     (Colo.,  Dec,  1913.) 

54.  Define:  Cash  assets.     (N.  Y.,  Dec,  J896;  111.,  Nov.,  1903.) 

55.  Define:  Fixed  assets.  (N.  Y.,  Dec,  1896;  N.  Y.,  Dec,  1898 
N.  Y.,  June,  1899;  Cal.,  June,  1904*;  Mich.,  July,  1906;  111.,  May,  1908 
Fla.,  July,  1909;  Ohio,  March,  1910*;  Mich.,  June,  1910;  Va.,  Nov.,  1910 


BALANCE  SHEET  57 

La.,  May,  1913*;  Wash.,  Nov.,  1913;  Colo.,  Dec.,  1913;  Mich.,  June, 
1914*;  N.  D.,  July,  1916;  Ohio,  Nov.,  1916;  111.,  May,  1917;  N.  C,  Aug., 
1917;  Va.,  Nov.,  1918.*) 

56.  Define:  Net  quick  assets.     (III.,  Dec.,  1918.) 

57.  Define:  Passive  assets.     (N.  Y.,  June,  1900.) 

58.  State  cases  where  the  condition  known  as  "diminishing  assets"  is 
likely  to  arise.  How  should  such  cases  be  treated?  (N.  Y.,  June,  1902; 
N.  Y.,  June,  1914*;  A.  L  of  A.,  May,  1918*;  N.  Y.,  June,  1918.*) 

59.  Give  instances  of  the  manner  in  which  fixed  assets  and  floating 
assets  affect  the  stability  and  the  credit  of  a  business.     (Fla.,  July,  1909.) 

60.  Give  two  examples  of  fixed  assets  in  one  business  which  become 
floating  assets  in  another  business.  (Mich.,  June,  1912;  Mass.,  June, 
1913.) 

61.  State  three  examples  of  fixed  assets  in  some  particular  business, 
which  are  generally  floating  assets.     (Mass.,  June,  1913.) 

62.  In  the  Balance  Sheet  of  a  company,  as  prepared  by  the  secretary, 
you  find  the  following  items : 

Under  "Capital  Assets": 

(a)  Factory  real  estate,  buildings,  plant  and  machinery. 

(b)  Real  estate  held  for  investment. 

(c)  Investments  in  and  advances  to  another  company  for  purposes 

of  control. 

(d)  Franchises  having  a  fixed  term. 
Under  "Current  Assets" : 

(e)  Company's  treasury  stock  (carried  at  $.50  on  the  dollar). 

(/)     Raw  material,  finished  product,  and  inventory  of  oflBee  sup- 

plies  and  stationer3\ 
(g)     Advances  to  officials  of  the  company  (unsecured). 
(h)     Insurance  premiums  paid  by  the  company  on  a  policy  of  the 
life  of  the  company's  president,  in  which  the  company  is 
beneficiarj'. 
(i)     Due  by  customers, 
(j)     Sinking  fund  investments. 
(A;)     L^nexpired  fire  insurance  premium. 
(l)     Cash  in  bank  and  on  hand. 
Discuss  the  correctness  or  otherwise  of  the  above  classification  of  items 
under  capital  and  current  assets,  giving  reasons  for  your  opinions,  and 
criticizing  the  items  generally. 

(Mo.,  Dec,  1914;  Okla.,  Nov.,  1919.) 

63.  Define:  Resource.     (N.  Y.,  June,  1899.) 

Liabilities 

64.  Define:  Liabilities.  (N.  Y.,  Dec.,  1898;  N.  Y.,  June,  1899;  N.  J., 
1904-1909.) 

65.  Define:  Current  liabilities.    (N.  Y.,  June,  1901;  Mich.,  July,  1906; 


58  C.  P.  A.  ACCOUNTING 

Va.,  Nov.,  1910;  Wash.,  Nov.,  1913*;  Mich.,  June,  1914*;  Kan.,  Dec, 
1915;  Mo.,  Dec,  1915;  Ohio,  Nov.,  1916;  W.  Va.,  May,  1917;  Va.,  Nov., 
1918*;  Iowa,  Dec,  1918.) 

66.  Define:  Floating  liabilities.  (N.  Y.,  Dec,  1896;  N.  Y.,  Dec,  1897*; 
N.  Y.,  June,  1900;  Wash.,  May,  1903*;  N.  Y.,  Jan.,  1904;  Mich.,  Dec, 
1906*;  Md.,  Jan.,  1909*;  Mass.,  June,  1910*;  Colo.,- Dec,  1913*;  N.  D., 
July,  1916;  Ohio,  Nov.,  1916.) 

67.  Define:  Fixed  liabilities.  (N.  Y.,  Dec,  1896;  N.  J.,  1904-1909; 
Mich.,  July,  1906;  La.,  May,  1913*;  Mich.,  June,  1914*;  N.  D.,  July, 
1916;  Ohio,  Nov.,  1916;  N.  C,  Aug.,  1917;  Va.,  Nov.,  1918.*) 

68.  Define:  Funded  debt.  (N.  Y.,  Dec,  1897;  Wash.,  May,  1903; 
N.  Y.,  Jan.,  1904;  Md.,  Jan.,  1909;  Ohio,  March,  1910*;  Mass.,  June, 
1910;  Va.,  Nov.,  1910;  Colo.,  Dec,  1913;  Kan.,  Dec,  1915;  Mo.,  Dec, 
1915.) 

69.  Define:  Bonded  indebtedness.     (Wash.,  Nov.,  1913.) 

70.  Define:  Capital  liabilities.  (La.,  May,  1913*;  Kan.,  Dec,  1915; 
Mo.,  Dec,  1915;  Iowa,  Dec,  1918.) 

71.  Define  the  following  accounting  and  business  term :  Passive  lia- 
bilities.    (N.  Y.,  June,  1900.) 

72.  Explain  the  difference  between  gross  and  net  floating  debts,  and 
what  is  generally  meant  by  the  term  "floating  debt"  without  the  word 
"gross"  or  "net"  preceding  it.     (111.,  Nov.,  1904.) 

73.  A  distinction  is  made  between  funded  debt  and  unfunded  debt. 
Please  define  and  compare,  discussing  the  advantages  and  disadvantages, 
if  any,  attaching  to  each.     (Mich.,  Dec,  1916*;  A.  I.  of  A.,  Nov.,  1918.) 

Accrued  Items 

74.  Define:  Accrued.  (N.  Y.,  Jan.,  1911*;  Mich.,  Dec,  1914*;  Kan., 
May,  1916.) 

75.  State  the  difference  between  the  accrual  and  the  cash  basis  of  ac- 
counting.    (Kan.,  Dec,  1915;  Mo.,  Dec,  1915.) 

76.  Define:  Accrued  liability.     (111.,  Dec,  1916.) 

77.  In  the  preparation  of  a  Balance  Sheet,  explain  the  basis  upon 
which  you  would  ascertain  that  accrued  liabilities  were  properly  valued. 
(Wis.,  April,  1914.) 

78.  Name  some  accrued  liabilities.  (Md.,  Dec,  1917;  Mich.,  June, 
1919.») 

79.  An  auditor  is  called  upon  to  verify  a  Balance  Sheet  and  upon  in- 
vestigation he  finds  that  unexpired  insurance,  interest  paid  in  advance 
on  discounted  notes,  taxes  accrued,  interest  accrued  on  demand  notes, 
bonded  indebtedness,  royalties,  etc.,  are  not  included  in  the  same.  He 
is  informed  that  it  has  not  been  the  custom  of  the  corporation  to  include 
in  their  Balance  Sheet  such  items,  as  they  offset  one  another,  and  that 
the  directors  do  not  desire  any  change  in  the  practice  Ihey  have  adopted. 


BALANCE  SHEET  59 

Discuss  this  proposition,  stating  reasons  for  your  conclusions.     (111.,  May, 
1907.) 

80.  A  corporation  wishes  to  get  figures  of  its  earnings  early  each 
month.  Besides  its  regular  income,  it  has  bonds  and  stocks  from  which 
the  interest  and  dividends  are  received  either  quarterly  or  semi-annually. 
It  has  trouble  in  getting  some  of  its  expense  bills  promptly,  as  some  come 
in  quarterly,  semi-annually,  and  even  yearly.  State  the  method  of  getting 
out  promptly  with  as  little  work  as  possible  these  monthly  figures.  (Mass., 
June,  1913.) 

Deferred  Items 

81.  What  do  you  understand  to  be  the  meaning  of  the  word  "deferred" 
when  applied  to  accounts?     (Ind.,  May,  1918.) 

82.  Define :  Deferred  charges.  (N.  Y.,  Jan.,  1911;  Wash.,  Nov.,  1913; 
Colo.,  Dec,  1913;  N.  Y.,  Jan.,  1914;  111.,  May,  1914;  Mich.,  June,  1914; 
Wash.,  June,  1915;  Minn.,  Oct.,  1916;  Ohio.  Nov.,  1916;  Mich.,  Dec, 
1916*;  111.,  May,  1917;  Wash.,  July,  1917;  Mass.,  Oct.,  1917;  111.,  Dec, 
1918;  A.  I.  of  A.,  May,  1919.) 

83.  Name  some  deferred  charges  to  expense.  (Mass.,  Oct.,  1917;  Md., 
Dec,  1917;  Md.,  May,  1918*;  Mich.,  June,  1919.*) 

84.  Mention  two  common  kinds  of  deferred  charges.  (Ohio,  Nov., 
1916.) 

85.  Define:  Prepaid  expenses  and  state  how  you  would  treat  them  in 
a  Balance  Sheet.     (Va.,  Nov.,  1910.) 

86.  What  is  the  theory  applying  to  deferred  debits  shown  in  a  Balance 
Sheet?     (N.  Y.,  Jan.,  19i7.) 

87.  Under  what  circumstances  would  you  permit  a  client  to  defer  items 
of  unquestionable  expense?     (Cal.,  May,  1916.) 

88.  What  do  you  understand  by  deferred  credits?  Give  illustration. 
(Mich.,  June,  1914;  111.,  Dec,  1916.) 

89.  To  what  extent  would  you  consider  it  necessary  to  verify  expense 
paid  in  advance,  and  what  reference  to  such  verification  would  you  make 
in  your  report?     (Mass.,  June,  1913.) 

90.  State  the  nature  of  the  items  appearing  in  a  Balance  Sheet  which 
may  properly  be  classified  as  "deferred  credits"  by  a  private  concern. 
(N.  Y.,  Jan.,  1920.) 

91.  What  is  the  auditor's  attitude  with  respect  to  organization  ex- 
penses, advertising,  and  other  expenditures  which  the  client  claims  to 
have  a  value  extending  over  more  than  one  year?     (N.  Y.,  Jan.,  1916.) 

Contingent  Items 

92.  Define:  Contingent  assets.  (Md.,  Jan.,  1909;  La.,  May,  1913*; 
Kan.,  May,  1916;  N.  C,  Aug.,  1917;  A.  I.  of  A.,  Nov.,  1920.*) 


60  C.  P.  A.  ACCOUNTING 

93.  Give  illustrations  of  contingent  assets.  (Md.,  Jan.,  1909;  Wis., 
April,  1914;  Kan.,  May,  1916;  A.  I.  of  A.,  Nov.,  1920.) 

94.  How  would  you  treat  contingent  assets  on  the  books?  (Md.,  Jan., 
1909;  Mo.,  Dec,  19i4;  A.  I.  of  A.,  Nov.,  1920.) 

95.  How  would  you  treat  contingent  assets  on  a  Balance  Sheet?  (Md., 
Jan.,  1909;  Wis.,  April,  1914;  Mo.,^Dee.,  1914;  A.  I.  of  A.,  Nov.,  1920.*) 

96.  Define:  Contingent  liabilities.  (N.  J.,  1904-1909;  Mich.,  Dec, 
1906;  Md.,  Jan.,  1909;  N.  Y.,  Feb.,  1910;  Mass.,  June,  1910;  Ya.,  Nov., 
1910;  111.,  May,  1913*;  La.,  May,  1913*;  Colo.,  Dec,  1913;  Mich.,  June, 
1914;  Mass.,  Oct.,  1914;  Kan.,  Dec,  1915;  Mo.,  Dec,  1915;  Kan.,  May, 
1916;  Ind.,  June,  1916*;  N.  D.,  July,  1916*;  Ohio,  Nov.,  1916;  Mich., 
Dec,  1916*;  111.,  May,  1917;  N.  C,  Aug.,  1917;  N.  D.,  Aug.,  1917; 
A.  I.  of  A.,  Nov.,  19i7;  Ind.,  Nov.,  1918;  111.,  Dec,  1918;  Iowa,  Dec, 
1918;  A.  I.  of  A.,  May,  1919;  A.  I.  of  A.,  Nov.,  1920.*) 

97.  Give  illustrations  of  contingent  liabilities.  (Fla.,  April,  1907; 
Fla.,  April,  1908;  Md.,  Jan.,  1909;  N.  Y.,  Feb.,  1910;  Wash.,  June,  1912; 
111.,  May,  1913;  Wis.,  April,  1914;  Mass.,  Oct.,  1914;  Mass.,  Oct.,  1915; 
Kan.,  Mav,  1916;  Ind.,  June,  1916;  N.  D.,  July,  1916;  Ohio,  Nov.,  1916; 
N.  D.,  Aug.,  1917;  111.,  Dec,  1918*;  A.  I.  of  A.,  Nov.,  1920.) 

98.  How  should  contingent  liabilities  be  shown  on  the  Balance  Sheet? 
(Fla.,  April,  1907;  Fla.,  April,  1908;  Md.,  Jan.,  1909;  N.  Y.,  Feb.,  1910 
Mass.,  June,  1910*;  Va.,  Nov.,  1910;  Wash.,  June,  1912;  111.,  May,  1913* 
Wis.,  April,  1914;  Mass.,  Oct.,  1914*;  Mo.,  Dec,  1914;  Mass.,  Oct.,  1915 
Ind.,  June,  1916;   Ohio,  Nov.,  1916;  N.  D.,  Aug.,  1917.*) 

99.  State  how  contingent  liabilities  should  be  treated  in  the  books. 
(Fla.,  April,  1907;  Fla.,  April,  1908;  Md.,  Jan.,  1909;  Wash.,  June,  1912; 
Mich.,  June,  1913*;  Colo.,  Dec,  1913*;  Mo.,  Dec,  1914;  Mich.,  Dec, 
1916;  A.  I.  of  A.,  Nov.,  1920.*) 

100.  Explain  how  you  would  treat  actions  pending  against  your  client 
in  your  report  on  audit.     (Iowa,  Dec,  1918.) 

101.  How  should  a  guarantee  given  that  machinery  sold  will  last  five 
years  be  treated  on  the  Balance  Sheet?     (Ind.,  May,  1918.) 

102.  (o)  Explain  the  treatment  you  would  give  the  following  in  the 
books  of  Account;  (6)  State  the  counterbalancing  or  offsetting  accounts; 
(c)  Explain  how  they  would  appear  in  the  Balance  Sheet:  (1)  Notes  re- 
ceivable discounted;  (2)  actions  pending  against  your  client;  (3)  cumu- 
lative preferred  dividends  payable;  (4)  liability  as  guarantor  for  third 
parties;  (5)  liability  as  accommodation  signer  on  note;  (6)  contingent 
liabilities  under  contract;  (7)  unpaid  balances  on  contracts  not  yet  ful- 
filled; (8)  collateral  in  possession  of  your  banker  to  secure  paj'ment  of  a 
note.     (Wis.,  April,  1915.) 

103.  What  contingent  liabilities  may  be  encountered  and  what  steps 
would  you  take  to  ascertain  their  existence  in:  (a)  a  wholesale  hardware 
corporation;  (b)  a  manufacturing  corporation;  (c)  a  partnership?  (Cal., 
May,  1916.) 


BALANCE  SHEET  *  61 

104.  What  is  an  auditor's  responsibility  and  what  steps  should  he  take 
in  connection  with  contingent  liabilities?     (111.,  Dec.,  1918.) 

105.  Under  what  conditions  would  you  carry  contingent  liabilities 
among  other  liabilities  on  a  Balance  Sheet  and  what  occurs  when  the  con- 
tingency ceases  to  exist?     (Cal.,  June,  1917.) 

106.  Give  examples  of  such  assets  and  liabilities  not  usually  found  on 
books  of  account,  as  should  be  considered  by  the  auditor  when  preparing 
an  Income  and  Profit  and  Loss  account  at  the  close  of  the  fiscal  period. 
(N.  Y.,  June,  1911.) 

107.  In  the  preparation  of  a  Balance  Sheet  of  a  manufacturing  com- 
pany how  would  you  handle  large  patent  infringement  suits  which  your 
client  would  probably  lose?     (Cal.,  June,  1917.) 

108.  Define:  Actual  liability.     (A.  I.  of  A.,  May,  1919.) 

Net  Worth 

109.  State  the  object  of  the  Capital  account.  (N.  Y.,  Dec,  1898; 
Ohio,  Nov.,  1913*;  Ohio,  Nov.,  1915.) 

110.  Differentiate  between  the  economic  and  the  accounting  use  of 
the  term  ''capital."  (Ohio,  Nov.,  1913;  Kan.,  Dec,  1915;  Mo.,  Dec, 
1915.) 

111.  Define:  Capital.  (N.  Y.,  Dec,  1896;  N.  Y.,  Dec,  1897;  111.,  Nov., 
1903;  Cal.,  June,  1904;  N.  J.,  1904-1909;  Wash.,  Sept.,  1907;  N.  Y., 
Jan.,  1911;  N.  D.,  June,  1914;  N.  C,  Aug.,  1917;  Iowa,  Dec,  1918.*) 

112.  Define  briefly:  Guarantee  capital.     (Cal.,  May,  1916.) 

113.  Define:  Working  capital.  (N.  Y.,  Dec,  1897;  Wash.,  March, 
1909;  Wash.,  Nov.,  1913;  Mo.,  Dec,  1913*;  Mich.,  Dec,  1914;  Cal.,  Nov., 
1916;  Ohio,  Nov.,  1916;  A.  I.  of  A.,  May,  1921.) 

114.  From  what  items  or  classes  of  items  in  a  Balance  Sheet  would 
you  determine  the  amount  of  working  capital  in  the  business?  (Wash., 
March,  1909.) 

115.  Define:  Floating  capital.     (N.  Y.,  June,  1900.) 

116.  Define:  Fixed  capital.     (N.  Y.,  June,  1900.) 

117.  How  would  you  differentiate  between  circulating  and  fixed  capital? 
Illustrate  the  distinction  that  you  would  draw,  giving  examples.  (N.  Y., 
Jan.,  1919.) 

118.  Define:  Loan  capital.  (N.  Y.,  Dec,  1896;  III.,  Nov.,  1903;  Cal., 
May,  1916.) 

119.  Discuss  from  both  standpoints  the  proposition  that  capital  may 
be  properly  regarded  in  any  set  of  books  as  a  liability.  (Md.,  Oct.,  1903*; 
111.,  Nov.,  1903;  N.  Y.,  June,  1909;  Mich.,  June,  1915;  Ohio,  Nov.,  1915*; 
Kan.,  Dec,  1915;  Mo.,  Dec,  1915;  Iowa,  Dec,  1918.) 

120.  Why  is  capital  always  shown  on  a  Balance  Sheet  as  a  liability? 
(N.  Y.,  Dec,  1897.) 

121.  Define:  Deficit.     (Va.,  Nov.,  1918.) 


62  C.  P.  A.  ACCOUNTING 

122.  Distinguish  between  capital  and  capital  stock.  (Ohio,  Nov.,  1915; 
Cal.,  May,  1916*;  Ohio,  Oct.,  1919.) 

123.  How  do  the  accounts  of  a  corporation  and  of  a  copartnership 
differ  in  the  statement  of  capital?     (N.  Y.,  Jan.,  1904.) 

12-4.  What  names  are  given  to  accounts  that  represent  the  excess  of 
assets  over  liabilities?  Differentiate  these  names  in  their  application  to 
various  kinds  of  business.     (N.  Y.,  June,  1898.) 

125.  Define  the  statement  of  net  worth,  also  describe  its  function. 
(N.  C,  Nov.,  1918.) 

126.  What  is  the  net  worth  of  a  business  and  name  the  accounts  be- 
longing to  the  net  worth  division  of  a  Balance  Sheet?     (N.  C,  June,  1916.) 

127.  How  would  you  determine  the  net  worth  or  net  insolvency? 
(N.  D.,  July,  1916.) 

128.  State  how  you  would  verify  the  net  worth  of  a  business.  (N.  C, 
June,  1916.) 


P.AT.ANCK  SHEET 


63 


PROBLEMS 


BALANCE  SHEET 


1.    From  the  following  comparative  Balance   Sheet,   make  statement 
showing  disposition  of  income: 


Assets 

Plant  and  Equipment 

Inventory,  Material 

Inventory,  Supplies 

Cash  in  Bank 

Cash  on  Hand 

Accounts  Receivable 

Interest  on  Accounts  Receivable 

Notes  Receivable 

Workmen's  Compensation  Commission 

Deferred  Insurance 

Office  Supplies 

Taxes 

Treasury  Stock 

Total 

Liabilities 

Capital  Stock 

Surplus .  . . .  : 

Accounts  Payable 

Notes  Payable 

Reserve  for  Interest 

Reserve  for  Taxes 

Reserve  for  Freight  and  Allowances  .  .  . 

Reserve  for  Hospital  Fund 

Reserve  for  Depreciation 

Total 


Net  profit  for  the  year,  $33,492.86. 

Dividends  paid  during  the  year,  $28,850. 

Surplus  adjustment,  debit,  $290.21. 

(W.  Va.,  May,  1919.) 

2.  In  an  investigation  of  the  accounts  of  the  American  Products 
Company  you  find  that  the  Balance  Sheets  of  the  company  for  the  past 
four  years  are  as  follows : 


1916 


$37,277.11 

$36,551.03 

24,334.05 

29,650.17 

6,077.69 

6,070.91 

2,100.90 

1.716.31 

6.44 

1.44 

10,792.56 

6,512.01 

34.58 

2.848.71 

121.71 

■  287.98 

650.28 

90.00 

95.00 

41.67 

400.00 

$84,413.40 

$81,247.15 

$26,000.00 

$26,000.00 

47,378.46 

43,025.81 

789.42 

147.81 

4,000.00 

6.12 

387.57 

1,051.41 

215.85 

750.00 

8,885.98 

7,022.12 

$84,413.40 

$81,247.15 

1915 


C4 


C.  P.  A.  ACCOUNTING 


Year  Ending  December  31 


1915 

1916 

1917 

1918 

$30,000 
75,500 
48,600 
8,000 
10,000 
25,000 
40,000 
36,000 

189,000 

15,000 

5,000 

$30,000 
80,500 
53,800 
8,500 
10,000 
25,000 
58,000 
48,000 

196,000 

12,500 

6,000 

$45,000 

80,500 

60,000 

8,500 

10,000 

$45,000 

95,000 

83,500 

9,000 

5,000 

66,000 

202,000 

20,000 

7,000 

125,000 

229,500 

9,500 

8,000 

$482,100 

$528,300 

$499,000 

$609,500 

$75,000 

50,000 

30,000 

,  75,000 

162,500 
10,000 
10,000 
24,000 
5,000 
40,600 

$75,000 
50,000 
20,000 
85,000 

177,200 
16,500 
12,500 
32,000 
8,000 
52,100 

$75,000 
60,000 
10,000 
90,000 
87,400 
18,000 
40,000 
42,000 
10,000 
66,600 

$75,000 
100,00j 

100,000 
90,600 
25,000 
65,000 
63,000 
12,000 
78,900 

$482,100 

$528,300 

$499,000 

$609,500 

Assets 

Land 

Buildings 

Machinery 

Furniture  and  Fixtures 

Bonds 

Capital  Stock  in  Subsidiarvr  Co..  . 
Advances  to  Subsidiary  Company 

Inventories 

Accounts  Receivable 

Cash 

Prepaid  Expenses 

Liabilities 

Common  Stock 

Preferred  Stock 

Bonded  Debt 

Bills  Payable 

Accounts  Payable 

Accrued  Wages 

Accrued  Taxes 

Reserve  for  Depreciation 

Reserve  for  Contingencies 

Surplus 


The  net  profits  were  double  the  amount  of  dividends  paid  each  year. 

Prepare  a  statement  reflecting  the  disposition  of  the  profits  earned  and 
other  funds  provided  each  year  and  the  change  in  working  capital  since 
December  31,  1915.  (Wis.,  May,  1919.) 

3.  From  the  following  comparative  Balance  Sheets  of  the  A.  B.  C. 
Company  at  December  31,  1917,  and  December  31,  1918,  prepare  a  short 
statement  showing  the  funds  realized  during  the  year  and  the  disposition 
made  thereof: 


* 

Dec.  31,  1917 

Dec.  31,  1918 

Assets 

Caoital  Assets 

$600,000 

]  ,000,000 

850,000 

200,000 

20,000 

$900,000 

1,160,000 
800  000 

(Replacement  values  as  shown  by  appraisal 
were  used  at  December  31,  1918) 
Inventories 

Accounts  Receivable 

Cash 

550  000 

Deferred  Charges 

10,000 

$2,670,000 

$3,420,000 

BALANCE  SHEET 


Oo 


Liabilities 

Capital  Stock 

Bonds  (issued  at  par) 

Capital  Surplus  representing  excess  of  sound 
replacement  value  of  appraisal  at  December 
31  1918,  over  the  book  value  of  capital  assets 
at  that  date 

Bank  Loans  

Accounts  Payable 

Reserve  for  Depreciation  and  Replacements  . . . 
(The  reserve  at  December  31.  1918,  represents 
the  difference  between  the  replacement  and 
sound  value  of  the  appraisal  at  December  31, 
1918) 

Surplus 


Dec.  31,  1917 


$1,000,000 


750,000 
500,000 
100,000 


320,000 


Dec.  31,  1918 


$2,670,000 


$1,000,000 
500,000 


150,000 
400,000 
600,000 
200,000 


570,000 


$3,420,000 


Note — The  profits  for  the  year  were  $450,000,  and  dividends  were  paid 
during  the  year  amounting  to  $200,000.  The  sum  of  $100,000  was  charged 
to  operation  for  depreciation  during  the  year  and  $50,000  was  charged 
against  the  Reserve  for  Replacements.  (A.  I.  of  A.,  Nov.,  1919.) 

4.  (a)  What  is  the  amount  of  the  net  working  capital  in  the  following 
Balance  Sheet: 

Balance  Sheet,  Jan.  1,  1915 


Real  Estate $10,000 

Patents 8,000 

Building.s 55.260 

Cash 9,320 

Inventories 32.600 

Interest  Prepaid 1,600 

Accounts  Receivable 40,200 


$156,980 


Capital  Stock $50,000 

Bonds 20,000 

Notes  Payable 10,000 

Reserve  for  Bad  Debts. . . .  5,350 

Accounts  Payable 32,502 

Reserve   for   Depreciation, 

Buildings 20,000 

Surplus 19,128 

$156,980 


(&)   What  is  the  amount  of  the  net  working  capital  in  the  Balance 
Sheet  of  the  same  company  as  of  January  1,  1916,  which  here  follows: 

Balance  Sheet,  Jan.  1,  1916 


Real  Estate $17,000 

Patents 7,000 

Buildings 63,520 

Cash 3,260 

Inventories 38,710 

Interest  Prepaid 820 

Accounts  Receivable 42,200 


$177,510 


Capital  Stock $50,000 

Bonds 30,000 

Notes  Payable 20,000 

Reserve  for  Bad  Debts. . . .  6,240 

Accounts  Payable 25,620 

Reserve  for   Depreciation, 

Buildings 35,200 

Surplus 10.450 

$177,510 


66  C.  P.  A.  ACCOUNTING 

Profits,  as  per  Profit  and  Loss  account  for  the  year,  amounted  to  $6,322, 
and  a  30  per  cent  dividend  was  declared  and  paid. 

(c)   Submit  a  statement  accounting  for  the  increase  or  decrease  in  the 

working:  capital. 

(Wis.,  May,  1916.) 

5.  The  bookkeejer  of  a  manufacturing  concern  could  produce  only 
the  following  statement  from  its  records  on  January  1,  1907 : 

Manufacturing  Fxpenses $4,622.89 

Capital  Stock 10,000.00 

Plant  and  Equipment 17,500.00 

Gross  Sales 8,469.10 

First  Mortgage  bond  (due  December  31,  1907) 15,000.00 

Material  and  SuppUes  (inventory) 4,289.34 

Notes  Payable   5,000.00 

Accounts  Receivable 5,423.23 

Accounts  Payable   2,436.28 

Interest  on  Bonds  ,(7  months) 393.75 

Interest  on  Notes  and  Accounts  Payable 282.40 

Cash 832.14 

On  January  1,  1907.  the  management  changes,  and  you  are  later  re- 
tained as  a  public  accountant  to  conduct  an  examination  and  prepare  a 
Balance  Sheet  as  of  January  1,  1908, 

You  find  that  during  the  preceding  year  the  directors  have  subscribed 
in  cash  to  $7,500  additional  capital  stock  and  have  retired  all  the  notes 
and  old  accounts  payable  and  that  no  interest  was  paid  on  these  accounts 
for  the  year.  You  also  find  that  the  plant  and  the  equipment  was  re- 
valued at  $15,000  and  5  per  cent  of  this  amount  was  charged  off  to 
provide  for  depreciation,  while  an  additional  2^/2  per  cent  was  ordered 
placed  in  Reserve  account  to  cover  repairs  and  renewals,  the  entire  7V2 
per  cent  being  charged  direct  to  Profit  and  Loss.  The  bond  outstanding 
fell  due  on  December  31,  1907,  and  was  paid,  principal  and  interest,  in 
cash. 

An  inventory  of  materials  and  supplies  placed  their  value  at  $2,328.19, 
the  practice  being  to  charge  all  purchases  direct  to  Manufacturing  Ex- 
penses and  to  credit  back  the  amount  of  the  inventory. 

The  accounts  payable  (all  for  material  and  non-interest  bearing)  amount 
to  $546.28. 

Of  the  accounts  receivable  January  1,  1907,  $4,968.18  was  collected  and 
the  balance  charged  off  as  uncollectible. 

In  addition  to  the  material  used  from  stock  during  the  year  and  the 
amount  still  due  for  material  purchased,  the  manufacturing  expenses  were 
$3,720.52,  all  paid  in  cash,  the  total  manufacturing  expenses  being  31 
per  cent  of  the  gross  sales  for  the  year  ending  January  1,  1908. 

Of  these  91.3  per  cent  were  collected  in  cash  and  the  balance,  all  of 
which  is  considered  good,  remains  on  the  books  in  Accounts  Receivable. 

Produce  a  comparative  Balance  Sheet  of  January  1,  1908-1907,  and 
state  the  amount  of  gross  sales  for  the  year. 

(N.  Y.,  Feb.,  1908.)      . 


CHAPTER  IV 
PROFIT  AND  LOSS  STATEMENT 

Income — Income  is  the  gain  or  remuneration  which  proceeds  from 
property,  labor,  or  business/  It  may  be  classified  into  (a)  rent  or  income 
from  real  estate,  (b)  wages  or  remuneration  of  labor,  (c)  interest  or 
income  from  liquid  capital,  and  (d)  profits  or  gain  from  investment  in  a 
business  organization/  Income  may  be  also  classified  as  primary  or 
operating  revenue  and  secondary  or  non-operating  revenue/ 

Revenue — The  term  "revenue"  is  frequently  used  interchangeably  with 
the  term  "income,"^  the  terms  "gross  revenue"  and  "net  revenue"  being 
used  synonymously  with  "gross  income"  and  "net  income,"  respectively.' 
There  is  a  slight  distinction,  however,  as  the  term  "i-evenue"  is  used 
especially  by  non-trading  concerns/ 

Earnings — Earnings  are  that  part  of  income  derived  as  remuneration 
for  personal  services,  the  terra  being  used  in  the  statements  of  professional 
men  and  public  service  coi'porations/ 

Profits — Pi'ofit  to  the  economist  means  onlj!-  the  reward  to  the  ability 
and  enterprise  of  the  successful  entrepreneur,*  while  to  an  accountant  it 
means  the  realized  increment  in  value  of  the  whole  amount  invested  in  a 
business/  If  a  sole  proprietor  made  a  net  income,  the  accountant  would 
call  all  of  the  net  income  profit,^"  while  the  economist  would  classify  it 
as  wages,  rent,  intei-est,  and  pi'ofits/' 

Gross  profits  represent  the  excess  of  the  selling  price  over  the  actual 
cost  of  the  article  sold/''  Net  profits  represent  the  excess  of  earnings  over 
all  costs,  expenses,  and  reserves  for  accrued  or  probable  losses/' 

A  broad  distinction  must  be  made  between  capital  profits  and  operating 
profits.  A  capital  profit  or  loss  is  one  that  arises  from  a  change  in  the 
value  of  fixed  assets,  and  should  be  shown  on  the  books  only  if  the  fixed 
assets  are  sold  or  destroyed."  An  operating  profit  or  loss  is  one  that 
ai'iscs  fi'om  the  ordinary  operation  of  the  business."  Capital  profits  and 
losses  should  not  be  included  in  the  operating  profit  and  loss  statement, 
but  should  be  deferred  until  the  operating  profit  or  loss  is  ascertained  and 
then  charged  or  credited  directly  to  surplus." 

Generally,  profits  and  losses  are  made  only  on  completed  transactions,'' 
accruals  being  a  recognized  exception  to  this  rule.  This  will  prevent  appre- 
ciation on  real  estate  from  being  treated  as  a  profit  until  the  real  estate  is 
sold.'"  Profit  may  not  be  taken  on  goods  made  for  stock,  for  the  profit 
occui-s  Avhcn  the  sale  is  made.'*  No  profit  may  be  taken  unless  a  cause 
of  legal  action  has  arisen.'" 


'For  cxplniiiitioii  of  superior  figure's  see  pnge  .337. 

67 


68  C.  P.  A.  ACCOUNTING 

The  estimated  profit  on  goods  made  to  order  may  be  taken  in  the  pro- 
portion the  share  of  work  done  during  the  period  is  to  the  total  under- 
taking.^' This  necessitates  a  predetermination  of  profit  on  the  whole 
undertaking,  an  estimate  of  the  portion  of  the  contract  completed,  and 
an  adequate  provision  for  unforeseen  difficulties  in  completing  the  work." 

Equitable  treatment  of  stockholders  requires  that  the  profit  on  long 
term  contracts  be  taken  as  the  work  is  completed."  If  liljeral  allowances 
for  contingencies  are  made,  the  estimated  profit  may  be  prorated  to  periods 
on  the  basis  of  the  estimates  of  the  portion  of  work  completed  made  by 
the  supervising  engineer  to  secure  the  periodic  payments  on  the  contract.^* 
If  the  contract  price  is  based  upon  a  series  of  units  in  the  contract,  profit 
should  be  taken  on  the  number  of  units  completed."  Cost-plus  contracts 
may  have  the  accrued  profits  taken  without  consideration  of  the  unfinished 
portion,  as  the  profits  are  always  a  definite  percentage  of  the  cost.''" 

As  commercial  practice  allows  buyers  to  cancel  orders  for  standard 
goods  up  to  the  date  of  a  shipment,  profits  on  luidelivered  sales  should 
not  be  taken.^' 

The  excess  of  market  price  over  production  cost  of  goods  sold  by  one 
subsidiary  companj'  to  another  or  transferred  from  one  department  to 
another  is  not  a  profit,  as  no  profit  can  be  made  unless  the  goods  are 
sold  to  an  outside  party .^ 

Revenue  and  Capital  Receipts — Revenue  receipts  are  the  receipts  of 
cash  and  other  assets  on  account  of  the  regular  operation  of  the  business, 
and  include  receipts  from  sales,  interest  on  investments,  etc."  Capital 
receipts  consist  of  the  cash  and  other  assets  received  through  the  sales  of 
capital  stock,  the  issue  of  capital  liabilities,™  or  the  sale  of  a  fixed  asset." 

Revenue  and  Capital  Expenses — Revenue  or  operating  expenses  are  in- 
curred to  conduct  the  business  in  an  endeavor  to  make  a  profit.''  Capital 
or  non-operating  expenses  are  the  financial  management  expenses  incurred 
in  providing  the  capital  needs  of  a  business.^' 

Revenue  and  Capital  Expenditures — The  term  "revenue  expenditures" 
is  synonymous  with  "revenue  expenses.'"*  Revenue  expenditures  appear  as 
assets  on  the  balance  sheet  only  as  defen-ed  charges  to  income.^"  Capital 
expenditures  are  the  expenditures  made  for  additions  or  improvments  to 
the  more  or  less  permanent  plant.^  The  accounting  for  capital  expendi- 
tures is  outlined  in  Chapter  IV,  Volume  II. 

Fixed  Charges  and  Operating  Expenses — Fixed  charges  are  the  expenses 
of  more  or  less  fixed  amount,  which  must  be  met  periodically  without  ref- 
erence to  the  volume  of  business.''  Interest  on  bonds  is  a  fixed  charge," 
but  interest  on  notes  payable  is  not."  Operating  expenses  are  those  in- 
curred directly  in  connection  with  operations  and  which  roughly  fluctuate 
with  the  business  done.^ 

Summaries  of  Nominal  Accounts — The  business  executive's  need  for 
statistics  has  caused  many  varied  groupings  of  the  nominal  accounts,  viz., 
the  manufacturing,  trading,  administrative,  and  profit  and  loss  accounts. 
These  summai-y  accounts  have  largely  been  replaced  by  reports  or  state- 


PROFIT  AND  LOSS   STATEMENT 


69 


ments  containing  the  same  data  in  more  available  form.  Examples  of  such 
statements  ai'e  the  statements  of  cost  of  manufacture,  cost  of  sales,  and 
overhead.  The  need  for  all  these  data  in  one  statement  has  led  to  the 
modern  profit  and  loss  statement.  Synonyms  for  the  profit  and  loss  state- 
ment are  statements  of  income  and  expenditures,  revenues  and  expendi- 
tures, trading  and  profit  and  loss,  income  and  profit  and  loss,  etc.  "Revenue 
and  expenditui'es"  is  usually  limited  to  profit  and  loss  statements  rendered 
by  non-profit-making  concerns,  "income  and  expenses"  to  institutions,  and 
"income  statements"  to  large  industrial  concerns.** 

Account  Form  of  Profit  and  Loss  Statement — The  origin  of  the  profit 
and  loss  statement  is  shown  in  its  account  form,  for  each  one  of  the  original 
summary  accounts  has  a  section  in  the  account  form  of  profit  and  loss 
account.** 

A.  B.  COMPANY 
PROFIT  AND  LOSS  STATEMENT 
For  Period  Ending  (Date) 


Manufacturing  Section 

Raw  Materials. .  .  . 
Productive  Labor .  . 
Factory  Overhead: 

Indirect  Labor.  . 

Superintendence. 

Supplies 

....s 

....$ 

Manufacturing  Cost,  carried 

to  Trading  Section $ 

Power 

% 

...  $ 

% ' 

$ 

Trading  Section 


Manufacturing  Cost,  brought 
down  from  Manufacturing 
Section % 

Finished  Goods  (initial  inven- 
tory)   % 


Less  Finished  Goods  (final  in- 
ventory)   $ . 


Cost  of  Sales 

Selling  Expense: 
Salesmen's   Sal- 
aries   

Traveling   Ex- 
penses   

Advertising 


Gross  Profit  on  Trading  car- 
ried to  Administration  Sec- 
tion  '. % 


Sales $ . 

Less  Returned  Sales $ . 

Net  Sales $. 


70 


C.  P.  A.  ACCOUNTING 
Administration  Section 


Office  Expense $ . 

Directors'  Fees $ . 

Legal  Expense .  .  $ . 

Net  Profit  on  Trading  carried 
to  Profit  and  Loss  Section. .  $ . 


Gross  Profit  on  Trading 
brought  down  from  Trad- 
ing Section 


Profit  and  Loss  Section 


Bad  Debts $ . 

Interest  Expense $ . 

Net  Profit  for  period  carried 
to  Appropriation  Section .  .  $ . 


Net  Profit  on  Trading  brought 
down  from  Administration 
Section 

Interest  Earned 


Appropriation  Section 


Dividends  Declared. 
Addition  to  Surplus. 


Net  Profit  for  Period  brought 
from  Profit  and  Loss  Sec- 
tion   


Report  Form  of  Profit  and  Loss — As  the  account  or  non-technical  form 
of  profit  and  lo^s  statement  is  not  adaptable  to  the  use  of  the  typewriter 
and  is  rather  too  complicated  for  the  lay  reader,  most  accountants  use 
a  report  form  of  profit  and  loss  statement/'  although  a  few  well  known 
authorities  still  advocate  the  account  form." 

As  is  the  case  with  the  balance  sheet,  thei-e  are  numerous  forms  of 
I)rofit  and  loss  statements.  The  chief  advantages  of  the  following  model 
form  are,  (a)  the  fact  that  only  three  money  columns  are  used,  thus 
increasing  the  speed  and  accuracy  of  the  student,  (b)  the  subtraction  from 
the  gross  profit  on  sales  of  the  total  of  the  selling,  administrative,  and 
general  expenses,"  thereby  eliminating  the  unimportant  selling  profit  sub- 
total, (c)  the  subtraction  from  or  addition  to  tlie  net  profit  on  operations 
of  the  net  figure  for  the  non-operating  expenses  and  incomes,**  thus 
eliminating  the  insignificant  total  income  figure,  and  (d)  the  introduction 
of  an  appropriation  of  profit  section,"  which  is  valuable  in  that  it  shows 
a  summary  of  the  effect  the  profits  had  on  the  vested  proprietorship 
accounts. 


PROFIT  AND  LOSS   STATEMENT 


71 


A.  B.  COMPANY 
PROFIT  AND  LOSS  STATEMENT 
For  Period  Ending  December  31,  1916 


Gross  Sales 

S 

S 

s 

$ 

S 

Less  Returns  and  Allowances  on  Sales. .  .  . 

Net  Sales 

$ 

$ 

s 

$ 

$ 

$ 

Cost  of  Sales: 
Material : 

Inventory,  Jan.  1,  1916      

Freight  and  Cartage  in 

Purchases 

Deduct  Inventory,  Dec.  31,  1916 

Labor 

$ 

$ 

$ 

$ 

$ 

$ 

$ 

$ 

$ 

$ 

Manufacturing  Expense: 

Depreciation,  Buildings — 1916 

Depreciation,  Machinery — 1916 

Factory  Expense 

Fuel 

Insurance,  Buildings  and  Machinery .  .  . 

Repairs,  Building 

Repairs,  Machinery 

Taxes,  Real  Estate 

Inventory  Variation — Work  in  Process: 

Jan.  1,  1916 

Dec.  31,  1916 

« 

$ 

Inventory  Variation — Finished  Goods: 

Jan.  1,  1916 

Dec.  31,  1916 

$ 

$ 

$ 

Gross  Profit  on  Sales      

$ 

$ ■. 

$ 

$ 

$ 

Operating  Expenses: 
SeUing  Expense: 

Advertising 

Commissions  Paid  Salesmen 

Insurance,  Finished  Goods 

Salaries,  Salesmen 

General  Administrative  Expense : 

Office  Expense 

Office  Payroll 

$ 

$ 

$ 

Salaries,  General  Officers 

$ 

Net  Profit  on  Operations 

$ 

$ 

$ 

$ 

$ 

Non-Operating  Items: 
Income: 

Discount  on  Purchases   

Interest  Received 

Expense: 

Discount  on  Sales 

$ 

$ 

$ 

Interest  Paid 

Doubtful  Accounts  Receivable 

$ 

Net  Income  for  Period 

$ 

$ 

$ 

$ 

$ 

Appropriation : 

Organization  Expense  Written  Off 

Subscriptions  and  Donations 

Income  Taxes     

Surplus  Additional 

$ 

72  C.  P.  A.  ACCOUNTIKG 

Statements  of  Non-Trading  Concerns — The  profit  and  loss  statements  of 
non-trading  concerns  such  as  railroads,  telephones,  banks,  and  hospitals 
have  the  same  general  divisions  as  the  statements  of  trading  companies.'" 
Fii-st,  all  the  operating  incomes  are  added,  from  which  sum  is  subtracted 
the  total  of  the  operating  expenses.  To  the  resulting  net  profit  on  opera- 
tions is  added  the  non-operating  income.  Then  the  non-operating  ex- 
penses are  deducted,  leaving  the  net  profit  for  the  period.* 

Comparative  Profit  and  Loss  Statements — The  Federal  Reserve  Board 
has  developed  the  form  of  comparative  profit  and  loss  statement  given 
below."  The  form  is  compact,  yet  clear,  and  may  be  followed  to  advan- 
tage. Of  course,  some  variation  in  the  form  may  be  made.  For  instance, 
outward  freight  may  be  treated  as  a  selling  expense  if  the  accountant  so 
desires.  The  increases  or  decreases  between  periods  or  percentage  figures 
may  be  shown  in  parallel  columns  to  the  right  of  the  annual  figures. 

Use  of  Schedules  in  Profit  and  Loss  Statements — To  eliminate  detail  on 
the  profit  and  loss  statement,  schedules  may  be  appended  showing  such 
items  as  cost  of  goods  sold,  the  gi'oup  of  selling  expenses,  the  group  of 
general  and  administrative  expenses,  etc."  A  profit  and  loss  statement 
where  the  detail  is  relegated  to  schedules  is  called  a  condensed  profit  and 
loss  statement." 

Working  Sheet — As  the  working  sheet  is  not  an  aim  in  itself  but 
mei'ely  a  means  of  increasing  speed  and  accuracy  in  the  jireparation  of 
financial  statements,  the  twelve  column  working  sheets  should  not  be  used. 
Two  columns  each  for  the  original  trial  balance,  adjustments,  balance 
sheet,  and  profit  and  loss  are  adequate." 

When  the  trial  balance  does  not  contain  all  the  accrued  and  defeiTed 
accounts,  these  accounts  may  be  opened,'**  or  the  adjustment  may  be  kept 
within  the  original  nominal  account  by  showing  the  debit  and  credit  entries 
in  the  adjustment  columns  under  the  original  account  with  the  letter  "B" 
after  the  item  affecting  the  balance  sheet."  For  instance,  the  new  mer- 
chandise inventory  would  appear  in  the  adjustment  columns  as  debit 
"15,000  B"  and  credit  "$15,000,"  the  debit  item  later  being  extended  to 
debit  balance  sheet  column  and  the  credit  item  to  the  credit  profit  and  loss 
column.  This  second  method  is  preferable,  as  it  is  quicker  and  leaves  the 
working  sheet  in  a  more  compact  form. 

There  are  tAvo  ways  of  treating  inventories  in  the  profit  and  loss  columns 
of  the  working  sheet,  namely,  (a)  extending  into  the  profit  and  loss  col- 
umns only  the  difference  between  the  old  and  the  new  inventories,"^  and 
(b)  extending  the  old  inventory  as  a  debit  and  the  new  inventory  as  a 
credit."  The  latter  method  is  the  better,  as  both  the  old  and  new  inventories 
are  used  in  the  profit  and  loss  statement  while  the  difference  between  the 
two  inventories  is  not  usually  entered. 

By  arranging  all  the  asset  and  expense  accounts  on  one  page  and  the 
liability,  capital,  and  income  accounts  on  the  other,  the  number  of  money 
columns  can  be  reduced  to  one  for  the  trial  balance,  two  for  adjustments, 
and  one  each  for  the  balance  sheet  and  profit  and  loss  statement.** 


PROFIT  AND  LOSS   STATEMENT 
COMPARATIVE  STATEMENT  OF  PROFIT  AND  LOSS 


73 


Year 

Ended 

19— 

Year 

Ended 

19— 

Year 

Ended 

19— 

Gross  Sales 

Less    Outward    Freight,  Allowances,    and 
Returns 

$ 

$ 

$ 

$ 

$ 

$ 

Net  Sales   .        .                            ... 

$ 

S 

$ 

Inventory  beginning  of  year 

$ 

$ 

$ 

$ 

$ 

$ 

Purchases,  Net 

Less  Inventory  end  of  year    . 

$ 

$ 

$ 

S 

$ 

$ 

Cost  of  Sales 

$ 

$ 

$ 

Gross  Profit  on  Sales 

$ 

$ 

$ 

Selling    Expenses    (itemized    to    correspond 
with  ledger  accounts  kept) 

$ 

$ 

$ 

Total  Selling  Expense 

S 

$ 

$ 

General  Expenses   (itemized  to  correspond 
with  ledger  accounts  kept) 

$ 

$ 

$ 

Total  General  Expenses 

$ 

f 

$ 

Administrative  Expenses  (itemized  to  corre- 
spond with  ledger  accounts  kept) 

$ 

$ 

$ 

Total  Administrative  Expenses 

$ 

$ 

$ 

Net  Profit  on  Sales 

$ 

$ 

•f 

Other  Income: 

Income  from  Investments 

$ 

$ 

$ 

$ 

$ 

.•1 

Interest  on  Notes  Receivable,  etc 

$ 

.$ 

$ 

Deductions  from  Income: 

Interest  on  Bonded  Debt 

$ 

$ 

$ 

$ 

$ 

$ 

Interest  on  Notes  Payable. .  .  .  , 

Total  Deductions 

$ 

$ 

$ 

Net  Income — Profit  and  Loss 

$ 

$ 

$ 

$ 

$ 

$ 

S 

« 

$ 

Add  special  credits  to  Profit  and  Loss 

Deduct  special  charges  to  Profit  and  Loss. . . 

Profit  and  Loss  for  Period 

$ 

$ 

f 

$ 

$ 

Surplus  beginning  of  period 

Dividends  Paid 

$ 

$ 

$ 

$ 

$ 

$ 

Surplus  ending  of  period 

$ 

$ •• . 

$ 

74  C.   F.  A.  ACCOL'NTINMJ 


QUESTIONS 

profit  axd  loss  statement 

Definitions 

1.  Define:  Trading-  aceonnt.  (N.  Y.,  Dee.,  1896;  X.  Y.,  June.  1890*: 
Pa.,  Nov..  1901;  Pa.,  May.  1902*;  Cal.,  June,  1904*;  X.  Y..  June,  1904*; 
N.  J.,  1904-1909*;  N.  Y..  Oct.,  1907*;  R.  I..  Dec,  1907*;  Cal..  May, 
1908;  Wash.,  Aug.,  1908*;  Va.,  Nov.,  1910;  Wash.,  June,  1912;  Va.. 
Oct.,  1912*;  Wash.,  Nov..  1913;  Colo.,  Dec,  1913;  W.  Va.,  May,  1917; 
N.  Y.,  Aug.,  1917;  S.  C,  Sept.,  1919.*) 

2.  Define:  Profit  and  Loss  account.  (N.  Y..  Dec.  1896;  N.  Y.,  Dec, 
1897*;  Pa.,  Nov.,  1899*;  Pa..  May,  1902*;  Pa.,  May.  1903*:  Md..  Oct.. 
1903;  Cal.,  June.  1904;  N.  Y.,  June,  1904*;  Va.,  *Nov..  1910;  Wash., 
Nov.,  1913*;  Colo.,  Dec,  1913;  W.  Va.,  May,  1917;  N.  C,  Aug.,  1917; 
S.  C,  Sept.,  1919.*) 

3.  Define:  Loss  and  Gain  account.  (N.  J.,  1904-1909*;  Iowa,  Dec, 
1918.) 

4.  Define:  Revenue  account.  (N.  Y.,  Dec,  1896;  Pa.,  Nov.,  1901*; 
Md.,  Oct.,  1903;  Cal.,  June,  1904*;  N.  J.,  1904-1909;  Cal.,  May,  1908; 
Wash.,  June,  1912;  W.  Va.,  May,  1917;  S.  C,  Sept.,  1919.*) 

5.  Define:  Income  account.     (111.,  Nov.,  1903.) 

6.  Define:  Net  income.     (Mich.,  June,  1910.) 

7.  Define:  Income.  (N.  Y.,  Dec,  1897;  Wash..  June,  1915*;  Cal.,  Nov., 
1916*;  N.  C,  Aug.,  1917;  Iowa,  Dec,  1918.) 

8.  Define:  Revenue.  (N.  Y.,  Dec,  1896;  Pa.,  Nov.,  1899;  Pa.,  Mav, 
1903*;  N.  Y.,  Jan.,  1904;  Va.,  Oct.,  1911*;  Mich.,  Dec,  1915;  Iowa,  Dec, 
1918.») 

9.  Define:  Expense.  (N.  Y.,  Dec,  1896*;  Pa..  Mav,  1903*;  N.  Y., 
Jan.,  1904;  Ohio,  March,  1910*;  W.  Va.,  May,  1917*;  n!  C,  Aug.,  1917.) 

10.  Define:  Cost  of  goods  sold.  (Wash.,  Nov.,  1913*;  N.  C,  Nov., 
1918.*) 

11.  Define:  Operating  expenses.  (N.  Y.,  Dec,  1896;  N.  J.,  1904- 
1909;  Wis.,  April,  1914;  Wash.,  June  1917.) 

12.  Define:  Commercial  expenses.     (Mich.,  June,  1908.) 

13.  Define:  Fixed  charges.  (N.  Y.,  Dec,  1896;  N.  Y.,  Dec,  1897; 
Wash.,  May,  1903*;  N.  J.,  1904-1909;  Mich.,  Dec,  1906;  Md..  Jan.,  1909; 
Mass.,  June,  1910;  Wis.,  April,  1914;  Mich.,  June,  1914*;  111.,  Dec,  1916; 
Wash.,  July,  1917.) 


PROFIT  AND  LOSS  STATEMENT  75 

14.  What  expenses  would  you  classify  as  fixed  or  overhead  expenses? 
(Va.,  Od.,  1911.) 

15.  Define:  Income  and  expenditures.  (Pa.,  May,  1902*;  Wash.,  June, 
1912;  N.  D.,  Aug.,  1917.) 

16.  Define:  Statement  of  income  and  expenditure.  (N.  Y.,  Dec,  1896*; 
Wis.,  April,  1914;  Del.,  elune.  1915*;  Wis.,  May,  1916;  Mich.,  Dec.,  1916; 
111.,  May,  1917;  W.  Va.,  May,  1917.*) 

17.  Define:  Statement  of  Revenue  and  Expenditures.  (Kan.,  May, 
1916.) 

18.  Define:  Statement  of  Revenue  and  Expenses,  and  also  describe 
its  function.     (Ohio,  Dee.,  1908*;  N.  C,  Nov.,  1918.) 

19.  Define:  Statement  of  Overhead  or  Administrative  Expense;  also 
describe  its  function.     (N.  C,  Nov.,  1918.) 

20.  What  is  a  Trading  statement?     (Iowa,  Dec,  1918.) 

21.  Define:  Cost  statement.     (N.  C,  June,  1920.) 

22.  What  is  a  Manufacturing  igtatement?  (N.  C,  Nov.,  1918*;  Iowa, 
Dec,  1918.) 

23.  What  is  a  Profit  and  Loss  statement?  (Iowa,  Dec,  1917;  N.  C, 
Nov.,  1918*;  N.  C,  June,  1920.*) 

24.  What  are  revenue  receipts?  (N.  Y.,  Dec,  1896;  N.  Y.,  Oct.,  1907; 
Mich.,  June,  1910;  Mich.,  June,  1912;  Wis.,  April,  1914;  Wis.,  April, 
1917.) 

25.  Define:  Capital  receipts.  (N.  Y.,  Dec,  1896;  N.  Y.,  Oct.,  1907; 
Mich.,  June,  1910;  Va.,  Oct.,  1911*;  Wis.,  April,  1914;  Wis.,  April, 
1917.*) 

26.  Define:  Capital  revenue.     (Cal.,  Nov.,  1916.) 

27.  Define:  Statement  of  the  Cost  of  Sales;  also  describe  its  function. 
(N.  C,  Nov.,  1918.) 

28.  Define:  Statement  of  Capital  Receipts;  also  describe  its  function. 
(N.  C,  Nov.,  1918.) 

29.  Illustrate  accounts  that  are  considered  as  deductions  from  income. 
(Mich.,  June,  1919.) 

30.  Define:  Statement  of  Capital  Expenses;  also  describe  its  func- 
tion.    (N.  C,  Nov.,  1918.) 

31.  Define  and  distinguish  between  capital  profit  and  capital  loss. 
(Wis.,  April,  1914.) 

Operating   Statements 

32.  Name  and  define  five  commonly  used  subdivisions  of  revenue  ex- 
penditure accounts.  (N.  Y.,  June,  1909* ;  Mich.,  June,  1912;  Wash.,  June, 
1912*;  Cal.,  Nov.,  1916*;  N.  Y.,  June,  1909.*) 

33.  Submit  a  pro  forma  Balance  Sheet  and  Profit  and  Loss  account, 
without  figures,  for  a  manufacturing  company.     (Wash.,  Aug.,  1908*;  Pa., 


70  C.  P.  A.  ACCOUNTING 

Nov.,  1908*;  Wash..  May,  1911*;  Cal.,  May,  1916;  Kan.,  May,  1916*; 
Ind.,  June,  1916*;  Mass.,  Oct.,  1917.*) 

34.  How  do  the  accounts  of  a  corporation  and  of  a  copartnership 
differ  in  the  statement  of  Operation  of  Business  and  Determination  of 
Profits?     (N.  Y.,  Jan.,  1904.) 

35.  A  manufactunn«r  corporation  havinsr  several  plants  decides  to  shut 
down  one  plant  because  it  cannot  be  run  economically.  Under  what 
classification  in  the  Operatinir  statement  would  you  include  the  expenses 
attending  the  care  and  upkeep  of  the  idle  plant?     (N.  Y.,  June,  1919.) 

36.  The  books  of  a  corporation  show  balances  at  the  debit  or  credit 
of  the  following:  accounts:  Rents  from  Tenements,  Reserve  for  Accounts 
Receivable,  Depreciation  on  Machinery,  Depreciation  on  Furniture  and 
Fixtures,  Bond  Redemption  account,  Bills  Receivable,  Dividend  on  Pre- 
ferred Stock.  State  which  should  enter  into  Profit  and  Loss  account  and 
which  should  appear  in  the  Balance  Sheet,  and  why.     (N.  Y.,  June,  1911.) 

37.  The  following  accounts  are  found  in  the  books  of  a  corporation. 
State  which  of  them  would  enter  the  Profit  and  Loss  account  and  Balance 
Sheet,  and  which  would  show  debit  and  credit  balances;  Reserve  Fund, 
Depreciation  on  Furniture,  Bad  Debt  Reserve,  Bond  Redemption  account. 
Bills  Receivable,  Rent  on  Properties  Owned,  Dividend  on  Preferred  Stock. 
(N.  Y.,  Oct.,  1907;  R.  I.,  Dec,  1907.) 

38.  Name  the  principal  elements  of  a  statement  of  the  Cost  of  Sales 
of  a  manufacturing  business  and  prepare  a  fonn  which  vou  would  use. 
(N.  Y.,  Nov.,  1919.) 

39.  Prepare  a  skeleton  foi"m  of  a  Manufacturing  statement  showing 
classification  of  accounts.     (Ind.,  June.,  1916*;  Iowa,  Dec,  1918.) 

40.  Explain  the  functions  of  a  Manufacturing  account.  (N.  C,  Aug., 
1917.) 

41.  Prepare  a  skeleton  form  of  a  Tr^'ding  statement  showing  classifica- 
tion of  accounts.     (Mich.,  Dec,  1913*;  Iowa,  Dec,  1918.) 

42.  What  is  the  purpose  of  a  Trading  account,  and  what  general 
result  should  it  show?  In  closing  the  Ledger  what  disposition  should  be 
made  of  the  balance  of  the  Trading  account?     (N.  Y.,  Jan.,  1901.) 

43.  In  making  up  a  Profit  and  Loss  statement  at  the  end  of  a  fiscal 
year,  are  you  stating  a  fact  or  an  opinion?  Give  reasons.  (111.,  Nov., 
1908;  Colo.,  Dec,  1913;  Del.,  June,  1915.) 

44.  State  what  is  indicated  by  the  Loss  and  Gain  account  (a)  when 
the  account  shows  a  debit  balance;  (b)  when  the  account  shows  a  credit 
balance.  Explain  fully.  (N.  Y.,  Dec,  1897*;  N.  Y.,  Jan.,  1900;  N.  Y., 
June,  1911.*) 

45.  What  accounts  on  the  Ledger  are  generally  considered  as  "de- 
duction from  income"?  Why?  As  "other  income"?  (Md.,  Dec,  1917; 
N   Y.,  June,  1919.) 

46.  From  what  accounts  is  a  Profit  and  Loss  account  prepared?  (N.  Y., 
June,  1901;  N.  C,  June,  1920.*) 


PROFIT  AND  LOSS  STATEMENT  77 

47.  How  is  the  pi'ofit  or  loss  anivefl  at  from  a  double-entry  set  of 
books?  (N.  Y.,  Oct.,  1907*;  R.  I.,  Dec,  1007*;  Cal.,  May,  1008*;  N.  Y., 
June,  1909*;  N.  Y.,  Feb.,  1010*;  Va.,  Nov.,  1010;  \Ya.sh.,  June,  1912*; 
Va.,  Nov.,  1018.*) 

48.  Describe  tbe  different  methods  of  determining  the  loss  or  gain  of 
a  business.     (N.  Y.,  Dec,  1898.) 

49.  In  a  general  way  what  distinguishes  manufactm*ing  expenses  from 
commercial  expenses'?     (Mich.,  June,  1910.) 

50.  May  interest  on  floating  debt  be  properly  considered  a  fixed 
charge?  (N.  Y.,  Dec,  1897;  Wash.,  May,  1903;  Md.,  Jan.,  1909;  N.  Y., 
June,  1909;  Mass.,  June,  1910.) 

51.  State  explicitly  and  fully  the  function  of  the  Profit  and  Loss 
account.     (N.  Y.,  June,  1899.) 

52.  Show  the  relationship  between  trial  balance,  Trading  statement. 
Profit  and  Loss  account,  assets  and  liabilities.  (Fla.,  July,  1908;  Cal., 
Nov.,  1916.*) 

53.  Differentiate  as  fully  as  possible  a  Manufacturing  account,  a 
Trading  account,  and  a  Profit  and  Loss  account,  and  state  what  the 
balance  in  each  indicates.  (N.  Y.,  June,  1898;  N.  Y.,  June,  1899;  N.  Y., 
June,  1906*;  Md.,  Jan.,  1909*;  N.  Y.,  Feb.,  1910*;  111.,  May,  1913*; 
Mich.,  June,  1915;  N.  D.,  Aug.,  1917*;  N.  D.,  July,  1918*;  Md.,  Oct., 
1919*;  N.  Y.,  Jan.,  1920.*) 

54.  Give  your  mode  of  procedure  in  ascertaining  the  correctness  of  the 
Profit  and  Loss  account.  (N.  Y.,  June,  1898*;  N.  Y.,  June,  1901*;  N.  Y., 
Jan.,  1902;  Pa.,  May,  1903;  Md.,  Oct.,  1903*;  Pa.,  Nov.,  1904;  Mich., 
June,  1913.*) 

55.  What  particular  accounts  in  a  mercantile  business  would  you  in- 
vestigate to  see  that  all  entries  had  been  made  before  ascertaining  the 
correct  profit  or  loss  of  the  business?  Answer  very  fully.  (Pa.,  Nov., 
1901.) 

56.  A  manufacturing  concern  on  closing  the  books  at  the  end  of  the 
fiscal  year  finds  that  they  show  a  loss,  whereas  there  is  good  ground  for 
the  belief  that  a  profit  was  earned.  With  a  view  to  settling  the  question, 
what  particular,  etc.,  would  you  examine,  and  how?     (Mass.,  Oct.,  1915.) 

57.  In  a  test  audit  what  verification  should  be  made  of  the  accuracy 
of  the  Profit  and  Loss  account?  Assume  that  there  is  no  reason  to 
suspect  any  intent  to  misstate  the  profits. 

Would  it  make  any  difference  if  you  were  employed  by  the  prospec- 
tive purchaser  of  the  business  and  he  suspected  that  the  profits  had  been 
overstated  so  as  to  increase  the  selling  price  of  the  business? 

(111.,  May,  1917.) 

58.  In  preparing  a  Profit  and  Loss  account  and  Balance  Sheet,  how 
are  gains,  losses,  assets,  liabilities,  capital,  drawings,  and  expenditures 
ascertained  and  dealt  with?     (111.,  Nov.,  1908.) 


78  C.  P.  A.  ACCOUNTING 

59.  Give  proper  disposition  of  any  bal  moe  appearing  in  a  Profit  and 
Loss  account  at  the  end  of  the  fiscal  year.  (111.,  May,  1907;  111.,  Maj-, 
1909.*) 

60.  (a)  What  is  the  purpose  of  a  Profit  and  Loss  account  and  how  is  it 
made  up? 

(&)   What  does  the  balance  of  a  Profit  and  Loss  account  represent? 

(111.,  May,  1909.) 

61.  An  attorney-at-law  joined  an  established  firm  in  partnership  with- 
out having  the  books  investigated,  relying  on  the  firm's  .statement  that 
they  were  making  certain  profit.  After  several  months  have  elapsed, 
the  new  partner,  not  being  satisfied,  instructs  you  to  audit  the  accounts 
for  the  year  prior  to  his  joining  the  firm.  You  discover  that  no  break 
has  been  made  in  the  books  for  years.  What  steps  would  you  take  to 
ascertain  the  exact  profit  for  the  year?     (111.,  May,  1910.) 

62.  Give  examples  of  such  assets  and  liabilities  not  usually  found  on 
books  of  accounts,  as  should  be  considered  by  an  auditor  when  preparing 
an  Income  and  Profit  and  Loss  account  at  the  close  of  a  fiscal  year. 
(N.  D.,  July,  1916.) 

Profits 

63.  Define:  Net  profit.  (N.  Y.,  June,  1900;  Pa.,  Nov.,  1900;  N.  Y., 
Jan.,  1901*;  Wash.,  Aug.,  1908;  Mich.,  June,  1910;  Va.,  Nov.,  1910; 
Colo.,  Dec.,  1913;  Kan.,  May,  1916;  N.  D.,  June,  1917.) 

64.  Define:  Gross  profit.  (Pa.,  Nov.,  1900;  Wash.,  Aug.,  1908*; 
Mich.,  June,  1910;  Va.,  Nov.,  1910;  Colo.,  Dec,  1913;  Wash.,  June,  1915.*) 

65.  Define:  Profits.     (Wash.,  Sept.,  1907.) 

66.  Define:  Trading  profit.     (Cal.,  Nov.,  1916.) 

67.  Define:  Manufacturing  profit.     (N.  Y.,  Jan.,  1919.) 

68.  Explain  the  significance  of  the  term  "profits"  as  used:  (1)  in 
ordinary  parlance;  (2)  in  commercial  parlance;  (3)  in  law;  (4)  in 
economies.     (Mo.,  Dec.,  1913*;  Mo.,  Dec,  1914.) 

69.  Legally,  income  is  cash  received  in  excess  of  expenses.  What  is 
the  theory  of  income  of  a  partnership  as  expressed  by  their  Profit  and 
Loss  account,  where  values  are  not  actually  reduced  to  a  cash  basis? 
(N.  Y.,  June,  1917.) 

70.  How  may  the  gross  profit  or  loss  on  merchandise  be  ascertained? 
(N.  Y.,  Dec,  1897.) 

71.  State  your  understanding  of  the  difference  between  gross  profit 
and  net  profit.     (111.,  May,  1908;  Mass.,  June,  1910.) 

72.  What,  in  your  opinion,  is  the  difference,  if  any,  between  net  in- 
come and  net  profit?    Answer  fully.     (Kan.,  Dec,  1915;  Mo.,  Dec,  1915.) 

73.  When  is  a  profit  made?     (Iowa,  Dec,  1918.) 

74.  State  the  general  rule  as  to  anticipation  of  profits.  What  excep- 
tion, if  any,  is  there  to  this  rule?     (Ohio,  Oct.,  1919.) 


PROFIT  AND  LOSS  STATEMENT  79 

75.  May  any  fliictiiation  in  the  value  of  permanent  assets  be  permitted 
to  affect  the  result  of  the  Profit  and  Loss  account?  Give  reasons.  (Fla., 
April,  1907*;  N.  Y.,  Jan.,  1918.) 

76.  What  methods  have  your  audits  revealed  by  which  uianufacturiui;' 
concerns  attempted  to  inflate  profits?  If  you  have  not  met  any,  ex2)Iain 
how  it  could  be  done.     (Mich.,  Dec,  1913.) 

77.  When  the  cost  of  making  a  product  is  known,  also  the  overhead  and 
other  expenses,  and  the  amount  of  the  sales,  what  other  information 
would  you  require  to  determine  the  profits  or  losses  in  the  sales  of  the 
product?     (N.  C,  June,  1916*;  N.  C,  Sept.,  1919.) 

78.  In  auditing  a  manufacturer's  accounts  would  you  incorporate  as  a 
part  of  profits  work  in  process  of  manufacture  and  finished  stock  on 
hand?     (Ind.,  June,  1916.) 

79.  The  Profit  and  Loss  account  of  a  manufacturing  company,  for  six 
months  ending  June  30,  1897,  contains  on  the  debit  side  (a)  stock  on 
hand  Januaiy  1,  1896,  (b)  purchases  of  raw  material,  (c)  manufacturing 
expenses,  (d)  expenses  of  selling;  on  the  credit  side  (a)  sales,  (6)  stock 
on  hand  June  30.  Does  the  balance  of  these  amounts  constitute  the  net 
profit  for  the  six  months,  or  should  other  charges  be  taken  into  account? 
If  so,  state  them.     (N.  Y.,  Dec,  1897;  Ohio,  March,  1910.) 

80.  A  manufacturer  has  become  convinced  that  he  is  not  making  the 
profit  he  should,  and  you  are  asked  to  find  out  the  condition  of  affairs. 
Explain  fully  what  should  be  done  to  analyze  the  situation  and  determine 
the  facts  in  the  case.     (Del.,  June,  1915.) 

81.  State  the  final  disposition  of  net  profit  in  the  books  of  a  partner- 
ship.    (N.  Y.,  June,  1900.) 

82.  State  briefly  what  the  effect  is,  if  any,  on  the  current  operating 
profits  of  each  of  the  following:  (a)  Failure  to  provide  adequate  reserve 
for  cash  discounts;  (b)  return  of  goods  sold  in  previous  period;  (c)  cre- 
ating a  reserve  for  a  bond  sinking  fund;  (d)  creating  a  reserve  for  con- 
tingencies; (e)  sale  of  machinery  in  excess  of  book  value;  (/)  excess  re- 
serve for  depreciation;  (g)  revaluing  merchandise  inventory  at  beginning 
of  period;  (h)  writing  up  the  value  of  securities  from  cost  to  market;  (i) 
bonds  issued  at  a  discount;  (j)  allowing  interest  on  partners'  capital. 
(Mass.,  Oct.,  1917.) 

83.  State  briefly  the  difference  between  income,  profits,  and  gain. 
(N.  Y.,  June,  1917.) 

Capital  and  Rfa'enue  Expenditures 

84.  Name  and  define  two  classes  of  expenditures.     (R.  I.,  Dec,  1907.) 

85.  Define:  Capital  expenditures.  (N.  Y.,  Dec,  1896;  N.  Y.,  Jan., 
1901*;  111.,  Nov.,  1903;  N.  Y.,  Oct.,  1907;  Cal.,  May,  1908*;  Mich,,  June, 
1910 ;  Va.,  Nov.,  1910 ;  Va.,  Oct.,  1911* ;  Mich.,  June,  1912 ;  Wis.,  April, 
1914;  III,  May,  1914;  Del.,  June,  1915;  Wash.,  June,  1915;  Wis.,  May, 
1916;  Wis.,  April,  1917;  Va.,  Nov.,  1918;  S.  C,  Sept.,  1919.) 


80  C.  P.  A.  ACCOUNTING 

86.  Define:  Revenue  expenditures.  (N.  Y.,  Dec,  1S96;  N.  Y.,  June, 
1899*;  N.  Y.,  Qct.,  1907;  Mich.,  June,  1910;  Ya.,  Nov.,  1910;  Ya.,  Oct., 
1911*;  Mich.,  June,  1912;  Wis.,  April,  1914;  Wis.,  May,  1916*;  Wis., 
April,  1917.*) 

87.  (a)  Define  charges  to  capital;  (6)  charges  to  revenue.  (Mass., 
Oct.,  1917.) 

88.  Explain  the  general  principles  to  be  observed  in  differentiating 
between  capital  and  revenue  expenditui-es.  (N.  Y.,  Jan.,  1902*;  Wash., 
May,  1903*;  111.,  May,  1904*;  N.  Y.,  Jan.,  1907*;  N.  Y.,  June,  1908*; 
Wash.,  Aug.,  1908*;  Md.,  Jan.,  1909*;  111.,  May,  1909*;  Mass.,  June, 
1910*;  Va.,  Nov.,  1910*;  Va.,  Oct.,  1912;  Ohio,  Nov.,  1913*;  Wis.,  April, 
1914;  Ohio,  Nov.,  1915*;  CaL,  May,  1916*;  Ohio,  Nov.,  1916*;  Mass., 
Oct.,  1917*;  Ohio,  Nov.,  1917*;  A.  I.  of  A.,  May,  1918;  Ohio,  Nov., 
1918*;  A.  I.  of  A.,  May,  1919*;  Ohio,  Oct.,  1919*;  Md.,  Oct.,  1919.*) 

89.  What  class  of  expenditures  should  be  treated  as  assets  at  the  close 
of  a  fiscal  period?     (N.  Y.,  Jan.,  1901;  111.,  May,  1910.*) 

90.  It  has  been  stated  that  "revenue  expenditures  do  not  create  assets." 
What  is^our  opinion?     (111.,  May,  1909.) 

91.  Under  what  condition  would  a  revenue  expenditure  appear  in  the 
Balance  Sheet  as  an  audit?     (Mo.,  Dec.,  1913.) 

92.  Explain  the  difference  between  capital  expenditures  and  revenue 
expenditures.  What  rule  controls  in  determining  whether  certain  pay- 
ments belong  to  capital  or  to  revenue?  When  in  doubt,  to  which  should 
the  payment  be  charged?     (N.  Y.,  June,  1901.) 

93.  Distinguish  between  capital  expenditure  and  expense.  (N.  D., 
July,  1916.) 

94.  If  in  the  course  of  an  audit  it  should  be  found  that  capital  ex- 
penditures had  been  charged  up  against  Profit  and  Loss  account,  what 
would  be  the  duty  of  the  auditor  in  respect  to  such  charges?  (N.  Y.,  Dec, 
1897;  A.  I.  of  A.,  May,  1919.*) 

95.  Under  what  circumstances,  if  any,  may  capital  expenditures  be 
charged  against  revenue?  (Ohio,  Nov.,  1915*;  Ohio,  Nov.,  1917*;  Ohio, 
Nov.,  1918.) 

96.  What  on  the  books  of  a  company  would  be  the  proper  method  of 
treating  cash  payments  for  capital  expenditures?     (Ohio,  May,  1910.) 

97.  In  the  audit  of  the  accounts  of  a  corporation  for  a  year  in  which 
the  business  has  made  a  loss  and  at  the  close  of  which  no  profits  are  avail- 
able for  dividends,  would  you  consider  it  your  duty  to  direct  the  same 
attention  as  usual  in  distinguishing  between  expenditure  on  fixed  assets 
chargeable  to  capital  and  that  chargeable  against  operations?  Give  your 
reasons.     (111.,  Dec,  1916.) 

98.  What  is  the  auditor's  general  duty  in  regard  to  capital  expendi- 
tures?    (A.  I.  of  A.,  May,  1919.) 

99.  State  which  of  the  following  should  be  charged  or  credited  to 
capital  and  which  to  revenue:  (a)  Repairs  to  machinery  and  plant;  (h) 


PROFIT  AND  LOSS  STATEMENT  81 

rejlacements  of  maeliinery  and  plant;  (c)  royalties  on  machines  owned 
and  used  by  the  company  owning  the  patents,  similar  machines  being 
leased  under  royalty  to  competitors;  (d)  brokerage  on  a  piece  of  property 
purchased;  (e)  costs  attending  a  mortgage  given;  (/)  costs  of  patents, 
including  lawyers  charges  and  govei'ument  fees;  (g)  expenses  of  incor- 
porating a  company;  (h)  discount  on  bonds  sold;  (i)  premium  on  bonds 
sold.     (Wis.,  April",  1915;  Mass.,  Oct.,  1917.*) 

100.  Expenditures  are  made  by  a  corporation  for  items  of  each  of  the 
following  classes:  (a)  Taking  dow-n  a  machine  in  one  pai't  of  a  factory, 
moving  it  and  putting  it  up  in  another  part;  (b)  expenses  of  incorporat- 
ing the  company,  including  State  charges  and  lawyer's  services;  (c)  bi'ok- 
erage  on  purchase  of  a  piece  of  property;  (d)  commission  on  an  issue  of 
debenture  bonds;  (e)  costs  attending  a  mortgage;  (/)  furniture  and 
fittings  of  a  city  office  and  salesroom;  (g)  cost  of  patents,  including 
solicitor's  charges  and  government  fees.  Which  items  should  be  charged 
to  capital  and  which  to  revenue?  State  reason  for  your  answer  in  each 
case.     (N.  Y.,  June,  1900.) 

101.  Would  an  auditor  be  justified  in  certifying  the  accuracy  of  ac- 
counts in  which  a  capital  expenditure  appeared  without  making  inquiry 
into  the  real  character  of  same?  If  not,  why  not?  If  he  would,  why? 
(Cal.,  May,  1908.) 


82 


C.  P.  A.  ACCOUNTING 


PROBLEMS 


PROFIT   AND   LOSS    STATEMiaTT 


1.     Following  is  condensed  trial  balance  of  the  Blank  Manufacturing 
Company  on  December  31,  1908,  before  closing  the  books  for  the  year: 


Debits 


Credits 


Inventories,  raw  materials  and  work  in  progress  Januarj- 

1,  1908 

Purchases  of  raw  materials  and  freight  thereon 

Productive  labor 

Factory  expenses  other  than  labor  and  materials 

Inventories  of  manufacturing  goods.  Januarj'  1,  1908. . . 

Administration  and  general  expenses 

Sales,  gross 

Freight  outwards 

Discounts  allowed 

Bad  debts  written  off 

Interest 

Discounts  received 

Cash  and  accounts  and  bills  receivable 

Accounts  and  bills  payable 

Real  estate  and  buildings 

Machinery  and  tools 

Furniture 

Capital  stock 

Undivided  profits 

Profit  from  sale  of  real  estate  in  1908 


$31,500 

253,o00 

56,o(X) 

11.500 

4,100 

23,700 

25,800 

1,600 

600 

3,500 

172.700 

20.000 

67.000 

1,300 


S673.300 


$408,700 

2,300 
95  000 


100,000 

63,300 

4.000 


$673,300 


The  inventories  of  December  31,  1908,  are:  Raw  materials  and  work 
in  progress,  $65,648;  manufactured  goods,  $1,991. 

A  reserve  of  $7,000  is  to  be  set  up  for  depreciation  on  machinery  and 
tools,  and  another  reserve  of  $3,000  is  to  be  provided  for  doubtful  ac- 
counts. 

A  commission  of  5  per  cent  on  the  net  profits  for  the  year  is  to  be 
credited  to  the  manager. 

After  taking  the  foregoing  items  into  account,  prepare  the  following: 
Lists  of  ledger  balances  after  closing  books ;  Manufacturing  account ; 
Trading  account;  Profit  and  Loss  account;  Balance  Sheet.  (Note:  Use 
technical  form  of  statements.)  (Wash.,  March,  1909.) 

2.     From   the   following  trial  balance  of   Maker   and    Sellers'   books, 


PROFIT  AND  LOSS  STATEMENT 


83 


extracted  on  December  31,  covering  six  months'  operations,  prepare  a 
Manufacturing,  Trading  and  Profit  and  Lo«s  account  and  Balance  Sheet, 
(Note:  Use  technical  form.) 


Cash  at  bank 

Petty  cash  in  hand 

Bills  receivable  on  hand 

Sundry  debtors 

Buildings 

Plant  and  machinery 

Sundry  creditors 

Loan  on  mortgage 

Material  on  hand  July  1  (raw  material) .  . . . 

Purchases 

Wages 

Discounts  allowed  on  purchases 

Discount  allowed  customers 

Returns  (customers'  returns  for  half-year) . . 

Sales 

Patent  rights  (expenses) 

Rent  and  taxes 

Advertising 

Traveler's  salary 

Carriage,  outward 

Bad  debts  written  off 

Repairs 

Patent  royalties  received  in  advance 

Royalties  on  patents  attributed  to  half-year 

General  expense 

Interest  on  loans 

Reserve  for  bad  and  doubtful  debts 

Reserve  for  discounts  on  book  debts 

Maker  capital  account 

Maker  drawing  account 

Seller  capital  account 

Seller  drawing  account 


Debits 


$3,000 
15 
1,000 
36,825 
20,000 
15,000 


13,705 

42,000 

7,020 

4,690 
1,650 

250 

500 

2,300 

2,150 

1,950 

500 

420 


2,510 
600 


6,000 
3,600 


$165,685 


Credits 


$9,850 
22,500 


1,950 
80,000 


2,500 
200 


2,700 

985 

30,000 

15,000 


$165,685 


The  goods  on  hand  (raw  material)  on  December  31,  are  valued  at 
$17,500. 

Write  off  5  per  cent  from  plant  and  machinery  for  depreciation  for 
the  half  year.  The  profits  are  to  be  apportioned  as  follows :  Maker,  two- 
thirds;  Seller,  one-third.  (Minn.,  Oct.,  1916.) 

3.  The  American  Manufacturing  Company  commenced  business  on 
January  1,  1918,  with  a  paid-up  cash  capital  equal  to  the  sales  for  the 
year  1918. 

The  net  profits  for  the  year  1918  were  $26,100, 

Of  the  total  charges  to  manufacturing  during  the  year,  40  per  cent  was 
for  materials,  30  per  cent  for  productive  labor,  and  30  per  cent  for  manu- 
facturing expenses  (including  5  per  cent  depreciation  on  plant  and  machin- 
ery, amounting  to  $3,000). 


84 


C.  P.  A.  ACCOUNTING 


The  value  of  the  materials  used  was  80  per  cent  of  the  amount  pur- 
chased, and  90  per  cent  of  the  amount  purchased  was  paid  during  the 
year. 

The  inventory  value  of  finished  goods  on  hand  at  December  31,  1918, 
was  10  per  cent  of  the  cost  of  finished  units  delivered  to  the  warehouse, 
and  the  work  in  process  at  that  date  was  equal  to  50. per  cent  of  the  cost 
of  units  delivered  to  the  warehouse. 

The  selling  and  administrative  expenses  were  equal  to  20  per  cent  of 
the  sales;  also  to  40  per  cent  of  the  cost  of  goods  sold.  Ninety  per  cent 
of  th&se  expenses  were  paid  during  the  year  1918. 

Plant  and  machinery  purchased  during  the  year  were  paid  for  in  cash. 

All  labor  and  manufacturing  expenses  (exclusive  of  depreciation)  were 
paid  in  full  up  to  and  including  December  31,  1918. 

Of  the  total  sales  for  the  year,  80  per  cent  was  collected  and  1  per  cent 
charged  off  as  worthless. 

From  the  given  data  you  are  required  to  prepare  a  Balance  Sheet,  and 
a  Profit  and  Loss  statement,  showing  cost  of  goods  delivered  to  the  ware- 
house, cost  of  goods  sold,  and  net  profit  for  the  year.  (Ohio,  Oct.,  1919.) 

4.  The  following  figures  are  shown  on  a  Balance  Sheet  for  January 
1,  1913: 


Real  Estate $100,000 


Machinery 

Merchandise 

Accounts  Receivable . 
Cash 


500,000 

150,000 

50,000 

50,000 


$850,000 


Capital  Stock $400,000 

Pounded  Debt 200,000 

Bills  Payable 50,000 

Accounts  Payable 75,000 

Surplus 125,000 


$850,000 


The  following  is  the  Income  Sheet  of  the  year  1912 : 


Sales 

Goods  in  process,  December  31,  1912  . 
Stores  on  hand,  December  31,  1912  .  . 
Merchandise,  December  31,  1912  .  .  .  . 


Less: 

Selling  costs . 


Goods  in  process,  January  1,  1912 

Stores  on  hand,  January  1,  1912 

Merchandise  on  hand,  January  1,  1912 

Supplies  purchased 

Wages  paid 

Wages  due  and  unpaid 

General  manufacturing  expenses 


Cost  of  Product , 


Net  profit 

Dividends  declared,  but  not  yet  paid  , 

Surplus  for  the  year 


$1,000,000 

30,000 

20,000 

100,000 


1,150,000 
250,000 


$20,000 
15,000 
80,000 

175,(K)0 

320,000 
30,000 

200,000 


$900,000 


$840,000 


$60,000 
32,000 


$28,000 


PROFIT  AND  LOSS   STATEMENT 


85 


Is  the  Balance  Sheet  consistent  with  the  Income  Sheet?  If  not,  assume 
the  Ledger  and  the  totals  of  both  sides  of  the  Balance  Sheet  to  be  correct, 
and  any  error  to  have  been  caused  by  unwarranted  combinations  or  can- 
cellations of  accounts,  and  then  correct  the  Balance  Sheet. 

Show  the  Ledger  with  both  sides  of  all  manufacturing  accounts  when 
closed  for  December  31,  1912. 

The  Balance  Sheet  on  January  1,  1912,  was  as  follows: 


Real  Estate S100,000 

Machinery 500,000 

Goods  in  Process 20,000 

Finished  Goods 80,000 

Stores 15,000 

Accounts  Receivable 75,000 

Cash 35,000 

$825,000 


Capital  Stock $400,000 

Funded  Debt 200,000 

Bills  Payable 40,000 

Accounts  Payable 60,000 

Surplus 97,000 

Dividends 28,000 


$825,0(X) 


Is  this  consistent  with  the  Income  Sheet  and  with  the  1913  Balance 
Sheet  as  corrected? 

(Cal.,   June,  1917.) 

5.  The  Metal  Products  Company  rents  the  ground  on  which  its  plant 
stands  for  a  long  term  of  years ;  it  sells  its  own  product  only  and  maintains 
a  downtown  office.  Its  books  disclose  the  following  data  for  its  last  fiscal 
year;  25  per  cent  of  the  office  expenses  and  wages  and  salaries  of  officers 
are  considered  as  entering  into  the  cost  of  production ;  65  per  cent  of 
these  accounts  are  deemed  to  be  selling  expenses  and  10  per  cent  are 
unallocated. 

Metal,  Products  Company,  Balances  of  Operating  Accounts 


Advertising $54,600 

Cartage  on  purchases 120 

Cartage  on  sales 1,000 

Depreciation : 

Factory  furniture 320 

Office  furniture 580 

Machinery  and  buildings  .  19,120 

Discount  on  sales 10,150 

Discount  on  purchases 12,500 

Factory  expenses 5,100 

Office  expenses  and  wages . . .  20,200 

Interest  paid 70 

Insurance,     machinery    and 

buildings 3,880 

Inventory  beginning  of  period  100,000 

Labor 37,900 


Light,  fuel,  and  power $8,350 

Purchases 381,500 

Royalty  on  output 30,000 

Repairs  to  equipment 7,600 

Rent  of  factory 4,300 

Salesmen's  salary  and  expense  2 1 ,000 
Superintendent  and  factory 

clerks 9,050 

Salaries  of  officers 24,000 

Supplies  at  factory 19,000 

Taxes  on  real  estate 1,700 

Plating  expenditures 20,500 

Reserve  for  doubtful  accounts  1,000 

Sales 660.000 

Inventory  at  close  of  period .  160,000 


Prepare  from  the  figures  (1)  the  cost  of  goods  sold;  (2)  the  manu- 
facturing profit;  (3)  the  gross  and  net  realization  from  sales;  (4)  the 
trading  profit;  (5)  the  other  profits,  losses  and  expenses,  as  the  case  may 
be;  and,  lastly,  the  net  profit  for  the  period. 

(Kan.,  Dec,  1915;  Mo.,  Dec,  1915.) 


CHAPTER  V 
PARTNERSHIP  AT  ORGANIZATION 

Partnership  Defined — A  partnership  may  be  defined  as  an  association, 
not  incorporated,  of  two  or  more  persons  who  have  agreed  to  combine 
their  labor,  propertj^,  and  skill,  or  some  of  them,  for  the  purpose  of  engag- 
ing in  ?i\\y  lawful  trade  or  business  and  sharing  profits  and  losses  as  well 
l)etween  them/  A  partnership  may  also  be  defined  as,  (a)  a  contract  of 
mutual  agency,  each  partner  acting  as  a  principal  in  his  own  behalf,  and 
as  an  agent  for  his  copartners,*  and  (b)  the  contract  relation  existing 
l)etween  persons  who  have  combined  their  property,  labor,  and  skill  in  an 
enterprise  or  business  as  principals,  for  the  purpose  of  a  joint  profit/ 

It  is  extremely  difficult  to  lay  down  any  definite  tests  for  determining 
the  existence  of  a  partnership,  as  each  case  must  be  judged  upon  pertinent 
facts,  it  being  necessary  to  prove  the  intention  of  the  parties  to  form  a 
partnership*  in  addition  to  proving  the  other  necessary  elements  of  an 
ordinary  contract.' 

Kinds  of  Partnerships — There  ai'e  two  broad  classes  of  partnerships, 
namely,  (a)  general,  or  partnerships  in  which  all  the  partners  are  tenants 
in  common,  each  partner  bearing  with  his  associates  the  liabilities  incuiTed 
by  individual  partners  acting  as  agents  for  their  copartners,'  and  (b) 
limited,  or  partnerships  composed  of  general  partners  who  are  liable  for 
the  debts  of  the  firm  as  in  the  general  partnei'ships,  and  special  partners 
who  are  liable  only  for  the  amount  of  their  investments.' 

The  law  also  recognizes  the  following  kinds  of  pai'tners : 

(a)  Silent  (sometimes  called  dormant),  or  partners  who  take  little  if 
any  active  part  in  the  partnership's  activities." 

(b)  Secret,  or  partners  who  are  not  known  as  partners,  although  they 
may  be  actively  engaged  in  the  business.' 

(e)  Nominal,  or  those  who  are  not  partners  by  contract  or  agreement, 
but  may  legally  be  so  considered,  due  to  having  knowingly  permitted  them- 
selves to  be  held  out  to  the  public  as  partners.'" 

(d)  Ostensible  (sometimes  called  public),  or  those  Avho  hold  themselves 
out  and  are  known  as  partners." 

Advantages  and  Disadvantages  of  Partnership — The  chief  advantages  of 
the  partnership  form  of  organization  are  (a)  larger  capital  and  therefore 
access  to  fields  closed  to  sole  proprietorships,  (b)  the  combining  of  the 
l)usiness  wisdom,  experience,  and  skill  of  several,  and  (c)  the  subdivision 
of  duties  with  the  resulting  increase  in  specialization."  The  chief  disad- 
vantages are  (a)  the  possibility  of  friction  between  partners,   (b)  .slower 


'For  explanation  of  superior  figures  see  page  337. 

86 


PAKTNEKSHIP  AT  ORGANIZATION  87 

action  than  in  sole  piopiielorships,  and  (c)  unlimited  liability  of  partners 
to  extent  of  entire  personal  fortunes." 

Liabiliti/  of  Partners — Except  for  the  limited  partners  who  i-isk  only 
their  investments/''  partners  are  considered  as  sureties  for  their  copai-tners, 
and  so  ai-e  ultimately  liable  even  to  the  amount  of  their  private  estates 
for  the  full  extent  of  the  firm's  debts."  They  are,  however,  entitled  to 
contributions  from  copartners  for  the  proportionate  share  of  the  Arm's 
debts  which  the  copartners  should  have  paid.'" 

Retiring  partners  are  liable  as  sureties  for  the  due  and  undue  obligations 
at  the  time  of  retirement  even  though  the  vendees  have  agreed  to  liquidate 
the  liabilities  and  have  given  the  vendors  an  indemnity  bond  against  any 
liability  for  the  firm's  debts.''  Continuing  partners  are  jointly  liable  with 
the  retiring  partners  for  the  deljfs  of  the  old  firm.''  Incoming  partners 
are  not  liable  for  the  debts  of  the  old  firm  unless  they  agree  to  undertake 
such  liability,  oi-  unless  they  perfonn  an  act  indicating  such  intent." 
However,  they  may  be  considered  to  have  accepted  the  liability  if  they  fail 
to  state  their  freedom  from  such  liability.^" 

Authority  of  Partners — Every  general  partner  is  an  agent  of  the  partner- 
ship in  the  transaction  of  business,  and  has  authority  to  transact  the  ordi- 
nary business  of  the  concern."  However,  unless  his  partners  have  left  the 
administration  of  the  busitiess  entirely  to  him  or  are  incapable  of  acting, 
he  cannot  legally  do  such  acts  as  (a)  dispose  of  the  goodwill  of  the  busi- 
ness, (b)  as  would  render  the  continuation  of  the  ordinary  business  im- 
possible, (c)  sell  at  one  time  all  the  partnership  property,  unless  it  con- 
sists entirely  of  merchandise,  (d)  confess  a  judgment,  (e)  submit  a  part- 
nership claim  to  ai-bitration,  and  (f )  assign  any  or  all  of  the  partnership 
propej'ty  to  a  creditor,  or  to  a  trustee  for  the  benefit  of  one  or  more 
crecUtors.^^ 

Articles  of  Copartnership — The  leading  features  which  should  be  pro- 
vided for  in  the  articles  of  copartnership  are  (a)  date  of  commencement, 
(b)  parties,  (c)  nature  of  business,  (d)  location  of  biisiness,  (e)  firm  name, 
(f)  duration  of  partnership,  (g)  individual  investments  (if  other  than 
cash,  the  basis  of  valuation  should  be  stated),  (h)  rights  and  duties  of 
partners,  (i)  di\dsion  of  profits  and  losses,  (j)  salaries,  (k)  personal  draw- 
ings, (1)  books  of  account,  (m)  auditing  of  books,  (n)  interest  on  invest- 
ments and  drawings,  and  (o)  settlement  at  dissolution  including  valuation 
of  goodwill  and  possession  of  firm  name." 

General  Partnership  Rules — Unless  specific  provisions  to  the  contrary 
are  contained  in  the  articles  of  copartnership,  or  unless  it  can  be  proved 
that  the  partners  intended  otherwise,  the  following  rules  prevail : 

(a)  Partners  are  entitled  to  interest  on  their  advances  and  on  payments 
personally  made  foi-  the  benefit  of  the  firm.''^ 

(b)  Interest  is  not  allowed  on  investments.'"' 

(c)  Interest  is  not  charged  on  Avithdrawals.'"' 

(d)  Interest  is  not  charged  on  money  held  by  a  pai-tner  during  liquida- 
tion of  partnership.'^ 


88  C.  P.  A.  ACCOUNTING 

(e)  If  no  rate  is  mentioned,  the  legal  rate  of  interest  prevails  in  interest- 
bearing  transactions.™ 

(f)  Profits  and  losses  are  shared  equally.^ 

(g)  Salaries  are  not  allowed  to  partners.^ 

(h)  All  partners  have  an  equal  duty  to  keep  books  of  account  to  which 
each  has  free  access.*^ 

Partners'  Proprietorship  Accounts — The  capital  accounts  of  a  partner- 
ship record  the  difference  between  the  assets  and  liabilities  of  the  business." 
The  components  of  the  equity  of  partners  are  sometimes  classified  thus:" 

Negative  Positive 

Deferred  Debit  Items  Deferred  Credit  Items 

Drawing  Account  (Debit  Balance)  Drawing  Account  (Credit  Balance) 

Undivided  Profits 

This  classification,  however,  differs  from  the  ordinary  one  in  that  de- 
ferred charges  and  credits  to  income  are  considered  to  be  capital  items 
instead  of  assets  and  liabilities,  respectively. 

The  proprietorship  accounts  of  partnerships  may  be  divided  into  three 
classes : 

(a)  Capital,  or  accounts  which  show  the  original  investment  plus  the 
profits  left  in  the  business  minus  withdrawals  of  capital,  if  the  articles  of 
copartnership  permit  this  to  be  done.**  Where  a  partner  has  agreed  to 
invest  a  specific  sum,  his  capital  account  should  show  this  sum,^  and  any 
deficiency  due  to  insufficient  actual  investment  should  be  shown  on  the 
books  as  a  charge  against  the  partner's  personal  account.^* 

(b)  Personal,  or  accounts  which  show  the  charges  for  losses,  with- 
drawals, or  interest  on  same,  and  credits  for  additional  investments,  interest 
on  investment,  salary,  and  profits."  The  personal  account  may  be  sub- 
divided into  a  salary  account  and  a  di'awings  account.^  Where  charges 
to  a  partner's  personal  account  represent  payments  made  in  currency, 
the  auditor  should  have  the  partner  approve  the  account  as  it  appears  in 
the  ledger," 

(c)  Undivided  profits  accounts,  or  those  representing  the  balance  of  the 
profit  and  loss  account  which  has  not  yet  been  allocated  to  the  partners.*" 

The  proprietorship  accounts  of  copartnerships  appear  in  the  appropria- 
tion section  of  the  profit  and  loss  statements,  as  follows :" 

Net  Profit  for  Period    remainder  of  statement 

omitted) xxxxxxx 

Appropriation: 

Jones,  Personal  Account  (3/5) xxxxxxx 

Smith,  Personal  Account  (2/5) xxxxxxx    xxxxxxx 

The  partners'  net  worth  accounts  may  be  shown  on  the  balance  sheet 
without  detail,  thus:" 

Net  Worth: 

Jones,  Capital xxxxxxx 

Smith,  Capital xxxxxxx     xxxxxxx 


PARTNERSHIP  AT  ORGANIZATION  89 

If  it  is  desired  to  show  all  elements  of  the  proprietorship  accounts  of 
the  partners  on  the  balance  sheet,  they  would  appear  as  follows:" 

Net  Worth: 

Jones,  Capital,  Jan.  1,  1920 xxxxxxx 

Share  of  net  profits  for  1920 xxxxxxx 

Salary  for  1920 xxxxxxx 

xxxxxxx 
Drawings  for  1920 xxxxxxx     xxxxxxx 

Smith,  Capital,  Jan.  1920 xxxxxxx 

Share  of  net  profits  for  1920 xxxxxxx 

Salary  for  1920 xxxxxxx 

xxxxxxx 
Drawings  for  1920 xxxxxxx    xxxxxxx    xxxxxxx 


Some  accountants  classify  a  partner's  undrawn  salary  as  a  current 
liability,  and  the  debit  and  credit  balances  to  a  partner's  drawing  account 
as  a  current  asset  and  current  liability,  respectively.'"  As  a  partner  cannot 
bring  suit  on  these  items  and  as  the  individual  creditors  of  the  partner 
cannot  consider  such  items  as  part  of  the  partner's  private  estate  until  the 
firm's  debts  have  been  paid,  it  seems  preferable  to  list  these  items  separately 
under  the  net  worth  accounts. 

Loans  of  Partners — A  partner  may  loan  money  to  his  firm  either  by  con- 
tributing capital  in  excess  of  the  amount  agreed  upon  or  by  leaving  his 
profits  in  the  business."  These  loans  which  may  or  may  not  be  covered  by 
notes,  are  sometimes  included  in  the  ordinary  capital  accounts,  but  it  is 
preferable  to  show  the  amounts  in  special-loan  accounts.*'  Partners'  loans 
rank  with  equal  status"  after  the  claims  of  outside  creditors  and  before 
the  ordinary  proprietorship  accounts  in  case  of  insolvency  and  dissolution 
of  the  partnership.**  Partners'  loan  accounts,  like  partners'  undrawn 
salary  accounts,  should  be  considered  as  net  worth  items  rather  than 
liabilities.  Unless  specifically  stated  otherwise  in  the  articles  of  copartner- 
ship, partners'  loans  bear  interest,  and,  if  no  rate  of  interest  is  specified, 
the  court  will  allow  the  legal  rate.*  If  profits  or  losses  are  distributed  on 
the  basis  of  capital  invested,  partners'  loans  are  not  counted  as  capital.*" 

Opening  Entry  for  Partnership  Books — When  a  partnership  agreement 
is  made  and  it  is  decided  how  much  capital  each  partner  should  furnish, 
the  opening  entry  would  be  framed  as  follows :" 

A,  Personal  Account xxxxxxx 

B,  Personal  Account xxxxxxx 

A,  Capital  Account xxxxxxx 

B,  Capital  Account xxxxxxx 

Then  when  each  partner  contributes  the  capital  agreed  upon,  the  follow- 
ing entries  would  be  made :" 


90  C.  P.  A.  ACCOUNTING 

Sundry  Assets  (itemized) xxxxxxx 

A,  Personal  Account xxxx  xx 

Sundry  Assets  (itemized) xxxxxxx 

Sundry  Liabilities  (itemized) xxxxxxx 

B,  Personal  Account xxxxxxx 

Guaranteed  Assets — Sometimes  a  partner  turns  into  the  new  fii-m  assets 
which  he  guarantees  to  be  worth  the  value  assigned  to  them.  Upon  subse- 
quent default  the  loss  must  be  borne  by  the  partner.  The  entry  to  be 
made  in  case  a  guaranteed  account  receivable  proved  wortliless  would  be." 

(Partner),  Personal  Account xxxxxxx 

Accounts  Receivable  (Customer) xxxxxxx 

Losses  on  assets  accepted  by  the  firm  as  of  a  certain  value,  without  the 
guarantee  of  the  contributing  partner,  cannot  be  charged  to  that  partner, 
but  must  be  charged  to  the  firm's  profit  and  loss  account." 

Buying  Interest  in  Business — The  purchase  of  an  interest  in  the  profits 
must  be  distinguished  from  the  purchase  of  a  share  in  the  business." 
Assimiing  that  B  wishes  to  purchase  a  half  interest  in  the  business  of  A, 
whose  net  worth  is  $5,000,  there  are  ten  possible  entries,  viz. : 

(a)  If  B  pays  into  the  business  exactly  book  value  of  interest  in  busi- 
ness purchased,  the  entry  would  be :" 

Cash $5,000 

B,  Capital  Account $5,000 

(b)  If  B  pays  to  A  personally  exactly  book  value  of  interest  in  business 
purchased,  the  entry  would  be:" 

A,  Capital  Account $2,500 

B,  Capital  Account $2,500 

(c)  If  B  pays  into  the  business  less  than  book  value  of  interest  in  busi- 
ness purchased,  without  bringing  in  goodwill,  the  entry  would  be  :** 

Cash. $4,000 

A,  Capital  Account 500 

B,  Capital  Account $1,500 

(d)  If  B  pays  into  the  business  less  than  book  value  of  interest  in  busi- 
ness purchased  but  brings  in  goodwill,  the  entry  would  be :'" 

Cash $4,000 

Goodwill 1 .  000 

B,  Capital  Account $5 ,  000 

(e)  If  B  pays  A  personally  less  than  book  value  of  interest  in  business 
l)urchased  without  bringing  goodwill  into  the  business,  the  entry  would  be." 

A,  Capital  Accoimt $2 .  500 

B,  Capital  Account $2 ,  500 

(f )  If  B  pays  to  A  personally  less  (ban  book  value  of  interest  in  busi- 
ness purchased  but  brings  goodwill  into  the  business,  the  entry  would  he : 

Goodwill $1 ,000 

A,  Capital  .\ccount 2 ,  000 

B,  Capital  Account $3 ,000 


PARTNERSHIP  AT  ORGANIZATION  91 

(j?)  If  B  pays  into  the  Jjiisiness  more  than  book  value  of  interest  in 
business  purchased,  the  entry,  if  goodwill  is  eliminated,  would  be:"* 

Ca»sh $6,000 

A,  Capital  Account $500 

B,  Capital  Account 5,500 

(h)  If  B  pays  into  the  business  more  than  ))ook  value  of  interest  in 
business  purchased,  the  entry,  if  goodwill  is  shown,  would  be :"' 

Goodwill $1,000 

A,  Capital  Account $1 ,  000 

Cash 6,000 

B,  Capital  Account 6,000 

( i )  If  B  pays  to  A  personally  more  than  book  value  of  interest  in  busi- 
ness purchased,  the  entry,  if  goodwill  is  eliminated,  Avould  be :"" 

A,  Capital  Account $2,500 

B,  Capital  Account $2, 500 

(j)  If  B  pays  to  A  personally  more  than  book  value  of  interest  in 
business  purchased,  the  entry,  if  goodwill  is  shown,  would  be  :"* 

Goodwill $1 ,000 

A,  Capital  Account $1 ,  000 

A,  Capital  Account 3 ,  000 

B,  Capital  Account 3,000 

Buying  Share  in  Profits — As  profits  are  not  necessaiily  distributed 
according  to  capital  investments,  all  that  is  necessary  to  do,  when  a  share 
in  the  profits  is  purchased  by  the  incoming  partner,  is  to  make  the  following 
entry  /' 

Assets  (itemized) xxxxxxx 

(New  Partner)  Capital  Account xxxxxxx 

Audit  of  Partnerships — Having  partnership  books  adjusted  by  profes- 
sional accountants  is  especially  important,  because  they  will  act  impartially 
and  comply  fully  with  the  articles  of  copartnership,  thus  eliminating  fric- 
tion between  partners.''"  In  the  case  of  the  limited  partnerships,  an  inde- 
jjendent  audit  is  absolutely  necessary  for  the  protection  of  the  limited 
partners,  who  have  no  legal  right  to  interfere  with  the  active  management 
of  the  business."' 

In  an  audit  of  a  partnership,  the  partnership  agreement  should  be  called 
for  and  very  detailed  notes  taken  of  its  contents."*  An  attempt  should  be 
made  to  secui-e  a  certified  copy  for  the  auditor's  permanent  files.™ 

The  auditor  should  conform  to  the  spirit  of  the  partnership  agreement, 
and  where  this  is  impossible,  his  first  resort  should  be  to  interpret  the 
articles  in  the  light  of  ordinary  business  practices,  and  this  failing,  to 
seek  assistance  from  the  law  of  partnei'ships.'" 

Joint  Stock  Companies — A  joint  stock  company  is  a  partnership  whose 
capital  is  divided,  or  agreed  to  be  divided,  into  shares  transferable  without 
tlie  express  consent  of  all  the  copartners." 

Joint  stock  companies  are  created  by  contract."  In  some  states  a  cer- 
tificate of  association  containing  the  name  of  the  joint  .stock  company. 


92  C.  P.  A.  ACCOUNTING 

the  date  of  its  organization,  the  number  of  its  stockhoklers,  the  names 
and  residences  of  its  directors,  and  the  principal  place  of  business  must 
be  tiled  with  the  Secretary  of  State." 

Joint  stock  companies  are  governed  by  directors,  the  members  not  being 
agents  as  is  the  case  in  partnerships.'*  The  legal  relation  of  the  members 
to  the  company,  and  the  liability  of  members  to  creditors  is  practically 
the  same  as  in  partnerships." 

Joint  stock  companies  are  not  terminated  by  transfer  of  shares,  bank- 
ruptcy or  death  of  members,  or  war,  but  terminate  on  lapse  of  period 
shown  in  articles  of  association,  on  bankruptcy  of  company,  on  dissolution 
by  court,  or  if  all  shares  are  owned  by  same  member." 


PAKTNKRSrilP   AT    ORGANIZATION  9:} 


QUESTIONS 

PARTNERSHIP    AT   ORGANIZATION 

1.  Explain  fully  a  partnership.     (Pa.,  Nov.,  1903;  Iowa,  Dec,  1918.) 

2.  Explain  fully  a  limited  partnership.  (Pa.,  Nov.,  1903;  Iowa, 
Dec,  1918.) 

3.  What  is  a  joint  stock  company'?     (Iowa,  Dec,  1918.) 

4.  Define:  Partners'  Capital  account.  (Wash.,  Nov.,  1913*;  Wash., 
July,  1917.) 

5.  Give  an  example  of  opening  the  books  of  a  partnership,  using 
your  own  figures.     (Pa.,  Nov.,  1903;  Iowa,  Dec,  1918.*) 

G.  Give  an  example  of  opening  the  books  of  a  limited  partnership, 
using  your  own  figures.     (Pa.,  Nov.,  1903.) 

7.  Give  the  necessary  books  and  papers  in  a  partnership.  (Pa., 
May,  1902;  Mich.,  July,  1906.*) 

8.  Explain  the  ways  in  which  the  books  and  accounts  kept  by  a  firm 
conducting  any  line  of  business  would  differ  from  those  of  an  individual 
conducting  the  same  business.     (Mich.,  June,  1913;  Mich.,  Dec,  1914.*) 

9.  Wherein  do  the  books  of  a  copartnership  differ  from  those  of  a 
corporation  in  the  same  line  of  business?     (Mich.,  Nov.,  1907.) 

10.  What  points  should  be  covered  in  a  partnership  agreement? 
(Mich.,  June,  1910*;  111.,  May,  1913;  Del.,  June,  1915.) 

11.  What  is  the  value  of  the  partnership  papers  and  what  effect  have 
they  in  opening  and  closing  of  books?     (Pa.,  Nov.,  1901.) 

12.  What  records  and  documents  should  an  auditor  have  access  to 
in  an  audit  of  a  partnership?  Give  reasons.  (N,  Y.,  Jan.,  1901*;  Pa., 
May,  1902*;  Kan.,  May,  1916.) 

13.  In  making  an  audit  of  a  partnership,  what  information  of  im- 
portance would  an  auditor  get  from  the  partnership  agreement  (a)  in 
determining  the  partners'  profits;  {b)  in  determining  the  partners'  in- 
terest in  business?  (Md.,  Oct.,  1903*;  N.  Y.,  Jan.,  1916*;  Mass.,  Oct., 
1917.) 

14.  How  would  you  organize  a  set  of  books  for  a  partnership? 
(Ohio,  Dec,  1908.) 

15.  In  arranging  accounts  for  a  partnership,  how  should  original 
capital  be  treated?     (Fla.,  July,  1909.) 

16.  In  auditing  the  accounts  of  a  private  firm  where  there  are  several 


94  C.  P.  A.  ACCOUNTING 

partners,  to  what  points  should  the  auditor  look  in  order  to  be  assured 
that  the  partnei''s  accounts  are  correctly  stated  in  the  Balance  Sheet? 
(N.  Y.,  June,  1899;  N.  Y.,  June,  1902*;  Wash.,  March,  1909*;  Ind.,  June, 
1916.*) 

17.  Name  some  of  the  distinctive  elements  in  an  audit  of  partnerships. 
(N.  C,  June,  1916.) 

18.  Analyze  and  discuss  the  following  clause  taken  from  a  certain 
partnership  agreement : 

"VIII.  And  it  is  further  agreed  that  the  said  party  of  the  second  part 
is  to  pay  to  the  said  party  of  the  first  part  the  sum  of  three  thousand 
dollars  ($3,000)  ;  for  which  the  said  party  of  the  second  part  shall  receive 
a  one-third  (1/3)  interest  in  said  business  of  the  said  party  of  the  first 
part." 

(N.  Y.,  Jan..  1917.) 

19.  In  making  a  general  audit  of  a  Minnesota  corporation  of  which 
C.  D.  Boyd  is  secretaiy  and  manager,  you  find  among  its  papers  what 
appeal's  to  be  a  contract  of  partnership  between  it  and  George  Owens, 
signed  respectively  by  Owens  and  by  C.  D.  Boyd  for  the  corporation, 
but  not  witnessed  as  to  either  signer  and  without  the  seal  of  a  corporation. 
No  other  papers  concerning  this  arrangement  are  on  file,  but  the  Ledger 
shows  a  cash  charge  to  the  partnership  for  the  amount  of  capital  required 
to  be  contributed,  by  the  corporation,  to  the  partnership. 

In  making  your  report  how  will  you  treat  this  transaction,  both  as  to 
the  initial  payment  and  any  unstated  accrued  results  of  the  partnership 
from  its  commencement  down  to  the  date  of  your  audit  ? 

(Minn.,  Oct.,  1916.) 


PARTNEHSTTIP  AT   OHfJAXTZATTOX  95 


PROBLEMS 


PARTNERSHIP  AT  ORGANIZATION 


1.  "A"  has  $5,000  invested  in  a  business.  He  sells  "B"  a  half  interest 
for  $2,000  an^  places  the  money  in  the  business.  Make  the  entry.  (N.  Y., 
Dec,  1898*;  Pa.,  Nov.,  1903*";  Mich.,  Nov.,  1907*;  Wash.,  Aug.,  1908*; 
Mich.,  June,  1915.) 

2.  "A"  has  .$5,000  invested  in  a  business.  He  sells  "B"  a  half  interest 
for  $3,000  and  keeps  the  money.  Make  the  entry.  (N.  Y.,  Dec,  1898*; 
Pa.,  Nov.,  1903*;  Mich.,  Nov.,  1907*;  Mich.,  June,  1914.) 

3.  "X"  and  "Y"  bought  merchandise  to  the  amount  of  $12,000.  "X" 
contributed  $7,500;  "Y"  $4,500.  They  afterwards  sold  "Z"  a  one-third 
interest  for  $6,000.  How  much  of  this  amount  should  "X"  and  "Y" 
receive  respectively  in  order  to  make  "X,"  "Y"  and  "Z"  equal  partners, 
assuming  (a)  money  paid  into  business  with  no  goodwill;  (b)  money  paid 
into  business  with  goodwill;  (c)  money  not  paid  into  business?  (N.  Y., 
June,  1914.*) 

4.  Two  partners,  Wilson  and  Peters,  find  at  the  end  of  the  first  year's 
business  that  the  Balance  Sheet  shows  Wilson's  interest  to  be  worth  $18,000 
and  Peters'  $9,000.  The  goodwill  of  the  firm  is  worth  $3,000.  Each 
partner  draws  profits  in  proportion  to  his  investment, 

Thej'  conclude  to  take  in  another  partner  and  to  give  him  a  one-quarter 
interest  in  the  new  firm.  What  sum  must  the  new  partner  contribute? 
How  will  the  partnership  accounts  appear  after  the  payment  in  of  the 
new  capital?     How  will  the  profits  be  divided?  (Cal.,  May,  1903.) 

5.  "A"  and  "B"  enter  into  a  partnership  and  will  share  profits  in  the 
proportions  indicated  by  their  investments.  "A"  furnishes  $25,000  and 
"B"  $15,000,  which  is  invested  in  lands  and  buildings,  $10,000,  and 
merchandise  $30,000.  However,  before  they  have  actually  commenced 
business,  "C,"  realizing  that  "A"  and  "B"  have  a  promising  venture, 
offers  to  buy  a  one-third  interest  in  the  business  for  $20,000.  "A"  agrees 
to  sell  provided  "B"  will  consent  to  pay  him  a  bonus  of  $4,000  out  of  his 
("B's")  share.     This  "B"  agrees  to  do  and  consents  to  the  sale. 

How  should  the  $20,000  be  divided  between  "A"  and  "B"  so  that  the 
interest  of  all  three  partners  will  be  equal?  (111.,  May,  1914.) 

6.  "A"  and  "B,"  each  carrj'ing  on  a  similar  business,  agree  to  form  a 
partnership  under  the  name  of  "A  and  B"  and  that  the  new  firm  should 
take  over  the  assets  of  each  partner  in  the  business,  and  assume  liabilities 
of  each. 

The  following  trial  balances  were  presented  by  each  of  the  partnerships 
on  commencing  business: 


1)G 


C.  P.  A.  ACCOUNTING 


Capital 

Machin  ry  and  Fixtures  . . 

Cash 

Bills  Receivable 

Book  Accounts  Recei. able 
Inventory  Merchandise. . . 

Wages 

Unpaid  wages 

Expense  Account 

Bills  Payable 

Suspense  Account 

Merchandise  Account .... 
Book  Accounts  Payable .  . 

Repairs  Account 

Rentals 

Rent 


$20,000 

$35,000 

$25,0(X) 

$45,500 

4,000 

8,000 

3,000 

3.000 

20,000 

25.000 

10,000 

15.000 

2.000 

4,000 

500 

3,000 

15,000 

18,000 

20,000 

26,000 

1,000 

3,000 

26,000 

35,000 

15,000 

25,000 

2,000 

3,000 

1,000 

1.000 

500 

500 

$82,500 

$82,500 

$125,000 

$125,000 

Formulate  the  opening  entries  for  new  partnei-ship.  (Pa.,  Nov.,  1906.) 

7.  "A"  and  "B"  carried  on  business  in  partnership  and  divided  profits 
and  losses  in  proportion  to  their  capital,  three-fifths  and  two-fifths,  re- 
spectively. On  January  1,  1915,  "A's"  capital  was  $52,500  and  "B's", 
$35,000,  as  shown  by  a  Balance  Sheet  of  that  date.  They  agreed  to  admit 
"C"  as  a  partner  from  the  same  date  on  the  following  terms:  (1)  Assets 
and  liabilities  and  capital  to  be  taken  as  shown  in  the  Balance  Sheet; 
(2)  $12,500  to  be  added  to  the  assets  for  goodwill;  (3)  the  amount  of 
goodwill  to  be  added  to  "A's"  and  "B's"  capital  in  the  proportion  in 
which  they  divide  profits;  (4)  "C"  to  pay  to  the  partnership  such  a  sum 
as  will  give  him  a  one-fifth  share  in  the  business. 

(a)  State  what  amoimt  of  capital  "C"  has  to  bring  in.  (b)  Set  out 
the  Capital  accounts  of  each  partner  in  the  new  partnership;  and  (c) 
state  in  what  proportions  the  profits  will  be  divided  in  the  future,  "A" 
and  "B,"  as  between  themselves  sharing  in  the  same  proportions  as  before. 

(Wash.,  May,  1916.) 

8.  "A"  and  "B,"  who  had  hitherto  been  in  business  separately,  decide 
to  enter  into  partnership  on  July  1,  1905.  The  Balance  Sheets  of  "A"  and 
"B"  were  on  that  date  as  follows : 


A 

B 

A 

B 

Liabilities 

Accounts  Payable 

Capital  Account 

$1,000 
5,000 

$1,500 
3,000 

Assets 

Furniture 

Accounts  Receivable 
(face  value) 

$750 

2,500 

2,550 

200 

$600 
1,500 

Merchandise 

2,000 

Cash 

400 

$6,000 

$4,500 

$6,000 

$4,500 

PARTNERSHIP  AT  ORGANIZATION 


97 


It  was  agreed  that  "A"  and  "B"  should  take  over  their  respective 
Accounts  Receivable  at  $200  and  $150  less  than  the  face  values  shown  in 
the  Balance  Sheets,  these  amounts  to  be  charged  against  their  Capital 
accounts  and  carried  on  the  partnei-ship  books  as  a  Reserve  for  Bad 
and  Doubtful  accounts.  Of  "B's"  furniture  only  $250  was  to  be  taken 
over  by  the  partnership.  With  the  above  exceptions  the  assets  and  lia- 
bilities of  the 'parties  were  to  be  taken  over  by  the  partnership  at  the 
Balance  Sheet  figures,  except  that  "B"  was  to  invest  in  the  partnership 
in  cash,  a  sum  which,  after  making  the  adjustments  above  referred  to, 
would  make  his  Capital  account  the  same  as  that  of  "A." 

Draw  the  Balance  Sheet  of  the  "A"  and  "B"  partnership  on  July  1, 
1905,  giving  effect  to  the  foregoing  provisions. 

(Wash.,  April,  1906.) 

9.  New,  Knott  and  Moore  are  partners,  sharing  profits  in  the  pro- 
portion of  their  investments.  On  December  31,  1915,  the  Balance  Sheet 
of  the  partnership  is  as  follows : 


Cash $18,000 

Other  Current  Assets 23,000 

Fixed  Assets 20,000 


$61,000 


Accounts  Payable $1,000 

Moore  Capital 24,000 

New  Capital 24.000 

Knott  Capital 12,000 

$61,000 


Moore  decides  to  retire  from  active  business  and  agrees  to  sell  his 
interest  to  the  other  two  partners  for  $26,400,  taking  $14,400  in  cash 
and  the  balance  in  three  equal  instalments  payable  July  2,  1916,  January 
2  and  July  2,  1917,  evidenced  by  notes  payable. 

The  business  is  very  prosperous,  but  it  becomes  increasingly  evident  that 
more  capital  is  required,  especially  in  view  of  the  approaching  maturity 
of  the  first  note  given  to  Moore.  New  and  Knott  decide  to  admit  John 
Less  as  partner  as  of  date  July  1,  1916,  at  which  time  the  current  assets 
have  increased  by  $16,000,  accounts  payable  by  $10,000  and  the  partners' 
Capital  accounts  by  $6,000.    They  value  the  goodwill  at  $12,000. 

Less  buys  a  one-third  interest,  but  stii^ulates  that  all  he  pays  must 
remain  in  the  business  and  that  the  goodwill  shall  not  appear  upon  the 
books. 

How  much  must  he  pay  for  the  one-third  interest  ?  Present  the  Balance 
Sheet  of  the  firm  of  New,  Knott  and  Less  as  of  date  July  1,  1916.  (Ig- 
nore accrued  interest  on  Moore  notes.) 

(Cal.,  May,  1916.) 

10.  January  1,  1913,  "A"  and  "B"  signed  articles  of  copartnership 
to  engage  in  a  mercantile  business,  agreeing  to  invest  $15,000  and  $25,000 
respectively.  Profits  were  to  be  divided  in  proportion  to  capital  eoo- 
tributed,  and  interest  at  5  per  cent  was  to  be  allowed  on  investments  in 
excess  of  the  agreed  contributions  and  was  to  be  charged  on  deficits  under 
the  agreed  contributions. 

The  trial  balance  of  their  books  on  December  31,  1913,  was  as  follows: 


ns 


C.  p.  A.  ACCOUNTTNG 


Purchases 

Office  Expense 

Real  Estate 

Building 

Accounts  Receivable 

Cash  on  Hand 

Notes  Receivable 

Furniture  and  Furnishings 

Discounts  Earned 

Accounts  Payable 

Salaries  and  Wages 

Notes  Payable 

Sales 

A 

B 


$60,000 

1,000 

5,000 

10,000 

12,000 

1,000 

.8,000 

2,000 


4,000 


$1,000 
7,000 

4,000 
55,000 

9,000 
27,000 


The  merchandise  on  hand  is  valued  at  $10,000. 

After  allowing  for  interest  on  invastments,  divide  the  net  profits  ac- 
cording to  the  agreement.  Give  the  Capital  accounts  of  each  partner 
and  the  Balance  Sheet  as  of  January  1,  1914. 

On  this  date  a  third  party,  "C,"  desired  to  enter  the  partnership,  and  it 
was  agreed:  (a)  That  "C"  pay  $9,000  in  cash  for  a  one-fourth  (i/4)  interest 
in  the  new  concern;  {b)  that  goodwill  of  "A"  and  "B"  be  valued  at  $3,000; 
(c)  that  "A"  and  "B"  adjust  their  capitals  so  that  they  hold  a  one-fourth 
(1/4)  and  one-half  (Yz)  interest,  respectively,  in  the  new  firm;  (d)  that 
profits  and  losses  are  to  be  divided  according  to  capital  eontnbutions. 

In  the  adjustment,  "A"  received  cash  (out  of  the  $9,000  paid  in  by 
"C")  for  his  excess  investment,  while  "B"  received  the  remainder  of  the 
$9,000  paid  in  by  "C"  and  the  balance  due  him  was  considered  a  loan  to 
the  partnership. 

Give  Journal  entries  necessary  to  record  the  above  facts  and  the  opening 
Balance  Sheet  of  the  new  firm. 

April  1,  1914,  "A,"  "B"  and  "C"  agreed  to  sell  the  business.  Ui>  to 
this  time  each  partner  had  withdrawn  $500.  The  assets  were  disposed  of 
for  $24,500  cash,  and  the  vendee  assumed  the  liabilities  of  the  partnersliip. 
How  should  this  sum  of  $24,500  be  divided  among  the  partners?  Show 
the  exact  relation  of  the  partners  to  one  another  by  accounts  or  statements. 

(Wis.,  April,  1914.) 

11.     The  firm  of  "A"  and  "B"  have  the  following  statement : 


Store $15,000 

Accounts  Leceivable. 12,000 

Cash 9,000 

Furniture  and  Fixtures 2,800 

Merchandise 37,000 

Miscellaneous  Equipment.  . .  .  4,200 


$80,000 


Accounts  Payable $10,000 

BillsJayable 5,000 

A,  Capital 30,000 

B,  Capital 35,(X)0 


$80,000 


"C"  is  admitted  as  a  special  partner  with  the  following  arrangement: 
"C"  to  contribute  $30,000  and  to  be  entitled  to  one-third  of  the  profit  for 


PARTNERSHIP  AT  ORGANIZATION  90 

one  year.  Before  making  the  contribution,  the  following  changes  to  be 
made  in  the  books:  Store  to  be  marked  down  5  per  cent;  allowance  for 
doubtful  accounts  to  be  created  amounting  to  2  jier  cent;  merchandise  to 
be  revahied  at  $35,000;  furniture  and  fixtures  to  be  valued  at  $2,500.  At 
the  end  the  amount  of  goodwill  to  be  fixed  at  three  times  the  net  profits 
for  the  year  in  excess  of  $20,000,  this  goodwill  to  be  set  up  on  the  books, 
the  corresponding  credit  being  to  "A"  and  "B"  equally — "A,"  "B"  and 
"C"  each  to  draw  $3,000  in  cash,  the  remaining  profits  to  be  carried  to 
their  Capital  accounts. 

During  the  year  the  following  transactions  took  place:  Merchandise 
bought  on  credit,  $240,000;  cash  purchases,  $25,000;  cash  sales,  $125,000; 
sales  on  credit,  $175,000;  accounts  payable  paid  (face,  $245,000,  discount 
2  per  cent),  $240,100;  accounts  receivable  collected  (face,  $170,000,  all 
net  except  $50,000,  on  which  2  per  cent  allowed),  $169,000;  buying  ex- 
penses, paid  cash,  $1,500;  selling  expenses,  paid  cash,  $21,000;  delivery 
expenses,  paid  cash,  $9,000;  management  expenses,  paid  cash,  $4,500; 
miscellaneous  expenses,  paid  cash,  $3,000;  interest  on  notes  payable,  paid 
cash,  $250;  partners  each  withdrew  $3,000  cash  as  agreed. 

In  closing  the  books  for  determining  profits  and  goodwill,  the  following 
were  agreed  upon :  Value  of  merchandise  on  hand,  $60,000 ;  depreciation 
on  store,  $285;  additional  allowance  for  doubtful  debts,  $165;  furniture 
and  fixtures  written  down,  $200. 

Goodwill  having  been  estimated  and  duly  entered,  "C"  then  contributes 
enough  cash  so  that  his  Capital  account  equals  just  one-third  of  the  total 
capital. 

Prepare  statements  showing  how  the  accounts  are  to  be  adjusted  and 
the  balance  sheet  after  the  final  adjustment, 

(A.  I.  of  A.,  June,  1917.) 

12.  "Y"  and  "Z"  are  partners,  drawing  equal  amounts  for  services  and 
sharing  profits  in  accordance  with  capital  investment,  after  allowing  5 
per  cent  on  capital.  In  order  to  acquire  additional  capital  they  agree 
to  admit  "X"  to  the  firm,  who  is  to  own  a  one-fourth  interest  in  the 
business.  According  to  the  balance  sheet,  "Y"  has  $8,000  and  "Z,"  $4,000 
invested.  Goodwill  is  valued  at  $4,000.  What  sum  must  "X"  contribute? 
How  will  partners'  accounts  appear  after  payment  into  the  firm  of  "X's" 
capital,  and  how  will  future  profits  be  divided?  Show  the  accounts  in 
skeleton  form.     (Ind.,  Jan.,  1916.) 

13.  John  Jones,  William  Brown,  and  Alexander  White  are  partners  in 
a  business,  their  respective  interests  in  the  profits  of  the  business  being 
five-tenths,  four-tenths,  and  one-tenth.  It  is  agreed  between  the  partners 
that  Mr.  Jones's  son  be  taken  into  the  business  as  at  January  1,  1916,  on 
the  understanding  tliat  White's  interest  in  the  business  be  increased  to 
12  per  cent,  which  increased  share  is  now  considered  to  be  applicable  to 
the  four  previous  years,  while  the  shares  of  Mr.  Jones  and  Mr.  Brown 
are  to  be  40  per  cent  and  39  per  cent,  respectively,  while  Mr,  Jones's  son 
is  to  be  given  9  per  cent  interest  in  the  profits  of  the  business.  It  is 
further  ajrreed  that  the  value  of  the  goodwill  of  the  business,  amounting  to 


100  C.  p.  A.  ACCOUNTING 

$50,000,  be  set  up  on  the  books.  This  amount  is  to  be  divided  between 
Mr.  Jones  and  Mr.  Brown  in  proportion  to  their  original  interests  in  the 
profits.  Mr.  Jones  is  to  transfer  the  sum  of  $6,000  to  his  son's  credit, 
which  wiir  be  in  addition  to  the  sum  to  be  allowed  him  out  of  profits  of 
previous  years.  The  profits  divided  during  the  four  years  to  December 
31,  1915,  were  as  follows:  1912,  $41,030;  1913,  $49,000;  1914,  $52,000; 
1915,  $48,000;  total,  $190,030. 

The  balances  at  credit  of  the  Capital  accounts  at  December  31,  1915, 
were,  John  Jones,  $230,310 ;  William  Brown,  $185.112 ;  Alexander  White, 
$21,809. 

Prepare  a  detailed  statement  showing  the  balances  at  credit  of  the 
various  partners  on  January  1,  1916,  after  giving  effect  to  the  provisions 
of  the  new  partnership  agreement  as  above  indicated.  Ignore  any  question 
of  interest. 

(111.,  May,  1916.) 


CHAPTER  VI 
PARTNERSHIP  DURING  OPERATION 

Division  of  Profits — In  the  absence  of  a  specific  agreement  to  the  con- 
trary, all  profits  are  shared,  and  all  losses  are  borne,  equally  among  part- 
ners/ The  partners,  however,  may  agree  that  profits  and  losses  be  divided 
according  to  (a)  original  investments,''  (b)  net  investments/  (c)  average 
investments,*  and  (d)  arbitrary  ratios/ 

For  an  ilhistration  of  these  four  methods  of  distributing  profits,  assume 
that  Jones  and  Smith,  whose  proprietorship  accounts  are  given  below, 
made  a  net  profit  of  $6,000  f<n'  the  year  ending  December  31st : 

Jones,  Capital  Account 

I   January  1 $10,000. GO 

Jones,  Personal  Account 

May  1 $1,000.00   i   Oct.! $1,000.00 

June  1 1,000.00   | 

Smith,  Capital  Account 

I   January  1 $5,000.00 

Smith,  Personal  Account 


Feb.  1 $1,000.00 

Mar.  1 1,000.00 

Aug.  1 2,000.00 


Nov.  1 $2,000.00 

Dec.l 3,000.00 


If  the  profits  are  divided  according  to  original  investments,  Jones  would 
receive  two-thirds,  and  Smith  one-third  of  the  net  profits,  or  $4,000  and 
$2,000,  respectively,  as  they  contributed  the  original  capital  of  $15,000 
in  the  ratio  of  2  to  1. 

If  the  profits  are  divided  according  to  net  investments,  Jones  and  Smith 
would  receive  profits  according  to  their  net  investment  at  the  end  of  the 
period,  viz.,  capital  accounts  together  with  personal  accounts.  Jones 
then  would  receive  9/15  of  $6,000,  or  $3,600,  while  Smith  would  receive 
6/15,  or  $2,400. 

If  the  profits  aie  divided  according  to  average  investments,  the  basis 
of  profit-sharing  would  be  the  amount  of  capital  furnished  by  each  partner 
and  the  duration  of  its  use  in  the  business.     There  are  two  methods  of 


'For  explanation  of  superior  figures  see  page  337. 

101 


102 


C.  P.  A.  ACCOUNTING 


determining  this  basis,  which  may  be  calculated  either  on  the  day-dollar 
or  month-dollar  basis. 

The  first  method,  illustrated  below,  in  finding  Jones'  average  invest- 
ment, multiplies  each  investment  and  withdrawal  by  the  number  of  months 
elapsing  between  date  of  transaction  and  date  of  profit  determination,  the 
difference  between  the  month-dollars  for  the  investments  and  the  with- 
drawals showing  the  average  investment  in  terms  of  month-dollars." 

The  second  method,  illustrated  below  in  finding  Smith's  avei-age  invest- 
ment, calculates  the  average  investment  in  terms  of  month-dollars  by 
adding  the  products  obtained  by  multiplying  the  net  investments  by  the 
number  of  months  they  remained  unchanged.' 

Both  methods  of  calculating  average  investment  necessarily  are  equally 
accurate,  but  the  second  method  is  slightly  preferable  in  that  the  "months" 
column  will  always  total  the  same  as  the  length  of  the  fiscal  pei-iod,  if  each 
partner  had  an  investment  at  the  start  of  the  period,*  and  the  last  net 
investment  figure  Avill  always  be  the  balance  of  the  capital  and  personal 
accounts  of  the  partner,'  thus  giving  two  checks  on  the  mathematical 
accuracy  of  the  calculation. 

First  Method    (Jones)  : 


Amount 

Months 

Month 
Dollars 

Month 
Dollars 

Investments: 

Jan.l 

$10,000 
1,000 

1,000 
1,000 

12 

3 

8 
7 

120,000 
3,000 

Oct.  1 

123.000 

Withdrawals : 
May  1 

8,000 
7,000 

June  1 

15,000 

108,000 

Second  Method    (Smith): 


Investment 

Months 

Month 
Dollars 

$5,000 
4,000 
3,000 
1,000 
3,000 
6,000 

1 
1 
5 
3 

1 
1 

5,000 
4,000 
15,000 
3,000 
3,000 
6,000 

12 

36,000 

PARTNERSHIP  DURING  OPERATION  103 

In  distributing  the  profit  of  $6,000  on  the  basis  of  average  investment, 
Jones  receives  108/144,  or  $4,500,  while  Smith  receives  36/144,  or  $1,500. 

Profits  may  be  distributed  without  reference  to  investment."  Indeed, 
in  some  cases  partners  receive  a  share  in  the  profits  without  having  an 
investment  at  all.  This  disparity  between  profit  and  capital  I'atios  is 
usually  due  to  one  man  having  unusual  skill  or  to  the  fact  that  one  partner 
devotes  more  time  to  the  business  than  the  others." 

Interest  on  Capital — Allowing  interest  on  partners'  investments  has  no 
effect  when  the  capital  and  profit  sharing  ratios  are  identical,  but,  where 
these  ratios  are  dissimilar,  it  benefits  the  partner  having  the  greater  capital 
at  the  expense  of  the  partner  having  the  smaller  investment,  and  operates 
to  the  detriment  of  the  partner  whose  profit-sharing  ratio  is  the  greater 
and  benefits  the  partner  with  the  smaller  profit-sharing  ratio." 

The  intei'est  on  investments  is  usually  credited  to  the  partners'  personal 
accounts,"  but  there  is  little  uniformity  as  to  the  debit  entry,  as  sometimes 
the  profit  and  loss  account'*  is  charged  directly,  while  frequently  either  the 
interest  account"  or  the  interest  on  partnership  investments  account"  is 
debited.  The  charge  to  the  interest  account  is  faulty,  because  the  interest 
account  is  commonly  interpreted  as  the  account  reserved  for  interest  pay- 
ments between  the  firm  and  outsiders,  while  interest  on  capital  affects 
only  the  proprietors." 

The  adjustment  between  partners  on  account  of  interest  on  capital  may 
well  be  made  by  crediting  each  partner's  personal  account  with  the  amount 
due  him,  and  then  chai'ging  the  same  account  with  its  prorata  share  of 
the  total  allowance  made  to  all  partnei's  as  interest  on  investments."  For 
instance,  if  A  and  B,  whose  profit  and  loss  ratio  is  equal,  are  allowed  $100 
and  $50,  respectively,  as  interest  on  capital,  the  entry  would  be : 

A,  Personal  Account $75. GO 

B,  Personal  Account 75 .  GO 

A,  Personal  Account $100. 00 

B,  Personal  Account 50. 00 

or,  showing  only  net  results : 

B,  Personal  Account $25. 00 

A,  Personal  Account $25. 00 

The  interest  on  capital  investment  is  sometimes  treated  as  a  financial 
expense,''  and  at  other  times  as  an  allocation  of  profit."  The  latter  treat- 
ment seems  preferable,  unless  the  articles  of  copartnership  require  interest 
on  investment  to  be  treated  as  an  expense. 

Interest  on  capital  must  be  considered,  if  provided  for  in  the  articles 
of  copartnership,  even  though  there  are  not  enough  net  pi'ofits  to  satisfy 
these  requirements."^ 

There  is  no  complexity  in  the  calculation  of  the  amount  of  interest  to 
be  allowed  on  investment,  when  it  is  allowed  on  either  original  investment 
or  net  investment.  However,  the  calculation  is  quite  complicated  when 
interest  is  allowed  on  the  average  investment.     The  two  methods  of  eal- 


104  C.  P.  A.  ACCOUNTING 

culating  the  interest  to  be  allowed  on  average  capital  given  below  use  the 
Jones,  Capital  Account,  and  Jones,  Personal  Account,  shown  on  page  — . 

First  Method: 


Interest  on  Investment: 

$10,000.00  for  12  mo.  @  6%= 
1,000.00  for    3  mo.  @  6%= 

Interest  on  Withdrawals : 

$1,000.00  for    8  mo.  @  6%= 
1,000.00  for    7  mo.  @  6%= 

Interest  to  be  allowed 


$600.00 
15.00 

$40.00 
35.00 

$615.00 


75.00 


$540.00 


Second  Method: 


Investment 

Months 

Dollar 
Months 

$10,000 
9,000 
8,000 
9,000 

4 
1 
4 
3 

40,000 

9,000 

32,000 

27,000 

12 

108,000 

$108,000.00  @  6%  for  one  month =$540.00. 


The  second  method  is  much  simpler  in  the  cases  which  occur  in  actual 
practice,  where  an  uneven  number  of  days  must  be  used  in  the  calculation. 

Interest  on  Excess  or  Deficit  of  Capital — Interest  allowed  to  a  partner 
on  excess  capital  is  really  interest  on  borrowed  money,  while  the  interest 
charged  a  partner  for  a  deficit  of  capital  is  an  income  offsetting  the 
interest  the  firm  probably  has  to  pay  on  money  borrowed  to  operate  the 
business.^^  Therefore,  interest  on  excess  or  deficit  of  capital  is  an  item 
to  be  entered  into  the  profit  and  loss  account  through  the  interest  account." 

A,  B,  and  C,  partners  sharing  profits  and  losses  equally,  agreed  to 
furni.sh  $20,000  each  with  interest  on  an  excess  or  deficit  of  capital  to  be 
figured  at  6%.  If  they  actually  contributed  $25,000,  $20,000,  and  $10,000, 
respectively,  the  entry  for  the  adjustment  between  partners  would  be : 

C,  Personal  Account  (6%  on  $10,000) $600. 00 

A,  Personal  Account  (6%  on  $5,000) $300. 00 

Interest  Account 300 .  00 

Interest  Account 300. 00 

A,  Personal  Account 100. 00 

B,  Personal  Account 100. 00 

C,  Personal  Account 100. 00 


PARTNERSHIP  DURING  OPERATION 


105 


Some  accountants  diiTei*  from  the  above  method  of  treating  interest  on 
excess  or  deficit  of  capital,  considering  it  merely  an  adjustment  between 
proprietors." 

Interest  on  Drawings — Interest  can  not  be  charged  on  drawings  unless 
the  articles  of  copartnership  so  provide."  The  intention  to  charge  such 
interest  can  not  be  implied  even  if  interest  is  allowed  on  investments,  as 
partners'  drawings  should  be  regarded  as  drafts  on  accruing  profit  rather 
than  on  initial  investment.™  Interest  also  can  not  be  charged  if  the  part- 
ners allow  one  partner  to  draw  more  than  the  amount  stipulated  in  the 
agreement,  unless  interest  was  agreed  upon  at  the  time  of  the  overdraft." 
Partners'  drawings  are  charged  directly  against  their  net  worth  accounts 
and  not  against  the  profit  and  loss  account.^* 

Partners'  Salaries — Adjustment  for  variation  in  the  ability  or  activity 
of  partners  may  be  made  by  compensating  the  more  skilled  or  more  active 
partners  by  salaries.^'  Salaries  are  not  allowed  to  partners  unless  the 
articles  of  copartnership  so  provide.^"  Salaries  of  partners  are  some- 
times treated  as  administrative  expense,^  and  sometimes  as  allocations  of 
profit,^^  the  latter  treatment  seeming  pi-eferable.  Partners'  salaries  must 
be  considered,  if  provided  for  in  the  articles  of  copartnership,  even  though 
there  are  not  enough  net  profits  to  satisfy  these  requirements.^ 

Tabular  Form  of  Partners'  Accounts — While  the  capital  accounts  of 
partners  are  in  the  regulation  ''T"  form  in  the  ledger,  it  is  customary  to 
present  these  accounts  in  tabular  form  when  the  detail  of  the  accounts  is 
desired  in  a  schedule  or  report."  This  tabular  form  gives  one  money 
column  for  the  combined  capital  and  personal  accounts  of  each  partner. 
The  following  is  a  simple  example : 


Total 

A 

B 

Original  Investment 

$15,000.00 
5,000.00 
1,500.00 

$10,000.00 
3,000.00 
1,000.00 

$5,000.00 

Salaries 

2,000.00 

Interest  on  Investment 

500.00 

Drawings 

$21,500.00 
7,500.00 

$14,000.00 
4,000.00 

$7,500.00 
3,500.00 

Profits    in    Excess    of    Interest    and 
Salaries 

$14,000.00 
10,000.00 

$10,000.00 
5,000.00 

$4,000.00 
5.000.00 

New  Capital 

$24,000.00 

$15,000.00 

$9,000.00 

106  C.  P.  A.  ACCOUNTING 


QUESTIONS 

PARTNERSHIP    DURING    OPERATION 

1.  Give  a  rule  for  ad.jusiing  partners'  accounts:  (a)  when  the  gains  or 
losses  are  to  be  divided  in  proportion  to  each  partner's  investment  and  the 
time  it  remains  in  use;  (6)  when  the  proportion  of  gain  or  loss  is  fixetl, 
and  interest  is  calculated  on  excess  or  deficit  of  capital.  (N.  Y.,  Jan., 
1901;  W.  Va.,  May,  1917.) 

2.  Ex])lain  a  method  for  apportioning  the  profits  or  losses  of  a  partner- 
ship, where  the  partners  may  make  additional  investments  or  withdrawals 
during  the  year.     (N.  D.,  June,  1914.) 

3.  In  arranging  accounts  for  a  partnership,  how  should  increases  to 
capital  be  treated?     (Fla.,  July,  1909.) 

4.  In  arranging  accounts  for  a  partnei*ship,  how  should  accumulated 
profits  of  each  partner  be  treated?    (Fla.,  July,  1909.) 

5.  In  case  of  a  partnership,  how  would  you  dispose  of  the  balance  of 
the  profit  and  loss  account?  (Mich.,  Dec,  1906*;  N.  Y.,  Oct.,  1907;  R.  I., 
Dec.,  1907.) 

6.  Where  do  a  joint  stock  company  and  a  copartnership  differ  in 
method  of  profit  distribution?     (N.  Y.,  June,  1909.) 

7.  How  is  the  profit  of  a  cooperative  association  divided?  (Mich., 
Dec.,  1906.) 

8.  Upon  the  audit  of  the  partnership  accounts  of  a  manufacturing 
business  the  following  condition  is  revealed :  A  loan  to  the  firm  has 
been  credited  by  mutual  consent  to  the  capital  account  of  one  of  the 
partners.  What  would  you  deduce  from  this  fact,  and  what  would  you 
feel  called  upon  to  do?     (Kan.,  May,  1916.) 

9.  How  should  the  auditor  deal  with  the  ledger  accounts  of  pai'tners 
(a)  in  the  division  of  profits,  (h)  in  the  assessment  of  losses,  (c)  in  case 
a  partner's  drawings  exceed  the  amount  specified  in  the  partnership  agree- 
ment, (d)  in  case  a  partner's  drawings  are  less  than  the  amount  to  which 
he  is  entitled?     (N.  Y.,  June,  1901.)  • 

10.  What  distinction,  if  any,  would  you  make  as  to  salaries  and  draw- 
ings of  the  partners  in  a  partnership  firm,  as  affecting  profit  and  loss? 
(Ind.,  Nov.,  1917.) 

11.  Explain  fully  in  what  way,  if  at  all,  partners'  salaries  should  enter 
into  Trading  and  Profit  and  Loss  statements,  with  reason  for  inclusion  or 
exclusion.  (N.  Y.,  June,  1898*;  N.  Y.,  Jan.,  1911;  Mass.,- June,  1913*; 
Kan.,  Mar,  1916.) 


PARTNERSHIP  DURING  OPERATION  107 

12.  John  Smith  owns  a  painting  and  decorating'  business,  employing 
twenty  men.  He  is  in  active  charge  and  prepares  estimates  and  supervises 
all  the  work.  He  draws  no  salary.  You  are  asked  to  advise  him  relative 
to  the  propriety  of  this  procedure,  and  as  to  whether  he  should  charge 
a  salary  for  himself,  and  how  much,  and  if  it  should  be  regarded  as  a  part 
of  his  overhead  expense  or  as  a  general  administrative  expense.  (111.,  May, 
1917.) 

13.  Explain  fully  in  what  way,  if  at  all,  partners'  drawings  should 
enter  into  Trading  and  Profit  and  Loss  statement,  with  reason  for  inclusion 
or  exclusion.     (N.  Y.,  Jan.,  1911;  Mass.,  June,  1913*;  Kan.,  May,  1916.) 

14.  Classify  the  Interest  on  Invested  Capital  account  properly,  accord- 
ing to  the  subdivision  of  assets,  liabilities,  proprietary  interest,  income  and 
expenses,  under  which  it  should  be  grouped.     (Wis.,  Maj',  1919.) 

15.  "A,"  "B,"  and  "C"  are  equal  partners,  eadi  having  subscribed 
$5,000  to  the  partnership.  "A"  pays  in  $3,000,  leaving  .$2,000  still  due 
the  partnership  on  his  capital  account.  It  is  agreed  for  the  present  that 
this  $2,000  can  remain  unpaid,  provided  "A"  pays  interest  on  the  same, 
which  he  does.  Later  a  dispute  arises  as  to  how  this  interest  shall  be 
credited.  "A"  claims  that  it  should  be  included  with  the  earnings  of  the 
business,  the  profits  of  which  are  to  be  divided  equally  among  the  three 
partners.  "B"  and  "C^'  claim  that  this  interest  should  be  divided  between 
them  only,  as  they  fully  lived  up  to  their  obligations  under  the  partnership 
agreement,  while  "A"  had  only  partially  done  so.  To  what  account  should 
the  interest  on  the  deferred  payment  be  credited?     (N.  Y.,  June,  1913.) 

16.  In  examining  the  partnership  accounts  of  Black  and  Brown  you 
ascertain  that  the  capital  of  $20,000  has  been  provided  equally,  and  the 
articles  of  partnership  provide  that  if  any  excess  capital  is  supplied 
either  partner,  interest  at  the  rate  of  5  per  cent  per  annum  shall  be 
allowed.  Black  pays  $5,000  additional  and  is  credited  at  the  end  of  the 
year  5  per  cent  on  same  which  equals  .$250,  which  is  debited  to  Brown. 
State  whether  you  consider  this  correct  and  give  reasons  for  your  answer. 
(N.  Y.,  June,  1898*;  111.,  Nov.,  1908;  Mich. ,'^  Dec,  1913.*) 

17.  What  method  should  be  pursued  in  adjusting  interest  on  capital 
among  partners  whose  investments  differ  in  amount?  Give  reasons  for 
such  book  entries  as  you  would  recommend  in  the  premises.  (N.  Y.,  Jan., 
1904.) 

18.  Under  what  conditions  may  interest  on  partners'  capital  invested 
in  a  firm  be  charged  and  cretlited  to  the  partners'  accounts  in  the  absence 
of  an  agreement  to  that  effect  under  the  terms  of  a  partnership?  (Mo., 
Dec.,  1914.) 


108  C.  P.  A.  ACCOUNTING 


PROBLEMS 

PARTNERSHIP   DURIXG   OPERATION 

1.  "A,"  "B,"  and  "C"  are  partners.  "A"  is  to  receive  a  salary  of 
$2,000  per  annum;  "B"  $2,500  and  "C"  $3,000.  The  balance  of  profits 
after  payment  of  salaries  is  to  be  divided  as  to  the  first  $20,000,  2/3  to 
"A,"  and  1/6  each  to  "B"  and  "C" ;  and  profits  above  $20,000  are  to  be 
divided  equally  among  the  three.  "A"  retires  from  active  business,  and 
gives  up  his  right  to  salary  for  1906.  The  profits  for  the  year,  before 
charging  salaries,  amount  to  $35,000.  To  what  extent  are  "A,"  "B,"  and 
"C,"  respectively,  affected  by  "A's"  concession?     (Fla.,  April,  1907.) 

2.  Jones  and  Johnson  form  a  copartnership,  January  1,  1909,  each  in- 
vesting $10,000 ;  April  1,  Jones  pays  in  an  additional  $2,500,  and  Johnson 
draws  out  $1,500.  August  1,  Johnson  pays  in  $3,000  and  Jones  withdraws 
$1,000.    The  profits  for  the  year  ending  December  31,  1909,  are  $5,000. 

Prepare  statements  showing  each  partner's  investment  and  portion  of 
profits,  the  profits  being  divided  in  proportion  to  capital  invested  and  the 
time  it  is  employed. 

(Ind.,  June,  1916.) 

3.  "A"  and  "B"  form  a  partnership,  "A"  investing  $30,000  and  "B," 
$50,000.  They  agree  to  share  expenses,  profits  and  losses  equally.  They 
further  agree  to  and  do  leave  their  original  investments  intact.  At  the 
end  of  the  first  year,  the  pi-ofits  from  the  operations  of  the  business  amount 
to  $30,000,  against  which  "A"  has  drawn  in  twelve  equal  monthly  install- 
ments on  the  last  day  of  each  month  an  aggregate  amount  of  $9,000.  "B" 
has  drawn  against  his  profits  on  the  last  day  of  each  quarter  the  sum  of 
$2,500. 

Prepare  Journal  entries  adjusting  interest  at  5  per  cent  per  annum 
between  the  partners  in  respect  to  both  their  investment  and  drawing  ac- 
counts and  render  statements  showing  the  amount  each  partner  has  in  the 
business  at  the  end  of  the  year. 

(Mass.,  June,  1912.) 

4.  The  capital  of  three  partners,  "A,"  "B,"  and  "C,"  in  a  manufactur- 
ing business  January  1,  1896,  was  $26,000,  of  which  "A"  owned  1/5,  "B" 
2/5,  "C"  2/5.  On  December  31,  1896,  one  year  thereafter,  the  condition 
was  found  to  be  as  follows:  Real  Estate,  $15,000;  Plant  and  Machinery, 
$7,000;  Stock  on  hand,  $2,000;  Book  Debts  Receivable,  $6,000;  Cash  in 
Bank,  $2,500 ;  Creditors'  Notes  Payable,  $8,000 ;  Partners'  Withdrawals— 
"A"  (including  interest),  $1,500;  Partners'  Withdrawals— "B"  (including 
interest),  $1,200;  Partners'  Withdrawals— "C"  (including  interest),  $2,000. 


PARTNERSHIP  DURING  OPERATION  109 

After  crediting  up  interest  on  Capital  at  the  rate  of  6  per  cent,  show 
the  net  result  for  the  year,  and  distribute  the  same,  in  proper  proportions, 
to  the  partners'  accounts. 

Prepare  individual  partners'  accounts,  showing  the  condition  of  each  at 
the  end  of  the  year. 

(Mass.,  June,  1910.) 

5.  "A"  and  "B,"  partners,  finding  themselves  in  want  of  further  capi- 
tal in  their  business,  and  both  being  possessed  of  real  property,  "A"  de- 
posited deed  with  the  bankers  of  the  firm  as  security  for  a  loan  of  $2,000 
to  the  firm.  "B"  arranged  on  some  of  his  own  property  a  mortgage  for 
$1,500  with  a  private  friend  and  paid  the  proceeds  into  the  firm's  bank 
account.  The  bankers  were  eventually  obliged  to  realize  the  security  held 
by  them  which  produced,  after  payment  of  all  expenses,  the  sum  of 
$2,850. 

Prepare  entries  recording  these  transactions  in  the  firm's  books. 

(Mass.,  Oct.,  1916.) 

6.  "A,"  "B,"  and  "C"  agree  to  start  in  business  with  a  capital  of 
$200,000,  of  which  "A"  is  to^furnish  $100,000  and  "B"  and  "C,"  $50,000 
each.  "A"  is  to  have  one-half  interest  in  the  business  and  "B"  and  "C," 
each  one-quarter.  Interest  at  5  per  cent  is  to  be  credited  on  excess,  or 
charged  on  deficiency  of  capital.  "A"  contributes  $100,000;  "B,"  $45,000; 
and  "C,"  $40,000.  How  would  the  capital  accounts  stand  on  the  books 
after  adjusting  the  interest  at  the  end  of  the  year?     (N.  Y.,  Dec,  1898.) 

7.  "A"  offers  to  take  "B"  into  partnership  on  equal  terms,  upon  pay- 
ment by  "B"  of  a  premium  of  $12,500  but  as  "B"  is  unable  to  pay  the 
money  for  three  years,  the  following  arrangement  is  agreed  upon.  The 
profits  are  to  be  divided  in  the  proportions  of  two-thirds  to  "A"  and  one- 
third  to  "B."  "B"  will  draw  one-third  of  his  share  of  the  profits,  leave 
one-third  in  the  business,  and  hand  over  the  remainder  to  "A"  in  part 
payment  of  the  premium  as  above. 

The  profits  for  the  first  three  years  are  as  follows :  First  year,  $8,000 ; 
second  year,  $10,000;  third  year,  $11,000. 

Draw  up  the  partnership  accounts  for  these  three  years,  and  show  the 
amount  due  from  "B"  to  "A"  at  the  end  of  the  period. 

(Wash.,  May,  1910.) 

8.  "A"  having  a  capital  of  $10,000,  took  "B"  into  partnership  on 
condition  of  his  bringing  in  $5,000,  In  ascertaining  profits  each  year, 
"A"  was  to  have  $3,000  salary  and  "B"  $1,500  and  5  per  cent  interest 
upon  capital  was  to  be  allowed  to  each  partner,  but  no  interest  charge  was 
to  be  made  on  withdrawals.  The  profits  thereafter  were  to  be  divided: 
Up  to  $9,000,  two  thirds  to  "A"  and  one  third  to  "B,"  any  excess  equally. 

No  limitations  as  to  withdrawals  in  anticipation  of  profits  were  shown 
in  the  deed  of  partnership  and  at  the  end  of  the  first  year  "A"  had  drawn 
$3,000,  and  "B"  $750  in  excess  of  their  salaries.  The  profit  for  the  year, 
before  making  the  above  charges,  amounted  to  $17,500. 

Complete  the  Profit  and  Loss  account  and  show  the  partners'  accounts 
as  they  should  appear.  (Wash.,  Sept.,  1907.) 


110  C.  p.  A.  ACCOUNTING 

9.  "X"  and  "Y"  enter  into  partnership.  "X's"  capital  being  $20,000 
and  "Y's"  $15,000.  Capital  is  to  bear  interest  at  10  per  cent  per  annum ; 
profitB  are  to  be  divided  equally  between  the  parties.  The  profits  for  the 
first  two  years  (after  chargins:  interest  on  eapi(al)  were:  First  year, 
$6,000;  second  year,  $7,500.  The  drawings  of  the  partners  (in  excess 
of  salaries)  were:  First  vear,  "X."  $1,500;  "Y,"  $1,200;  and  second 
year,  "X,"  $1,750;  "Y,"  $1,500. 

At  the  end  of  the  second  year  "Z"  was  admitted  to  partnership,  and  put 
into  the  business  the  same  amount  of  capital  as  "Y"  had  in  the  business 
at  that  time,  and  on  the  same  conditions  as  to  interest  and  division  of 
profits.  The  profits  of  the  business  for  the  third  year  were  $12,000,  and 
the  partners'  drawings  in  excef.s  of  salaries  were:  "X,"  $1,750;  "Y," 
$1,600;  "Z."  $1,500. 

Construct  the  Capital  accounts  of  the  partners  for  each  of  the  three 
years,  showing  the  balance  on  each  at  the  end  of  the  third  year. 

(Wash.,  April,  1906.) 

10.  "A,"  "B,"  and  "C"  formed  a  partnership.  "A"  agreed  to  furnish 
$10,000,  "B"  and  "C"  each  $7,000.  "A"  was  to  manage  the  business  and 
receive  one  half  of  the  profits;  "B"  and  "C"  were  each  to  receive  one 
fourth.  "A"  supplied  merchandise  worth  $8,500,  but  no  additional  cash. 
"B"  turned  over  to  "A,"  as  managing  partner,  $9,000  cash  and  "C"  turned 
over  $5,500.  The  business  was  conducted  by  "A"  for  some  time,  but  with- 
out keeping  exact  books.  While  managing  the  business  "A"  purchased 
additional  merchandise  amounting  altogether  to  $75,000  and  made  sales  of 
$100,000.  The  cash  received  and  paid  out  for  the  partnership  was  not 
kept  separate  from  "A's"  personal  cash.  In  order  to  straighten  out  mat- 
ters, "B"  took  over  the  management.  lie  found  receivables  amounting  to 
$20,000  and  of  these  he  collected  $4,500.  The  merchandise  still  on  hand 
he  sold  for  $500.  These  receipts  he  deposited  in  a  bank  to  the  credit  of 
the  firm.  The  remaining  accounts  proved  worthless.  The  outstanding 
accounts  payable  amounted  to  $2,000  of  which  $1,500  bad  been  incurred 
in  purchasing  merchandise  and  $500  for  expenses.  These  accounts  he 
paid.  "A"  presented  vouchers  showing  that  during  his  management  he 
had  paid  other  expenses  of  $2,400.  By  mutual  agi-eeraent  "P."'  was  held  to 
be  entitled  to  $100  on  account  of  interest  on  excess  capital  contributed  and 
"A"  and  "C"  were  to  be  charged  $75  each  for  shortage  in  contribution  of 
capital. 

(a)  Prepare  Trading  and  Profit  and  Loss  accounts  and  accounts  of 
each  of  the  partners,  indicating  the  final  adjustment  to  be  made  in  closing 
up  the  partnership. 

(b)  ShoAv  how  the  above  final  adjustment  would  be  modified  if  "A" 
proved  to  have  no  as.sets  or  liabilities  outside  the  partnership. 

(A.  I.  of  A.,  June,  1917;  Ind.,  Nov.,  1917.*) 

11.  John  Jones  and  Samuel  Smith  go  into  the  hardware  business 
January  1,  1910,  investing  as  follows:  Jones,  $23,874;  Smith,  $19,228.50; 
total  $43,102.50. 

Within  the  year  Jones  draws  out  for  private  use  over  the  additional  in- 


PARTNERsnrp  DuruNa  operation  111 

vestment  made  by  him  $7,863;  Smith  draws  out  over  investments 
$10,057.30.  A  year  later  Jones  makes  additional  investments  from  private 
sources,  and  over  and  above  the  amount  drawn  out  for  private  use, 
$11,269.70.  Smith  also  invests  over  amounts  drawn  out  for  his  private 
use  $4,732.60.  At  the  end  of  the  year  lOU  they  make  the  following  state- 
ment of  assets  and  liabilities: 


Assets 

Merchandise   

$27,860.00 
10,246.00 
12,354.30 
16,452.00 
10,000.00 
5,324.70 

Liabilities 

Bills  Payable 

Personal  Accounts 

Firm  owes  John  Jones . 

$20,480.00 

16,553.40 

3,406.70 

Cash  on  Hand 

Personal  Accounts  Due  . 

Bills  Receivable   

Real  Estate 

Samuel  Smith  owes  firm. 

$82,237.00 

$40,440.10 

By  agreement  they  are  to  divide  the  plains  or  losses  pro  rata  according 
to  investment  at  the  commencement  of  business,  the  partner  of  the  small 
investment  not  paying  to  the  other  partner  any  interest  upon  the  differ- 
ence of  capital. 

Show  the  Drawing  account  and  Investment  account  of  each  and  the  net 
investment  or  worth  of  each  partner  and  also  of  the  firm  at  December  31, 
1911. 

(Wash.,  June,  1912.) 

12.  H.  Pratt,  F.  Jones  and  J.  Todd  entered  into  partnership  on  July  1, 
1914.  Pratt  brought  in  as  capital  $15,000;  Jones,  $10,000;  and  Todd, 
$5,000.  They  were  to  share  profits  in  the  proportion  of  one-half,  one- 
third,  and  one-sixth,  but  as  Jones  and  Todd  were  the  working  partners, 
they  were  to  be  credited  at  the  close  of  each  current  year,  by  way  of  salary, 
with  the  respective  sums  of  $1,250  and  $750.  Pratt  was  to  be  allowed  to 
draw  each  year,  as  against  profits,  $2,500;  Jones,  $1,650;  and  Todd, 
$1,250;  interest  at  6  per  cent  was  to  be  charged  on  such  drawings.  The 
partnership  agreement  also  provided  that  Jones  and  Todd  should  have  the 
right  to  bring  in  extra  capital  not  exceeding  $8,000  each,  and  that  upon 
such  capital  they  were  to  be  credited  with  6  per  cent  interest.  Upon 
closing  the  books  on  June  30,  1915,  it  was  found  that  the  partners  had 
drawn  as  follows: 


Pratt 

Jones 

Todd 

Sept.  1 . 
Nov.  1 . 
Dec.  1  . 

....    $500.00 
...       760.00 
...    1,000.00 

Aug.  1 $400.00 

Sept.  1 350.00 

Oct.  1 500.00 

Dec.  1 400.00 

Aug.  I . 
Sept.  1. 
Nov.  1. 
Jan. 1. . 

$300.00 
250.00 
400.00 
100.00 

On  October  1,  Jones  brought  into  the  business  as  additional  capital  the 
sum  of  $1,200  and  Todd  $2,000.     On  closing  the  books  at  June  30,  1915, 


112  C.  P.  A.  ACCOUNTING 

and  before  the  salary  or  interest  to  partners  had  been  dealt  with,  the 
balance  to  the  credit  of  Profit  and  Loss  stood  at  the  sum  of  $13,000.  Make 
the  closing  entry  and  prepare  Capital  and  Drawing  accounts  showing  the 
exact  position  of  the  partners  on  July  1,  1915. 

(Wash.,  May,  1916.) 

13.  In  making  an  audit  of  the  books  of  the  partnership  of  "A"  and 
"B"  you  find  that  the  agreed  division  of  profits  was  to  be  on  the  basis  of 
the  capitals  and  of  the  time  that  they  were  left  in  the  business. 

The  books  show  as  follows:  "A's"  account  paid  in  January  1,  $6,000; 
March  1,  $2,000;  June  1,  $4,000;  November  1,  $1,000;  withdrew  April  1, 
$3,000;  October  1,  $2,000. 

"BV  account,  paid  in  January  1,  $4,000;  February  1,  $1,000;  August 
1,  $3,000;  withdrew  May  1,  $2,000;  December  1,  $1,000.  Prepare  a  state- 
ment showing  method  of  arriving  at  correct  profit  distribution. 

(Ind.,  Nov.,  1918.) 


CHAPTER  VII 

PARTNERSHIP  AT  LIQUIDATION 

Causes  of  Dissolution — Partnerships  may  be  dissolved  by  (a)  the  with- 
drawal of  any  partner,  (b)  sale  of  a  partner's  interest  or  admission  of  a 
new  partner,  (c)  lapsing  of  time  limitations  and  completion  of  object 
specified  in  partnership  agreement,  (d)  mutual  consent  of  the  partners, 
(e)  misconduct,  insanity,  death,  disability,  assignment,  or  bankruptcy  of 
a  partner,  (f)  illegality  of  object,  (g)  war  between  nations  represented 
by  partners,  (h)  bankruptcy  of  the  firm,  and  (i)  sale  and  transfer  of  all 
property  of  firm/  As  a  safeguard,  it  is  important  that  all  special  reasons 
which  will  dissolve  the  firm,  be  mentioned  specifically  in  the  articles  of 
copartnership." 

Adjustment  Upon  Partner's  Death — To  avoid  the  necessity  of  taking 
an  inventory  and  an  appraisal  of  all  the  firm's  assets,  the  articles  of 
copartnership  may  provide  for  the  continuation  of  the  business  and  the 
purchase  of  the  interest  of  the  deceased  partner  by  the  remaining  partner/ 
In  such  cases,  the  deceased  partner's  share  in  the  current  period's  profits 
is  usually  the  proration  of  the  profits  over  the  part  of  the  period  prior 
to  his  death/  The  deceased  partner's  estate  is  usually  paid  an  allowance 
for  the  deceased  partner's  share  in  the  goodwill,^  and  usually  allowed 
interest  from  the  date  of  death  until  settlement  is  made/ 

Liquidating  Partners — All  partners  have  equal  rights  to  share  in  the 
work  of  liquidating  the  firm,  but,  as  there  is  seldom  sufficient  work  for 
all,  it  is  customary  to  appoint  one  partner  (or  an  outside  party)  as 
liquidator/  After  the  fact  of  the  dissolution  and  name  of  liquidator  have 
been  announced  through  the  local  newspapers,  the  liquidator  proceeds  to 
sell  the  assets,  fulfill  the  existing  contracts  and  complete  the  partly  manu- 
factured goods/  If  necessary,  he  may  purchase  such  materials  as  are 
needed  to  realize  upon  the  assets/  The  salary  or  commission  of  the 
liquidating  partner  may  be  paid  privately  by  the  other  partners"  or  by 
the  representatives  of  a  deceased  partner,"  but  it  is  preferable,  for  record 
purposes,  to  have  the  remuneration  charged  to  the  firm's  liquidation  ex- 
penses/" 

Value  of  Assets  at  Dissolution — The  usual  valuation  rules  used  for  going 
concerns  should  not  be  followed  when  assets  are  appraised  upon  the 
dissolution  of  a  partnership/^  For  instance,  if  a  machine,  whose  estimated 
life  is  ten  years,  cost  $1,000  three  years  ago,  it  would  be  carried  on  the 
books  at  $1,000  minus  $300  (depreciation)  or  $700,  in  spite  of  the  fact 
that  the  selling  price  may  have  doubled.     The  machine  would,  however,  be 


'For  explanation  of  superior  figures  see  page  337. 

113 


114  C.  P.  A.  ACCOUNTING 

valued  at  its  repi-oduetion  value  less  aocnmulated  depreciation  on  the  date 
of  dissolution,  say  .$1,500,  in  calculating  the  retiiing  partner's  interest." 

Liquidation  of  Uncompleted  Contractu — The  uncompleted  contracts  at 
the  dissolution  of  a  jjartnership  would  be  valued  at  market  price,  which 
would  approximate  the  cost  of  the  work  done  plus  that  proportion  of 
the  estimated  profit  on  the  completed  contract  that  the  cost  of  the  work 
done  is  to  the  estimated  cost  of  the  completed  contract. 

If  the  partners  do  not  wish  to  estimate  the  profits  on  the  contracts,  the 
books  of  the  firm  may  be  held  open  until  the  uncompleted  contracts  are 
finished  by  one  of  the  partners  or  by  an  outside  fiim  acting  as  the  agent 
of  the  partners.  After  the  completion  of  the  contracts,  the  assets  would 
be  realized  upon,  the  liabilities  liquidated,  and  the  profit  on  the  contracts 
distributed  to  the  old  partners." 

Reserves  at  Dissolution — In  eases  where  the  interest  of  a  retiring  partner 
is  purchased  by  the  continuing  partners,  the  valuation  reserves  are  closed 
into  the  related  assets  which  are  then  raised  or  lowered  to  their  appraised 
value.  Contingency,  sinking  fund,  and  other  surplus  reserves  are,  how- 
ever, part  of  the  net  worth  of  the  business  and  directly  affect  the  amount 
to  be  allowed  for  the  interest  of  the  retiring  partner. 

Application  of  Assets  at  Dissolution — The  proceeds  from  the  assets  are 
applied  in  the  following  order:  first,  against  outside  liabilities;  second, 
against  partners'  loans  and  advances;  third,  against  partners'  capital  ac- 
counts; and  fourth,  in  the  distribution  as  profits  of  the  residue  to  the 
partners  on  the  profit-and-loss  ratio."  If,  however,  there  is  a  loss,  this 
must  be  distributed  in  the  profit-and-loss  ratio  before  the  capital  con- 
tributions may  be  withdrawn."  If  losses  have  been  so  great  as  to  wipe 
out  the  capital  account  of  a  partner  who  has  loaned  money  to  the  firm, 
a  sufficient  amount  should  be  transfei-red  from  his  loan  account  to  his 
capital  account  to  cancel  the  debit  balance  in  the  latter  account,  before 
pajTnent  is  made  on  the  loan  account." 

Losses  and  Expenses  of  Liquidation — There  is  considerable  dispute 
among  accountants  as  to  the  distribution  of  liquidation  losses  and  gains, 
.some  prorating  them  according  to  capital  investments,"  others,  on  the 
ordinary  profit-and-loss  ratio.""  As  the  partnership  is  not  dissolved  until 
the  capital  investments  are  returned,  the  latter  view  is  the  correct  one. 

Purchasing  Partner's  Interest — When  a  partner  agrees  to  sell  his  interest 
in  the  business,  the  liooks  are  closed  and  the  profit  or  loss  up  to  date 
of  sale  is  distributed  to  the  partners'  personal  accounts.  The  retiring 
partner's  personal  account  is  then  closed  into  his  capital  account,  which 
is,  in  turn,  closed  by  a  debit  offsetting  the  cash,  notes,  or  other  property 
given  the  retiring  partner  for  his  interest.'*  These  entries  take  the  fol- 
lowing form : 

Jones,  Capital  Account xxxxxxx 

Jones,  Personal  Account xxxxxxx 

Jones,  Capital  Account xxxxxxx 

Cash xxxxxxx 

Notes  Pavable xxxxxxx 


PARTNERSHIP  AT  LIQIIDATION  115 

If  the  retiring  partner  sells  his  interest  for  less  than  the  book  value,  the 
ditference  is  credited  to  the  continuing  partner's  personal  account."  The 
transaction  would  be  journalized  thus : 

Jones,  Capital  Account xxxxxxx 

Cash xxxxxxx 

Smith,  Personal  Account xxxxxxx 

However,  if  the  retiring  partner  receives  more  than  the  book  value  of 
his  interest,  the  difiference  may  be  debited  either  to  the  continuing  partner's 
personal  account  or  to  the  goodwill  account."  The  fii-st  method  is  jour- 
nalized thus : 

Jones,  Capital  Account xxxxxxx 

Smith,  Personal  Account xxxxxxx 

Cash xxxxxxx 

The  second  method  is  recorded  by  the  following  entry : 

Jones,  Capital  Account xxxxxxx 

Goodwill xxxxxxx 

Cash xxxxxxx 

Personal  Insolvency  of  One  Partner — Any  loss  on  a  debt  owed  to  the 
firm  by  an  insolvent  partner,  whether  occasioned  by  withdrawings  of 
capital,  operating  losses,  or  liquidating  losses,  should  be  borne  by  the 
other  partners  according  to  the  ordinary  profit-and-loss  ratio.^* 

Illustrative  Problem — To  illustrate  the  effect  of  dissolution  of  a  part- 
nership upon  the  accounts  of  the  firm,  assume  that  the  following  balance 
sheet  shows  the  condition  after  the  realization  of  the  assets.  The  profit- 
and-loss  ratio  of  Jones,  Smith,  and  Johnson  is  3:2:1,  respectively;  Jones 
is  bankrupt  and  wull  not  be  able  to  pay  anything  on  his  debts.  The 
pi'oblem  is  to  close  the  books. 


BALANCE  SHEET  OF  JONES,  SMITH,  AND  JOHNSON 


Cash .  . 

Liquidation  Loss. 


$8,500.00 
12,000.00 


$20,500.00 


Accounts  Payable 

Jones,  Loan  Account .  . 
Smith,  Loan  Account.  . 
Jones,  Capital  Account. 
Smith,  Capital  Account 
Johnson,  Capital  Account 


$3,000.00 
2,000.00 
1,500.00 
1,000.00 
5,000.00 
8,000.00 

$20,500.00 


The  order  and  character  of  the  journal  entries  required  to  close  the 
partnership  books  are  as  follows : 

(a)  Prorating  loss  on  liquidation: 

Jones,  Capital  Account  (3/6) $6,000.00 

Smith,  Capital  Account  (2/6) 4,000.00 

Johnson,  Capital  Account  (1/6) 2 ,  000 .  00 

Liquidation  Loss $12,000.00 


JKJ  C.   P.   A.    ACCUL.NTINU 

(b)  Transferring  Jones'  Loan  to  Capital: 

Jones,  Loan  Account $2,000. 00 

Jones,  Capital  Account $2,000.  00 

(c)  Prorating  lo?s  on  Jones: 

Smith,  Capital  Account  (2/3) $2,000.00 

Johnson,  Capital  Account  (1/3) 1,000.00 

Jones,  Capital  Account $3 ,000. 00 

(d)  Transferring  Part  Smith's  Loan  to  Capital: 
Smith,  Loan  Account $1,000.00 

Smith,  Capital  Account $1 ,000. 00 

(e)  Payment  of  Accounts  Payable: 

Accounts  Payable $3,000. 00 

Cash $3,000.00 

(f)  Payment  of  Smith's  Loan: 

Smith,  Loan  Account $500 .  00 

Cash $500.00 

(g)  Payment  of  Johnson's  Capital: 

Johnson,  Capital  Account $5,000.00 

Cash $5,000.00 

Capital  Deficit — As  general  partners  are  liable  to  the  extent  of  their 
personal  fortunes  for  the  debts  of  the  fii'm,  any  deficits  in  the  partners' 
capital  accounts  created  by  the  distribution  of  operating  and  liquidating 
losses  must  be  canceled  by  additional  contributions."  For  instance,  if 
Jones  and  Srnith,  who  shared  profits  and  losses  equally  and  whose  capital 
contributions  were  $10,000  and  $2,000,  respectively,  had  only  $3,000  in 
cash  after  realizing  on  the  assets  and  licjuidating  the  liabilities,  then  Jones 
would  receive  all  the  casli  the  firm  had,  together  with  the  $2,500  which 
Smith  would  pay  into  tlie  firm.     The  journal  entries  would  be : 

Jones,  Capital  Account $4,500.00 

Smith,  Capital  Account 4,500.00 

Profit  and  Loss $9,000.00 

Cash 2,500.00 

Smith,  Capital  Account 2,500.00 

Jones,  Capital  Account 5,500.00 

Cash 5.500.00 

Goodwill  in  Liquidation — When  a  Inisiness  is  sold  as  a  unit  for  more 
than  its  book  value,  this  excess  is  goodwill  which  may  be  recorded  in  the 
books  by  debiting  the  goodwill  account  and  dividing  the  credit  among  the 
partnei's'  capital  accounts  on  the  profit-and-loss  ratio.''*  Goodwill  would 
then  be  closed  with  the  other  assets  when  the  sale  is  recorded."  If  the 
sale  is  made  at  decease  of  one  partner,  his  estate  is  entitled  to  its  share 
of  the  goodwill.'"' 


PARTNERSHIP  AT  LIQUIDATION 


117 


When  goodwill  is  allowed  to  a  retiring  partner,  the  value  for  the 
goodwill  for  the  firm  must  be  calculated,  but  only  the  goodwill  allowed  to 
the  retiring  partner  may  be  placed  on  the  books  because  the  share  of  the 
goodwill  belonging  to  the  continuing  partners  has  not  been  purchased.^ 

Closing  Partnership  Books  After  Sale — If  a  partnership  sells  out,  the 
old  books  must  be  closed.  This,  of  course,  can  be  done  by  simply  debiting 
the  liability  and  capital  accounts  and  crediting  the  assets.  However,  it 
is  preferable  to  show  the  closing  by  steps  as  follows  :"^° 


Entry  No.  1 

Jones-Smith  Company,  Vendee xxxxxxx 

Assets  (itemized) 

Entry  No.  2 

Liabilities xxxxxxx 

Jones-Smith  Company,  Vendee 

Entry  No.  3 

Cash xxxxxxx 

Jones-Smith  Company,  Vendee 

Entry  No.  4 

Jones,  Capital  Account xxxxxxx 

Smith,  Capital  Account xxxxxxx 

Cash 


xxxxxxx 


If  stock  is  received  instead  of  cash,  the  first  and  second  entries  would 
be  as  above,  but  the  third  and  fourth  entries  would  be  as  follows :'' 

Jones-Smith  Company  Stock xxxxxxx 

Jones-Smith  Company,  Vendee xxxxxxx 

Jones,  Capital  Account xxxxxxx 

Smith,  Capital  Account xxxxxxx 

Jones-Smith  Company  Stock xxxxxxx 


Liquidation  by  Installments — As  the  liquidator  is  personally  responsible 
if  he  overpays  a  partner,^^  it  is  important  when  partnerships  are  liquidated 
by  installments  to  have  the  capital  investments  reduced  to  the  profit-and- 
loss  ratio  as  soon  as  possible."^  The  rule  for  finding  the  amount  that 
should  be  given  to  each  partner  is  to  consider  that  all  the  property  besides 
the  dividend  is  worthless,  and  that  any  partner,  who  may  owe  the  partner- 
ship when  this  loss  is  prorated,  is  bankrupt."*  After  these  assumed  losses 
are  prorated,  the  i-emaining  capital  represents  the  correct  distribution  of 
the  dividend." 

As  an  example,  a.ssume  that  the  capital  accounts  of  Jones,  Smith,  and 
Johnson,  after  prorating  the  operating  losses,  are  $20,000,  $10,000,  and 
$5,000,  res])ectively.  If  the  partners  shai'e  profits  and  losses  equally, 
liquidating  dividends  of  $14,000,  $9,000,  and  $9,000  would  be  prorated 
in  the  manner  shown  on  page  118. 


118 


C.  P.  A.  ACCOUNTING 


Summary 


Total 

Jones 

Smith 

Johnson 

1st  Capital 

$.35,000.00 
14,000.00 

$20,000.00 

$10,000.00 

$-5,000.00 

1st  Dividend 

7,000.00 

7,000.00 

Assumed  Loss 

$21,000.00 

7,000.00 

1st  Dividend — Unadjusted . 
Adjustment 

$14,000.00 

$1.3,000.00 
1,000.00 

$3,000.00 
1,000.00 

$2,000.00 
2,000.00 

1st  Dividend — Adjusted.  .  . 

$14,000.00 

$12,000.00 

$2,000.00 

nil 

2nd  Capital 

$21,000.00 
9,000.00 

$8,000.00 

$8,000.00 

$5,000.00 

2nd  Dividend 

4,000.00 

4,000.00 

Assumed  Loss 

$12,000.00 

4,000.00 

2nd  Dividend — Adjusted. .  . 

$9,000.00 

$4,000.00 

$4,000.00 

$1,000.00 

3rd  Capital 

$12,000.00 
9,000.00 

$4,000.00 

$4,000.00 

$4,000.00 

3rd  Dividend 

1,000.00 

1,000.00 

Assumed  Loss 

$3,000.00 

1,000.00 

3rd  Dividend — Adjusted .  . . 

$9,000.00 

$3,000.00 

$3,000.00 

$3,000.00 

Dividend 

Jones 

Smith 

Johnson 

1st 

$12,000.00 
4,000.00 
3,000.00 

$2,000.00 
4,000.00 
3.000.00 

nil 

2nd 

$1,000.00 

3rd 

3  000.00 

As  the  capital  accounts  in  the  above  problem  are  in  the  pvofit-aud-loss 
ratio,  after  the  second  dividend,  the  actual  loss  of  $3,000  could  be  directly 
prorated  on  the  profit-and-loss  ratio,  leaving  the  capital  accounts  totaling 
the  amount  of  the  last  dividend. 


PARTNERSHIP  AT  LIQUIDATION  119 


QUESTIONS 

PAltTXERSHlP    AT   LIQUIDATION 

1.  As  the  bookkeeper  of  a  firm  that  had  no  articles  of  copartnership, 
what  would  be  j'onr  duty  on  learning:  of  the  death  of  a  partner?  (N.  Y., 
June,  1898.) 

2.  Describe  two  or  more  bases  of  valuation  of  a  merchandising  business, 
in  contemplation  of  purchase  or  sale  thereof.  Illusti'ate  a  basis  of  valua- 
tion of  a  professional  business  in  contemplation  of  admission  of  a  third 
l)artiier  and  sIioav  how  the  capital  accounts  and  profits  would  work  out. 
(III.,  Dec.,  1910.) 

3.  In  arranging-  accounts  for  a  partnership,  how  should  Losses  on 
Dissolution  of  Partnership  be  treated?     (Fla.,  July,  1909.) 

4.  How  may  a  partnership  terminate?  Name  the  different  ways. 
(Iowa,  Dec.  1918.) 

5.  A  firm  of  three  partners  divided  their  profits  as  follows:  "A," 
11/25;  "B,"  8/25;  "C,"  6/25.  By  the  partnership  agreement  it  was  pro- 
vided that  in  the  event  of  the  death  of  either,  the  survivors  should  take 
the  deceased's  share  in  the  proportion  they  already  shared  the  profits, 
"A"  dies.  What  proportion  of  the  profits  would  "B"  and  "C"  respectively 
take  afterwards?     (111.,  May,  1910.) 

6.  In  dissolving  a  solvent  partnership  in  what  order  should  the  pro- 
ceeds of  the  assets  be  distributed?     (N.  D.,  July,  1916.) 

7.  What  is  the  status  of  a  partner's  loan  in  the  liquidation  proceedings? 
(N.  D.,  July,  1916.) 


120 


C.  P.  A.  ACCOUISTl^G 


PROBLEMS 


PARTNERSHIP    AT   LIQUIDATIOX 


1.  The  capital  of  a  partnership  is  contributed  as  follows:  "A," 
$90,000;  "B,"  $45,000;  "C,"  $15,000. 

The  partnership  agreement  provides  for  profit-sharing  as  follows:  "A," 
50  per  cent;  "B,"  30  per  cent;  "C,"  20  per  cent. 

The  partnei-s'  salaries  are  as  follows:  "A,"  $5,000;  "B,"  $3,000;  "C," 
$2,000. 

At  the  close  of  the  first  year's  business,  "C"  dies.  The  books  are  closed 
and  the  net  assets  of  the  business  are  shown  to  be  $152,500.  "A"  and 
"B"  liquidate  the  affairs  of  the  partnership  and  distribute  the  surplus 
assets  as  follows:  First  distribution,  $42,410.20;  second  distribution, 
$74,622.30;  final  distribution,  $31,967.50. 

Prepare  a  statement  of  the  partners'  accounts,  showing  how  the  distribu- 
tion of  the  assets  should  be  made  and  the  apportionment. 

(Md.,  Feb.,  1915;  N.  Y.,  June,  1917;  S.  C,  Sept.,  1919.) 

2.  "A,"  "B,"  and  "C"  were  in  partnership,  "A's"  capital  being  $90,000, 
"B's"  $50,000  and  "C's"  $50,000.  Their  agreement  is  to  share  profits 
in  the  following  ratio :  "A,"  60  per  cent ;  "B,"  15  per  cent ;  "C,"  25  per 
cent.  During  the  year  "C"  withdrew  $10,000.  Net  losses  on  the  business 
during  the  year  were  $15,000,  and  it  is  decided  to  close  out  the  business. 
It  is  uncertain  how  much  the  assets  will  ultimately  yield,  although  none  of 
them  is  known  to  be  bad.  The  partners  therefore  mutually  agree  that  as 
the  assets  are  liquidated,  distribution  of  cash  on  hand  shall  be  made 
monthly  in  such  a  manner  as  to  avoid,  so  far  as  feasible,  the  possibility  of 
paying  to  one  partner  cash  which  he  might  later  have  to  repay  to  another. 
Collections  are  made  as  follows:  May,  $15,000;  June,  $13,000;  July, 
$52,000.  After  this  no  more  can  be  collected.  Show  the  partners'  ac- 
counts indicating  how  the  cash  is  distributed  in  each  installment,  the 
essential  feature  in  the  distribution  being  to  observe  the  agreement 
given  above.     (Ind.,  May,  1917;  A.  I.  of  A.,  June,  1917.) 

3.  "A,"  "B,"  and  "C"  were  partners  in  a  business  on  the  following 
basis : 


Capital  Contributed 

Share  of  Profits 

Salaries 

"A" $45,000 

"B" 22.500 

••C" 7,500 

.50  per  cent 
40  per  cent 
10  p>er  cent 

$6,000 
4,000 
2,400 

PARTNERSHIP  AT  LIQUIDATION  121 

At  the  end  of  the  second  year's  business,  "A"  died.  The  partners'  draw- 
ing accounts  before  crediting  their  year's  salaries  appeared  with  the  fol- 
lowing debit  balances:    "A,"  $2,572;  ''B,"  $1,218;  "C,"  $1,710. 

The  net  assets  of  the  business  after  finally  closing  the  books  were  found 
to  be  $74,780.  "B"  and  "C"  liquidate  the  affairs  of  the  partnership. 
Three  distributions  of  the  proceeds  of  liquidation  were  made  as  follows: 
$25,000,  $35,000,  $11,780. 

You  are  asked  to  prepare  a  tabulation  showing  the  share  of  each  of  the 
distributions  to   each  of  the  partners. 

(Wis.,  Nov.,  1919.) 

4.  "A"  and  "B,"  on  winding  up  their  partnership,  found  their  assets 
realized  as  follows:  Factory  premises  standing  in  their  books  at  $10,000 
realized  $4,000;  machinery  standing  in  their  books  at  $7,500  realized 
$2,500;  merchandise  standing  in  their  books  at  $5,500  realized  $4,500; 
accounts  receivable  standing  in  their  books  at  $9,500  realized  $6,500, 

Their  unpaid  liabilities  were  $10,500.  "A's"  capital  stood  at  $15,000, 
and  "B's"  capital  at  $7,000.  In  respect  of  profits  and  losses  they  were 
equal  partners. 

Divide  the  proceeds  of  the  realization  between  them  after  paying  off 
the  liabilities,  and  debit  them  as  having  been  paid  the  proportion  to 
which  each  was  entitled,  and  show  what  amount  would  be  payable  (if  any) 
by  either  partner  to  the  other  to  settle  the  accounts, 

(111.,  May,  1910;  N.  D,,  Aug.,  1917.) 

5.  "A,"  "B,"  and  "C"  are  in  partnership.  "A"  invested  $11,000;  "B" 
invested  $5,000 ;  and  "C"  invested  $1,200.  Their  agreement  provides  that 
profits  or  losses  shall  be  divided  as  follows:  ^"A,"  4/9;  "B,"  3/9; 
"C,"  2/9. 

The  partnership  has  become  insolvent  and  has  therefore  decided  to  dis- 
solve. The  cash  value  of  assets  is  $10,000.  The  deficit  is,  therefore,  $7,200. 
How  should  the  assets  be  divided  and  how  much  money  will  each  partner 
receive?  (Wis.,  May,  1916.) 

6.  "A"  and  "B"  are  partners  sharing  losses  and  gains  equally.  "A" 
invested  $3,000,  "B"  invested  $4,000.  They  are  ready  to  wind  up  the 
business.  The  firm  owes  $5,000  of  which  $1,000  is  due  "A,"  and  $500  is 
due  "B."  They  have  $7,000  in  cash.  Prepare  the  accounts  showing  the 
closing.     (Mich.,  June,  1914.) 

7.  Jones  and  Brown  are  partners,  sharing  profits  equally.  Their  capi- 
tal as  it  appears  on  the  books  of  the  partnership  on  June  20,  1908.  the 
date  on  which  they  dissolve  partnership,  is  Jones  $2,000  and  Brown  $500. 
The  total  amount  owing  by  the  firm  is  $5,000  which  includes  $1,000  due  to 
Jones  on  a  loan  and  $500  due  to  Brown  on  a  loan.  The  whole  of  the 
assets  of  the  firm  realize  $6,000.  Prepare  accounts  closing  up  the  part- 
nership and  show  the  position  in  which  the  partners  stand  with  each 
other.     (Mich.,  July,  1909.) 

8.  Two  parties,  "A"  and  "B,"  have  been  in  business  for  the  three  years 
ending  December  31,  1904,  on  which  date  they  agree  to  dissolve  partner- 


12-. 


C.  P.  A.  ACCOUNTING 


ship.  "A"  takes  over  the  business,  paying  *'B"  $7,500  for  his  share  of  the 
goodwill.  "A"  has  drawn  out  each  year  $2,000  and  ''B"  $3,000.  "A's" 
capital  at  start  was  $10,000,  and  "B's"  $12,500,  and  the  profits  of  each 
year  have  been  $3,500,  $4,200  and  $4,600,  respectively.  There  was  no  deed 
of  partnei-ship,  nor  any  agi'eement  as  to  interest  on  capital.  Draft  ac- 
counts showing  "A's"  capital  on  taking  over  the  business,  and  the  amount 
"B"  will  receive  on  retiring.     (Wash.,  April,  1906.) 

9.  "A,"  "B,"  and  "C"  engage  in  business,  "A"  contributing  $10,000 
capital,  "B"  contributes  $5,000,  Avhile  "C"  in  lieu  of  any  capital  contribu- 
tion, agrees  to  undertake  the  active  management,  at  a  salary  of  $3,000  a 
year,  to  be  paid  monthly. 

After  allowing  5  per  cent  interest  on  capital,  they  are  to  divide  the  net 
result  in  the  proportions  of  5,  3  and  2,  respectively. 

At  the  end  of  eighteen  months  they  ascei1:ain  the  position  to  be 
unfavorable  and  decide  to  wind  up.  The  assets  realize  $12,500;  there  are 
no  liabilities  except  for  capital  and  interest  thereon  and  one  month's 
salary,  due  to  "C." 

Make  up  the  partners'  accounts  showing  the  amount  to  be  received  by 
each. 

(Mass.,  Oct.,  1914.) 

10.  "A"  and  "B,"  partnei-s  in  a  commercial  enterprise,  share  profits 
and  losses  equally.  At  the  end  of  five  years  the  partnership  terminates 
and  the  balance  sheet  shows  as  follows : 


Assets 

Cash  on  hand  and  in  bank ....  $5,600 

Accounts  Receivable 28,000 

Merchandise  Inventory 38,000 

Plant  and  Machinery 15,400 


$85,000 


Liabilities 

Accounts  Payable $30,000 

•Bills  Payable 10,000 

Capital : 

"A" $30,000 

"B" 15,000     45,000 

$85,000 


After  studying  the  Balance  Sheet  an  offer  to  buy  at  $30,000  (except 
cash)  is  accepted.  Make  final  adjustments  and  closing  entries  and  show 
amount  each  partner  receives.  (Ind.,  June,  1916.) 

11.  By  partnership  agreement  existing  between  Brown  and  Gray, 
Brown  has  2/3  of  the  profits  and  Gray  1/3.  Brown's  capital  account 
stands  credited  $50,000.  Gray's  capital  account  stands  credited  $40,000. 
The  assets  of  the  partnership  consist  of  the  following:  Factory  and  Ma- 
chinery, $75,000;  Stock  as  per  inventories,  $30,000;  Accounts  Receivable, 
$15,500.  The  liabilities  are  as  follows :  Accounts  Payable,  $15,000 ;  Notes 
Payable,  $10,000;  Overdraft  Bank,  $5,500. 

Gray  takes  over  the  liabilities  as  above  mentioned,  and  the  assets  at  the 
following  agreed  valuation:  Factory  and  Machinery,  $70,000;  Stock  as 
per  inventories,  $26,000;  Accounts  Receivable,  $14,000. 

An  arrangement  is  made  whereby  Gray  received  $500  from  Brown  for 
accepting  sole  liability  for  the  discounted  bills  receivable. 


PARTNERSHIP  AT  LIQUIDATION  123 

Make  up  a  statement  showing  the  amount  that  Brown  should  receive  it 
being  understood  that  the  losses  on  capital  are  borne  by  the  partners  in 
the  proportion  in  which  the  profits  are  divided.  (W.  Va.,  May,  1919.) 

12.  Brown,  Black  and  Green  were  engaged  in  a  merchandise  business  as 
partners.  On  December  31,  1913,  a  Balance  Sheet,  to  which  all  of  the 
partners  agreed,  showed  the  following  financial  position  in  the  business: 


Assets 

Real  Estate $10,000 

Horses,  Wagons,  and  Fixtures  5,000 

Merchandise  on  hand 14,000          Brown 


LiahilUies 

Mortgage  Payable $4,000 

Accounts  Payable 30,000 


Accounts  Receivable 38,000 

Cash  in  bank  and  on  hand. .  .      14,000 
Green 1,000 


$82,000 


Capital  account    $25,000 
Drawing  account    10,000     35,000 


Black: 

Capital  account    $15,000 
Drawing  account      2,000      13,000 


$82,000 


The  profits  are  shared  in  the  following  proportions :  Brown,  i/^ ;  Black, 
1/3;  and  Green,  1/6.  It  was  decided  to  dissolve  the  partnership  as  at 
the  date  of  the  above  Balance  Sheet.  The  real  estate  was  sold  for  $13,000. 
Bad  debts  and  discounts  allowed  amounted  to  $5,000.  The  merchandise 
realized  $12,000,  and  the  horses,  wagons  and  fixtures  were  sold  for  $3,000. 
The  mortgage  on  the  property  was  paid  off,  interest  thereon  amounting  to 
$80.  The  accounts  payable  were  paid,  less  discounts  amounting  to  $1,000. 
The  expenses  incurred  during  the  period  of  realization  amounted  to  $3,000. 

After  the  assets  had  been  realized  and  all  the  liabilities  had  been  dis- 
charged. Green  became  bankrupt,  and  a  claim  was  made  against  his  estate 
for  the  amount  due  from  him  to  the  firm  on  the  dissolution  and  a  dividend 
thereon  at  the  rate  of  20  cents  on  the  dollar  received. 

By  the  end  of  the  year  1914  all  matters  were  disposed  of.  Write  up 
the  Realization  (Profit  and  Loss)  account,  Cash  account  (in  summary 
form),  and  the  Capital  and  Drawing  accounts  of  the  partners,  and  close 
the  books  of  the  firm.  (Wash.,  June,  1915.) 

13.  "A"  and  "B"  were  partners  trading  under  the  name  of  "A,"  "B" 
and  Company.  June  30,  1908,  the  following  balances  appear  on  their 
ledger : 

"A"  Capital  account $70,000 

"B"  Capital  account 50,000 

Real  estate 22;000 

Buildings 20,000 

Machinery  and  tools 44,000 

Furniture  and  fixtures 2,000 

Accounts  receivable 50  000 

Cash 7,000 

Materials  and  merchandise 53,000 

Accounts  payable ^^'^92 

Bills  payable f 'JSn 

Bills  receivable 5,000 


124 


C.  P.  A.  ACCOUNTING 


On  June  30,  1908,  the  business  is  incorporated  as  the  "X"  Company,  on 
the  following  plan: 

(1)  Capital  Stock,  $150,000. 

(2)  "X"  Company  takes  over  entire  assets  and  liabilities  of  "A,"  "B" 
and  Company,  at  the  book  figures  as  above,  except  (a)  real  estate  of  the 
book  value  of  $5,000,  which  is  retained  by  "A,"  "B"  and  Company;  (b) 
the  Accounts  Receivable,  which  are  taken  over  at  $48,000,  and  (c)  the 
Capital  accounts  of  the  partners. 

(3)  "X"  Company  pays  "A,"  "B"  and  Company  $30,000  for  the  good- 
will of  the  business. 

(4)  Payments  to  "A,"  "B"  and  Company  are  made  as  follows:  viz., 
$50,000  in  first  mortgage  bonds,  and  the  Balance  in  capital  stock  of  the 
"X"  Company. 

(5)  After  paying  off  "A,"  "B"  and  Company  the  remainder  of  the 
capital  stock  is  sold  for  cash  to  sundry  persons. 

The  real  estate  which  is  retained  by  "A,"  "B"  and  Company  is  bought 
from  "A,"  "B"  and  Company  by  "A"  for  $7,000  and  is  to  be  charged  to 
"A's"  Capital  account. 

After  the  completion  of  the  foregoi'ig  described  transactions  "A"  and 
"B"  dissolve  partnership. 

You  are  required : 

(a)  To  prepare  closing  entries  for  the  books  of  "A,"  "B"  and 
Company. 

(6)  A  statement  setting  forth  the  partners'  accounts  down  to  their 
final  closing,  beginning  with  the  balances  shown  bv  the  books  on  June  30, 
1908. 

(Wash.,  Aug.,  1908.) 

14.  "A"  and  "B"  trading  in  partnership  decide  to  admit  "C"  as  from 
January  1,  1919.    They  agree  with  "C"  as  follows: 

"C"  is  unable  to  contribute  any  tangible  assets  as  his  capital  investment, 
but  agrees  to  allow  his  share  of  the  profits  to  be  credited  to  his  capital 
account  until  he  shall  have  one  fifth  interest.  "C"  is  to  share  profits  and 
losses  to  the  extent  of  one  fifth. 

"C"  is  to  receive  a  salary  of  $3,000  per  annum,  payable  monthly  in 
addition  to  his  share  of  profits. 

Balance  Sheet  of  "A"  and  "B"  at  December  31,  1918,  lis  as  follows: 


Assets 

Cash $1,.'>00 

Accounts  receivable 10.000 

Merchandise 7..500 

Furniture  and  fixtures 1 ,500 

Goodwill 2,.500 

$23,000 


Liabilities 

Accounts  payable $8,000 

Capital  accounts 

"A" $10,000 

"B" 5,000     15,000 


$23,000 


During  the  six  months  endeil  June  30,  1919,  the  business  has  sustained 
unusual  losses  and  it  is  decided  to  dissolve  the  partnership. 


PARTNERSHIP  AT  LIQUIDATION 
The  Balance  Sheet  at  that  date  is  as  follows: 


125 


Assets 

Cash $.")0() 

Accounts  Receivable r2,.5(X) 

Merchandise 5  000 

Furniture  and  fixtures 1,500 

Goodwill 2,500 

Deficit 

Being  loss  on   trading  for 

six  months 5,500 

$27,500 


Liabilities 

Accounts  Payable $12  500 

Capital  Accounts 

"A" $10,000 

"B" 5,000     15,000 


$27,500 


Accounts  Receivable  were  sold  for  $9,000,  the  buyer  assuming  all  re- 
sponsibility for  collection  and  loss,  if  any. 

Merchandise  realized  $6,500,  and  furniture  and  fixtures,  $500. 

You  are  asked  to  make  an  examination  of  the  accounts  from  January  1 
and  to  prepare  statements  showing  the  realization  of  assets,  the  adjust- 
ment of  the  partnership  accounts,  and  the  distribution  of  the  funds. 

In  your  examination  you  find  that  "C"  has  not  drawn  his  salary  for 
four  months  and  that  "B"  has  advanced  to  the  partnership  $2,500  by  way 
of  a  temporary  loan.  These  liabilities  you  find  are  included  in  the  sum 
of  $12,500  shown  as  Accounts  Payable. 

"C"  is  ascertained  to  be  worthless. 

(A.  I.  of  A.,  Nov.,  1919.) 


CHAPTER  VIII 

CORPORATION  STOCK 

Orrrporation  Defined — Chief  Justice  Marshall  has  defined  a  corporation 
as  "an  artificial  being,  invisible,  intangible,  and  existing  only  in  contem- 
plation of  law."  A  corporation  is  an  artificial  legal  person,  or  entity, 
having  an  existence  distinct  from  that  of  the  members  composing  it,  and 
having  in  addition  to  the  powers  granted  by  its  charter,  the  implied  powers 
of  a  natural  person/ 

Classes  of  Corporations — From  the  standpoint  of  purpose,  corporations 
may  be  divided  into  two  classes,  viz.:  (a)  those  for  profit,  and  (b)  those 
not  for  profit.*  From  the  standpoint  of  ownership,  corporations  may  be 
classified  into  public  or  governmental  organizations,  and  private  corpora- 
tions.* Quasi-public  corporations  are  those  privately  owned  companies, 
which,  although  operated  for  profit,  promote  some  public  enterprise.* 
Private  corporations  are  of  two  types,  (a)  stock  companies,  or  firais 
operated  for  profit,  and  (b)  non-stock  companies,  or  non-profit-making 
enterprises,  such  as  libraries,  churches,  etc.° 

Stock  corporations  are  called  sole  corporations  if  all  the  stock,  except 
a  few  shares,  is  held  by  one  person,*  open  corporations  if  the  stock  is  freely 
bought  and  sold,'  and  close  corporations  if  the  stock  is  not  traded  in  and 
the  control  is  retained  within  a  small  compass  of  ownership.' 

From  the  standpoint  of  the  sovereignty  to  which  allegiance  is  due,  cor- 
porations are  either  domestic  or  foreign,  depending  upon  whether  the 
corporation  is  in  the  state  or  country  within  which  it  was  organized.* 
From  the  standpoint  of  the  fact  of  incorporation,  corporations  may  be 
classed  as  (a)  de  jure,  or  those  legally  incorporated,  and  (b)  de  facto, 
or  those  which  have  not  fully  met  all  legal  requirements  but  are  to  all 
intents  and  purposes  corporations  in  fact." 

Advantages  and  Disadvantages  of  Incorporation — The  chief  advantages 
of  incorporating  are  (a)  limited  liability  of  stockholders,  (b)  distinct  legal 
entity,  (c)  stability  and  permanency  of  organization,  (d)  transferability 
of  shares,  (e)  increased  available  capital,  and  (f )  centralized  control." 

The  chief  disadvantages  of  incorporating  are  (a)  lack  of  motivation 
on  the  part  of  officials,  (b)  onerous  taxation,  (c)  required  filing  of  many 
reports,  (d)  limitations  on  business  activities,  (e)  limitation  of  credit  to 
the  amount  of  net  assets,  and  (f )  illegality  in  some  states  of  owning  stock 
of  other  corporations." 

Procedure  of  Incorporation — The  state  laws  governing  incorporation 
usually  require  that  three  or  more  persons  prepare  an  application  for 

'For  explanation  of  superior  figures  see  page  337. 

126 


OORPORATTON  STOCK  127 

incorporation,  wliidi,  after  proper  publicity  and  when  approved  by  the 
proper  oflkial  after  the  payment  of  prescribed  fees,  becomes  tJie  charter 
or  certificate  of  incorporation  of  the  company."  The  stockholders  then 
complete  the  organization  by  electing  directors,  adopting  by-laws,  and 
making  necessary  arrangements  for  the  conduct  of  the  corporate  business." 

Charter — The  chartei',  or  certificate  of  incorporation,  recites  (a)  name 
of  corporation,  (b)  purpose  of  corporation,  (c)  amount  and  kinds  of 
capital  stock,  (d)  number  and  par  value  of  share  of  stock,  (e)  location 
of  principal  office,  (f)  duration  of  corporation,  (g)  number  of  directors, 
(h)  names  and  addresses  of  fii'st  directors,  and  (i)  names  and  addresses 
of  subscribers  to  the  eertiiicate  with  number  of  shares  subscribed  for  by 
each/' 

By-Laws — By-laws  are  the  rules  of  corporate  action  which  are  adopted 
at  stockholders'  meetings."  By-laws  regulate  the  relations  of  the  stock- 
holders with  the  directors,  of  the  directors  with  the  stockholders  and 
officers,  and  of  the  corporation  with  outsiders  who  have  knowledge  of  the 
existence  and  import  of  the  by-laws."  If  especially  authorized  by  the 
charter  or  by  the  state  laws,  directors  can  adopt  by-laws.'*  The  auditor 
should  ascertain  the  duties  and  powers  of  each  officer  from  the  by-laws," 
which  information  is  needed  before  the  authenticity  of  the  records  can  be 
thoroughly  verified. 

Stockholders — Stockholders  have  the  common  law  right  to  make  or 
change  the  by-laws  of  the  corporation.""*  They  may  insist  on  having  cer- 
tificates of  stock  issued  to  them  showing  the  number  of  shares  they 
own."  When  this  ownership  of  the  certificates  is  recorded  in  the  books 
of  the  corporation,  the  owners  are  called  stockholders  of  record."  Stock- 
holders of  record  alone  may  vote  and  receive  dividends,  and,  although  a 
transferee  has  equitable  title  to  any  dividend  declared  after  date  of  pur- 
chase, he  can  not  receive  it  except  by  recovery  through  the  payee,  the 
stockholder  of  record."*'  Stockholders  have  a  common  law  right  to  inspect 
the  books  of  the  corporation,  but  state  statutes  and  corporate  by-laws 
usually  require  that  good  reason  be  shown  for  such  inspection,  which, 
when  arbitrarily  enforced,  reduces  to  nil  the  stockholders'  right  to  inspect 
the  books  of  account."  Stockholders  may  vote  to  elect  directors,  increase 
or  decrease  the  capital  stock,  amend  the  charter,  and  dissolve  the  cor- 
poration." Stockholders  may  subscribe  to  their  due  proportion  of  all 
increases  in  capital  stock."'  At  stockholders'  meetings  each  stockholder  is 
entitled  to  a  vote  for  each  share  of  stock  standing  in  his  name  on  the 
books."  In  the  election  of  directors,  some  states  allow  each  share  of 
stock  one  vote  for  each  director  to  be  elected,  which  votes  may  be  cumulated 
upon  one  director  or  scattered  at  the  option  of  the  stockholders."  When 
a  stockholder  desires,  he  may  delegate  his  voting  powers  to  another  stock- 
holder who  is  said  to  be  his  proxy."  Auditors  are  .sometimes  called  upon 
lo  make  an  audit  of  proxies,  which  is  done  by  obtaining  a  list  of  the 
>lockholders  of  record  from  the  stockholders'  ledger  and  then  verifying 
the  transferal  of  the  voting  privileges.'* 


128  C.  P.  A.  ACCOUNTING 

In  the  event  of  insolvency,  stockholders  are  not  liable  for  the  firm's 
debts,  provided  their  stock  is  fully  paid,''  but  they  are  liable  for  the  dif- 
ference between  that  which  they  paid  and  that  which  thej'  should  have 
paid  to  make  their  stock  fully  paid,  even  though  the  corporation  agreed 
to  accept  the  payments  made  as  full  consideration.''  This  liability  is  fi'e- 
quently  avoided  by  having  the  directors,  whose  valuation  in  the  absence 
of  the  proof  of  fraud  is  final,"  issue  all  or  a  large  part  of  the  stock  for 
property,  the  former  ow-ners  of  which  then  donate  part  of  this  stock  to  the 
corporation  which  can  then  sell  this  once-issued  stock  below^  par."  Innocent 
purchasers  for  value  are  not  liable  for  the  unpaid  balances  on  their 
holdings.'' 

Directors — The  powers  of  directors  vary  largely  because  of  the  lack  of 
uniformity  in  state  statutes  and  corporate  by-laws.  In  general,  directors 
have  the  power  to  borrow  money,  to  create  new  debts,  to  liquidate  old 
debts,  to  lease  corporate  property  if  such  lease  does  not  force  the  dis- 
continuation of  corporate  business,  to  declare  dividends  out  of  surplus, 
to  conduct  corporate  litigation,  to  ratify  a  debt  which  has  been  barred 
by  the  statute  of  limitations,  to  make  and  transfer  negotiable  paper,  to 
fix  the  salaries  of  corporate  officers,  to  prepay  wages,  and  to  make  con- 
tracts with  persons  for  ser%'ices  for  periods  of  one  year  and  more." 

Unless  especially  authorized  by  state  statutes  or  corporate  by-laws, 
directors  must  secure  the  sanction  of  the  stockholders  before  they  can 
make  any  change  in  the  authorized  capital  stock  of  the  company,  issue 
mortgages,  cancel  subscriptions  for  stock,  or  dissolve  the  company."  They 
can  not  issue  more  stock  than  is  authorized,'*  or  execute  leases  divesting 
the  corporation  of  its  physical  assets.*  Unless  permitted  by  statutes, 
directors  can  not  issue  stock  below  par."  They  can  not  vote  by  proxy 
at  meetings  of  the  directors  as  the  position  of  director  is  a  fiduciary  one 
invohnng  trust  and  confidence." 

Since  directors  are  trustees,  both  for  the  stockholders  and  creditors, 
they  can  not  secure  for  themselves  any  advantage  which  the  stockholders 
can  not  share,  or  which  would  cause  the  rights  of  the  creditors  to  become 
inferior  to  theirs." 

As  many  states  hold  directors  personally  responsible  for  debts  con- 
tracted under  certain  circumstances  and  in  excess  of  certain  sums,  it  is 
important,  in  examinations  for  credit  purposes,  for  the  auditor  to  ascer- 
tain from  the  statutes  whether  or  not  the  directors  have  automatically 
made  themselves  liable  for  any  part  of  the  indebtedness."  Directors  are 
held  personally  liable  for  declaring  a  dividend  in  excess  of  profits  or 
surplus." 

Minute  Books — There  is  no  special  form  of  minute  book,  but  the  most 
modern  and  convenient  form  is  a  loose-leaf  book.*'  In  large  corporations, 
separate  books  are  kept  for  the  stockholders'  and  directors'  minutes,  but 
in  small  corporations  the  stockholders'  and  directors'  minutes  are  recorded 
in  the  front  and  back,  respectively,  of  the  same  book.**  The  minute  book 
should  contain,  first,  a  copy  of  the  charter;"  second,  a  copy  of  the  by- 
laws;*' and  lastly,  a  complete  record  of  the  kind  of  meeting,  whether 


CORPORATION  STOCK  129 

regular,   special,  or  adjourned,"   and   of  the   action   taken  at  the  stock- 
holders' and  directors'  meetings."" 

The  minutes  are  examined  by  auditors  with  regard  to  the  election  of 
officers,  the  compensation  of  officers,  extra  compensation,  bond  of  the 
treasurer,  depositories,  contracts  with  manager,  contracts  for  the  purchase 
of  a  business,  contracts  for  additions  to  plant,  contracts  for  future  de- 
livery of  materials  in  unusual  quantities,  valuation  fixed  on  purchased 
property,  depreciation  rates,  possibility  of  litigation,  settlement  of  pending 
litigation  for  a  sum  in  excess  of  liability  carried  therefor  in  the  books, 
and  contingent  liabilities."  If  the  auditor  is  denied  an  inspection  of  the 
minute  book,  he  should  mention  that  fact  in  his  report  and  in  a  footnote 
to  his  balance  sheet." 

Corporate  Officers — The  duties  of  the  officers  of  a  corporation  are  as- 
signed by  by-laws,  custom,  common  consent,  or  action  of  the  board  of 
directors."  The  number,  titles,  and  powers  of  these  officers  vary  accord- 
ing to  the  size  and  internal  organization  of  the  corporation."  A  corpora- 
tion usually  has  a  president,  one  or  more  vice-presidents,  a  secretary,  a 
treasurer,  and  an  auditor."^  These  officers  are  chosen  either  by  the  directors 
or  stockholders,"  usually  the  former.  No  officer  is  personally  liable  for 
the  acts  of  another  officer,  and  no  officer  can  involve  the  corporation  by 
unauthorized  acts." 

Capital  Stock — The  capital  stock  of  a  corporation  is  the  amount  of 
stock  which  it  is  authorized  to  issue."  The  amount  of  authorized  capital 
stock  can  be  changed  only  by  compliance  with  the  requirements  of  the 
state  laws."  It  is  sometimes  claimed  that  capital  stock  represents  the 
total  number  of  shares  of  stock  outstanding,*^"  but  the  term  is  more  inclusive 
than  that,  as  unissued  authorized  stock  is  also  capital  stock."  The  capital 
stock  may  be  called  the  nominal  or  share  capital  of  the  corporation."*  If 
the  actual  capital  stock  of  the  corporation,  the  excess  of  assets  over 
liabilities,'^  is  greater  than  its  capital  stock,  the  difference  is  called 
surplus."  This  difference  between  the  nominal  and  actual  capital  of  a 
corporation  causes  the  capital  stock  to  have  two  kinds  of  value,  namely, 
par  value,  or  the  nominal  value  printed  on  the  certificate,  and  book  value, 
or  the  quotient  of  the  net  worth  of  the  corporation  divided  by  the  number 
of  shares  outstanding. 

No  Par  Stock — Ten  states  permit  the  issue  of  capital  stock  with  no 
par  value."  The  stock  certificates  must  show  the  number  of  shares  they 
represent  and  the  number  of  shares  the  corporation  is  authorized  to 
issue.*'  The  value  of  no  par  stock  is  its  book  value  found  by  dividing 
the  net  worth  by  the  number  of  shares  outstanding,'^  each  share  repre- 
senting an  aliquot  interest."*  When  a  company  issues  both  par  and  no 
par  stock,  care  must  be  taken  to  show  the  portion  of  the  profits  belonging 
to  each,  though  this  division  may  or  may  not  be  shown  on  the  balance 
sheet." 

When  all  the  stock  is  without  par  value,  and  there  is  but  one  class  of 
stock,  there  is  no  necessity  for  maintaining  a  surplus  or  undivided  profits 
account,  for  the  result  of  current  operations  can  be  closed  into  the  capital 


130  C.  P.  A.  ACCOUNTING 

stock  account.'"  However,  no  par  stock  may  be  held  on  the  books  at  its 
book  value  just  after  the  organization,  and  fluctuations  in  value  may  be 
shown  in  a  surplus  or  deticit  account.'*  The  latter  method,  which  is  called 
holding  the  no  par  stock  at  its  stated  value,  seems  preferable.  In  either 
ease,  specific  projirietorship  reserves  should  be  stated  separately."  It  is 
sometimes  contended  that,  if  cui'rent  profits  are  closed  into  the  capital 
stock  account  instead  of  into  the  surplus  account,  the  profits  would  not 
be  available  as  dividends  on  the  ground  that  combining  the  capital  stock 
and  current  profits  has  the  same  effect  as  a  stock  dividend  in  an  ordinary 
corporation."  This  contention  overlooks  the  fact  that  stock  dividends 
involve  the  issue  of  new  shares  of  stock. 

The  no  par  capital  stock  account  should  be  credited  for  the  value  re- 
ceived for  the  stock.^*  The  number  of  shares  outstanding  should  be  shown 
either  on  the  balance  sheet'"  or  in  a  footnote  thereto."'  Unsubscribed  no 
par  stock  should  not  be  shown  at  all  on  the  balance  sheet  as  it  may  be 
subscribed  for  at  prices  othei-  than  the  stated  value."  When  unsubscribed 
stock  is  sold  after  a  company  has  accumulated  a  surplus,  the  amount 
received  in  excess  of  the  stated  value  of  the  then  outstanding  stock  should 
be  credited  to  the  surplus  account,  if  that  account  is  maintained."  The 
number  of  donated  shares  may  be  earned  without  value  in  the  treasury 
stock  account,'*  but,  if  a  surplus  account  is  maintained,  it  would  seem 
preferable  to  value  the  donated  shares  at  their  stated  value  and  credit 
the  capital  surplus  account  therefoi-.  No  par  stock  repurchased  may 
be  debited  to  the  treasury  stock  account  at  cost  price,*"  but,  if  a  surplus 
account  is  maintained,  it  would  seem  preferable  to  debit  the  treasury  stock 
account  for  the  stated  value,  the  difference  between  the  stated  value  and 
cost  being  charged  or  credited  to  the  capital  surplus  account. 

The  advantages  of  issuing  no  par  stock  are  (a)  the  checking  of  the 
tendency  to  inflate  value  of  assets  in  order  to  offset  the  nominal  value  of 
certificates  issued  therefor,*'  (b)  the  assurance  given  the  investor  that 
.stock  is  fully  paid  and  non-assessable,'"  (c)  the  special  facilities  afforded 
in  ease  of  consolidation  or  i-eorganization,*''  and  (d)  the  fact  that  the 
investor  is  put  on  liis  guard  to  find  out  the  real  value  of  the  stock."  The 
objection  that  no  par  stock  provides  more  oj)portunities  for  improper 
manipulation  does  not  seem  to  be  justified,  as  there  are  many  well  known 
legal  methods  of  evading  the  statutory  requirements  as  to  a  fixed  par 
value."' 

Kinds  of  Stock  with  Par  Value — The  coi'porate  stock  having  par  value 
may  be  divided  into  the  following  classes : 

(a)  Unsubscribed,  or  shares  not  only  unissued  but  fo)-  which  no  sul)- 
scription  has  been  received.*' 

(b)  Unissued  (.sometimes  called  potential),  or  sliares  for  which  cer- 
tificates have  not  been  issued."' 

(e)  Treasury,  or  shares  of  issued  fully  paid  stock  reacquired  and  held 
by  the  corporation  subject  to  disposal  by  directoi's.''* 

(d)  Authorized,  or  total  number  of  shares  the  corporation  has  permi-<- 
sion  to  issue. 


CORPORATION  STOCK  131 

(e)  Canceled,  or  the  shares  of  authorized  stock  which  have  been  de- 
clared void,  thus  reducing  the  number  of  shares  the  corporation  can  issue. 

(f )  Outstanding,  or  issued  shares  in  the  hands  of  the  public." 

(g)  Non- voting,  or  shares  not  giving  owners  votes  in  stockholders' 
meetings.'" 

(h)  Common,  or  shares  with  no  financial  preference  over  any  other 
stock  of  the  company."' 

(i)  Preferred,  or  shares  with  financial  preference,  either  as  to  divi- 
dends or  principal,  or  both,  over  other  stock  of  the  company.*'' 

(j)  Non-participating,  or  shares  of  pi-ef erred  stock  which  do  not  share 
in  the  profits  beyond  their  preferential  dividends.*^ 

(k)  Non-convertible,  or  shares  which  cannot  be  exchanged  for  some 
other  form  of  ownership  or  obligation." 

(1)  Cumulative,  or  shares  whose  dividends,  if  not  paid  in  one  year, 
continue  as  a  charge  against  the  profits  of  succeeding  years.'" 

(m)  Redeemable,  or  shares  issued  under  contract  to  redeem  them  after 
a  certain  length  of  time,  at  a  named  figure."" 

(n)  Founders,  or  shares  of  English  origin  (issued  to  promoters)  which 
are  preferred  as  to  their  share  of  dividends;  for  instance,  founders'  stock 
might  receive  one-half  more  dividends  than  the  ordinary  common  stock." 

(o)  Guaranteed,  or  shares  which,  as  to  dividend  or  principal,  or  both, 
have  been  actually  guaranteed  by  some  person,  concern,  or  corporation 
other  than  the  issuing  corporation."* 

(p)  Debenture,  or  shares  issued  under  contract  to  pay  absolutely  there- 
on, at  specified  intervals,  a  specified  return."'  In  England  debentures  are 
unsecured  loans  issued  in  irregular  amounts.'*"  Debenture  stocks  are  to  be 
classified  as  liabilities."" 

(q)   Watered,  or  shares  fictitiously  designated  as  paid-up  stock.'" 

(r)  Forfeited,  or  shares  taken  back  because  of  failure  to  make  the 
agreed  purchase  payments."' 

(s)  Bonus,  or  shares  given  extra  upon  the  purchase  of  other  stocks  or 
bonds.'" 

(t)  Donated,  or  shares  of  issued  paid-up  stock  which  have  been  given 
back  to  the  corporation.'""  Donated  stock  should  be  entered  on  the  books 
as  treasury  stock.'*' 

As  each  of  these  classes  contains  only  shares  having  special  characteris- 
tics, stocks  not  having  these  characteristics  must  fall  into  classes  whose 
titles  are  generally  the  opposite  of  the  above,  viz.,  subscribed,  participat- 
ing, irredeemable,  etc. 

Capital  Stock  on  Balance  Sheet — While  unsubscribed  stock  is  not  an 
asset  as  far  as  the  outside  creditors  are  concerned,  yet  it  is  a  matter  of 
which  the  stockholders  should  have  knowledge.'"'  The  unsubscribed  stock 
should,  therefore,  be  deducted  from  the  authorized  stock,  as  this  would  not 
list  the  unsubsci'ibed  stock  as  an  asset."*  As  unissued  stock  is  not  an  asset, 
it  should  be  deducted  from  the  authorized  capital  stock."* 

Some  accountants  believe  that  treasury  stock  is  an  asset,  as  it  represents 
an  actual  value  received  by  a  corporation,  either  through  purchase  or 


132  C.  P.  A.  ACCOUNTING 

donation,  ""  and  they  so  list  it  on  the  balance  sheet/"  Some  aeeountants 
classify  treasury  stock  as  an  "accrued  item,"""  but  this  seems  incorrect, 
for,  even  if  treasury  stock  were  an  asset,  it  does  not  accrue.  Other 
accountants  see  in  treasui-y  stock  a  decrease  in  stock  proprietoi'ship,  and 
deduct  it  at  par  from  the  authorized  capital  stock.""  It  seems  that  the 
last  view  is  preferable,  especially  as  treasury  stock  does  not  fulfill  the 
functions  of  an  asset  when  the  corporation  is  dissolved;  it  liquidates 
neither  the  liabilities  nor  the  outstanding  capital  stock. 

Stock  Certificate  Book — Stock  certificates  are  usually  bound  together 
with  a  stub  for  each  certificate.'"  Both  the  certificate  and  the  stub  are 
numbered  consecutively,"'  and  contain  spaces  for  the  purchaser's  name 
and  the  number  of  shares  issued.""  The  stub  also  lists  for  what  the  stock 
was  issued,  name  and  address  of  former  owner  of  shares,  number  of 
original  certificate,  number  of  shares  on  original  certificate,  number  of 
shares  transferred,  and  a  receipt  for  the  certificate."'  The  certificate  is  a 
formal  document  under  coi'porate  seal,  and  with  the  signatures  of  the 
authorized  oflBcers."'  On  the  back  of  the  certificate  there  is  usually  a 
blank  for  assignment  of  the  stock.^"  The  certificates  for  the  stock  that 
has  been  transferred  or  canceled  should  always  be  pasted  back  on  the 
stubs  from  which  they  were  taken.'*"  The  stubs  that  have  no  certificates 
attached  would  then  represent  the  outstanding  stock.'" 

Stockholders'  Ledger — Except  in  New  York  and  a  few  other  states  where 
the  form  of  the  stockholders'  ledger  is  prescribed  by  law,  there  is  little  or 
no  uniformity  in  stockholders'  ledgers."*  The  object  of  this  ledger  is  to 
keep  with  each  stockholder  an  account  which  will  show  the  date  and  num- 
ber of  the  certificates  and  the  number  of  shares  on  each  certificate  issued 
to  him,  and  if  any  certificates  are  transferred,  the  date,  name  of  trans- 
feree, number  of  surrendered  certificates,  number  of  reissued  certificates, 
and  the  number  of  shares  transferred.'"  Any  form  tabulating  this  data 
conveniently  is  satisfactorj'. 

A  capital  stock  account  is  sometimes  opened  on  the  front  page  of  the 
stockholders'  ledger,  and  debited  with  the  aggi'egate  amount  of  shares 
credited  to  the  stockholders'  accounts,  which  practice  makes  the  ledger 
self-balancing  and  shows  at  any  time  the  amount  of  outstanding  stock.'" 

Opening  Entries  for  Corporation — There  are  so  many  satisfactory 
methods  of  opening  corporate  books  that  most  authors  give  more  than  one 
way.  The  following  procedure'"  is  recommended  for  three  reasons,  namely, 
(a)  unissued  stock  is  shown  on  the  books,  (b)  the  practice  of  not  issuing 
stock  until  fully  paid  is  recognized,  and  (c)  the  distinction  between  un- 
issued and  unsubscribed  stock  is  drawn : 

Entry  No.  1 

Unissued  Capital  Stock xxxxxxx 

Capital  Stock  Authorized xxxxxxx 

Entry  No.  2 

Subscribers xxxxxxx 

Subscriptions xxxxxxx 


CORPORATION  STOCK  133 

Entry  No.  3 

Cash xxxxxxx 

Subscribers xxxxxxx 

Entry  No.  4 

Subscriptions xxxxxxx 

Unissued  Capital  Stock xxxxxxx 

For  a  briefer  procedure  the  following  is  recommended."^ 

Unsubscribed  Capital  Stock xxxxxxx 

Subscribed  Capital  Stock xxxxxxx 

Authorized  Capital  Stock xxxxxxx 

Cash xxxxxxx 

Subscribed  Capital  Stock xxxxxxx 

Subscriptions — Unconditional  accepted  subscriptions  for  capital  stock 
bind  the  subscribers,  as  they  can  be  enforced  either  by  the  company  itself 
or  the  creditors  after  insolvency.'"  The  uncalled  subscriptions  for  capital 
stock  are  accounts  receivable,  but  they  must  be  kept  in  a  class  by  them- 
selves."* Sometimes  unpaid  calls  are  deducted  from  the  authorized  capital 
stock,'"  but  it  seems  preferable  to  treat  them  as  assets.  The  practice  of 
classifying  subscriptions  as  "accrued  items'"^  seems  incorrect,  as  they  do 
not  accrue. 

Subscription  Ledger — There  is  no  set  form  to  the  subscription  books, 
but  it  usually  contains  columns  showing  when  the  calls  are  to  be  made, 
when  actually  made,  when  paid,  and  the  balance  due."*  In  general,  the.se 
columns  are  grouped  so  that  the  ledger  follows  roughly  the  "T"  form, 
the  asset  subscriptions  being  entered  on  the  left  and  payments  thereof,  on 
the  right.'"  The  subscription  account  acts  as  the  controlling  account  for 
this  ledger."" 

Transfer  Journal — The  stock  transfer  journal  or  register  contains  the 
original  entries  of  stock  transfers  which  are  posted  to  the  stock  ledger."^ 
Sometimes  the  transfer  journal  is  combined  with  the  stock  ledger,  but  the 
laws  of  many  states,  including  New  York,  require  separate  books  and 
prescribe  the  form  of  the  transfer  journal.""  The  New  York  form  records 
in  separate  columns  the  date,  serial  number  of  canceled  certificate,  number 
of  shares,  by  whom  surrendered,  to  whom  issued,  serial  number  of  new 
certificate,  number  of  shares,  and  number  and  value  of  revenue  stamps."" 
The  transfer  journals  outside  the  state  of  New  York  approximate  the  same 
form  except  for  the  data, on  revenue  stamps."'  Sometimes,  the  transfer 
book  is  merely  a  duplication  of  the  assignment  ordinarily  appearing  on 
the  back  of  each  sfoclc  certificate.""  In  such  cases,  the  book  used  as  a 
posting  medium  for  stock  transfers  is  called  the  register  of  transfers.'" 

Installments — If  the  subscribers  do  not  settle  for  their  stock  in  one  pay- 
ment, the  first,  second,  and  fourth  entries  in  the  proposed  detailed  pro- 
cedure for  opening  corporate  books  would  be  used,  but,  instead  of  the 
third  entry,  the  following  would  be  used:'*" 


134  C.  P.  A.  xiCCOUNTING 

Entry  No.  3  (a) 

Installment  (or  Call)  No.  1 xxxxxxx 

Installment  (or  Call)  No.  2 xxxxxxx 

Subscribers xxxxxxx 

Entry  No.  3  (b) 

Cash xxxxxxx 

Installment  (or  Call)  No.  1 xxxxxxx 

Installment  Ledger — The  purpose  of  the  installment  lx)ok  or  ledger  is 
to  give  a  classified  record  of  original  subscriptions,  installments  due  and 
paid,  and  balances  still  unpaid.'"  Separate  records  should  be  kept  for 
each  installment.'"  There  are  two  general  forms  of  installment  books. 
One  of  these  is  based  on  the  "T"  form  and  contains  on  tlie  left  side  columns 
for  date,  shares,  how  acquired,  percentage  unpaid,  certificate  number  of 
scrip,  and  amount ;  and  on  the  right  side,  columns  for  date,  shares  trans- 
ferred, how  settled,  percentage  unpaid,  certificate  number  of  scrip,  how 
many  shares  paid,  installment  number,  and  amount.'"  The  other  form  is 
based  on  the  Boston  ledger  form,  and  contains  a  list  of  subscribers,  num- 
ber of  shares  subscribed  by  each,  amount  of  installment,  date  of  payment, 
amount  received,  and  remarks  relating  to  the  particular  installment.*" 
The  installment  account  is  the  controlling  account  for  the  installment 
ledger.'" 

Installment  Scrip  Book — The  installment  scrip  book  contains  blank  re- 
ceipts to  be  filled  out  and  signed  by  the  authorized  corporate  officers  when 
the  installments  are  paid  by  the  subscribers."'  The  details  of  each  receipt 
issued  are  recorded  on  its  stub,  from  which  the  proper  entries  may  be 
obtained  for  the  stockholders'  ledger  and  for  the  other  books.'"  Upon  the 
payment  of  the  last  installment,  the  scrip  is  taken  up  and  replaced  by  a 
certificate  of  stock.'"  Instead  of  giving  individual  receipts  for  the  pay- 
ment of  each  installment,  sometimes  a  more  elaborate  receipt  called  a  stock 
scrip  is  given  on  the  first  payment,  and  subsequent  installments  are  en- 
dorsed on  the  back  of  the  stock  scrip.'*  Both  the  installment  scrip  and 
the  stock  scrip  are  transferable."* 

Forfeited  Stock — Unpaid  subscriptions  to  capital  stock  can  be  declared 
forfeited  only  if  the  statutes  specifically  grant  the  corporation  that  power.'" 
In  some  states  the  entire  amount  paid  in  may  be  declared  forfeited,'"  while 
in  others  only  enough  to  cover  the  cost  of  placing  the  corporation  in  as 
good  condition  as  it  would  have  been  had  the  subscriber  fulfilled  his  obli- 
gation, may  be  retained  by  the  corporation.'"  When  a  corporation  declares 
a  subscriber's  stock  forfeited  for  nonpayment  of  any  balance,  all  liability 
on  the  part  of  tlie  subsci'iber  ceases  both  to  the  creditors  of  the  corpora- 
tion and  to  the  corporation  on  account  of  the  subscription."*  The  profit 
arising  from  the  forfeiture  of  stock  is  sometimes  credited  to  profit  and 
loss,'"  but  as  the  profit  is  not  an  operating  item,  it  is  preferable  to  credit 
it  directly  to  surplus.""  Any  discount  or  premium  on  the  sale  of  forfeited 
stock  should  be  closed  into  the  surplus  arising  from  the  foi'feiture.'" 
Surplus  from  forfeited  stock  should  not  be  declared  as  dividends,  although 


CORPORATION  STOCK 


135 


such  action  is  not  illegal."^*     The  auditor  should   verify  forfeited  stock 
with  data  from  the  minute  book."' 

By  way  of  an  illustration  of  the  entries  for  forfeited  stock  under  the 
proposed  detailed  proeediii'e  for  opening  corporate  entries,  assume  an 
authorized  capitalization  of  $1,000,000,  one-half  subscribed  and  subscrip- 
tions payable  in  four  equal  installments.  The  following  entries""  would 
be  necessary  if  one  hundred  shares  were  declared  forfeited  for  failure  to 
pay  the  second,  third,  and  fourth  installments  and  these  shares  were  resold 
at  $90  a  share.  The  entries  have  been  divided  into  four  parts  to  show 
what  entries  take  the  place  of  the  four  usual  entries  in  the  proposed  pro- 
cedure for  opening  corporate  books. 


Parti 

Unissued  Capital  Stock $1,000,000.00 

Capital  Stock  Authorized $1 ,  000 ,  000 .  00 

Part  2 

Subscribers 500,000. 00 

Subscriptions 500,000. 00 

Parts 

Installment  No.  1 125,000.00 

Installment  No.  2 125,000.00 

Installment  No.  3 125,000.00 

Installment  No.  4 125,000.00 

Subscribers 500,000. 00 

Cash 492,500.00 

Installment  No.  1 125,000.00 

Installment  No.  2 122, 500. 00 

Installment  No.  3 122,500.00 

Installment  No.  4 122,500.00 

Subscribers 7,500.00 

Installment  No.  2 2,500.00 

Installment  No.  3 2,500.00 

Installment  No.  4 2,500.00 

Subscribers 2,500.00 

Surplus  from  Forfeited  Stock ....  2 ,  500 .  00 

Subscriptions 10,000.00 

Subscribers 10,000.00 

Subscribers 9,000.00 

Surplus  from  Forfeited  Stock 1 ,000. 00 

Sub.scriptions 10,000.00 

Cash 9,000.00 

Subscribers -  . .  9,000.00 

Part  4 

Subscriptions 500,000.00 

Unis.sued  Capital  Stock 500 , 000. 00 


136  C.  P.  A.  ACCOUNTING 

Payment  for  Stock — As  is  shown  by  the  above  illustration,  the  method 
of  paj'ment  for  stock  affects  only  the  third  section  in  the  proposed  pro- 
cedure for  opening  corporate  books.  This  section  may  be  further  compli- 
cated because  stock  may  be  paid  for  by  cash,  services,  or  property.  In 
all  these  cases,  the  credit  is  to  subscribers.  However,  if  cash  is  received, 
then  the  cash  account  is  debited;"'^  if  services  are  received,  accounts  like 
manager's  salary,  commissions,  etc.,  are  debited;"'  and  if  property  is 
received,  the  fixed  asset  accounts  are  debited.^*'  When  going  concerns  are 
taken  over  by  new  corporations,  it  is  customary  to  debit  the  itemized  assets 
and  credit  a  vendor  account,  to  credit  the  itemized  liabilities  and  debit  the 
vendor  account,  and  then  to  close  the  vendor  account  against  the  sub- 
scriber's account."*  These  entries  are  illustrated  in  Chapter  VI,  Volume  II, 
under  the  caption  "opening  entries  for  amalgamated  companies."  Some- 
times a  sundry  asset  account  and  a  sundry  liabilities  account  are  used 
instead  of  the  vendor  account."^'  In  auditing  the  payments  for  stock,  the 
auditor  should  refer  to  the  minute  book  to  verify  the  authorization  of 
issues  for  property  or  for  cash  at  varying  rates  of  discount  or  premium.'"* 

Premium  and  Discount  on  Stock — Since  the  subscriber's  account  used  in 
the  second  section  of  the  proposed  detailed  procedure  for  opening  cor- 
porate books  is  an  asset  representing  the  amount  the  subscriber  promised 
to  pay  for  the  stock,  premium  and  discount  on  capital  stock  are  recorded 
in  the  second  section.  If  the  stock  is  .sold  at  a  premium,  the  debit  to 
subscribers  offsets  credits  to  both  the  subscriptions  and  premium  on  stock 
accounts.""  If  the  stock  is  sold  at  a  discount,  the  debits  to  subscribers  and 
discount  on  stock  accounts  are  offset  with  the  single  credit  to  subscriptions 
account.*'^* 

Discount  on  stock  should  not  be  regarded  as  organization  expense.*'*  It 
may  and  should  be  extinguished  by  premiums,  assessments,  surplus,  or  by 
the  retirement  of  an  equivalent  amount  of  stock.""  Some  accountants  show 
discount  on  capital  stock  on  the  balance  sheet  as  a  deferred  asset,'"  while 
others  list  it  as  a  deduction  from  the  par  value  of  the  outstanding  stock."' 
The  latter  method  seems  preferable  as  the  discount  really  causes  a  capital 
deficit. 

Premium  on  stock  is  a  capital  surplus,"'  and  .should  therefore  not  be 
distributed  as  dividends,  although  such  action  is  not  illegal."*  The  premium 
on  stock  account  should  be  kept  open  as  a  special  surplus  account  and 
this  special  surplus  should  not  be  transferred  into  general  surplus  where 
it  will  be  looked  upon  as  available  for  dividends."' 

The  Interstate  Commerce  Commission  requires  that  the  discount  on 
issued  stock  be  subtracted  from  the  premium  on  issued  stock,  and  that  any 
balance  be  shown  on  the  balance  sheet  as  a  deferred  item."' 

Donated  Stock — When  stock  is  donated  to  the  corporation,  it  is  recorded 
at  par  value."'  There  are  two  methods  of  handling  donated  stock,  which 
are  illustrated  in  the  following  entries  required  for  a  donation  of  shares 
having  a  par  value  of  $10,000,  but  which  realized  only  $7,500. 


CORPORATION  STOCK  137 

Entry  No.  1  (Same  both  methods) ''* 

Treasury  Stock $10 ,  000 ,  00 

Donation  Account  (or  Working  Capi- 
tal Donated) $10 ,  000. 00 

Entry  No.  2  (Same  both  methods) '-^ 

Cash 7,500.00 

Donation  Account 2 ,  500 .  00 

Treasury  Stock 10,000.00 

Entry  No.  3  (First  Method) '«« 

Donation  Account 7,500. 00 

Contributed  Surplus 7,500. 00 

Entry  No.  3  (Second  Method)'" 

Donation  Account 7,500.00 

(blank)    Account    (Insert    name    of 
fictitiously  valued  asset) 7 ,  500 .  00 

The  second  method  is  preferable  in  that  it  gives  a  conservative  pres- 
entation of  conditions,  but  the  choice  of  method  is  a  matter  to  be  decided 
by  the  treasurer  and  directors  of  the  corporation.'** 

If  desired,  the  discount  or  premium  on  treasury  stock  may  be  shown  in 
separate  accounts  under  those  titles,  instead  of  being  recorded  directly  in 
the  donation  account,  but  such  accounts  would  be  merged  into  the  donation 
account  when  all  the  stock  was  sold.'"  Sometimes  the  subscriptions  to 
treasury  stock  and  subscribers  to  treasury  stock  accounts  are  also  used."* 

Bonus  Stock — Stock  given  as  a  bonus  to  subscribers  for  the  issue  of 
other  corporate  securities  is  usually  treated  as  organization  expense  and 
written  off  over  a  period  of  years."°  However,  as  a  bonus  is  practically 
discount  on  treasury  stock,  it  seems  preferable  to  treat  a  bonus  as  discount 
on  stock  instead  of  as  organization  expense."*  Bonus  stock  is  usually 
treasury  stock,  as  original  stock  cannot  be  issued  below  par  as  paid-up 
stock."' 

Redemption  of  Stock — Reduction  in  capital  stock  cannot  be  accom- 
plished without  the  consent  of  at  least  a  majority  of  the  stockholders"* 
and  authorization  by  the  state.'*°  In  most  states  a  corporation  cannot 
reduce  its  capital  stock  to  an  amount  less  than  that  of  its  liabilities."" 
Redemption  at  book  value  is  accomplished  by  debiting  capital  stock  for  its 
par  value  and  surplus  for  the  pro  rata  share  in  the  surplus,  and  crediting 
cash."'  Redemption  at  par  affects  only  the  capital  stock  account,  but 
redemption  at  other  than  par  affects  both  the  capital  stock  and  surplus 
accounts.'" 

Audit  of  Capital  Stock — The  charter,  by-laws,  and  minutes  should  be 
examined  by  the  auditor  for  information  relative  to  authorized  capital, 
method  of  payment,  and  all  other  provisions  governing  the  issuance  of 
stock.""  A  complete  record  of  the  authorized  and  issued  stock  should  be 
made.'** 

The  stock  certificate  book  should  be  examined  and  reconciled  with  the 
stockholders'  ledger,""  all  canceled  certificates  being  inspected  or  accounted 


138  C.  P.  A.  ACCOUNTING 

for.""  The  uncanceled  certificates  repiesent  the  outstanding  stock.^"'  An 
auditor  is  not  required,  as  a  general  thing,  to  examine  the  transfers."'  He 
should,  however,  obtain  a  certificate  from  the  transfer  agent  as  to  the 
capital  stock  outstanding.'** 

A  trial  balance  should  be  taken  of  the  stockholders'  ledger  to  see  that 
the  aggregate  outstanding  stock  agi-ees  with  the  general  ledger  account.^"" 

The  auditor  should  see  that  the  certificates  of  treasury  stock  have  been 
endorsed  over  to  the  corporation  or  its  trustee,  and,  if  the  treasury  stock 
was  donated,  the  intent  of  the  donors  should  be  ascertained.*"'  The  pro- 
ceeds of  stock  sold  during  the  period  should  be  verified."'" 

The  auditor  must  assure  himself  that  all  issued  shai-es  have  been  fully 
paid,""  but  he  is  not  called  upon  to  challenge  the  good  faith  of  the  di- 
rectors.^"* If  stock  has  been  sold  on  the  installment  plan,  he  shoidd  ascer- 
tain whether  or  not  the  calls  have  been  promptly  met."** 

Stock  pledged  as  collateral  should  be  properly  designated  and  shown 
on  the  balance  sheet  as  a  deduction  from  the  outstanding  stock.""* 

Organization  Expense — Organization  expenses  are  those  costs  neces- 
sarily incurred  for  the  purpose  of  getting  a  corporation  started.^*"  Some 
accountants  stress  the  fact  that  a  corpoi-ation  organized  and  ready  to 
commence  business  is  in  a  better  position  than  an  unorganized  company, 
and  the  fact  that  the  value  of  these  costs  will  last  as  long  as  the  corporate 
existence.""  They  therefore  classify  organization  expense  as  an  intangible 
asset.""  Other  accountants,  while  admitting  these  two  facts,  stress  the 
need  of  keeping  the  balance  sheet  free  from  assets  which  have  no  salable 
value,  and  therefore  regard  organization  expense  as  a  deferred  charge  to 
income,  which  should  be  Avritten  off  in  about  five  years.""  A  few  account- 
ants are  still  more  conservative  and  advocate  charging  otf  organization 
expenses  as  they  are  incurred,  or,  at  most,  over  tlie  first  two  years'  opera- 
tion."' Since  organization  expense  is  an  unsalable  asset,  it  should  be 
written  off,  but  it  would  be  better  to  write  it  off  in  five  years  than  in  two, 
as  this  would  not  reduce  the  dividends  so  severely.  Organization  expense 
written  off  is  sometimes  ti'eated  as  a  financial  expense'"  and  sometimes  as 
an  administrative  expense,'"  but  it  seems  preferable  to  charge  it  directly 
to  surplus.'"*  As  organization  expense  should  be  written  off  directly 
against  surplus,  it  should  be  shown  on  the  balance  sheet  as  a  deduction 
from  the  net  worth  accounts. 

Investments  in  Stocks — Sometimes  stock  investments  are  listed  under  a 
special  caption,"'  but  temporary  stock  investments,  or  stocks  of  ready 
marketability  in  which  current  funds  are  tied  up  temporarily,'"  may  be 
classified  as  current  assets,""'  and  permanent  stock  investments,  or  stocks 
representing  control  in  affiliated  or  subsidiary  companies,"*  may  be  classi- 
fied as  fixed  assets."* 

Temporary  investments  in  stocks  should  be  valued,  for  balance  sheet 
purposes,  at  cost  or  market,  whichever  is  the  lower.""  Many  accountants 
advocate  valuing  them  at  market,  regardless  of  cost;'"  indeed,  insurance 
companies  are  legally  compelled  to  do  so.^"  It  is,  however,  j>referable,  in 
order  to  avoid  bringing  unrealized  profits  into  the  cui-rent  period,  to  value 


CORPORATION  STOCK  139 

them  at  cost  or  market,  whichever  is  the  lower.  If  desired,  the  market 
value  may  be  shown  in  a  footnote  to  the  balance  sheet."^ 

As  it  is  undesirable  to  have  the  value  of  the  temporary  investments 
changed  on  the  books  every  time  the  })Ooks  are  closed,  these  stocks  should 
be  booked  at  cost,  and,  whenever  the  market  value  is  less  than  cost,  the 
discrepancy  should  be  shown  in  the  reserve  for  investment  fluctuations 
account  which  would  be  subtracted  fi'om  the  investment  account  on  the 
balance  sheet.'" 

Many  accountants  value  at  cost  the  permanent  investments  in  stocks  of 
other  companies.""  However,  it  is  preferable  to  use  the  actual  value  of  the 
stock  as  shown  by  the  certified  balance  sheet  of  the  affiliated  company."" 

The  dividends  received  on  stock  investments  should  be  entered  as  non- 
operating  income  in  the  profit  and  loss  statement,""  unless  the  stock  is 
booked  at  its  actual  value  as  per  the  certified  balance  sheet  of  the  affiliated 
company,  in  which  case  the  dividend  would  be  credited  to  the  investment 
account  instead  of  to  the  income  account."'  The  auditor  should  list  all  the 
stocks  held  during  the  period  of  the  audit  so  as  to  ascertain  that  all  divi- 
dends have  been  })roperly  recorded."^" 

All  the  stocks  owned  should  be  verified  by  actual  inspection,""  and  a  very 
complete  and  detailed  list  of  them  prepared."*  Securities  out  as  collateral 
should  be  verified  by  correspondence,""  and  those  in  transit  to  the  firm 
under  audit  should  be  checked  upon  their  receipt,  and  the  transmitter 
circularized."^  Stocks  purchased  during  the  pei-iod  of  audit  should  be 
verified  by  correspondence,  by  biokers'  notices,  and  by  inspection."* 


140  C.  P.  A.  ACCOUNTING 


QUESTIONS 
c»rporation  stock 

General 

1.  What  is  a  corporation?     (Mich.,  July,  1906;  Iowa,  Dec,  1918.) 

2.  What  is  a  close  corporalion?     (Iowa,  Dec,  1918.) 

3.  What  are  the  distinguishing:  characteristics  of  the  "corporation"  as 
compared  with  other  forms  of  business  orsranization  ?  (Mich.,  Julv,  1906*; 
A.  I.  of  A.,  Nov.,  1918.) 

4.  State  the  advantages  and  disadvantages  of  conducting  a  mercantile 
business  as  a  corporation  as  compared  with  a  partnership.  (Mich.,  June, 
1913;  Mass.,  Oct.,  1914;  Del.,  June,  1915*;  A.  I.  of  A.,  Nov.,  1918.) 

5.  In  the  case  of  an  important  public  corporation  having  a  number  of 
stockholders  throughout  the  country,  vigorous  action  is  displayed  between 
two  opposing  interests  to  obtain  control  of  the  directorate  and  officers,  ren- 
dering it  important  that  the  proxies  from  the  stoekholdei's  should  be  cor- 
rectly reported  at  the  meeting.  For  this  purpose  you  are  requested  to 
certify  to  the  number  of  proxies  received  in  favor  of  the  respective  in- 
dividuals. State  what  information,  papei-s,  or  records  you  would  require 
to  examine,  and  how  you  would  proceed.  Also  draft  a  form  of  certificate 
you  would  furnish.     (111.,  May,  1914.) 

6.  A  trading  corporation  incorporated  under  the  laws  of  Maine,  whose 
stockholders  are  citizens  of  Massachusetts,  decides  to  reincorporate  under 
the  laws  of  Massachusetts,  (a)  State  a  strong  reason  to  influence  the 
stockholders  in  this  decision,  (h)  Describe  briefly  what  the  minutes  of 
each  corporation  should  contain  to  complete  this  change.  (Mass.,  Oct., 
1914.) 

7.  You  are  called  upon  by  the  organizers  of  a  manufacturing  company 
to  advise  as  to  process  of  securing  charter.  Company  to  have  a  capital  of 
$50,000.  To  be  incorporated  under  the  laws  of  Pennsylvania.  Give 
method  of  procedure.     (Pa.,  May,  1900.) 

8.  A.  B.  has  a  chance  to  buy  49  per  cent  of  the  stock  of  the  Johnson 
Sales  Company.  All  of  the  stock  is  now  owned  by  C.  J.  and  he  will  con- 
tinue to  hold  the  balance  of  51  per  cent.  A.  B.  is  anxious  to  invest,  as  the 
business  is  very  profitable,  but  hesitates  for  fear  that  C.  J.,  who  is  heavily 
involvetl  in  outside  matters,  may  be  forced  to  sell  his  stock,  and  that  the 
new  owners  might  try  to  "freeze  him  out."  C.  J.  has  offered  to  protect 
A.  B.  as  far  as  possible  in  this  respect.  What  would  you  advise  A.  B.  to 
do?    (111.,  May,  1915.) 


CORPORATTON  STOCK  141 

0.  Sketch  the  history  of  corporations.  Define  a  modei-n  corporation 
giving  examples  of  the  various  kinds.  Descrihe  safei^uards  ajjainst  the 
abuse  of  coi-jiorate  poAvers  or  i)rivileges  which  are  in  force  in  countries 
other  than  Florida,  and  give  reasons  for  and  against  the  adoption  of 
similar  safeguards  here.     (Fla.,  July,  1909.) 

10.  What  is  the  principle  underlying  capital  stock  having  no  par  value? 
How  should  it  appear  on  the  Balance  Sheet  of  the  corporation  issuing  it  ? 
How  should  it  appear  on  the  Balance  Sheet  of  the  owner  thereof!  (111., 
Dec,  1918.) 

11.  In  setting  up  the  Balance  Sheet  of  a  corporation  which  has  an 
issue  of  100,000  shares  of  stock  of  no  par  value  but  a  stated  value  of  $5  a 
share  and  an  excess  of  assets  over  liabilities  of  $1,500,000,  how  would  you 
show  the  capital  on  the  balance  sheet?     (A.  I.  of  A.,  Nov.,  1919.) 

12.  What  evidence  would  you  consider  satisfactory  for  the  correctness 
of  the  following  expenditure:  Commissions  on  sale  of  stock?  (N.  C,  Nov., 
1919.) 

13.  What  are  the  underlying  principles  of  corporation  accounting? 
(N.  Y.,  Jan.,  1920.) 

14.  Name  accounts  and  use  of  each  that  are  peculiar  to  corporation 
accounting.     (N.  Y.,  June,  1901*;  Pa.,  Nov.,  1904*;  Mich.,  June,  1914.) 

15.  What  is  the  meaning  of  the  word  "stock"  in  reference  to  a  corpora- 
tion? (a)  What  stock  does  it  represent?  (h)  How  is  it  ti'ansf erred ?  (c) 
What  right  have  stockholders  in  and  to  the  corporate  profit?  (Mich.,  July, 
1906.) 

16.  How  should  the  right  to  remuneration  claimed  by  officers  and  direc- 
tors of  a  corporation  be  verified?     (N.  Y.,  Dec.,  1898.) 

17.  In  an  audit  of  the  books  of  a  company,  it  is  found  that  the  Presi- 
dent's Salai-y  account  is  credited  with  a  bonus  of  $5,000  for  "extra  service," 
in  addition  to  the  usual  salary  paid  him ;  what  steps  should  be  taken  to 
ascertain  that  this  item  is  con-ect?  (N.  Y.,  Dec,  1897;  N.  Y.,  Feb.,  1910*; 
Ind.,  June,  1916.) 

18.  What  would  you  consider  satisfactory  evidence  of  the  correctness 
and  propriety  of  expenditures  of  directors'  remuneration.  (A.  I.  of  A., 
May,  1918;  N.  C,  Nov.,  1919.*) 

19.  In  examining  the  books  of  a  corporation  you  find  that  an  officer  is 
a  partner  in  a  business  from  which  the  corporation  makes  purchases. 
Upon  scrutiny  you  find  that  all  the  transactions  are  at  proper  figures. 
Would  you,  under  such  conditions,  call  attention  to  the  fact  when  making 
your  report  to  the  directors?     (R.  I.,  Dec,  1907.) 

20.  After  having  been  employed  as  auditor  of  a  firm  for  several  years, 
the  partners  advise  you  that  they  have  decided  to  conduct  their  business 
as  a  corporation  and  have  secured  a  charter  under  the  general  laws  of  the 
state  accordingly.  In  Avhat  way  would  your  duties  as  an  auditor  be  affected 
by  such  a  change?     (111.,  Nov.,  1908.) 

21.  The  Best  Store  Company  was  incorporated  for  $50,000  on  March 


142  C.  P.  A.  ACCOUNTING 

15,  1918,  by  the  three  partners,  "A,"  "B,"  and  "C."  The  change  in  or- 
sanization  was  not  iiiven  eflfeet  upon  the  books  of  the  company.  Proprie- 
tary interest  on  Januaiy  1,  1918,  was  $75,000.  The  profits  for  the  year 
1918,  determined  in  January,  1919,  are  found  to  be  $10,000.  Specifically 
state  how  yon  would  correct  this  condition  on  the  books  of  the  Company 
when  you  are  called  in  February,  1919.     (Wis.,  Nov.,  1919.) 

22.  A  company  is  organized  under  the  laws  of  Florida  to  purchase  a 
patent  and  to  manufacture  and  .sell  the  patented  article.  The  patent  is 
jiaid  for  by  the  entire  issue  of  stock,  and  the  patentee  gives  back  to  the 
company  49  per  cent  of  the  entire  stock,  to  be  used  for  developing  pur- 
poses. Describe  the  formalities  which  must  be  complied  with,  and  draw 
the  opening  entries  in  the  books,  stating  what  books  are  required.  (Fla., 
July,  1909.) 

23.  A  corporation  has  a  capital  stock  of  $100,000.  It  ha.s  assets  at 
inventory  value  amounting  to  $160,000.  With  a  view  to  reducing  the 
number  of  its  enterprises,  it  sells  two  of  its  stores  for  $85,000  at  inventory 
value.  This  $85,000  is  distributed  among  its  stockholders.  What  entries 
should  be  made  upon  the  books,  and  what  procedure  would  you  recommend 
in  order  to  safeguard  all  interest  in  making  such  distribution?  (111.,  Nov., 
1904.) 

24.  Give  an  example  of  that  portion  of  the  Balance  Sheet  of  a  corpora- 
tion which  deals  with  the  Share  and  Debenture  Capital  account.  State  and 
set  out  the  same  in  proper  columns,  assuming  the  following  to  be  the 
position  of  the  company's  share  and  debenture  capital:  (a)  share  capital, 
authorized,  $100,000;  (6)  share  capital.  Issued  or  subscribed,  $80,000;  (c) 
share  capital,  called  up,  $60,000;  (d)  calls  paid  in  advance,  $5,000;  (e) 
calls  in  arrear,  $1,000;  (/)  debenture  capital,  authorized,  $50,000;  (g)  de- 
benture capital,  issued  or  subscribed,  $40,000;  (h)  debenture  capital,  paid 
up,  $35,000.     (111.,  Dec..  1910.) 

25.  Name  the  diffei'ent  kinds  of  ])ei*sons  who  would  have  an  interest  in 
audit  reports  prepared  for  a  corporation ;  indicate  what  the  interest  is, 
and  show  the  responsibilitv  of  the  auditor  to  each  class.  (Mich.,  June, 
1914.) 

26.  Formulate,  in  an  imaginary  ease,  the  Journal  entry  or  entries  for 
the  conversion  of  a  partnership  into  a  joint  stock  company  with  the 
same  resources  and  liabilities.  (N.  Y.,  June,  1898;  Wash.,  Mav,  1903; 
Va.,  Nov.,  1918.*) 

27.  State  the  entries  necessary  to  open  a  set  of  corporation  books  so 
that  the  assets  may  appear  properly  on  the  Ledger.  (N.  Y.,  Dec,  1898; 
Pa.,  Nov.,  1903*;  Va.,  May,  1917;  Va.,  Nov.,  1918*;  Iowa,  Dec,  1918.*) 

28.  A  corporation  is  owned  by  four  persons  who  have  an  equal  number 
of  shares.  During  the  operations  of  the  company  each  one  of  the  above 
parties  is  ci-edited  through  the  Cash  Book  at  various  times  with  equal 
amoimts  as  loans  to  the  corporation,  and  the  Cash  Book  is  kept  in  balance 
by  increasing  the  payroll  by  the  amounts  credited.  Each  of  the  parties 
is  credited  also  with  interest  on  undrawn  credit  balances  thus  obtained. 


CORPORATION  STOCK  143 

In  what  manner  does  such  action  aifect  the  results  of  the  business?    Please 
explain  fully.     (Pa.,  Nov.,  1904.) 

29.  A  corporation  has  the  following:  items  in  its  Balance  Sheet :  Ac- 
counts Payable,  Accounts  Receivable,  Cash,  Capital  Stock,  Expense  Ac- 
crued not  due,  Expense  Paid  in  Advance,  Goodwill,  Merchandise,  Machin- 
ery, Notes  Payable,  Patents,  Real  Estate,  Reserve  for  Depreciation  on 
Plant,  Surplus,  Trade  Marks,  and  Treasury  Stock. 

You  are  asked  to  figure  the  value  of  the  stock.  State  which  items  you 
would  take  to  get  the  gross,  and  which  items  you  would  deduct  from  the 
gross  to  get  the  net  amount,  and  how  you  would  obtain  the  value  of  each 
share.  (Mass.,  June,  1913.) 

30.  What  knowledge  must  a  transfer  agent  possess  in  order  to  safe- 
guard his  com)>any  in  the  transfer  of  its  certificates  of  stock?  (N.  Y., 
June,  1913.) 

31.  A  manufacturer  having  turned  his  business  into  a  joint  stock  com- 
pany and  as  yet  owning  all  the  stock  himself — only  a  few  shares  standing 
nominally  on  the  company's  books  as  belonging  to  friends — finds  after  a 
time  that  the  business  needs  more  capital.  Thereupon  he  agrees  to  sell  a 
portion  of  the  stock  to  outside  parties  for  cash,  stipulating  that  the 
money  so  obtained  shall  be  put  into  the  business;  that  is  to  say,  shall  be 
at  once  expended  in  purchasing  new  machinery  and  in  repairs  and  im- 
provements, which  is  done  accordingly.  How  is  the  transaction  properly 
to  be  brought  into  the  company's  accounts?     (Mich.,  June,  1910.) 

Corporate  Books 

32.  List  books  and  give  purpose  of  each,  used  by  a  corporation  but  not 
by  a  partnership.  (N.  Y.,  Dec,  1898*;  Pa.,  Nov.,  1899*;  N.  Y.,  Jan., 
1901;  Pa.,  Nov.,  1901*;  N.  Y.,  Jan.,  1902;  Pa.,  May,  1902;  Md.,  Oct., 
1903*;  Mich.,  July.  1906*;  Mich.,  Dec,  1906*;  Ohio,  Dec,  1908*;  Va., 
Oct.,  1911*;  La.,  May,  1913*;  Mich.,  June,  1913*;  Mich.,  June,  1914; 
W.  Va.,  May,  1917*;  A.  I.  of  A.,  May,  1918.*) 

33.  How  would  you  organize  a  set  of  books  for  a  corporation?  (Ohio, 
Dec,  1908.) 

34.  What  would  be  your  procedure  in  examining  the  capital  stock 
books  of  a  corjioration  ?  What  books  would  you  require?  Give  reasons 
why  such  an  audit  may  be  necessary.     (N.  Y.,  June,  1911.) 

35.  In  auditing  the  accounts  of  a  corporation,  for  the  first  year  of  its 
existence,  what  records  and  documents  should  be  examined  in  addition  to 
the  books  of  accounts  and  the  vouchers?     (Ind.,  May,  1917.) 

36.  In  auditing  the  accounts  of  a  corporation  for  the  first  time,  for 
what  books  and  i-ecords  would  you  ask  and  what  purpose  would  you  expect 
each  to  .serve  in  connection  with  the  audit?  (N.  Y.,  Dec,  1896;  N.  Y., 
Jan.,  1900*;  Wash.,  May,  1903*;  N.  Y.,  Jan.,  1904*;  Cal.,  May,  1908*; 
Mass.,  June,  1910;  Va.,  Nov.,  1910*;  A.  I.  of  A.,  June,  1917*;  Ohio,  Oct., 
1919.*) 


144  C.  P.  A.  ACCOUNTING 

37.  What  records  and  documents  should  an  auditor  have  access  to  in 
an  audit  of  a  corporation?  (N.  Y.,  Dec.,  1898*;  Pa.,  Nov.,  1899*;  Pa., 
May,  1902*;  111.,  Nov.,  1903*;  Mich.,  July,  1906*;  N.  Y.,  June,  1908*; 
Kan.,  May,  1916;  A.  I.  of  A.,  May,  1918*;  Iowa,  Dec.,  1918.*) 

38.  In  the  case  of  a  corporation  which  has  been  in  existence  for  several 
years,  if  you  were  engaged  to  audit  the  accounts  for  the  last  year  only, 
would  you  consider  it  necessary  to  inspect  the  Minute  Book,  Stock  Ledger, 
Bond  Register,  or  any  of  the  entries  in  the  books  of  accounts  at  dates 
prior  to  that  covered  by  your  audit?  Give  reasons.  (N.  Y.,  Jan.,  1906*; 
Ya.,  Nov.,  1910.) 

39f  Describe  the  correct  manner  of  writing  up,  and  keeping  the  fol- 
lowing books:  Stock  Certificate  Book,  Stockholders'  Journal,  Transfer 
Record,  Stockholders'  Ledger.  Sketch  a  simple  form  of  each  of  the  above 
books.     (111.,  Nov.,  1904.) 

40.  Distinguish  between  the  registration  of  stock  certificates  and  the 
registration  of  negotiable  bonds  of  a  corporation  and  explain  fully  the 
purposes  of  such  registrations.     (Md.,  Jan.,  1909.) 

41.  What  is  a  Stock  (or  Shares)  Ledger?  (N.  Y.,  .Jan..  1897*;  N.  Y., 
June,  1900.) 

42.  Describe  how  the  Stock  Ledger  (Shares  Ledger)  of  a  corporation 
is  kept.  (N.  Y.,  Jan.,  1897;  N.  Y.,  Jan.,  1900;  Pa.,  May,  1903*;  N.  Y., 
Jan.,  1906* ;  N.  Y.,  Feb.,  1910 ;  Mass.,  April,  1911* ;  N.  Y.,  June,  1911* ; 
Va.,  Oct.,  1912.*) 

43.  Give  the  ruling  of  a  Stock  or  Shares  Ledger  for  a  corporation. 
(N.  Y.,  Jan.,  1900;  N.  Y.,  Feb.,  1910;  N.  Y.,  June,  1912.*) 

44.  State  the  full  procedure  leading  up  to  the  entry  of  the  following 
transactions  in  the  shares  of  a  corporation,  the  par  value  of  which  is  $100. 
April  5,  1901,  James  Williamson  received  certificate  No.  75  for  100  shares 
full  paid.  May  3,  1901,  James  Williamson  requests  a  transfer  to  Geo.  T. 
Jenkins  of  30  of  his  100  shares.  Outline  a  form  of  Stockholders'  Ledger 
and  properly  enter  the  above  items  therein.     (N.  Y.,  Jan.,  1902.) 

45.  What  relation  does  the  Stock  or  Shares  Ledger  bear  to  the  general 
books  of  the  corporation?  (N.  Y.,  Jan.,  1900;  N.  Y.,  June,  1900;  N.  Y., 
Feb.,  1910.) 

46.  Explain  the  nature  of  the  records  of  a  Stock  (or  Shares)  Ledger, 
and  describe  the  manner  in  which  they  are  made.     (N.  Y.,  June,  1900.) 

47.  What  is  the  purpose  and  method  of  registering  stock  certificates 
and  other  corporate  securities?     (Pa.,  Nov.,  1906.) 

48.  Describe  the  method  of  determining  the  number  of  shares  of  capital 
stock,  both  common  and  preferred,  held  by  each  of  the  several  stockholders 
of  a  corporation.    (N.  Y.,  Jan.,  1906;  N.  Y.,  June,  1911* ;  Va.,  Oct.,  1912.) 

49.  Give  reasons  for  or  against  the  necessity  of  an  auditor's  verifying 
the  Stock  Ledger  of  a  corporation.  (N.  Y.,  Jan.,  1900;  Wash.,  May, 
1903*;  N.  Y.,  Feb.,  1910* ;  N.  Y.,  Jan.,  1918.*) 

50.  How  would  you  audit  the  Stock  Certificates  book  of  a  corporation 


CORPORATION  STOCK  145 

(a)  when  the  corporation  has  no  register,  (h)  when  the  corporation  hav, 
a  register?     (Pa.,  Nov.,  I!)n4;  Md.,  Jan.,  1900.) 

51.  I3i  an  audit  of  the  accounts  of  a  corporation,  should  the  auditor 
accept  as  conclusive  the  certificate  of  the  rej^istrar  of  the  stock  as  to  th« 
total  amount  of  capital  stock  outstandin":'?  If  not,  why  not?  (N.  Y., 
June,  1904.) 

52.  In  making:  an  audit  of  the  accounts  of  a  corporation,  would  you 
consider  it  part  of  your  duty  to  verify  the  transfers  of  the  certiticat&s  of 
capital  stock  occurrins:  during  the  period  covered  bv  vonr  examination? 
(111.,  May,  1908.) 

53.  Submit  rulings  of  Transfer  Journal  suitable  to  record  heavy  trans- 
fers of  a  listed  stock  and  all  necessary  transfer  records  to  be  used  therewith. 
Explain  fully  the  use  of  each  record  and  its  relation  to  the  others.  (N.  Y., 
Jan.,  1911.) 

54.  Describe  the  Minute  Book  and  how  used.     (Pa.,  May,  1903.) 

55.  Do  you  consider  it  necessary,  and  if  so,  why,  for  an  auditor  of  a 
corporation  to  peruse  the  minutes  of  directors'  and  shareholder'  meetings? 
(N.  Y.,  June,  1909;  Wash.,  Nov.,  1913;  Del.,  June,  1915;  N.  Y.,  June, 
1917.*) 

56.  Of  what  use  is  the  Minute  Book  of  a  corporation  to  an  auditor? 
(Pa.,  Nov.,  1899;  Md.,  Oct.,  1903;  N.  Y.,  June,  1909;  Md.,  Jan.,  1909; 
Va.,  Oct.,  1911*;  Del.,  June,  1915*;  Ohio,  Nov.,  1917.) 

57.  What  should  an  auditor  do  in  case  he  is  refused  access  to  the 
Minute  Book?    (N.  Y.,  June,  1909* ;  Del.,  June,  1915;  N.  Y.,  June,  1917.*) 

58.  State  how  you  would  deal  with  the  following  items  in  auditing  a 
cash  book  of  a  large  corporation  :  (a)  Payments  of  an  exceptional  nature, 
such  as  the  expenses  of  officials  visiting  the  corporations  in  England,  a 
large  political  contribution,  and  gifts  to  members  of  the  board  of  directors ; 

(b)  payments  to  directors  for  special  services  in  addition  to  the  ordinary 
and  regular  directors'  fees.     (N.  D.,  July,  1918.) 

59.  What  is  the  value  of  a  Minute  Book  in  opening  the  books  of  "a 
corporation?     Explain  fully.     (Pa.,  Nov.,  1904.) 

60.  Mention  three  classes  of  transactions  the  verification  of  which 
in  a  corporation  audit  would  lead  you  to  refer  to  the  Minute  Books  of  the 
corporation.    Give  reasons  in  each  ease.     (Ohio,  Nov.,  1918.) 

61.  (a)  Would  you  refuse  to  sign  an  audit  certificate  if  you  had  been 
refused  access  to  the  Minute  Book  of  a  corporation? 

(b)  If  the  answer  is  "yes"  would  you  sign  the  certificate  with  a  quali- 
fication ? 

(c)  Mention  five  items  for  which  you  would  look  in  examining  a 
Minute  Book. 

(d)  If  the  corporation  were  a  "close"  one  and  practically  no  minutes 
were  kept,  what  action  would  you  take  and  why?  (A.  I.  of  A.,  May, 
1919.) 

62.  Describe  the  necessary  precautions  to  be  taken  by  a  transfer  agent 


140  C.  P.  A.  ACCOUNTING 

to  safeguard  his  corporation,  when  an  executor  presents  a  certificate  of 
stock  for  transfer.  Outline  entries  which  should  appear  on  the  Transfer 
Books.     (N.  Y.,  Jan.,  1918.) 

63.  To  what  extent  should  the  auditor  of  a  corpoi'ation  check  the  jne- 
fen-ed  and  common  stock  certificate  registers'?     (N.  D.,  July.  1918.) 

(54.  What  method  would  you  adopt  to  prove  the  outstanding  certificates 
of  stock  to  be  correct  as  represented  on  the  Transfer  Ledger?  (N.  Y.. 
June,  1913.) 

Classes  of  Stock 

65.  Define:   Stock.     (Mich.,  June,  1908.) 

66.  Name  the  various  forms  of  capital  stock  and  how  created,  stating 
the  rights  and  privileges  of  each.  (Mich.,  July,  1906*;  Mich.,  Dec,  1906; 
Mich.,  June,  1913.*) 

67.  Define:  Capital  stock.  (X.  Y.,  Dec,  1896;  Pa..  May.  1903;  111., 
Nov.,  1903:  Wash.,  Sept.,  1907;  Ohio,  March,  1910;  La.,  May,  1913*;  Cal,, 
May,  1916.) 

68.  How  would  you  verify  the  liability  of  a  corporation  as  to  capital 
stock?     (III.,  May,  1904;  Ohio,  Dec,  1908.) 

69.  To  what  extent  would  you  consider  it  necessary  to  verify  the 
capital  stock,  and  what  reference  to  such  verification  would  you  make  in 
your  report?     (Mass.,  June,  1913.) 

70.  How  would  you  proceed  to  determine  the  book  value  of  capital 
stock?     (N.  Y.,  June,  1919.) 

71.  State  what  verification  you  would  make  of  stock.  (N.  C,  Aug., 
1917.) 

72.  In  making  a  detailed  audit,  what  procedure  would  you  follow  to 
verify  the  capital  stock  issued?  (N.  Y.,  June,  1909*;  Mo.,  Dec,  1915; 
Kan.^  Dec,  1915;  Ohio,  Nov.,  1916*;  A.  I.  of  A.,  June,  1917*;  Wash., 
July,  1917.*) 

73.  To  what  extent  do  you  think  it  is  necessary  to  veiify  the  outstand- 
ing capital  stock  of  a  corporation?     (A.  I.  of  A.,  Jime,  1917.) 

74.  State  what  course  you  would  advocate  to  prevent  an  over  issue  of 
stock.     (Mass.,  April,  1911.) 

75.  If  called  upon  to  verify  the  outstanding  capital  stock  of  a  cor- 
poration, what  records  would  you  consult?  (Mass.,  April,  1911*;  Ohio, 
Nov.,  1916.) 

76.  Define:  Common  stock.     (N.  Y.,  Dec,  1896;  N.  Y.,  June,  1898* 
N.  Y.,  Dec,  1898;  Wash.,  May,  1903*;  Pa.,  Nov.,  1903;  Mich.,  Nov.,  1907* 
Wash.,  Mav,  1911;  Wash.,  June,  1915;  Cal.,  May,  1916;  N.  D.,  July,  1916* 
W.  Va..  May,  1917;  N.  C,  Nov.,  1918;  S.  C,  Sept.,  1919;  N.  C,  Nov., 
1919;  N.  C,  June,  1920.) 

77.  Define:  Preferred  stock.  (N.  Y.,  Dec,  1896;  N.  Y.,  Dec,  1898; 
Pa.,  May,  1903 ;  Wash.,  May,  1903* ;  Pa.,  Nov.,  1903* ;  Mich.,  Nov.,  1907* ; 
Wash.,  June,  1915;  Cal.,  May,  1916;  N.  D.,  July,  1916*;  W.  Va.,  May, 


CORPORATION  STOCK  147 

1917;  Wash.,  July,  1917;  N.  C,  Nov.,  1918;  S.  C,  Sept.,  1919;  N.  C, 
Nov.,  1919;  N.  C,  June,  1920.) 

78.  How  would  you  verify  the  Preferred  Capital  Stock  account  in  mak- 
ing a  Balance  Sheet  audit?     (Mass.,  Oct.,  1914;  Ohio,  Nov.,  1915.) 

79.  You  are  elected  the  auditor  of  a  corporation  by  the  holders  of  both 
common  and  prefeiTed  stock  atid  it  is  your  duty  to  safeguard  the  interests 
of  both  classes  of  stockholders.  The  preference  stock  bears  7  per  cent, 
and  is  noncumulative.  Mention  what  precautions  you  would  adopt  to 
safejfuard  the  interests  of  the  preference  shareholders,  giving  reasons 
therefor.  (111.,  May,  1907;  Mich.,  June,  1912*;  Wis.,  April,  1914*;  Ind., 
June,  1916*;  Ohio,  Nov.,  1916.*) 

80.  Define:  Cumulative  preferred  stock.  (Wash.,  May,  1911;  Wash., 
Nov.,  1913;  Cal.,  May,  1916;  N.  D.,  July,  1916.*) 

81.  In  its  prospectus  a  corporation  represents  that  it  has  an  issue,  of 
"cumulative,  nonvoting,  nonparticipating,  6  per  cent  preferred  stock." 
(jrive  your  interpretation  of  this  expression.     (Mass.,  Oct.,  1915.) 

82.  Define:  Founders'  stock.     (N.  C,  Nov.,  1918;  N.  C,  Nov., .1919.) 

83.  Define :  Guaranteed  stock.  (N.  C,  Nov.,  1918;  N.  C,  Nov.,  1919; 
N.  C,  June,  1920.) 

84.  Define:  Debenture  stock.  (111.,  May,  1914*;  N.  C,  Nov.,  1918; 
N.  C,  June,  1920.) 

85.  Define:  Share  capital.     (N.  Y.,  Dec.,  1896;  Cal.,  May,  1916.) 

86.  Explain  fully  and  state  how  Authorized  Capital  Stock  account 
should  be  carried  upon  the  books.     (Mich.,  Nov.,  1907.) 

87.  What  is  "watered  stock"?  (N.  Y.,  Dec.,  1898;  Mich.,  Dec,  1906; 
Wash.,  May,  1911;  Mich.,  June,  1914*;  Wash.,  June,  1915;  Cal.,  Nov., 
1916;  W.  Va.,  May,  1917;  Cal.,  June,  1917;  Mich.,  June,  1919.) 

88.  How  should  watered  stock  appear  on  the  books?  (Mich.,  Dec, 
1906;  Cal.,  June,  1917.) 

89.  How  should  watered  stock  be  treated  in  audit  reports?  (Mich., 
June,  1919.) 

90.  A  company  Avhose  stock  is  widely  distributed  and  much  dealt  in, 
increases  its  capital  stock  of  $500,000  by  a  stock  dividend  of  100  per  cent. 
Some  years  subsequently  an  original  stockholder  brings  suit  for  elimination 
from  the  capital  stock  of  what  he  claims  is  "water."  How  can  the  stock 
issued  as  dividend  be  eliminated  from  the  $1,000,000  of  stock  outstanding? 
(111.,  May,  1905.) 

91.  How  do  you  determine  the  amount  of  watered  stock?  (Cal.,  June, 
1917.) 

92.  Define:  Donated  Stock.     (Mass.,  Oct.,  1917.) 

93.  How  would  donated  stock  affect  the  book  value  of  capital  stock? 
(Mass.,  Oct.,  1917.) 

94.  State  how  you  would  show  capital  stock  donated  to  the  company 
on  the  Balance  Sheet.     (Ohio,  Nov.,  1917.) 


148  C.  P.  A.  ACCOUNTING 

95.  Mention  and  explain  two  common  views  concerninj;:  tlie  trcalineiit 
of  donated  capital  stock.    (A.  I.  of  A.,  Nov.,  J918;  Pa.,  Nov.,  1919.) 

96.  Does  unsubscribed  stock  in  a  corporation  constitute  a  liability  f  If 
it  does,  under  what  account  would  it  appear  in  a  ledprer?  (Mich.,  Juno, 
1908.) 

97.  Say  how  you  would  show  capital  stock  authorized  but  not  sub- 
scribed on  the  Balance  Sheet.     (Ohio,  Nov.,  1917.) 

98.  How  would  you  treat  in  the  accounts  bonus  stock  Issued  with 
bonds?     (Cal.,  Nov.,  1916.) 

99.  A  corporation  has  two  classes  of  stock  fully  issued:  $5,000,000 — 7 
per  cent  cumulative  preferred  a.s  to  dividends  and  assets,  10  per  cent  divi- 
dends are  in  arrears;  $12,000,000 — common,  on  which  no  dividend  has  been 
paid.  The  corporation  proposes  to  retire  by  purchase  $2,000,000  com- 
mon. What  would  be  the  effect,  if  any,  on  the  interests  of  the  preferred 
stockholders'?  Give  reasons  supporting?  your  answer.  (Mass.,  April, 
19n.) 

100.  A  corporation  is  fonned  whose  capital  stock  has  no  par  value. 
Shares  issued,  1,000;  as.sets,  $8,000;  liabilities,  $4,000.  Prepare  the 
Journal  entries  to  open  the  books.     (N.  Y.,  Jan.,  1919.) 

101.  Differentiate  unsubscribed  stock,  unis.sued  stock,  issued  stock  and 
treasury  stock.     (N.  Y.,  Jan.,  1916.) 

Treasury  Stock 

102.  Define :  Treasury  stock.  (N.  Y.,  Jan.,  1902*  ;  Pa.,  May,  1903» ;  111., 
Nov.,  1904*;  N.  D.,  June,  1914;  Wa.sh.,  June,  1915;  Wis.,  Mav,  1916;  Cal., 
May,  1916;  Pa.,  Nov.,  1917;  N.  Y.,  June,  1918;  Va.,  Nov.,  1918;  N.  C, 
Nov.,  1918;  Iowa,  Dec.,  1918;  A.  I.  of  A.,  May,  1919;  N.  C,  June,  1919; 
N.  C,  Sept.,  1919';  N.  C,  Nov.,  1919;  N.  C,  June,  1920.) 

103.  "Treasury  stock  or  bonds  are  merely  so  many  legalized  pieces  of 
paper,  and  cannot  in  any  sense  be  considered  as  assets  of  the  corporation 
creating  and  issuing  them."     (Dickinson.)     Defend.     (Kan.,  May,  1916.) 

104.  Discuss  briefly  the  following  statement:  "Treasury  stock  is  unis- 
sued capital  stock."    (Cal.,  Nov.,  1916.) 

105.  In  your  opinion,  what  is  the  correct  accounting  procedure  in 
cases  where  an  incorporated  company  sells  its  treasury  stock  at  premium? 
(Wash.,  June,  1912.) 

106.  A  corporation  having  issued  its  capital  stock  at  par  buys  1,000 
shares  at  95.  It  later  sells  500  of  these  shares  at  98,  and  300  at  85,  and 
200  at  101.  Give  the  journal  entries  covering  these  transactions.  How 
should  the  items  appear  cm  the  Balance  Sheet  inuiiediately  after  purchas- 
ing the  stock,  and  inunediately  after  each  of  the  sales?  (A.  I  of  4.,  June, 
1917.) 

107.  On  which  side  of  the  ledger  should  the  balance  of  the  treasury 
stock  account  appear?     (Mo.,  Dec,  1914.) 


CORPORATION  STOCK  149 

lOS.  Explain  fully  and  state  how  accounts  should  be  carried  on  tb« 
books  with  treasury  stock.     (Mieh.,  Nov.,  1907;  Mich.,  June,  1915.*) 

109.  How  would  you  show  treasury  stock  on  a  Balance  Sheet  t  If  your 
answer  is  "as  an  asset,"  state  your  reason.  (Cal.,  May,  1916;  Pa.,  Nov., 
1917*;  N.  C,  Nov.,  1918*;  N.  C,  June,  1919;  N.  C,  si^t.,  1919.) 

110.  (a)  How  would  you  deal  in  the  Balance  Sheet  of  a  corporation 
with  shares  recovered  from  a  vendor  to  whom  they  had  been  issued  as 
fully  paid  and  who  had  returned  them  in  s^lement  of  a  claim  for 
fraudulent  misrepresentation  in  respect  of  the  property  sold  by  him  to  the 
corporation  ? 

(h)  How  would  yon  deal  with  these  shares  for  the  purposes  of  a 
dividend? 

(A.  L  of  A.,  June,  1917;  Ind.,  Nov.,  1917.*) 

111.  A  corporation  is  organized  with  an  authorized  capital  stock  of 
$50,000  of  which  only  $10,000  is  sold,  and  stock  certificates  issued  therefor. 
Two  conflicting:  methods  of  recordingr  the  capital  stock  on  the  books  are 
urged  by  rival  accountants  as  follows:  (a)  Treasury  stock  to  capital  stock 
$50,000,  cash  and  properties  to  treasury  stock  $40,000;  (6)  cash  proper- 
ties to  capital  stock  $40,000.  Which  method  is  the  better  and  why? 
(N.  Y.,  Jan.,  1906;  Cal.,  June,  1917.) 

112.  A  company  has  acquired  at  $90  per  share  100  shares  of  its  own 
jpital  stock,  of  the  par  value  of  $100  per  share.     Its  Balance  Sheet 

shows  treasury  stock  $9,000.  Is  this  correct?  If  so,  why?  If  not,  state 
how  vou  would  adjust  the  books.  (N.  Y.,  June.  1912;  Mich.,  Dec,  1913; 
Ohio,'  Nov.,  1917;  N.  Y.,  June,  19ia*) 

113.  What  is  involved  in  the  verification  of  treasury  stock?  (Cal., 
May.  1916:  Mich.,  Dec.,  1916.*) 

114.  It  frequently  happens  that  a  corporation  contracts  to  purchase 
property  at  an  agreed  price,  which  on  the  face  of  the  contract  is  declared 
to  be  its  value,  and  that  by  another  clause  in  the  contract,  or  by  another 
contract,  the  vendors  agree  to  provide  in  addition  to  the  property,  a  cer- 
tain sum  for  working  capital  or  even  for  free  surplus. 

It  is  sometimes  maintained  that  this  free  sum  so  provided  is  a  profit  or 
surplus  of  the  new  corjxjration  available  for  payment  of  dividends  if  the 
directors  so  determine. 

Write  a  brief  expression  of  your  opinion  as  to  the  proper  treatment  of 
the  sum  turned  back. 

(A.  I.  of  A.,  May,  1919.) 

115.  Prepare  Journal  eitry  for  retiring  treasury  stock  of  the  par  value 
of  $100  acquired  at  $50  and  the  book  value  of  $125.     (N.  C,  June.  1920.) 

116.  A  company  with  an  authorized  capital  stock  of  $3,000,000,  $100 
par  value,  issues  $2,400,000  of  shares  in  pajTuent  of  various  properties. 
In  order  to  secure  working  capital  the  shareholders  return  to  the  eom- 
1  any  three  eightlis  of  their  holdings  to  be  sold  at  $50,  and  on  the  same 
day  3.000  shares  are  so  sold  and  |>aid  for.  How  would  you  treat  this 
matter? 


150  C.  P.  A.  ACCOUNTING 

(a)     Draft  entries  and  show  Ledger  accounts  and  balances. 
(6)     After  all  the  treasury  stock  has  been  sold,  state  what  should  be 
done  as  to  entries,  and  the  resultant  effect  upon  property  account. 

(Mich.,  Dec,  1916*;  Okla.,  Nov.,  1919.) 

117.  A  financing  corporation  which  had  paid  $450,000  for  six  patents 
of  equal  value,  sold  one  of  these  patents  during  the  first  year  of  its  exist- 
ence and  received  as  the  consideration  for  the  sale  1,500  shares  of  pre- 
ferred stock  (par  value  $100)  in  a  subsidiary  company  organized  for  the 
purpose  of  working  the  patent.  During  the  second  year  of  its  life  the 
financing  corporation  sold  the  1,500  shares  of  preferred  stock  for  $100,000. 
State  how  you  would  treat  the  accounts  in  respect  to  these  tw'o  transactions 
in  the  financing  corporation  at  the  end  of  the  first  and  second  years 
respectively.      (N.  D.,  July,  1918.) 

Organization  Expense 

118.  What  are  organization  expenses?  (Wash.,  Nov.,  1913;  A.  I.  of 
A.,  June,  1917;  Ind.,  May,  1918.) 

119.  What  class  of  expenditures  do  you  consider  properly  belong  to 
"organization  expense  account,"  on  the  books  of  a  corporation?  Give 
examples  and  reasons.     (N.  Y.,  Jan.,  1919.) 

120.  How  should  organization  expense  be  treated  on  the  books  of  a  cor- 
poration? (N.  Y.,  Jan.,  1904*:  N.  Y.,  Jan.,  1906* ;  Mass.,  June,  1910; 
Va.,  Nov.,  1910*;  Mich.,  June,  1913* ;  Colo.,  Dec,  1913;  A.  I.  of  A., 
June,  1917;  Mass.,  Oct..  1917*;  Ohio,  Nov.,  1917;  Ind.,  May,  1918*;  Va., 
Nov.,  1918;  Iowa,  Dec,  1918*;  A.  I.  of  A.,  May,  1920*.) 

121.  At  what  point  do  expenses  cease  to  be  organization  expenses  and 
become  operating  expenses?     (A.  I.  of  A.,  June,  1917;  Ind.,  May,  1918.) 

122.  What  is  usually  included  in  the  account,  Organization  Expenses,  in 
the  books  of  a  company?     (Mass.,  June,  1910;  Colo.,  Dec,  1913.) 

123.  Upon  what  basis  should  organization  expense,  carried  as  a  de- 
ferred asset,  be  valued?     (Cal.,  Nov.,  1916.) 

124.  Classify  the  accounts  properly  recording  the  following  items  ac- 
cording to  the  subdivision  of  assets,  liabilities,  proprietary  interest,  in- 
come and  expenses  under  which  it  should  be  grouped,  (a)  organization  ex- 
pense,  (b)   organization  expenses  written  off.     (Wis.,  May,  1919.) 

125.  A  pei-son  is  interested  in  the  profits  of  a  corporation,  but  is  not 
a  shareholder  therein.  He  objects  to  having  the  preliminary  expenses 
enter  into  the  Profit  and  Loss  accounts.  Is  his  position  tenable?  Give 
reasons.     (R.  I.,  Dec,  1907.) 

Subscriptions 

120.    Define:   Subscription.     (N.  D.,  June,   1914.) 

127.  Describe  the  nature  of  the  Subscription  account.  (N.  Y.,  Dec, 
1S96;  N.  Y.,  June,  1899;  Wash.,  Aug.,  1908;  Ya.,  Oct.,  1912*;  W.  Va., 
May,  1917.) 


CORPORATION  STOCK  151 

128.  On  which  side  of  the  ledj^er  should  the  balance  of  the  Capital 
Stock  Subscription  account  appear?     (Mo.,  Dec,  1914.) 

129.  Explain  fully  and  state  how  accounts  should  be  carried  upon  the 
books  with  unsubscribed  stock.     (Mich.,  Nov.,  1907.) 

130.  Of  the  authorized  capital  stock  of  $100,000  one-half,  or  $50,000 
has  been  subscribed  and  only  $25,000  has  been  paid  in  in  subscriptions. 
Submit  the  Balance  Sheet,     (Cal.,  May,  1916.) 

131.  Define:   Stock  right.     (N.  Y.,  Jan.,  1911;  Kan.,  May,  1916.) 

132.  (a)  State  the  purposes  for  which  subscription  privileges  or 
"rights"  may  be  given  stockholders,  (b)  How  may  a  stockholder  use  the 
"right'"?  (c)  What  is  the  value  of  a  "right"  in  the  following  case:  The 
par  value  of  the  outstanding  capital  stock  of  a  corporation  is  $1,000,000; 
market  value,  $150  per  share.  The  stockholders  of  a  certain  date  are  of- 
fered $500,000  more  of  this  same  class  of  stock  at  $125.  (Wis.,  May, 
1919.) 

133.  "B"  contracts  for  $5,000  of  capital  stock  in  the  Stone  Manu- 
facturing Company,  which  is  to  be  paid  for  in  ten  equal  installments.  He 
fails  to  pay  the  last  installment  and  forfeits  the  stock  to  the  corporation 
who  sells  it  for  $3,800.  The  cost  to  the  corporation  was  $50.  Journalize 
the  transaction.     (N.  D.,  June,  1914.) 

134.  If  you  found  a  Journal  entry  in  the  General  Journal  of  a  cor- 
poration cancelling  an  obligation  due  from  subscriber  to  capital  stock,  for 
his  subscription  to  capital  stock,  what  evidence  would  you  require  in 
support  of  such  entrj^?     (N.  Y.,  Jan.,  1920.) 

135.  How  should  money  received  on  account  of  stock  subscriptions  and 
forfeited  by  nonpayment  of  installments  as  they  mature,  be  treated  on  the 
books  of  the  corporation"?     (N.  Y.,  June,  1906;  Fla.,  July,  1909.*) 

136.  A  corj)oration  is  organized  under  the  laAvs  of  this  state,  with  an 
authorized  capital  of  $50,000  divided  into  shares  having  a  par  value  of 
$100  each.  Six  men  agree  to  subscribe  to  10  shares  each.  Omitting  the 
explanations  that  should  accompany  original  entiies,  draft  three  types  of 
opening  entries  for  the  corporation  and  point  out  which  one  you  would 
favor.     Give  reasons.     (N.  Y.,  Jan.,  1917.) 

137.  A  corpoT'ation  was  duly  authorized  to  do  business  in  this  state, 
capitalized  at  $100,000,  which  amount  was  fully  subscribed  for  before  in- 
corporating. Only  50  per  cent  of  that  amount  was  called  in,  viz.,  $50,000; 
and,  after  completing  the  first  year's  opeiations,  it  was  decided  that  it 
would  be  unnecessary  to  call  in  the  balance  of  the  amount  subscribed  for. 

At  a  general  meeting  of  the  membeis  of  the  corporation,  a  resolution  was 
passed  unanimously,  and  recorded  in  the  minutes,  reducing  the  capital  to 
$50,000,  and  authorizing  and  directing  the  bookkeeper  to  clear  off  the  bal- 
ance of  $50,000  standing  at  the  debit  of  Subscription  account. 

As  auditor,  would  you  consider  that  those  enti-ies  called  for  comment? 
If  so,  state  fully  for  what  reason. 

(Wash.,    April,    1906.) 


152  C.  P.  A.  ACCOUNTING 

Stock  Discount  and  Premium 

138.  How  should  the  losses  on  shares  of  stock  issued  at  a  discount  be 
dealt  with  in  the  accounts  of  a  corporation?  (N.  Y.,  June,  1898;  AVash., 
May,  1903;  Ind.,  June,  1916;  Va.,  Nov.,  1918.) 

139.  How  should  stock  sold  at  a  premium  be  treated  in  the  books  of  a 
corporation?  Give  reasons  for  your  answer.  (N.  Y.,  June,  1901*;  R.  I., 
Dec,  1907*;  Wash.,  May,  1910;  Ohio,  Nov.,  1917*;  Ohio,  Nov.,  1918*; 
N.  Y.,  Jan.,  1919.*) 

140.  May  premiums  received  on  account  of  sales  of  capital  stock  be 
applied  against  discounts  of  former  years  ?     (Cal.,  Nov.,  1916.) 

141.  How  should  the  discount  and  premium  arisinjr  from  the  sale  of  a 
company's  own  securities  held  in  its  treasury  be  treated  on  the  books? 
Give  examples.     (N.  Y.,  June,  1902.) 

142.  Is  it  proper  for  a  corporation  to  pay  a  dividend  out  of  surplus 
arising  from  the  sale  of  treasury  stock  at  a  premium?  Whv?  (N.  D., 
July,  1918* ;  Ohio,  Nov.,  1918* ;  N.  Y.,  Jan.,  1919.) 

143.  A  corporation  increases  its  capital  stock,  which  it  sells  at  auction, 
receiving  therefor,  as  premiums  above  the  par  value,  $3,000.  The  treasurer 
credits  this  amount  to  Profit  and  Loss  account  and,  in  his  statement,  shows 
it  as  a  part  of  the  profits.  State:  (a)  your  opinion  thereof:  (b)  to  what 
account,  if  other  than  Profit  and  Loss,  the  amount  should  be  credited;  (c) 
how  it  should  be  shown  in  the'treasurer's  statement;  and  (d)  your  reasons. 
(Mass.,  April,  1911.) 

144.  What,  in  your  opinion,  is  the  coriect  method  of  recording  on  its 
books  of  account  the  purchase  of  property  and  plant  by  a  corporation 
where  payment  is  made  in  capital  stock  of  the  purchasing  company,  the 
par  value  of  such  stock  being  greatly  in  excess  of  the  actual  value  of  the 
assets  acquired?     (N.  Y.,  June,  1918.) 

Securities 

145.  Under  what  circumstances,  if  any,  would  you  examine  the  securi- 
ties owned  by  a  corporation,  and,  if  you  think  this  course  unnecessary, 
give  your  reasons  why,  and  should  you  think  it  necessary,  state  how  you 
would  proceed  in  the  event  the  securities  are  not  in  the  company's  posses- 
sion.    (111.,  May,  1913:  Ohio,  Nov.,  1915.) 

146.  How  would  you.  as  auditor  of  an  incorporated  corai)any,  satisfy 
yourself  of  the  existence  of  shares  or  bonds  given  as  security  for  loans  to 
the  company?     '(Wash.,  Aug.,  1908.) 

147.  In  examining  se<'uri(ie'^  what  data  .should  be  recorded  to  protect 
the  auditor?  What  is  to  be  apprehended?  (N.  Y.,  Jan.,  1914;  Ohio, 
Nov.,  1915;  Ohio,  N«v.,  1916.*) 

148.  What  precautions  should  be  taken  to  avoid  substitution  or  other 
xnanipulation  during  the  inspection  of  securities?     (111.,  Dec,  1918.) 


CORPORATION  STOCK  153 

149.  How  may  the  correctness  of  securities  in  a  Balance  Sheet  be  de- 
termined? (N.  Y.,  June,  1897*;  N.  Y.,  Dec,  1897*;  N.  Y.,  Jan.,  1902*; 
N.  Y.,  Jan.,  1906* ;  N.  Y.,  June,  1906* ;  N.  Y.,  Feb.,  1908* ;  Ohio,  March, 
1910*;  Mass.,  June,  1910;  Mich.,  June,  1912*;  Kan.,  May,  1916;  N.  C, 
June,  1916;  Mass.,  Oct.,  1916*;  Ind.,  Nov.,  KIT*;  111.,  Dec,  1918.*) 

150.  To  what  extent  are  you  justified  in  accepting  the  certificates  of 
officials  of  a  corporation  as  to  bonds  and  stocks  of  outside  companies 
owned?     (Ind.,  Nov.,  1917.) 

151.  How  would  you  record  on  the  books  of  account  the  purchase  of 
securities  which  had  been  only  partially  j^aid  for?     (N.  Y.,  June,  1919.) 

152.  In  the  course  of  an  audit  you  find  the  following  classes  of  security : 
(1)  Real  estate,  (2)  mortgages  on  real  estate,  (3)  chattel  mortgages,  (4) 
stocks  and  bonds  held  as  collater-al  security.  For  each  of  these  classes 
state  (a)  the  method  of  examination,  (b)  special  points  to  be  examined, 
(c)  form  of  report.     (Fla.,  July,  1909.) 

153.  A  manufacturer  makes  extensive  investments  in  stocks  and  bonds, 
buying  and  selling  from  time  to  time  as  the  market  conditions  warrant  and 
clearing  all  such  transactions  through  his  regular  books  of  account.  How 
should  such  transactions  be  isolated  from  his  manufacturing  operations 
and  what  books  and  accounts  should  he  employ  to  record  the  details  of 
the  principal  and  income  from  such  investments?     (N.  Y.,  Jan.,  1906.) 

154.  Your  vei'ification  of  the  securities  of  a  corporation  has  to  be  made 
at  a  date  about  two  months  subsequent  to  the  date  of  the  Balance  Sheet 
you  are  asked  to  certify.  Can  you  suggest  steps  which  will  enable  you  to 
do  this  without  risk  of  overlooking  serious  overstatement?  (A.  I.  of  A., 
Nov.,  1918;  III.,  Dec,  1918.) 

155.  In  making  a  detailed  audit  what  procedure  would  you  follow  to 
verify  the  proceeds  from  sale  of  marketable  investment?  (Kan,,  Dec, 
1915;  Mo.,  Dec,  1915.) 

156.  In  prei)aring  a  Balance  Sheet  of  a  corporation  how  would  you 
classify  or  deal  with  securities  representing  (a)  an  interest  in  a  competing 
company,  (6)  the  entire  ownership  of  a  plant,  (c)  an  investment  of  a 
temporary  surplus  of  cash?     (A.  I.  of  A.,  June,  1917.) 

157.  Classify  the  Income  from  Stocks  and  Bonds  Owned  account  prop- 
erly, according  to  the  subdivision  of  assets,  liabilities,  proprietary  interest, 
income  and  expenses  under  which  it  should  be  grouped.  (Wis.,  May, 
1919.) 

158.  A  concern  has  an  authorized  issue  of  bonds  to  the  amount  of 
$100,000;  $40,000  are  sold  at  par,  $10,000  are  sold  at  10  per  cent  pre- 
mium, $30,000  are  put  up  as  collateral  to  a  $25,000  loan  at  the  bank,  and 
$20,000  are  on  hand.  Pre])are  a  Balance  Sheet  showing  the  above  trans- 
actions, supplying  the  other  needed  accounts.  (N.  Y.,  June,  1908* ;  Mich., 
June,  1910*;  "n.  Y.,  Jan.,  1914*;  Mich.,  June,  1914;  Md.,  Oct.,  1919.) 

159.  The  Oak  Furniture  Company  i)laced  $50,000  of  its  undivided  earn- 
ings in  the  hands  of  a  broker  to  invest  in  United  States  4  per  cent  bonds. 
The  bonds  were  for  $1,000  each  and  cost  101%,  commission  Vs-    Prepare 


154  C.  P.  A.  ACCOUNTING 

detailed  entries  to  record  properly  the  transacticn  of  the  company's  books. 
(N.  Y.,  June,  1915.) 

160.  The  Bristol  Manufacturing  Company  issued  and  sold  on  the  1st  of 
January,  1911,  to  "A"  and  "B"  (50  to  each  at  the  same  price),  first  mort- 
gage bonds  of  $500  each,  bearing  interest  at  1  per  cent  per  annum,  and 
received  $4S,000  in  cash.  What  records  of  the  transactions  should  be 
made,  and  in  what  books?     (111.,  May,  1911.) 

161.  An  investment  company  purchased  for  investment  $100,000  of  6 
per  cent  10-year  municipal  debentures  at  96,  and  $200,000  of  5  per  cent 
industrial  bonds,  15  j-ears  to  run,  at  104.  How  would  you  treat  the  dis- 
count and  the  premium  in  the  accounts?  Give  the  Journal  entries.  (Wis., 
April,  1914*;  Wash.,  June,  1915.) 

162.  In  auditing  the  books  of  a  corporation  you  find  record  of  the 
ownership  of  stocks  and  bonds,  some  of  which  are  in  hand,  some  are 
deposited  with  bankers  or  others  for  safekeeping,  and  other?  are  lodged 
as  security  for  loans.  State  what  kind  of  evidence  you  would  require  in 
each  case,  specifying  particularly  in  the  case  of  stocks  and  registeretl 
bonds;  if  not  registered  in  the  name  of  the  corporation,  what  would  you 
consider  necessary  to  protect  your  client's  interests?  (A.  I.  of  A.,  May, 
1920.) 

163.  What  means  should  be  employed  to  veiify  the  value  of  investment 
securities?  (N.  Y.,  Dec.,  1896*;  N.  Y.,  June,  1898*;  N.  Y.,  Dec.,  1898*; 
Pa.,  May,  1900*;  N.  Y.,  Jan.,  1907;  N.  Y.,  June,  1909*;  Ohio,  March, 
1910*;  N.  Y.,  Jan.,  1911*;  Va.,  Oct.,  1912*;  Cal.,  May,  1916*;  Mass.,  Oct., 
1917.*) 

164.  What  procedure  would  you  follow  in  detennining  the  connect 
valuation,  for  Balance  Sheet  purposes,  of  investment  securities  purchased 
at  a  premium  or  at  a  discount?     (Ohio,  Oct.,  1919.)  "■ 

165.  Finance  corporations  holding  a  large  number  of  shares  in  other 
corporations  are  in  the  habit  of  valuing  their  securities  for  Balance  Sheet 
purposes  at  either  (a)  cost  price,  or  (6)  market  price  at  the  date  of  the 
balance  sheet.  Discuss  the  respective  merits  of  the  two  methods  and  say 
which  you  consider  the  soundest  from  an  accountant's  point  of  view.  (111., 
May,  1905.) 

166.  Discuss  generally  the  duty  of  an  auditor  in  relation  to  the  question 
of  depreciation  of  outside  investments  of  fluctuating  value.  (Mich.,  June, 
1913*;  Wash.,  Nov.,  1913;  Mass.,  Oct.,  1917.*) 

167.  A  corporation  formed  to  invest  in  certain  classes  of  securities  has 
made  a  serious  loss  on  paper,  by  a  fall  in  the  ])rice  of  some  of  its  pur- 
chases, while  it  has  earned  enough  on  income  to  pay  the  usual  dividend. 
How  should  this  be  dealt  with  in  the  annual  accounts?  (111.,  May,  1910; 
Mich.,  June,  1913.*) 

168.  Where  would  you  place  the  Appreciation  of  Securities  account 
in  the  Income  and  Profit  and  Loss  account?     (Md.,  Dec,  1917.) 

169.  How  would  you  classify  investments  in  stocks  in  preparing  a 


CORPORATION  STOCK  155 

certificate  of  condition,  to  be  filed  with  the  Secretary  of  the  Common- 
wealth?    (Mass.,  April,  1911.) 

170.  A  corporation  has  a  number  of  investments  in  stocks  and  bonds 
wliich  are  listed,  and  have  a  definite  market  price  from  day  to  day.  It 
carries  them  at  their  cost  prices  in  the  Ledger  and  wishes  to  retain  these 
cost  prices,  but  at  the  same  time  wishes  to  have  them  show  in  the  Balance 
Sheet  at  the  market  prices.  State  a  convenient  method  of  doing  this  with- 
out changing  the  cost  values  in  the  Ledger.     (Matis.,  June,  1913.) 

171.  A.  B.  is  a  heavy  speculator  on  margin  on  the  stock  exchange, 
buying  and  selling  in  numerous  transactions  a  large  variety  of  seciu-ities. 
His  broker,  C.  D.,  renders  him  monthly  a  statement  of  transactions  and 
an  account  current.  State  the  steps  you  would  take  to  completely  audit 
C.  D.'s  monthly  account  cui'rent  and  statement.     (Mass.,  June,  1913.) 

172.  In  consolidating  two  banks,  at  what  value  should  the  bond  invest- 
ments be  set  up?     (Mich.,  June,  1919.) 

173.  In  auditing  the  accounts  of  a  large  corporation  you  find  an 
account  with  Liberty  Bonds,  charged  with  $200,000.00,  representing  the 
cost  of  bonds  subscribed  and  paid  for  by  the  company.  At  the  date  of 
the  balance  sheet  to  which  you  are  to  certify,  the  bonds  had  a  market 
value  of  $187,500.00.  What  attitude  would  you  take  as  to  their  valuation 
in  the  balance  sheet?     (A.  I.  of  A.,  Nov.,  1920.) 

174.  What  steps  should  be  taken  in  the  most  complete  possible  verifica- 
tion of  the  capital  stock  account  of  a  corporation  in  an  audit  for  a  year, 
if  during  that  year  the  authorized  capital  was  increased  and  new  stock 
was  issued,  and  certain  shares  were  acquired  and  held  by  the  company 
(assuming  that  it  was  legal  to  do  so),  and  if  (a)  there  is  a  registrar  of  the 
stock  and  (b)  there  is  not?     (A.  I.  of  A.,  Nov.,  1920.) 


156  C.  P.  A.  ACCOUNTING 


PROBLEMS 

CORPORATION   STOCK 

1.  A  corporation  incorporated  under  the  laws  of  the  State  of  South 
Dakota  with  an  authoiized  capitalization  of  $2,000,000  offers  stock  for 
subscription  under  the  following  terras  and  conditions : 

The  sale  of  shares  of  preferred  stock,  par  value  $100,  at  a  discount  of 
25  per  cent,  payable  in  five  installments.  To  each  purchaser  of  preferred 
stock  shall  be  donated  one  share  of  common  stock,  par  value  $100. 

At  the  end  of  the  year  it  was  found  that  the  money  had  been  received 
from  installments  paid  on  subscriptions  to  preferred  stock  as  follows: 
First  installment,  $120,750;  second  installment,  $96,600;  third  install- 
ment, $96,600;  fourth  installment,  $96,600;  fifth  installment,  $96,600. 

The  organizer  of  the  corporation  had  purchased  a  vacant  building  and 
real  estate  suitable  for  the  factory  site,  paying  therefor  $27,500.  The 
property  purchased  was  appraised  by  disinterested  appraisers  and  valued 
conservatively  at  $45,000. 

The  owner  (organizer)  then  turned  the  said  property  over  to  the  cor- 
poration at  the  appraised  value,  viz.,  $45,000,  and  received  therefor  pre- 
ferred stock  at  same  price  as  subscribers,  which  was  at  25  per  cent  dis- 
count, and  also  received  one  share  of  common  stock  (donated)  for  each 
share  of  preferred  stock. 

The  expense  of  organization  and  sale  of  stock  at  the  end  of  the  year  was 
found  to  be  as  follows:  Commissions  on  sale  of  stock,  $10,000;  office 
expenses,  clerk  hire,  heat  and  light,  stationery  and  other  expenses,  $3,000 ; 
appraisal,  $250;  betterments  and  remodeling  building  for  occupancy, 
$2,000. 

Draw  up  a  statement  showing  the  condition  of  organization,  using  re- 
ceipts and  disbursements  as  above  and  showing  the  condition  of  stock  sub- 
scriptions and  stock  issue.  (Iowa,  Dec,  1918.) 

2.  The  Unique  Manufacturing  Company,  a  corporation,  was  organized 
July  1,  1913,  with  an  authorized  capital  stock  of  $215,000,  par  value  of 
shares,  $100  each,  for  the  purpose  of  manufacturing  novelties.  The  five  in- 
corporatoi'S  subscribed  and  paid  for  five  shares  each,  organization  expenses 
were  incurred  to  the  amount  of  $5,000  and  paid  for  in  stock,  and  tlie  bal- 
ance of  the  stock  was  disposed  of  on  the  following  conditions:  10  per  cent 
upon  subscription,  and  thi-ee  eciual  calls  for  the  balance  at  30,  60  and  00 
days. 

On  July  31,  the  Unique  Manufacturing  Company  secured  an  option  for 
30  days  on  the  plant  of  "A"  and  "B"  for  $10,000,  agreeing  to  take  over 
the  assets,  exclusive  of  cash,  and  assume  the  liabilities  of  the  partnershii) 


CORPORATION  STOCK 


157 


as  at  July  31,  for  the  sum  of  $200,000,  payable  $90,000  immediately  after 

taking  over  the  business,  and  the  balance  in  90  days.    At  the  expiration  of 
the  option,  the  corporation  took  over  the  plant  as  agreed. 

The  following-  is  a  transcript  of  "A"  and  "B's"  ledger  balances  as  at 
July  31,  1913: 

Land $30,000 

Buildings 35,000 

Machinery 20,000 

Furniture  and  fixtures -. 5,000 

Raw  Material 10,000 

Tools 2,500 

Finished  goods 10,000 

Work  in  process 5,000 

Supplies 7,500 

Accounts  Receivable 25,000 

Cash 8,200 

Mtortgages  on  Buildings 10,000 

Reserve  for  Depreciation — machinery 2,500 

Reserve  for  Bad  Debts 1,000 

Accounts  Payable 15,000 

"A" 77,820 

"B" 51,880 

During  the  interval  "A"  and  "B,"  with  the  consent  of  the  corporation, 
had  sold  finished  goods  for  $5,000,  which  was  2o  per  cent  above  cost. 

The  subscriptions  to  the  stock  of  the  corporation  were  met  on  call  with 
the  exception  that  on  the  second  call  a  subscriber  for  25  shares  notified 
the  corporation  that  lie  was  unable  to  complete  his  agreement,  and  he  was 
released  witliout  further  liability.  The  forfeited  stock  was  sold  for  cash, 
at  par. 
From  the  foregoing,  draft: 

(a)     Journal  entries  necessary  to  close  the  books  of  the  partnership. 
(6)     To  open  the  books  of  the  corporation  and  to  show  all  transactions 
on  the  Unique  Company's  books. 

(c)  Balance  Sheet  of  the  Unique  Manufacturing  Company,  September 
1,  1913.  (Ohio,  Nov.  1913;  Minn.,  Oct.,  1916.*) 

3.  On  June  30,  1917,  Mace  &  Morgan,  partners  operating  a  manu- 
facturing plant,  decided  to  incorporate  under  the  general  laws  of  the 
State  of  Virginia.  The  corporation  purchased  all  the  assets  and  assumed 
all  the  liabilities  of  the  partnership  as  set  forth  in  the  Balance  Sheet 
dated  June  30,  1917. 

The  corporation  has  an  authorized  capital  of  $500,000  and  it  gave  its 
entire  issue  of  capital  stock  as  the  consideration  for  the  purchase  of  the 
partnership.    The  entire  stock  was  taken  by  Mace  &  Morgan. 
Balance  Sheet,  June  30,  1917 


Assets 

Plant  and  Machinery $175,000 

Inventory 102,625 

Accounts  Receivable 113,750 

Notes  Receivable 7,500 

Cash 32,125 

$431,000 


Liabililies 

Mace's  Investment $300,000 


Morgan's  Investment 

Accounts  Payable 

Notes  Payable 

Wages  Due  and  Unpaid  . . 


100,000 

26,250 

3,500 

1,250 

$431,000 


158 


C.  P.  A.  ACCOUNTING 


The  change  in  organization  was  not  retlected  on  the  books  at  the  time 
of  incoi-poration,  but  at  the  close  of  the  fust  fiscal  year  (June  30,  1918) 
of  the  corporation's  existence  the  condition  of  the  books  was  shown  by  the 
following  trial  balance : 


Debits 


Credits 


Mace's  Investment 

Morgan's  Investment 

Plant  and  Machinery 

Material  Inventory,  June  30,  1917 

Sales 

Purchase 

Labor 

Office  Salaries 

Traveling  Expenses 

Interest 

Stationery  and  Printing 

Rent  and  Taxes 

Allowances  on  Sales 

Cash  Discounts 

Fuel 

Insurance 

Freight  Inward 

Commission 

Advertising 

Notes  Receivable 

Notes  Payable 

Accounts  Receivable 

Accounts  Payable 

Cash 


$187,500 
102,625 

240,000 

172,500 

35,000 

12,000 

3,000 

875 

21,000 

1,250 

10,000 

23,000 

875 

8,750 

31,875 

2,500 

30,575 

180,575 

37,875 


$300,000 
100,000 


657,025 


5,500 
39,250 


$1,101,775 


$1,101,775 


Depreciation  on  Plant  and  Machineiy,  5  per  cent;  Unexpired  Insurance, 
$375;  Bad  Debts,  $4,625;  Inventory  of  Material  on  hand  June  30,  1918, 
$98,025. 

The  bad  debts  represent  a  part  of  the  Accounts  Receivable  taken  over  by 
the  corporation  June  30,  1917,  which  accounts  were  guaranteed  by  Mace 
&  Morgan  individually  on  a  basis  of  the  investment  of  each  at  the  time 
the  business  was  incorporated. 

Make  such  entries  as  would  convert  the  partnership  books  into  those  of 
the  corporation  and  prepare  an  Income  account  and  analysis  of  Profit  and 
Loss  for  the  year  ending  June  30,  1918,  and  a  Balance  Sheet  as  of  June 
}0,  1918. 

(Va.,  Nov.,  1918.) 

4.  A  stockholder  desires  to  know  the  book  value  of  his  stock  in  a  cor- 
poration for  the  purpose  of  accepting  an  offer  from  a  purchaser  who 
agrees  to  buy  his  stock  at  the  book  value  as  shown  by  the  following  Balance 
Sheet.    Prepare  a  statement  showing  the  book  value  of  the  stock. 


CORPORATION  STOCK 


150 


Assets 

Cash  on  hand $2r),()nn 

Accounts  Receivable 100,000 

Notes  Receivable 30,0(K) 

Inventories 1(X),000 

Treasury     Stock      (1,(X)0 

shares) 125,000 

Cost  of  i  lant 800,000 


$1,180,000 


Liabilities 

Accounts  Payable $15,000 

Notes  Payable 50,000 

Bonds  Outstanding 200,000 

Reserve  for  Shrinkage  of 

Inventories 25,000 

Reserve  for  Depreciation         50,000 

Reserve  for  Extinguish- 
ment of  Bonds 100,000 

Reserve  for  Additions  to 

Plant 50,000 

Capital     Stock      (3,000 

shares) 300,000 

Reserve     for      Working 

Capital 200,000 

Undivided  Profits 190,000 


$1,180,000 


(N.  C,  Nov.,  1918.) 

5.  In  Mr.  Jones'  Private  Ledger  he  keeps  accounts  with  each  invest- 
ment he  makes,  one  of  which  is  an  investment  of  1,000  shares  (par  value 
$100)  of  the  A.  B.  Company  which  he  acquired  in  July,  1914,  for  $85,000. 
After  this  date  and  up  to  December  31,  1918,  he  makes  further  purchases 
and  sales  of  this  stock.  A  certified  public  accountant  called  in  to  prepare 
Mr.  Jones'  income  tax  return  for  1918  finds  that  these  and  other  transac- 
tions have  been  wiitten  up  in  the  following  manner,  no  effort  to  show  the 
profit  of  the  sale  of  1,000  shares  on  June  1,  1918,  having  been  attempted. 

Investment  A.  B.  Company  Account 


July  1,  1914 
Dec.  31,  1914 
May  31,  1915 
Nov.  30,  1915 
Dec.  31,  1915 
July     1,  1916 


Feb.  28   1917 
Dec.  31,  1917 

June    1,  1918 


1,000  shares  purchased 

Entry  to  carry  this  stock  at  par 

Purchased  1,500  shares  at  par 

Sold  300  shares  at  125 

Profit  and  loss — profit  on  sale  of  300  shares 

Stock  dividend  of  50  per  cent  on  2,200 
shares  declared  from  profits,  accumulated 
prior  to  Mar.  1.  1913 

Sold  700  shares  at  110 

Profit  and  Loss,  profit  made  on  sale  of  700 
shares 

Sold  1,000  shares  at  125 


Rewrite  this  entire  account  to  show  how  it  should  have  been  kept  in  order 
to  show  actual  profit  on  each  sale  and  also  calculate  the  actual  profit  on 
the  last  sale  of  1,000  shares.  What  is  the  book  value  of  the  total  shares  on 
hand  December  31,  1918? 

(A.  L  of  A.,  Nov.,  1919.) 


160  C.  P.  A.  ACCOUNTING 

6.  The  directors  of  the  Clinrles  Manufactiuing-  Company  decide  to 
change  their  plan  of  capitalization  by  retiring  their  common  stock  and 
issuing  preferred  stock  and  new  comiuon  stock  in  place  of  it.  On  De- 
cember 31,  1917,  their  books  shewed  $1,000,000  common  stock  and  $300,000 
surplas.  The  new  plan  offered  each  common  stockholder  1.3  shares  of  pre- 
ferred stock  and  1  share  of  new  common  stock  for  each  share  of  old  com- 
mon stock,  fractional  shares  amounting  to  $6,400  to  be  redeemable  in 
cash.  Amendments  to  the  articles  of  incorporation  were  duly  made  pro- 
viding an  authorized  amount  of  $1,500,000  of  preferred  stock  and 
$2,000,000  of  common  stock.  On  April  1,  all  exchanges  had  been  com- 
pleted with  the  following  excej^tions:  unissued  common  .stock,  1,000  shares; 
unissued  preferred  stock,  1,300  shares;  fractional  shares,  $300.  The  par 
value  of  each  kind  of  stock  is  $100  per  share. 

Draft  the  necessary  Journal  entry  or  entries  to  record  the  above 
changes. 

(Wis.,  April,  1918.) 

7.  The  authorized  capital  stock  of  a  corporation  is  $500,000,  divided 
into  5,000  shares,  par  value  $100.  Of  this  amount  $400,000  has  been  sub- 
scribed and  paid  for  in  full.  The  corporation  purchases  ten  shares  of  a 
dissatisfied  stockholder  for  $75  a  share,  and  five  other  stockholders  each 
donate  five  shares  to  the  company.  Five  shares  of  the  purchased  stock  and 
all  the  donated  stock  are  sold  for  $50  a  share. 

(a)  Draft  proper  entries  and  show  the  Ledger  accounts  and  balances. 
(6)  How  would  the  balances  of  the  accounts  in  (a)  appear  in  a  Balance 
Sheet?  (c)  Give  the  entries  and  show  the  Ledger  accounts  and  balances 
if  the  capital  stock  were  of  no  specified  par  value,  but  5,000  shares  had 
been  issued  at  $80  and  the  other  conditions  remain  as  stated  in  the  first 
paragraph,  (d)  How  would  the  balances  of  the  accounts  in  (c)  appear 
in  a  Balance  Sheet? 

(Wis.,  April,  1917.) 

8.  J.  B.  Brown  and  L.  C.  Smith  are  partners,  and  in  order  to  raise 
more  capital  and  to  preserve  the  organization  they  decide  to  incorporate. 
A  company  was  duly  incorporated  under  the  name  of  The  Eclipse  Com- 
pany, with  an  authorized  capital  of  $800,000,  di\-ided  into  8,000  shares 
of  the  par  value  of  $100  each. 

The  partners  agi-eed  to  sell  for  the  sum  of  $800,000,  payable  in  capital 
stock  of  the  corporation  at  par,  all  rights  to  and  title  in  the  net  assets  of 
the  partnership,  exclusive  of  the  cash,  which  was  divided  between  the 
partners  in  proportion  to  their  several  interests  at  the  time  of  the  sale  of 
the  property. 

According  to  the  articles  of  partnership,  Brown  and  Smith  weie  ecjually 
interested  in  the  assets,  but  the  profits  and  losses  were  on  a  basis  of  60 
per  cent  and  40  per  cent,  respectively. 

The  partnership  Balance  Sheet  at  the  time  of  the  sale  was: 


CORPORATION  STOCK 


161 


Land  and  Buildings $2(M),(K)0 

Cash 10,000 

Inventories 100,0(K) 

Accounts  Receivable 150,000 

Machinery  and  Equipment  100,000 

$560,000 


Notes  Payable $100,000 

Accounts  Payable 40,000 

Brown's  Capital 210,000 

Smith's  Capital 210,000 


$560,000 


For  the  purpose  of  providing  workin^^  capital,  the  partnership  donated 
$300,000  of  the  capital  stock  to  the  corporation,  which  was  subsequently 
sold  at  $50  per  share. 

You  are  required  to  (1)  close  the  partnership  books,  showing  Ledger 
accounts  of  partners  only;  (2)  open  corporation  books;  (3)  prepare  a 
Balance  Sheet  of  the  corporation  before  sale  of  donated  stock;  (4) 
prepare  a  Balance  Sheet  after  sale  of  donated  stock. 

(Ohio,  Oct.,  1919.) 


CHAPTER  IX 


INTEREST 

Interest  and  Discount — Although  both  disooiint  and  interest  are  fre- 
quently recorded  in  the  same  account/  they  must  not  be  confused.  Dis- 
count is  the  allowance  made  for  the  realization  upon  an  undue  asset/  while 
interest  is  the  payment  made  for  the  use  of  capital/  Discount  includes 
allowances  made  for  prompt  payment,  or  cash  discounts;  allowances  made 
to  customers  from  list  prices,  or  trade  discounts;  differences  between  par 
and  a  purchase  price  less  than  par,  or  discount  on  securities;  and  interest 
paid  in  advance,  or  commercial  discount,*  the  last  being  the  only  kind  of 
discount  included  in  the  interest  and  discount  account.'  If  simple  interest 
is  collected  in  advance,  it  is  called  commercial  discount ;  if  compound  in- 
terest is  collected  in  advance,  it  is  called  true  discount." 

It  has  been  claimed  that  as  bank  discount  is  prepaid  interest,'  it  should 
be  designated  intei'est  and  not  discount.*  However,  interest  is  calculated 
on  principal  only,  while  bank  discount  is  calculated  on  both  principal  and 
intei'est.'  The  interest  on  a  note  for  $1,000  due  at  six  per  cent  for  one 
year  would  amount  to  $00  at  the  end  of  the  year,  but  if  money  is  worth  six 
per  cent,  the  bank  discount  at  the  beginning  of  the  year  would  be  six 
per  cent  of  $1,060,  or  $63.60. 

If  interest  is  figured  on  a  constant  principal,  it  is  called  simple  interest, 
while  if  it  is  figured  on  a  principal  periodically  increased  by  the  simple 
interest,  it  is  called  compound  interest.'" 

Prepaid  Interest — Prepaid  interest  on  notes  payable  and  receivable  are, 
respectively,  deferred  charges  to  operation"  and  deferred  credits  to  in- 
come." Both  kinds  of  prepaid  interest  may  be  created  directly  upon  the 
receipt  or  payment  of  the  money,  in  Avhich  case  the  prepaid  interest  would 
be  prorated  between  the  accounting  pei-iods,"  or  by  adjusting  entries  which 
separate  the  real  and  nominal  elements  in  the  interest  expense  and  interest 
income  accounts." 

Accrued  Interest — Accrued  interest  receivable  and  accrued  interest  pay- 
able are  current  asset'''  and  current  liability  accounts,*"  respectively.  They 
are  created  by  charging  or  crediting  the  accounts,  as  the  case  may  be,  with 
the  amounts  of  the  accrued  interest,  the  ofTsetting  entries  being  made  to 
the  interest  expense"  and  intei'est  income  accounts."  While  it  is  desirable 
to  show  the  full  accrued  interest  on  doubtful  notes  and  accounts  receivable 
for  which  reserves  have  been  established,  the  interest  should  be  credited 
to  the  reserve  instead  of  to  income." 

There  has  been  a  tendency  to  neglect  adjusting  the  books  for  prepaid 
and  accrued  interest  on  the  ground  that  the  amount  at  the  beginning  is 


'For  explanation  of  superior  figures  see  page  337. 

162 


INTEREST  163 

about  the  same  as  at  the  end  of  the  period.  This  is  absolutely  unscientific 
and  untrustworthy.  Financial  institutions  have  met  this  situation  by 
having  the  amounts  of  the  loans  i-ecorded  on  a  tickler  under  the  due  date. 
When  the  adjustments  are  calculated,  the  unpaid  loans  are  totaled  each 
day  and  then  the  interest  is  figured. 

Interest  Expense  and  Income — It  is  advisable,  though  not  absolutely 
necessary,  to  have  separate  accounts  for  the  expenses  and  incomes  arising 
from  interest.^"  The  interest  expense  account  may  be  divided  into  interest 
on  notes  payable,  interest  on  bonds  payable,  etc. ;  and  the  interest  income 
account  also  may  be  divided  so  as  to  indicate  the  source  of  the  income." 
All  of  these  accounts  should  be  classed  as  non-operating  items  in  the  profit 
and  loss  statement,"  although  some  accountants  treat  interest  on  bonds 
and  loans  payable  as  an  administrative  item." 

Calculation  of  Interest — In  calculating  time  in  connection  with  interest, 
the  first  day  should  be  excluded  and  the  last  day  included,  or  vice  versa." 
When  the  interest  period  is  expressed  in  months,  the  due  date  is  the  same 
in  the  proper  month,  but  if  the  period  is  expressed  in  days,  the  due  date 
is  found  by  counting  the  actual  days."  For  instance,  a  note  dated  April 
11  would  be  due  on  July  11  if  the  interest  period  is  three  months,  or 
would  be  due  on  July  10  if  the  interest  period  is  ninety  days.  In  calculat- 
ing discount,  banks  always  count  the  number  of  days  regardless  of  the  way 
the  interest  period  is  stated  on  the  notes.^"  The  legal  riile  for  calculating 
interest,  which  is  used  by  the  government  and  by  accountants  in  computing 
interest  on  judgments  and  whenever  judicial  actions  and  decisions  are 
involved,  is  based  on  a  365-day  year,  while  the  ordinary  business  rule  is 
based  on  a  360-day  year." 

Interest  on  Partial  Payments — The  United  States  rule  for  calculating 
interest  on  partial  payments  applies  a  payment  first  against  the  interest 
due.  If  payment  exceeds  the  interest,  the  surplus  reduces  the  principal 
on  which  the  future  interest  is  computed.  If  a  payment  is  less  than  the 
accrued  interest,  it  is  held  in  reserve,  and  interest  on  the  old  principal 
continues  until  a  payment,  together  with  the  payments  held  in  reserve, 
exceeds  the  acci'iied  interest,  in  which  case  the  excess  is  used  as  above.** 

The  merchants'  rule  charges  interest  on  the  principal  from  its  creation 
to  its  cancellation,  and  allows  interest  on  all  payments  from  the  date  of 
each  payment  till  the  date  the  whole  principal  is  canceled." 

Audit  of  Interest — When  the  auditor  is  verifying  the  assets  and  liabili- 
ties, he  should  make  lists  showing  the  securities  and  notes  receivable  owned 
and  the  notes  and  bonds  payable  owed  during  the  period.'"  All  interest 
arising  therefrom  should  then  be  checked  against  the  interest  income  and 
expense  accounts.""  The  verification  of  interest  paid  is  especially  impor- 
tant, as  notes  payable  which  are  not  shown  on  the  books  may  be  dis- 
covered.'' The  interest  on  bank  deposits  should  be  verified  by  ascertaining 
the  rate  of  interest  and  by  roughly  checking  the  average  daily  balances." 

Interest  as  a  Cost  of  Manufacture — One  school  of  accountants  believes 
that  interest  should  be  included  on  the  books  as  a  cost  of  manufacture,** 
while  another  school  holds  the  opposite  view." 


164  C.  P.  A.  ACCOUNTING 

The  contention  that  intei-est  is  an  element  in  the  selling  price  cannot  be 
denied.  Whether  interest  should  be  shown  on  the  books  as  a  cost  of  manu- 
facture, however,  depends  on  whether  it  is  preferable  to  include  interest 
in  the  manufacturing  burden  or  in  the  profit  charged  on  the  product ;  in 
either  case  the  interest  charge  affects  the  selling  price.""" 

The  objections  frequently  raised  to  including  interest  as  a  cost  of  manu- 
facture are:  first,  the  difficulty  of  determining  the  rate  to  be  charged;" 
second,  that  interest  is  usually  charged  only  on  fixed  investments;™  third, 
the  difficulty  of  handling  the  credit  offsetting  the  interest ;'°  fourth,  that  a 
more  or  less  constant  element  representing  interest  tends  to  obscure  fluctua- 
tions in  the  other  costs  ;^  and  fifth,  that  the  inclusion  of  interest  as  a  cost 
in  financial  statements  confuses  the  business  world.*' 

The  first  of  these  objections  is  not  unsurmountable,  for  the  interest  rate 
to  be  charged  does  not  represent  the  actual  cost  of  the  capital  invested 
but  only  the  market  rate  of  interest.*^  The  second  objection  is  faulty,  as 
interest  is  unimportant  except  whei'e  the  element  of  time  is  vital."  Thus, 
interest  need  not  be  charged  on  inventories  whose  turnover  is  rapid,  to 
justify  the  inclusion  of  interest  as  a  cost  in  the  case  of  seasoned  lumber, 
etc."  The  fourth  objection  may  be  an.swered  by  stating  that  correct  cost 
figures  may  be  preferable  to  incorrect  cost  figures  even  though  the  correct 
figures  may  be  harder  to  analyze. 

The  third  and  fifth  objections  to  the  inclusion  of  interest  as  a  cost  of 
manufacture  are  very  real  and  vital.  The  credit  offsetting  the  interest 
charge  is  usually  made  to  profit  and  loss,  which  is  absolutely  incorrect  as 
it  anticipates  a  profit.  The  charge  to  manufacturing  burden  is  transferred 
to  finished  goods  where  it  may  enhance  the  inventory — another  illustration 
of  the  anticipated  profit."  Some  cost  accountants  are  meeting  these 
objections  by  crediting  a  reserve  account  when  interest  is  charged  to  manu- 
facturing burden,*'  and  then  writing  off  to  profit  and  loss  all  of  the  reserve 
except  an  amount  equal  to  the  interest  included  in  the  inventories  at  the 
end  of  the  period."  This  i-eserve  account  is  then  deducted  from  the  inven- 
tories on  the  balance  sheet." 

As  the  calculation  of  the  interest  in  the  final  inventories  is  very  compli- 
cated, and  as  most  of  the  advantages  of  including  interest  as  a  cost  manu- 
facture can  be  obtained  by  including  it  in  statistical  studies  and  excluding 
it  on  the  books,  it  would  seem  expedient  to  refrain  from  including  interest 
in  burden,  except  in  cases  where  the  time  element  is  unusually  important 
and  where  the  accounting  departments  of  the  firms  are  competent  to  clear 
the  interest  charge  through  the  books  in  such  a  way  that  it  will  not  cause 
an  anticipated  profit. 

Compound  Interest  Processes — A  knowledge  of  compound  interest  pro- 
cesses is  necessary  in  dealing  with  the  valuation  of  bonds,  the  writing  off 
of  bond  discounts  and  premiums,  the  calculation  of  depreciation  by  cer- 
tain methods,  the  computation  of  sinking  funds  or  the  amounts  required 
to  redeem  an  obligation  at  maturity,  and  the  handling  of  all  transactions 
involving  annuities."  Annuities  are  payments  of  specified  sums  of  money 
made  at  regular  intervals  for  a  stated  duration  of  time.'^" 


INTP]REST  1G5 

Compound  interest  processes  are  all  based  upon  the  ratio  of  increase, 
or  one  added  to  the  rate  of  interest  (1  +  i)."  They  may  be  classified,  how- 
ever, into  those  involving  interest  and  those  involving  discount.  The 
class  involving  interest  is  calculated  by  using  the  rate  of  increase  as  a 
multiplier,  while  the  latter  class  is  calculated  by  using  the  rate  of  increase 
as  a  divisor. 

Process  Involving  Interest — Compound  interest  processes  involving  in- 
terest are  used  in  calculating  (a)  the  amounts  of  principals  payable  at 
future  dates  at  compound  interest,  (b)  the  amount  of  annuities,  and 
(c)  sinking  fund  installments. 

(a)  The  amount  of  a  given  pi-incipal  payable  after  a  given  number  of 
periods,  interest  being  compounded  at  a  given  rate,  is  found  by  multiply- 
ing the  given  sum  by  the  ratio  of  increase  multiplied  by  itself  as  many 
times  as  there  are  periods." 

Illustration. 
Formula  for  Find  the  amount  that  $10,000  will  aggregate 

Amount  of  Principals     if  invested  for  four  periods  at  two  per  cent  per 
P  X  (l  +  i)"  period. 

$10,000  X  (1.02)'  =  $10,824.32 

(b)  The  amount  of  an  annuity  at  the  end  of  a  given  number  of  periods, 
if  the  payments  have  been  invested  at  compound  interest  at  a  given  rate  of 
interest,  is  found  by  multiplying  the  amount  of  each  payment,  called  the 
rent  of  the  annuity,  by  the  quotient  of  the  total  interest  (amount  of  $1  for 
the  given  time  at  compound  interest  at  given  rate  minus  one)  divided  by 
the  rate  of  interest."' 

Illustration. 
Formula  for  Find   what   a   four-payment   annuity,   whose 

Amount  of  Annuity       rent  is  $10,000,  will  aggregate  if  payments  are 
(l-fi)"  —  1  invested  at  compound  interest  at  two  per  cent 

^  i  per  period. 

$10,000  X  ^      ^' =  $41,216.08 

(c)  The  amount  of  annuity  payments  (sinking  fund  installments)  re- 
quired to  produce  a  given  sum  in  a  given  time,  if  the  payments  are  invested 
at  a  given  rate  of  interest,  is  found  by  dividing  the  desired  sum  by  the 
amount  of  an  annuity  of  $1  for  the  same  time  at  the  same  interest  rate." 
The  terms  "sinking  fund  installments"  and  "rents  of  annuities"  are 
synonyms. 

Illustration. 
Formula  for  Find  the   amount   which,   set   aside   at  four 

Sinking  Fund  equal  intervals  at  compound  interest  at  two  per 

Installments  cent  per  period,  will  amount  to  $41,216.08  at 

(1  +  i)"  —  1         end  of  the  four  periods. 

'  $41,216.08  ^  ^"''^^Q^)"^  =  $10,000.00 


166  C.  P.  A.  ACCOUNTING 

The  connection  between  the  formulas  for  finding  the  amounts  of  an- 
nuities and  of  sinking  fund  installments  can  be  seen  from  the  following 
equations : 

Rent  X  amount  of  annuity  of  $1  =  Amount  of  given  annuity. 

Rent  =^  amount  of  given  annuity  -f-  amount  of  annuity  of  $1. 

The  following  illustration  may  show  the  reasoning  back  of  the  above 
formulas.  If  $1  were  left  on  deposit  in  one  bank  at  two  per  cent  interest 
for  four  years,  and  the  annual  interest  of  two  cents  withdrawn  annually 
and  deposited  in  another  bank  at  two  per  cent,  compounded  annually, 
formula  (a)  shows  that  at  the  end  of  the  four  years  there  would  be  the 
original  $1  in  the  first  bank  and  $.082432  in  the  second  bank.  The  latter 
sum  is  the  amount  with  interest  of  the  periodic  payment  (an  annuity)  of 
two  cents.  If  the  amount  of  an  annuity  of  two  cents  is  $.082432,  the 
amount  of  an  annuity  of  $1  must  be  $4.1216  ($.082432  X  V2  X  100). 

Processes  Involving  Discount — Compound  interest  processes  involving 
discount  are  used  in  calculating  (a)  the  present  worth  of  payments  to  be 
made  at  future  dates,  (b)  the  present  worth  of  annuities,  (c)  the  rent  of 
annuities,  (d)  premium  on  bonds,  and  (e)  discount  on  bonds. 

(a)  The  present  worth  of  an  amount  payable  at  a  future  date  is  found 
by  dividing  the  amount  bj^  the  product  of  the  ratio  of  increase  multiplied 
by  itself  as  many  times  as  there  are  periods.'" 

Illustration. 
Formula  for  Find  the  present  worth  of  $10,824.32  payable 

Present  Worth  at  the  end  of  four  periods,  if  money  is  com- 

of  Future  Amounts       pounded  at  two  per  cent  per  period. 
A  ^  (1  +  i)n  $10,824.32^(1.02)  =$10,000.00. 

(b)  The  present  worth  of  an  annuity  is  fouiul  ])y  multiplying  the  rent 
of  the  annuity  by  the  quotient  of  the  total  discount  (one  minus  the  present 
worth  of  $1  payable  at  the  end  of  the  annuity)  divided  by  the  rate  of 
interest."  Illustration. 

Formula  for  Find  the  pre.sent  worth  of  a  four-period  an- 

Present  Worth  nuity  of  $10,000  when  the  money  is  worth  two 

of  Annuity  per  cent   per  period. 

(1  +  i)n  (1.02)* 

R  X '  $10,000.00  X ^Ti =  $38,077.29 

1  .UJ 

(c)  The  rent  of  an  annuity  that  can  be  purchased  for  a  given  sum, 
when  money  is  worth  a  given  rate  of  interest,  is  found  I)y  dividing  the 
sum  by  the  present  worth  of  an  annuity  of  $1  for  the  same  number  of 
periods  at  the  same  rate  of  interest."  Illustration 

Formula  for  Find  the  rent  of  a  four-pei-iod  annuity  that 

Rent  of  Annuity  can   be  purchased  for  $38,077.29  wlien  money 

1  is  worth  two  per  cent  per  period. 

1- nTFTj^  i_ 

^~ i (].02)' 

1  $38,077.29  -f- — — -  =  $10,000.00 


INTEREST  167 

(<l)  The  premium  on  a  bond  is  the  present  worth  of  an  annuity  whose 
rent  equals  the  excess  of  the  actual  interest  receivable  over  the  interest  it 
would  bear  at  the  current  rate  of  interest,  whose  life  is  the  life  of  the 
bond,  and  whose  interest  rate  is  the  current  rate.^' 

Illustration. 

Formula  for  Find    the   premium   on   a   bond    whose   par, 

Bond  Premium  $1 0,000,  due  after  four  periods,  bearing  interest 

1  at  two  and  one-half  per  cent  per  period,  if  the 

n  -I-  n "     current  interest  rate  is  two  per  cent  per  period. 

(AT— Cl)X ^  ^  1 

1  1 ;—, 

$50.00  X ^^^f^^  =$190.39 

(e)  Bond  discount  is  calculated  like  premium  on  bonds,  except  that 
the  rent  of  the  annuity  whose  present  worth  must  be  computed,  is  the 
excess  of  the  interest  that  the  bond  would  bear  at  the  current  rate  of 
interest  over  the  actual  interest  receivable.™ 

Illustration. 
Formula  for  Find  discount  on  above  mentioned  bond  if 

Bond  Discount  current  rate   of  money  is   three  per   cent   per 

1  period. 

(CI— xVI)X- 


:i  +  i)»  -J L 


'        $50.00  X yr-^  =  ^^  ^^-^^ 

The  connection  between  the  formulas  for  finding  the  present  worth  and 
rents  of  annuities  can  be  seen  from  the  following  equations : 

Rent  X  present  worth  of  annuity  of  $1  =  present  worth  of  given  annuity. 
Rent  =  present  worth  of  given  annuity  -^  present  worth  of  annuity  of  $1. 

The  connection  betAveen  the  formulas  for  finding  the  amount  and  present 
worth  of  annuities  can  be  seen  from  the  fact  that  the  amount  of  the 
annuity  is  a  sum  payable  in  "n"  periods,  money  worth  "i"  per  cent  interest. 
The  amount  of  the  annuity  discounted  back  to  the  present  would  be  its 
present  worth,  or 

1 
(1-f  i)"  — 1  1  ^  1  — (1  +  i)" 

i  '  ^     (l  +  i)n  i 

Valuation  of  Serial  Bonds — When  bonds  mature  at  dififerent  dates,  the 
initial  value  of  the  series  can  be  found  only  by  calculating  the  initial  values 
of  each  installment  and  then  adding  these  initial  values.'"  After  the  value 
of  the  .series  has  been  thus  calculated,  the  amortization  of  premium  or 
discount  would  be  computed  as  if  the  bonds  were  not  in  a  series."' 

Valuation  of  Short  Terminal  Bonds — When  bonds  do  not  mature  on 
one  of  their  interest  dates,  there  is  a  short  terminal  period  to  be  considered 
in  evaluating  the  bonds.  The  rule  for  evaluating  bonds  with  short  terminals 


168 


C.  P.  A.  ACCOUNTING 


is  to  ascertain  the  value  for  the  full  number  of  periods,  disreprardin^  the 
terminal;  and  to  this  value  add  the  short  interest  (that  part  of  the  nominal 
interest  per  period  that  the  short  term  is  to  the  normal  interest  period)  and 
then  to  divide  the  sum  by  the  short  ratio  (one  plus  that  part  of  the  effective 
interest  period  that  the  short  term  is  to  the  normal  interest  period)/' 

Suppose  a  five  per  cent  bond,  par  $10,000,  yielding  four  per  cent, 
maturing  October  1,  1919,  interest  payable  May  1  and  Nov.  1,  was  pur- 
chased May  1,  1914;  the  bond  runs  10  5/6  periods;  short  ratio  rate  is 
1.016667;  and  short  interest  rate  .020833.  The  bond  for  the  ten  periods  is 
worth  $10,449.13,  which  together  with  $208.33,  is  $10,657.46,  which  amount 
divided  by  1.016667  gives  $10,482.75,  the  value  of  the  ]>ond.  If  a  bond 
were  issued  between  interest  dates  and  matured  between  interest  dates,  this 
process  is  applied  twice." 

Amortization  of  Bond  Premium  and  Discount — The  scientific  adjusting 
of  the  valuation  of  bonds  purchased  above  or  below  par  so  that  the  bonds 
will  be  valued  at  par  when  they  mature,  is  called  the  amortization  of  bond 
premium  and  discount.*^^  Bond  investments  have  two  interest  rates,  namely, 
the  coupon  or  nominal  interest  rate,  and  the  effective  rate  or  the  real 
income  rate  on  the  basis  of  the  price  paid  for  the  bonds."  At  the  end  of 
each  period  the  bonds  are  revalued  by  adding  the  excess  of  the  effective 
over  nominal  intei'est  to  the  value  of  the  bonds  if  the  bonds  wei*e  puichased 
below  par,  or  by  subtracting  the  excess  of  the  nominal  over  the  effective 
interest  from  the  value  of  the  bonds  if  the  bonds  were  purcha.sed  at  a 
premium.'' 

The  following  table"'  shows  the  amortization  of  bond  premium  and  the 
revaluation  of  the  bond  at  each  interest  date. 

5%  bond,  par  $10,000;  purchased  Jan.  1,  1917,  on  a  4%  basis  for 
$10,190.39 ;  2  years  to  run ;  interest  January  and  July. 


Date 


Nominal 

Interest 

(a) 


Effective 

Interest 

(b) 


Amorti- 
zation 

(c) 


Value  of 
Bond 

(d) 


1/1/17 

7/1/17 
1/1/18 
7/1/18 
1/1/19 


$250.00 
250.00 
250.00 
250.00 


$203.81 
202.88 
201 . 94 
200.98 


$46. 19 
47.12 
48.06 
49.02 


Sl.000.00 


$809.61 


$190. 39 


$10,190.39 
10,144.20 
10.097.08 
10,049.02 
10,000.00 


(a)  2}/^  per  cent  of  par. 

(b)  2  per  cent  of  (d). 


(c)  (a)  minus  (b). 

(d)  Value  on  previous  date  minus  (c). 


A  similar  table,**  showing  the  amortization  of  bond  di.scount  and   the 
periodic  revaluation  of  the  bond,  is  3.3  follows ; 


INTEREST 


169 


5%   bond,  par  $10,000;   purchased  Jan.  1,  1917,  on  a  6%   basis  for 
$9,814.15;  2  years  to  run;  interest  January  and  July. 


Date 


Nominal 

Interest 

(a) 


Effective 

Interest 

(b) 


Amorti- 
zation 

(c) 


Value  of 

Bond 

(d) 


1/1/17 
7/1/17 
1/1/18 
7/1/18 
1/1/19 


$250.00 
250.00 
250.00 
250.00 


$294.42 
295.75 
297.13 
298.55 


$44.42 
45.75 
47.13 
48.55 


$9,814.15 
9,858.57 
9,904.32 
9,951.45 

10,000.00 


$1,000.00 


$1,185.85 


$185.85 


(a)  2J/^  per  cent  of  par. 

(b)  2  per  cent  of  (d). 


(c)  (b)  minus  (a). 

(d)  Value  on  previous  date  plus  (c). 


In  both  of  the  above  tables  the  nominal  interest  is  the  product  of  the 
nominal  interest  rate  multiplied  by  the  par  value  of  the  bonds,  while  the 
effective  interest  is  the  product  of  the  purchase  price  of  the  bonds  multi- 
plied by  the  rate  of  interest  the  bonds  actually  yield  on  the  money  invested. 

As  the  amortization  of  bond  premium  and  discount  is  accomplished 
through  the  bond  interest  account,  this  subject  will  be  further  discuissed  in 
Chapter  XI. 

Bond  Purchased  at  Intermediate  Dates — When  bonds  are  purchased 
between  interest  dates,  the  buyer  pays  the  seller  accrued  (simple)  interest 
on  the  bonds,  and  the  premium  or  discount  is  considered  to  vanish  by  an 
equal  portion  each  month.™  If  the  bond  mentioned  in  the  preceding  section 
as  purchased  at  a  premium  on  January  1,  1917,  was  sold  on  a  4%  ba  is 
on  February  1,  1917,  the  accrued  simple  interest  would  be  $41.67 
(1/6  X  $250)  and  the  amortizement,  $7.70  (1/6  X  $46.19),  so  the  seller 
would  receive  $10,224.36  (.$10,190.39  — $7.70  +  $41.67).  A  simpler  rule 
is  to  add  the  simple  interest  for  the  lapsed  time  on  tlie  value  at  the  previous 
interest  date  calculated  at  the  effective  interest  ra^e.'"  By  this  rule  the  seller 
would  receive  $10,224.36    ($10,190.39  +  ($10,190.39  X  .04  X  1/12)). 

Interest  Bearing  Debt  Paid  by  Equal  Annual  Imtallments — The  amount 
of  equal  periodic  installments  including  principal  and  interest  which  will 
repay  an  interest-bearing  debt  in  a  given  numl)er  of  payments  is  found 
by  multiplying  the  principal  by  the  amount  of  the  sum  of  $1  under  the 
given  conditions  and  by  dividing  that  product  by  the  amount  of  an  an- 
nuity of  $1  under  the  same  conditions."' 

For  example,  what  equal  annual  payments  including  pi-incipal  and 
interest  will  repay  in  foui-  annual  payments  a  debt  of  $10,000  l)eaiing  two 
per  cent  interest? 

;1.02)^  — 1 


($10,000  X  (1.02)^)  ^ 


.02 


.$2,626.24,  annual  payment. 


170  C.  P.  A.  ACCOUNTING 

Optional  Bond  Redemption — When  the  issuer  has  the  option  of  redeem 
ing  bonds  prior  to  maturity,  the  premium  or  discount  on  the  bonds  will 
be  affected,  if  it  is  to  the  issuer's  advantage  to  call  the  bonds.  If  the 
issuer  can  redeem  at  par  before  maturity,  the  purchase  price  should  be 
calculated  on  the  assumption  that  the  right  will  be  used  if  the  bonds  are 
bought  at  a  premium,  or  that  the  riglit  will  not  be  used  if  the  bonds  are 
bought  at  a  discount." 

Bonds  which  will  be  redeemed  above  par  before  maturity,  should  be 
valued  by  separately  calculating,  at  the  effective  interest  rate,  the  present 
worth  of  the  principal  as  a  future  sum  and  the  present  worth  of  the 
coupons  as  an  ordinary  annuity." 

For  example,  find,  on  a  four  per  cent  basis,  the  value  of  a  five  per  cent 
$10,000  bond,  due  in  ten  years,  but  to  be  redeemed  at  110  in  two  years. 

Value  of  $11,000  in  two  years: 

$11,000  ^  ( 1.02 ) '  =  $10,162.30 

Value  of  coupons : 

"^~(1.()2)' 
$250  X -rp-^  =        ^^'^•^^ 

Sum  of  value  of  pi-incipal  and  intei-est  $11,114.23 

Computing  an  Unknown  Rate — The  C.  P.  A.  candidate  need  not  fear 
questions  in  compound  interest  when  the  rate  is  the  unknown,  as  such 
problems  are  solved  by  refeience  to  annuity  tables,  but  he  should  be  able 
to  approximate  the  rate,  if  given  access  to  annuity  tables  or  data  therefrom. 

Assume  a  problem  asking  at  what  i-ate  an  ordinary  annuity  of  four  rents 
of  $10,000  would  amount  to  $41,216.08,  if  the  available  annuity  table  did 
not  show  the  amounts  at  a  2  per  cent  basis,  but  showed  the  amounts  at 
1%  and  21/4  pei"  cents.  The  closest  amounts  in  the  four-period  line  would 
be  taken,  and  the  diffei-ence  made  by  an  increase  in  one  per  cent  ascertained. 

Amount  at  2M% $4.1.3703639 

.       Amount  at  1M% 4.10623036. 

Differences    }4% .f  .03080603,  or 

Differences      1% '    $.06161206 

Amount  at  unknown  rate $4.12160800 

Next  lower  amount 4.10623036 

Difference $.015  7764 

If  an  increa.se  $.06161206  is  caused  by  an  increase  of  1  per  cent,  the 
increase  of  $.01537764  must  be  caused  by  an  increase  of  .249  ( .01537764  -H 
.06161206)  per  cent.  Hence  the  approximate  rate  is  1.999  (1.75 +  .249) 
per  cent.'* 

Computing  Effective  Bond  Rates — When  a  bond  is  purchased  at  a  price 
not  listed  on  a  bond  table,  the  effective  interest  i-ate  may  be  approximated 
by  interpolation."     Assume  that  a  $10,000  5  per  cent  bond,  due  in  two 


INTEREST  171 

years,  Avas  sold  for  $10,190.39.  The  purchase  price  should  be  looked  up  on 
a  bond  table  on  the  four-period  line,  which  would  show  the  price  in  the 
two  per  cent  column.  If  the  price  was  not  given,  the  nearest  prices  on  the 
four-period  line  Avould  be  taken,  and  the  difference  made  by  an  increase 
of  one  per  cent  ascertained. 

Price  at        1%% $10,287.32 

Price  at       2}4% 10,094.62 

Differences    ^% $192.70,  or 

Differences      1  % $385.40 

Next  higher  price $10,287.32 

Price  at  unknown  rate 10,190.39 

Difference $96.93 

If  an  increase  of  $385.40  is  caused  by  an  increase  of  1  per  cent,  the 
increase  of  $96.93  must  be  caused  by  an  increase  of  .2515  (96.93  -^  385.40) 
per  cent.  Hence  the  approximate  effective  bond  rate  is  2.0015  (1.75  + 
.2515)  per  cent. 

Computing  an  Unknown  Time — The  unknown  time  can  be  approximated 
if  the  other  facts  regarding  the  annuity  are  available."  For  example, 
find  in  how  many  periods  rents  of  $5,000  will  amount  to  $93,196.43  at  2 
per  cent. 

Dividing  $93,196.43  by  5,000,  the  amount  of  an  annuity  of  $1  is  found 
to  be  18.639286. 

18.639286      =  ( ( 1 .02 ) "  —  1 )  -^  .02 
.37278572=  (1.02)"  — 1 
1.37278572=  (1.02)" 
1.37278571  =  (1.02)'", 
so  annuity  runs  sixteen  periods. 

It  should  be  noticed  that  in  the  above  illustration  sixteen  periods  is  not 
quite  correct,  but  it  is  close  enough  for  all  practical  purposes.  The  raising 
of  1.02  to  a  high  power  in  the  effort  to  match  1.37278572  can  be  readily 
done  by  rememl)ering  that  in  raising  numbers  to  powers  the  exponents 
are  added.  (1.02)"  can  be  calculated  bv  four  successive  multiplications, 
viz.,  (a)  1.02X1.02=  (1.02)';  (b)  "(1.02)' X  (1.02)' =  (1.02)*;  (c) 
(1.02)*  X  (1.02)*=  (1.02)';  and  (d)   (1.02)' X  (1.02)' =  (1.02)". 

Amount  of  Annuities  with  Interim  Interest  Dates — If  the  interest  is 
compounded  more  frequently  than  the  rents  are  payable,  the  amount  of 
such  an  annuity  would  be  found  by  dividing  the  total  compound  interest 
by  the  effective  interest  rate  per  rent  period."  Letting  the  annual  interest 
rate,  the  frequency  of  conversion,  and  the  number  of  years  be  respectively 
represented  by  "j,"  "ni,"  and  "n,"  the  formula  for  the  amount  of  an 
annuity  with  interim  interest  dates  is'" 

(1  + j/m)'""  — 1 
^  (1  +  j/m)'"   -1 


172  C.  P.  A.  ACCOUNTING 

For  example,  find  the  amount  of  an  ordinary  annuity  of  four  annual 
rents  of  $100,  if  the  rents  are  invested  at  4  per  cent  per  annum,  com- 
pounded semi-annually. 

(1.02)'  — 1 
The  amount  would  be :  $100  X  , =  $424.90 

y  ±  ,\)ct )    -L 

Present  Worth  of  Annuities  tvith  Interim  Interest  Dates — If  the  interest 
is  compounded  more  frequently  than  the  rents  are  payable,  the  present 
worth  of  the  annuity  would  l)e  found  by  di\dding  the  total  compound 
discount  by  the  effective  interest  rate  per  rent  period.'*  Letting  the  annual 
interest  rate,  the  frequencj'  of  conversion,  and  the  number  of  years  be 
respectively  represented  by  "j,"  "m,"  and  "n,"  the  formula  for  the  present 
worth  of  an  annuity  with  interim  interest  dates  is*" 

1 
1 


RX  ^^  +  -i/"^^' 


(H-j/m)rn  — 1 

For  example,  find  the  present  worth  of  an  ordinary  annuity  of  four 
annual  rents  of  $100,  if  the  rents  are  invested  at  4  per  cent  per  annum, 
compounded  semi-annually. 

""(1.02)" 
The  present  worth  would  be :  $100  X  ,_      =  $362.65 

Present  Worth  of  Deferred  Annuities — If  the  rents  of  an  annuity  do 
not  commence  to  run  until  after  a  certain  number  of  periods  have  elapsed, 
the  annuity  is  called  a  deferred  annuity."  If  an  annuity  of  $1  for  "x" 
periods  is  deferred  for  *S"  periods,  its  present  worth  can  be  found  either 
(a)  by  multiplying  the  present  worth  of  an  ordinai*y  annuity  of  $1  for 
"x"  periods  by  the  present  worth  of  $1  due  "y"  periods  hence,*"  or  (b)  by 
subtracting  the  present  worth  of  an  ordinary  annuitj'  of  $1  for  "y"  periods 
from  the  present  worth  of  an  ordinary  annuity  of  $1  for  "x"  plus  "y" 
periods."  If  a  bond  table  is  available,  the  second  method  is  preferable,  but 
otherwise  the  first  should  be  used. 

For  example,  find  the  present  worth  of  a  four-period  annuity,  whose  rent 
is  $10,000,  when  money  is  worth  two  per  cent  per  period,  if  the  annuity 
is  deferred  five  periods. 

First  Method. 
1 


(1.02)'  1 

X  ,X  $10,000  =  $34,487.77,  present  worth. 


.02 


Secon<l  Method. 


:i.02)'  (1.02)'^ 

-^02 Q2~J  X  $10,000  =  $34,487.77,  present  worth. 


INTEREST  173 

Amount  of  Annuities  Due — Annuities  whose  rents  are  due  at  the  end  of 
the  periods  are  "ordinary"  or  "immediate"  annuities,  but  annuities  whose 
rents  are  due  at  the  lie^inning  of  the  periods  are  called  "annuities  due."" 
The  amount  of  an  annuity  due  of  "n"  rents  can  be  found  either  (a)  by 
multiplying  the  amount  of  an  ordinary  annuity  under  the  same  conditions 
by  the  ratio  of  increase/''  or  (b)  by  subtracting  one  rent  from  the  amount 
of  an  ordinary  annuity  under  the  same  conditions  except  for  "n  +  1" 
periods/"  or  (c)  by  adding  the  compound  interest  on  the  first  rent  for  "n" 
periods  to  the  amount  of  an  ordinary  annuity  of  "n"  periods.*' 

For  example,  find  the  amount  of  a  four-period  annuity  due,  whose 
rent  is  $10,000,  when  money  is  worth  two  per  cent  per  period. 

First  Method. 

tl  02  V 1 

-— — ^— X  1.02  X  $10,000  =  $42,040.40,  amount  of  annuity  due. 

Second  Method. 
I  (1.02)  —]_  _  -^  I.  ^  $10,000  =  $42,040.40,  amount  of  annuity  due. 

Third  Method. 
[(1.02)^  —  1]  +  ^'^'^^\~~^[x  $10,000  =  $42,040.40,  amoimt  of  annuity 


.02 


due. 


Present  Worth  of  Annuities  Due — The  present  worth  of  an  annuity 
due  of  "n"  rents  can  be  found  (a)  by  multiplying  the  present  worth 
of  an  ordinary  annuity  under  the  same  conditions  by  the  ratio  of  increase," 
or  (b)  by  adding  one  rent  to  the  present  worth  of  an  ordinary  annuity 
under  the  same  conditions  except  for  "n  — 1"  periods,"'  or  (c)  by  adding 
the  compound  discount  on  the  first  rent  for  "n"  periods  to  the  present 
worth  of  an  ordinary  annuity  of  "n"  periods. 

For  example,  find  the  present  worth  of  a  four-period  annuity  due, 
whose  rent  is  $10,000,  when  money  is  worth  2  per  cent  per  period. 


1 


First  Method. 
1 


^"^•'^•^     X  1.02  X  $10,000  =  $38,838.83,  present  worth. 


Second  Method. 


r.  1 


V 


(1  02  V  I 

_ir — L+  1  fx  $10,000  =  $38,838.83,  present  worth. 


Third  Method. 


1 


(1.02)V, 
002?'  "^ 02~    ^  $10,000  =:  $38,838.83,  present  worth. 


174  C.  P.  A.  ACCOUNTINd 


QUESTIONS 

INTEREST 

1.  Distino:insh  between  sini))le  aiul  compound  interest.  (N.  Y.,  June. 
1913.) 

2.  Define:  Discount.     (Wash.,  July.  1917.) 

3.  Explain  four  different  uses  of  the  word  "discount"  as  used  in  ac- 
counting.    (Colo.,  Dec.,  1913.) 

4.  Classify  the  Interest  account.     (Iowa,  Dec.,  1918.) 

5.  How  should  interest  appear  on  a  Trading  statement?  (Iowa, 
Dec,  1918.) 

6.  (a)  What  are  the  elements  of  interest?  (6)  What  is  the  rate  of 
discount  corresponding  to  2  per  cent  interest?  (c)  In  a  4  per  cent 
bond  to  net  2^/2  per  cent,  what  is  the  difference  of  rates?  (Mich.,  June, 
1912.) 

7.  Why  is  unearned  interest  on  bills  receivable  seldom  taken  into  con- 
sideration in  ordinary  business  houses?     (S.  C,  Sept.,  1919.) 

8.  How  would  you  treat  such  aci;'ount  as  the  following,  if  closing  the 
books  at  a  certain  period,  with  a  view  to  ascertaining  the  loss  or  gain : 
Mortgages,  Notes,  and  other  paper  as  to  interest?     (N.  J.,  1904-1909.) 

9.  In  making  aii  audit  of  a  company,  you  find  that  they  have  notes 
receivable  bearing  interest,  some  paid  in  advance  and  included  in  the  face 
of  the  notes,  othei"s  with  interest  accruing.  Exi)lain  your  handling  of  this 
interest  in  your  report.     (Mich.,  June,  1910.) 

10.  State  what  is  indicated  by  the  Interest  account  (a)  when  the  ac- 
count shows  a  debit  balance;  (b)  when  the  account  shows  a  credit  bal- 
ance.    Explain  fully.     (N.  Y.,  Jan.,  1900.) 

11.  In  order  to  facilitate  the  preparation  of  monthly  Profit  and  Loss 
statements  by  a  corporation,  how  would  you  recommend  that  the  Interest 
Payable  be  treated  on  it.s  books  from  month  to  month?  (Wash.,  April, 
1906.) 

12.  Explain  the  inaccuracies  from  an  accounting  standpoint  of  the 
ordinary  Interest  and  Discount  account.  (Mich.,  Dec,,  1913;  N.  Y.,  Jan., 
1919.*) 

13.  Describe  fully  the  Interest  accoimt,  showing  what  entry  may  be 
made  on  each  side  and  what  disposition  should  be  made  of  the  balance. 
(N.  Y.,  June,  1898.) 


INTEREST  175 

14.  Explain  by  entries  the  proper  accounting  for  each  case  in  which 
interest  and  discount  is  involved  in  handling  both  Bills  Receivable  and 
Bills  Payable.     (Mich.,  Dec,  1913;  N.  Y.,  Jan.,  1919.*) 

15.  "A"  purchases  "BV  business  as  a  going  concern,  paying  part  of 
the  purchase  price  in  casli  on  the  date  of  the  transfer,  and  giving  interest- 
bearing  notes  for  the  balance.  How  would  you,  as  auditor,  expect  to  find 
the  interest  on  the  notes  treated  in  "A's"  books?     (Wash.,  April,  1906.) 

16.  A  large  hotel  is  furnished  on  the  installment  plan.  Explain,  giving 
reason,  whether  the  question  of  interest  is  of  importance  to  (a)  the  seller; 
(6)  the  buyer.     (N.  Y.,  June,  1913.) 

17.  In  making  a  detailed  audit  what  procedure  would  you  follow  to 
verify  the  Interest  Prepaid  on  Notes  Payable?  (Kan.,  Dec,  1915;  Mo., 
Dec,  1915.) 

18.  In  preparing  the  Balance  Sheet  of  a  business  at  the  close  of  a 
year,  how  should  j'ou  treat  Interest  Paid  in  Advance  on  Notes  Payable 
Discounted?     (Mass.,  June,  1910.) 

19.  In  a  statement  of  condition,  requiring  classification  of  assets  and 
liabilities  into  fixed  assets,  current  assets,  etc.,  in  what  class  would  you 

^lace  prepaid  interest?     State  reasons  for  your  answer.     (Mass.,  June, 
1912.) 

20.  A  corporation  redeems  part  of  its  outstanding  mortgage  bonds 
and  holds  the  bonds  as  an  asset,  collecting  the  interest  thereon  through 
its  fiscal  agent.  The  interest  thus  collected  is  regarded  by  the  officers 
of  l>e  company  as  income.  In  pieparing  a  statement  for  publication 
how  should  the  account  treat  such  interest?     (N.  Y.,  June,  1917.) 

21.  On  which  side  of  the  Ledger  should  the  balance  on  the  Interest 
Collected  in  Advance  account  appear?     (Mo.,  Dec,  1914.) 

22.  Define:  Accrued  interest.     (A.  I.  of  A.,  May,  1919.) 

23.  How  would  you  verify  the  correctness  of  accrued  interest?  (N.  Y., 
June,  1909.) 

24.  Formulate  Journal  entries  to  express  fully  the  following  transac- 
tion: The  adjusting  of  an  Interest  account  for  interest  earned  but  not  yet 
collected.     (Wash.,  May,  1903.) 

25.  What  in  your  opinion  is  the  proper  treatment  of  the  accounts 
named  below  in  arriving  at  the  profit  or  loss  of  a  business  for  a  specific 
fiscal  period:  Accrued  Interest  on  Notes  Receivable;  Accrued  Interest 
on  Notes  Payable?     (Ohio,  Dec,  1908.) 

26.  On  which  side  of  the  Ledger  should  the  balance  of  the  Interest 
Accrued  on  Notes  Receivable  account  appear?     (Mo.,  Dec,  1914.) 

27.  In  what  section  of  the  Balance  Sheet  and  in  what  order  would 
you  show  the  Interest  Accrued  Payable?     (A.  I.  of  A.,  Nov.,  1919.) 

28.  State  in  the  form  of  Journal  entries  the  following  transaction: 
the  adjustment  of  interest  accrued  but  not  yet  payable  on  a  mortgage. 
(N.  Y.,  June,  1898.) 


176  C.  P.  A.  ACCOrNTIN(J 

29.  Define:  Annuity.     (Ohio.  Nov.,  1916.) 

30.  Define:  Ratio  of  increase;  accumulation.  (Kan.,  Dec,  1915;  Mo., 
Dec.,  1915.) 

31.  Define:  Effective  rate;  Nominal  rate.  (N.  Y.,  June,  1913*;  Kan., 
Dee.,  1915;  Mo.,  Dec.,  1915.) 

32.  What  do  you  understand  by :  "present  worth  of  a  deferred  pay- 
ment"; "amount  of  an  annuity"?     (Kan.,  Dec,  1915;  Mo.,  Dec,  1915.) 

33.  Define:  Amortization.  (N.  Y..  Jan.,  1911;  Colo.,  Dec,  1913;  111., 
May,  1914*;  Mich.,  Dec.  1914;  Del.,  June,  1915*;  Kan.,  Dec,  1915;  Mo., 
Dec,  1915;  Ohio,  Nov.,  1916;  Wis.,  April,  1917;  A.  I.  of  A.,  Nov.,  1917; 
Ind.,  Nov.,  191S*;  111.,  Dec,  1918;  A.  I.  of  A.,  May,  1921.) 

34.  What  is  an  Amortization  account?     (N.  D.,  July,  1916.) 

35.  (a)  What  is  the  principle  of  amortization?  (ft)  How  and  when 
used?    Illustrate.     (Iowa,  Dec,  1918.) 

36.  Explain  the  method  of  valuinj;  permanent  investments  in  bonds 
on  an  amortization  principle.     (Mass.,  Oct.,  1914.) 

37.  Under  what  theoiy  is  the  present  value  of  deferred  payments  ascer- 
tained?    (N.  Y.,  Jime,  1917.) 

38.  Describe  the  annuity  method  of  writing  off  the  premium.  Is  there 
any  objection,  in  your  opinion,  to  this  method?  If  so,  state  it.  (Mo., 
Dec,  1913.) 

39.  What  mathematical  procedure  would  you  adopt  to  amortize  the 
premium  on  bonds  having  a  number  of  years  to  run?  (N.  Y.,  Jan.,  1914; 
N.  D.,  Aug.,  1917.) 

40.  How  would  you  determine  the  amount  of  a  sinking  fund  install- 
ment if  you  were  unfamiliar  with  the  use  of  mathematical  tables  and 
knew  no  mathematics  beyond  arithmetic?  Explain  and  illustrate.  (N.  Y., 
Jan.,  1917.) 


INTEREST 


17i 


PROBLEMS 


INTE31EST 


1.  What  sum  invested  at  6  per  cent  compounded  annually  will  amount 
to  $1,000  in  five  years?     (Cal.,  June,  1904.) 

2.  The  present  value  of  an  annuity  of  $1  for  four  periods  at  2  per 
cent  is  $3.80772870.  What  is  tlie  value  on  January  1,  1914,  of  a  5  per  cent 
per  annum  bond  issue  of  $100,000  bought  on  a  4  per  cent  per  annum  basis 
(semiannual  coupons),  due  January  1,  1916? 

Prepare  amortization  table  as  follows : 


Date 
1-1-14 
7-1-14 
1-1-15 
7-1-15 
1-1-16 


Totallnterest       Income       Amortization    Book  Value 


2J^  per  cent 


2  per  cent 


Par  Value 


$100,000 


Insert  values  under  the  various  heads  to  the  nearest  cent. 

(Kan.,  Dec,  1915;  Mo.,  Dec,  1915.) 

3.  In  auditing  the  books  of  a  corporation  you  find  that  in  order  to 
provide  a  sum  to  redeem  a  mortgage  of  $100,000  falling  due  at  the  end  of 
ten  years,  a  reserve  of  $8,000  per  annum  has  been  set  aside  annually  for 
three  years,  but  that  contrary  to  intention,  tlie  company  has  failed  to 
accumulate  interest  thereon.  Assuming  interest  at  4  per  cent  (convertible 
annually),  what  should  have  been  the  total  accumulations  to  date,  and 
what  amount  should  now  be  set  aside  annually  for  the  next  seven  years 
in  order  to  complete  the  sinking  fund?  (1.04)^=1.31593.  (A.  I.  of  A., 
May,  1918.) 

4.  A  lease  has  5  years  to  run  at  $1,000  a  year,  payable  at  the  end  of 
each  year,  with  an  extension  for  a  further  5  years  at  $1,200  a  year.  On 
a  6  per  cent  basis,  what  sum  should  be  paid  now  in  lieu  of  10  years' 
rent  (V^  at  6  per  cent=.7473)?     (A.  I.  of  A.,  May,  1919.) 

5.  "A"  owns  an  annuity  of  $50  per  annum,  the  first  payment  on  which 
falls  due  1  year  hence  and  which  continues  for  a  period  of  20  years 
certain.  State:  (a)  the  present  value  of  the  benefit;  (b)  the  amount  which 
he  will  have  aeeunuilated  at  the  end  of  the  period  if  he  invests  each  moiety 
as  it  becomes  due. 


178  C.  P.  A.  ACCOUNTING 

Assume  interest  at  4  per  cent  annually.  In  -this  connection  tlic  value 
of  1.042"  js  gtatotl  to  be  equal  to  2.191123. 

(A.  I.  of  A.,  Nov.,  1917.) 

6.  An  annuily  of  $3,G00  at  6  per  cent  compound  interest  is  to  be  paid 
in  four  quarterly  payments.  Furnish  the  annuity  depreciation  account 
with  quarterly  balances.     (N.  Y.,  June,  1919.) 

7.  The  present  value  of  an  annuity  of  $1,000,  payable  in  half-yearly 
installments  for  45  years,  at  the  rate  of  6  per  cent,  semiannually,  bein<? 
$15,501.20,  what  is  the  value  of  the  same  annuity  if  paid  annually,  the 
rate  being  the  same  as  before?  (Ohio,  Nov.,  1915.)  (Note:  45th  power 
of  1.06  is  13.76401083,  Avhose  reciprocal  is  .07205007.) 

8.  Purchased  May  1,  1904,  ex  coupon,  $100,000  of  5  per  cent  bonds, 
interest  payable  May  1  and  November  1,  on  the  basis  of  4  per  cent  semi- 
annually, principal  to  mature  May  1,  1929.  Required  the  amortizement 
of  the  premium  for  the  half  year  endius:  November  1,  1924.  (Ohio,  Nov., 
1915.)  (Note:  9th  power  of  1.02  is  1.19509257,  whose  reciprocal  is 
.83675527.) 

9.  The  discount  on  $1  for  35  years,  at  3V2  per  cent,  semiannually, 
being  $0.7031133,  what  is  the  value  of  $100,000  of  bonds  paj'ing  5  per 
cent,  semiannually,  and  maturing  in  34%  years,  the  investment  rate  being 
the  same  as  before?     (Ohio,  Nov.,  1915.) 

10.  If  $100,000  of  bonds,  paying  $3,000  each  six  months  (May  1  and 
November  1)  is  worth  $123,408.34  on  May  1,  1915,  ex  coupon,  and  the 
investment  rate  is  4  per  cent  semiannually,  what  is  the  present  value  of 
an  annuity  of  $1,750,  payable  annually  during  the  life  of  the  bonds,  the 
investment  rate  being  the  same  as  before?  (Ohio,  Nov.,  1915.)  (Note: 
32nd  power  of  1.02  is  1.88454059,  whose  reciprocal  is  .53063330.  10th 
power  of  1.04  is  1.87298125,  whose  reciprocal  is  .53390818.) 

11.  Smith's  baby  owns  an  annuity  of  $100  per  annum,  the  first  pay- 
ment on  which  falls  due  1  year  hence,  which  continues  for  a  period  of  20 
years  certain. 

You  are  asked  to  state:  (a)  The  present  value  of  the  benefit;  (ft)  the 
amount  which  he  will  have  accumulated  at  the  end  of  the  period,  if  he 
invests  each  payment  as  it  becomes  due;  (c)  an  arithmetical  formula  for 
computing  compound  interest  for  the  period.  Assume  interest  at  4  per 
cent,  payable  annually.  The  value  of  $1  at  4  per  cent  compound  interest 
is  assumed  to  be  $2.191123. 

(Ind.,  May,  1918.) 

12.  If  a  principal  of  $1  will  amount  to  $3.3863549  in  25  years,  at  5 
per  cent  per  annum,  what  would  be  the  present  value  of  an  annuity  of 
$1,250  for  30  years  at  the  same  rate?  (Ohio,  Nov.,  1915.)  (Note:  30th 
power  of  1.05  is  4.32194238,  whose  reciprocal  is  .23137745.) 

13.  A  manufacturer  owes  $100,000  on  his  plant  at  5  per  cent  per 
annum,  due  at  the  end  of  the  5  years  from  date.  He  secures  the  agree- 
ment, however,  to  pay  the  debt  in  equal  installments,  which  will  include 
interest  and  principal.     What  amount  is  he  required  to  pay  each  year? 


INTEREST  179 

(Pa.,  Nov.,  1912.)     (Note:  5th  power  of  1.05  is  1.27628156,  whose  recip- 
rocal is  .78352617.) 

14.  A  corporation  needing  some  additional  capital  for  a  short  term 
of  years,  issues  .f.300,00()  of  debenture  bonds  carrying  6  per  cent  interest, 
payable  one-fifth  each  year  for  five  years.  Coupons  are  attached  to  the 
bonds  maturing  every  six  months;  the  bonds  are  sold  at  90  flat. 

What  average  rate  of  interest  does  the  company  pay  for  the  money, 
excluding  interest  on  interest? 

(Ill,  Nov.,  1904.) 

15.  A  certain  issue  of  $100,000  4  per  cent  bonds  is  dated  September 
1,  1908,  and  interest  begins  at  that  date,  but  interest  is  payable  on  Feb- 
ruarj"-  1  and  August  1,  and  the  principal  (with  4  months'  interest)  is 
payable  December  1,  1912.  What  is  the  value  of  these  bonds  on  a  3.60 
basis  at  that  date  of  issue?  What  is  their  value  on  the  same  basis,  if  pur- 
chased on  December  1,  1908?  (Note  that  you  are  interloping  into  a  five- 
month  period,  not  a  six-month,  in  the  beginning.)  (Mich.,  July,  1909.) 
(Note:  7th  power  of  1.018  is  1.1330121,  whose  reciprocal  is  .882603.) 


CHAPTER  X 

RESERVES  AND  FUNDS 

Classes  of  Beserves — The  term  "reserve  accounts"  has  been  indiscrimi- 
nately applied  to  items  which  are  essentially  different/  In  financial  insti- 
tutions, the  term  "reserve"  indicates  the  amount  of  cash  and  cash  items 
on  hand  and  on  deposit,  which  may  legally  be  counted  as  a  cash  reserve 
fund  held  against  deposits." 

The  following  table  shows  the  classes  of  reserves  found  in  ordinary 
businesses : 

A.  Valuation  Reserves : 

(a)  Asset  valuation  reserves. 
I.  Depreciation  reserves. 

1.  For  buildings. 

2.  For  machinery. 

3.  For  equipment. 

4.  For  merchandise,  etc. 
II.  Depletion  reserves. 

1.  For  mines. 

2.  For  timber. 

3.  For  oil,  etc. 

III.  Bad  debts  reserves. 

1.  For  uncollectible  accounts  receivable. 

2.  For  sales  discounts. 

3.  For  uncollectible  notes  receivable,  etc. 

IV.  Market  fluctuations  reserves. 

1.  For  merchandise  inventories. 

2.  For  securities,  etc. 

(b)  Liability  valuation  reserves. 
I.  Operating  reserves. 

1.  For  taxes. 

2.  For  accrued  wages. 

3.  For  accrued  salaries. 

4.  For  accrued  rents,  etc. 

B.  Proprietorship  Resei'ves : 
(a)  Recorded  reserves. 

I.  Appreciation  reserves. 

1.  For  buildings  and  machinei'y. 

2.  For  land. 


'For  explanation  of  superior  figures  see  page  337. 

180 


RESERVES  AND  FUNDS  181 

II.  Contingency  reserves. 

1.  For  fire  insurance. 

2.  For  flood  loss. 

3.  For  accidents. 

4.  For  guarantees. 

5.  For  pending  law  suits. 

6.  For  obsolescence,  etc. 
III.  Open  reserves. 

1.  For  debt  extinguishment, 

2.  For  plant  extension, 

3.  For  betterment  and  improvements. 

4.  For  working  capital, 

(b)  Unrecorded  reserves. 
I.  Secret  reserves, 

1,  Understatement  of  assets. 

2,  Overstatement  of  liabilities, 

3,  Overstatement  of  asset  valuation  reserves. 

Asset  Valuation  Reserves — Asset  valuation  reserves  represent  the  provi- 
sion made  for  losses  actually  sustained  in  the  value  of  assets,  or  anticipated 
losses  known  to  be  certain,'  Their  balances  are  suspended  or  deferred  cred- 
its to  the  corresponding  assets,  and  are  neither  liabilities  nor  surplus.*  Al- 
though occasionally  found  on  the  right  side  of  the  balance  sheet,"  asset 
valuation  reserves  should  be  deducted  from  their  related  assets  on  the 
balance  sheet,' 

The  depreciation  and  bad  debts  reserves  are  thoroughly  discussed  in 
Chapter  XV,  Volume  I,  and  Chapter  III,  Volume  II,  I'espectively,  As 
leasehold  property  and  patents  become  valueless  after  a  certain  length 
of  time,  reserves  for  amortization  of  improvements  on  leasehold  property' 
and  for  expiration  of  patents  should  be  created ;'  both  reserves  being  asset 
valuation  reserves  of  the  depreciation  gi'oup. 

The  depletion  reserves  are  credited  with  the  estimated  amount  of  wasting 
assets  consumed,*  The  related  asset  minus  the  reserve  gives  the  net  book 
value  of  the  remaining  wasting  asset. 

Market  fluctuation  reserves  are  credited  for  excess  of  the  cost  price  of 
the  related  asset  over  its  market  price,*"  At  the  close  of  each  period  the 
reserve  is  adjusted  so  that  it  equals  this  excess,  the  offsetting  entry  being 
to  profit  and  loss,"  Fluctuations  above  cost  are  disregarded  on  the  balance 
sheet  except  for  parenthetical  references," 

Operating  Resettles — Operating  reserves  are  not  appropriations  of  sur- 
plus," but  are  liabilities."  They  may  be  divided  into  those  offsetting 
estimates  of  expenses,  the  exact  amounts  of  which  are  not  known,  and 
those  offsetting  expenses,  the  exact  amounts  of  which  are  known  but  are 
unpaid,"  The  reserve  for  taxes  is  an  illustration  of  the  first  group,  while 
the  reserves  for  accrued  wages,  salaries,  and  rents  are  illustrations  of  the 
second  group," 


182  C.  P.  A.  ACCOUNTING 

The  use  of  the  title  "reserve"  for  items  covered  by  the  operating  reserves 
is  unfortunate,  except  in  cases  where  the  amount  of  the  expense  items  can 
not  be  approximately  predetei'mined.''  Thus  it  would  be  preferable  to  use 
the  title  "wages  accrued"  rather  than  "reserve  for  wages,"  but  the  use  of 
the  title  "reserve  for  taxes"  may  be  justified."  Many  accountants,  however, 
use  the  term  "accrued  taxes"  instead  of  "reserve  for  taxes."" 

Appreciation  Reserves — An  appreciation  reserve  represents  the  un- 
realized profit  put  on  the  books  when  the  assets  were  raised  to  the  appraisal 
value,  the  reserve  being  credited  instead  of  surplus  to  prevent  the  un- 
realized profit  from  being  declared  as  dividends.*" 

Contingency  Reserves — It  is  extremely  difficult  to  draw  a  dividing  line 
between  operating  and  contingency  reserves,  it  being  a  matter  of  degi-ee 
as  to  the  certainty  of  the  events  happening  for  which  provision  is  being 
made."  Since  the  liabilities  covered  by  contingency  reserves  are  not  likely 
to  materialize,  the  reserves  are  surplus  temporarily  debarred  from  dividend 
distribution.*^ 

The  operation  of  a  contingency  reserve  is  shown  in  Chapter  XIV, 
Volimie  I,  under  the  caption  "reserve  for  insurance." 

Secret  Reserves — Secret  reserves  represent  the  excess  of  the  actual  net 
worth  of  a  concern  over  the  amount  indicated  on  its  balance  sheet."  They 
may  be  created  either  by  undervaluing  the  assets"  or  overstating  the 
liabilities;^  the  former  is  the  more  popular  method  as  the  creditors  them- 
selves check  up  all  actual  liabilities.*"  The  actual  value  of  the  firm's  net 
worth  should  be  conservatively  valued,  but  an  auditor  should  not  counte- 
nance secret  reserves  as  they  prevent  the  balance  sheet  from  being  a  true 
statement  of  condition.*' 

An  advantage  claimed  for  secret  reserves  is  that  extraordinary  losses, 
which  would  result  in  very  unfavorable  fluctuations  in  stock  values,  may 
be  charged  against  them.*'  However,  an  extraordinary  loss  should  not  be 
concealed  from  the  prospective  investor.  The  claim  that  secret  reserves 
have  the  advantage  of  permitting  the  maintenance  of  a  uniform  di\'idend 
rate**  must  be  admitted,  but  the  desired  result  can  also  be  obtained  with 
an  open  surplus  account. 

Secret  reserves  are  objectionable  because  they  prevent  stockholders  from 
recei\nng  their  share  of  the  profits  made  during  the  period  of  their  stock 
ownership,*"  and  from  receiving  the  true  value  of  the  stock  in  case  of  sale." 
Secret  reserves  are  also  objectionable  as  they  afford  the  directors  an 
opportunity  of  concealing  losses  due  to  speculation  and  mismanagement." 

Open  Proprietorship  Reserves — The  open  proprietorship  reserves  repre- 
sent profits  reinvested  in  the  business  and  withheld  from  distribution  to 
stockholders."  These  reserves  are  frequently  called  the  true  reserves.** 
They  are  nothing  but  subdivisions  of  surplus  and  should  be  shown  as  such 
on  the  balance  sheet."  The  separation  of  a  portion  of  surplus  into  open 
proprietorship  reserves  is  almost  entirely  a  matter  of  bookkeeping,  de- 
signed to  inform  stockholders  that  the  amounts  so  set  aside  will  not  be 
distributed  as  dividends."    A  surplus  account  is  an  open  proprietorship 


RESERVES  AND  FUNDS  183 

reserve  account  representing  pj-ofits  reserved  for  general  rather  than  for 
speeifie  purposes."  Open  proprietorsliip  reserves  are  created  by  debiting 
surplus  directly/*  and  after  the  purposes  of  accumulating  the  capital, 
such  as  debt  extinguishment  and  plant  extension,  have  been  accomplished, 
the  reserves  should  be  closed  back  into  surplus.'* 

Funds — Funds  are  debit  accounts  consisting  of  money  or  securities  set 
aside  for  specific  purposes.^"  They  are  financial  expedients  to  ensure  the 
possession  of  ready  money  to  meet  the  object  for  which  the  fund  is  estab- 
lished when  the  necessity  for  its  use  arises." 

Unless  protected  by  directors'  resolutions,  funds  can  be  used  for  pur- 
poses other  than  those  for  which  they  were  created,"  or  can  be  reverted  to 
general  cash."     Funds  are  current  assets." 

Reserve  Funds — Some  accountants  use  the  term  "reserve  fund"  to  mean 
a  surplus  reserve."  This  use  is  incorrect,  for  reserve  funds  are  assets  set 
aside  for  a  purpose,  for  the  accomplishment  of  which  a  reserve  has  also 
been  established.'^  If  money  is  set  aside  for  a  definite  purpose,  a  fund  is 
created."  If  profits  are  retained  in  the  business  for  the  same  purpose,  a 
reserve  is  created.^*  If  both  a  fund  and  reserve  are  created  for  the  same 
purpose,  the  fund  is  a  re.serve  fund." 

Sinking  Fund — A  sinking  fund  is  assets  set  aside  for  the  purpose  of 
meeting  a  debt  or  retiring  an  issue  of  stock."  Sinking  funds  are  usually 
increased  by  more  or  less  regular  amounts  set  aside  at  regular  intervals." 
These  installments  are  usually  placed  in  the  hands  of  trustees,"  who  either 
place  the  money  in  banks  or  purchase  gilt-edged  securities  with  it."  Re- 
gardless of  the  fact  that  many  deeds  of  trust  require  that  sinking  funds 
be  created  out  of  pi'ofits,  the  sinking  fund  should  be  still  created  by  setting 
aside  cash,  but,  Avhen  this  is  done,  an  equal  amount  should  be  reserved  from 
profits  thus  creating  a  sinking  fund  reserve  in  addition  to  the  sinking 
fund." 

If  the  sinking  fund  is  in  the  hands  of  the  company,  the  auditor  should 
verify  the  sinking  fund  securities  and  deposits."  If  the  sinking  fund  is 
controlled  by  a  trustee,  the  auditor  should  secure  a  certificate  as  to  the 
funds.'"  If  the  sinking  fund  has  been  invested  in  the  corporation's  own 
bonds  which  have  been  surrendered  by  the  trustee,  the  auditor  must  ex- 
amine them  and  ascertain  whether  illegal  use  of  them  has  been  rendered 
impossible." 

When  sinking  funds  are  shown  on  the  balance  sheet,  differentiation 
should  be  made  between  the  investments  which  have  been  made  and  the 
cash  on  hand.''  Sinking  fund  holdings  of  the  corporation's  own  bonds 
payable  may  be  either  listed  on  the  balance  sheet  among  the  assets'"  or 
deducted  from  the  outstanding  bonds  payable,*"  the  latter  method  seeming 
preferable  as  treasury  bonds  are  not  assets  any  more  than  treasury  stock. 
If  the  sinking  fund  I)onds  are  canceled,  they  must  either  be  deducted  from 
the  outstanding  bonds  payable  on  the  balance  sheet''  or  only  the  net  out- 
standing bonds  shown,"'  tlie  latter  method  seeming  preferable  as  canceled 
debts  are  not  usuallv  shown  on  the  balance  sheet. 


184  C.  P.  A.  ACCOUNTING 

Income  derived  from  sinking  funds  may  be  turned  over  to  the  company, 
retained  by  the  trustee  as  an  addition  to  the  sinking  fund,  or  applied  as  a 
deduction  from  the  next  sinking  fund  installment."'  This  income  should 
be  shown  in  a  special  account,  which  should  be  closed  into  the  corpora- 
tion's ordinary  profit  and  loss  account.*^ 

Although  premium  or  discount  on  bonds  piirchased  as  sinking  fund 
investments  is  frequently  written  oft"  at  once  to  the  profit  and  loss  account," 
it  should  be  amortized  over  the  life  of  the  bonds."  If  the  bonds  are  can- 
celed, the  pi'emiura  or  discount  thereon  should  be  written  otf  at  once  to 
profit  and  loss." 

When  the  bonds  covered  by  the  sinking  fund  mature,  the  sinking  fund 
(outside)  securities  are  converted  into  cash,'*  any  ditferencc  in  the  realized 
and  book  values  of  the  securities  being  written  oft"  to  the  profit  and  loss 
account.  The  cash  is  then  used  by  the  trxistee  to  redeem  the  bonds,'*  and 
any  cash  remaining  after  the  redemption  of  the  bonds  is  turned  over  to 
the  corporation.'*  Treasury  bonds  in  sinking  fund  would  then  be  canceled 
and  written  off  against  bonds  outstanding.'' 

Entries  for  Sinking  Fund — The  following  entries  show  a  complete  his- 
tory of  a  sinking  fund.  The  account  titles  are  suggestive  only,  as  many 
varying  titles  are  used. 

Entry  No.  1 

Sinking  Fund  Cash  Held  by  Trustee xxxxxxx 

Cash xxxxxxx 

(For  sinking  fund  installment.) 

Entry  No.  2 

Sinking  Fund  Investments xxxxxxx 

Premiums  on  Sinking  Fund  Investments xxxxxxx 

Sinking  Fund  Cash  Held  by  Trustee xxxxxxx 

(Purchase  at  premiiun  of  bonds  for  sink- 
ing fund.) 

Entry  No.  3 

Sinking  Fund  Expenses xxxxxxx 

Sinking  Fund  Cash  Held  by  Trustee xxxxxxx 

(For  expenses  chargeable  to  fund.) 

Entry  No.  4 

Sinking  Fund  Cash  Held  by  Trustee xxxxxxx 

Sinking  Fund  Income xxxxxxx 

Premium  on  Sinking  Fund  Investments ....  xxxxxxx 

(For  interest  received  on  sinking  fund 
bonds  and  amortization  of  premium 
on  same.) 

Entry  No.  5 

Sinking  Fund  Income xxxxxxx 

Sinking  Fund  Expenses xxxxxxx 

Profit  and  Loss  Account xxxxxxx 

(For  closing  net  sinking  fund  income 
into  profit  and  loss.) 


RESERVP]S  AND  FUNDS  185 

Entry  No.  6 

Sinking  Fund  Cash  Held  by  Trustee xxxxxxx 

Loss  on  Sale  of  Sinking  Fund  Investments .  .  .     xxxxxxx 

Sinking  Fund  Investments xxxxxxx 

(For  cash  received  and  loss  incurred  on 
sale  of  sinking  fund  securities.) 

Entry  No.  7 

Profit  and  Loss  Account xxxxxxx 

Loss  on  Sale  of  Sinking  Fund  Investments .  .  xxxxxxx 

(To  close  loss  on  sale  into  profit  and  loss.) 

Entry  No.  8 

First  Mortgage  6%  Bonds xxxxxxx 

Sinking  Fund  Cash  Held  by  Trustee xxxxxxx 

(For  redemption  of  bonds.) 

Entry  No.  9 

Cash xxxxxxx 

Sinking  Fund  Cash  Held  by  Trustee xxxxxxx 

(For  cash  turned  over  by  trustee  to  com- 
pany.) 

Reserve  for  Sinking  Fund — While  it  is  not  necessary  to  have  a  sinking 
fund  reserve  to  have  a  sinking  fund,  and  vice  versa,  accounts  are  usually 
seen  together  due  chiefly  to  the  fact  that  so  many  deeds  of  trust  require 
that  sinking  funds  be  created  out  of  profits."  As  sinking  funds  can  only 
be  created  out  of  assets,  a  sinking  fund  reserve  account  is  usually  created 
by  charging  surplus"  immediately  on  the  creation  of  the  fund  (Entry 
No.  1,  page  184).  The  profit  and  loss  account  is  frequently  debited  at  the 
creation  of  sinking  fund  reserves,'*  but  it  is  preferable  to  debit  surplus 
directly  as  such  action  emphasizes  the  fact  that  the  reserve  is  a  part  of 
surplus. 

If  the  premium  or  discount  on  sinking  fund  investments  is  gradually 
amortized,  as  shown  in  entry  No.  4  above,  no  adjustment  with  the  sinking 
fund  reserve  is  necessary.  If  the  premium  and  discount  are  written  off 
immediately,  the  former  should  be  debited  and  the  latter  credited  to  the 
sinking  fund  reserve,  as  the  reserve  should  be  kept  in  harmony  with  the 
fund."  After  the  net  profit  or  loss  from  the  sinking  fund  has  been  trans- 
ferred to  profit  and  loss  (see  entry  No.  5  above)  and  then  to  surplus,  the 
net  increment  or  decrement  should  be  transferred  from  surplus  to  the 
sinking  fund  reserve." 

After  the  losses  and  gains  on  sale  of  sinking  fund  investments  have  been 
transferred  to  profit  and  loss  (see  entry  No.  7  above),  the  sinking  fund 
reserve  must  be  brought  into  agreement  with  the  sinking  fund,"  the  off- 
setting entries  being  to  surplus. 

After  the  bonds  have  been  redeemed  and  the  excess  cash  has  been  turned 
over  by  the  trustee  to  the  company  (see  entries  No.  8  and  No.  9  above),  the 
sinking  fund  reserve  is  turned  back  into  surplus," 


18G  C.  P.  A.  ACCOUNTING 


QUESTIONS 

reskives  and  funds 

Reserves 

1.  Define:  Reserve  account.  (Md.,  Oct.,  1903* ;  N.  Y.,  Feb.,  190S* ; 
Mich.,  June,  1908*;  Ohio,  March,  1910*:  Va.,  Nov.,  1910:  Wash..  May, 
1911*:  Wash.,  June,  1912;  Mich.,  June,  1912*;  Wis.,  April,  1914;  Wis., 
May,  1916;  Ohio,  Nov.,  1916*:  N.  D.,  Aug.,  1917;  Mass.,  Oct.,  1917; 
Va.,  Nov.,  1918;  Iowa,  Dec,  1918.) 

2.  Define:  Qualified  reserve.     (111.,  Dec.,  1918.) 

3.  Name  various  ways  in  which  reserves  may  appear  on,  or  affect  the 
items  of,  a  Balance  Sheet.     (Wis.,  April,  1914.) 

4.  Discuss  briefly  the  various  kinds  of  reserves  you  are  familiar  with 
and  state  how  they  should  be  handled  on  the  books  and  on  the  Balance 
Sheet.     (Pa.,  Nov!,  1917.) 

5.  Distinguish  between  "voluntary  reserves"  and  "necessar\'  reserves" 
and  how  would  you  treat  such  on  a  Balance  Sheet?     (N.  C,  June,  1916.) 

6.  Are  sinkinof  fund  or  debt  extinguishment  reserves  a  charge  against 
income  or  are  they  allocations  of  the  surplus?     (N.  C,  June.  1916.) 

7.  Under  what  conditions  would  you  group  reserves  with  surplus  upon 
a  general  Balance  Sheet?     (Cal.,  Nov.,  1916.) 

8.  Distinguish  and  name  some  of  the  resei'v^e  accounts  created  or  pro- 
duced by  a  charge  to  operating  expenses,  from  that  class  of  reserve  ac- 
counts created  or  produced  by  a  charge  to  the  surplus  account.  (N.  C, 
June,  1919;  N.  C,  Sept.,  1919;  N.  C,  Nov.,  1919.*) 

9.  Define  operating  reserves  and  illustrate  briefly  their  uses  in  two 
separate  instances.     (Minn.,  Oct.,  1916;  111.,  Dec.,  1918.) 

10.  What  is  your  understanding  of  the  difference  between  Liability 
Reserves  and  Surplus  Reserves,  and  name  examples  of  each  class?  (N.  C, 
Nov.,  1918.) 

11.  Explain  the  difference  between  real  reserves  and  nominal  reser\'es. 
Give  two  examples  of  each.     (A.  I.  of  A.,  May,  1920.) 

12.  From  the  viewpoint  of  a  Balance  Sheet,  what  is  the  logical  place 
of  (a)  reserves  for  depreciation  of  physical  a.ssets,  created  by  charges  to 
operations;  (h)  operating  reserves;  (c)  reserves  for  redemption  of  lia- 
bilities; {d)  reserves  for  contingencies?  Give  reasons  for  your  opinion 
(N.  Y.,  Jan.,  1914.) 


RESERVES  AND  FUNDS  187 

13.  Under  what  condition  and  upon  what  basis  would  you  set  up  the 
following  reserves  and  how  would  you  finally  dispose  of  them:  (a)  Sinking 
funds;  (&)  appreciation  of  leasehold;  (c)  depreciation  of  leasehold;  (d) 
exhaustion  of  oil  wells?     (Cal.,  Nov.,  1916.) 

14.  (a)  State  whether  each  of  the  following  accounts  is  a  liability 
account  or  a  proprietary  interest  account:  (1)  Reserve  for  Depreciation; 
(2)  Reserve  for  Extensions;  (3)  Reserve  for  Bad  Debts;  (4)  Reserve  for 
Contingencies;  (5)  Reserve  for  Inventory  Adjustment;  (6)  Reserve  for 
Sinking  Fund. 

(b)  Discuss  the  accuracy  of  the  terminology  employed  in  each  of  the 
above  account  names. 

(Wis.,  April,  1917.) 

15.  How  are  reserves  created?     (Wash.,  May,  1911.) 

16.  For  what  purpose  are  reserves  created?  (Md.,  Oct.,  1903;  Mich., 
June,  1912.) 

17.  How  are  the  Reserve  accounts  used?     (Ohio,  March,  1910.) 

18.  Show  proper  classification  of  a  reserve  on  the  debit  and  credit 
sides  of  a  Balance  Sheet.     (Mich.,  June,  1908.) 

19.  How  may  a  Reserve  account  be  properly  established  and  for  what 
purpose?  What,  if  any,  contra  account  should  be  maintained?  Under 
what  circumstances  should  these  accounts  be  maintained?  Why?  (N.  Y., 
Feb.,  1908.) 

20.  How  would  a  Reserve  affect  the  book  value  of  capital  stock? 
(Mass.,  Oct.,  1917.) 

21.  Discuss  briefly  the  following  statement:  A  reserve  is  always  cap- 
ital.    (Cal.,  Nov.,  1916.) 

22.  A  corporation  having  issued  first  mortgage  bonds  to  the  amount 
of  $50,000  sets  aside  out  of  the  profits  $5,000  each  year  and  pays  off  at 
par  bonds  to  a  similar  amount.  How  .shall  these  items  appear  in  a 
Balance  Sheet  at  the  end  of  five  years?     (Mass.,  Oct.,  1916.) 

23.  What  is  a  "contributed  reserve"?  Commonly  employed  by  what 
kind  of  organization?     (Iowa,  Dec,  1918.) 

24.  Explain  "contingency  reserve."  How  would  it  affect  the  book  value 
of  capital  stock?    (Mass.,  Oct.,  1917;  A.  I.  of  A.,  May,  1921.*) 

25.  In  the  Balance  Sheet  of  a  corporation  how  should  the  Reserve  for 
Contingencies  be  stated?     (111.,  May,  1912.) 

26.  In  connection  with  the  audit  of  the  books  of  a  corporation  describe 
briefly  the  procedure  you  would  consider  to  be  essential  in  verifying  the 
Reserve  for  Allowances  and  Discounts.     (Wash.,  July,  1917.) 

27.  Define :  Reserve  for  Amortization  of  Improvements  on  Leasehold 
Property.     (Wash.,  July,  1917.) 

28.  Define:  Reserve  for  Expiration  of  Patent  Rights.  (Wash.,  July, 
1917.) 

29.  Classify  Reserve  for  Depreciation  on  Buildings  according  to  the 
subdiNasion    of    assets,    liabilities,    proprietary    interest,    income   and    ex- 


188  C.  P.  A.  ACCOUNTING 

penses  under  which  it  should  be  ^ouped.     (N.  Y,,  Jan.,  1907*;  Mass., 
June,  1912*;  Wis.,  May,  1919.) 

30.  Classify  Reserve  for  Income  and  Excess  Profits  Taxes  according 
to  the  subdivision  of  assets,  liabilities,  proprietary  interest,  income  and 
expense  under  which  it  should  be  grouped.     (Wis.,  May,  1919.) 

31.  To  what  extent  would  you  consider  it  necessary  to  verify  the 
Reserve  for  Depreciation  of  Plant  and  the  Reserve  for  Sales  Discount, 
and  what  reference  to  such  verification  would  you  make  in  your  report? 
(Mass.,  June,  1913.) 

32.  What  provision,  if  any,  would  you  make  for  income  and  excess 
profits  taxes  in  closing  accounts  before  the  passing  of  a  pending  act 
levj'ing  these  taxes,  either  in  general  circumstances  or  when  profits  are 
partlv  divisible  under  some  special  contract  or  arrangement?  (A.  I.  of  A., 
Nov.,' 1918.) 

33.  As  at  December  31,  how  would  you  djs'^lnse  upon  a  Balance  Sheet 
the  surplus  appropriated  for  additions  and  betterments  on  the  books  of  a 
steel  plant?     (Cal.,  May,  1916.) 

34.  A  corporation  owns  nearly  all  of  a  block  of  land.  The  remaining 
portion  is  purchased  subject  to  an  existing  lease.  The  corporation  sets 
aside  out  of  surplus  an  amount  believed  to  be  sufficient  to  extend  its  plant 
over  the  entire  block  at  tlie  expiration  of  the  lease.  What  ledger  title 
should  be  given  to  the  amount  set  aside  and  how  should  the  amount  be 
set  up  in  the  Balance  Sheet?     (A,  I.  of  A.,  Nov.,  1919.) 

35.  An  interurban  railway  company,  wishing  to  provide  against  pos- 
sible accidents,  adopted  the  plan  of  depositing  2  per  cent  of  their  gross 
7'eceipts  each  month  in  a  local  savings  bank  as  a  reserve  for  that  purpose, 
charging  the  funds  so  set  aside  to  an  account  which  they  designated  "Re- 
serve for  Accidents."  The  total  fund  for  the  j'ear  amounted  to  $4,869.26, 
out  of  which  they  paid  $950  for  accidents  occurring  and  settled  during  the 
twelve  months,  debiting  such  payment  to  Accident  account,  and  leaving 
cash  balance  in  the  bank  on  December  31  of  $3,919.26. 

The  bookkeeper  endeavored  to  close  the  books  by  showing  the  $4,869.25 
as  a  charge  against  operating  for  the  year  arising  out  of  accident  liability, 
carrying  over  the  balance  in  bank  ($3,919.26)  to  provide  for  future  acci- 
dents, and  making  a  corresponding  credit  to  the  "Reserve  for  Accidents" 
account.  This  left  the  company  with  cash  assets  of  $3,919.26  not  repre- 
sented on  the  books. 

Wherein  did  the  bookkeeper  err,  and  what  entries  should  have  been 
made  to  show  the  transaction  correctly? 

(111.,  Dec.,  1907.) 

36.  Define:  Reserve  for  Bad  and  Doubtful  Accounts.  (Wash.,  July, 
1917.) 

37.  When  preparing  a  Trading  and  Profit  and  Loss  account  at  the 
end  of  a  fiscal  year,  in  what  manner  would  you  treat  the  Reserves  for  Bad 
Debts?     (N.  Y.,  Jan.,  1907.) 


RESERVES  AND  FUNDS  18!) 

38.  Would  a  Reserve  for  Doubtful  Accounts,  the  account  of  which  is 
based  entirely  upon  the  amount  of  loss  of  the  year  preceding,  be  shown 
on  a  Balance  Sheet  as  a  liability,  or  as  an  allocation  of  the  surplus? 
State  your  reason.     (N.  C,  June,  1910;  N.  C,  Sept.,  1919.) 

39.  In  connection  with  the  audit  of  the  books  of  a  corporation  describe 
briefly  the  procedure  you  would  consider  to  be  essential  in  verifying  the 
Reserve  for  Bad  and  Doubtful  accounts.     (Wash.,  July,  1917.) 

40.  In  what  respects,  if  any,  does  a  Reserve  for  Replacement  differ 
from  a  provision  for  depreciation?     (Ohio,  Oct.,  1919.) 

41.  In  conducting  an  audit:  (a)  What  would  be  your  object  in  ex- 
amining reserve  accounts  that  were  allocations  of  the  surplus  and  are 
not  liabilities?  (b)  If  you  found  liability  reserve  excessive  or  deficient, 
what  would  be  your  procedure?     (N.  C,  Nov.,  1918.) 

42.  State  fully  how  you  would  deal  with  excessive  and  deficient  re- 
serves.    (N.  C,  June,  1916;  N.  C,  Nov.,  1919.*) 

43.  What  is  a  secret  reserve?  (Wa.sh.,  Sept.,  1907*;  Mich.,  June, 
1908;  Fla.,  July,  1909*;  Wash.,  May,  1911*;  Mo.,  Dec,  1914*;  Ohio, 
Nov.,  1916;  Cal.,  Nov.,  1916*;  N.  C,  Aug.,  1917;  Mass.,  Oct.,  1917*;  Md., 
Dec,  1917;  Iowa,  Dec,  1918.) 

44.  (a)  How  are  secret  reserves  usually  created?  (6)  What  is  the 
effect  of  such  practice?  (c)  What,  if  anything,  can  be  said  in  extenuation 
of  this  practice?     (Ohio,  Nov.,  1913*;  Ohio,  Oct.,  1919.) 

45.  (a)  Illustrate  different  kinds  of  secret  reserves.  (6)  Are  secret 
reserves  legal?     (Mich.,  June,  1919.) 

46.  Briefly  discuss  the  question  of  an  understatement  of  the  financial 
condition  and  state  whether  this  is  ever  justifiable,  giving  your  reasons. 
(Ohio,  Oct.,  1919.) 

47.  Discuss  briefly  the  ethics  of  charging  expenditures  for  construc- 
tion and  betterments  to  profit  and  loss.     (Cal.,  Nov.,  1916.) 

48.  Describe  the  object  and  effect  of  a  Secret  Resei*ve.  (Wash.,  May, 
1911.) 

49.  You  are  employed  to  make  an  audit  by  a  stockholder  who  believes 
the  management  of  the  corporation  is  piling  up  large  secret  reserves  with 
the  view  of  buying  up  the  stock  of  the  minority  holders.  You  are  given 
free  access  to  the  books.  Explain  in  detail  what  investigations  you  would 
make  to  detennine  the  truth  or  falsity  of  this  belief.     (Mich.,  Dec,  1913.) 

50.  Give  your  opinion  as  to  the  advantages  or  dangers  arising  from 
secret  reserves.     (Fla.,  July,  1909.) 

51.  Give  some  examples  of  secret  reserves  and  state  your  opinion  aq 
to  the  propriety,  or  otherwise,  of  the  creation  of  such  reserves,  giving 
reasons.     (Wash.,  Sept.,  1907;  Mich.,  June,  1908*;  Md.,  Dec,  1917.*) 

52.  State  what  you  would  consider  an  auditor's  duties  relative  to 
secret  resei-ves.  (Fla.,  July,  1909*;  N.  Y.,  Feb.,  1910*;  Mo.,  Dec,  1914; 
Mich.,  June,  1919.*) 


I 


190  C.  P.  A.  ACCOUNTING 

53.  How  would  a  secret  reserve  affect  the  book  value  of  capital  stock? 
(Mass.,  Oct.,  1917.) 

54.  Where  do  secret  or  hidden  reserves  differ  from  ordinary  reserves? 
How  are  such  reserves  created  and  for  what  purposes?  (Mo.,  Dec,  1913; 
111.,  May,  1914.) 

55.  Define:  Hidden  reserve.  (111.,  May,  1906*;  Pa.,  Nov.,  1906*;  Pa. 
Nov.,  1912*;  Mich.,  Dec,  1914;  Mass.,  Oct.,  1916.*) 

56.  Describe  two  instances  of  hidden  reserves  and  their  treatment  in 
the  accounts.  How  would  you  deal  with  these  in  a  Balance  Sheet? 
(Mass.,  Oct.,  1916.) 

57.  How  far  in  your  opinion  is  an  auditor  justified  in  omitting  to 
mention  hidden  reserves  in  a  certified  statement?  Would  you  treat  them 
differently  in  a  statement  for  credit  purposes,  a  statement  to  stockholders, 
and  a  statement  to  the  State  of  Massachusetts?  If  so  (a)  how,  and  (h) 
why?     (Mass.,  April,  1911.) 

58.  Express  your  opinion  of  the  soundness  of  hidden  reserves  from 
an  auditor's  standpoint.  (111.,  May,  1906;  Pa.,  Nov.,  1906;  Pa.,  Nov., 
1912.*) 

59.  Support  your  views  on  the  use  of  secret  reserves.  (N.  D.,  June, 
1914.) 

60.  Express  your  opinion  of  the  soundness  of  hidden  resei'ves  from 
the  viewpoint  of  a  shareholder,  and  a  director,  respectively.  (111.,  May, 
1906;  Mich.,  June,  1913.*) 

61.  Define:  Reserve  for  Sinking  Fund.     (Wis.,  April,  1917.*) 

62.  State  clearly  under  what  conditions  reserves  for  sinking  funds 
should  be  carried.     (Cal.,  June,  1917.) 

63.  Indicate  the  pro  forma  entries  for  the  creation  of  a  sinking  fund 
reserve.     (Cal.,  June,  1917*;  Wis.,  May,  1919.) 

64.  If  a  reserve  has  been  created  out  of  profits  to  pro\ide  for  a 
sinking  fund  to  pay  off  a  bonded  debt  at  maturity,  what  does  the  Reserve 
account  represent  after  the  redemption  and  cancellation  of  the  bonds? 
(Ohio,  Nov.,  1917.) 

65.  Discuss  the  disposition  of  a  Sinking  Fund  Reserve  account  which 
is  no  longer  necessary.  (Wis.,  May,  1916;  Cal.,  June,  1917*;  Ohio,  Nov., 
1917*;  Wis.,  May,  1919.) 

66.  The  officers  of  a  company  of  which  you  are  the  auditor  elected  by 
the  shareholders,  submit  to  you  for  audit  a  Balance  Sheet  in  which  the 
following  item  appears:  "Miscellaneous  Reserves  (including  premium  on 
stock),  $248,000."  On  investigation  you  find  the  item  is  made  up  as 
follows:  General  Reserve,  $86,000;  Operating  Reserves,  $6,000;  Provision 
for  Plant  Depreciation,  $46,000;  Provision  for  Amortization  of  Lease- 
holds, $40,000;  Provision  for  Bad  Debts,  $36,000;  Premium  on  Capital 
Stock  sold,  $34,000;  Total,  $248,000. 

What  recommendation  would  you  make  to  the  oflieers  and  what  course 
would  you  take  if  your  recommendation  were  not  followed  ? 

(A.  I.  of  A.,  June,  1917.) 


RESERVES  AND  FUNDS  191 

67.  The  Balance  Sheet  of  a  corporation  shows  the  following  credit 
balances:  Reserve  for  Depreciation;  Reserve  for  Extension  of  Plant; 
Reserve  for  Bad  and  Doubtful  Debts;  Sinking  Fund  Reserve;  Insurance 
Reserve;  Reserve  for  Pensions;  Reserve  for  Contingencies;  Reserve  for 
Taxes. 

What  would  you  assume  to  be  the  nature  of  each  of  these  items'  Can 
better  terms  be  submitted  for  any  of  those  used?  In  what  circumstances 
would  each  of  the  above  accounts  be  debited,  and  when  debited,  what 
would  be  the  corresponding  credit?  If  the  business  were  to  be  sold  for 
the  amount  of  its  net  worth,  as  shown  by  the  Balance  Sheet,  which  of 
these  items  would  represent  a  proper  addition  to  the  capital  stock  in 
determining  the  selling  price? 

(A.  I.  of  A.,  June,  1917.) 

68.  With  the  present  trend  toward  prohibition,  what  accounting  propo- 
sition would  you  make  in  the  books  of  a  corporation  owning  a  brewery 
in  order  to  maintain  its  financial  integrity  before  authorizing  the  declara- 
tion of  a  dividend?     (Mich.,  Dec,  1916.) 

69.  Assuming  that  a  Balance  Sheet  consists  of  three  principal  divisions 
of  accounts,  viz.,  assets,  liabilities,  and  net  worth,  state  in  what  division 
you  would  place  the  following  accounts:  (a)  Reserve  for  Depreciation; 
(b)  Reserve  for  Improvements  and  Additions;  (c)  Reserve  for  Shrinkage 
of  Inventory  Values;  (d)  Reserve  for  Losses  on  Hedging  Contracts;  (e) 
ReseiTe  for  Federal  Taxes  (Income  and  Excess  Profits);  (/)  Reserve  for 
Unpaid  or  Accumulated  Dividends  of  Preferred  Stock;  (g)  Reserve  for 
Doubtful  Accounts;  (h)  Reserve  for  Contingent  Losses;  (t)  Reserve  for 
Non-admitted  Assets;  (_;)  Reserve  for  Extinguishment  of  Bonds.  (N.  C, 
Nov.,  1918.) 

70.  You  are  called  upon  to  examine  the  accounts  of  a  corporation  for 
the  purpose  of  certifying  its  Balance  Sheet.  An  item  is  carried  on  the 
"liability"  side  described  "Sundry  Reserves— $375,000."  You  find  that 
this  amount  is  made  up  as  follows: 

Reserve  for  Contingencies $50,000 

Plant  Depreciation  Reserve 80,000 

Reserve  for  Bad  Debts 10,000 

Reserve  for  Collection  Expenses 15,000 

Premium  on  Sale  of  Capital  Stock 12,00J 

Reserve  for  Personal  Injury  Suit  which  has  just  been 

decided  against  the  Company 8,000 

Reserve  for  Patent  Litigation  pending 20,000 

Reserve  for  Income  and  War  Excess  TProfits  Taxes 40,000 

Special  Reserve  against  a  possible  drop  in  market  values 

of  merchandise  on  hand 30,000 

Sinking  Fund  for  Retirement  of  bonds 48,000 

Provision  against  dismantlement  of  Aurora  Works 34,000 

Pension  Fund 28,000 

$375,000 


The  president  is  unwilling  to  change  the  company's  Balance  Sheet  with- 
out consulting  the  board  of  directors  and  states  that  if  you  take  exception 


192  C.  P.  A.  ACCOUNTING 

to  this  item  you  should  write  him  your  views  so  that  he  may  bring  the 
matter  formally  before  them.  Write  such  a  letter  stating  clearly  the 
reasons  for  any  exceptions  you  may  take.  (Ill-,   Dec.,   1918.) 

71.  A  company  incorporated  M'ith  a  capital  of  $200,000,  fully  paid  up, 
has  sold  its  stock  at  a  premium  of  25  per  cent,  thus  realizing  in  cash 
$250,000.  The  by-laws,  which  cannot  be  amended  except  in  a  stock- 
holders' meeting  and  after  proper  notice  of  such  amendment,  having 
been  mailed  to  each  stockholder  ten  daj's  prior  to  the  meeting,  contain 
a  provision  that  the  $50,000  so  received  over  and  above  the  capital  stock  at 
par,  shall  be  placed  to  the  credit  of  a  Special  Reserve  account,  and  that 
this  fund  shall  not  be  applicable  towards  the  payment  of  dividends.  At 
the  close  of  the  first  fiscal  year,  it  is  found  that  the  company  has  made  a 
net  profit  of  $4,000,  after  charging  $6,000  for  depreciation  on  the  buildings 
and  machiner}\  The  directors  desire  to  pay  a  cash  dividend  of  5  per  cent 
and  pa.ss  a  resolution  ordering  that  the  depreciation  refened  to  above 
shall  be  charged  against  the  above  Special  Reserve  account  instead  of 
against  Profit  and  Loss,  and  they  then  pi'oceed  to  declare  a  dividend  of 
5  per  cent.  Discuss  the  above  situation  from  the  standpoint  of  an  ac- 
countant.    (111.,  Nov.,  1904.) 

72.  A  public  utility  company  issues  $500,000  first  mortgage  bonds  due 
in  equal  installments  from  the  tenth  to  the  twentieth  year.  The  mortgage 
required  that  a  reserve  for  sinking  fund  nuist  be  provided  at  the  rate  of 
$15,000  per  year. 

What  is  the  effect  of  such  a  provision?  What  entries  should  be  made 
each  year  and  what  entries  when  the  bonds  are  paid? 

(111.,  May,  1915.) 

73.  What  reason  can  you  give  for  the  creation  of  a  reserve  for  a 
sinking  fund  when  the  reserve  is  not  to  be  funded?  (N.  Y.,  June,  1912; 
N.  D.,  July,  1916.) 

74.  A  sinking  fund  reserve  is  created  out  of  annual  earnings.  How  is 
the  book  value  of  the  company's  stock  affected  by  such  policy?  Explain. 
(Mass.,  Oct.,  1917.) 

75.  How  would  you  classify  the  sinking  fund  reserve  in  the  Balance 
or  Profit  and  Loss  account?     (Kan.,  Dec,  1915;  Mo.,  Dec,  1915.) 

76.  Classify  Reserve  for  Sinking  Fvmd  according  to  the  subdivision  of 
assets,  liabilities,  proprietary  interest,  income  and  expenses  under  which 
it  should  be  grouped.     (Wis.,  May,  1919.) 

77.  What  is  involved  in  the  verification  of  reserves  for  sinking  funds? 
(Cal.,  May,  1916.) 

78.  How  should  a  Reserve  account  and  a  sinking  fund,  both  relating  to 
the  redemption  of  the  same  debt,  be  simultaneously  operated?  What  pur- 
pose is  accomplished  thereby  and  how  do  said  accounts  respectively  appear 
on  the  Balance  Sheet?  (N.  Y.,  Jan.,  1906;  Cal.,  June,  1917;  N.  Y., 
Jan.,  1920.*) 

79.  Are  sinking  fund  reserve  appropriations  a  satisfactory  protection 
to  the  bondholder?     (Wis.,  May,  1916.) 


RESERVES  AND  FUNDS  193 

Funds 

80.  Distinguish  between  a  fund  and  a  reserve.     (Mo.,  Dec,  1913.) 

81.  Define:  Reserve  fund.  (N.  Y.,  Dec.,  1897*;  N.  Y.,  June,  1900; 
Pa.,  May,  1903*;  111.,  May,  1904;  Wash.,  April,  1906;  Wash.,  June,  1912; 
La.,  May,  1913*;  Wis.,  May,  1916;  W.  Va.,  May,  1917*;  N.  D.,  Aug., 
1917;  Mass.,  Oct.,  1917.*) 

82.  State  the  object  of  the  Reserve  Fund  account.  (N.  Y.,  Dec.,  1898; 
Mich.,  July,  1906.*) 

83.  How  are  reserve  funds  created  on  the  books'?     (Mich.,  July,  1906.) 

84.  If  asked  to  criticize  a  Balance  Sheet  prepared  by  a  client's  book- 
keeper, state  what  you  Avould  say  regarding  the  following  caption  found 
on  the  liability  side  of  the  Balance  Sheet :  "Reserve  Fund  for  Redemption 
of  Bonds— .$50,000."    Explain  fully.     (N.  Y.,  June,  1918.) 

85.  How  would  a  reserve  fund  affect  the  book  value  of  capital  stock? 
(Mass.,  Oct.,  1917.) 

86.  Support  your  views  on  the  use  of  reserves  that  are  funded  and 
reserves  that  are  not  funded.     (N.  D.,  June,  1914.) 

87.  Distinguish  between  a  reserve  account  and  a  reserve  fund.  (N.  Y., 
June,  1902;  N.  Y.,  Feb.,  1908;  111.,  Dec.,  1908;  Mo.,  Dec.,  1914;  111.,  Dec, 
1916;  Mich.,  Dec,  1916.*) 

88.  On  which  side  of  the  Balance  Sheet  should  a  reserve  fund  be 
found?     (N.  Y.,  Jan.,  1917.) 

89.  State  whether  or  not  you  consider  the  keeping  of  reserve  funds 
advisable,  and  j'our  reasons.     (Wash.,  May,  1911.) 

90.  Under  what  circumstances,  if  any,  may  a  reserve  fund  or  a  reserve 
appear  on  the  books  of  a  company  without  the  other?     (111.,  Dec,  1916.) 

91.  Should  a  reserve  fund  be  invested  in  interest-bearing  securities? 
State  the  custom.  If  so  invested,  what  account  should  be  credited  with  the 
income?     (N.  Y.,  June,  1897.) 

92.  State  your  views  fully  of  (a)  how  Reserve  Investment  Funds  shoul^J 
be  invested.  (6)  Why  is  it  not  advisable  to  keep  reserve  funds  in  the 
business?     (Md.,  Oct.,  1903.) 

93.  Define:  Sinking  fund.  (N.  Y.,  Jan.,  1900*;  N.  Y.,  June,  1900; 
Pa.,  Mav,  1903*;  111.,  May,  1904*;  Pa.,  Nov.,  1904*;  N.  Y.,  Jan.,  1906*; 
Md.,  Jan.,  1909*;  Mass.,  June,  1910*;  Va.,  Nov.,  1910*;  111.,  May,  1912*; 
Mich.,  June,  1912*;  Wis.,  April,  1914;  Cal.,  Nov.,  1916;  Wis.,  April, 
1917;  W.  Va.,  May,  1917*;  N.  Y.,  June,  1918*;  Va.,  Nov.,  1918;  Iowa, 
Dec,  1918;  Wis.,  May,  1919.) 

94.  What  is  the  purpose  of  a  sinking  fund?  (Md.,  Oct.,  1903* ;  Mich., 
July,  1906;  111.,  May,  1909*;  Mich.,  June,  1912;  Ohio,  Nov.,  1913*;  Cal., 
Nov.,  1916.) 

95.  State  the  plan  most  frequently  approved  by  writers  on  corporation 
finance  as  being  the  most  satisfactory  for  accomplishing  the  purpose  for 
which  a  "sinking  fund"  is  created.     (Wis.,  May,  1919.) 


J!J4  C.   V.   A.  ACC()L'XTIX(; 

96.  What  is  the  eflPect  on  a  business  of  a  sinking  fund?  (Pa.,  May, 
1906.) 

07.  State  how  a  sinking'-  fund  should  be  treated  in  the  books  of  a 
corporation.  (N.  Y.,  Jan.,  1900*;  N.  Y.,  Jan.,  1901';  N.  Y.,  Jan.,  1906; 
III.,  May,  1906*;  N.  Y.,  June,  1909;  Va.,  Nov.,  1910;  Mich.,  June,  1919.*) 

98.  Indicate  the  pro  forma  entries  for  the  creation  of  a  sinking  fund. 
(N.  Y.,  June,  1902*;  Mich.,  July,  1906*;  Wash.,  March,  1909*;  111.,  May, 
1914*;  Mich.,  June,  1915*;  Mass.,  Oct.,  1915*;  Wis.,  May,  1919.*) 

99.  Give  the  entries  for  sinking  fund  required  when  the  bonds  are  paid 
off  at  maturity.     (111.,  May,  1912* ;  Mass.,  Oct.,  1915;  N.  Y.,  Jan.,  1920.*) 

100.  (a)  Stout  and  Company,  acting  as  trustees  in  an  issue  of  $250,000 
thirty  year  bonds  of  the  Modern  Gas  and  Electric  Conii)any,  engage  you 
as  an  accountant  to  advise  them  as  to  the  merits  of  the  various  methods  of 
recording  funded  debt  transactions  upon  the  books  of  the  utility. 

In  your  report  construct  hypothetical  balance  sheets  to  show  four 
methods  of  handling  the  funded  debt  and  explain  fully  the  advantages 
and  disadvantages  of  each,  bearing  in  mind  that  you  are  working  for  the 
trustee  whose  chief  desire  is  security  for  the  bondholder. 

(b)  A  director  of  the  utility  argues  that  if  appropriations  are  made  out 
of  profits  for  sinking  fund  purposes,  then  it  is  unnecessary  to  make  pro- 
visions for  depreciation  on  the  property  of  the  utility.  Include  in  your 
report  to  your  client  recommendations  upon  this  point. 

(Wis.,  April,  1918.) 

101.  Argument  has  been  strongly  urged  that  aside  from  any  question 
of  possible  mismanagement,  or  of  the  difficulty  of  making  satisfactory 
investments  to  yield  the  same  rate  as  is  paid  on  the  bonds,  a  sinking  fund 
for  bonds  is  more  expensive  than  an  arrangement  for  the  serial  payment 
of  bonds.  This  is  illustrated  by  the  case  of  $20,000  5  per  cent  bonds.  If 
these  are  paid  off  in  a  series,  one  each  year,  the  total  payment  made  will 
be  principal  $20,000,  interest  .$10,500,  total  $30,500.  The  annual  sinking 
fund  to  pay  these  bonds  would  on  a  5  per  cent  basis  amount  to  $604.85, 
making  in  twenty  years  $12,097,  and  the  interest  paid  on  the  bonds  would 
be  $20,000,  total  payments  $32,097.  The  apparent  excess  burden  is  ac- 
cordingly $1,597. 

Discuss  the  above  argimient  and  show  clearly  just  what  the  figures  mean 
and  in  what  the  apparent  saving  actually  consists. 

(A.  I.  of  A.,  June,  1917.) 

102.  State  the  theory  and  purpose  of  e.ich  of  the  following,  and  show 
wherein  they  differ:  (a)  reserve  fund;  (b)  sinking  fund.  (N.  Y.,  June, 
1899;  Dec.,  June,  1915*;  N.  D.,  July,  1916.*) 

103.  Name  various  ways  in  which  sinking  funds  may  appear  on,  or 
affect  the  items  of  a  Balance  Sheet.  (Wis.,  April,  1914;  Wis.,  May, 
1916.*) 

104.  Should  a  sinking  fund  represent  actual  investments  or  may  it  be 
offset  by  contra  entries?     (N.  Y.,  Jan.,  1918.) 

105.  Classify  the  account  Sinking  Fund  for  Bond  Redemption,  ac- 


RESERVES  AND  FUNDS 


195 


cording'  to  the  subdivision  of  assets,  liabilities,  proprietary  interest,  in- 
come and  expenses  under  which  it  should  be  grouped.     (Wis.,  May,  1919.) 
106.     You   are  auditinp:   the  accounts  of  a   corporation   and   you   find 
the  following  entries  in  one  month  without  sufficient  explanation: 


Profit  and  Loss 

To  Accrued  Sinking  Fund . 


Accrued  Sinking  Fund 

To  Reserve  for  Sinking  Fund  . 

Union  Trust  Company,  Trustee. 
To  Accounts  Payable 


Accounts  Payable , 
To  Cash 


Sinking  Fund  No.  1 

To  Union  Trust  Company 


Reserve  for  Sinking  Fund  , 
To  Profit  and  Loss .... 


$8,333.33 
100,000.00 
100,000.00 

100,000.00 
100,000.00 
100,000.00 


$8,333.33 
100,000.00 
100,000.00 

100,000.00 
100,000.00 
100,000.00 


What  would  you  conceive  the  situation  to  be  and  what  recommendation 
would  you  make? 

(Cal.,  May,  1916;  Mich.,  Dec,  1916.*) 

107.  On  which  side  of  the  Ledger  should  balance  of  the  Sinking  Fund 
aecoimt  appear?     (Mo,,  Dec,  1914.) 

108.  Name  the  various  ways  of  figuring  a  sinking  fund.  (111.,  May, 
1912*;  Wis.,  April,  1914;  Wis.,  May,  1916.) 

109.  What  are  the  usual  sinking  fund  provisions  to  be  found  in  a 
trust  deed  securing  an  issue  of  bonds  of  a  corporation?  (111.,  May,  1912*; 
111.,  May,  1914;  Wis.,  May,  1919.*) 

110.  Compare  the  sinking  fund  provision  of  typical  preferred  stock 
issues  with  the  sinking  fund  provisions  of  typical  bonds  issues  as  to 
entries  to  be  made:  (1)  When  the  sinking  fund  is  set  up;  (2)  when  the 
purpose  of  creating  the  sinking  fund  has  been  attained;  (3)  when  any 
balances  remain  in  the  sinking  fund  or  allied  accounts.  (Wis.,  Nov., 
1919.) 

111.  A  city  wishes  to  borrow  $90,000  for  30  years,  this  debt  to  be 
extinguished  either  by  a  sinking  fund  or  by  an  issue  of  serial  loan  bonds, 
payable  so  much  per  annum.  Which,  in  your  opinion,  would  be  the 
better  way,  and  why?  Answer  fully  and  differentiate  clearly  between 
these  two  methods  of  extinguishing  the  debt.     (Mass.,  April,  1911.) 

112.  A  mortgage  provides  for  a  sinking  fund  to  be  accumulated  in  the 
hands  of  a  trustee  from  profits  prior  to  dividend  payments.  Prepare 
skeleton  Balance  Sheet  to  disclose  the  state  of  the  fund,  dividends  declared 
and  payable,  appropriation  of  profits  for  purpose  of  the  fund  and  an 
unappropriated  surplus.    What  effect  would  losses  in  excess  of  such  un- 


196  C.  P.  A.  ACCOUNTING 

appropriated  surplus  have  on  the  sinking  fund?     (N.  Y.,  Jan.,  1911.) 

113.  From  a  theoretical  point  of  view,  are  the  contributions  to  a  sink- 
ing fund  a  proper  charge  against  profits?  Give  your  reasons.  (Ohio, 
Nov.,  1913.) 

114.  From  what  source  is  a  sinking  fund  derived  and  what  account 
is  charged?     (Pa.,  Nov.,  1904;  Md.,  Jan.,  1909.) 

115.  May  a  sinking  fund  be  established  by  appropriating  profits? 
(Cal.,  Nov.,  1916.) 

116.  Where  the  establishment  of  a  sinking  fund  is  compulsory,  is  it 
theoretically  correct  to  charge  income  with  the  annual  appropriations 
required  by  the  mortgage?     Give  reasons.     (Ohio,  Nov.,  1917.) 

117.  If  not  set  aside,  in  compliance  with  requirements  of  a  bond  issue, 
how  may  a  sinking  fund  afifect  a  corporation?  (Pa.,  Nov.,  1904;  Md., 
Jan.,  1909.) 

118.  In  the  course  of  a  Balance  Sheet  audit  state  how  you  would 
proceed  to  verify  the  correctness  of  a  sinking  fund.     (Ohio,  Oct.,  1919.) 

119.  In  preparing  a  Balance  Sheet  of  a  corporation,  how  would  you 
classify  or  deal  with  securities  representing  the  investment  of  a  sinking 
fund?     (111.,  May,  1906*;  N.  Y.,  Jan.,  1907*;  A.  I.  of  A.,  June,  1917.) 

120.  How  may  sinking  funds  be  invested?     (Wis.,  May,  1916.) 

121.  A  concern  has  established  a  sinking  fund  for  the  retirement  of  a 
mortgage.  An  investment  has  been  made  in  bonds,  the  present  market 
value  of  which  is  below  cost.  Would  you  inventory  them  at  market  value 
or  at  book  value?     Why?     (Mich.,  Dec,  1914.) 

122.  Define:  Sinking  Fund  Investment  account.  (Wash.,  Nov.,  1913*; 
Wash.,  July,  1917.) 

123.  Explain  in  some  detail  the  procedure  you  would  follow  as  an 
auditor  in  verifying  Sinking  Fund  Investment  account  appearing  in  the 
books  and  accounts  of  a  company.     (Wash.,  June,  1915.) 

124.  How  should  interest  on  sinking  fund  investments  be  treated  in 
the  accounts?     (Cal.,  Nov.,  1916.) 

125.  Define  briefly  the  following  terms:  (a)  Redemption  fund;  (6) 
Depreciation  fund;  (c)  Contingent  fund;  {d)  Investment  fund.  (111., 
May,  1904.) 

126.  Describe  the  nature  of  the  following  accounts :  (a)  Sinking  fund; 
(b)  Reserve  Fund;  (c)  Redemption  Fund;  (d)  Depreciation  Fund;  (e) 
Contingent  Fund;  (/)  Investment  Fund.  (N.  Y.,  Dec.,  1896;  W.  Va., 
May,  1917;  S.  C,  Sept.,  1919.) 

127.  State  the  object  of  the  following:  Redemption  Fund  account. 
(N.  Y.,  Dec,  1898.) 

128.  Define  the  difference,  if  any,  between  a  sinking  fund  and  a  depre- 
ciation fund.  Explain  fully.  (N.  Y.,  June,  1906*;  Mass.,  Oct.,  1915*; 
Cal.,  Nov.,  1916;  N.  Y.,  June,  1918*;  Md.,  Oct.,  1919.) 


RESERVES  AND  FUNDS  197 

129.  A  municipality  has  built  a  public  building  from  the  proceeds  of 
a  bond  issue.  Should  the  municipality  write  off  depreciation  of  the 
building  and  at  the  same  time  also  create  a  sinking  fund?  (Wis.,  May, 
1916.) 

130.  Explain  in  some  detail  the  procedure  you  would  follow  as  an 
auditor  in  verifying  the  invested  funds  appearing  in  the  books  and 
accounts  of  a  company.     (Wash.,  June,  1915.) 

131.  Define  and  differentiate  sinking  fund  and  Reserve  account.  Where 
does  each  appear  on  the  Balance  Sheet,  and  what  does  each  represent? 
(Va.,  Oct.,  1912.) 

132.  Under  the  conditions  of  a  general  mortgage  given  by  the  Red  Clay 
Brick  Company  to  protect  its  issue  of  bonds,  provision  is  made  for  pay- 
ments to  the  trustee  at  stated  periods,  which,  together  with  all  accretions 
from  interest  and  profits,  are  to  be  held  and  disbursed  by  the  trustee  as 
one  fund.  Should  any  distinction  be  shown  on  the  books  of  the  brick 
company  between  interest  and  profit  so  obtained?  If  so,  give  reasons. 
(N.  Y./June,  1918.) 


198  C.  P.  A.  ACCOUNTING 


PROBLEMS 


RESERVES  AXD   FUNDS 


1.  On  the  1st  of  July,  1905,  a  company  borrowed  $100,000  at  4  per  cent 
per  annum,  payable  half-yearly,  and  payment  of  loan  to  be  made  at  end 
of  10  years  at  105  per  cent.  It  was  decided  to  set  aside  annually  out  of 
the  profits  such  a  sum  as  would  with  interest  at  4  per  cent  per  annum 
pronde  for  the  payment  of  the  premium  on  the  loan  at  the  end  of  the 
term.     (What  was  this  sum?)     (Fla.,  April,  1907.) 

2.  A  corporation  wants  to  retii'e  a  debt  of  $103,000.  bearino;  5  per  cent 
interest,  payable  annually.  The  tenth  payment,  including  interest,  is  to 
be  $15,000.  The  other  nine  periodical  payments  are  all  to  include  interest 
and  to  be  of  the  same  amount.  Required :  the  amount  of  each  of  such 
nine  payments.     (1.05»=1.551.328.)      (A.  I.  of  A.,  Nov.,  1918.) 

3.  A  company  is  under  obligations  to  pay  $10,000  to  sinking  fund 
trustees  "out  of  protits,"    The  following  transactions  take  place: 

1914 

Pe*^.  31,  $10,000  cash  paid  to  sinking  fund  trustees. 

1915 

Jan.  5,  Trustees  invest  in  $10,000  of  the  5  per  cent  bonds  of  the  company  at 

98  and  interest  (from  Jan.  1). 
July  1,  Coupons  on  above  bonds  collected. 
Dec.  .31,  $10,000  paid  to  sinking  fund  trustees. 
1916 

Jan.  1,  Coupons   collected. 

Jan.  2,  $11,000  bonds  bought  for  sinking  fund  at  95. 
July  1,  Coupons  collected. 

Dec.  31,  $125  paid  for  expenses  of  sinking  fund. 
Dec.  31,  $10,000  paid  to  sinking  fund  trustees. 
1917 

Jan.  1,  Coupons  collected. 
Jan.  10,  $10,000   bonds  bought  at  101   and   interest. 

Give  the  Journal  entries  on  the  company's  books  for  the  above  transac- 
tions. (A.  I.  of  A.,  June,  1917.) 

4.  "C,"  "D"  and  "E"  are  partners  with  ecjual  capital  and  share  e<^pially 
in  the  profits.  After  trading  for  three  years,  "E"  wishes  to  retire  and 
"D"  elects  to  remain  and  purchase  the  share  of  the  former.  The  part- 
nership agreement  provides  that  the  retiring  partner  shall  receive  a  share 
of  the  goodwill.  The  value  of  the  latter  to  be  equal  to  two-thirds  of  the 
average  of  the  ]irofits  of  the  last  three  years  preceding  his  retirement. 

The  following  are  the  figures  and  we  are  requested  to  prepare  a  Balance 
Sheet  and  a  Profit  and  Loss  account  as  of  June  30,  1916,  and  an  account 
showing  the  amount  due  to  "E"  from  the  remaining  partners: 


RESERVES  AND  FUNDS  199 

Capital  Account,  "C" $8,000 

Capital  Account.  "D" g  000 

Capital  Accour.t,  "E" 8*000 

Plant  and  Equipment 14*840 

Trade-marks 4',500 

Inventory,  June  30   191G 7*600 

Inventory,  July  1,  1915 4*800 

Accounts  lloceivable 19,400 

Merchandise  Creditors 13  402 

Sales ;      5.5^188 

Purchases  .  . 27,804 

Wages  and  Salaries 4,600 

General  Expenses l|560 

Partners'  Drawing  Accounts: 

"C "  Debit  Balance 5,500 

"  D  "  Debit  Balance 5,500 

"E"  Debit  Balance 5*500 

Cash  in  bank  and  on  hand 3,974 

The  following'  adjustments  are  to  be  made  for  the  year  just  closed: 
Ten  per  cent  depreciation   on  tlie  i)lant  and  equii)ment;   15  per  cent 
on  the  trade-marks;  10  per  cent  reserve  for  bad  and  doubtful  debts. 

There  is  on  the  books  a  special  Reserve  account  to  cover  depreciation 
of  the  stock  on  hand,  which  is  of  a  very  perisliable  nature.  The  reserve 
amounts  to  $5,388  and  must  be  equitably  dealt  with  in  the  dissolution  of 
the  partnership. 

The  previous  two  years'  profits  were  $17,816  and  $22,020,  respectively. 

(Minn.,  Oct.,  1916;  Cal.,  Nov.,  1916.) 
5.  The  Jones  Company,  Inc.,  acquired  the  business  from  S,  R.  Jones, 
who  took  bonds  amounting  to  $50,000  in  part  payment.  These  mature 
in  20  years  and  can  be  canceled  by  payment  after  15  years,  and  bear 
interest  at  the  rate  of  6  per  cent  per  annum.  A  sinking  fund  is  to  be 
established  for  their  redemption  by  payment  to  the  Bankers  Trust  Co., 
Trustees,  of  $2,500  a  year,  the  interest  on  this  fund  to  accumulate  to  help 
in  retiring  the  bonds  before  20  years. 

November  30,  1918,  was  the  end  of  the  first  fiscal  year  and  the  trial 
balance  of  that  date  is  as  follows: 


Real  Estate 

Buildings 

Machinery 

Accounts  Receivable 

Cash 

Merchandise 

Labor 

Office  Expense 

Miscellaneous  Investments . 

Bond  Sinking  Fund 

Interest  Paid  on  Bonds  ... 

Capital  Stock 

Bonds  Payable 

Net  Sales 

Notes  Payable 

Accounts  Payable 

Miscellaneous  Income 


$30,000.00 
30  000.00 
43;  150.00 

4,260.00 
12.759.60 
46^540.00 
20,000.00 

1,950.00 
440.40 

2,500.00 

3,000.00 


$194,600.00 


$.50,000.00 

.50,000.00 

68,090.00 

5.000.00 

21,450.00 

60.00 


$194,600.00 


200  C.   P.   A.  ACCOUNTING 

Inventory  of  merchandise  is  $28,500. 

Under  date  of  May  30,  1918,  the  company  paid  the  Bankers'  Trust 
Co.  $2,500.  Under  date  of  November  30.  1018  the  Bankers'  Trust  Co. 
reported  that  they  had  received  the  $2,500;  that  on  June  15,  1918, 
they  purchased  two  $1,000  bonds  at  par  and  accrued  interest.  Rate  of 
interest  5  per  cent,  payable  in  November  and  May  each  year;  that  they 
collected  $50  interest  on  November  1,  1918;  and  that  they  have  allowed 

4  per  cent  interest  per  annum,  computed  semiannually,  on  the  lowest 
amount  of  cash  in  the  fund  during-  the  period. 

They  show  the  cash  in  the  fund  which  you  are  asked  to  compute  from 
the  above  to  verify  the  correctness  of  their  balance. 

(1)  Make  the  necessary  Journal  entries  to  close  the  books  for  the 
fiscal  year.  (2)  Construct  Income  and  Expense  account  for  the  year.  (3) 
Construct  Balance  Sheet  as  of  November  30,  1918.  (4)  Prepare  statement 
of  sinking  funds  in  hands  of  Bankers'  Trust  Co. 

(Iowa,  Dec.,  1918.) 

6.  A  corporation  issued  10-year  first  mortgage  bonds  on  April  1,  1914, 
bearing  6  per  cent  interest,  payable  semiannually  (April  and  October). 
The  bonds  provided  for  annual  contributions  to  a  sinking-fund  trustee, 
a  trust  company,  that  allowed  2  per  cent  compound  interest.  The  bonds 
were  sold  at  a  premium  and  payment  was  received  therefor  on  April  1, 
1914.  Show  pro  forma  entries  as  of  the  following  dates:  (a)  April  1, 
1914;  (&)  October  1,  1914;  (c)  April  1,  1915;  {d)  April  1,  1924.  (N.  Y., 
June,  1915.) 

7.  A  contractor  proposes  to  build  a  bridge  to  Belle  Isle  and  accept  the 
city's  4  per  cent  20-year  bonds  to  the  amount  of  $2,000,000  in  payment. 
He  advocates  as  a  means  of  retiring  the  bonds  the  establishment  of  a  toll 
system  on  foot  passengers  and  automobiles  at  the  respective  rates  of  1  and 

5  cents  each.  Assuming  the  ratio  of  foot  passengers  to  automobiles  to  be 
10  to  1,  how  many  of  each  would  be  necessary  to  pay  the  interest  annually 
and  create  a  fund  which,  placed  at  the  same  rate  of  interest,  would  be  suf- 
ficient to  retire  the  bonds  at  maturity.  (Note:  $1  compounded  at  4  per 
cent  for  20  years  equals  $2.19112314.)  (Mich.,  Dec,  1915;  Mich.,  June, 
1919.) 

8.  You  are  called  upon  to  state  what  is  the  annual  sinking  fund  neces- 
sary to  redeem  a  principal  sum  of  $1,000,000  due  30  years  hence,  it  being 
assumed  that  the  annual  sums  set  aside  are  invested  at  compound  interest 
at  5  per  cent.  (Note:  30th  power  of  1.05  is  4.32194238.)  (A.  I.  of  A., 
Nov.,  1917.) 

9.  A  municipality  borrowed  $150,000  for  5  years  at  4  per  cent  interest, 
payable  annually.  To  meet  the  debt  when  it  became  due,  a  sinking  fund 
was  created  by  depositing  at  5  per  cent  compound  interest  an  equal  sum  at 
the  expiration  of  each  of  the  five  years.  What  was  the  annual  amount 
deposited?     (Mich.,  June,  1910.) 

10.  A  company  wants  to  set  aside  a  sinking  fund  on  a  3  per  cent 
basis  so  as  to  retire  $100,000  in  three  equal  annual  payments.     Required 


RESERVES  AND  FUNDS 


201 


the  amount  necessary  to  be  annually  invested  at  3  per  cent  per  annum. 
(Kan.,  Dec,  1915;  Mo.,  Dec,  1915.) 

11.  What  amount,  including  interest,  must  be  set  aside  annually  to 
provide  a  sinking  fund  sufficient  to  retire  an  issue  of  bonds  amounting 
to  $100,000,  originally  sold  at  90,  interest  payable  semiannually  at  6  per 
cent,  and  maturing  in  ten  equal  annual  installments  after  the  fifth  year? 
Show  method.  (Mich.,  Dec,  1914.)  (Note:  Assume  sinking  fund  install- 
ments set  aside  at  beginning  of  each  year  and  first  bonds  redeemed  at 
beginning  of  sixth  year.) 

12.  A  company  is  issuing  $100,000  of  4%  20-year  bonds,  which  it 
wishes  to  pay  at  matuiity  bj^  means  of  a  sinking  fund,  in  which  equal 
annual  deposits  are  to  be  made.  The  board  of  directors  wishes  to  assume 
that  this  fund  will  earn  5^/^%  interest  for  the  first  five  years,  5%  for  the 
next  five  years,  and  4%  for  the  last  ten  yeai's.  What  is  the  annual  deposit 
required  ? 


Given: 

5y2% 

5% 

4% 

Ss 

5.581 

12.875 

1.307 

1.708 

5.526 

12.578 

1.276 

1 .  629 

5.416 

12.006 

(1-1- i)» 

1.217 

(l+i)^'' 

1.480 

(A.  I.  of  A.,  May,  1920.) 


CHAPTER  XI 

BONDS 

Classes  of  Bonds — Bonds  are  sealed  documents  promising  to  pay  a  cer- 
tain sum  of  money  with  interest  thereon  at  a  specified  rate.*  Bonds  have 
been  classified  by  Lawrence  Chamberlain  in  ''The  Principles  of  Bond  In- 
vestment" under  the  following  headings : 

A.  Character  of  Issuing  Corporation 

(a)  Governmental 

(b)  Commercial 

B.  Character  of  Security 

(a)  Unsecured 

(b)  Secured 

I.  Cruaranty  Security 
II.  Lien  Security 

(1)  On  Personality 

(a)  Paper  Collateral 

(b)  Rolling  Stock 

(c)  Funds 

(2)  On  Realty 

(a)  General  Property 

(b)  Specific  Property 

(c)  Mortgage  Priority 

C.  Purpose  of  Issue 

D.  Payment  of  Interest 

(a)  Conditional 

(b)  Unconditional 

(c)  Registei-ed 

(d)  Coupon 

E.  Payment  of  Principal 

F.  Maturity  of  Principal 

(a)  Fixed  Date 

(b)  Unsettled  Date 

I.  Option  of  Payer 

II.  Option  of  Payee 

(c)  Perpetual 

Unsecured  bonds  or  debenture  bonds  are  merely  formal  acknowledg- 
ments of  debts  under  seal."     Income  bonds  may  or  may  not  secure  the 

'For  explanation  of  superior  figures  sec  page  337. 

202 


BONDS  203 

principal  by  a  pledge  of  specific  property  or  by  a  preference  claim  against 
corporate  property,  but  payment  of  interest  always  depends  entirely  on 
the  existence  of  net  profits.' 

Guaranty  security  bonds  are  those  which,  in  case  of  default,  will  be 
paid  both  as  to  principal  and  interest  by  the  sponsor/  The  chief  kinds 
of  guaranty  security  bonds  are  assumed  bonds,  guaranteed  bonds,  en- 
dorsed bonds,  stamped  bonds,  and  joint  bonds. 

Collateral  trust  bonds  are  good  examples  of  bonds  secured  by  a  lien  on 
personality  consisting  of  securities."  Car  trust  and  equipment  bonds  are 
likewise  secured  by  lien  on  personality,  but  the  property  is  in  the  form  of 
railroad  rolling  stock.'  The  personalty  securing  sinking  fund  bonds  is  in 
the  form  of  funds. 

Bonds  secured  by  liens  against  real  estate  nuiy  be  safeguarded  by 
mortgages  on  the  general  corporate  property,  or  on  realty  of  a  certain 
class,  or  on  sections  of  the  corporate  property.'  Land  grant  bonds  are 
railroad  bonds  secured  by  liens  on  lands  granted  l)y  the  government.* 
Railroads  sometimes  issue  divisional  bonds  secured  l)y  mortgages  on  specific 
railroad  divisions.* 

The  matter  of  mortgage  piiority  is  vital  as  the  pledged  property  may 
be  sufficient  to  liquidate  only  the  senior  issues,  thus  leaving  the  junior 
issues  unprotected  except  for  the  general  corporate  property.'" 

Bonds  may  be  designated  according  to  the  purpose  for  which  the  money 
was  raised.  For  instance,  funding  bonds  represent  the  consolidation  of 
the  corporate  unfunded  debt.  Purchase  money  bonds  are  issued  in  full  or 
part  payment  for  a  property."  Refunding  bonds  are  issued  in  lieu  of 
maturing  obligations  and  have  the  same  security  as  the  redeemed  bonds." 
Consolidated  bonds  replace  several  prior  issues  and  unite  the  securities  for 
the  retired  issues.^' 

Ordinary  bonds  contain  unconditional  promises  to  pay  a  specified 
amount  of  interest.  However,  there  are  some  exceptions,  like  income 
bonds  which  pay  interest  only  if  a  net  profit  is  made  by  the  corporation," 
or  participating  bonds,  the  interest  of  which  may  run  from  a  specified 
minimum  to  a  specified  maximum  (in  limited  participation),  or  to  what- 
ever is  earned  (in  unlimited  participation)."  Bonds  may  be  registered  as 
to  interest,  in  which  case  the  interest  is  remitted  by  check,  or  they  may 
have  interest  coupons  attached,  which  are  cut  and  cashed  when  due." 
Bonds  may  be  registered  as  to  principal,  regardless  of  whether  they  are 
registered  as  to  interest.^' 

Bonds  are  sometimes  designated  according  to  the  exchange  medium  in 
which  thev  are  redeemable,  viz.:  Gold  bonds,  silver  boiuls,  and  currency 
bonds." 

The  bonds  which  mature  at  a  settled  date  are  either  straight  bonds,  those 
nominally  maturing  all  at  one  time,  or  serial  bonds,  those  maturing  in 
regular  installments."  Redeemable  bonds  and  callable  bonds  are  illustra- 
tions of  bonds  the  maturity  of  which  is  affected  by  the  payer's  right  to 
retire  them  before  the  obligatory  maturity  date.*"  Convertible  bonds  are, 
at  the  option  of  the  payee,  eonvei-til)le  generally  into  other  securities  of  the 


204  C.  P.  A.  ACCOUNTING 

same  corporation."     Perpetual  bonds  are  those  containing  no  promise  to 
liquidate  the  principal." 

Unissued  Bonds — Some  accountants  list  unissued  bonds  among  the  assets 
on  the  balance  sheet,"  while  other  accountants  deduct  the  unissued  bonds 
from  the  authorized  bonds."  The  latter  procedure  seems  preferable,  as 
it  shows  the  right  to  issue  the  bonds  and  yet  does  not  tend  to  confuse  the 
lay  reader.  Unissued  bonds  are  sometimes  called  reserved  bonds,"  but  this 
practice  is  not  good  because  the  title  is  not  self-explanatory. 

Treasury  Bonds — As  bonds  may  be  issued  below  par,  it  is  not  very  im- 
portant to  distinguish  between  unissued  and  reacquii'ed  bonds."  However, 
it  is  preferable  to  reserve  the  term  "treasury  bonds"  for  reacquired  bonds." 
The  method  of  presenting  treasury  bonds  (sinking  fund  securities)  on  the 
balance  sheet  is  discussed  in  Chapter  X  under  the  caption  "sinking  funds." 
It  must  be  remembered,  however,  that  the  Interstate  Commerce  Commission 
compels  transportation  companies  to  classify  authorized  bonds,  signed  and 
sealed  by  the  mortgage  trustee,  as  working  assets. 

Bonds  as  Collateral — As  treasury  bonds  out  as  collateral  are  still  the 
property  of  the  borrowing  company,  they  need  be  accounted  for  only  by 
memorandum  entries,**  although  the  fact  that  they  are  pledged  should  be 
indicated  on  the  balance  sheet  by  showing  the  pledged  and  unpledged 
bonds  separately.^ 

Issue  of  Bonds — There  are  two  methods  of  recording  the  issue  of  bonds. 
Assuming  the  authorized  issue  of  $1,000,000,  of  which  $750,000  are  sold  at 
par  for  cash,  the  balance  remaining  unissued,  the  entries  would  be : 


First  Method^" 

Unissued  Bonds $1,000,000.00 

Bonds  Payable $1,000,000.00 

Cash 750,000.00 

Unissued  Bonds 750,000.00 

Second  Method^^ 

Ca.sh $750,000.00 

Bonds  Payable $750,000. 00 


The  second  method,  which  presupposes  that  the  authorized  issue  would 
be  carried  parenthetically  in  the  account  title,  is  preferred  by  accountants 
who  depreciate  the  use  of  memorandum  accounts."  The  first  method  seems 
preferable,  however,  as  the  accounts  themselves  then  give  a  complete  his- 
tory of  the  transaction. 

If  the  above-mentioned  bonds  were  sold  on  the  installment  basis  instead 
of  for  cash,  the  bonds  would  not  be  issued  until  the  subscriptions  were 
paid."     The  entries  would  be  as  follows: 


BONDS  205 

Unissued  Bonds SI  ,000,000.00 

Bonds  Payable $1 ,000,000. 00 

Bond  Subscribers 750,000. 00 

Bond  Subscriptions 750,000.00 

Cash  (1st  Installment) 375,000. 00 

Bond  Subscribers 375,000.00 

Cash  (2nd  Installment) 375,000.00 

Bond  Subscribers 375,000. 00 

Bond  Subscriptions 750,000.00 

Unissued  Bonds 750,000.00 

Bond  premium  and  discount  are  entered  on  the  books  in  the  same  manner 
as  the  premium  and  discount  on  stock.'*  Using  the  above  data,  and 
assuming  that  $150,000  of  the  unissued  bonds  were  sold  at  102  and  the 
remaining  at  99,  the  entries,  illustrating  the  first  method  only,  would  be: 

Cash $153,000.00 

Unissued  Bonds $150,000.00 

Premium  on  Bonds  Payable 3,000.00 

Cash 99,000.00 

Discount  on  Bonds  Payable 1 ,000. 00 

Unissued  Bonds 100,000. 00 

Discount  on  bonds  payable  is  an  asset,  as  it  represents  the  privilege  of 
using  money  at  less  than  the  market  interest  rate,  while  premium  on  bonds 
payable  is  a  liability,  as  it  represents  an  obligation  to  pay  more  than  the 
market  rate  of  interest.'"  Discount  and  premium  on  bonds  payable  should 
be  shown  on  the  balance  sheet  as  deferred  charges^'  and  defended  credits 
to  income,"  respectively.  This  procedure  is  prescribed  for  transportation 
companies  by  the  Interstate  Commerce  Commission.  Discount  on  bonds 
is  sometimes  treated  as  a  valuation  item,^*  but  this  seems  incorrect.  Bond 
discount  should  not  be  deducted  from  bonds  payable  because  the  incurred 
liability  is  the  par  value  of  the  bonds  issued. 

Expenses  incurred  in  floating  a  bond  issue  may  be  shown  in  a  bond 
issue  expense  account,^'  or  an  underwriting  expense  account,*'  or  may  be 
shown  together  Avith  bond  discount  in  a  bond  discount  and  expense 
account."  The  last  method  seems  preferable  as  the  expense  should  be 
prorated  on  the  amortization  principle.  It  is  sometimes  advocated  to 
charge  the  issuing  expenses  to  organization  expense,"  but  this  seems  in- 
advisable. 

The  opening  entries  for  an  issue  of  collateral  trust  bonds  should  be 
preceded  by  an  entry  debiting  pledged  investments  and  crediting  invest- 
ments for  the  amount  of  the  collateral." 

When  equipment  trust  bonds  are  issued,  the  transaction  is  in  the  nature 
of  a  conditional  sale  or  lease,  the  company  paying  installments  annually 
and  the  trustee,  acting  for  the  vendees,  holding  a  lien  on  the  entire  prop- 
erty until  the  last  installment,  or  rental,  is  paid,  when  a  release  is  given." 


206  C.  P.  A.  ACCOUNTING 

The  entries  for  tlie  insue  of  $100,000  of  equipment  trust  bonds  and  the 
payment  of  the  first  installment  would  he:" 

Leased  Equipment $100,000.00 

Equipment  Trust  Bonds $100,000.00 

Equipment  Trust  Bonds 10,000.00 

Cash  (assuming  ten  equal  payments)  10,000.00 

Equipment 10,000.00 

Leased  Equipment 10,000.00 

Bond  Interest — Interest  on  registered  bonds  is  sent  to  the  bondholders 
either  by  the  treasurer  or  fiscal  agent  of  the  corporation/"  As  coupons  are 
frequently  unredeemed  at  the  end  of  the  fiscal  period,  it  is  unsatisfactory 
to  record  the  interest  only  as  it  is  paid.  The  correct  procedure  would  be 
to  debit  bond  interest  and  credit  bond  interest  accrued  at  the  end  of  each 
month  for  one-sixth  of  the  semi-annual  interest  charges,  and  to  debit  bond 
interest  accrued  and  credit  cash  Avhen  coupons  are  redeemed." 

Accountants  usual  h"  record  the  interest  on  treasury  bonds,"  although 
it  is  not  necessary  to  do  so,  unless  tlie  treasury  Jjonds  are  held  by  a 
trustee  under  a  sinking  fund  provision  requiring  that  the  trustee  receive 
the  interest  and  control  its  investment. 

Interest  on  guaranteed  bonds  is  normally  paid  by  the  issuing  company 
in  the  usual  way,  but,  in  cases  of  default,  the  guaranteeing  company  must 
make  the  payment."  The  entry  would  be  a  debit  to  advances  to  subsidiary 
company  and  a  credit  to  ca.sh  on  the  books  of  the  parent  company,  and  a 
debit  to  the  bond  interest  and  a  credit  to  advances  from  parent  company 
on  the  books  of  the  suljsidiary  company."" 

Although  intei-est  on  income  bonds  is  unusual  in  that  it  is  paid  only 
in  the  event  that  the  company's  profits  are  sufiicient  to  pay  it,"  there  are 
no  irregularities  in  the  accounting  procedure  in  recording  the  interest 
payments."' 

Since  bond  premium  represents  the  present  worth  of  an  annuity  the 
rent  of  which  is  the  excess  of  the  stated  bond  interest  over  the  el¥ective 
interest,  the  periodic  payments  extending  over  the  life  of  the  bonds,  the 
premium  is  not  an  earning  but  an  offset  to  excess  bond  interest."  On  the 
other  hand  discount  on  bonds  is  an  addition  to  the  interest  paid  on  bonds."* 
Bond  premium  and  bond  discount  are  therefore  gradually  written  off  as 
a  credit  and  debit,  respectively,  into  the  bond  interest  account."  The 
usual  way  of  prorating  bond  premium  and  discount  is  to  write  off  an 
equal  amount  annually  over  the  life  of  the  bonds.""  The  scientific  or  effec- 
tive rate  method  is  to  amortize  the  difference  between  the  stated  and  effec- 
tive bond  interest  through  the  bond  interest  account." 

If  a  bond  $10,000  par  value,  bearing  five  per  cent,  life  two  years,  interest 
]>ayable  January  and  July,  was  sold  on  a  four  per  cent  basis  for  $10,190.39 
on  January  1,  1917,  the  entry  on  July  1,  1917,  for  bond  interest  and  the 
amortization  of  bond  premium  would  be  as  follows : 


BONDS  207 

Bond  Interest $203.81 

Premium  on  Bonds  Payable 46. 19 

Cash .' $250.00 

If  the  bonds  were  sold  on  a  six  per  cent  l)asis  for  $9,814.15,  the  entry 
to  be  made  would  be : 

Bond  Interest $294 .  42 

Cash $250. 00 

Discount  on  Bonds  Payable 44. 42 

The  mathematics  for  the  above  entries  is  shown  in  the  bond  premium  and 
discount  amortization  tables  given  in  Chapter  IX  under  the  caption  "Amor- 
tization of  Bond  Premium  and  Discount." 

Bond  Register — While  no  record  of  ownership  need  be  kept  for  coupon 
bonds,  a  record  of  the  holders  of  registered  bonds  must  be  kept  either  by 
the  corpoi'ation  or  by  its  transfer  agent.'*  This  bond  ledger  or  register 
consists  of  a  sheet  or  card  for  each  class  of  bonds  ruled  into  columns  for 
number  and  date  of  bond,  name  and  address  of  person  to  whom  bond  is 
issued,  names  and  addresses  of  transferees,  par  value  of  bond,  and  interest 
payments  under  the  different  due  dates.™ 

Coupon  Register — Paid  and  canceled  interest  coupons  may  be  recorded 
in  a  coupon  register  in  which  an  entire  page  is  allotted  to  each  bond  for 
the  pasting  of  the  coupons  in  spaces  numbered  to  correspond  with  the 
particular  coupons.""  However,  the  modern  practice  is  to  deliver  all  paid 
and  canceled  coupons  to  the  trustee  of  the  mortgage,  who  destroys  them 
and  issues  a  certificate  to  that  effect. 

Audit  of  Bonds  Payable — The  auditor  must  read  the  bond  .agreement 
and  ascertain  that  all  provisions  have  been  fulfilled."^  A  complete  schedule 
of  authorized  and  issued  bonds  should  be  made."^ 

All  bonds  certified  by  a  trustee  and  delivered  to  a  corporation  must  be 
accounted  for.'"  The  proceeds  of  all  bonds  sold  should  be  traced  into  the 
proper  accounts."^ 

On  account  of  the  safeguards  placed  by  bond  holders  on  bonds,  little 
difficulty  is  incurred  in  determining  the  outstanding  bonds."  They  may  be 
shown,  (a)  by  the  bond  register  which  should  be  checked  against  the  ag- 
gregate shown  in  the  general  ledger;"'  (b)  from  the  cash  receipts  journal 
if  sold  for  cash,  or  from  general  journals  if  sold  for  property  or  other  as- 
sets;*" (c)  through  the  interest  checks  or  interest  coupons  if  all  have 
been  returned;"*  (d)  by  obtaining  final  proof  through  a  certificate  of  the 
trustee.** 

The  auditor  must  see  that  bond  interest  is  correctly  shown  in  the 
accounts."  He  should  see  that  interest  on  income  bonds  is  taken  care  of 
before  any  dividends  are  paid  out  of  profits." 

Investments  in  Bonds — Bonds  purchased  may  be  classified  into  those  held 
temporarily  and  those  held  permanently.  The  former  are  valued  and 
accounted  for  in  the  same  way  as  temporary  stock  investments  (see 
Chapter  VIII),  while  permanently  held  bonds  are  accounted  for  on  the 


208  C.  P.  A.  ACCOUNTING 

amortization  principle.  As  to  the  application  of  the  amortization  prin- 
ciple to  bonds  purchased  at  a  discount,  it  is  desirable  to  set  up  a  reserve 
for  possible  losses,  if  the  discount  on  the  bonds  was  influenced  by  the  bad 
credit  of  the  issuing  company." 

The  entries  covering  the  purchase  and  the  first  interest  payment  of  a 
bond,  $10,000  par  value,  bearing  five  per  cent,  life  two  years,  interest 
receivable  January  and  July  on  a  four  per  cent  basis,  for  $10,190.39 
would  be: 

Bond  Investment  Account $10,000.00 

Premium  on  Bonds  Purchased 190. 39 

Cash $10,190.39 

Cash.  . 250.00 

Premium  on  Bonds  Purchased 46. 19 

Interest  on  Bonds  Purcha.sed 203.81 

if  the  the  bonds  had  been  purchased  on  a  six  per  cent  basis  for  $9,814.15, 
the  entries  would  be: 

Bond  Investment  Account $10,000.00 

Discount  on  Bonds  Purchased $185 .  85 

Cash 9,814.15 

Cash 250.00 

Discount  on  Bonds  Purchased 44 .  42 

Interest  on  Bonds  Purchased 294.42 

The  calculation  of  the  amounts  that  should  be  written  off  from  the  bond 
premium  and  bond  discoimt  is  illustrated  in  the  amortization  tables  given 
in  Chaptel"  IX  under  the  caption,  "Amortization  of  Bond  Premium  and 
Discount." 

If  there  are  only  a  few  transactions  involving  the  purchase  of  bonds,  no 
special  ledger  is  needed,  it  being  satisfactory  if  a  separate  account  is  kept 
in  the  general  ledger  for  each  purchase.  If  such  transactions  are  numerous 
a  special  "bonds  owned  register"  should  be  used,  which  would  allot  to 
each  transaction  a  page,  containing  space  for  the  name  of  the  issuing 
company,  name  of  bonds,  how,  when,  and  where  payable,  nominal  and 
effective  interest  rates,  numbers  of  the  bonds,  and  columns  for  the  date, 
voucher,  principal,  par  value,  original  cost,  book  value,  and  market  value." 

The  bonds  owned  should  be  verified  by  actual  inspection,'*  and  a  very 
complete  and  detailed  list  of  them  prepared."  The  auditor  should  also  list 
all  bonds  owned  during  the  period  of  the  audit  and  ascertain  therefrom 
whether  all  interest  has  been  properly  collected  and  recorded."  Bonds 
out  as  collateral  should  be  verified  by  con-espondence."  Bonds  purchased 
during  the  period  under  audit  should  be  verified  by  correspondence,  by 
brokers'  notices,  and  by  inspection."  The  auditor  should  ascertain  that 
the  coupons  on  bonds  are  intact.™ 


BONDS 


209 


QUESTIONS 


BONDS 


General 

1.  What  is  a  bond?     (Iowa,  Dec,  1918.) 

2.  Do  unsold  treasury  bonds  constitute  a  liability?  Why?  (Mich,, 
June,  1908;  N.  Y.,  June,  1919.) 

3.  Do  unsold  bonds  of  a  railroad  company  constitute  a  liability?  If 
they  do,  under  what  accounts  would  they  appear  on  the  Ledger?  (N.  Y., 
Jan.,  1919.) 

4.  In  examining  the  accounts  of  a  corporation  you  find  the  following 
accounts  on  the  books  in  connection  with  Liberty  loans : 


Company's  subscriptions 

Subscription  for  employees 

Loans  from  banks  secured  by  $100,000  Liberty  bonds . 
Payments  by  employees 


Cr. 


$72,000 
12,000 


(a)  State  how  you  would  proceed  to  verify  each  of  the  above  accounts, 
assuming  the  examination  is  made  two  months  after  the  close  of  the  year. 

(&)  State  how  you  would  enter  these  transactions  in  the  Balance  Sheet 
of  a  corporation. 

(111.,  May,  1915.) 

5.  A  corporation  has  issued  $1,000,000  5  per  cent  debenture  bonds, 
redeemable  at  par,  out  of  profits,  at  the  end  of  twenty  years.  State 
what  method  should  be  adopted  to  provide  for  such  redemption  so  that 
each  year's  profit  may  bear  its  due  proportionate  burden  of  contribution. 
(N.  D.,  July,  1918.)  ' 

6.  The  Ledger  of  a  corporation  has  an  account  entitled :  "First  Mort- 
gage Bond  Script,"  showing  a  credit  balance  of  $967.54.  What  does 
this  balance  represent  and  how  would  you  treat  the  item  in  the  Balance 
Sheet?     (N.  Y.,  Jan.,  1904.) 

7.  Explain  fully  in  what  way,  if  at  all,  the  loss  on  bonds  held  and 
disposed  of  during  the  period  should  enter  into  the  Trading  and  Profit 
and  Loss  statements  of  a  mercantile  concern.  Give  reasons  for  including 
or  excluding.     (Kan.,  May,  1916.) 


210  C.  P.  A.  ACCOUNTING 

8.  Sketch  the  form  of  a  Bond  Ledger  which  will  provide  the  purchaser 
of  a  bond  at  a  premium  with  a  perpetual  detail  record  of  each  bond 
transaction.     (Wis.,  April,  1015.) 

9.  Make  entries  in  journal  form,  with  ])roper  explanations,  for  all  the 
various  ways  in  which  bonds  issued  by  a  corporation  may  be  sold  by  it 
and  afterwards  paid.     (Mich.,  Dec.,  1914.) 

10.  A  company,  having  $500,000  of  debentures,  bearing  5  per  cent 
interest,  which  have  been  in  existence  for  some  years,  and  which  are  re- 
payable February  1,  1907,  arranges  to  provide  the  necessary  capital  by 
the  issue,  at  par,  of  $500,000  4  per  cent  permanent  debentui-e  stock,  the 
interest  on  which  runs  from  January  1,  1907 ;  the  accounts  of  the  company 
are  made  up  to  June  30,  1907.  What,  in  your  opinion,  is  the  proper 
amount  of  debenture  interest  to  be  charged  against  the  profits  of  the  half 
year?  Give  the  reasons  upon  which  your  opinion  is  based.  (Ill,,  Dec., 
1910.) 

11.  Outline  an  entry  recording  bond  interest  due  but  not  paid  at  time 
of  making  the  entry.  What  are  the  advantages  of  such  an  entry?  (N.  Y., 
June,  1902.) 

12.  How  would  you  disclose  on  a  Balance  Sheet  dated  December  31 
bond  interest  due  January  1?     (Cal.,  June,  1917.) 

13.  What  is  the  advantage  of  amortization  in  regard  to  the  valuation 
of  bonds?     (Del.,  June,  1915.) 

14.  A  company  has  issued  bonds  of  a  par  value  of  $100,000  redeemable 
20  years  hence  at  a  premium  of  $105.  The  trust  deed  provides  that  the 
company  must  set  aside  out  of  profits  and  invest  at  the  end  of  each  year 
the  sum  of  $5,000;  the  bonds  issued  were  sold  for  $95,000. 

1.  Included  in  the  sinking  fund  are  certain  of  the  company's  bonds 

purchased  at  98.     Should  these  bonds  purchased  be  stated  at 
98,  100  or  105? 

2.  How  should  the  interest  from  the  investment  of  the  sinking  fund 

be  treated  in  the  preparation  of  the  accounts  of  the  company? 

3.  Would  it  be  correct  to  write  off  the  discount  of  $5,000  on  the  bond 

issue  during  the  first  year's  operations  because  the  company 
had  a  successful  year? 

4.  How  should  the  company  provide  for  the  repayment  of  the  bonds 

at  tbe  end  of  20  years?    Would  this  provision  apply  to  the 
bonds  purchased  as  part  of  the  sinking  fund  investment? 

Explain  the  reasons  for  the  answers  of  this  question. 

(Kan.,  Dec,  3915;  Mo.,  Dec.,  1915.) 

15.  How  would  you  show  in  the  Balance  Sheet  bonds  which  have  been 
put  up  as  collateral  and  the  indebtedness  for  which  they  are  collected? 
(Mich.,  June,  1919.) 

16.  In  the  Balance  Sheet  of  a  corporation,  how  should  the  following 
item  be  stated:  5  per  cent  mortgage  bonds  authorized,  $1,000,000;  certified 


BONDS  211 

and  issued  by  the  trustees,  $750,000.  viz.,  (1)  in  the  hands  of  the  public, 
$400,000;  (2)  pledged  as  collateral  to  secure  the  company's  notes  pay- 
able, $100,000;  (3)  in  the  custody  of  the  treasury,  $250,000.  (111.,  May, 
1912.) 

17.  On  March  15,  1919,  you  received  instructions  to  audit  the  accounts 
of  a  larjje  corporation  whose  fiscal  year  ended  December  31,  1918,  The 
corporation  has  subscribed  for  Liberty  bonds  through  various  banks  and 
at  December  31,  1918,  certain  subscriptions  had  been  paid  for  in  full  and 
delivery  of  bonds  accepted,  while  in  other  eases  part  payments  only  had 
been  made.  Bonds  have  also  been  delivered  to  employees  who  have  sub- 
scribed and  paid  for  them  in  full.  Describe  how  you  would  proceed  to 
verify  the  asset  shown  on  the  Balance  Sheet  at  December  31,  1918,  repre- 
senting bonds  on  hand  and  part  payments  made  on  subscriptions,  so  that 
you  can  give  an  unqualified  certificate  to  your  clients  as  of  December  31, 
1918.     (A.  I.  of  A.,  Nov.,  1919.) 

18.  How  would  you  deal  with  Bond  Issue  Expense  account  in  prepar- 
ing the  annual  accounts  of  a  company?  Comment  briefly  on  any  points 
which  would  need  special  consideration.     (A.  I.  of  A.,  May,  1920.) 

19.  When  preparing  a  Trading  and  Profit  and  Loss  account  at  the 
end  of  a  fiscal  year,  in  what  manner  would  yon  treat  the  interest  on 
bonds?     (N.  Y.,  Jan.,  1907.) 

20.  A  company  authorizes  its  officers  to  borrow  for  its  account  $100,000 
and  give  as  security  $200,000  of  the  first  mortgage  bonds  of  the  company. 
How  should  this  transaction  be  treated  in  the  Balance  Sheet?  (Ohio,  Nov., 
1917.) 

21.  How  would  you  disclose  on  a  Balance  Sheet  dated  December  31, 
bonds  put  up  as  collateral  when  there  is  a  strong  probability  of  their  not 
being  released?     (Cal.,  June,  1917.) 

22.  What  advantage,  if  any,  has  the  serial  plan  of  paying  bonds  over 
the  sinking  fund  plan?     (Del.,  June,  1915;  Wis.,  May,  1916.*) 

23.  A  certified  public  accountant,  having  perfonned  the  duty  at  the 
close  of  several  years,  takes  up  for  the  year  just  past  the  examination  of 
bonds — the  property  of  an  institution.  At  his  last  examination  he  found 
coupon  bonds  in  three  classes,  registered  as  to  principal,  registered  as  to 
principal  and  income,  and  unregistered.  In  his  examination  of  the  books 
of  account  of  the  institution,  he  found  that  bonds  had  been  bought  during 
the  year.  To  make  an  efficient  examination  in  the  most  expeditious  way, 
how  might  he  treat  these  various  classes?     (Mass.,  June,  1913.) 

24.  If  called  upon  to  verify  the  integrity  of  a  trust  fund  consisting 
principally  of  bonds,  state  specifically  what  particulars  of  said  bonds  you 
would  examine  critically.     (N.  Y.,  Jan.,  1918.) 

25.  What  entries  would  you,  as  auditor,  deem  proper  to  record  the 
redemption  of  bonds  by  a  company  with  the  cash  deposited  with  its  fiscal 
agent,  where  such  bonds  were  cancelled?     (N.  Y,,  June,  1918.) 

26.  A  corporation  borrows  $120,000  for  a  period  of  ten  years  to  pay 
off  an  existing  loan  at  a  higher  rate  of  interest,  paying  therefor  in  broker- 


212  C.  P.  A.  ACCOUNTING 

nge  and   costs  $2,750.     How  would   you   treat  this  item   on   the  books? 
(ind.,  Nov.,  1917.) 

Classes  op  Bonds 

27.  Name  five  classes  of  bonds,  describing  briefly  each  class  with  regard 
to  issue,  purpose,  redemption,  etc.  (Mich.,  June,  1908;  Mich.,  June, 
1913*;  N.  Y.,  Jan.,  1917*;  N.  C,  Aug.,  1917.*) 

28.  Define :  Mortgage  bond.  (N.  Y.,  June,  1898* ;  Wash.,  May,  1903* ; 
Pa.,  Nov.,  1906*;  N.  C,  June,  1916;  N.  C,  Nov.,  1918;  N.  C,  Nov.,  1919.) 

29.  Define:  First  mortgage  bonds.     (Wash.,  Sept.,  1907.) 

30.  Define:  Second  mortgage  6  per  cent  serial  gold  bonds.  (Wash., 
July,  1917.) 

31.  Define:  Collateral  trust  bonds.  (Pa.,  Nov.,  1906*;  N.  C,  Nov., 
1918;  N.  C,  Nov.,  1919.) 

32.  Define:  Guaranteed  bond.     (N.  C,  Nov.,  1918;  N.  C,  Nov.,  1919.) 

33.  Define:  Income  bonds.  (N.  Y.,  June,  1898*;  Wash.,  May,  1903*; 
Pa.,  Nov.,  1906*;  Wash.,  June,  1912;  111.,  May,  1914*;  Wis.,  May,  1916; 
N.  C,  June,  1916;  N.  D.,  Aug.,  1917;  N.  C,  Nov.,  1918;  N.  C.,  Nov., 
1919*;  A.  I.  of  A.,  May,  1921.) 

34.  Define:  Serial  bond.     (Kan.,  Dec.,  1915;  Mo.,  Dec,  1915.) 

35.  Define:  Coupon  bonds.     (N.  C,  June,  1916.) 

36.  Define:  Registered  bonds.     (N.  C,  June,  1916.) 

37.  Define :  Debenture  bond.  (N.  Y.,  June,  1898* ;  Wash.,  May,  1903* ; 
Cal.,  May,  1916;  N.  C,  June,  1916;  N.  C,  Nov.,  1918;  N.  C,  Nov.,  1919.) 

38.  Define:  Underlying  bond.     (Cal.,  May,  1916.) 

Bond  Discount  and  Premium 

39.  Define:  Bond  discount  and  bond  premiums.  How  should  each  be 
treated  in  the  annual  statement  of  a  concern?     (Minn.,  Oct.,  1916.) 

40.  Show  the  method  by  which  entries  would  be  made  for  receipts 
from  bonds  sold  below  or  above  par.  (Mich.,  June,  1913;  Mich.,  June, 
1919.) 

41.  How  would  you  treat  the  discount  on  bonds  on  the  Balance  Sheet 
of  issuing  company?  (111.,  Mav,  1914*;  Mo.,  Dec,  1914*;  111.,  May, 
1915.*) 

42.  In  the  Balance  Sheet  of  a  manufacturing  concern,  how  should 
commissions  paid  and  discounts  allowed  on  bonds  sold  be  treated?  (111., 
May,  1912.) 

43.  On  which  side  of  the  Ledger  should  the  balance  on  the  Bond  Dis- 
count Amortization  account  appear?     (Mo.,  Dec,  1914.) 

44.  The  Fort  William  Manufacturing  Company  has  created  a  first  and 
second  issue  of  mortgage  bonds  which  have  been  placed  through  a  syn- 
dicate.    The  first  mortgage  bonds  are  to  run  20  years  and  were  sold  at  115. 


BONDS  213 

The  second  mortiraire  bonds  are  to  run  40  years  and  were  sold  at  60. 
State  how  the  same  should  be  entered  on  the  books  of  the  company,  as- 
suming the  total  par  value  of  the  first  mortgage  to  be  $5,000,000  and  the 
second  $2,500,000.     (Md.,  Oct.,  1919.) 

45.  A  corporation  has  issued  $1,000,000  of  6  per  cent  20-year  bonds 
at  90  and  for  8  years  has  written  off  5  per  eait  of  the  discount  each  year. 
Last  year  an  opportunity  occurred  to  buy  in  $200,000  at  85,  which  was 
done  and  the  bonds  cancelled.  The  directors  propose  to  take  up  into  their 
year's  revenue  $30,000,  the  discount  saved  upon  extinction  of  this  liability. 
Do  you  approve?  If  not.  what  course  would  you  advise,  or,  if  they 
insist,  how  would  you  act?     (A.  I.  of  A.,  Nov.,  loiS;  W.  Va.,  May,  1919.) 

46.  What  proportion  of  $15,000 — commission  paid  for  n^x»tiating  a 
sale  of  bonds,  to  run  10  years — should  be  treated  as  an  asset  at  the  ead  of 
the  first  year?     Give  reasons.     (N.  Y.,  Jan.,  1901.) 

47.  If  a  company  sells  its  own  bonds  at  a  premium,  is  the  pranium 
received  a  legitimate  profit  of  the  company?  (Ohio,  Nov.,  1915;  W.  Va., 
May,  1919;  N.  D.,  July,  1919.) 

48.  Explain  the  different  ways  in  which  the  discount  on  bonds  sold 
below  par  may  be  written  off  bv  the  issuing  concern.  Which  is  the  best? 
(Mich.,  Pec.,  1913;  Mich.,  Dec.,'l914.») 

49.  "Discounts  and  premiums  on  bonds  are  in  effect  an  addition  to  or 
a  deduction  from  the  interest  rate  paid  on  the  bonds  over  their  life." 
(Dickinson.)  Defend  and  illustrate  this  statement  in  view  of  your  defi- 
nition of  interest.     (Kan.,  May,  1916.) 

50.  Classify  according  to  the  subdivision  of  assets,  liabilities,  pro- 
prietary interest,  income  and  expense  under  which  they  should  be  grouped : 
(a)  discount  on  bonds  purchased;  (6)  amortization  of  discount  on  bonds 
purchased;  (r)  amortization  of  premium  on  bonds  issued;  (d)  premium 
on  bonds  issued.     (Wis.,  May,  1919.) 

51.  State  the  various  accounts  which  should  be  kept  for;  (1)  recordii^ 
bonds  issued  and  bonds  purchased;  (2)  the  premium  and  discount  on  these 
bonds;  and  (3)  outline  four  methods  of  treating  premium  and  discounts 
on  bonds.     (Wis.,  Nov.,  1919.) 

52.  A  corporation  sells  its  first  mortgage  bonds  at  $10,000  premium 
and  its  second  mortgage  bonds  at  $10,000  discount.  Give  your  views  as 
to  the  proper  treatment  of  these  items  of  premium  and  discount  (N.  Y, 
Jan.,  1901:  Wis.,  April,  1914.*) 

53.  What  does  the  Interstate  Commerce  Commission  require  in  con- 
nection with  the  recording  and  the  financial  presentation  of  premiums  and 
discounts  on  bonds,  and  premiums  and  discounts  on  stocks?  State  the 
theoretic  reason  for  the  difference,  if  any,  in  the  handling  of  these  two 
classes  of  facts.     (N.  Y.,  Jan.,  1914.) 

54.  Frame  any  entries  necessary  to  record  the  action  of  the  directors 
as  it  appears  in  the  minutes  of  the  meeting.  The  president  reported  that 
a  finn  of  bankers  had  offered  to  purchase  $200,000  of  the  company's  20- 
year  5  per  cent  bonds  to  be  dated  October  1,  1917,  at  93  and  accrued 


214  C.  P.  A.  ACCOUNTING 

interest.    He  was  authorized  to  accept  the  offer  and  deliver  bonds  on 
that  date.     (A.  I.  of  A.,  Nov.,  1917.) 

55.  A  company  having  sold  its  bonds  at  a  rate  above  par,  how  should 
the  premium  received  be  treated  on  the  books  of  the  company?  (N.  Y., 
June,  1906*;  AVash.,  Sept.,  1907*;  Ohio,  Nov.,  1913.) 

56.  What,  in  your  opinion,  is  the  most  equitable  basis  of  writing  off 
discount  incurred  on  an  issue  of  serial  bonds  by  a  corporation?  Mention 
at  least  two  other  bases  which  might  be  followed,  stating  wherein  the 
method  you  suggest  is  more  equitable  than  the  others.     (111.,  May,  1914.) 

57.  A  firm  purchased  ten  $1,000  bonds  at  97^/2,  due  January  1,  1915, 
bearing  5  per  cent  interest,  payable  semiannually.  What  procedure  would 
you  adopt  to  care  for  the  discount  at  maturity?  (N.  Y.,  Jan.,  1914;  N.  D., 
Aug.,  1917.*) 

58.  How  should  the  interest  received  en  a  bond  bought  at  a  premium 
be  treated?     (N.  Y.,  June,  1897.) 

59.  Give  various  methods  of  treating  premiums  and  discounts  on  bonds 
purchased.  Give  the  advantages  and  disadvantages  of  each  method  and 
state  which  you  consider  the  best,  with  reasons  for  your  belief.  (111., 
May,  1907*;  Fla.,  July,  1909;  Va.,  Nov.,  1910*;  Mich.,^  Dec,  1916.*) 

60.  A  company  having  an  issue  of  $1,000,000  of  bonds  running  for 
the  period  of  30  years  and  bearing  interest  at  the  rate  of  6  per  cent  per 
annum,  payable  annually,  has  asked  your  advice  as  to  an  equitable  method 
of  disposing  in  its  accounts  of  a  discount  of  $50,000  paid  to  its  brokers 
for  the  negotiation  of  the  bonds.  Y''ou  have  ascertained  that  the  bonds 
are  payable  in  annual  installments  of  $25,000  during  the  fii-st  20  years 
and  in  installments  of  $50,000  per  annum  during  the  next  5  years,  the 
final  payment  of  $250,000  being  due  at  the  end  of  the  30th  year.  In 
answering  this  question,  submit  details  of  the  process  you  adopt  in  arriv- 
ing at  your  conclusion.     (Wash.,  July,  1917.) 

61.  Illustrate  in  the  form  of  Journal  entries  the  accrual  of  discount 
and  the  amortization  of  the  premium  on  bond  investments.  Explain  or 
illustrate  the  relation  of  each  to  the  interest  receipts  and  to  the  income 
returns.     (Md.,  Jan.,  1909.) 

62.  A  concern  of  which  you  are  the  auditor  buys  for  investment  $10,000 
worth  of  7  per  cent  15-year  gold  bonds  at  115.  How  would  you  show 
these  in  the  Balance  Sheet  of  the  concern  and  what  disposition  would 
vou  make  of  the  premium  on  the  books?  (N.  Y.,  Dec,  1897*;  N.  Y., 
Feb.,  1910*;  Ohio,  Nov.,  1918.) 

63.  In  case  of  bonds  purchased  at  a  premium  or  at  a  discount,  to  be 
held  until  maturity,  state  how  the  price  should  be  disposed  of  on  the 
books  at  purchase,  at  maturity,  and  at  any  intervening  time.  (N.  Y., 
June,  1898.) 

64.  How  would  you  treat  in  the  accounts  bonds  bought  at  90,  with  11 
years  yet  to  run?     (Cal.,  Nov.,  1916.) 


BONDS  215 

Auditing 

65.  State  how  yon  would  verify  the  Bonds  Payable.  (Ohio,  Dec.,  1908; 
N.  C,  June,  1916;  N.  C,  Aug.,  1917*;  111.,  Dec,  1918.*) 

66.  State  in  detail  what  steps  you  would  consider  necessary  to  verify 
the  First  Mortgage  Gold  Bonds.     (111.,  May,  1904*;  111.,  May,  1916.) 

67.  How  would  you  verify  the  Investment  Bonds  in  making  a  Balance 
Sheet  audit?  (Mich.,  June,  1912;  Mass.,  Oct.,  1914*;  Ohio,  Nov., 
1915.) 

68.  State  for  Balance  Sheet  purposes  the  rules  of  valuation  that  apply 
in  long-time  bonds  bought  at  a  discount  for  speculation,  the  market  value 
of  which  has  advanced.     (N.  Y.,  Jan.,  1911.) 

69.  State  for  Balance  Sheet  purposes  the  rules  of  valuation  that 
apply  in  long-time  bonds  bought  at  a  jiremium  for  investment,  the  market 
value  of  which  has  advanced.     (N.  Y.,  Jan.,  1911.) 

70.  Under  what  circumstances  would  you  pei'mit  a  client  to  carry  bonds 
at  cost  when  the  market  is  lower?     (Cal.,  May,  1916.) 

71.  What  is  the  general  rule  as  to  bonds  of  other  corporations  held 
as  an  investment?  (N.  Y.,  Dec,  1898*;  Va.,  Oct.,  1912*;  Ohio,  Nov., 
1918.) 

72.  "What  Avould  you  consider  satisfactory  evidence  of  the  correctness 
and  propriety  of  expenditures  of  Commission  Paid  to  Bankers  for  Sale 
of  Bonds?     (A.  I.  of  A.,  May,  1918.) 

73.  How  would  you  proceed  to  verify  the  Liberty  bonds  on  hand  rep- 
resenting company's  and  employees'  subscriptions?     (111.,  Dec,  1918.) 


216  C.  P.  A.  ACCOUNTING 


PROBLEMS 

BONDS 

1.  A  $10,000  5  per  cent  semiannual  coupon  (bond)  is  bought  on  a  4 
per  cent  basis,  due  1^  years  hence.  What  did  it  cost?  (A.  I.  of  A., 
Nov.,  1918.) 

2.  Determine  the  price  of  a  4^  per  cent  bond  in  the  amount  of  $10,000, 
with  four  years  to  run,  purchased  so  as  to  net  3^/^  per  cent.  The  interest 
is  payable  semiannually.  Construct  a  schedule  of  amortization.  (Pa., 
Nov.,  1918.)  (Note:  8th  power  of  1.0175  is  1.14888178,  whose  reciprocal 
is  .87041157.) 

3.  What  must  be  paid  for  a  bond  $1,000  par  maturing  in  10  years 
Avith  4  per  cent  interest,  payable  semiannually,  to  net  the  purchaser  6 
per  cent?  (Mich.,  Dec,  1913.)  (Note:  20th  power  of  1.03  is  1.80611123, 
whose  reciprocal  is  .55367575.) 

4.  "X"  and  "Y"  are  dealers  in  bonds  and  securities,  sharing  profits 
and  losses  in  the  proportion  of:  "X,"  three-fourths  and  "Y,"  one-fourth. 
They  employ  "Z"  to  sell  securities,  agreeing  to  pay  him,  in  lieu  of  a  salary, 
an  amount  equal  to  25  per  cent  of  the  net  profits  to  be  divided  between 
the  partners.  During  the  period  of  "Z's"  employment  the  firm  purchased 
$100,000  Topeka  Traction  Company  first  mortgage  5  per  cent  bonds,  on 
a  3  per  cent  basis.  The  bonds  mature  in  one  year  and  one-half.  Interest 
is  payable  semiannually.  These  bonds  are  held  by  "X"  and  "Y"  until 
maturity. 

Prepare  a  statement  of  the  Topeka  Traction  Company  bond  accounts, 
showing  cost,  amortization,  and  interest.  The  total  profit  to  be  adjusted 
in  the  contract  with  "Z"  is  $15,000.     Show  the  division  of  this  profit. 

(Kan.,  May,  1916.) 

5.  A  bond,  bearing  interest  at  5  per  cent  per  annum,  payable  annually, 
and  repayable  in  five  years,  with  bonus  of  10  per  cent,  is  for  sale.  What 
price  can  a  purchaser  pay  who  desires  to  realize  6  per  cent  on  his  invest- 
ment?    (V^  at  6  per  cent  =r  .7473.)     (A.  I.  of  A.,  May,  1919.) 

6.  On  Januar\'  1,  1915,  an  investor  purchased  {ex  interest)  a  5  per 
cent  $10,000  bond,  the  interest  being  payable,  $250  on  January  1  and  $250 
on  July  1,  of  each  year. 

Assuming  that  the  bond  matured  on  July  1,  1917,  and  yielded  2  per 
cent  semiannuallv  on  his  investment,  what  price  did  he  pav?  It  is  given 
that  (1.02)' =  1.104081. 

Using  the  figures  derived  from  your  eoiiii)iitation,  make  illustrative 
entries   for   income   and    amortization    covering   the   first   two   periods. 


BONDS  217 

(Mass.,  Oct.,  1917.)     (Note:  5th  power  of  1,02  is  1.10408080,  whose  re- 
ciprocal is  .90573081.) 

7.  A  savings  bank  on  July  1,  1916,  purchased  50  bonds  of  par  value 
$1,000  each  of  the  "A.  B."  R.  R.  Company.  The  bonds  mature  January 
1,  1925,  and  bear  interest  at  the  rate  of  41/2  per  cent  per  annum,  payable 
semiannually.  They  were  purchased  on  a  semiannual  basis  of  2  per  cent, 
(1)  What  was  the  total  cost  of  the  bonds?  (2)  Construct  a  schedule  of 
amortization  for  the  premium  on  the  bonds.  (3)  Set  up  an  account  with 
the  bond  issue,  showing  what  entries  would  be  made  on  each  interest  date. 
(Pa.,  Nov.,  1916.)  (Note:  17th  power  of  1.02  is  1,40024142,  whose 
reciprocal  is  .71416256.) 

8.  A  corporation  decided  to  issue  and  sell  bonds  to  the  amount  of 
$100,000.  The  denomination  of  such  bonds,  $1,000  each;  term  of  bonds, 
15  years;  interest  rate,  5  per  cent,  payable  semiannually.  On  January  1, 
1914,  these  bonds  were  sold  for  $105,411,33,  or  on  a  414  per  cent  return 
basis,     July  1,  1914,  interest  was  paid,  amounting  to  $2,500, 

(a)     What  entry  should  the  corporation  have  made  when  the  bonds 

were  sold? 
(&)     What  entry  should  the  corporation  have  made  when  it  paid  the 

$2,500  interest  referred  to  above? 
(c)     What  entiy  should  the  purchaser  of  these  bonds  have  made  when 

he  received  the  first  interest  payment? 

(Wis,,  April,  1915,) 

9.  An  issue  of  $500,000  of  bonds  is  made  by  a  corporation,  redeemable 
at  par  in  10  years.  The  trust  deed  provides  for  the  creation  of  a  sinking 
fund  for  the  redemption  of  the  bonds,  and  for  investment  of  the  sinking 
fund.     The  bonds  are  issued  at  a  discount  of  2  per  cent. 

You  are  required : 

(a)     To  prepare  entries  setting  up  the  bond  issue  on  the  books  of  the 

corporation, 
(&)     To  prepare  (for  one  year  only)  the  entries  which  must  be  made 

on  the  books  each  year  with  respect  to  the  sinking  fund  and  the 

sinking-fund  investment. 

(c)  To  prepare  the  entries  which  will  be  required  on  the  redemption 

of  the  bonds. 

(d)  To  show  how,  during  the  life  of  the  bonds,  the  various  accounts 

relative  thereto  should  appear  on  the  books  of  the  corporation. 
Note:  In  your  answer  disregard  interest.  (Wash.,  May,  1910.) 

10.  On  December  15,  1914,  the  stockholders  of  the  "A"  Corporation 
authorized  an  issue  of  $100,000  10-year  5  per  cent  first  mortgage  bonds. 
These  bonds  were  sold  on  January  1,  1915,  at  95.  On  January  1,  1916, 
another  duly  authorized  issue  of  $100,000  20-year  6  per  cent  bonds  was 
sold  at  102. 

In  accordance  with  the  terms  of  the  bond  recitals  the  sinking  fund  in- 
stallments were  to  be  invested  in  outside  securities,  and  on  January  1, 
1916,  a  portion  of  the  pro  rata  installment  of  the  first  bond  issue  was 


218  C.  P.  A.  ACCOUNTING 

used  in  purchasing  100  51/2  per  cent  bonds  of  the  "X"  Corporation  at  9S. 
On  January  1,  1917,  the  pro  rata  installments  were  invested  as  follows: 
Issue  No.  1,  100  61/2  per  cent  bonds  of  the  "T"  government  at  102 ;  issue 
No.  2,  50  7  per  cent  bonds  of  the  "W"  government  at  par. 

Draft  the  proper  entries  to  record  the  above  transactions,  and  show 
the  ledger  accounts  and  balances  as  of  January  2,  1917.  Interest  calcula- 
tions need  not  be  given. 

(Wis.,  April,  1917.) 

11.  Corporation  "A"  issues  50  bonds,  par  value  $50,000,  bearing  5 
per  cent  interest,  payable  annually.  The  bonds  are  numbered  serially, 
and  are  to  be  retired  in  consecutive  groups  of  10  each  year.  They  are 
to  be  sold  at  date  of  issue  for  an  average  price  of  $950. 

(a)     Submit,  in  form  of  ledger  accounts,  all  entities  required  to  handle 
this  bond  issue,  in  what  you  consider  the  most  equitable  manner, 
from  date  of  issue  to  retirement. 
(6)     Corporation  "B"  buys  bonds  Nos.  21  to  40,  inclusive,  on  date  of 
issue,  at  $950  each,  and  sells  Nos.  21  to  30  at  the  end  of  two 
years  for  $1,000  each.     The  other  ten  bonds  are  retired  when 
due. 
Submit,  in  the  form  of  ledger  accounts,  all  necessary  entries  in  Cor- 
poration "B's"  books  for  handling  the  matter  in  what  you  consider  the 
most  equitable  manner. 

(Wis.,  May.  1916.) 

12.  What  could  a  purchaser  who  wished  to  realize  3  per  cent  on  his 
investment  give  for  a  bond  for  $10,000  which  had  four  j-ears  to  run  at  5 
per  cent  interest,  payable  yearly,  and  thereafter  w^as  payable  with  a  bonus 
of  10  per  cent?  (Note:  3rd  power  of  1.03  is  1.12550881;  whose  recipro- 
cal is  .88848705.)  (A.  I.  of  A.,  Nov.,  1920.) 

13.  The  4%  per  cent  Victory  notes  mature  at  par  on  May  20,  1923. 
If  a  purchaser  buys  at  $96.20  on  May  20,  1920,  calculate  the  approximate 
yield  per  cent. 


Given: 


A,. 

V». 


2H7o 


5.4264 
.8498 


3% 


5.4172 
.8375 


(A.  I.  of  A.,  May,  1920.) 


CHAPTER  XII 

DIVIDENDS  AND  SURPLUS 

Classes  of  Dividends — Dividends  are  those  profits  of  a  corporation  which 
are  divided  among  the  stockholders/  However,  in  common  parlance,  any 
payment  made  to  a  stockholder  Ls  called  a  dividend.  Dividends  may  be 
classified  according  to  source  of  funds  and  manner  and  time  of  payment. 
There  aj-e  two  classes  of  dividends  divided  according  to  source  of  funds, 
namelj^  those  paid  out  of  profits  and  those  paid  out  of  capital.'  The  five 
kinds  of  dividends,  classified  according  to  manner  of  payment,  are  the 
dividends  paid  in  cash,'  property,*  stock,'  bonds,'  and  scrip.'  If  the  com- 
pany is  unusually  successful,  a  special  dividend  in  addition  to  the  regular 
dividend  may  be  declared,  or  an  interim  dividend,  a  dividend  between 
regular  dividend  dates,  may  be  declared.* 

Dividends  Out  of  Capital — Except  in  case  of  liquidation  or  a  legally 
authorized  reduction  of  capital  stock,  di%'idends  paid  out  of  capital  are 
illegal,  and  the  directors  responsible  for  them  are  usually  jointly  and 
severally  liable  therefor."  It  is  legal  in  some  states  for  extractive  industry 
corporations  to  disregard  the  wasting  assets  and  thus  declare  dividends  in 
excess  of  real  profits.'"  Modern  practice  is  now  requiring  the  setting  up 
of  reserves  for  wasting  assets,  an  improvement  the  adoption  of  which  has 
been  greatly  popularized  by  the  federal  income  tax  requirements.  If  an 
extractive  industry  corporation  wishes  to  pay  its  stockholders  more  than 
the  net  profit  after  depletion  has  been  charged,  the  excess  should  be 
charged  to  the  capital  payments  account,  which  account  would  be  deducted 
from  the  capital  stock  account  on  the  balance  sheet." 

Dividends  Out  of  Profits — Both  operating  and  capital  profits,  if  realized, 
may  be  declared  as  dividends."  Unrealized  profits"  and  premium  on 
capital  stock,"  however,  should  not  be  so  distributed.  The  old  English 
rule  for  dividends  was  that  fixed  assets  could  be  lost  and  yet  the  excess 
of  current  receipts  over  current  payments  could  be  divided,  but  that  the 
current  assets  must  be  maintained."  This  rule  has  not  Ijeon  changed  as 
to  current  assets,  but  all  waste,  both  of  fixed  and  current  assets,  inci- 
dental to  the  process  of  making  profits  must  now  be  made  good  out  of 
the  profits  earned  l>efore  dividends  can  be  paid." 

Declaration  of  Dividends — The  declaration  of  di\'idends  is  subject  to 
the  provisions  of  the  corporate  charter  and  by-laws,  and  the  rights  of 
other  stockholders,"  but  otherwise,  in  the  absence  of  fraud,  is  under 
the  control  of  the  directors."  However,  obligation  is.  incumbent  on  the 
directors    if    any    improper   withholding   can    he    shown."      The   minute 


'For  explanatiou  of  superior  figures  see  page  337. 

219 


220  C.  P.  A.  ACCOUNTING 

books  should  give  the  dividend  rate,  date  and  manner  of  payment,  and 
to  whom  payable.'"  Dividends  are  paid  to  stockholders  of  record  as 
of  a  set  date ;"  they  may  be  postdated  but  may  not  be  antedated."  Stock- 
holders are  entitled  to  be  notified  of  each  dividend  declaration."  The 
declaration  of  dividends  may  be  revoked  up  to  the  time,  but  not  after, 
notice  has  been  given  except  in  the  case  of  illegal  dividends  which  may 
be  revoked  any  time  previous  to  payment,"  and  after  payment  to  stock- 
holders cognizant  of  the  source."    Treasury  stock  does  not  draw  dividends.^' 

Payment  of  Dividends — When  dividends  are  declared  and  notice  thereof 
given,  a  liability  is  incurred."  This  liability  should  be  shown  in  the 
dividends  payable  account,  if  the  dividend  is  not  payable  on  the  day 
it  was  declared.**  Sometimes  declared  dividends  are  not  recorded  until 
paid,  but  this  overlooks  a  current  liability  which  should  appear  on  a 
balance  sheet  prepared  between  the  dividend  declaration  and  dividend 
payment  dates,*  and  also  overlooks  unclaimed  but  payable  dividends 
which  should  be  shown  among  the  current  liabilities.^  Unclaimed  dividends 
can  be  returned  to  surplus  only  by  action  of  the  directors."  When  the 
dividends  paj'^able  are  set  up,  either  the  current  profit  and  loss  account" 
or  the  surplus  account  may  be  charged,"  but,  as  dividends  frequently  ex- 
ceed the  profits  of  any  one  year,  it  seems  preferable  to  debit  the  surplus 
account.  If  the  dividends  are  charged  to  profit  and  loss,  the  debit  is 
frequently  made  through  the  dividends  declared  account,"  which  account 
would  appear  in  the  appropriation  section  of  the  profit  and  loss  state- 
ment." 

If  dividends  are  payable  in  cash,  the  entries  to  record  them  would  be :" 

Entry  No.  1 

Surplus  (or  Profit  and  Loss) xxxxxxx 

Dividends  Payable xxxxxxx 

Entry  No.  2 

Dividends  Payable xxxxxxx 

Cash xxxxxxx 

Sometimes  property,  instead  of  cash,  is  distributed  as  dividends.''  On 
account  of  the  difficulty  of  equitably  distributing  property,  dividends 
paid  in  property  are  rare,  except  for  the  property  dividends  which 
distribute  corporate  securities."  Property  dividends  are  recorded  the  same 
as  cash  dividends,  except  that  the  property  distributed,  instead  of  cash,  is 
credited  in  the  second  entry  above.* 

Directors  sometimes  declare  dividends  payable  in  either  unissued  or 
treasury  stock,  the  entries  being  the  same  as  for  cash  dividends  except  that, 
instead  of  cash,  unissued  stock  or  treasury  stock,  as  the  case  may  require, 
is  credited  in  the  second  entry.** 

If  directors  are  unwilling  to  issue  stock  and  cannot  borrow  money  to 
advantage,  they  may  declare  a  scrip  dividend,  if  they  wish  to  liquidate  the 
debt  within  a  short  time,"  or  a  bond  dividend,  if  the  obligation  is  not  to 
mature  in  the  near  future."  The  entries  for  such  dividends  would  consist 
of  entry  No.  1,  as  al)ove,  followed  by  a  debit  to  dividends  payable  and  a 


DIVIDENDS  AND   SURPLUS  221 

credit  to  bonds  payable  or  scrip  payable,  and,  at  the  maturity  of  the  obliga- 
tions, by  a  debit  to  bonds  payable  or  scrip  payable  and  a  credit  to  cash." 
The  entry  for  the  interest  on  the  bonds  or  scrip  would  be  a  debit  to  interest 
and  a  credit  to  cash." 

Dividends  are  sometimes  applied  on  unpaid  installments  on  capital 
stock  subscriptions."  In  such  cases  the  dividends  payable  account  is 
debited  and  the  installment  on  stock  account  credited  for  the  amount  so 
applied.*" 

Cumulative  Dividends — When  a  preferred  dividend  is  "passed,"  the  sur- 
plus account  may  be  debited  and  the  unpaid  preferred  dividend  account 
credited,  but  this  procedure  is  poor  in  that  it  reduces  the  surplus,  and, 
indeed,  may  change  a  surplus  into  a  deficit,  for  the  sake  of  a  liability  which 
is  only  contingent."  A  substitute  entry  is  to  debit  preferred  stock  divi- 
dends account  (an  asset)  and  credit  unpaid  preferred  dividends  account, 
a  procedure  which  offsets  a  contingent  asset  with  a  contingent  liability.** 
Another  way  of  handling  "passed"  cumulative  dividends  is  simply  to  men- 
tion them  in  a  footnote  to  the  balance  sheet."  The  last  method  seems  the 
best,  as  cumulative  dividends  are  not  really  liabilities  until  they  are  de- 
clared. 

Dividend  Book — The  dividend  book  is  a  bound  or  loose-leaf  book,  con- 
taining for  each  declared  dividend  an  alphabetical  list  of  the  stockholders 
entitled  to  receive  dividends,  together  with  their  addresses,  number  of 
shares  held,  amount  of  dividends  paid,  and,  if  the  dividends  are  paid  by 
mailed  checks,  the  check  numbers,  or  if  the  dividends  are  paid  in  the  office, 
the  stockholders'  signatures."" 

Audit  of  Dividends — The  authority  for  the  dividend  should  be  verified 
from  the  minute  book."  The  list  of  stockholders'  holdings  in  the  dividend 
book  should  be  verified  by  checking  a  few  of  the  larger  amounts,  taken  at 
random,  with  the  capital  stock  ledger,  and  by  checking  the  addition  of  the 
number  of  shares,  as  listed  in  the  dividend  book,  against  the  aggregate 
shares  outstanding,  as  shown  in  the  general  ledger."  The  audit  of  the 
dividend  payments  consists  in  comparing  the  verified  list  of  the  amounts 
due  to  stockholders  with  the  dividend  checks  or  warrants  which  have  been 
paid  after  endorsement  by  the  stockholders."  The  outstanding  dividends 
payable  must  then  be  traced." 

Profit  Sharing — When  a  part  of  the  profits  is  distributed  to  employees 
as  such,  the  process  is  known  as  profit-sharing  and  should  be  recorded  by 
debiting  surplus  or  profit  and  loss  and  crediting  the  "bonus  to  employees" 
account,  which  account  is  closed  when  payment  is  made." 

Undivided  Profits  Account — Most  corporations  make  little  or  no  dis- 
tinction between  undivided  profits  and  surplus,'"  but  banks  usually  treat 
surplus  as  a  part  of  the  permanent  capital  created  by  systematic  alloca- 
tions of  current  profits,  and  treat  undivided  profits  as  a  reservoir  to  hold 
profits  remaining  after  the  declaration  of  dividends  and  the  allocations  to 
surplus."  .     , 


222  C.  P.  A.  ACCOUNTING 

Surplus  Defined — In  the  broadest  sense,  surplus  I'epresents  the  excess 
of  the  net  worth  of  a  corporation  over  the  par  value  of  the  capital  stock 
outstanding."*  Under  this  definition,  true  reserves  are  the  part  of  surplus 
called  restricted  or  appropriated  surplus,"*  while  the  surplus  available  for 
dividends  is  called  free  surplus.'"  In  the  narrower  sense,  surplus  repre- 
sents the  retained  past  profits  which  are  available  for  dividends."  As 
the  narrower  use  of  the  word  tends  toward  clearness,  it  is  the  preferable. 

Kinds  of  Surjiliis — Surplus  may  be  classified  by  sources  into  (a)  con- 
tributed, or  surplus  created  by  contribution  either  at  the  sale  of  stock  by 
issue  at  a  premium*"  or  after  donation  of  stock •,"^  (b)  revaluation,  or 
surplus  created  either  by  increasing  the  book  valuations  of  fixed  assets,**  or 
by  reducing  par  value  of  capital  stock,°°  and  (c)  revenue,  or  surplus 
accumulated  out  of  past  profits.''  This  classification  is  noteworthy  as  con- 
tributed and  revaluation  surplus  should  not  be  declared  in  dividends 
although  such  declaration  is  not  illegal,"  while  revenue  surplus  may  be 
distributed  as  dividends.  "Capital  surplus"  is  sometimes  used  as  a 
synonym  for  "contributed  surplus.""* 

Statement  of  Surplus — All  mistakes  made  in  prior  periods  in  the  valua- 
tion of  inventories,  or  in  the  estimate  of  losses  due  to  depreciation  and  bad 
debts,  or  in  the  failure  to  differentiate  between  capital  and  revenue  ex- 
penditures should  be  adjusted  directly  into  the  surplus  account  and  not 
through  the  pi'ofit  and  loss  account.™  Losses  and  gains  not  occasioned  by 
the  ordinary  operation  of  the  business  are  also  recorded  directly  in  the 
surplus  account.'"  If  these  adjustments  are  few  and  simple,  a  statement 
of  surplus  may  be  either  annexed  to  the  profit  and  loss  statement"  or  em- 
bodied in  the  balance  sheet."  If  the  adjustments  are  numerous  and  com- 
plex, a  separate  statement  of  surplus  should  be  prepared." 

The  statement  of  surplus  is  prepared  somewhat  as  follows  :'* 

Statement  of  Surplus,  December  31,  1920 

Balance  of  Surplus  as  of  December  31,  1919. .  . .  xxxxxxx 

Adjustments  applicable  to  period  ending  Decem- 
ber 31,  1919: 

Additions  (itemized) xxxxxxx 

Deductions  (itemized) xxxxxxx     xxxxxxx 

True  Surplus  as  of  December  31,  1919 xxxxxxx 

Extraordinary  Profits  (or  Losses)  this  period.. .  .     xxxxxxx 

Net  Profit  this  period xxxxxxx     xxxxxxx 

Amount  available  for  appropriations xxxxxxx 

Appropriations  of  Surplus: 

Reserves  (itemized) xxxxxxx 

Dividends  declared xxxxxxx     xxxxxxx 

Net  Surplus  as  on  December  31,  1920 xxxxxxx 

Audit  of  Surplus — The  auditor  must  secure  a  detailed  analj'sis  of  the 
surplus  account."  This  is  accomplished  through  the  preparation  of  a 
statement  reconciling  the  surplus  balances  on  the  balance  sheets  of  the 


DIVIDENDS  AND   SURPLUS  223 

previous  and  current  years,  viz.,  tlie  drawing  up  of  a  statement  of  surplus." 
The  books  should  be  examined  to  ascertain  if-  the  surplus  was  earned,  and, 
if  it  was  not,  a  careful  record  of  its  creation  should  be  made." 

Deficit — A  deficit  is  the  excess  of  the  par  value  over  the  actual  value  of 
the  stock  issued.'"  Although  the  deficit  account  is  sometimes  shown  on  the 
balance  sheet  as  an  asset,™  it  should  be  deducted  from  the  other  net  worth 
accounts  as  it  is  not  an  asset." 


224  C.  P.  A.  ACCOUNTING 


QUESTIONS 

dividends  and  surplus 

Dividends 

1.  Explain  the  meaning  and  use  of  the  Dividend  account.  (La.,  May, 
1913.) 

2.  Describe  the  nature  of  the  Dividend  account.  (N.  Y.,  Dec,  1896; 
W.  Va.,  May,  1917.) 

3.  Describe  fully  the  Dividend  account.  (N.  Y.,  Jan.,  1902;  Pa.,  May, 
1903;  Ohio,  March,  1910.*) 

4.  Give  the  theory  of  dividends.     (N.  Y.,  Jan.,  1920.) 

5.  What  is  a  dividend?     (Mich.,  July,  1906;  Iowa,  Dec,  1918.) 

6.  What  is  an  interim  dividend?     (Wash.,  June,  1915.) 

7.  What  is  a  cash  di\ddend?     (Wash.,  June,  1915.) 

8.  State  where  the  Dividend  account  is  employed.  (N.  Y.,  Jan.,  1902* ; 
Ohio,  March,  1910.*) 

9.  When  mav  dividends  on  common  stock  be  paid?  (Iowa,  Dec,  1918; 
S.  C,  Sept.,  1919.*) 

10.  What  examination  should  the  auditor  make  of  the  items  in  a  Bal- 
ance Sheet  to  determine  whether  a  dividend  is  justified?  (S.  C,  Sept., 
1919.) 

11.  (o)  Out  of  what  funds  may  dividends  be  paid?  (h)  If  dividends 
are  not  paid  in  accordance  with  your  answer  to  the  preceding  question, 
have  the  creditors  of  the  corporation  any  grounds  for  action  against  the 
company  or  its  stockholders?     (N.  D.,  July,  1916.) 

12.  From  what  funds  are  dividends  payable  and  what  penalties  attach 
to  improperly  declaring  or  paying  them?     (Fla.,  April,  1907.) 

13.  State  how  declaration  and  payment  of  dividends  are  usually  re- 
corded in  books  of  accounts.  (Mich.,  July,  1906;  A.  I.  of  A.,  May, 
1919.*) 

14.  How  would  you  treat  the  following  dividends  on  common  stock 
in  preparing  a  Balance  Sheet :  Dividends  declared  and  receivable  on 
stock  of  subsidiary  company  which  is  controlled  and  whose  accounts  are 
consolidated?     (Mo.,  Dec,  1914.) 

15.  How  would  you  treat  the  following  dividends  on  common  stock  in 
preparing  a  Balance  Sheet:  Dividends  declared  on  stock  in  treasury; 
declared  in  reduction  of  capital?     (Mo.,  Dec,  1914.) 


DIVIDENDS  AND  SURPLUS  225 

16.  How  would  you  classify  the  followinj?  item  in  the  Balance  Sheet 
or  Profit  and  Loss  account:  Capital  stock  dividend?  (N.  Y.,  Jan.,  1907*- 
Kan.,  Dec,  19155  Mo.,  Dec,  1915.)  ' 

17.  How  would  you  indicate  on  the  Balance  Sheet,  December  31 :  Or- 
dinary dividends  for  the  year,  declared  the  following  January  22?  (A  I 
of  A.,  Nov.,  1918;  W.  Va.,  May,  1919.) 

IS.  How  would  you  deal  with  the  noncumulative  preferred  dividends 
declared  June  30,  1914,  in  auditing  a  Balance  Sheet  as  at  June  30,  1914, 
to  be  certified  by  you?  (Mo.,  Dec,  1914;  A.  I.  of  A.,  Nov.,  1918* ;  W.  Va., 
May,  1919.*) 

19.  How  would  you  deal  with  the  preference  dividends  declared  July 
1,  1914,  in  auditing  a  Balance  Sheet  as  at  June  30,  1914,  to  be  certified 
by  you?     (Mo.,  Dec,  1914.) 

20.  Classify  Dividends  Declared  and  Dividends  Payable  according  to 
the  subdivision  of  assets,  liabilities,  proprietary  interest,  income  and  ex- 
penses under  which  they  should  be  grouped.     (Wis.,  May,  1919.) 

21.  In  preparing  the  Balance  Sheet  of  a  corporation,  how  would  you 
treat  arrears  of  cumulative  dividends  on  preferred  stock?  (N.  Y.,  June, 
1901*;  N.  Y.,  June,  1902*;  Wash.,  Sept.,  1907*;  111.,  May,  1908*;  N.  Y., 
Feb.,  1910*;  Va.,  Nov.,  1910;  Va.,  Oct.,  1912*;  La.,  May,  1913*;  Wis., 
April,  1914*;  111.,  May,  1914*;  Mass.,  Oct.,  1914*;  Ohio,  Nov.,  1915*; 
Ohio,  Nov.,  1916;  Md.,  Dec,  1917*;  N.  Y.,  June,  1918*;  Pa.,  Nov.,  1918*; 
A.  I.  of  A.,  Nov.,  1918*;  W.  Va.,  May,  1919*;  Mich.,  June,  1919.*) 

22.  How  would  you  handle  cumulative  dividends  in  arrears  on  the 
Balance  Sheet  in  the  case  of  a  corporation  owning  the  stock  with  respect 
to  which  the  cumulative  dividends  are  in  arrears?     (Pa.,  Nov.,  1918.) 

23.  When  is  it  proper  to  record  on  the  books  of  a  corporation  a  divi- 
dend on  cumulative  preferred  capital  stock?     (N.  Y.,  June,  1919.) 

24.  When  do  dividends  on  preferred  stock  become  an  obligation  of 
the  company?     (Iowa,  Dec,  1918.) 

25.  How  would  you  classify  the  Dividend  Declared  on  Preferred  Stock 
but  not  paid  until  the  following  year,  in  the  Balance  Sheet  or  Profit  and 
Loss  account?     (Kan.,  Dec,  1915;  Mo.,  Dec,  1915;  Iowa,  Dec,  1918.*) 

26.  What  relation  do  cumulative  preferred  stock  dividends  bear  to  the 
cost  of  operating?     (Mich.,  July,  1906.) 

27.  Explain  in  some  detail  the  procedure  you  would  follow  as  an  audi- 
tor in  verifying  the  Preferred  Stock  Dividends  in  Arrears,  appearing  in 
the  books  and  accounts  of  a  company.     (Wash.,  June,  1915.) 

28.  What  do  you  understand  by  the  term  "Dividends  Paid  Out  of 
Capital"?  What,  in  your  opinion,  would  constitute  such  payment,  and 
mention  any  circumstances  that  may  occur  to  you  to  justify  such  pay- 
ment? (111.,  May,  1913;  Wis.,  April,  1915*;  Ind.,  June,  1916;  Md.,  Oct., 
1919.) 

29.  A  corporation  has  been  accused  by  a  stockholder  of  paying  divi- 
dends unlawfully.     How  should  he  proceed  in  the  matter?    You  as  an 


22()  V.  P.  A.  ACCOUNTING 

aceonntanl  have  been  called  in,     Wbat  would  you  do  (o  prove  or  disprovr 
(he  accusation?     (Del.,  June,  1915.) 

30.  The  Hayward  Company  has  declared  a  dividend  of  10  per  cent  on 
its  capital  stock  of  $100,000,  payable  July  1,  1910;  stock  books  close  on 
June  15,  1910.  Describe  the  accounting  procedure  incident  thereto  and 
state  who  may  participate  in  the  dividends.     (N.  Y.,  June,  1911.) 

31.  On  which  side  of  the  Ledger  should  the  balance  on  the  Unclaimed 
Dividends  account  appear?     (Mo.,  Dee.,  1914.) 

32.  How  would  you  deal  with  the  Unclaimed  Dividends  account  in 
preparing  the  annual  accounts  of  a  company?  Comment  briefly  on  any 
points  which  would  need  special  consideration,     (A.  I.  of  A.,  May,  1920.) 

33.  What  evidence  would  you  consider  satisfactory  for  the  correctness 
of  the  dividends?     (N.  C,  Nov.,  1919.) 

34.  How  would  you  treat  unclaimed  dividends?  (A.  I.  of  A.,  May, 
1919.) 

35.  What  is  your  method  of  checking  dividends  paid?  (R.  I.,  Dec, 
1907.) 

36.  In  case  a  corporation  of  five  persons  owning  all  the  stock  should 
credit  each  party  in  proportion  to  their  several  holdina-s  with  the  profits 
as  shown  by  the  books,  without  formally  declaring  a  dividend,  and  the  said 
stockholders  were  also  credited  with  interest  on  the  undrawn  credit  bal- 
ance, what  would  be  your  action?     (Pa.,  Nov.,  1904.) 

37.  Discuss  the  subject  of  dividends:  (a)  when  declared  out  of  pre- 
mium secured  from  sale  of  capital  stock;  (b)  when  company's  sole  invest- 
ments are  in  diminishing  (or  wasting)  assets.     (Wis.,  April,  1915.) 

38.  Can  you  mention  any  distinction  between  dividends  declared  out 
of  income  and  dividends  declared  out  of  pi'ofits  realized  from  the  incre- 
ment of  invested  values?     (A.  I.  of  A.,  May,  1919.) 

39.  How  would  excessive  dividends  affect  the  individual  holders  or 
subscribers?     (Mich.,  June,  1913.) 

40.  It  has  been  stated  that  the  right  to  declare  a  dividend  depends 
upon  the  state  of  a  company's  finances  at  the  time  when  the  dividend  is 
declared.  Give  your  opinion  as  to  conditions  under  which  a  company  may 
borrow  monev  for  the  purpose  of  paying  a  dividend.  (111.,  May,  1910; 
N.  D.,  Aug.,  1917.) 

41.  Under  the  law  permitting  payment  of  dividends  solely  out  of 
surplas  earned,  a  corporation  pays  a  dividend  out  of  general  surplus  after 
carrying  its  losses  for  the  period  against  the  account.  Would  you  make 
mention  of  the  fact  in  the  recital  accompanying  your  statements  or  Avould 
you  let  it  go  without  specific  mention?     Give  reasons.     (R.  I.,  Dec.,  1907.) 

42.  What  is  a  stock  dividend?    (Wash.,  June,  1915;  Iowa,  Dec,  1918.) 

43.  In  what  respect  do  stock  dividends  differ  from  cash  dividends? 
(N.  Y.,  Jan.,  1920.) 

44.  What  must  be  the  condition  of  the  business  to  justify  the  issue  of 
stock  dividends?     (Iowa,  Dec,  1918.) 


DIVIDENDS  AND  SURPLUS  227 

45.  In  three  successive  fiscal  years  a  niannfacturing  corporation  values 
its  supplies,  etc.,  in  hand  at  cost,  with  deductions  for  deterioration  as 
follows :  At  end  of  fir.st  year  5  per  cent ;  at  end  of  second  year  10  per 
cent,  and  at  end  of  third  year  15  per  cent.  With  the  inventory  taken  on 
this  basis  the  profits  for  the  second  year  did  not  equal  the  dividends  de- 
clared and  in  the  third  year  the  dividend  paid  was  so  much  in  excess  of 
profits  that  the  surplus  was  entirely  exhausted  and  a  debit  balance  created 
in  the  Profit  and  Loss  account.  In  auditing  the  books,  how  would  you  treat 
the  above  condition  in  your  report?     (Pa.,  May,  1905.) 

46.  Prepare  Journal  entries  for  dividend  paid  out  of  treasury  stock 
acquired  at  half  of  its  book  value.     (N.  C,  June,  1920.) 

47.  The  stockholders  of  the  Farmers  Cooperative  Store  share  the  store's 
earnings  in  proportion  to  purchases  made  during  the  year,  and  dividends 
may  be  withdrawn  in  trade  or  in  cash.  The  fiscal  year  corresponds  to  the 
calendar  year,  but  the  dividend  year  runs  from  March  15  to  March  14. 

(a)  The  following  facts  are  given  you  with  the  request  that  you  ascer- 
tain the  status  of  the  Surplus  account  on  December  31,  1917,  and  indicate 
the  disposition  of  any  dividends  which  may  have  been  paid  out  in  excess 
of  available  surplus. 

The  balance  of  the  Reserve  for  Dividends  account  on  December  31, 1916, 
was  $2,540.15.  This  represented  the  dividends  which  might  be  withdrawn 
in  cash  during  the  period,  January  1,  1917,  to  March  15,  1917. 

The  dividends  withdrawn  in  cash  during  the  year  1917  amount  to 
$1,015.37,  and  those  withdrawn  in  trade  during  1917  amount  to  $24,786.29. 
In  order  to  reduce  trade  dividends  to  a  cash  basis,  20  per  cent  is  deducted 
from  the  selling  price  of  goods  so  withdrawn  and  charged  back  against  the 
sales. 

The  balance  of  surplus  available  for  dividends  on  December  31,  1916, 
was  $20,710.43. 

An  examination  of  the  accounts  shows  that  dividends  to  the  amount  of 
$2,496.63  (cash  basis)  may  be  withdrawn  between  January  1,  1918,  and 
March  14,  1918. 

During  the  period  January  1,  1917,  to  March  14,  1917,  dividends  were 
withdrawn  to  the  amount  of  $1,720.15,  cash  basis. 

(b)  The  sales  to  members  for  the  year  1917  are  $162,280  and  the  net 
profits  are  $20,285.  In  your  judgment,  what  dividends  should  be  declared 
in  trade  and  in  cash  for  the  year  1918? 

(e)  Briefly  criticize  the  plan  followed  by  this  company  in  distributing 
the  earnings  to  the  stockholders. 

(Wis.,  April,  1918.) 

48.  A  firm  is  incorporated  under  the  laws  of  the  State  of  New  York 
to  do  business  within  the  state,  with  an  authorized  capital  of  $100,000. 
Its  assets,  including  $10,000  for  goodwill,  aggregate  $30,000;  the  lia- 
bilities other  than  capital  stock,  aggregate  $30,000.  When  the  books  were 
closed  at  the  end  of  the  first  year,  the  net  profits  shown  amounted  to 
$8,000.  Are  the  directors  warranted  in  declaring  a  dividend?  If  so,  for 
what  amount?     (N.  Y.,  June,  1917.) 


228  C.  P.  A.  ACCOUNTING 

49.  The  books  of  a  corporation  (with  a  capital  stock  of  $800,000)  at 
the  beginning  of  the  last  fiscal  year  showed  a  surplus  of  $28,450. 

During  your  examination,  immediately  subsequent  to  the  close  of  the 
fiscal  year,  you  learn  the  following  facts : 

That  the  net  profit  on  goods  delivered  to  customers  during  the  year 
amounted  to  $115,350. 

That  prior  to,  and  at  the  close  of  the  year,  the  company  owned  bona 
fide  contracts  for  the  delivery  of  goods  during  the  next  few  months. 

That  the  company  had  purchased  a  sufficient  quantity  of  merchandise 
in  order  to  fill  these  contracts. 

That  the  company  after  making  due  allowances  for  all  production  cost, 
expenses  incidental  to  the  delivery  of  the  contract  goods,  cost  of  selling,' 
etc.,  had  arrived  at  a  net  profit  amounting  to  $51,120,  which  was  carried 
to  Profit  and  Loss  account.     This  made  a  total  net  balance  of  $106,470. 

That  tliere  was  declared  and  paid  a  dividend  of  20  per  cent  (or  $160,000) 
and  that  $6,470  was  carried  to  surplus  account. 

Would  you  consider  it  necessary  to  call  particular  attention  to  this 
matter  in  your  report  to  the  stockholders?     State  your  reasons. 

(Wis.,  April,  1914.) 

50.  An  Indiana  company  is  incorporated  with  a  capital  stock  of 
$100,000,  of  which  $90,000  was  paid  in.  In  making  their  application  for 
the  charter  all  the  stock  was  subscribed.  The  full  capital,  however,  was 
not  required. 

It  was  recently  decided  that,  inasmuch  as  a  surplus  of  $15,000  had 
accumulated,  they  would  declare  a  stock  dividend,  pro  rata,  for  the  $10,000 
of  stock  unissued  but  subscribed  for  by  one  of  the  present  stockholders. 
No  action  on  the  transaction  was  taken  by  the  board  of  directors,  and  no 
record  of  it  appears  in  the  minutes  of  the  corporation. 

Is  it  necessary  that  a  transaction  of  this  character  be  recorded  in  the 
minutes,  inasmuch  as  all  of  the  stock  was  originally  subscribed? 

Will  not  this  stock  dividend  have  to  be  reported  as  income  for  the 
current  year  if  the  surplus  was  earned  since  1910? 

What  would  be  your  recommendations  in  regard  to  the  proper  procedure 
in  this  matter? 

(Ind.,  May,  1918.) 

51.  By  a  charter  provision  a  company  is  directed  annually  to  retire 
out  of  the  ordinary  profits  arising  from  the  conduct  of  the  business  5 
per  cent  of  its  7  per  cent  dividend  paying  preferred  stock.  This  pre- 
ferred stock  is  now  worth,  say,  110  in  the  market  and  during  the  past 
year  the  company  succeeded  in  buying  in  1,000  shares  of  the  par  value  of 
$100  at  $80,  borrowing  for  that  purpose  $80,000  for  four  months  at  41/2 
and  paying  the  note  at  maturity. 

(1)  What  is  the  amount  of  the  profit?  (2)  How  should  the  amount  be 
treated  in  the  company's  published  accounts?  (3)  Would  the  directors 
be  justified  in  utilizing  the  amount  for  the  payment  of  a  common  stock 
dividend,  and  should  this  be  treated  as  a  cash  dividend  or  a  repayment  of 
capital?  (Kan.,  Dec,  1915;  Mo.,  Dec,  1915.) 


dividp:nds  and  surplus  229 

52.  Under  what  circumstances  would  you  permit  a  client  to  distribute 
capital  stock  as  dividends  prior  to  dissolution?     (Cal.,  May,  1916.) 

53.  Define:  Scrip.     (N.  Y.,  Jan.,  1911.) 

54.  Explain  briefly  the  following  term:  Dividend  scrip.  (III.,  Nov.. 
1903.)  ' 

55.  May  di\idends  be  paid  in  other  than  cash?  Explain.  (Iowa.  Dec. 
1916.)  ^  V         ,         , 

56.  State  when  and  how  dividends  become  effective.  (Mich.,  July, 
1906;  Fla.,  April,  1907.) 

Surplus 

57.  Define:  Surplus.  (N.  Y.,  Dec,  1896;  N.  Y.,  Dec,  1897*;  N.  Y., 
June,  1900;  N.  Y.,  Jan.,  1901*;  N.  Y.,  Jan.,  1902*;  Pa.,  May,  1903*; 
111.,  Nov.,  1903*;  N.  J.,  1904-1909;  Ohio,  March,  1910*;  La.,  May,  1913*; 
Wash.,  Nov.,  1913*;  Wis.,  April,  1914;  N.  D.,  June,  1914;  Cal.,  Nov., 
1916;  Mass.,  Oct.,  1917*;  Va.,  Nov.,  1918;  Iowa,  Dec,  1918;  N.  C,  June, 
1920.) 

58.  Define:  Appropriated  surplus.  (Wis.,  May,  1916;  Wis.,  April, 
1917*;  Mass.,  Oct.,  1917.*) 

59.  Define:  Free  surplus.     (Wis.,  April,  1917.) 

60.  What  do  you  understand  by  capital  surplus  ?  Is  it  invested  capital  ? 
(Wis.,  Nov.,  1919.) 

61.  How  is  the  Surplus  account  used?  (N.  Y.,  Jan.,  1902*;  N.  J., 
1904-1909;  Ohio,  March,  1910.) 

62.  What  is  the  purpose  of  the  surplus  of  a  stock  company?  (N.  Y., 
Dec,  1898*;  N.  Y.,  Jan.,  1901.) 

63.  How  would  surplus  and  appropriated  surplus  affect  the  book 
value  of  capital  stock?     (Mass.,  Oct.,  1917.) 

64.  May  surplus  be  created  by  "marking  up"  the  value  of  fixed  assets? 
May  such  surplus  be  used  as  a  basis  for  declaring  dividends?  (Iowa, 
Dec,  1918.) 

65.  Mention  and  differentiate  between  accounts  properly  belonging  to 
Profit  and  Loss  and  Surplus  accounts.    (Mass.,  June,  1913.) 

66.  (o)  What  items  do  you  consider  should  be  charged  or  credited 
direct  to  surplus?  (6)  Would  you  regularly  make  small  adjustments  of 
subsequently  discovered  errors  through  this  account?  (c)  Is  the  balance  at 
credit  of  surplus  ever  in  any  circumstances  a  liability,  and  if  so,  to  whom  ? 
(A.  L  of  A.,  Nov.,  1918.) 

67.  When  preparing  a  Trading  and  Profit  and  Loss  account  at  the  end 
of  a  fiscal  year,  in  what  manner  would  you  treat  the  surplus  or  deficit 
brought  forward  from  prior  year?    (N.  Y.,  Jan.,  1907.) 

68.  Distinguish  between  reserve  and  surplus.  (Wis.,  April,  1914; 
Ohio,  Nov.,  1916.*) 


230  C.  P.  A.  ACCOUNTING 

69.  Define:  Undistributed  Profit  account.     (Iowa,  Dec,  1918.) 

70.  How  do  the  accounts  of  a  corporation  and  of  a  copartnership  differ 
in  the  statement  of  division  and  distribution  of  profits'?  (N.  Y.,  Jan., 
1904;  Va.,  Nov.,  1910*;  Cal.,  Nov.,  1916.*) 

71.  How  is  the  profit  of  the  following  concern  divided :  Stock  com- 
pany?    (Mich.,  Dec,  1906.) 

72.  Give  thi'ee  sound  accounting  principles  governing  the  determination 
of  profits  distributable  as  dividends  of  a  corporation.     (Mass.,  Oct.,  1915.) 

73.  When  is  surplus  available  for  dividends?  When  not  available? 
(Cal.,  Nov.,  1916.) 

74.  On  which  side  of  the  Ledger  should  the  balance  on  the  Undivided 
Profits  account  appear?     (Mo.,  Dec,  1914.) 

75.  A  corporation  purchased  a  business  as  a  going  concern  on  January 
1,  1908,  with  a  right  to  the  profits  from  October  1,  1907.    Its  capital  is : 

Five  per  cent  first  preferred  stock,  $250,000 ;  6  per  cent  second  preferred 
stock,  $250,000;  common  stock,  $124,000. 

The  year's  profits  to  September  30,  1908,  are  found  to  have  been 
$38,320.  What  appropriation  of  such  profits  would  you  consider  to  be 
correct  ? 

(111.,  May,  1910;  N.  C,  Nov.,  1918.*) 

76.  What  in  your  opinion  would  be  the  proper  accounting  record  for  a 
business  corporation  to  make  for  an  appropriation  from  its  surplus  profits 
for  the  account  of  a  permanent  investment  in  property?  (Md.,  Jan.,  1909.) 

77.  On  which  side  of  the  Ledger  should  the  balance  on  the  Building 
Appropriation  account  appear?     (Mc,  Dec,  1914.) 

78.  What  support  is  usually  necessary,  in  addition  to  approval  vouch- 
ers, for  the  distribution  chargeable  to  Undivided  Profits  or  Surijlus  ac- 
count?    (N.  C,  Nov.,  1918.) 

79.  If  assets  taken  over  by  a  company  are  in  excess  of  its  capital 
stock,  would  you  credit  the  excess  to  surplus?  If  not,  why  and  to  what 
account  would  yau  credit  such  excess?     (111.,  May,  1906.) 

80.  How  would  you  treat  the  deficiency  in  the  early  years  of  a  corpora- 
tion's activities?     (Ind.,  May,  1918.) 

81.  Can  surplus  be  created  in  any  way  other  than  through  profits 
earned  from  operations?     Explain.     (A.  I.  of  A.,  May,  1920.) 

82.  The  books  of  a  company,  when  closed  for  the  fiscal  year,  show  a 
substantial  net  profit.  No  dividends  were  paid  during  the  year  and  there 
is  no  cash  available  with  which  to  pay  dividends,  (a)  What  would  account 
for  this  condition?  (6)  Illustrate  your  explanation  by  a  statement  sup- 
plj'ing  the  figures.    (Mass.,  Oct.,  1916.) 

83.  Is  the  deficiency  in  the  early  j'ears  of  a  corporation's  activities 
(whether  an  actual  loss  or  a  deficiency  between  the  earnings  and  the  nor- 
mal rate  of  return)  similar  to  organization  expenses?  How  should  such 
deficiencies  be  treated  in  the  accounts?    To  what  extent  is  such  a  deficiency 


DIVIDENDS  AND  SURPLUS  231 

similar  to  interest  paid  during  construction?  Should  such  deficiencies  be 
carried  on  the  Balance  Sheet?  If  so,  should  they  be  written  off,  and 
how  and  when?  May  the  deficiencies  representing  the  difference  between 
actual  earnings  and  normal  rate  of  return  be  capitalized,  in  the  strict 
sense  of  having  capital  stock  issued  to  a  corresponding  sum?  State 
clearly  just  who  is  affected,  and  how,  by  the  different  methods  of  treating 
the  items  mentioned  above.     (A.  I.  of  A.,  June,  1917.) 

84.  How  would  you  distinguish  between:  (a)  Earned  surplus,  (b) 
paid-in  surplus,  (c)  capital  surplus,  (d)  appropriated  surplus?  (A.  I.  of 
A.,  May,  1921.) 


232  C.  P.  A.  ACCOUNTING 


PROBLEMS 


DIVIDENDS  AND  SURPLUS 


1.  A  corporation's  profits  for  the  year  ended  December  31,  1908, 
amount  to  $451,000.  The  by-laws  require  a  reserve  equal  to  10  per  cent 
of  any  dividend  paid  to  common  stockTiolders,  and  any  surplus  remaining 
after  such  dividend  has  been  paid  is  also  to  be  applied  to  the  resen'e, 
until  such  reserve  account  amounts  to  $250,000,  The  reserve  at  December 
31,  1907,  was  $156,020.  The  capital  is  $2,000,000,  one-half  cumulative 
preference  6  per  cent  and  one-half  common,  all  fully  paid.  On  December 
31,  1908,  the  preferred  dividend  is  two  and  one-half  years  in  arrear.  On 
December  31,  1907,  Profit  and  Loss  account  was  in  debit  $202,000.  Set 
out  your  treatment  of  the  profit  for  1908.  (111.,  Dec.,  1910;  Colo., 
Dec,  1913.) 

2.  The  profits  of  a  corporation  with  a  paid-up  capital  of  $5,000,000 
amount  to  $337,193.08  for  a  given  year,  without  allowing  for  its  mort- 
gage interest.  At  the  end  of  the  previous  financial  year,  there  was  left  a 
balance  of  undivided  profits  of  $27,806.92. 

Its  4  per  cent  mortgages  are  $500,000  and  its  6  per  cent  mortgages  are 
$750,000.  How  much  must  be  taken  from  the  pre\'ious  year's  surplus 
balance  to  pay  the  stockholders  a  dividend  of  6  per  cent? 

(111.,  May,  1911.) 

3.  Frame  any  entries  necessary  to  record  the  action  of  the  directors 
as  it  appears  in  the  minutes  of  the  meeting. 

The  treasurer  reported  that  the  profits  for  the  year  as  audited  amounted 
to  $59,287.  Voted  that  a  dividend  of  $40,000  be  paid  on  October  1  to  the 
stockholders  of  record  September  15  and  that  $10,000  of  the  profits 
be  appropriated  as  a  reserve  for  relief  of  employees  disabled  while  in 
the  service  of  the  United  States  and  invested  in  Libertv  Bonds. 

(A.  I.  of  A.,  Nov.,  1917.) 

4.  Before  making  the  charges  referred  to  below,  the  Profit  and  Loss 
account  of  a  corporation  for  the  year  shows  a  credit  balance  of  $60,000. 
The  accounts  receivable  are  $40,700  and  the  Plant  and  Machinery  account 
is  $55,000.  The  6  per  cent  preferred  stock  is  $50,000,  and  the  common 
stock  $150,000.  It  is  decided  (a)  to  provide  out  of  the  above-named 
Profit  and  Loss  balance  7y2  per  cent  depreciation  on  plant  and  machinery; 
(b)  to  write  off  as  uncollectible  $1,500  of  the  accounts  receivable  and  to 
make  a  reserve  of  2  per  cent  on  the  remainder  of  the  accounts  receivable 
to  provide  for  possible  losses  thereon;  (c)  to  provide  for  the  preferred 
gtock  dividend  for  the  year;  (d)  to  provide  for  a  bonus  of  $7,500  to  the 


DIVIDENDS  AND  SURPLUS  233 

employees;  (e)  to  provide  for  a  dividend  on  the  common  stock  of  15  per 
cent  for  the  year,  and  (/)  to  carry  the  balance  then  remaining  on  Profit 
and  Loss  account  to  Undivided  Profits  account. 
Draft  entries  to  comply  with  the  above  provisions. 

(Wash.,  April,  1906;  Mo.,  Dec.,  1913.) 

5.  In  an  audit  of  the  Acme  Motor  Car  Company  you  find  the  Reserve 
for  Depreciation  account  and  the  Surplus  account  (jpmposed  of  the  items 
as  here  enumerated : 

The  Reserve  for  Depreciation  account  was  opened  on  December  31, 1915, 
the  close  of  the  first  business  year,  by  debiting  the  depreciation  accounts  of 
the  various  assets  with  $265,000. 

The  Reserve  for  Depreciation  account  was  also  credited  with  $25,000 
on  December  31,  1916,  and  with  $20,000  on  December  31,  1917. 

During  1916  and  1917  the  following  items  have  been  charged  against 
this  Reserve  for  Depreciation  account:  assets  scrapped,  $125,000;  bad 
debts,  $25,000;  repairs,  $10,000;  fire  loss  on  building  and  equipment, 
$7,500;  organization  expense,  $65,000;  salesmen's  extra  commission, 
$12,000. 

The  Surplus  account  for  1915  and  1916  has  been  closed,  the  balance 
having  been  paid  out  in  dividends. 

The  Surplus  account  on  December  31,  1917,  is  found  to  consist  of  the 
following  credit  items:  reserve  for  car  guarantees,  $50,000;  premium  on 
stock  sold,  $50,000;  reserve  for  obsolescence,  $50,000;  bonus  from  Com- 
mercial Club,  $50,000;  reserve  for  income  and  excess  profits  taxes,  1917, 
$80,000;  operating  profits,  $750,000. 

You  are  requested  to  make  such  adjustments  in  the  Reserve  for  Depre- 
ciation accoimt  and  Surplus  account  as  are  appropriate,  and  to  show  how 
the  several  items  and  accounts  should  appear  in  the  financial  statement. 

(Wis.,  April,  1918.) 

6.  The  books  of  the  "X"  Manufacturing  Company  were  audited  to  De- 
cember 31,  1913,  and  in  making  up  the  Balance  Sheet  and  Profit  and  Loss 
account  at  that  date  the  auditors  recommended  the  following  adjustments: 
(a)  Transferred  to  profit  and  loss,  $4,231.07,  which  had  been  charged  to 
real  estate  and  buildings  in  error;  (6)  provided  for  depreciation  of  build- 
ings, etc.,  $7,200;  (c)  adjusted  salaries  amounting  to  $1,400,  due  for  1913 
services  but  not  entered  on  the  books  until  January,  1914;  (d)  reduced 
tlio  amount  of  inventory  because  of  errors,  $12,000. 

The  same  auditors  were  again  called  in  to  audit  the  books  to  June  30, 
1914,  and  found  that  the  above  adjustments  had  not  been  entered  on  the 
books.  They  also  found  that  during  the  half-year  $1,000  had  been  charged 
to  real  estate,  buildings,  etc.,  instead  of  to  expense ;  that  no  provision  had 
been  made  for  depreciation  for  the  period,  amounting  to  $3,600 ;  and  that 
the  inventory  had  been  footed  $10,000  too  much.  Also  that  the  unexpired 
insurance  amounted  to  $750  more  than  was  entered  on  the  books.  The 
following  are  condensed  trial  balances  of  the  "X"  Manufacturing  Com- 
pany's books  as  the  auditor  found  them  as  of  December  31,  1913,  aud 
June  30,  1914: 


234 


C.  r.  A.  ACCOUNTING 


Real  Estate,  Buildings,  Etc. 

Capital  Stock 

Debentures 

Cash 

Accounts  Payable 

Accounts  Receivable 

Loans 

Stocks  and  Bonds 

Inventory 

Unexpired  Insurance 

Surplus 

Profit  and  Loss,  1914 


Dec.  31,  1913 


June  30,  1914 


$102,840.26 

14,672.14 

22,436.10 

17,502.50 

246,153.42 

1,471.23 


$405,075.65 


$200,000.00 
100,000.00 

9,431.17 

10,000.00 

85,644.48 


$405,075.65 


$115,226.80 

22,143.21 

28,250.40 

19,150.00 

288,360.14 

742.26 


$200,000.00 
100,000.00 

11,698.21 

5,000.00 


85,644.48 
71,530.12 


$473,872.81   $473,872.81 


From  the  above  facts  prepare:  (1)  A  correct  Balance  Sheet,  June  30, 
1914;  (2)  state  the  adjusted  ambunt  of  profits  for  the  half-year  to  June 
30,  1914;  (3)  prepare  statement  reconciling  the  Balance  Sheet  figures 
with  the  original  trial  balance  of  June  30,  1914. 

(Mass.,  Oct.,  1914.) 

7.  On  January  1,  1919,  the  close  of  its  third  year's  business,  the  fol- 
lowing accounts  were  open  upon  the  General  Ledger  of  the  Winner  Manu- 
facturing Company: 


Preferred  Capital  Stock 

Common  Capital  Stock 

Cash  on  Deposit 

Imprest  Cash  Fund 

Real  Estate 

Buildings 

Notes  Receivable 

Factory  Equipment 

Accounts  Payable 

Notes  Payable 

Reserve  for  Depreciation,  Buildings 

Reserve  for  Depreciation,  Factory  Equipment 

Accounts  Receivable 

Patents 

Patterns 

Auto  Trucks 

Bonds  Issued 

Premium  on  Bonds  Issued 

Inventory  Raw  Material 

Inventory  Finished  Goods 

Inventory  Goods  in  Process 

Reserve  for  Depreciation,  Auto  Trucks 

Surplus,  January  1,  1918 

1918  Operating  Profit  and  Loss 


$50,000 
500 
250,000 
300,000 
8,000 
450,000 


75,000 

1 

25,000 

10,000 


180,000 
40,000 
70,000 


$376,000 
600,000 


53,000 

25,000 

6,000 

75,000 


200,000 
2,000 


4,000 
50,000 
67,501 


$1,458,501        $1,458,501 


DIVIDENDS  AND  SUKPLl'S  235 

The  preferred  capital  stock  was  $400,000  7  per  cent  cumulative,  and 
the  provisions  of  its  issue  require  that  3  per  cent  of  the  authorized 
amount  be  set  aside  annually  as  a  sinking  fund  for  its  redemption  at 
$125. 

The  common  capital  stock  of  the  company  is  without  par  value;  20,000 
shares  have  been  authorized ;  12,000  shares  issued. 

The  Real  Estate  account  is  found  to  consist  of  the  following  items: 

Factory    Real    Estate    $  20,000 

Fertile   Farms   Investment    120  000 

City   Real   Estate    Investment    110,000 

The  bonds  of  the  company  are  20-year,  7  per  cent  gold  bonds,  sold  on 
September  30,  1918,  for  101.  Interest  is  payable  October  1  and  April  1. 
The  bond  recital  provides  for  the  creation  of  a  pro  rata  sinking  fund  to  be 
reserved  out  of  the  profits  of  each  year  and  for  the  setting  aside  of  cash 
equivalent  to  such  resei-vation. 

The  income  taxes  for  the  year  are  estimated  at  $5,000. 

You  leam  that  the  directors  met  on  Januai-y  10,  1919,  and  declared  a 
dividend  of  7  per  cent  upon  the  preferred  stock,  authorized  the  purchase 
of  shares  of  preferred  stock  in  accordance  with  the  terms  of  issue  and 
declared  a  dividend  of  five  dollars  ($5)  per  share  of  common  stock.  The 
dividends  were  paid  on  January  15,  1919,  and  the  preferred  stock  was 
purchased  on  that  date. 

In  view  of  the  above  conditions,  you  are  asked  to  prepare  a  financial 
statement  of  the  Winner  Manufacturing  Company  as  of  January  1,  1919, 
after  the  books  for  the  year  have  been  closed  finally. 

(Wis.,  Nov.,  1919.) 

8.    At  the  close  of  its  fiscal  year,  December  31,  1915,  the  trial  balance 
of  the  Nau-Pace  Company  was  as  given  on  next  page. 
You  are  to  take  into  consideration  the  following  facts : 

1.  Real  Estate,  Machinery  and  other  Factory  Equipment,  and  Patents 

are  stated  at  cost. 

2.  Of  the  Real  Estate,  $25,000  is  for  land,  and  $200,000  is  for  buildings. 

3.  All  capital  stock  authorized  has  been  issued  and  is  outstanding. 

4.  Allowances  for  depreciation  are:  machinery  and  factory  equipment, 

$15,000;  buildings,  3  per  cent  on  cost;  patents,  1/17  of  cost. 

5.  $15,000  is  to  be  set  aside  as  a  reserve  for  bad  accounts. 

6.  Ten  per  cent  of  the  book  values  of  Stable  Equipment  and  Office 

Equipment  and  one-sixth  of  the  book  value  of  Drawings  and 
Patterns  are  to  be  charged  off. 

7.  Inventories  at  the  close  of  the  year  were:  raw  materials,  $63,580.40; 

finished  goods,  $58,864.56;  goods  in  process,  $27,024.52;  fuel, 
$4,823.43;  factory  supplies,  $1,525;  office  supplies,  $500;  prepaid 
insurance,  $500. 

8.  The  accruals  are:  taxes,  $7,000;  Direct  Labor,  $12,618.75;  Indirect 

Labor,  $2,040.50;  Interest  on  Bonds,  $1,000;  Advertising, 
$4,718.50. 


236  C.  P.  A.  ACCOUNTING 

Trial  Balance,  Nau-Pace  Company,  December  31,  1915 


Real  Estate 

Fixed  Machinery 

Movable  Equipment 

Shaftings,  Pulleys,  etc 

Stable  Equipment 

Office  Equipment 

Drawings  and  Patterns 

Patents 

Capital  Stock 

First  Mortgage  Bonds 

Profit  and  Loss 

Surplus 

Dividends 

Interest  on  Bonds 

Other  Interest  Paid 

Interest  Received 

Cash  Discount  on  Purchases 

Cash  Discount  on  Sales 

Sales 

Return  Sales - 

Cash 

Bills  Receivable 

Accounts  Receivable 

Raw  Materials 

Finished  Goods,  January  1,  1915 

Goods  in  Process,  January  1,  1915. . . 

Fuel 

Insurance 

Taxes 

Bills  Payable 

Accounts  Payable 

Reserve  for  Depreciation : 

Machinery  and  Equipment 

Buildings 

Patents 

Bad  Accounts 

Salaries,  Offices  and  Clerks  (General) 

General  Office  Supplies 

Postage,  Telegraph,  and  Phone 

Miscellaneous  General  Expenses 

Advertising 

Salaries  and  Expenses,  Salesmen 

Agents'  Commissions 

Credit  Department  Salaries 

Miscellaneous  Expenses,  Selling 

Stable  Expenses 

Direct  Labor,  Manufacturing  ....... 

Indirect  Labor,  Manufacturing 

Superintendence,  Factory 

Factory  Supplies 

Repairs,  Machinery  and  Equipment . 

Repairs  of  Buildings 

Power,  Heat,  and  Light 


$225,000.00 

150,000.00 

18,000.00 

10,500.00 

3,500.00 

2,915.90 

9,000.00 

75.000.00 


5,000.00 
1,323.10 


2.861.50 

8,258.25 

27,750.65 

50.750.00 

298;650.25 

622,190.90 

62.735.06 

24.747.27 

38.688.28 

4,000.00 

5,000.00 


56,150.00 

2.950.75 

1,560.00 

850.00 

35,000.00 

72,350.31 

30.141.40 

7,560.00 

610.00 

3,963.46 

508,311 .39 

44.981.01 

6,000.00 

8,547 . 18 

7,418.52 

2,860.47 

2.875.80 


$500,000.00 
100,000.00 

86,140.28 
300.00 


2,469.50 
13,389.52 

1,540.816.75 


40,000.00 
46,585.85 

50,000.00 

30,000.00 

22,058.80 

6,240.75 


$2,438,001 .45       $2,438,001 .45 


DIVIDENDS  AND  SURPLUS  237 

9.  The  depreciation  on  stable  equipment  (see  item  6)  is  to  be  charged 
to  Stable  Expenses,  and  one-third  of  the  latter  is  apportioned  to 
ManufacturiiiiT    Expenses,  and  two-thirds  to  Selling  Expenses. 

10.  The  cost  of  fuel  used  is  to  be  charged  to  Power,  Heat  and  Light. 

11.  Maintenance  of  Real  Estate  is  to  be  charged  with  cost  of  repairs 

to  buildings,  depreciation  on  building's,  20  per  cent  of  taxes  for 
the  year,  and  $1,000  for  insurance.  The  total  cost  for  such  main- 
tenance is  to  be  shown  as  an  item  of  manufacturing  expense  on 
the  Statement  of  Cost  of  Sales. 

12.  The  portion  of  insurance  remaining  after  charging  Maintenance  of 

Real  Estate  is  to  be  allocated  to  Manufacturing  Expenses. 

13.  Thirty  per  cent  of  the  taxes  for  the  year  is  to  be  apportioned  to 

Manufacturing  Expenses,  and  50  per  cent  is  to  be  charged  against 
Income. 

14.  Of  the  Salaries  of  Officers  and  Clerks,  General,  $3,600  should  be 

apportioned  to  selling  expenses. 

15.  Amongst  the  Bills  Receivable  is  a  note  for  $5,000,  pertaining  to  a 

previous  fiscal  year,  which  is  considered  to  be  worthless.  No 
provision  was  made  for  such  loss. 

16.  Regardless  of  theory,  cash  discounts  on  purchases  and  sales  are  to 

be  treated  as  pertaining  to  income. 

17.  On  the  10th  of  December,  1915,  a  dividend  of  10  per  cent  on  the 

capital  stock  was  declared  and  made  payable  on  January  10, 
1916,  for  which  no  entry  was  made  prior  to  taking  off  the  trial 
balance. 
Given  the  foregoing  information,  you  are  asked  to  prepare  the  following 
statements  in  approved  form  for  the  information  of  your  clients: 

1.  Cost  of  Sales. 

2.  Profit  and  Loss,  showing  (a)  the  gross  profit  and  the  per  cent  of 

same  on  sales;  (6)  selling  expenses  and  per  cent  of  same  on  gross 
profits;  (c)  general  expenses  and  the  percentage  that  such  ex- 
penses bear  to  gross  profits;  and  (d)  the  net  profits  and  the  per 
cent  of  same  on  sales. 

3.  Balance  Sheet,  showing  the  surplus  at  the  beginning  of  the  fiscal 

year,  and  the  amount  at  the  close  of  the  year. 

(Ohio,  Nov.,  1916.) 


CHAPTER  XIII 


TRADING  ACCOUNTS 


Old  Merchandise  Account — The  old  merchandise  account  was  an  ac- 
count in  which  were  gathered  all  the  factors  affecting  the  gi-oss  profit  on 
sales/    The  structure  of  the  merchandise  account  was  as  follows:' 


Debits 

Credits 

1. 

Initial  inventory. 

1. 

Sales. 

2. 

Purchases  of  the  period. 

2. 

Returned  purchases. 

3. 

Freight  and  cartage  inward  on  pur- 

3. 

Freight  and  cartage  inward,  paid  at 

chases. 

time  of  purchase,   on  purchases 

4. 

Freight   and   cartage   outward   on 

now  returned. 

sales. 

4. 

Trade  discounts  on  purchases. 

5. 

Returned  sales. 

5. 

Merchandise  destroyed  as  unfit  for 

6. 

Freight  and  cartage  inward  on  re- 

sale. 

turned  sales. 

6. 

Deterioration  of  merchandise. 

7. 

Trade  discounts  on  sales. 

7. 

Freight  and  cartage  inward  on  pur- 

8. 

Gross  profit  on  merchandise. 

chase  applicable  to  goods  unsold. 

8. 

Closing  inventory. 

In  objecting  to  the  use  of  the  old  merchandise  account  great  stress  is 
frequently  made  about  its  being  a  mixed  account.'  As  the  inventory 
account,  one  of  the  accounts  which  replaces  the  merchandise  account,  is  a 
mixed  account  as  soon  as  a  sale  has  been  made,  this  objection  does  not  seem 
vital.  The  real  objection  is  that  the  old  merchandise  account  makes  it 
impossible,  without  a  laborious  analysis,  to  give  the  executives  statistics 
which  will  show  how  business  is  progressing.*  Modern  accountants,  there- 
fore, use  a  separate  account  for  each  of  the  factors  listed  above,  although 
some  of  the  closely-related  factors  are  sometimes  combined  for  convenience.* 

Purchase  Records — When  the  storeskeeper  discovers  a  shortage  of  a  cer- 
tain material,  he  sends  a  requisition  for  an  order  to  be  placed  by  the  pur- 
chasing department,  and  files  a  carbon  of  the  requisition.'  The  purchasing 
agent  sends  the  order  to  the  jobber  and  a  carbon  of  the  order  (details  not 
usually  shown  on  this  copy)'  to  the  receiving  department  and  to  the  stores- 
keeper;  he  then  files  a  copy,  together  with  the  original  requisition,  in  his 
own  files."  When  the  jobber  ships  the  material,  he  submits  an  invoice 
which  the  purchasing  agent  files  with  the  order.'  On  receipt  of  material 
the  receiving  clerk  makes  out  a  report  of  material  received,  two  copies 
going  with  the  material  to  the  storesroom  and  the  third  copy  being  placed 
in  the  receiving  clerk's  files  with  the  carbon  of  the  original  order."  After 
inspection,  a  copy  of  the  report  of  material  received,  properly  signed,  is 
sent  by  the  storekeeper  to  the  purchasing  agent,  who  compares  it  with 


'For  explanation  of  superior  figures  see  page  337. 

238 


TRADING  ACCOUNTS  230 

the  order  and  invoice,  and  then  files  tlie  order  and  report  of  material  re- 
ceived and  sends  the  invoice,  properly  endoi'sed,  to  the  disbursing  officer, 
who  pays  it  with  a  voucher  check."  Of  course,  this  procedure  varies 
gi-eatly  in  difi'erent  concerns,  but  these  records,  or  substitutes  therefor,  are 
the  vouchers  the  auditor  uses  to  verify  the  purchases."  The  auditor  should 
never  attempt  to  verify  all  of  the  purchase  vouchers  but  should  only  make 
copious  checks.*^  Some  auditors  <'heck  each  invoice  against  the  purchase 
journal  for  only  three  or  four  months  out  of  the  twelve  months,  always 
including  the  last  month  of  the  period,"  while  others  check  them  all."  In 
either  case,  purchases  recorded  during  the  first  few  weeks  after  the  date 
of  closing  should  be  scrutinized  to  prevent  the  goods  from  being  included 
in  the  inventory  without  having  been  recorded  in  the  purchase  journal." 

The  form  of  the  purchase  book  or  register  depends  on  whether  the  busi- 
ness is  departmentalized,  as  separate  columns  should  be  assigned  to  each 
department.  Again,  purchases  may  be  classified  as  to  the  kind  of  ma- 
terial, viz.,  woolens,  and  trimmings,  in  the  clothing  industry.  However,  the 
accounting  procedure  is  the  same  whether  there  is  only  one  purchase 
account  or  whether  there  are  a  dozen.  The  auditor  must  check  the  dis- 
tribution of  the  charges,"  but  the  checking  of  the  distribution  of  about 
every  fifth  voucher  will  suffice,"  if  vouchers  for  substantial  sums  are 
chosen. 

In  a  fairly  large  concern,  the  purchase  journal  footings  of  about  every 
tenth  or  twelfth  page  in  addition  to  the  last  page  of  each  month  should 
be  verified,  but,  in  a  smaller  concern,  proving  every  fifth  or  sixth  page, 
including  always  the  last  page  of  each  month,  will  suffice." 

The  posting  of  the  totals  of  all  the  columns  in  the  purchase  book  must 
be  verified,^"  but  the  checking  of  the  credit  postings  can  usually  be  omitted" 
(see  audit  of  accounts  payable  in  Chapter  III,  Volume  II). 

Unrecorded  Purchase  Orders — Some  accountants  believe  that  small  un- 
recorded orders  for  merchandise,  w^hether  unshipped,  in  transit,"*  or  de- 
livered at  or  just  before  inventory  time,"  may  be  ignored  on  fmancial 
statements,  unless  there  is  a  wide  difference  between  the  contract  and 
market  price,  although  important  orders  must  be  recorded.  Other  ac- 
countants add  all  unrecorded  purchase  orders  to  the  inventory  and  to  the 
accounts  payable."  As  the  firm  under  audit  is  liable  for  the  purchase 
price  of  the  material,  the  second  view  seems  preferable. 

Sales  Egcords— Systematic  record  should  be  kept  of  all  sales  orders, 
which  should  be  checked  with  the  invoices  before  the  latter  are  sent  out." 
The  shipping  clerk  should  keep  an  independent  record  of  shipments. 
When  practicable,  the  sales  book  should  consist  of  the  first  carbon  of  the 
invoices,  as  this  makes  the  sales  book  admissible  in  court  as  an  exact  copy 
of  the  invoices.'"  The  auditor  should  vouch  the  sales  book  with  the  original 
orders  of  sales  and  the  original  record  of  shipments."  It  will  suffice,  how- 
ever, if  the  sales  for  three  or  four  months  scattered  over  the  year,  but 
always  including  the  last  month,  be  vouched  in  detail. 

Where  the  business  is  departmentalized,  an  elaborate  sales  journal  with 
columns  for  each  department  may  be  kept.'*    These  columnar  sales  journals 


240  C.  P.  A.  ACCOUNTING 

should  be  audited  as  to  the  total  sales  column  in  the  same  manner  as  the 
simpler  forms  and  the  distribution  of  the  sales  should  be  only  test-checked. 
The  monthly  or  other  aggregates  of  the  sales  postings  should  be  checked, 
but  the  individual  debit  postings  need  not  be  verified^  (see  audit  of  ac- 
counts receivable  in  Chapter  III,  Volume  II).  As  to  sales  journal  foot- 
ings, every  eighth  page  in  large  concerns  and  every  third  or  fourth  page 
in  small  concerns  should  be  verified,  but  in  every  case  the  last  and  some- 
times the  next  to  last  page  of  each  month  should  be  checked.^ 

Cash  Sales — In  regard  to  cash  sales,  the  auditor  should  first  examine  the 
system  of  internal  check;  any  weaknesses  discovered  should  be  mentioned 
in  the  report  and  should  be  thoroughly  tested  during  the  audit.  Where  the 
system  of  internal  check  is  good,  the  auditor  need  test  only  the  sum- 
maries of  the  cash  sales  as  posted  in  the  general  books,  but,  where  the 
system  is  not  extensive,  he  must  make  thorough  tests  of  the  cash  sales." 

C.  0.  D.  Sales — "When  merchandise  is  sold  on  a  C.  0.  D.  basis,  a  credit 
is  made  to  the  sales  account  and  a  debit  to  the  C.  0.  D.  account.''  When 
collections  are  made,  cash  account  is  debited  and  C.  0.  D.  account  credited. 
The  balance  of  the  C.  0.  D.  account  represents  undelivered  sales,"  and  the 
auditor  must  verify  this  balance."  The  C.  0.  D.  department  charges  each 
route  for  the  new  C.  0.  D.  sales  and  credits  it  with  the  cash  returned  and 
goods  returned  into  stock  as  refused,  and  prepai'es  for  the  driver  a  route 
sheet  which  shows  the  held-overs  from  the  previous  day,  the  new  C.  0.  D. 
sales,  and  the  i-eturns  made.^°  Some  accountants  make  only  memorandum 
entries  until  collection  has  been  made  for  C.  0.  D.  sales.'* 

Installment  Sales — Theoretically  the  profit  and  loss  account  should 
be  credited  with  the  profit  on  installment  sales  and  debited  with  the 
estimated  bad  debts  and  estimated  collection  expenses,  but  the  calculation 
of  the  necessary  reserves  is  too  difficult  for  practical  purposes.'^ 

Assume  that  an  installment  house,  whose  gross  profit  percentage  based 
on  sales  was  100  per  cent,  sold  an  article  for  $60,  terms  $10  down  and 
$5  a  month.  If  the  books  were  closed  for  the  end  of  the  fiscal  period 
after  the  first  monthly  pajanent,  the  entries,  according  to  the  procedure 
approved  by  the  income  tax  authorities,  would  be  :^ 

Installment  Sales  Contracts  (1920) $60. 00 

Goods  Sold $30.00 

Unrealized  Gross  Profit  on  Installment  Sales 

Contracts  (1920) 30. 00 

(For  sale) 

Cash. 15.00 

Unrealized  Gross  Profit  on  Installment  Sales  Con- 
tracts (1920)  7  50 
Installment  Sales  Contracts  (1920) ............  15. 00 

Realized    Gross    Profit    on    Installment    Sales 

Contracts 7. 50 

(For  deposit  and  first  monthly  payment.) 

Realized  Gross  Profit  on  Installment  Sales  Con- 
tracts           7 .  50 

Profit  and  Loss 7. 50 

(For  closing  books) 


TRADING  ACCOUNTS  241. 

Sales  for  Future  Delivery — Under  ordinary  conditions  sales  for  future 
delivery  are  not  booked  until  delivery/'  and  profit  thereon  should  not  be 
taken/"  If  a  valid  contract  exists,  upon  which  a  customer  can  be  sued, 
the  profit  may  be  taken  if  the  goods  ai-e  ready  for  shipment,  but  the  profit 
should  not  be  taken  if  the  goods  are  unmade."  Selling  expense  incurred  on 
undelivered  sales  is  a  deferred  charge  to  income.*^ 

Goods  for  current  shipment  delayed  in  delivery  should  be  added  to  the 
accounts  receivable  and  sales  accounts,  and  profit  thereon  taken/' 

Sales  to  Branches — Shipments  to  branches  should  be  credited  to  a  branch 
shipment  account  to  differentiate  them  from  the  real  sales/*  The  branch 
shipment  account  in  the  head  office's  books  is  offset  by  the  "purchases  from 
head  office"  account  in  the  branch  office's  books,  when  the  accounts  of  the 
main  office  and  the  branch  are  consolidated  at  the  end  of  the  period/"  If 
the  goods  have  been  billed  to  the  branch  at  a  figure  other  than  cost,  an 
adjustment  entry  must  be  made  at  the  end  of  the  period  to  bring  the 
branch  inventory  to  a  cost  basis/" 

Consignment  Sales — As  shipments  to  commission  merchants  are  still 
owned  by  the  consignors,  they  must  be  differentiated  from  the  real  sales/' 

The  accounting  for  consignments  is  treated  in  Chapter  XV,  Volume  II. 

Sales  of  Scrap  or  By-Products — Sales  of  scrap  material  or  by-products 
are  credited  to  a  special  account  instead  of  to  the  regular  sales  account." 

The  accounting  for  sci*ap  material  and  by-products  is  treated  in  Chapter 
XVI,  Volume  I. 

Sales  to  Proprietor — Theoretically,  sales  to  the  proprietor  should  be 
charged  at  cost  to  the  proprietor's  personal  account  and  credited  to  the 
purchase  account."  If,  for  convenience,  they  aie  recorded  in  the  sales 
journal,  a  correcting  entry  in  the  general  journal,  debiting  the  proprietor's 
personal  account  and  crediting  the  accounts  receivable,  must  be  made  for 
the  aggregate  of  such  transactions.'^  As  these  transactions  are  too  small 
to  vitiate  percentages  based  on  sales,  the  transferal  of  the  credit  from  the 
sales  account  to  the  purchase  account  may  be  omitted." 

Sales  on  Approval — As  goods  sent  out  on  approval  are  not  sold  until 
they  have  been  accepted,  they  must  not  be  entered  directly  into  the  sales 
account.""  The  prevailing  practice  is  to  charge  approval  accounts  receiv- 
able at  selling  price  and  credit  approval  sales,  reversing  the  entry  if  the 
goods  are  returned."  If  the  sale  is  consummated,  it  is  recorded  as  a 
regular  cash  or  credit  sale,"  and  the  original  entry  to  approval  accounts 
receivable  and  approval  sales  is  reversed.  For  balance  sheet  purposes,  all 
goods  out  on  approval  should  be  shown  in  the  inventory."  On  account  of 
the  risk  involved,  a  very  liberal  reserve  for  losses  should  be  set  off  against 
this  part  of  the  inventory. 

Sales  of  Fixed  ^sse^s— Sales  of  fixed  assets  should  not  b^  credited  to 
the  sales  account.'"  The  special  features  involved  in  such  sales  are  treated 
in  Chapter  IV,  Volume  II. 

The  Turnover — Turnover  is  sometimes  defined  as  the  prime  cost  of 
sales."    This  use  of  the  term  is  unfortunate,  as  it  is  frequently  confused 


242  C.  P.  A.  ACCOUNTING 

with  the  rate  of  turnover,  or  the  number  of  times  tlie  stock  has  been 
turned  over  in  tlie  period."*  The  rate  of  turnover  is  the  quotient  of  the 
cost  of  sales  divided  by  the  initial  (if  normal)  inventory."' 

Merchandise  Expenses — The  merchandise  expenses  which  affect  the  cost 
of  purchases  are  freight  in;'"  cartage  in;"'  warehouse  rent,  wages,  and 
supplies;*'  insurance  during  transit  and  storage;"'  duty;"  seasoning  or 
aging  costs  ;'^  and  buying  expense."  Sometimes  warehouse  expenses  are 
classified  as  selling  expense,'''  but  it  seems  preferable  to  prorate  them 
betw^een  the  purchases  and  sales.***  Sometimes  buying  expense  is  shown  on 
the  profit  and  loss  statement  as  a  separate  kind  of  operating  expense," 
but,  where  practicable,  it  seems  preferable  to  have  the  buying  expense 
affect  the  cost  of  purchases.  These  costs  are  recorded  separately,  and 
added  to  the  invoice  price  of  the  stock-in-trade  at  the  end  of  the  period.'* 

Freight  outward  and  cartage  outward  are  considered  as  deductions  from 
sales  by  some  accountants,"  and  as  selling  expenses  by  others.''  If  the 
firm  ordinarily  sells  goods  F.  0.  B.  destination,  such  fi-eight  and  cartage 
outward  should  be  deducted  from  sales,  but,  if  this  is  not  the  custom, 
these  outlays  should  be  treated  as  selling  expenses."  The  cost  of  delivering 
goods  to  local  customers  should  be  regarded  as  a  selling  expense.'*  Ship- 
ping expenses,  salesmen's  salaries,  traveling  expenses,  commissions,  and 
advertising  are  selling  expenses." 

The  auditor,  in  examining  the  contracts  with  the  salesmen  to  verify  the 
payments  made  to  them,  should  pay  special  attention  to  the  bases  on 
which  commissions  should  be  calculated  and  to  any  limits  that  may  have 
been  placed  on  traveling  expenses.'"  Advertising  in  current  new^spapers 
and  magazines  should  be  closed  into  profit  and  loss,"  but  ispecial  advertis- 
ing campaigns  entailing  heavy  outlays,  the  results  of  which  can  not 
possibly  become  evident  until  some  f utm-e  date,  may  be  treated  as  deferred 
assets  and  written  off  as  soon  as  pi-acticable." 

Although  sometimes  classified  as  selling  expense,'*  credit  department 
expense  appears  to  be  a  general  administrative  expense.*" 

Returns  on  Sales  and  Purchases — Returned  sales  and  returned  pur- 
chases are  deductions  from  the  sales  and  purchases  accounts,  respectively." 
For  the  recording  of  these  transactions,  separate  journals  called  returned 
sales  books  and  returned  purchases  books,  respectively,  should  be  used 
as  posting  media.*^ 

In  auditing  the  returned  sales  book  the  auditor  should  prove  the  foot- 
ings, check  enti'ies  against  approved  credit  slips  and  stock  records  for 
returned  merchandise,  and  check  the  postings  to  the  controlling  accounts." 
The  footings  in  the  returned  purchases  book  must  be  checked,**  the  entries 
vouched  with  the  stock  records,  shipment  records,  and  credit  memos  from 
the  vendors,"  and  the  postings  to  the  controlling  accounts  verified. 

Rebates  and  Allotcances — Rebates  are  of  much  the  same  nature  as 
allowances  and  have  the  same  effect  as  special  allowances.*'  The  allowances 
on  sales  and  on  purchases  should  be  separated  from  the  returns  on  sales 
and  on  purchases,  respectively."'     It  is  sometimes  claimed   that  rebates 


TRADING  ACCOUNTS  243 

aiul  allowances  on  sales  are  deductions  from  sales  when  they  are  beyond 
the  control  of  the  sales  department,  and  selling  expenses  when  they  are 
controlled.'"  Some  accountants  treat  allowances  to  customers  as  a  financial 
expense,™  Avhile  other  accountants  always  regard  them  as  deductions  from 
sales.""  The  latter  view  appeal's  prefei-able,  as  transactions  occurring  after 
the  sales  have  been  consummated  can  hardly  create  selling  or  financial 
expenses.  For  administrative  purposes  it  is  sometimes  advisable  to 
classify  the  allowances  on  sales  according  to  the  reason  therefor,  viz., 
defective  goods,  breakage,  damage,  loss,  etc."  Rebates  and  allowances  on 
purchases  should  be  deducted  from  the  cost  of  the  purchases." 

The  procedure  in  auditing  the  sales  allowances  and  purchases  allowances 
should  be  to  check  them  with  the  credit  memoranda."  An  auditor  has 
little  to  fear  in  regard  to  purchases  allowances.  He  must,  however,  take 
unusual  care  in  auditing  the  sales  allowances,  as  they  are  frequently  used 
as  a  cover  for  fraud.** 

Guaranteed  Sales — Guarantees  of  goods  are  contingent  liabilities,  and 
an  estimated  amount  based  upon  past  experience  should  be  charged  to  a 
suitable  expense  account  and  credited  to  an  appi'opriate  liability  reserve 
therefor.*" 

The  treatment  of  contingent  liabilities  is  discussed  in  Chapter  III. 

Sales  in  Transit — If  goods  are  lost  in  transit  and  another  shipment  made, 
the  railroad  company  would  be  debited  and  a  "lost  in  transit"  account 
credited.*"  When  the  damages  are  received,  cash  is  debited,  the  railroad 
company  credited,*'  and  the  "lost  in  transit"  account  closed  into  sales.  If 
a  financial  statement  is  prepared  prior  to  settlement  of  the  damages,  the 
goods  lost  in  transit  would  appear  at  cost  in  the  inventory,  as,  if  located, 
they  can  be  returned  by  the  railroad  company. 

Book  and  Physical  Inventories — Inventories  may  be  defined  as  itemized 
lists  of  goods  or  valuables  with  prices  attached.*'  If  the  lists  are  prepared 
after  actual  count  of  the  goods,  the  inventories  are  called  physical  inven- 
tories.** If  separate  accounts  showing  the  various  receipts  and  disburse- 
ments are  kept  for  each  kind  of  goods,  a  list  of  the  balances  of  these 
accounts  would  constitute  a  perpetual  or  book  inventory.'** 

Even  where  book  inventories  are  maintained,  their  accuracy  should  be 
verified  by  physical  inventories  taken  from  time  to  time."'  This  can  be 
readily  done  by  inventorying  each  kind  of  goods  when  its  stock  is  low.' 
Where  the  large  bulk  of  stocks  like  ore,  pig  iron,  etc.,  makes  a  physical 
inventory  impracticable,  and,  where  it  may  be  several  years  before  the 
supply  is  materially  reduced,  it  is  customary  that  a  small  percentage  on 
the  consumption  be  allowed  for  waste  or  loss.'*' 

Verifying  Inventories — In  checking  merchandise  inventories,  an  auditor 
should  (a)  have  the  stock  sheets— if  possible,  the  originals'*"— certified  by 
persons  responsible  therefor;'*'  (b)  test  the  calculations  including  all  large 
items;'**  (e)  prove  the  footings;'"  (d)  compare  the  book  and  physical  in- 
ventories;'"' (e)  see  that  invoices  for  inventoried  goods  are  ^recorded ; 
(f )  verify  the  handling  of  consignments  and  branch  shipments;"*  (g)  check 


244  C.  P.  A.  ACCOUNTING 

the  additions  of  duties,  freight,  etc.,  to  cost  of  purchases;'"  (h)  compare 
prices  with  those  of  the  latest  piu'chases  of  similar  goods;'"  (i)  determine 
whether  the  quantities  of  goods  on  hand  are  reasonable  and  in  accordance, 
in  particular  instances,  with  the  average  consumption  and  average  pur- 
chases over  a  fixed  period;"^  (j)  use  the  "gross  jDrotit  check"  and  compare 
the  percentage  of  gross  profit  with  that  of  former  years;"*  (k)  compare 
quantities  and  prices  of  classes  of  stock  with  corresponding  data  in  pre- 
vious inventories;'"  (1)  see  that  proper  allowances  have  been  made  for 
damaged  and  obsolete  goods;"*  and  (m)  compare  the  inventory  prices  and 
the  selling  prices  of  some  of  the  goods  sold  after  the  computation  of  the 
inventory,  to  determine  whether  there  is  sufficient  margin  between  the  two 
prices  to  indicate  a  profit ;  otherwise  it  may  be  inferred  that  the  inventory 
prices  have  been  padded."' 

In  complying  with  rules  (g)  and  (h)  above,  care  should  be  taken  to 
have  the  merchandise  inventories  valued  at  cost  or  market,  whichever  is 
the  lower."*  If  it  is  desired  to  show  inventories  at  cost  when  the  prices 
have  declined,  a  market  fluctuation  reserve  should  be  used.'"  The  use  of 
market  fluctuation  reserves  is  discussed  in  Chapter  X. 

The  "gross  profits  check"  mentioned  in  rule  (j)  above  is  applied  by 
calculating  the  average  gross  profit  in  the  past  and  then  computing  a 
theoretical  final  inventory  by  using  the  actual  figures  for  the  sales,  pur- 
chases, and  initial  inventory,  and  an  estimated  figure  for  the  cost  of  sales 
based  upon  the  average  gross  profit  percentage.  If  the  gross  pi'oflt  per- 
centage is  based  on  sales,  the  cost  of  sales  is  the  product  of  the  sales 
multiplied  by  unity  minus  the  gross  profit  percentage,'""  but,  if  the  gross 
profit  percentage  is  based  on  the  cost  of  sales,  the  cost  of  sales  is  the 
quotient  of  the  sales  divided  by  unity  plus  the  gross  profit  percentage.'" 
The  average  gross  profit  percentage  is  sometimes  calculated  as  the  simple 
arithmetic  average  of  the  annual  averages.'^  but  greater  accuracy  would 
be  obtained  by  using  a  w^eighted  arithmetic  average. 

Trade  Discount — A  trade  discount  is  a  deduction  from  list  price  made 
by  wholesalers  to  retailers.'"  Usually  trade  discounts  do  not  appear  on 
the  books,  being  deducted  directly  from  invoices.'"*  Where  trade  discount 
is  shown  on  the  books,  it  should  be  deducted  from  the  sales'"  or  pur- 
chases'^" as  the  case  may  be.  Sometimes  trade  discounts  have  a  time  limit 
such  as  when  seven  per  cent  is  allowed  for  payment  within  thirty  days. 
The  test  of  a  trade  discount  is  the  rate,  it  being  the  usual  rule  that  terms 
exceeding  "2  per  cent  10  days,  net  30  days"  are  trade  discounts."' 

The  auditor  is  supposed  to  have  a  general  knowledge  of  the  prices  of 
commodities  and  trade  discounts,  and  in  vouching  the  purchase  journal 
should  be  able  to  detect  any  flagrant  irregularities.'^' 

Cash  Discount  on  Purchases — An  allowance  from  the  billed  price  for 
paj-ment  within  a  specified  time  is  a  cash  discount,  if  the  rate  of  discount 
does  not  exceed  what  a  business  could  afford  to  pay  for  the  use  of  the 
money  for  the  credit  period.'™  Some  accountants  look  upon  cash  discounts 
on  purchases  as  a  reduction  in  the  price  of  the  goods,  and  therefore 
deduct  it  from  purchases  ;'*"  while  other  accountants  emphasize  the  element 


TRADING  ACCOUNTS  245 

of  financing  the  business  and  treat  the  discount  as  a  financial  profit."'  As 
the  realization  of  the  reduction  in  price  depends  on  having  sufficient 
capital,  the  latter  treatment  seems  preferable. 

Where  payments  are  made  by  checks,  the  auditor  has  little  to  fear  about 
discounts  on  purchases.  He  should,  however,  scrutinize  in  a  general  way 
the  payment  of  accounts  payable  on  which  no  discounts  are  recorded."' 

Cash  Discount  on  Sales — Some  accountants  regard  a  cash  discount  on 
sales  as  an  overstatement  of  a  revenue,  and  therefore  deduct  it  from  the 
sales  ;"^  others  regard  it  as  an  inducement  offered  to  prospective  customers, 
and  treat  it  as  a  selling  expense  ;"*  while  still  others  regard  it  as  composed 
of  the  two  factors,  interest  and  bad  debts  expense,  and  accordingly  treat 
it  as  a  financial  expense."'.  The  last  treatment  seems  preferable. 

The  auditor  should  secure  from  his  client  an  authoritative  list  of  cash 
discounts  allowed  and  use  it  as  the  basis  for  a  fairly  exhaustive  test  of  the 
discounts  stated  in  the  cash  book.""  As  overstating  discount  allowances  is 
a  frequently  used  method  of  fraud,  this  test  should  be  exhaustive  for  the 
period  covered,  but  it  need  only  extend  over  a  few  days  or  weeks,  depend- 
ing on  the  volume  of  collections."' 

Neglected  Discounts — Some  accountants,  who  consider  that  cash  dis- 
counts should  be  deducted  fi'om  the  invoiced  prices,  consider  it  advisable 
to  have  the  neglected  discounts  shown  on  the  books."'  This  is  accomplished 
hj  debiting  purchases  discount  account  for  all  the  discounts  which  are 
offered  and  crediting  it  with  those  taken."*  Another  method  of  handling 
this  matter  is  to  debit  a  neglected  discounts  account,  and  to  credit  cash  for 
the  discount  not  taken,  the  purchase  being  originally  charged  to  purchases 
and  credited  to  accounts  payable  at  the  best  available  price."" 

Reserve  for  Cash  Discounts — Some  accountants  favor  the  use  of  reserves 
for  cash  discounts,'"  while  others  do  not.'*^  The  use  of  the  reserves  seems 
preferable,  for,  if  prospective  sales  discounts  are  ignored,  the  balance 
sheets  do  not  reflect  true  condition  of  the  business.  Reserves  for  sales  dis- 
counts should  be  deducted  from  the  accounts  receivable  on  the  balance 
sheet,'"  although  they  are  sometimes  classified  as  current  liabilities,"*  and 
are  sometimes  shown  on  the  right  side  of  the  balance  sheet  under  the 
caption  "reserves."'"  The  reserves  for  purchase  discounts  should  be  de- 
ducted from  the  accounts  payable  on  the  balance  sheet.'"  Unless  the  pros- 
pective discounts  are  of  a  considerable  amount,  however,  they  may  be 
disregarded.'*' 


246  C.  P.  A.  ACCOUNTING 


QUESTIONS 

trading  accounts 
General 

1.  How  should  the  guarantee  given  that  machinery  sold  will  last  five 
years  be  treated  in  the  Balance  Sheet?     (A.  I,  of- A.,  June,  1917.) 

2.  In  an  accounting  system  embodying  a  storeroom,  how  should  pay- 
ments on  account,  covering  an  investment  charge,  be  treated,  and  how 
should  the  final  bill  be  treated?     (111.,  May,  1905.) 

3.  Define:  Freight  in  and  freight  out.     (Mich.,  June,  1908.) 

4.  Describe  the  proper  method  of  handling  discounts,  freights  and 
haulage  on  goods  and  machinei*y  purchased.  On  goods  sold.  (Ind.,  Nov., 
1917.) 

5.  A  corporation  that  has  been  lax  in  its  accounting  methods  carries  a 
freight  account  into  which  it  charges  all  payments  of  whatever  nature  it 
makes  to  railroad  companies,  even  payments  made  on  delivery  of  goods 
purchased  f.  o.  b.  location  of  said  corporation.  If  called  upon  to  reor- 
ganize their  methods,  what  suggestions  and  altei'ations  would  you  make 
in  the  matter?    Give  reasons.     (111.,  May,  1906;  Ind.,  June,  1916.) 

6.  In  examining  the  affairs  of  a  manufacturing  concern  you  find  among 
the  assets  finished  goods  inventory  of  $175,798  and  ascertain  that  included 
in  the  above  total  is  the  sum  $50,000  covering  goods  deposited  as  collateral 
to  secure  notes  which  are  included  in  the  Notes  Payable  account.  How 
would  you  treat  this  in  statement  prepared  for  credit  purposes?  Explain 
why.     (A.  I.  of  A.,  May,  1919.) 

7.  A  grain  dealer  charges  his  customers  15  cents  apiece  for  sacks  that 
cost  him  10  cents.  He  agrees  to  receive  back  any  sacks  returned  in  good 
condition  at  12  cents  each,  calculating  that  they  would  be  worth  7i/'2  cents 
each.  How  should  these  transactions  be  treated  on  the  dealer's  books? 
(N.  Y.,  June,  1906;  N.  Y.,  Jan.,  1918.*) 

8.  A  manufacturing  company  ships  its  products  in  packages  costing 
7%  cents  each.  They  are  charged  to  customer  at  10  cents  each  but  subject 
to  credit  when  returned  in  good  order  at  same  price  as  charged.  At  close 
of  year,  package  account  shows  an  apparent  gain,  being  the  difference 
between  the  cost  of  package  and  amount  of  contingent  sales.  What 
disposition  should  be  made  of  the  ledger  gain  at  close  of  year?  (Mich., 
Dec.,  1906.) 

9.  Define:  Turnover.  (Mich.,  June,  1912*;  N.  Y.,  Jan.,  1916*;  Ohio, 
Nov.,  1910;  Iowa,  Dec,  1918;  A.  I.  of  A.,  May,  1921.) 


TRADING  ACCOUNTS  247 

10.  Wiiat  is  meant  by  percentage  on  turnover  and  how  is  it  ascertained? 
(N.  Y.,  June,  1917.) 

11.  How  can  the  amount  of  turnover  be  shown  in  the  Trading  ac- 
count?    (Mich.,  June,  1912.) 

12.  The  ABC  Manufacturing  Company  shipped  a  bill  of  goods 
amounting  to  $300  to  a  customer,  rendering  the  usual  invoice  therefor.  The 
goods  were  destroyed  in  transit  by  a  railway  wreck.  The  ABC  Company 
subsequently  made  a  second  shipment  to  its  customer  to  replace  the  lost 
goods,  and  collected  $300  from  the  railway  company  in  payment  for  the 
lost  shipment.  To  what  account  should  this  last  $300  be  credited?  (N.  Y., 
June,  1915.) 

13.  Suppose,  in  examining  a  set  of  books  for  an  importer  of  tea,  you 
find  that  upon  arrival  all  tea  is  placed  in  bonded  warehouse  for  subse- 
quent withdrawal,  the  tea  having  been  received  by  the  importer  on  orders 
previously  placed  against  six  months'  acceptances  bearing  interest,  and 
that  the  importer  has  omitted  the  tea  from  his  inventory  and  the  ac- 
ceptances as  his  liability.  How  would  you,  as  auditor,  ti'eat  such  matters 
in  pi'eparing  a  certified  statement  of  the  importer's  financial  affairs? 
(N.  Y.,  Jan.,   1919.) 

14.  A  wool  merchant  has  entered  into  an  agreement  to  take  1,000,000 
pounds  of  wool,  in  the  grease,  on  joint  account  with  the  shipper,  to  have 
it  washed,  scoured,  combed,  etc.,  and  sell  the  product,  the  profit  or  loss 
to  be  shared  equally. 

The  grease  wool  is  sent  to  the  scourer  in  lots  as  received  from  the  shipper 
and  returned  from  day  to  day  in  smaller  quantities  as  grades  A,  B,  C, 
D,  and  E;  these  lots  (or  some  of  them)  and  combinations  of  said  lots  are 
sent  to  combers  and  again  returned  in  smaller  quantities  from  day  to  day 
as  grades  Nos.  1,  2,  3,  4,  and  5. 

The  wool  may  be  sold  in  any  form  that  is  in  the  grease  as  grades  A,  B, 
C,  D,  and  E,  or  as  grades  Nos.  1,  2,  3,  4,  and  5. 

It  will  be  necessaiy  for  the  merchant  to  report  the  results  of  the 
transactions  in  each  lot  separately. 

Submit  a  method  for  keeping  a  record  of  this  stock  and  accounts  from 
which  proper  statements  can  be  made  to  the  shipper  showing  the  loss  in 
the  various  processes,  the  quantities  sold,  to  whom,  prices,  etc.,  and  the 
net  loss  or  gain. 

(Pa.,  Nov.,  1908.) 

Inventory 

15.  Define:  Inventory.     (Wash.,  June,  1915.) 

16.  Give  your  definition  of  a  correctly  extended  inventory.  (111.,  May, 
1908.) 

17.  A  concern  inventories  its  property  and  makes  statement  of  assets 
and  liabilities  on  the  first  of  the  year.  The  business  continues  and  up  to 
October  1,  $10,000  additional  capital  is  paid  in.  Plant  amount  during 
the  same  period  increased  $5,000.    From  trial  balance  of  October  1,  how 


248  C.  P.  A.  ACCOUNTING 

would  you  determine  the  amount  of  merchandise  and  supplies  necessarily 
on  hand  to  show  neither  gain  nor  loss  for  the  period  between  January  1 
and  October  1?     (Mich.,  Dec,  1906.) 

18.  What  responsibility  attaches  to  an  auditor  as  to  the  inventory? 
(N.  Y.,  Dec,  1896*;  N.  Y.,  Jan.,  1904*;  Pa.,  Nov.,  1904;  Fla.,  April, 
1907* ;  Fla.,  April,  1908* ;  111.,  May,  1909* ;  Fla.,  July,  1909*  ;  Mich.,  June, 
1910*;  Va.,  Nov.,  1910*;  Wash.,  May,  1911*;  Mich,  June,  1913*;  111., 
May,  1914*;  111.,  May,  1915;  Ind.,  June,  1916*;  Ohio,  Nov.,  1916;  Mass-, 
Oct.,  1917*;  Ohio,  Nov.,  1918*;  N.  D.,  July,  1919*;  S.  C,  Sept.,  1919; 
A.  I.  of  A.,  May,  1919.*) 

19.  An  auditor  is  required  to  certify  to  the  correctness  and  value  of  an 
inventory  with  which  he  is  unfamiliar.  Should  he  decline  to  do  so?  Give 
reasons.     (N.  Y.,  Jan.,  1914;  Ohio,  Nov.,  1915.) 

20.  Woxild  you  examine  the  inventory  of  a  firm,  in  view  of  the  fact 
that  the  auditor  is  not  responsible  for  the  inventory?  If  so,  why?  If  not, 
why  not?     (N.  Y.,  June,  1915.) 

21.  In  making  the  audit  of  a  concern  doing  a  mercantile  business,  what 
would  you  require  to  enable  you  to  certify  to  the  eoiTectness  of  the  in- 
ventory of  merchandise  that  they  had  prepared?     (N.  D.,  July,  1916.) 

22.  How  would  you  verify  the  inventory?  (N.  Y.,  Jan.,  1902*;  111., 
Nov.,  1903*;  N.  Y.,  June,  1912*;  Mass.,  Oct.,  1914*;  Mo.,  Dec,  1914*; 
111.,  May,  1915*;  Mass.,  Oct.,  1915*;  Ohio,  Nov.,  1915*;  Kan.,  Dec,  1915*; 
Mo.,  Dec,  1915*;  N.  Y.,  Jan.,  1916*;  Cal.,  May,  1916*;  Ohio,  Nov.,  1916*; 
W.  Va.,  Mav,  1917;  Ind.,  May,  1917*;  A.  I.  of  A.,  June,  1917*;  Cal.,  June, 
1917*;  Pa.,  Nov.,  1917*;  N.  D.,  July,  1918*;  Ga.,  May,  1919*;  Mich., 
Jime,  1919.*) 

23.  To  what  extent  would  you  consider  it  necessary  to  verify  the  mer- 
chandise, and  what  reference  to  such  verification  would  you  make  in  your 
report?     (Mass.,  June,  1913.) 

24.  How  would  you  prove  in  quantities  the  inventory  of  materials  at 
beginning  of  year,  if  you  had  superintended  an  actual  inventory  at  end  of 
year?     (A.  I.* of  A.,  May,  1919.) 

25.  State  briefly  the  essential  points  to  be  considered  in  a.scertaining 
the  correct  profits  so  far  as  the  inventories  are  concerned?  (N.  D.,  July, 
1919.) 

26.  Under  what  circumstances  should  an  auditor  require  the  officials  of 
a  company  which  manufactures  and  sells  to  retailers,  to  furnish  him  with 
a  certificate  as  to  inventorj'  prices  and  any  other  items  that  may  be  con- 
sidered necessary?  Draft  such  a  certificate  or  certificates  and  indicate  the 
circumstances  under  which  the  different  paragraphs  or  clauses  should  be 
included  in  the  certificate.  Who  should  sign  the  certificate?  Is  the  auditor 
released  from  all  liability  for  the  accuracy'  of  the  items  covered  by  the 
certificate?    Why?     (Ill.l!  May,  1915.) 

27.  State  fully  how  you  would  test  the  accuracy  of  inventoi'y  values. 
(N.  Y.,  Dec,  1898*;  Pa.,  May,  1902*;  N.  Y.,  Feb.,  1910*;  N.  Y.,  June, 
1911*;  Mich.,  June,  1915*;  N.  D.,  July,  1918*;  N.  C,  Nov.,  1919.) 


TRADING  ACCOUNTS  240 

28.  At  what  value  should  inventories  be  shown  on  the  Balance  Sheet? 
(111.,  May,  1904*;  Mich.,  June,  1908*;  Va.,  Oct.,  1911;  Wash.,  June* 
1912*;  La.,  May,  1913*;  Wash.,  Nov.,  1913*;  Mich.,  June,  1914*;  N.  Y., 
Dec.,  1916*;  111.,  May,  1917*;  N.  C,  Aug.,  1917*;  Ohio,  Nov.,  1917*; 
A.  I.  of  A.,  Nov.,  1918*;  Iowa,  Dec,  1918;  A.  I.  of  A.,  May,  1919*; 
Ga.,  May,  1919*;  N.  Y.,  June,  1919*;  Ohio,  Oct.,  1919*;  N.  C,  Nov., 
1919.*) 

29.  (a)  What  is  the  reason  for  the  general  rule  as  to  valuation  of 
merchandise  for  inventory  purposes?  (b)  Is  it  based  on  any  accounting 
principle?     Explain.     (Ohio,  Oct.,  1919.) 

30.  For  inventory  purposes  what  principles  should  govern  the  valua- 
tion of  second-hand  goods  taken  in  trade?  In  your  opinion  how  should 
transactions  in  second-hand  goods  be  reflected  in  the  Profit  and  Loss  ac- 
counts?    (111.,  Dec,  1918.) 

31.  Can  you  suggest  circumstances  in  which  you  would  approve  a  de- 
parture from  the  general  rule  for  valuation  of  merchandise?  Would  you 
be  influenced  by  events  of  conditions  subsequent  to  the  date  of  closing  the 
accounts?  Give  reasons.  (111.,  Nov.,  1908* ;  111.,  May,  1913* ;  A.  I.  of  A., 
Nov.,  1918;  Ohio,  Oct.,  1919.*) 

32.  Discuss  fully  the  proper  principles  to  be  observed  in  the  valuation 
on  a  certified  Balance  Sheet  of  inventories  of  (o)  goods  of  last  season's 
styles,  (&)  goods  in  cases  awaiting  shipment.     (Pa.,  Nov.,  1919.) 

33.  The  theory  has  recently  been  advanced  that  inventory  should  be 
taken  up  at  cost  on  the  Balance  Sheet,  even  if  the  market  price  is  lower, 
on  the  ground  that  you  are  no  more  justified  in  anticipating  losses  than  in 
anticipating  profits.    Wherein  is  the  theory  faulty?     (Cal.,  May,  1916.) 

34.  A  wholesale  and  retail  company,  which  also  manufactures  most  of 
the  goods  sold  by  it,  determines  through  its  cost  system  in  the  factoi'y  the 
cost  of  manufacture,  and  proposes  to  bill  its  wholesale  department  for  all 
goods  manufactured  at  cost  plus  10  per  cent.  What  effect  will  such  pro- 
cedure have  on  statements  issued  by  this  company?  (A.  I.  of  A.,  Nov.,  1919.) 

35.  A  manufacturing  corporation  has  been  accustomed  to  estimate  its 
inventory  value  at  the  end  of  each  year,  and  did  so  December  31,  1915,  and 
December  31,  1916.  What  method  would  you  suggest  for  refiguring  the 
profits  of  1916?     (Mass.,  Oct.,  1917.) 

36.  In  valuing  inventories,  what  does  the  term  cost  mean?  What  does 
the  term   market   mean?     (Wis.,   May,   1919.) 

37.  In  an  inventory,  how  should  stock  purchased  on  credit  subject  to 
usual  discounts  be  valued?  (N.  Y.,  June,  1897;  Pa.,  Nov.,  1911*;  Mass., 
Oct.,  1915.*) 

38.  You  are  auditing  the  accounts  of  a  corporation  which  carries  on  its 
books  and  Balance  Sheet  an  account  called  "Adjustment  of  Inventory," 
which  represents  the  difference  between  the  actual  inventory  of  raw  ma- 
terial on  hand  and  the  book  inventory  carried  as  part  of  its  cost  records. 
What  would  you  recommend  doing  in  regard  to  ttis  account?  (Ohio,  Nov., 
1917.) 


250  C.  P.  A.  ACCOUNTING 

39.  If  the  market  price  of  items  shown  on  the  inventory  is  less  than 
inventory  value,  how  would  you  adjust  the  difference?  If  inventory  values 
were  less  than  market?     (Va.,  Oct.,  1912.) 

40.  Explain  fully  in  what  way,  if  at  all,  overvaluation  of  opening? 
inventory  should  enter  into  Trading:  and  Profit  and  Loss  statement,  with 
reason  for  inclusion  or  exclusion.  (N.  Y.,  Jan.,  1911;  Mass.,  June,  1913; 
Kan.,  May,  1916;  N.  C,  Nov.,  1919.*) 

41.  In  analyzing  the  inventory  of  a  corporation  you  find  that  the  fol- 
lowing have  been  included  therein:  (a)  raw  material,  (6)  in  process 
merchandise,  (c)  finished  merchandise,  (d)  factory  supplies,  (e)  advertis- 
ing, (/)  office  supplies,  (g)  insurance  prepaid,  (h)  construction  materials, 
(0  repair  parts  for  equipment. 

In  preparing  your  Balance  Sheet,  would  you  include  all  of  these  items 
as  Merchandise  Inventoi*y?  If  not,  how  would  you  classify  the  various 
items?    Give  reasons.  (Ohio,  Nov.,  1917.) 

42.  Give  some  reasons  why  the  professional  auditor  should,  under  pres- 
ent-day conditions,  give  even  more  attention  than  in  the  past  to  inventories. 
(A.  I.  of  A.,  May,  1920.) 

43.  Assuming  an  automobile  manufacturing  company  made  a  contract 
for  rubber  tires  at  $35  each  with  the  understanding  that  it  was  to  receive  a 
rebate  of  $5  a  tire  if  the  purchases  exceeded  40,000  tires,  and  that  at  the 
end  of  the  season  when  the  accounts  were  made  up,  say  on  July  31,  it 
was  found  that  45,000  tires  had  been  purchased  and  a  claim  for  the  rebates 
was  thereupon  made  and  a  check  in  settlement  was  received  on  August  31 
following.  On  July  31,  there  were  15,000  tires  on  hand.  At  what  price 
should  they  be  valued  for  inventory  purposes,  and  how  should  the  rebate  be 
dealt  with  in  the  accounts  for  the  year  ending  July  31?    (111.,  May,  1913.) 

44.  Smith,  Jones,  and  Smith,  as  a  finn,  have  transacted  business  for  a 
number  of  years,  dealing  in  Irish  linens.  Jones  decides  to  retire  from  the 
firm  on  January  1,  1919.  The  linens  still  on  hand  have  been  purchased 
from  time  to  time,  during  which  period  the  market  fluctuations  were  fre- 
quent and  extensive  in  range,  the  prevailing  prices  on  December  31,  1918, 
being  considerably  higher  than  any  previously  paid.  What  prices  should 
be  used  in  preparing  the  inventory  of  December  31,  1918,  in  determining 
Jones's  interest  in  the  business?     (N.  Y.,  Jan.,  1919.) 

45.  How  would  you  treat  through  the  books,  the  amount  of  the  un- 
sustained  loss  representing  the  difference  between  the  cost  and  the  market 
price  of  inventories,  in  case  the  market  price  is  less  than  cost,  and  your 
method  of  taking  inventories  as  reported  to  the  Commissioner  of  Internal 
Revenue  was  "cost  or  market  whichever  is  lower"  as  provided  in  T.  D. 
2609?     (N.  C,  June,  1920.) 

46.  What  effect  has  an  overestimated  inventory  on  the  business  of  a 
mercantile  firm?  AVhat  effect  has  an  underestimated  inventory  on  the 
business  of  a  mercantile  firm?     (Md.,  Jan.,  1909;  (^al.,  June,  1917.*) 

47.  Define:  Perpetual  inventorv.  (Wash.,  May,  1911*;  Pa.,  Nov., 
1912*;  Cal.,  Nov.,  1916;  Wash.,  July,  1917.) 


TRADING  ACCOUNTS  261 

i8.    Describe  a  perpetual  invenloi-y,  how  used,  its  advantag:es  and  dis- 
advantages, if  any.     (Wash.,  April,  1906*;  Pa.,  May,  1906;  Pa.    Nov 
1906*;  Fla.,  July,  1909*;  Va.,  Oct.,  1912;  Mass.,  Oct.,  1914*;  Mass.,  Oct.', 
1916;  N.  C,  Nov.,  1918.) 

49.  Explain  the  method  of  maintaining  a  perpetual  inventory.     (Pa 
May,  1902*;  Wash.,  Sept.,  1907*;  Fla.,  July,  1909*;  Pa.,  Nov.,  1912*; 
Mich.,  June,  1913*;  Mich.,  June,  1915;  Mass.,  Oct.,  1916.*) 

50.  How  may  a  perpetual  inventory  be  verified?  What  would  you 
require  in  relation  to  such  an  inventory  before  accepting  it  in  a  Balance 
Sheet  under  your  audit  without  qualifying  your  certificates.  (Pa.,  Nov, 
1912.) 

51.  Describe  the  method  of  determining  points  of  actual  stock-taking 
under  a  system  providing  for  a  perpetual  inventory.  (Wash.,  May, 
1911.) 

52.  To  what  extent  or  under  what  circumstances  could  you  certify  to 
book  inventories  in  the  absence  of  physical  inventories?  (Cal.,  Nov., 
1916*;  Cal.,  June,  1917.) 

53.  In  the  case  where  it  is  impossible  to  take  physical  inventories  but 
once  each  year,  how  would  you  ascertain  the  monthly  profits?  (N.  C, 
June,  1919;  N.  C,  Sept.,  1919;  A.  I.  of  A.,  Nov.,  1920.*) 

54.  What  safeguaids  in  accounting  would  you  suggest  to  a  client  to 
prevent  loss  of  stock  by  theft?     (Mass.,  June,  1910.) 

55.  Define :  Book  inventory.    (A.  I.  of  A.,  May,  1918* ;  111.,  Dee.,  1918.) 

56.  Indicate  in  what  circumstances  and  for  what  purposes  you  would 
consider  a  "book  inventory"  to  be  used  in  a  manufacturing  business:  (a) 
for  current  information;  (b)  for  use  in  the  preparation  of  interim  state- 
ments of  accounts;  (c)  for  use  in  the  preparation  of  final  yearly  or 
half-yearly  accounts.  Assuming  your  client  decided  to  rely  entirely  upon 
such  records,  what  steps  should  be  taken  to  guard  their  accuracy?  (A.  I. 
of  A.,  May,  1918.) 

57.  A.  B.,  a  manufacturer,  states  that  he  has  had  a  loss  in  his  factory 
during  his  last  fiscal  period  which  he  can  not  understand,  and  calls  you  in 
to  ascertain  where  the  loss  has  occurred.  State  the  nature  of  the  business 
used  for  illustration  and  describe  fully  what  methods  you  would  employ 
to  discover  where  the  loss  occurred,  and  how  you  would  prove  the  correct- 
ness of  your  deductions.     (Mass.,  June,  1912.) 

58.  For  several  successive  terms,  a  corporation  has  regularly  deducted 
$15,000  from  its  inventory  of  stock  in  process,  the  inventory  prices  being 
conservatively  below  market  value.  Then,  for  the  quarter  ended  June  30, 
1908,  while  following  the  same  conservatism  regarding  prices,  the  $15,000 
deduction  was  discontinued,  and  the  treasurer  made  his  statement  of 
earnings  during  the  term,  based  upon  the  inventory  amount  of  stock  in 
process.  State  (a)  whether  you  agree  therewith;  (b)  the  reasons  for 
your  agreement  or  disagreement,  and  (c)  if  you  disagi'ee  with  the 
treasurer's  statement  of  the  matter,  how  you  would  deal  with  the  item  on 
your  statement  of  earnings  and  of  surplus?     (Mass.,  April,  1911.) 


252  C.  P.  A.  ACCOUNTING 

Books 

59.  Prepare  a  ruling  for  a  Sales  Book  to  provide:  (1)  Total  monthly 
postings  to  three  good  accounts;  (2)  the  separation  of  cash  sales  from 
charge  sales,  (3)  supplementary  distribution  of  sales  among  four  sales- 
men's columns.    (N.  Y.,  Jan.,  1897;  N.  Y.,  Jan.,  1907.) 

60.  Prepare  a  ruling  for  an  invoice  book  to  provide  for  total  monthly 
charges  to  three  Material  accounts  and  two  Expense  accounts,  and  also  to 
detail  postings  to  sundry  accounts  of  capital  and  revenue.  (N.  Y.,  Jan., 
1897;  N.  Y.,^Jan.,  1907.) 

61.  Submit  rulings  for  correlated  Cash  Book,  Purchase  Book  and  Sales 
Book,  to  classify  purchases  and  sales  in  three  divisions  and  to  provide  for 
miscellaneous  purchases.  Provision  must  be  made  to  record  cash  sales  in 
Sales  Book  and  in  Cash  Book  and  for  customers'  and  creditors'  controlling 
accounts.  Submit  pro  forma  monthly  summary  entries  for  the  foregoing 
books.     (N.  Y.,  Jan.,  1911.) 

62.  The  proprietor  of  a  mercantile  store  asked  you  to  prepare  a  set  of 
blanks  to  be  used  by  him  for  checking  all  of  his  delivery  wagons,  in- 
cluding the  returned  goods  which  the  drivers  collected.  He  wants  to  be 
able  to  trace  every  package.     Outline  such  a  set.     (Mich.,  June,  1914.) 

63.  About  a  year  ago  the  Ford  Motor  Company  agreed  to  make  a 
refund  to  each  purchaser  during  the  succeeding  year,  if  a  stated  number 
of  machines  were  sold  during  the  year.  If  you  had  been  asked  to  deter- 
mine a  method  by  which  the  necessary  records  should  be  kept  in  order 
that  the  least  inconvenience  should  be  experienced  at  the  close  of  the 
year  when  this  refund  was  made,  it  being  apparent  that  the  requirement 
would  be  met,  what  orders  would  you  have  given  ?  Write  a  report  embody- 
ing same.     (Mich.,  June,  1915.) 

64.  What  do  vou  consider  a  complete  checking  of  the  Purchase  Journal  ? 
(R.  I.,  Dec,  1907.) 

65.  According  to  the  regulations  of  the  United  States  Food  Administra- 
tion, wholesale  and  retail  dealers  must  base  the  selling  price  of  certain 
commodities  upon  the  cost  price  rather  than  upon  the  replacement  or  the 
market  value.  Outline  a  stock  system  for  a  wholesale  grocer  to  use  for  this 
purpose  and  record  hj'pothetical  data  upon  such  forms  as  you  may  design, 
showing  in  detail  the  application  of  your  recommendations.  (Wis.,  April, 
1918.) 

Merchandise  Account 

66.  Define:  Merchandise  account.  (N.  Y.,  Jan.,  1897*;  N.  Y.,  June, 
1899*;  N.  Y..  June,  1901*;  Pa.,  Mav,  1903*;  Mich..  July,  1906;  N.  Y., 
June,  1909*;  Fla.,  July,  1909;  La.,  May,  1913*;  W.  Va.,  May,  1917.*) 

67.  State  how,  in  your  opinion,  the  Merchandise  account  should  be 
kept.     (N.  Y.,  Dee.,  1898;  Mich.,  July,  1906.*) 

68.  Is  the  common  old-fashioned  method  of  adding  the  inventory  of 
merchandise  on  hand  to  the  credit  side  of  the  Merchandise  account  before 


TRADING  ACCOUNTS  253 

closing  the  books  theoretically  correct?     Explain   fully,      (N.  Y.,   Jan., 
1901;  Mich.,  June,  1908.) 

69.  What  is  the  theoretic  aspect  of  the  Merchandise  account  when 
kept  in  modern  books?  Consider  the  proposition  in  connection  with  the 
modem  books  of  original  entry.     (N.  Y.,  Jan.,  1914.) 

70.  State  what  is  indicated  by  the  Merchandising  account  (a)  when  the 
account  shows  a  debit  balance;  {b)  when  the  account  shows  a  credit 
balance?    Explain  fully.     (N.  Y.,  Jan.,  1900.) 

71.  State  your  objections,  if  any,  to  the  Merchandise  account  fre- 
quently appearing  on  ledgers.  (N.  Y.,  June,  1904*;  N.  Y.,  June,  1909*; 
Mass.,  June,  1910*;  Ohio,  Nov.,  1913*;  Mo.,  Dec,  1914.) 

72.  Explain  the  proper  method  of  handling  Merchandise  account,  and 
outline  the  various  subdivisions.  (N.  Y.,  Jan.,  1897*;  N.  Y.,  June,  1904*; 
N.  Y.,  June,  1909*;  Fl.a.,  July,  1909*;  Mass.,  June,  1910*;  Mo.,  Dec, 
1914*;  Mich.,  June,  1915;  N.  D.,  Aug.,  1917.*) 

Purchases 

73.  State  how  you  would  verify  purchases.  (Va.,  Nov.,  1910;  Kan., 
Dec,  1915;  Mo.,  Dec,  1915.) 

74.  What  elements  enter  into  the  cost  of  goods  purchased  for  sale? 
(Ohio,  Oct.,  1919.) 

75.  If  you  should  contract  for  an  audit  which  provides  for  an  examina- 
tion of  all  vouchers,  what  would  you  accept  as  proper  vouchers  for  pur- 
chases? (Ohio,  March,  1910*;  Wis.,  May,  1916*;  Va.,  Nov.,  1918;  Ohio, 
Oct.,  1919.*) 

76.  State  how  you  would  deal  with  missing  invoices  for  goods  purchased 
in  auditing  .the  Cash  Book  of  a  large  corporation?     (N.  D.,  July,  1918.) 

77.  Tiace  the  various  operations  in  an  office  where  you  have  full  charge 
as  an  accountant  from  the  time  an  order  is  given  for  the  purchase  of 
material  until  such  material  is  paid  for,  to  protect  the  company  from  any 
possible  loss  in  the  transaction.  (N.  Y.,  Jan.,  1900*;  Mich.,  Nov.,  1907; 
Mass.,  June,  1910.) 

78.  Describe  a  means  for  the  protection  of  a  manufacturing  company 
in  the  purchase  of  necessary  materials  and  supplies  and  in  the  payment 
for  such  materials  and  supplies.     (N.  Y.,  Jan.,  1900.) 

79.  When  auditing  the  books  of  a  firm  you  find  that  no  invoice  transac- 
tions are  recorded  on  the  firm's  books  until  they  have  matured.  Give  your 
opinion  upon  this  method  and  show  how  the  Balance  Sheet  of  the  firm 
would  be  affected.     (Wash.,  June,  1915;  Va.,  Nov.,  1918.*) 

80.  What  check  has  the  auditor  on  fictitious  entries  showing  pur- 
chases?    (N.  Y.,  June,  1900;  Wash.,  May,  1903.) 

81.  In  the  course  of  an  audit  of  a  stove  manufacturer's  books  as  of 
March  31,  1912,  you  notice  that  three  invoices,  respectively  for  pig  iron, 
sand,  and  fire  brick,  dated  March  22,  27,  and  28,  were  entered  on  the 


254  C.  P.  A.  ACCOUNTING 

books  in   April.     What   does   this  sig-nify   and   how   may  it   affect   your 
report?     (111.,  May,  1913.) 

82.  When  you  audit  the  accounts  of  the  ABC  Dry  Goods  Company, 
you  find  that  the  books  do  not  show  any  asset  or  liability  in  respect  of 
spring  goods  in  transit.  What  would  you  report  on  this  situation,  and 
why?    (III.,  May,  1916;  III.,  Dec.,  1916*;  Cal.,  June,  1917.) 

83.  In  connection  with  the  audit  of  the  books  of  a  corporation,  describe 
briefly  the  procedure  you  would  consider  to  be  essential  in  verifying  the 
goods  in  transit.     (Wash.,  July,  1917.) 

84.  In  the  trial  balance  of  a  corporation,  December  31,  1910,  the  end 
of  the  fiscal  term,  there  is  a  debit  of  $50,000  against  John  Doe  for  a  pay- 
ment to  him  on  account  of  material  purchased  from  him.  The  material  is 
to  be  delivered  after  said  date.  How  should  this  be  classified  in  the 
Balance  Sheet,  December  31,  1910?     (Mass.,  June,  1912.) 

85.  Should  you  be  informed  that  liabilities  representing  merchandise 
received  but  not  inventoried  were  omitted,  would  the  explanation  be  satis- 
factory?    (Ohio,  Nov.,  1915.) 

86.  What  routine  would  you  recommend  in  the  checking  of  invoices  of 
goods  or  materials  purchased?     (N.  Y.,  June,  1901.) 

87.  How  would  you  classify  the  following  items  in  preparing  a  state- 
ment of  condition,  to  be  filed  with  the  Secretary  of  the  Commonwealth : 
(a)  Duty  paid  on  foreign  merchandise  received,  invoice  for  same  being 
unpaid;  (b)  an  ascertained  loss  on  a  contract  for  the  purchase  of  mer- 
chandise to  be  delivered  in  the  following  month?     (Mass.,  April,  1911.) 

Returns  and  Allowances 

88.  Define :  Returns  and  allowances.    (HI.,  May,  1908.) 

89.  Describe  a  safe  and  easy  system  of  keeping  the  account  of  goods 
returned,  (a)  as  buyer,  (6)  as  seller.  (N.  Y.,  June,  1899;  Pa.,  May, 
1900.*) 

90.  State  how  you  would  verify  the  returned  sales.  (N.  Y.,  Dec, 
1898* ;  N.  Y.,  June,  1900* ;  N.  Y.,  Jan.,  1901* ;  N.  Y.,  Feb.,  1909* ;  Mass., 
June,  1910*;  Va.,  Nov.,  1910;  Kan.,  Dec.,  1915*;  Mo.,  Dec.,  1915.*) 

91.  Classify  the  Returned  Sales  account  properly  according  to  the  sub- 
division of  assets,  liabilities,  proprietary  interest,  income  and  expenses, 
under  which  it  should  be  grouped.     (Wis.,  May,  1919.) 

92.  Under  instructions  calling  for  a  complete  audit  and  verification  of 
all  entries,  what  supporting  data  would  you  require  for  returned  sales? 
(Ohio,  March,  1910;  Va.,  Nov.,  1918*;  Ohio,  Oct.,  1919.*) 

93.  State  what  means  should  be  adopted  to  verify  allowances  on  sales. 
(N.  Y.,  Jan.,  1901*;  N.  Y.,  Feb.,  1909*;  Mass.,  June,  1910.) 

94.  Under  instructions  calling  for  a  complete  audit  and  verification  of 
all  entries,  what  supporting  data  would  you  require  for  returned  pur- 
chases?    (Ohio,  March,  1910;  Ya.,  Nov.,  1918.*) 


TRADING  ACCOUNTS  255 

Selling  Expenses 

95.  Define:  Selling  expenses.     (Wash.,  June,  1917.) 

96.  Where  would  you  i?lace  the  Entertaining-  Customers'  account  in  the 
Income  and  Profit  and  Loss  account?     (Md.,  Dec,  1917.) 

97.  Classify  Expenses  Advanced  Salesmen  according  to  the  subdivision 
of  assets,  liabilities,  proprietary  interest,  income  and  expenses,  under 
which  it  should  be  grouped.     (Wis.,  May,  1919.) 

98.  In  making  a  detailed  audit,  what  procedure  would  you  follow  to 
verify  the  advances  to  salesmen?     (Kan.,  Dec,  1915;  Mo.,  Dec,  1915.) 

99.  Would  you  consider  it  proper  under  any  circumstances  for  a  com- 
pany to  treat  all  or  any  of  its  advertising  expenses  incurred  during  the 
year  as  an  asset  for  the  purpose  of  its  annual  accounts  to  be  submitted 
to  the  stockholders?  In  your  answer  state  fully  the  grounds  for  your 
views  and  explain  how  you  would  recommend  such  expenses  should  be 
dealt  with.     (N.  Y.,  June,  1908*;  111.,  May,  1914;  Ind.,  June,  1916.) 

100.  In  closing  the  books  of  a  company  at  the  end  of  its  first  fiscal 
year,  how  would  you  treat  advertising  booklets  on  hand,  estimated  to  last 
another  year?     (Mass.,  Oct.,  1917.) 

101.  State  what  is  indicated  by  the  Commission  account  (a)  when  the 
account  shows  a  debit  balance,  (b)  when  the  account  shows  a  credit  bal- 
ance.    Explain  fully.     (N.  Y.,  Jan.,  1900.) 

102.  What  in  your  opinion  is  the  proper  treatment  of  the  unpaid  com- 
missions to  salesmen  in  arriving  at  the  profit  or  loss  of  a  business  for  a 
specific  fiscal  period?     (Ohio,  Dec,  1908.) 

103.  Would  you  consider  it  proper  to  include  as  an  asset  the  advertis- 
ing expenses?    (N.  Y.,  Jan.,  1897.) 

Discounts 

104.  Explain  the  various  kinds  of  discounts  found  in  the  business 
world  and  the  methods  by  which  they  should  be  distinguished  in  the  books 
of  account.     (Fla.,  July,  1909*;  Mich.,  June,  1913.) 

105.  Define:  Trade  discount.  (Ill,  May,  1908;  Va.,  Nov.,  1910;  Iowa, 
Dec,  1918.) 

106.  Classify:  Trade  discount.     (Iowa,  Dec,  1918.) 

107.  State  how  trade  discount  is  used  on  a  Profit  and  Loss  statement 
or  Balance  Sheet,  also  its  classification.     (Iowa,  Dec,  1918.) 

108.  Define:  Cash  discount.  (111.,  May,  1908;  Va.,  Nov.,  1910;  Iowa, 
Dec,  1918.) 

109.  State  two  methods  of  considering  merchandise  discount.  Give 
reasons  for  your  preference.     (N,  D.,  June,  1914.) 

110.  State  how  cash  discount  is  used  in  a  Profit  and  Loss  statement  or 
Balance  Sheet,  also  its  classification.     (Iowa,  Dec,  1918.) 


256  C.  P.  A.  ACCOUNTING 

111.  (a)  State  three  methods  of  treating  commercial  cash  discounts, 
(b)  Sketch  sufficient  rulings  of  Purchase  and  Cash  Book  to  show  the 
use  of  each  method,  respectively.     (Wis.,  May,  1916.) 

112.  Give  your  opinion  regarding  the  following,  and  reasons :  Should 
ordinary  discount  of  2y2  per  cent  a  month  be  considered  as  a  trade  or  a 
time  discount?     (Mich.,  June,  1908.) 

113.  In  a  given  trade  goods  are  purchased  on  terms  of  5  per  cent  dis- 
count for  cash  in  ten  days  or  net  thirty  days.  In  certifying  a  Balance 
Sheet  in  this  trade  how  would  you  deal  with  the  question  of  this  discount 
in  stating  the  value  of  the  inventoi-y  of  merchandise  on  hand?  Give  reason 
for  the  treatment  you  would  advocate.     (Pa.,  Nov.,  1911.) 

114.  Describe  the  system  of  running  a  discount  column  in  the  Cash 
Book  and  show  how  the  entries  are  made  both  in  the  General  Ledger  and 
in  the  Subsidiary  Ledger.     (N.  Y.,  Oct.,  1907.) 

115.  To  what  extent  should  an  auditor  hold  himself  responsible  for  the 
correctnes.s  of  discounts?  (N.  Y.,  Dec.,  1896;  Va.,  Nov.,  1910;  Mich., 
June,  1912.) 

116.  In  passing  upon  discounts,  how  would  you  state  them  in  reports? 
(111.,  May,  1905.) 

117.  State  what  means  should  be  adopted  to  verify:  (a)  Discounts  Al- 
lowed, (b)  Discounts  Received.  (N.  Y.,  Jan.,  1901*;  Mass.,  June,  1910; 
Mich.,  June,  1912.*) 

118.  How  should  cash  discounts  on  purchases  be  treated?  (N.  Y., 
June,  1902*;  Ohio,  Nov.,  1918.) 

119.  Assuming  that  all  cash  discounts  on  purchases  made  during  the 
year  have  been  taken  advantage  of,  and  the  Ledger  account  shows  a 
substantial  credit,  to  what  account  would  you  close  the  balance?  In  the 
case  of  a  jobbing  concern?  Of  a  manufacturer?  (111.,  Dec,  1907;  Mich., 
June,  1912*;  Ind.,  Nov.,  1918.) 

120.  Classify  Cash  Discounts  on  Merchandise  Sales  and  on  Mer- 
chandise Purchases  according  to  the  subdivision  of  assets,  liabilities,  pro- 
prietary interest,  income  and  expenses  under  which  they  should  be 
grouped.  (N.  Y.,  Jan.,  1897*;  N.  Y.,  June,  1908*;  N.  Y.,  Jan.,  1919*; 
Wis.,  May,  1919;  N.  Y.,  Jan.,  1920.*) 

121.  In  auditing  department  store  "A,"  you  find  that  cash  discounts  on 
purchases  are  regularly  deducted  from  invoices  when  they  are  entered  in 
the  books,  while  in  store  "B,"  the  invoices  are  entered  in  full  and  the  dis- 
counts are  credited  to  a  discount  account  as  and  when  received.  Discuss 
the  relative  advantages  and  disadvantages  of  the  two  methods,  and  state 
what  variations,  if  any,  would  occur  in  the  valuing  of  inventories  under 
the  two  methods.     (A.  L  of  A.,  May,  1920.) 

122.  In  preparing  the  Balance  Sheet  of  a  business  at  the  close  of  a 
year,  how  should  you  treat  Discount  on  Accounts  Pavable?  (Mich.,  June, 
1908*;  Mass.,  Jime,  1910.) 


TRADING  ACCOUNTS  257 

123.  What  is  the  proper  treatment  of  Cash  Discounts  on  Sales? 
(Ohio,  March,  1910*;  Ohio,  Nov.,  1918.*) 

124.  Should  provision  be  made  out  of  revenue  to  provide  for  Discount 
on  Accounts  Receivable  outstanding  after  the  date  of  closing?  Give 
reasons  for  your  answer.     (N.  Y.,  Jan.,  1904.) 

125.  In  preparing  the  Balance  Sheet  of  a  business  at  the  close  of  a 
year,  how  should  you  treat  Discount  on  Accounts  Receivable?  (Mass., 
June,  1910.) 

126.  A  manufacturing  company  buys  its  material  on  30  days'  credit,  or 
5  per  cent  cash  discount  if  paid  in  10  days.  In  order  to  take  advantage  of 
the  cash  discount,  the  company  borrows  money  from  the  bank  at  3  per 
cent.  How  should  this  discount  and  interest  be  treated  on  the  books  of 
the  company?     (N.  Y.,  Jan.,  1918.) 

127.  A  corporation  sells  all  of  its  manufactured  product  at  5  per  cent 
10  days,  3  per  cent  30  days,  or  net  60  days.  In  making  the  annual  audit 
you  find  that  ninety  per  cent  of  the  concern's  customers  take  one  of  the 
two  discounts.  What  provision  would  you  think  desirable  to  make  to  pro- 
vide for  the  discounts  that  will  be  taken  in  the  period  next  to  the  one 
under  audit?  Give  arguments  for  or  against  such  provision.  (N.  D.,  July, 
1918.) 

Sales 

128.  Under  instructions  calling  for  a  complete  audit  and  verification  of 
all  entries,  what  supporting  data  w^ould  you  require  for  Sales?  (Ohio, 
March,  1910;  Va.,  Nov.,  1918*;  Ohio,  Oct.,  1919.*) 

129.  State  how  you  would  verify  the  Sales.  (N.  Y.,  Jan.,  1901; 
Mass.,  June,  1910;  Va.,  Nov.,  1910.) 

130.  How  would  you  discover  and  subsequently  prevent  defalcations 
when  a  shipping  clerk  and  driver  steal  merchandise  ostensibly  sent  to 
customers?     (Cal,  May,  1916.) 

131.  How  would  you  discover  and  subsequently  prevent  defalcations 
when  the  manager  and  cashier  of  a  manufacturing  plant  in  collusion  de- 
stroy original  orders,  fail  to  enter  bills  against  customers  in  regular  books, 
and  keep  returns  on  same  when  made?     (Cal.,  June,  1917.) 

132.  A  firm  of  export  merchants  desires  a  thorough  investigation  of 
the  past  year's  transactions,  having  reason  to  suspect  fraud.  State  con- 
cisely upon  what  lines  you  would  proceed  to  satisfy  yourself  that :  (a) 
Goods  had  been  taken  out  of  the  store  on  bond  in  the  way  shown  by  the 
books;  (b)  they  had  been  shipped  to  account  of  proper  consignee;  (c) 
that  no  goods  had  been  removed  without  being  charged;  (d)  that  no  ficti- 
tious entries  had  been  made  in  the  books.     (Wash.,  Aug.,  1908.) 

133.  Sales  aggregating  $25,000  were  entered  on  the  books  as  at  Decem- 
ber 31,  1915,  but  the  goods  were  not  all  delivered  until  January  25,  1916. 
Indicate  what  adjustments  you  would  make  of  the  books  or  of  the  ac- 
counts for  the  purpose  of  statement  on  the  Balance  Sheet.  Give  Journal 
entries.     (111.,  May,' 1916.) 


258  C.  P.  A.  ACCOUNTING 

134.  If  in  auditin.af  the  accoiin(s  of  a  manufacturing  corporation  you 
found  that  certain  orders  booked  for  jiroducts  to  be  manufactured  subse- 
quent to  llie  closinc:  period,  had  nevertheless  been  entered  upon  the  books 
as  actual  sales,  how  would  you  treat  such  entries  in  the  sales  record? 
(N.  Y.,  June,  1919.) 

135.  Upon  the  audit  of  the  partnership  accounts  of  a  manufacturing 
business  the  followinjr  condition  is  revealed.  Sales  toward  the  end  of 
tlie  period  are  unusually  larije.  What  would  you  deduce  from  this  fact 
and  what  would  you  feel  called  upon  to  do?     (Kan.,  May,  191G.) 

136.  What  means  should  be  employed  to  detect  the  willful  omission  to 
enter  in  the  books  under  audit  sales  made?     (X.  Y.,  Dec,  1S96.) 

137.  A  bookkeeper  of  a  manufacturing:  concern  fails  to  make  all 
the  proper  charges  of  time  sales.  No  order  book  was  kept.  He  retains 
$1,200  to  $1,500  yearly  for  five  years  of  payments  received  on  account  of 
tune  sales  by  cashing  the  checks  received  in  due  course  out  of  the  cash 
drawer,  and  deposits  of  said  cheeks  regularly  in  the  firm's  bank  account, 
properly  endorsed  by  the  firm.  How  would  you  detect  this?  (Md.,  Jan., 
1909.)  ' 

138.  How  would  you  require  manufactured  goods,  shipped  to  customer 
for  sale  to  them,  as  and  when  used  by  them,  to  be  statetl  in  a  Balance 
Sheet  you  are  asked  to  certify?  How  would  you  ascertain  that  all  goods 
so  shipped  come  under  your  notice?     (111.,  Dec,  1916.) 

139.  "A"  operates  a  five  and  ten  cent  cash  store  and  for  the  purpose 
of  selling  out  the  strength  of  large  profits  shown  in  the  books,  makes  false 
entries  increasing  the  sales  and  corresponding  entries  on  the  disburse- 
ment side  of  the  Cash  Book  purporting  to  represent  withdrawals.  The 
records  by  which  the  sales  might  be  verified  are  destroye<l.  "A"  makes  a 
proposition  to  "B"  to  sell  the  business  and  gives  statements  drawn  from 
the  books  showing  the  profits  of  the  business.  "B"  engages  a  certified 
public  accountant  to  make  an  examination,  who  discovei-s  the  fraud. 

How  and  by  what  procedure  could  he  discover  the  fraud? 

(Mass.,  Oct.,  1916.) 

140.  A  house  sraids  out  many  goods  on  approval  and  treats  the  transac- 
tions as  sales.  How  should  such  items  be  treated  by  the  auditor  when  set- 
ting up  statements  for  the  period?     (N.  Y.,  Jan.,  1904.) 

141.  Illustrate  how  to  handle  goods  sent  out  for  sale  or  return.  (Mich., 
June,  1919.) 

142.  A  firm  is  in  the  habit  of  supplying  goods  on  the  principle  of  sale 
or  return,  taking  payments  by  installments  covering  principal  and  interest, 
the  purchaser  having  the  option  to  return  the  goods  at  any  time,  forfeiting 
the  installments  paid.  How  would  you  recommend  that  such  conditional 
sales  should  be  entered  in  the  books  of  the  selling  firm,  and  how  should 
the  outstanding  amounts  be  from  time  to  time  valued?     (111.,  Dec,  1910.) 

143.  A  manufacturer  disposes  of  a  special  machine,  partly  by  direct 
sales  to  customers  and  partly  by  lease  with  the  provision  that  it  may  later 


TRADING  ACCOUNTS  259 

be  pnreliased.  If  a  customer  takes  advantage  of  this  provision,  the  pay- 
ment made  on  the  machine  from  the  date  of  the  lease  may  be  applied  as  a 
reduction  of  the  purchase  price. 

Prepare  Journal  entries  supplying  figures  showing  the  necessary  ac- 
counts for  recording  the  sale  and  lease  transactions  of  this  business. 

(Mass.,  Oct.,  1916.) 

144.  Suggest  a  plan  whereby  a  department  store  might   adequately 
counts  for  recording  the  sale  and  lease  transactions  of  this  business, 
submit  forms.     (N.  Y.,  Jan.,  1914.) 

145.  Explain  fully  transaction  involving  the  sale  of  a  piano  on  what 
is  termed  an  installment  lease.  Explain  how  you  would  handle  this  on 
the  books  and  records  from  time  the  transaction  is  entered  into  until 
completed.     (Pa.,  Nov.,  1918.) 

146.  In  the  audit  of  the  books  of  an  installment  merchandise  dealer, 
in  what  particular  would  you  examine  the  books  that  would  not  be  neces- 
sary in  other  classes  of  business?     (N.  C,  June,  1919;  N.  C,  Nov.,  1919.) 

147.  On  certain  lines  of  business  (e.g.,  real  estate  companies,  sewing 
machine  agencies,  retail  furniture  houses)  a  large  proportion  of  the  sales 
is  made  on  the  installment  plan,  and  at  the  close  of  any  period  a  large 
number  of  such  transactions  may  be  uncompleted.  How  should  the 
profits  from  such  uncompleted  sales  be  treated  in  the  Balance  Sheet? 
(Fla.,  July,  1909.) 

148.  A  concern  located  in  a  village  and  employing  400  people,  fur- 
nishes them  with  coal,  wood  and  ice  at  cost,  collecting  for  same  in  weekly 
installments.  Describe  the  best  method  of  keeping  the  accounts.  (Mass., 
June,  1913.) 

149.  You  have  been  called  upon  to  audit  the  books  of  a  furniture  com- 
pany which  sells  on  the  installment  plan.  The  books  have  been  closed 
when  you  reach  the  office,  and  you  are  handed  a  completed  Profit  and 
Loss  account  and  Balance  Sheet.  The  company  has  been  in  business  one 
year  only. 

On  investigation  you  find  that  all  installment  sales  are  credited  to  an 
account  designated  "Installment  Sales."  A  controlling  account  and  sub- 
ledger  are  kept  for  the  installment  customers. 

You  find  that  the  total  of  the  installment  sales  has  been  credited  to 
"Installment  Sales"  account,  which  has  been  closed  into  Profit  and  Loss. 

All  accounts,  installment  and  otherwise,  that  were  known  to  be  uncol- 
lectible have  been  charged  off.  There  ai'e  no  reserves  against  balances 
due  from  customers  on  the  books. 

What  criticisms  or  corrections  have  you  to  suggest  as  to  the  correctness 
or  otherwise  of  the  Balance  Sheet  and  Profit  and  Loss  statement  handed 
to  you? 

(A.  I.  of  A.,  May,  1920.) 

150.  (o)  Of  the  total  sales  of  the  X  Company,  a  considerable  por- 
tion is  made  on  approval  for  5,  10,  or  30  days,  according  to  the  com- 
modity.    If   unsatisfactory,   the   articles   may   be   returned    within    the 


260  C.  P.  A.  ACCOUNTING 

specified  time.  State  how  such  sales  should  be  treated  in  the  following: 
(1)  books  of  original  entry,  (2)  ledgers,  (3)  revenue  account,  (4)  Balance 
Sheet. 

(b)  Retui-nable  packages  are  charged  to  customers  at  a  price  in 
excess  of  cost.  State  how  such  items  should  be  treated  at  time  of  sale 
and  of  return,  in  the  following:  (1)  books  of  original  entry,  (2)  cus- 
tomers' accounts,  (3)  revenue  account,  (4)  Balance  Sheet. 

(Wis.,  May,  1916.) 

151.  A  retail  bookstore  agrees  to  deliver  certain  sets  of  books  at  $20, 
on  payment  of  $2  down,  the  purchaser  agreeing  to  make  $3  payments  for 
each  of  the  six  months  next  following.  It  is  expected  that  sales  on  this 
plan  will  aggregate  several  hundred  sets.  Suggest  a  method  of  keeping 
the  accounts,  so  that  results  may  be  readily  shown.     (N.  Y.,  June,  1902.) 

152.  A  furniture  installment  dealer  made  a  sale  on  installments, 
amounting  to  $200,  which  was  double  the  cost.  During  the  year  install- 
ments were  collected  amounting  to  $50.  During  the  second  year  no  in- 
stallments were  collected,  but  goods  were  repossessed  which  were  un- 
damaged.    Journalize  the  transactions.     (N.  C,  June,  1919.) 

153.  The  Western  Farm  Machinery  Company  sells  its  product  to  farm- 
ers on  three  years'  credit,  payable  in  three  equal  annual  installments.  The 
cost  of  production  for  one  year  is  $10,987,600;  sales,  $13,210,900;  selling 
and  administration  exjienses,  $223,300,  including  all  contingencies.  Find 
an  equitable  method  of  stating  profits  for  the  year.  (N.  Y.,  Jan..  1914; 
N.  Y.,  June,  1917.*) 


TRADING  ACCOUNTS 


201 


PROBLEMS 


TRADING  ACCX)UNTS 


JONES  MANUFACTURING  COMPANY 
Trial  Balance,  December  31,  1913  (Before  Closing) 


Credits 


Accounts  Payable 

Accounts  Receivable 

Capital  Stock 

Cash  in  Banks 

Commissions 

Depreciation 

Discount  on  Sales 

Discount  on  Purchases 

Finished  Product  (Inventory  at  December  31, 

1912) 

Freight  Inward 

Freight  Outward 

Factory  Expense 

Insurance  

Interest 

Labor 

Machinery  and  Equipment 

Material  Purchased 

Material  Inventory  (at  December  31,  1912) .  .  . 

Notes  Payable 

Office  and  Selling  Expenses 

Petty  Cash 

Prepaid  Taxes 

Prepaid  Interest 

Power 

Reserve  for  Depreciation 

Repairs 

Rent 

Salaries 

Sales 

Supplies 

Surplus 

Taxes 

Traveling  Expenses 

Unexpired  Insurance 

Work  in  process  (Inventory  December  31,  1912) 


$42,739.66 

3,706.82 

7,750.71 

12,067.30 

4,986.22 


110,630.84 

4,709.81 

3,542.39 

52,796.57 

5,372.90 

3,850.00 

179,473.82 

120,672.96 

158,691.26 

10,786.90 

14,790.82 

150.00 

672.80 

375.00 

7,500.00 

5,281.76 
15,000.00 
32,250.00 

6,872.90 

2,937.50 

4,836.24 

6,821.16 

53,689.39 

$872,955.73 


$22,560.71 
150,000.00 

6,792.40 


60,000.00 

20,978.23 

570,478.31 
42,146.08 

$872,955.73 


262 


C.  P.  A.  ACCOUNTING 


Inventories  at  December  31,  1913,  are:  Material,  $9,877.44;  work  in 
process,  $56,091.29;  finished  product,  $71,170.10. 

From  the  above  trial  balance  and  facts  prepare:  (a)  Balance  Sheet,  De- 
cember 31,  1913;  (b)  statement  showing  cost  of  manufacture  and  goods 
sold;  (c)  Profit  and  Loss  account.  Importance  will  be  attached  to  the 
form  and  classification  of  these  statements  in  marking  answers. 

(Mass.,  Oct.,  1914.) 

2.  From  the  following  trial  balance  of  the  A.  B.  Stevens  Manufacturing 
Co.  prepare  a  Balance  Sheet  at  December  31,  1915,  and  a  statement  of 
profits  for  the  year. 


Accounts  Payable 

Accounts  Receivable 

Accrued  Taxes 

Advertising 

Allowances 

Capital  Stock 

Cash 

Depreciation  of  Equipment 

Discount  on  Sales 

Discounts  Received 

Electricity 

Factory  Supplies  and  Expenses 

Freight  of  Purchases 

Income  from  Bonds 

Insurance 

Interest  Paid 

Inventory  December  31,  1914 

Investment  in  Bonds 

Notes  Payable,  secured  by  deposit  of  $15,000  of 

Bonds 

Office  Expenses 

Out  Freight 

Plant  and  Equipment 

Purchases 

Rent 

Repair  Parts 

Return  Sales 

Salaries,  Office  and  Officers 

Salaries,  Salesmen 

Salaries,  Shipping  Department 

Salary,  Superintendent 

Sales 

Salesmen's  Expenses 

Sales  of  Waste 

Surplus 

Taxes 

Unexpired  Insurance 

Wages  in  Factory 


D(bits 


$70,000 

3,000 
2,200 

15,000 
3,500 
3,100 

1,300 
3,600 
9,000 

300 

300 

50,000 

15,000 


900 
8,500 

40,000 

100,000 

2,500 

1,100 

1,000 

10,000 
6,000 
1,500 
2,000 

2,100 


400 

800 

15,000 


$368,100 


Credits 


$15,000 
1,200 

100,000 
1,800 
1,000 

10,000 


200,300 

1,200 
37,600 


$368,100 


The  inventory  at  December  31,  1915,  amounted  to  .+45,000. 

(111.,  Dec,  1916.) 


TRADING  ACCOUNTS  263 

3.  Smith,  Hill  and  Davis  form  a  ])aitiiei>liii)  under  an  airreement  that 
Smith  is  to  have  a  salary  of  $200 ;  Hill,  $150 ;  and  Davis,  $100  a  month, 
respectively.  The  profits  are  to  be  divided  in  proportion  to  the  amount 
of  business  secured  by  each  partner.  The  partners  are  to  be  individually 
responsible  for  any  direct  losses  arising  from  their  own  business. 

They  are  in  business  nine  months,  at  the  end  of  which  time  their  books 
state  as  follows:  Smith's  sales,  $5,310;  Hill's  sales,  $3,100;  Davis'  sales, 
$3,200;  net  profits,  $2,468.50. 

They  then  decide  to  rescind  the  salary  agreement,  treating  any  salary 
drawn  as  an  advance,  but  otherwise  to  divide  the  profits  according  to  the 
original  arrangement. 

You  find  errors  during  the  nine  months*  period,  namely :  office  furniture, 
charged  to  operation,  $65;  funds  lent  by  Davis,  credited  to  his  salary 
account,  $400 ;  and  open  items  not  entered  on  the  books  as  follows :  Smith's 
salary  (ninth  month),  $200;  Hill's  salary  (ninth  month),  $150;  advertis- 
ing, $27.50 ;  clerk  hire,  $130 ;  telephone,  $6 ;  rent,  $50 ;  stationery,  $15 ;  ac- 
counts receivable.  Smith's  business,  uncollectible,  $210;  and  that  the 
sales  represent  a  gross  profit  of  100  per  cent  over  cost  of  merchandise. 

State  the  Journal  entries  necessary  to  readjust  the  accounts  and  prepare 
a  corrected  Profit  and  Loss  account  and  a  statement  of  the  distribution  of 
the  profits. 

(Va.,  Nov.,  1910*;  A.  I.  of  A.,  May,  1918.) 

4.  "A"  and  "B"  were  copartners,  sharing  equally  the  net  profits  of 
the  business ;  "B"  had  exclusive  charge  of  the  business  and  of  the  accounts 
of  the  firm,  with  duty  of  furnishing  to  "A"  at  the  close  of  each  year  a 
statement  of  the  condition  of  the  firm's  accounts,  together  with  details 
of  the  Profit  and  Loss  account  for  the  year. 

According  to  statements  rendered  by  "B"  for  the  years  1900,  1901,  1902, 
the  following  conditions  of  account  appeared : 

On  January  1,  1900,  "A's"  capital  account  was  $135,000  and  the  in- 
ventory amounted  to  $175,000 ;  the  purchases  during  the  year  were  $950,000, 
the  sales  were  $1,175,000,  expenses  were  $110,000,  and  the  inventory, 
December  31,  amounted  to  $160,000. 

For  the  year  1901  the  inventory,  January  1,  was  $160,000,  the  purchases 
were  $875,000,  the  sales  were  $1,180,000,  the  expenses  were  $125,000,  and 
the  inventory,  December  31,  amounted  to  $150,000. 

For  the  year  1902  the  inventory,  January  1,  was  $150,000,  the  purchases 
were  $910,000,  the  sales  were  $1,210,000,  the  expenses  were  $130,000,  and 
the  inventory,  December  31,  amounted  to  $110,000. 

"A"  had  an  examination  made  of  the  books,  and  the  following  irregu- 
larities in  the  accounts  were  thereby  discovered : 

(1)  There  were  included  in  the  sales  for  1902  goods  at  an  estimated 
cost  of  $20,000,  which  fonned  no  part  of  the  inventories  or  purchases. 

(2)  Goods  were  omitted  from  the  inventories  of  the  several  years  as 
follows:  January  1,  1901,  valued  at  $30,000;  January  1,  1902,  valued  at 
$50,000 ;  January  1,  1903,  valued  at  $50,000. 

State  "A's"  Capital  account  for  the  several  years  as  it  would  appear 


264 


C.  P.  A.  ACCOUNTING 


from  the  statements  rendered  by  "B"  and  determine  and  state  what  modi- 
fication of  it  will  result  from  a  correction  of  the  above  irregularities,  giving 
the  correct  amount  of  his  capital  for  each  year. 

Give  details  of  process  by  which  you  reach  your  results. 

(Pa.,  May,  1905;  Wash.,  Sept.,  1907.) 

5.  The  board  of  directors  of  the  "X,"  "Y,"  "Z"  Company  removed  their 
manager  on  April  30,  1915,  on  the  general  suspicion  that  his  books  mis- 
represented the  true  financial  condition  of  the  business.  Prepare  a  state- 
ment showing  the  nature  and  probable  extent  of  his  misrepresentation, 
also  an  approximate  Statement  of  the  Profit  and  Loss  for  the  four  months 
ending  April  30,  1915,  and  a  Balance  Sheet  as  of  April  30,  1915. 

The  following  is  a  trial  balance  taken  from  the  books  April  30,  1915 : 


Capital  Stock 

Fixtures 

Inventory,  Januarv  1,  1915 

Cash 

Accounts  Receivable 

Accounts  Payable 

Loans  Payable 

Sales 

Purchases 

Salaries,  Salesmen 

Advertising 

Salaries,  Office 

Rent 

Interest 

Insurance,  January  1  to  December  31,  1915 

Stationery  and  Printing 

Reserve  for  Depreciation  of  Fixtures 

Surplus,  January  1,  1915 


$10,000 

128,600 
15,450 
24,600 


40,700 

2,200 

1,650 

1,100 

400 

200 

999 

105 


$226,004 


$75,000 


39,000 
10,000 
51,000 


2,710 
48,294 


$226,004 


An  analysis  of  the  Purchases  and  Sales  accounts  revealed  the  follow- 
ing: purchases,  year  1912,  $122,000;  sales,  year  1912,  $153,750;  inventory, 
January  1,  1912,  $101,000;  purchases,  year  1913,  $123,000;  sales,  year 

1913,  $153,170;  inventory,  January  1,  1913,  $100,000;   purchases,  year 

1914,  $121,000;  sales,  year  1914,  $154,722;  inventorv,  Januarv  1.  1914, 
$102,000.  (N.  Y.,  Jan^,  1915.) 

6.     The  Good  Music  Company  sells  pianos  on  the  installment  basis. 

On  January  2,  1914,  Jones  purchases  a  piano  from  the  company  for 
$375,  to  be  paid  for  as  follows :  $25  down  and  the  balance  in  quarterly  in- 
stallments of  $50  each,  bill  of  sale  to  be  given  on  date  of  final  payment. 
The  piano  cost  the  company  $125.  The  four  installments  for  1914  were 
duly  received,  the  last  having  been  paid  on  December  31. 

(o)  Set  up  the  proper  Ledger  accounts  covering  this  sale  and  the 
payments  thereon. 

(b)  Give  the  Journal  entry  (at  the  close  of  the  year)  by  which  the 
year  will  be  credited  with  its  proper  proportion  of  the  profit  on  this 
transaction.  (Wis.,   April,  1915.) 


TRADING  ACCOUNTS 


265 


7.  From  the  following  data  (a)  prepare  one  Trading  account  for  the 
years  1916  and  1917,  stating  the  gross  profit  for  the  two  years;  (h)  pre- 
pare a  Trading  account  with  estimated  inventory  for  1918: 


Sales  1916 $290,696 .81 

Sales  1917 292,548.21 

Purchases  1916 241,709 .33 

Purchases  1917 253,791 .95 

Sales  discounted  1916  . .  .  1,238 .55 

Sales  discounted  1917  . .  .  1,955 .26 

Freight  in  1916 3,384 .35 

Freight  in  1917 4,094 .70 

Inventory  Jan.  1,  1916  .  .  63,784.78 

Inventory  Dec.  31,  1917.  107,317.80 


Sales  nominal $493,476.87 

Sales  in  carload  lots 39,351 .68 

Return  sales 15,304 .58 

Sales  discounts 4,267 .30 

Purchases 443,685 .  15 

Freight  in 6,211.53 

Gross  gain  on  carload  lots .  1,956.42 


(N.  Y.,  June,  1919.) 

8.  The  office  of  a  firm  of  traders,  doing  business  in  San  Francisco, 
was  destroyed  by  an  earthquake.  The  books  of  account,  which  had  been 
fully  posted,  were  badly  damaged.  The  following  Ledger  accounts  were 
found  to  be  legible:  Purchases,  net,  $69,000;  Discounts  lost,  $640;  Dis- 
counts gained,  $3,450;  Sales,  $54,000;  Bills  Receivable,  $33,000. 

Inquiry  at  the  bank  disclosed  a  balance  on  deposit,  $129,000.  Bills 
receivable  amounting  to  $45,000  had  been  discounted  at  the  bank.  An 
audit  of  the  checks  paid  by  the  bank  showed  that  $99,000  had  been  paid 
creditors  (including  $60,000  notes  payable). 

A  Balance  Sheet  prepared  at  the  last  closing  of  the  books  was  produced 
containing  the  following  items :  Cash,  $60,000 ;  Accounts  Receivable,  $126,- 
000;  Loans  Receivable,  $24,000;  Real  Estate,  $90,000;  Notes  Receivable, 
$78,000 ;  Capital,  $318,000 ;  Notes  Payable,  $60,000. 

Prepare  a  trial  balance  supplying  the  missing  accounts. 

(N.  Y.,  June,  1915.) 


CHAPTER  XIV 
INSURANCE  AND  GENERAL  EXPENSE 

Unexpired  Insurance — As  fire  in.surance  premiums  are  paid  in  advance 
for  services  extending  over  a  duration  of  time,  unexpired  insurance  is  a 
deferred  charge  to  income.*  In  case  of  cancellation  of  a  policy,  the  pre- 
mium for  the  period  when  the  insurance  was  in  force  would  he  figured  upon 
a  "short  rate,"  which  is  considerably  higher  than  the  usual  rate,  and  the 
excess  of  the  unexpired  actual  premium  over  this  "short  rate"  premium 
would  be  returned  by  the  insurance  company.^  For  financial  statement 
purposes  unexpired  insurance  is  valued  on  the  pro  rata  basis,  except  in 
case  of  bankruptcy  or  in  the  statement  of  affairs,  when  the  "short  rate" 
basis  would  apply.' 

In  the  case  of  a  fire,  that  proportion  of  the  imexpired  insurance,  which 
will  represent  the  ratio  of  the  amount  of  the  settlement  made  by  the  insur- 
ance company  to  the  face  of  the  policy,  should  be  charged  ott"  to  the  fire 
loss  account.* 

In  order  to  include  insurance  in  monthly  statements,  it  is  advisable  to 
debit  unexpired  insurance  or  prepaid  insurance  Avhen  the  premiums  are 
paid,  and  then  to  write  that  account  off  to  expense  (or  insurance)  monthly." 

Fire  Insurance  Register — "When  there  are  numerous  insurance  policies, 
an  insurance  register  may  be  used,  which  will  record  the  policy  numbers, 
names  of  companies,  dates  of  policies,  dates  of  expiration,  natui'e  and 
amount  of  insurance,  and  amounts  of  premiums.'  The  insurance  register 
contains  twelve  columns  in  which  the  insurance  expense  applicable  to  each 
month  is  recorded.'  The  use  of  an  insurance  register  greatly  aids  the 
auditor  in  verifjdng  the  unexpired  insurance.* 

Beserve  for  Insurance — If  a  large  firm  desires  to  carry  its  own  insurance, 
it  makes  an  annual  charge,  equivalent  perhaps  to  regular  insurance  pre- 
miums, to  profit  and  loss,  and  credits  a  reserve  for  insurance.'  Any  fire 
losses  would  be  written  against  this  reserve."  In  case  of  liquidation,  the 
reserve  is  a  part  of  profits,'*^  but  going  concerns  must  maintain  the  reserve, 
although  they  should  show  it  on  the  balance  sheet  as  a  surplus  reserve. 
If  the  reserve  increases  until  it  is  out  of  proportion  to  the  fire  risk,  it  may 
be  written  down  directly  into  surplus."  Frequently  the  insurance  reserve 
is  funded  in  order  to  insure  the  possession  of  liquid  assets  in  case  of  a  fire." 

Audit  of  Fire  Insurance — The  auditor  should  call  for  all  fire  insurance 
policies  for  the  purpose  of  assuring  himself  that  adequate  insurance  is 
carried"  and  that  the  policies  are  in  the  company's  name."    The  policies 


'For  explanation  of  superior  figures  see  page  337. 

266 


INSURANCE  AND  GENERAL  EXPENSE  267 

will  also  furnish  additional  proof  of  the  properties  not  having  been 
hypothecated."  The  auditor  should  examine  the  coinsurance  and  other 
restrictive  clauses  of  the  policies."  All  canceled  policies  should  be  noted 
and  the  return  premium  verified."  If  no  register  is  kept,  a  detailed 
schedule  of  unexpired  policies  should  be  prepared."  The  auditor  should 
ascertain  that  the  policies  properly  describe  and  locate  the  insured  prop- 
erty, and  that  mortgage  clauses  are  attached,  if  the  property  is  mortgaged.** 
Coinsurance  Clause — An  80%  coinsurance  clause,  sometimes  called  the 
average  clause,  makes  the  insured  a  coinsurer  with  the  insurance  company 
for  the  difference  between  80%  of  the  cash  value  of  the  property  and  the 
face  of  the  policy."  Let  "P"  represent  the  face  of  the  policy;  "L,"  the 
amount  of  the  loss;  "C,"  the  cash  value  of  the  pi'operty;  and  "A,"  the 
amount  that  may  be  recovered  from  the  insurance  company;  then,  in 
accordance  with  the  provisions  of  the  coinsurance  clause," 

PL 


.8C 

In  using  this  formula,  it  must  be  borne  in  mind  that  the  insurance  company 
is  never  liable  for  more  than  the  face  of  the  policy." 

Fire  Losses — After  a  fire,  a  fire  loss  account  should  be  charged  with  the 
full  amount  of  the  loss,  including  related  expenses  and  canceled  unexpired 
insurance,  and  credited  with  the  allowance  made  by  the  insurance  com- 
pany." The  balance  of  the  account  represents  the  net  loss  or  gain  due  to 
the  fire  and  should  be  closed  directly  into  surplus.*" 

Depreciation  accruing  from  close  of  last  fiscal  period  to  date  of  fire 
should  be  debited  to  the  depreciation  expense  account  and  credited  to  the 
related  asset  account.*'  The  portion  of  the  depreciation  reserve  applicable 
to  the  destroyed  asset  should  be  written  off  to  the  related  asset  account, 
and  then  the  balance  of  the  asset  account  should  be  closed  into  the  fire 
loss  account."  A  point  in  theory,  usually  disregarded  in  practice,  is  to 
adjust  any  difference  between  the  book  value  and  the  valuation  allowed 
by  the  insurance  company  directly  to  surplus  as  an  item  occasioned  by 
faulty  depreciation  allowances  in  foi'mer  periods.** 

When  perpetual  inventories  are  not  maintained,  losses  in  merchandise 
are  estimated  on  the  assumption  that  the  gross  profit  for  the  period  in 
which  the  fire  occurred  would  be  the  same  percentage  as  normal.*'  The 
procedure  is  that  used  in  the  "gross  profit  check"  on  inventories  (see 
page  244). 

Marine  Insurance — Professor  Solomon  S.  Huebner  in  "Property  Insur- 
ance" states  that  the  law  of  general  average  covers  all  those  losses  which 
result  from  the  sacrifice  of  any  interest  voluntarily  and  deliberately  made 
by  the  master  of  a  vessel  in  time  of  distress  for  the  common  safety  of  the 
ship,  cargo,  and  freight.  Such  losses  must  be  repaid  proportionately  by 
all  the  parties  benefited.  Assuming  a  loss  of  $5,000  when  the  ship,  cargo, 
and  freight  were  valued  at  $50,000,  $25,000,  and  $1,000,  respectively,  the 
loss  would  be  borne  in  the  ratio  50:25:1.    It  will  be  noted  that  jettisoned 


268  C.  P.  A.  ACCOUNTING 

property  bears  its  share  along  with  the  saved  property.  An  insurance 
company  pays  such  losses  in  the  proportion  which  the  insured  value  of 
the  property  bears  to  its  contributory  value. 

Life  Insurance — Premiums  on  straight  life  insurance  policies  of  which 
the  firm  is  the  beneficiary  may  be  capitalized  at  their  cash  surrender  value,^ 
the  excess  of  the  premiums  over  the  increase  in  the  cash  surrender  value  of 
the  policies  being  charged  to  operating  expense."  In  case  of  the  death  of 
the  insured,  the  surrender  value  account  would  be  closed  and  the  excess 
of  the  payment  over  the  surrender  value  would  be  credited  to  surplus." 

Employers'  Liability  Insurance — All  premiums  paid  to  insurance  com- 
panies for  insurance  against  claims  for  damages  for  injimes  to  employees 
are  proper  charges  to  factory  burden.^'  As  the  premiums  are  paid  in  ad- 
vance, the  unexpired  insurance  must  be  shown  as  an  asset  at  the  end  of 
a  fiscal  period.'^  The  auditor  should  secure  a  statement  from  the  firm's 
attorney's  as  to  pending  claims  and  suits,  and  should  provide  reserves  for 
possible  liabilities.^ 

Burglary  Insurance — Premiums  for  burglary  insurance  are  charged  to 
factory  burden  or  administrative  expense,  depending  on  whether  the 
property  so  safeguarded  belonged  to  the  manufacturing  or  sales  branch  of 
the  business.  Premiums  paid  for  the  bonding  of  employees  are  usually 
regarded  as  administrative  expense.^ 

Accrued  and  Prepaid  Wages — An  auditor  must  ascei'tain  that  the  ac- 
crued wages  at  both  the  beginning  and  end  of  the  fiscal  period  are  reflected 
in  the  accounts."  Under  ordinary  conditions,  accrued  wages  are  calculated 
on  the  fractional  basis  rather  than  by  reference  to  each  individual  time 
card."  Prepayments  of  wages  are  current  assets,  but,  although  they  are 
personal  accounts,  they  must  not  be  included  among  the  accounts  re- 
ceivable." 

Payroll  Book — The  payroll  book  is  a  subsidiary  book  to  the  general 
cash  book,  containing  the  names  of  employees  and  the  payments  made  to 
each.*"  In  its  form  and  content  there  is  little  uniformity.  Sometimes  it 
lists  all  the  employees,"  and  sometimes  it  omits  those  working  for  wages." 
It  may  or  may  not  contain  the  signatures  of  the  employees  for  each  pay- 
ment." The  pajToll  book  is  sometimes  merely  a  memorandum  record,  and 
sometimes  a  book  of  original  entry." 

Audit  of  Payroll — The  extent  of  an  audit  of  the  payrolls  depends  on  the 
system  of  internal  check,"  a  subject  treated  in  Chapter  I.  The  auditor 
should  call  for  all  the  payrolls,  which  should  be  signed  by  the  officials 
responsible  therefor."  Unsigned  payrolls  should  be  carefully  compared 
with  those  signed.  When  salaries  are  paid  by  checks,  the  pa\Toll  book 
should  be  thoroughly  tested  with  the  canceled  checks.*'  The  auditor  should 
verify  the  footings  of  the  payroll  book  of,  say,  every  third  week."*  Where 
employment  cards  are  kept,  they  should  be  used  to  vouch  the  payrolls." 
Accrued  wages  must  be  verified."  The  auditor  should  suggest  all  needed 
improvements  in  the  payroll  system."^  Officers'  salaries  should  be  verified 
from  the  directors'  minutes."" 


INSURANCE  AND  GENERAL  EXPENSE  269 

Insurance,  Bent,  and  Taxes  as  Costs — Most  accountants  hold  that  ex- 
penses incurred  for  insurance,  rent,  and  taxes  should  be  prorated  between 
factory  expense  and  administrative  expense  in  proportion  to  the  capital 
invested  in  the  factory  and  office  departments."  Others  view  insurance 
and  taxes  as  payments  made  for  the  protection  of  capital,  and  rent  as  an 
expense  due  to  lack  of  capital,  and  therefore  treat  them  as  non-operating 
expenses." 

Income  and  excess  profits  taxes  are  levied  on  the  assumption  that  the 
government  is  a  partner  in  the  business,  i.  e.,  that  the  government  shares 
in  the  profits  in  return  for  benefits  conferred  on  the  business  by  the  gov- 
ernment.'°  They  should,  therefore,  be  charged  directly  to  surplus^*  and 
shown  in  the  appropriation  of  profits  section  of  the  profit  and  loss  state- 
ment." This  procedure  is  not  universally  followed,  for  income  taxes  are 
sometimes  seen  classed  as  administrative  expense'*  and  sometimes  as 
financial  expense.'" 

Assessment  taxes  for  improvements  are  not  expenses  but  outlays  in- 
creasing the  value  of  the  property  taxed,  and  they  should,  therefore,  be 
charged  to  the  related  asset  account.'" 

Rent  as  Income — When  rent  is  received  on  property  owned,  it  is  non- 
operating  income;"*  but  if  the  I'ent-bearing  property  is  a  part  of  property 
which  the  firm  itself  is  leasing,  the  rent  received  may  be  deducted  from  the 
rent  paid/^  or  the  rent  paid  may  be  allocated  to  operating  or  non-operating 
expense,  and  all  of  the  rent  received  treated  as  non-operating  income.*" 
The  latter  procedure  gives  the  truer  costs. 

Postage — In  most  audits,  postage  is  unimportant  financially,  but,  as  it 
is  a  fi'equent  source  of  petty  theft,  the  auditor  should  scrutinize  carefully 
the  paj^ments  for  postage,  and,  if  possible,  suggest  safeguards  which  will 
reduce  the  future  losses  and  at  the  same  time  remove  a  source  of  serious 
temptation  to  junior  clerks."  Postage  is  usually  treated  as  a  general  and 
administrative  expense,"  although  it  may  be  allocated  to  the  various  de- 
partments of  the  business. 

Donations — Donations,  from  which  an  indirect  benefit  is  not  expected, 
should  be  charged  directly  to  surplus  and  shown  in  the  appropriation  of 
profits  section  of  the  profit  and  loss  statement,^  but  donations  incurring 
indirect  benefits  are  administrative  expenses." 

Adjustment  Entries — Entries  required  to  bring  the  ledger  into  accord 
with  actual  facts  ai*e  called  adjustment  entries."*  There  are  five  kinds  of 
adjustment  entries,  namely,  those  pertaining  to  (a)  merchandise  inven- 
tories, (b)  valuation  reserves,  (c)  deferred  assets  and  liabilities,  (d)  ac- 
crued assets  and  liabilities,  and  (e)  the  correction  of  errors. 

Closing  Entries — Closing  entries  are  those  made  for  the  purpose  of 
closing  the  nominal  accounts  into  the  vested  proprietorship  accounts.** 
Some  accountants  use  the  manufacturing  and  trading  accounts  in  addition 
to  the  profit  and  loss  account,'"  a  practice  which  seems  inadvisable  as  the 
modern  use  of  analytical  statements  makes  the  opening  of  manufacturing 
and  trading  accounts  unnecessary.     Some  accountants  close  the  subsidiary 


270  C.  P.  A.  ACCOUNTING 

sales  and  subsidiary  purchase  accounts  into  the  sales  account  and  purchase 
account,  respectively,  before  closing  these  accounts  into  the  profit  and  loss 
account.'*  Other  accountants  close  all  nominal  accounts  directly  into  profit 
and  loss,"  a  short-cut  which  seems  warranted  by  the  fact  that  the  analysis 
of  accounts  is  usually  studied  from  the  financial  statements  instead  of  from 
the  ledger. 

Accountants  prefer  the  journal  method  of  closing  to  the  direct  or  "red 
ink"  method  because  it  gives  a  complete  record  of  transfers  in  the  books  of 
original  entry,"  and  because  it  collects  all  the  closing  entries  in  one  place.'* 

The  auditor  should  examine  the  closing  entries  in  order  to  prevent  a 
misstatement  of  income." 


INSURANCE  AND  GENERAL  EXPENSE       271 


QUESTIONS 

insurance  and  general  expense 

Fire  Insurance 

1.  In  preparing  a  statement  of  the  cost  of  making  the  product  would 
you  include  as  cost,  expenses  of  the  fire  insurance  premiums  on  the  fire 
policies  eoverins:  the  plant?  State  your  reason.  (N.  C,  June,  1919; 
N.  C,  Sept.,  1919.) 

2.  In  order  to  facilitate  the  preparation  of  monthly  Profit  and  Loss 
statements  by  a  corporation,  how  would  you  recommend  insurance  to  be 
treated  on  its  books  from  month  to  month?     (Wash.,  April,  1906.) 

3.  State  what  is  indicated  by  the  Insurance  account  (a)  when  the 
account  shows  a  debit  balance,  {b)  when  the  account  shows  a  credit  bal- 
ance.    Explain  fully.     (N.  Y.,  Jan.,  1900.) 

4.  What  in  your  opinion  is  the  proper  treatment  of  the  Unexpired 
Insurance  Premiums  account,  in  arriving  at  the  profit  and  loss  of  a  busi- 
ness for  a  specific  fiscal  period?     (Ohio,  Dec.,  1908;  Mass.,  June,  1910.*) 

5.  On  which  side  of  the  Ledger  should  the  balance  on  the  Insuranct. 
Premiums  Unexpired  account  appear?     (Mo.,  Dec,  1914.) 

6.  Would  you  consider  it  proper  to  include  Insurance  Premium  Un- 
earned as  an  asset?  Explain  briefly  and  give  reasons.  (N.  Y.,  Jan.,  1897; 
N.  Y.,  June,  1908.) 

7.  How  would  you  treat  insurance  premiums  paid  in  advance,  if  clos- 
ing the  books  at  a  certain  period,  with  a  view  to  ascertaining  the  loss  or 
gain?     (N.  J.,  1904-1909.) 

8.  Classify  the  following  account :  Prepaid  Insurance.  (Mass.,  April, 
1911*;  Mass.,  June,  1912*;  Iowa,  Dec,  1918.) 

9.  A  concern  owning  a  fleet  of  twenty  vessels  decides  to  carry  its  own 
insurance.  How  will  this  be  dealt  with  on  the  books,  and  what  entries 
would  you  make  at  the  close  of  each  fiscal  vear?  (111.,  May,  1904;  Cal., 
Nov.,  191C.*) 

To.  A  corporation  invests  its  capital  in  a  number  of  subsidiary  com- 
panies, each  subsidiary  company  having  a  distinct  organization,  but  its 
dividends  being  payable  to  the  parent  company.  The  parent  company 
decides  to  carry  insurance  for  all  the  subsidiai-y  companies  and  each  pays 
in  to  the  parent  company  monthly  a  specific  sum.  Fire  losses  as  in- 
curred are  payable  by  the  parent  company.  How  are  such  entries  treated 
and  what  should  the  books  show ;  also,  what  entries  are  necessary  at  close 
of  fiscal  years?     (Mich.,  Dec,  1900.) 


272  C.  P.  A.  ACCOUNTING 

11.  The  Insurance  account  as  kept  upon  the  books  of  the  Good  Mer- 
chandise Company  is  charged  witli  the  premiums  paid  on  the  following 
kinds  of  insurance:  Fire  insurance  on  buildings,  merchandise  and  fix- 
tures; sprinkler  leakage;  employer's  guarantee  bond;  safe  burglary;  rob- 
bery and  hold-up;  automobile  fire;  theft  and  liability;  general  liability; 
elevator  liability;  steam  boiler;  tornado;  plate  glass;  use  and  occupancy; 
insurance  on  officers'  lives. 

You  are  asked  to  indicate  the  proper  treatment  to  be  given  each  of  the 
above  items;  i.e.,  indicate  the  name  of  the  account  or  accounts  to  which 
they  should  be  charged,  give  the  adjusting  entries,  state  the  section  of  the 
revenue  account  or  income  statement  in  which  each  would  appear,  etc. 

(Wis.,  Nov.,  1919.) 

12.  A  certain  concern  decides  that  instead  of  paying  insurance  pre- 
miums it  will  establish  its  own  insurance  fund,  depositing  therein  an 
amount  equal  to  the  premiums  which  it  would  otherwise  have  paid.  The 
iasurance  fund  is  invested  in  certificates  of  deposit  bearing  3  per  cent 
interest. 

Using  imaginary  figures,  construct  (a)  Journal  entries  covering  the 
insurance  fund  transactions  as  above  described  for  one  year;  and  (b) 
the  Journal  entries  required  in  the  event  of  a  fire  loss  which  the  said  fund 
does  not  fully  cover.  (Wash.,  May,  1910.) 

13.  Give  some  reasons  why  the  professional  auditor  should,  under 
present-day  conditions,  give  even  more  attention  than  in  the  past  to  in- 
surance carried.     (A.  I.  of  A.,  May,  1920.) 

14.  A  corporation  manufacturing  explosives  was  compelled  to  pay 
exorbitant  rates  for  a  very  limited  amount  of  insurance,  and  in  conse- 
quence was  obliged  to  install  an  automatic  sprinkler  system  at  a  cost  of 
$75,000.  This  additional  fire  protection  enabled  it  to  secure  a  full  line 
of  insurance,  though  in  mutual  companies  and  at  a  much  lower  rate  than 
was  obtainable  prior  to  such  installation.  At  the  end  of  the  fiscal  year 
the  company  received  dividends  from  these  mutual  insurance  companies 
aggregating  $2,000.  To  what  account  should  the  cost  of  the  sprinkler 
system  be  charged  and  to  what  account  should  this  dividend  be  credited? 
State  your  reasons  fully.     (N.  Y.,  June,  1915;  N.  Y.,  Jan.,  1920.*) 

15.  Assuming  that  you  are  a  certified  public  accountant  and  employed 
as  auditor  by  a  corporation,  state  what  you  would  consider  it  your  duty 
to  do  in  order  to  safeguard  your  clients  regarding  the  fire  insurance  that 
they  carry  on  their  stock  of  merchandise.  (N.  Y.,  June,  1912;  N.  Y., 
June,  1915.) 

16.  What  notice,  if  any,  should  an  auditor  take  of  the  fact  that  the 
client  has,  at  risk  of  loss  by  fire,  property  on  which  no  insurance  is  car- 
ried? 

What  is  the  duty  of  an  auditor  to  his  clients  regarding  the  amount  of 
insurance  that  they  carry  on  their  stock  of  merchandise? 

(Wis.,  April,  1915.) 

17.  Define:  Coinsurance.     (Wis.,  May,  1916;  A.  I.  of  A.,  May,  1921.)  . 


INSURANCE  AND  GENERAL  EXPENSE  273 

18.  Describe  the  practical  application  of  the  coinsurance  clause.  (Pa., 
Mny,  1906;  Mich.,  July,  1909.*) 

19.  Describe  the  practical  application  of  (a)  the  average  clause;  (&) 
the  three-fourths  value  clause;  (c)  the  three-fourths  loss  clause;  (d)  the 
use  and  occupancy  form,  in  the  insurance  business.     (Mich.,  July,  1909.) 

20.  A  company  shows  among  its  assets  $2,675  as  unexpired  insurance 
on  January  1,  1907.  On  February  1,  1907,  the  plant  is  destroyed  by  fire 
and  a  total  loss  of  $57,875  occurs,  which  the  insurance  company  pays. 
How  would  you  treat  the  $2,675  unexpired  insurance  item?  (111.,  May, 
1909;  Mass.,  Oct.,  1916.*) 

21.  How  would  you  determine  the  loss  in  case  of  a  fire  if  the  books 
had  been  saved  but  were  not  kept  so  as  to  show  a  perpetual  inventory? 
(Pa.,  May,  1900*;  N.  Y.,  June,  1902*;  Pa.,  May,  1903*;  111.,  May,  1905*; 
Pa.,  Nov.,  1906*;  Mich.,  Dec,  1906*;  Wash.,  Aug.,  1908*;  Md.,  Jan., 
1909*;  Mich.,  July,  1909*;  111.,  May,  1910*;  Va.,  Nov.,  1910*;  Mass,, 
June,  1913*;  Mich.,  Dec,  1913;  Mass.,  Oct.,  1914*;  Wis.,  April,  1915*; 
Ohio,  Nov.,  1915*;  Ind.,  June,  1916*;  Mich.,  Dec,  1916*;  A.  I.  of  A., 
June,  1917*;  Cal.,  June,  1917;  Ind.,  May,  1918*;  Md.,  Oct.,  1919*;  A.  I. 
of  A.,  May,  1921.*) 

22.  What  methods  would  you  adopt  to  determine  the  value  of  stock 
or  merchandise  on  hand  at  an  odd  date  in  case  of  total  loss  by  fire  in  a 
manufacturing  business,  under  each  of  the  following  several  conditions: 
(1)  Inventories  taken  in  detail  annually  on  closing  only?  (2)  Running 
inventories  kept  on  cards  by  quantities  only,  except  as  to  work  in  progress 
and  under  a  system  of  estimating  costs  for  finished  product?  (3)  Run- 
ning inventories  kept  on  Ledgers  according  to  true  cost  in  value  and 
quantities,  but  not  posted  for  three  months  back,  and  posting  media 
destroyed,  through  general  books  intact?     (Mich.,  June,  1914.) 

23.  State  in  the  form  of  Journal  entry  the  following  transaction :  Loss 
by  fire  of  buildings,  fixtures  and  merchandise;  loss  sustained  by  owner 
over  and  above  the  insurance  carried,  and  the  amounts  due  and  collected 
from  the  insurance  companies.     (111.,  Nov.,  1903.) 

24.  Draft  a  form  of  statement  you  would  prepare  to  support  a  claim 
for  loss  by  fire  which  was  suffered  by  a  retail  dry  goods  store,  six  months 
since  an  inventory  was  taken  and  allowing  for  all  depreciation,  discounts, 
and  freights.  A  portion  of  the  stock  was  not  damaged,  a  portion  was 
damaged  to  some  extent  and  the  remainder  was  entirely  destroyed. 
(Wash.,  Aug.,  1908*;  Fla.,  July,  1909.) 

25.  A  clothing  store  carries  fire  insurance  to  the  amount  of  $30,000  on 
a  stock  of  $50,000.  The  policies  contain  the  80  per  cent  coinsurance 
clause.  State  (a)  the  amount  collectible  if  a  partial  loss  of  $15,000  was 
suffered  and  (b)  the  amount  collectible  if  a  total  loss  occurred,  (c)  If 
you  believe  that  a  merchant  should  carry  100  per  cent  insurance  protection 
upon  his  property,  draft  the  entry  or  entries  to  record  the  annual  insur- 
ance charge.  Assume  that  it  is  only  necessary  for  him  to  carry  80  per 
cent  in  outside  companies  to  take  advantage  of  the  coinsurance  clause. 
(Wis.,  April,  1918.) 


274  C.  P.  A.  ACCOUNTING 

26.  What  is  the  diflferenee,  theoretieall.y,  between  expenditures  by  a 
manufacturing  concern  for  fire  and  burglar  insurance,  health  and  accident 
insurance  and  watchmen?  In  which  section  of  a  Profit  and  Loss  account 
should  each  item  be  allocated?     (N.  Y.,  June,  1917.) 

27.  An  audit  discloses  payments  for  insurance  during  a  year  amountino: 
to  $1,600,  which  is  api3roximately  a  normal  yearly  expenditure,  and  the 
unexpired  insurance,  unadjusted,  amounts  to  $600.  No  account  was 
raised  for  the  unexpired  insurance  at  the  beginning  of  the  year.  What 
adjustment  should  be  made  on  the  books  and  how  made?  (Ohio,  Nov.. 
1913.) 

28.  The  Ledger  of  a  manufacturing  corporation  contains  a  Fire  In- 
surance Fund  account.  The  treasurer  submits,  for  credit  purposes,  a 
statement  showing,  as  "Surplus  Funds,"  the  sum  of  the  amount  at  the 
credit  of  the  Insurance  Fund  account,  and  of  the  amount  at  the  credit  of 
the  "Surplus"  account.  State  (n)  your  comments  thereon;  (6)  the 
reasons  supporting  your  answei*.     (Mass.,  June,  1912.) 

29.  Under  what  circumstances  would  you  permit  the  inclusion  of 
profits  of  excess  amount  recovered  from  an  insurance  company  over  the 
book  value  of  a  plant  destroyed  by  fire?     (Mo.,  Dec,  1914.) 

30.  A  fire  in  a  manufacturing  concern  resulted  in  a  loss  on  machinery, 
$5,000;  merchandise  (raw  material),  $10,000;  manufactured  goods,  $25,- 
000 ;  which  amount  of  $40,000  was  agreed  upon  and  paid  by  the  insurance 
companies.  Give  the  entries  necessary  to  record  properly  the  above  trans- 
actions on  the  books  of  the  concern.     (N.  Y.,  June,  1914.) 

Life  Insurance 

31.  A  corporation  authorized  the  secureraent  of  life  insurance  of  its 
treasurer  for  $50,000  payable  at  death  to  the  corporation.  How  should 
this  subject  be  treated  in  the  accounts?     (Mass.,  Oct.,  1914.) 

32.  How  would  you  classify  the  following  item  in  the  Balance  Sheet 
or  Profit  and  Loss  account :  Premium  on  Life  Assurance  Policy  on  which 
there  is  a  surrender  value?     (Kan.,  Dec.,  1915;  Mo.,  Dec.,  1915.) 

33.  A  company  has  insured  the  life  of  its  president  for  its  own  benefit, 
and  is  carrying  the  amount  of  premium  paid  in  its  Balance  Sheet.  What 
position  would  you  as  an  auditor  take  in  regard  tp  these  premiums?  (Ind., 
May,  1917;  A.*L  of  A.,  Nov.,  1917.) 

34.  The  partnership.  Black  &  White,  has  insured  the  lives  of  its 
partners  for  equal  amounts.  The  policies  are  payable  to  the  firm.  Pre- 
miums have  been  paid  for  five  years,  (a)  Show  the  annual  entries  for  each 
of  the  five  years.  At  the  end  of  the  fifth  year  White  dies.  (6)  What 
would  be  the  proper  entries  to  make  upon  receij)t  of  the  amount  of  the 
policy?     (Wis.,  April,  1915;  Wash.,  June,  1915.*) 

35.  A  firm  of  jobbers,  Shea,  Kargeau  and  Company,  to  provide  against 
financial  strain  in  the  event  of  the  death  of  one  of  the  partners,  takes  out 
a  joint  life  insurance  policy  for  $15,000,  the  annual  i)remium  ($750)  being 


INSURANCE  AND  GENERAL  EXPENSE       275 

charged  as  a  feature  of  the  Profit  and  Loss  account.  Eight  years  there- 
after the  junior  partner  dies;  assuming  that  they  are  equal  partners,  you 
are  called  in  to  adjust  the  accounts  upon  receipt  of  the  insurance  money. 
Explain  the  method  you  would  follow.     (Ill,,  May,  1912.) 

36.  It  is  now  becoming  quite  the  common  practice  for  corporations  to 
insure  the  lives  of  their  principal  officers,  so  that  upon  their  deaths  the 
corporations  may  be  in  a  measure  reimbursed  for  their  loss  to  the  business. 
In  this  connection  you  are  asked  to  indicate  what  sort  of  entries  would  be 
made  by  a  company  from  time  to  time  if  it  paid  the  insurance  for  $50,000 
carried  on  the  life  of  its  president  under  the  four  classes  of  insurance 
policies  indicated  below: 

10-year  renewable  term  policy. 
20-payment  life  policy. 
Straight  life  policy. 
20-year  endowment  policy. 
Also  indicate  what  entries  should  be  made  in  the  books  for  the  receipt 
of  the  $50,000  principal,  of  the  different  classes  of  policies  supposing  the 
president  of  the  company  died  during,  say,  the  fifth  year  of  the  insured 
term. 

(Wash.,  July,  1917;  A.  I.  of  A.,  Nov.,  1920.) 

Payroll 

37.  Classify  the  following  accounts :  (a)  Payroll;  (b)  officers' salaries; 
(c)  bookkeepers'  salaries.     (Iowa,  Dec,  1918.) 

38.  Define:  Payroll  Bank  account.     (Wash.,  July,  1917.) 

39.  Define:  Payroll  advances.     (Wash.,  June,  1915.) 

40.  Classify  the  accounts  properly  recording  the  following  items  ac- 
cording to  the  subdivision  of  income  and  expenses,  assets,  liabilities,  pro- 
prietary interest,  under  which  they  should  be  grouped:  (a)  Wages  paid 
workmen  and  office  staff;  (&)  wages  due  workmen  and  office  staff.  (Wis., 
May,  1919.) 

41.  What  plan  would  you  suggest  for  minimizing  the  risk  of  fraud  in 
the  payment  of  wages  in  a  factory  where  large  numbers  of  men  are  em- 
ployed partly  by  piece-work  and  partly  at  fixed  wages?  (N.  Y.,  June, 
1899*;  Pa.,  Nov.,  1906;  Mass.,  Oct.,  1915*;  Pa.,  Nov.,  1916*;  111.,  Dec., 
1916.*) 

42.  In  handling  large  factory  payrolls,  which  do  you  consider  the 
better  practice  for  the  prevention  of  fraud,  the  taking  of  receipts  from 
each  employee  for  the  amounts  paid  or  the  establishment  of  a  good  system 
of  accounting  for  handling  payrolls.  Give  reasons  and  explain  why  you 
think  one  method  is  better  than  the  other.     (N.  Y.,  Feb.,  1909.) 

43.  Classify:  Night  watchman  wages.     (Iowa,  Dec,  1918.) 

44.  Would  you  consider  wages  in  a  manufacturing  concern  to  be  a 
nominal  account,  or,  if  not,  how  should  it  be  classified?  (Minn.,  Oct., 
1918.) 


276  C.  P.  A.  ACCOUNTING 

45.  In  what  section  of  the  Balance  Sheet  and  in  what  order  would 
you  show  wages?     (A.  I.  of  A.,  Nov.,  1919.) 

46.  An  accountant  is  employed  to  audit  the  personal  accounts  of  a 
retired  capitalist  who  maintains  several  estates.  What  data  is  required 
concerning  the  employees  to  form  an  important  schedule  of  the  report? 
(N.  Y.,  Jan.,  1914.) 

47.  Give  some  methods  used  by  dishonest  timekeepers  and  foremen  to 
falsify  records.     (Pa.,  Nov.,  1900.) 

48.  To  what  extent  do  you  consider  it  necessary  to  verify  the  payrolls? 
(N.  Y.,  Dec,  1896*;  Va.,  Nov.,  1910;  Mich.,  June.  1912*;  111.,  May,  1914*; 
Pa.,  Nov.,  1916.*) 

49.  As  an  auditor,  to  what  extent  would  you  charge  yourself  with  re- 
sponsibility for  the  accuracy  both  as  to  facts  and  figures  of  the  wages? 
(111.,  May,  1913.) 

50.  "What  would  you  consider  satisfactory  evidence  of  the  correctness 
and  propriety  of  expenditures  of  wages  paidf  (N.  Y.,  Jan.,  1899*;  Pa., 
Nov.,  1900*;  N.  Y.,  Jan.,  1901*;  Mich.,  June,  1915*;  Wis.,  May,  1916*; 
A.  I.  of  A.,  May,  1918.) 

51.  In  making  a  detailed  audit,  what  procedure  would  you  follow  to 
verify  the  payroll?  (N.  Y.,  June,  1897*;  Wash.,  May,  1903*;  N.  Y., 
Jan.,  1906*;  Mich.,  June,  1912*;  Pa.,  Nov.,  1913*;  Kan.,  Dec.,  1915;  Mo., 
Dec,  1915;  Mass.,  Oct.,  1916*;  Mich.,  Dec,  1916*;  N.  C,  Aug.,  1917.*) 

52.  State  how  you  would  satisfy  the  correctness  and  regularity  of  an 
unsigned  payroll.     (N.  C,  Jime,  1916;  N.  C,  Nov.,  1918.) 

53.  What  evidence  would  you  consider  satisfactory  for  the  correctness 
of  the  salary  of  officers?     (N.  C,  Nov.,  1919.) 

54.  What  would  you  consider  satisfactory  evidence  of  the  correctness 
and  propriety  of  expenditures  of  salary  of  president?  (A.  I.  of  A.,  May, 
1918.) 

.55.     The  president  of  a  corporation  engaged  four  salesmen  on  a  salary 
and  a  profit-sharing  basis.     To  one  he  gave  40  per  cent  of  the  profits, 
to  the  other  three,  20  per  cent  each.    The  profits  of  the  corporation  were 
$102,608.18.      Show    proportion    of    profits    payable   to    each    salesman 
(N.  Y.,  June,  1919.) 

Administrative  Items 

56.  Define:  Administration  expense.     (Mich.,  Dec,  1914.) 

57.  How  would  you  classify  Rents  Collected  in  Advance  in  the  Balance 
Sheet  or  Profit  and  Loss  account?     (Kan.,  Dec,  1915;  Mo.,  Dec,  1915.) 

58.  State  what  is  indicated  by  the  Rent  account  (a)  when  the  account 
shows  a  debit  balance;  (6)  when  the  account  shows  a  credit  balance.  Ex- 
plain fully.     (N.  Y.,  Jan.,  1900.) 

59.  The  Chicago  Grocery  Company  owns  and  occupies  a  warehouse 
which  cost  $1,000,000.     The  management  has  decided  to  charge  operations 


INSURANCE  AND  GENERAL  EXPENSE  277 

of  the  various  departments  with  an  annual  rental  aggregating  $75,000. 
The  accounts  as  finally  prepared  show  the  various  departments  charged 
and  surplus  account  credited  with  this  amount.  Does  this  transaction 
meet  with  your  approval,  and  why?     (111.,  May,  1917.) 

60.  The  Jones  Manufacturing  Company,  needing  a  larger  building  for 
its  increasing  business,  finds  a  property  desirable  in  every  respect  except- 
ing that  the  building  is  much  larger  than  is  necessary.  They  lease  the 
property  at  an  annual  rental  of  $18,000,  after  considering  that  they  can 
probably  sublease  part  of  the  building.  Owing  to  the  desirability  of  the 
property  and  other  favorable  conditions,  they  execute  a  sublease  for  one- 
half  of  the  building  at  an  annual  rental  of  $18,000.  How  would  you 
treat  these  facts  in  compiling  the  annual  income  statement  of  the  Jones 
Manufacturing  Company?     (Wis.,  April,  1915.) 

61.  In  making  a  detailed  audit,  what  procedure  would  you  follow  to 
verify  Rents  Collected?     (Kan.,  Dec,  1915;  Mo.,  Dec,  1915.) 

62.  In  large  establishments,  where  the  outlay  on  postage  is  consider- 
able, what  system  would  you  advise  to  prevent  loss  by  theft?  (111.,  May, 
1904.) 

63.  Where  would  you  place  the  Factory  Office  Expense  account  in  the 
Income  and  Profit  and  Loss  account?     (Md.,  Dec,  1917.) 

64.  What  would  you  consider  satisfactory  evidence  of  the  correctness 
and  propriety  of  expenditures  of  pensions  paid  to  ex-employees?  (A.  I. 
of  A.,  May,  1918.) 

65.  What  would  you  consider  satisfactory  evidence  of  the  correctness 
and  propriety  of  expenditures  of  expenses  of  president?  (A.  I.  of  A,, 
May,  1918.) 

66.  In  what  respect  would  you  satisfy  yourself  of  the  regularity  of 
"Unclassified  Expenses"  in  absence  of  any  support  other  than  the  itemi- 
zation on  the  voucher  or  check  stub?     (N.  C,  June,  1916.) 

67.  What  information  is  required  in  respect  of  a  full  and  complete 
voucher  for  expenses  incurred?     (Wis.,  May,  1916.) 

Taxes 

68.  Classify  the  following  account:  Taxes.  (Iowa,  Dec,  1918;  A.  I. 
of  A.,  Nov.,  1919.*) 

69.  In  order  to  facilitate  the  preparation  of  monthly  Profit  and  Loss 
statements  by  a  corporation,  how  would  you  recommend  Taxes  to  be 
treated  on  its  books  from  month  to  month?     (Wash.,  April,  1906.) 

70.  Would  you  consider  it  proper  to  include  Taxes  Paid  in  Advance 
as  an  asset?  Explain  briefly  and  give  reasons.  (N.  Y.,  Jan.,  1897;  N.  Y., 
June,  1908.) 

71.  Classify  the  accounts  propei'ly  recording  the  following  items  ac- 
cording to  the  subdivision  of  assets,  liabilities,  proprietary  interest,  in- 
come and  expenses  under  which  they  should  be  grouped:  (a)  Property 
Taxes;  (&)  Income  and  Excess  Profits  Taxes.     (Wis.,  May,  1919.) 


278  C.  P.  A.  ACCOUNTING 

72.  "Would  you  advise  showing  profits  for  prospectus  purposes  before 
or  after  deducting  war  profits  and  income  taxes?  State  your  reasons 
briefly.     (A.  I.  of  A.,  Nov.,  1919.) 

73.  Define :  Accrued  taxes.     (Iowa,  Dec.,  1918.) 

74.  To  what  extent  would  you  consider  it  necessary  to  verify  the 
Taxes  Accrued  and  what  reference  to  such  vex'ifieation  would  you  make 
in  your  report?     (Mass.,  June,  1913.) 

75.  State  how  Accrued  Taxes  are  used  in  a  Profit  and  Loss  statement 
or  Balance  Sheet;  also  state  classification.     (Iowa,  Dec.,  1918.) 

76.  On  which  side  of  the  Ledger  should  the  balance  in  the  Tax  Accruals 
account  appear?     (Mo.,  Dec.,  1914.) 

77.  In  connection  with  the  audit  of  the  books  of  a  corporation  de- 
scribe briefly  the  procedure  you  would  consider  to  be  essential  in  verifying 
the  Accrued  Taxes.     (Wash.,  July,  1917.) 

78.  You  ascertain  that  a  client  owes  a  substantial  amount  for  assess- 
ments against  local  benefits.  No  liability  therefor  appears  on  the  books. 
How  would  you  proceed  to  determine  the  amount  due?  How  would  you 
reflect  such  amount  on  the  Balance  Sheet?     (A.  I.  of  A.,  Nov.,  1919.) 

79.  Give  some  idea  of  what  taxes  you  would  charge  against  income 
and  what  against  surplus.  Of  the  former,  which,  if  any,  would  you  take 
up  into  manufacturing  costs?     (A.  I.  of  A.,  Nov.,  1918.) 

80.  In  a  statement  of  condition,  requiring  classification  of  assets  and 
liabilities  into  fixed  assets,  current  assets,  etc.,  in  what  class  would  you 
place  taxes  paid,  but  not  accrued?  State  reasons  supporting  your  answer. 
(Mass.,  June,  1912.) 

81.  In  auditing  the  accounts  of  a  corporation  for  the  first  time  you 
find  that  during  prior  years  the  federal  income  tax  returns  (which  had 
not  been  audited  by  the  Government)  were,  in  your  opinion,  inaccurate,  and 
that  a  substantial  additional  amount  of  tax  is  due  to  the  Government. 
Your  client  claims  that  the  returns  were  coi'rect,  having  been  prepared 
under  the  advice  of  counsel.  You  believe  that  the  client  is  acting  in  good 
faith.  What,  if  anything,  would  you  recommend?  What  bearing,  if  any, 
would  the  facts  have  upon  a  Balance  Sheet  to  be  certified  by  you?  (A.  I. 
of  A.,  Nov.,  1919.) 

Adjusting  and  Closing  Entries 

82.  Define:  Closmg  entries.     (Pa.,  Nov.,  1900.) 

83.  Describe  the  nature  and  purpose  of  closing  entries  at  the  closing 
of  a  fiscal  period.     (N.  Y.,  Jan.,  1907.) 

84.  State  the  process  of  closing  a  Ledger.  (N.  Y.,  Jan.,  1897*;  N.  Y., 
Dec,  1898* ;  N.  Y.,  June,  1900* ;  N.  Y.,  Jan.,  1901 ;  N.  Y.,  June,  1902* ; 
Pa.,  Nov.,  1903*;  N.  Y.,  Jan.,  1906;  N.  Y.,  Oct.,  1907;  R.  I.,  Dec.,  1907*; 
Iowa,  Dec,  1918.*) 


INSURANCE  AND  GENERAL  EXPENSE  279 

85.  How  should  inventories  be  treated  in  closing  the  Ledger  at  the  end 
of  a  fiscal  year?     (N.  Y.,  Jan.,  1901.) 

86.  Why  is  property  unsold  credited  to  the  account  to  which  it  belongs 
before  closing?     (N.  Y.,  Dec,  1898.) 

87.  Describe  in  detail  the  process  of  closing  the  accounts  of  a  business 
contained  in  Ledger  "A"  and  transferring  them  to  Ledger  "B."  (N.  Y,, 
Jan.,  1904.) 

88.  In  preparing  annual  accounts  from  a  set  of  books  maintained  on 
the  double-entry  system,  what  general  rules  or  principles  would  guide  you 
in  deciding  Avhether  an  account  should  be  closed  by  an  entry  debiting  or 
crediting  Profit  and  Loss  account,  or  whether  the  balance  of  an  account 
should  be  included  as  an  item  in  the  Balance  Sheet?     (111.,  May,  1916.) 

89.  In  summarizing  the  nominal  accounts  of  a  manufacturing  concern 
to  determine  the  results  of  operations  for  a  period  (a)  what  would  be 
the  order  and  character  of  the  three  closing  accounts?  {b)  What  nature 
of  accounts  form  the  elements  of  each?  (c)  Give  your  reasons.  (111., 
May,  1911.) 

90.  How  should  expiring  accounts,  such  as  premiums  on  fire  insurance, 
taxes,  etc.,  be  treated?     (Md.,  Oct.,  1903.) 

91.  In  closing  a  set  of  books  at  any  given  period,  how  would  you  treat 
the  different  accounts  therein  (a)  of  a  partnership;  (b)  of  a  corporation? 
Explain  fully.     (Pa.,  Nov.,  1900;  La.,  May,  1913.*) 

92.  Name  all  the  accounts  which  contribute  to  the  Profit  and  Loss 
account  when  closing  the  books.     (Pa.,  May,  1905.) 

93.  Define:  Adjusting  entries.     (Pa.,  Nov.,  1900;  N.  Y.,  Jan.,  1907.*) 

94.  How  would  you  deal  with  the  items  accrued  and  due  (such  as  rent, 
commission,  and  salaries)  when  closing  the  accounts  of  a  business  at  the 
end  of  a  fiscal  period?  (N.  Y.,  Dec,  1898*;  N.  Y.,  Jan.,  1904*;  N.  Y., 
Feb.,  1910 ;  Va.,  Oct.,  1912* ;  N.  Y.,  Jan.,  1914* ;  Kan.,  May,  1916* ;  W. 
Va.,  May,  1919.*) 

95.  Explain  the  methods  of  treating  accruals,  indicating  advantages 
and  weaknesses  of  each.     (W.  Va.,  May,  1919.) 

96.  Illustrate  prepaid  accounts  and  accrual  accounts  used  by  concerns 
making  monthly  showings  of  operations.     (Wis.,  April,  1914.) 

97.  How  would  you  treat  tax  assessments  on  real  estate,  if  closing  the 
books  at  a  certain  period,  with  a  view  to  ascertaining  the  loss  or  gain? 
(N.  J.,  1904-1909.) 

98.  What  is  the  necessity  of  taking  into  account  accruals  of  interest, 
taxes,  rent,  etc?  How  should  prepaid  rents,  insurance,  etc.,  be  treated 
at  the  end  of  a  fiscal  period?     (Ohio,  Oct.,  1919.) 

99.  In  auditing  the  books  of  a  company  whose  fiscal  year  ends  Decem- 
ber 31  you  find  charged:     (a)  Rent  for  the  year  ending  September  30;. 

(b)  insurance  premiums  for  tlie  year  ending  March  31,  paid  in  advance; 

(c)  discounts  actually  allowed  during  the  year;  (d)  taxes  paid  during  the 
year,  assessed  for  the  previous  calendar  year. 


280  C.  P.  A.  ACCOUNTING 

You  are  informed  that  in  the  opinion  of  the  company  no  adjustment 
should  be  made  as  a  full  year's  charge  for  each  of  these  items  has  been 
recorded  in  the  accounts.    State  your  views. 

(Wash.,  June,  1915.) 

100.  An  auditor  is  called  upon  to  verify  a  Balance  Sheet,  and  upon 
investigation  he  finds  that  unexpired  insurance,  interest  paid  in  advance 
on  discounted  notes,  taxes  accrued  on  demand  notes  and  bonded  indebted- 
ness, royalties,  etc.,  are  not  included  in  same.  He  is  informed  that  it 
has  not  been  the  custom  of  the  corporation  to  include  in  their  Balance 
Sheet  such  items,  as  they  offset  one  another,  and  that  the  directors  do 
not  desire  any  change  in  the  practice  they  have  adopted.  Discuss  this 
proposition,  stating  reasons  for  your  conclusions.     (Ohio,  Nov.,  1917.) 

101.  Describe  the  method  of  closing  the  books  of  a  concern,  stating 
how  you  provide  for  depreciation,  bad  debts,  surplus,  and  dividend  ac- 
counts.    (N.  Y.,  Jan.,  1906*;  Fla.,  July,  1909.) 

102.  In  closing  a  set  of  books,  state  how  you  would  treat  the  following 
on  Ledger  on  financial  statement:  Depreciation  on  machinery,  $1,500; 
expenses  prepaid,  $500;  discount  on  customers'  accounts,  3  per  cent, 
$1,080;  salaries  and  wages  accrued,  $675.     (N.  Y.,  Jan.,  1904.) 

lOSi'*  What  are  some  of  the  important  things  to  consider  before  closing 
a  set  of  books  for  a  fiscal  period?     (Iowa,  Dee.,  1918.) 

104.  What  is  meant  by  an  "Expense  Inventory"  and  how  would  you 
treat  this  item  in  drawing  up  the  Profit  and  Loss  account  and  Balance 
Sheet?     (Colo.,  Dec.,  1913.) 

105.  Describe  three  methods  of  treating  accruals  and  the  advantage 
or  weakness  of  each.     (Ind.,  Nov.,  1917.) 

106.  Give  the  treatment  of  deferred  accounts  at  the  close  of  the  ac- 
counting period.     (Ind.,  May,  1918.) 


INSURANCE  AND  GENERAL  EXPENSE 


281 


PROBLEMS 


INSURANCE  AND  GENERAL  EXPENSE 

1.  A  merchant  effected  $40,000  insurance  on  property  worth  $60,000; 
his  policies  contained  the  usual  80  per  cent  coinsurance  clause.  A  loss 
of  $30,000  was  sustained.  What  sum  should  the  merchant  receive  from 
the  underwriters  in  correct  settlement  of  the  loss?     (Colo.,  Dec,  1913.) 

2.  A  fire  in  a  manufacturing  concern  resulted  in  a  loss  on  machinery, 
$5,000;  merchandise  (raw  material),  $10,000;  manufactured  goods,  $25,- 
000,  of  which  amount  $40,000  was  agreed  on  and  paid  by  the  insurance 
companies.  Give  the  entries  necessary  to  record  properly  the  above  trans- 
actions on  the  books  of  the  concern.     (N.  Y.,  June,  1914.) 

3.  (a)  If  the  value  of  the  property  insured  is  $10,000  and  the  actual 
insurance  at  the  time  of  the  fire  is  80  per  cent,  what  should  be  the  settle- 
ment if  the  loss  was  50  per  cent  of  the  total  value?  (6)  If  the  property 
value  was  $10,000  with  an  80  per  cent  coinsurance  clause  and  the  actual 
insurance  in  force  at  the  time  of  the  fire  was  $6,000,  what  would  be  the 
owner's  deficiency?  (c)  How  much  if  the  loss  was  only  40  per  cent  of 
the  total?     (Mich.,  July,  1909.) 

4.  Robert  Adams  and  William  Stevens  are  equal  partners.  On  the 
night  of  July  3,  their  stock  and  fixtures  were  destroyed  by  fire.  A  trial 
balance  which  Adams  had  at  his  home  showed  the  following  condition  of 
the  Ledger  at  the  close  of  the  business  June  30 : 


Robert  Adams 

William  Stevens 

Cash 

Fixtures 

Merchandise  Purchases 
Merchandise  Sales .... 

Notes  Receivable 

Notes  Payable 

Interest 

Expense 

Customers 

Creditors 


The  property  is  fully  covered  by  insurance.     The  insurance  company, 
for  the  purpose  of  estimating  the  value  of  the  merchandise  destroyed,  has 


$600 

$7,450 

600 

7,450 

3,309 

1,500 

32,600 

24,800 

1,000 

2,000 

120 

50 

780 

4,500 

3,259 

$45,009 

$45,009 

282 


C.  P.  A.  ACCOUNTING 


agreed  to  allow  35  per  cent  as  the  average  gross  gain  on  the  sales  and  to 
pay  66%  per  cent  on  the  value  of  fixtures  as  shown  by  the  Ledger.  On 
the  basis  of  this  agreement,  state  the  result  of  the  business  and  the  capital 
of  each  partner. 

(Mass.,  June,  1910.) 
5.     On  October  31,  1919,  the  store  of  the  Good  Merchandise  Company 
was  destroyed  by  fire.    It  was  a  total  loss.     The  books  and  records  were 
found  to  be  complete,  and  the  trial  balance  as  of  October  31  was  as 
follows : 


Accounts  Receivable 

Accounts  Payable 

Cash 

Merchandise  Inventory,  January  1,  1919 

Merchandise  Purchases 

Real  Estate 

Dividends  Paid 

Buildings  and  Fixtures 

Furniture  and  Furnishings 

Reserve  for  Depreciation,  Buildings  and  Fixtures  .  .  . 
Reserve  for  Depreciation,  Furniture  and  Furnishings 

Unexpired  Insurance 

Merchandise  Sales 

Miscellaneous  Income 

Clerks'  Salaries 

Light,  Heat,  and  Power 

Advertising 

Office  Salaries , 

Officers'  Salaries 

Postage 

Treasury  Stock 

Taxes 

Telephone  and  Telegrams 

Sundry  General  Expense 

Capital  Stock 

Surplus 


$7,000 

2,000 
15,000 
85,000 

3,000 

4,000 
18,000 

5,000 


2,500 


5,000 

1,000 

2,000 

2,500 

5,000 

700 

10,000 

2,500 

150 

50C 


$170,850 


$5,000 


3,000 
300 

99,000 
1,500 


50,000 
12,050 


$170,850 


An  average  gross  profit  of  33j/^  per  cent  was  agreed  upon  by  all 
parties  concerned. 

Stock,  buildings,  fixtures,  and  furnishings  were  insured  under  the  80 
per  cent  clause,  the  buildings,  etc.,  for  $15,000  and  the  stock  for  $32,000. 
Settlement  is  made  on  the  basis  of  these  facts. 

You  are  asked  to  give  (a)  the  profit  or  loss  due  to  fire,  (&)  the  oper- 
ating statement  for  the  period  ending  October  31. 

(Wis.,  Nov.,  1919.) 

6.  The  Star  Department  Store,  organized  January  1,  1911,  suffers  a 
fire  loss  just  before  inventory  December  31,  1913.  The  books  disclose  the 
following  facts: 


INSURANCE  AND  GENERAL  EXPENSE 


283 


Dec.  31,  1911 


Dec.  31,  1912 


Dec.  31,  1913 


Inventory  Beginning 

Purchases  During  Year. . . 
Allowance  on  Purchases  .  . 

Sales 

Allowance  on  Sales 

Advertising 

Salaries 

Wages 

Delivery 

Depreciation  of  Furniture 
Stationery  and  Printing  .  . 

Gas 

Rent 

Insurance 

Interest 

Light 

Water 

Taxes 

Telephone 

General 

Traveling 

Furniture  and  Fixtures . . 


$44,244 .04 

171,133.33 

11,900.12 

197,474.49 

4,294.37 

3,487.39 

8,295.92 

16,684.30 

567.34 

169.31 

282.40 

75.09 

5,350.00 

927.15 

1,571.02 

612.53 

29.21 

438.80 

33.75 

2,041.30 

352.94 

9,000.00 


$51,894.68 

173,478.81 

15,182.62 

209,397.00 

4,594.50 

3,334.45 

9,196.72 

16,628.75 

567.38 

372.92 

309.30 

45.55 

5,400.00 

856.42 

1,506.89 

567.40 

28.51 

597.00 

52.27 

3,343.37 

351 .93 

9,500.00 


$50,396.40 

157,188.09 

8,293.54 

195,937.98 

5,179.19 

2,987.56 

9,196.72 

17,531.22 

1,053.34 

112.94 

300.00 

62.02 

5,400.00 

770.40 

1,695.80 

525.06 

27.63 

891 .07 

54.00 

2,581.32 

278.28 

10,500.00 


You  are  asked  by  the  appraisers: 

(a)     To  determine  the  value  of  assets  destroyed. 

(&)  To  arrive  at  correct  amounts  to  effect  a  complete  adjustment  under 
the  following  concurrent  policies  containino;  the  80  per  cent  coinsurance 
clause. 


Stock 

Furniture 

Home  Insurance  Co 

$10,000 

15,000 

5,000 

10,000 

$2,000 
3,000 
1,000 
2,000 

Glen  Falls  Insurance  Co 

Globe  In.surance  Co 

Equitable  Insurance  Co 

(c)  If  the  property  loss  had  been  50  per  cent,  what  amounts  would 
have  been  the  proper  adjustments'? 

(Mich.,  Dec,  1913.) 

7.  A  merchant's  ledger  disclosed  the  following  statement  on  March  31, 
1915: 

Inventory  of  goods  on  hand  January  1,  1914,  $26,000. 

January  31,  merchandise  purchases,  $10,800;  merchandise  returned  to 
store,  $700;  sales,  $13,800;  purchases  returned  to  seller,  $250.  February 
28,  merchandise  purchases,  $8,600;  merchandise  returned  to  store,  $375; 
sales,  $10,575;  purchases  returned  to  seller,  $325.  March  31,  mercliandi.se 
purchases,  $9,675;  purchases  returned  to  store,  $567;  sales,  $16,425;  mer- 
chandise returned  to  seller,  $435.    April  30,  merchandise  purchases,  $6,756 ; 


284  C.  P.  A.  ACCOUNTING 

merchandise  returned  to  store,  $437;  sales,  $12,150;  goods  returned  to 
seller,  $193.  May  31,  merchandise  purchases,  $7,693;  merchandise  re- 
turned to  store,  $842 ;  sales,  $14,236 ;  goods  returned  to  seller,  $178.  June 
30,  merchandise  purchases,  $6,957;  merchandise  returned  to  store,  $359; 
sales,  $20,789;  goods  returned  to  seller,  $237.  July  31,  merchandise  pur- 
chases, $10,764 ;  merchandise  returned  to  store,  $653 ;  sales,  $15,283 ;  goods 
returned  to  seller,  $364.  August  31,  merchandise  purchases,  $12,786; 
merchandise  returned  to  store,  $689;  sales,  $19,476;  goods  returned  to 
seller,  $113.  September  30,  merchandise  purchases,  $19,436;  merchandise 
returned  to  store,  $347;  sales,  $8,679;  goods  returned  to  seller,  $]47. 
October  31,  merchandise  purchases,  $10,798 ;  merchandise  returned  to  store, 
$878;  sales,  $17,124;  goods  returned  to  seller,  $236.  November  30,  mer- 
chandise purchases,  $12,745;  merchandise  returned  to  store,  $1,087;  sales, 
$22,187;  goods  returned  to  seller,  $475.  December  31,  merchandise  pur- 
chases, $8,649 ;  merchandise  returned  to  store,  $983 ;  sales,  .$4,851 ;  goods 
returned  to  seller,  $122,    Inventory  as  of  December  31,  1914,  $32,232. 

January  31,  1915,  merchandise  purchases,  $9,786;  sales,  $14,287;  goods 
returned  to  seller,  $342;  goods  returned  to  store,  $329.  February  28, 
sales,  $11,364;  goods  returned  to  store,  $597;  merchandise  purchases, 
$9,873;  merchandise  returned  to  seller,  $239.  March  31,  sales,  $17,747; 
goods  returned  to  store,  $324;  merchandise  purchases,  $11,397;  merchan- 
dise returned  to  seller,  $539. 

On  the  morning  of  April  1,  1915,  previous  to  time  of  opening  the  store 
for  business,  the  building  and  contents  were  destroyed  by  fire.  It  v?ill, 
therefore,  be  necessary  for  you  to  state  what  [was]  the  book  value  of  the 
merchandise  on  hand  at  the  time  of  the  loss,  as  of  March  31,  1915.  You 
will  make  a  Ledger  account  of  the  foregoing.  [Make  a]  Profit  and  Loss 
statement.  What  [did]  the  goods  cost  that  went  out  of  the  house,  as  of 
January  31,  1915.  What  was  the  ratio  of  profit  to  cost  of  goods  and  sell- 
ing price?     Show  how  you  arrive  at  same.    Make  statement  of  the  loss. 

(Texas,  June,  1915.) 

8.  A  severe  fire  occurred  on  March  1,  1917,  at  the  factory  of  the  Pitts- 
field  Woolen  Company,  by  which  practically  the  whole  of  the  buildings 
and  contents  were  destroyed,  with  the  exception  of  the  offices. 
,  The  insurance  adjusters  paid  a  lump  sura  of  $10,000  in  respect  of  the 
buildings,  $1,500  for  the  machinery,  and  $12,000  for  the  stock  on  hand, 
the  salvage  being  sold  to  the  Pittsfield  Woolen  Company  for  $1,000,  which 
salvage  was  deducted  from  the  insurance  money  on  settlement.  The 
Pittsfield  Woolen  Company  realized  $2,000  from  the  salvage. 

In  the  Pittsfield  Woolen  Company's  books,  land  and  buildings  stood  at 
$17,500,  and  the  value  of  the  buildings  not  destroyed  was  $1,000,  the  land 
being  valued  at  $5,000;  the  machinery  stood  at  $1,600  and  was  a  total 
loss;  the  stock  on  hand  at  date  of  last  inventory  (June  30,  1916)  stood 
at  113,500.  The  sales  for  the  eight  months  from  date  of  last  inventoiy 
(June  30,  1916)  to  date  of  fire  (March  1,  1917)  amounted  to  $30,000,  and 
the  purchases  and  expenses  for  the  period  to  $26,000.     The  gross  profit 


INSURANCE  AND  GENERAL  EXPENSE 


285 


on  sales  was  known  to  be  Sj/^   per  cent.     There  were  expenses  in  con- 
nection with  the  fire  insurance  adjustment  of  $200. 

Show  how  the  above  matters  should  be  entered  in  the  books  of  the 
Pittsfield  Woolen  Company. 

(N.  D.,  July,  1918.) 

9.  The  B.  Moose  Printing  Company,  a  corporation,  had  a  capital  of 
$50,000,  which  was  held  by  B.  Moose,  $25,000 ;  U.  Moose,  $15,000,  and  L. 
Moose,  $10,000. 

The  plant  was  destroyed  by  fire  December  15,  1917.  All  books  and 
records  were  saved,  except  the  sales  records,  which  were  not  written  up 
for  December.  The  insurance  companies  paid  $28,000  on  the  machinery 
and  equipment  and  $7,000  on  the  stock,  which  was  distributed  to  the 
stockholders  as  received  in  proportion  to  their  holdings.  On  December 
30th,  the  trial  balance  disclosed  the  following  conditions: 


Capital  Stock 

Machinery  and  Equipment. . . . 
Stock  on  Hand,  June  1,  1917  .  . 

Accounts  Receivable 

Accounts  Payable 

Reserve  for  Bad  Debts 

Insurance  Adjustment 

Cash 

Engraving 

Printing 

December  Sales,  not  segregated 

Merchandise  Purchases 

Wages 

Rent 

Salaries 

Surplus 

L.  Moose 

U.  Moose 

B.  Moose 


$30,000 

8,750 

19,640 


3,900 


58,800 

130,180 

1,800 

5,750 

7,000 
10,500 
17,500 


$293,820 


$50,000 


12,590 

1,250 

28,000 

77,600 
99,350 
24,175 


855 


$293,820 


The  accounts  receivable  realized  $18,320,  and  the  liquidation  expenses 
were  $1,850,  The  stockholders  turned  in  their  stock  for  cancellation  and 
received  their  proportionate  amount  in  cash.  Prepare  Journal  entries 
closing  the  books  of  the  corporation,  and  a  Profit  and  Loss  account.  In- 
clude Journal  entries  for  final  distribution  of  cash  and  for  final  disposition 
of  loss  or  gain. 

(Va.,  Nov.,  1918.) 

10.  A  firm  manufacturing  but  one  grade  of  cloaks,  insured  against 
burglary,  claims  to  have  been  robbed  on  the  night  of  September  10.  The 
proof  of  the  loss  filed  by  the  assured  contained  two  items  for  600  cloaks, 
$12,000;  silk,  1,000  yards,  $1,500.  An  inventory  of  the  stock  on  hand, 
consisting  of  cloaks,  cloth  and  silk,  had  been  taken  January  1,  amounting 
to  $118,500,  the  particulars  of  which  have  been  lost  or  destroyed. 


286  C.  P.  A.  ACCOUNTING 

An  analysis  of  the  firm's  books  produced  the  following  information: 
purchase  of  cloth,  37,500  yards,  at  $1;  purchases  of  silk,  10,000  yards,  at 
$2;  6,000  cloaks  were  manufactured,  consuming  cloth,  40,000  yards,  at 
$1;  silk,  10,000  yards,  at  $2;  9,000  cloaks  were  sold  between  January  1 
and  September  10.  Cost  of  sales,  per  cloak,  for  material,  $10;  cost  of 
sales,  per  cloak,  for  labor  and  sundries,  $7. 

Inventory,  September  11 :  2,500  cloaks,  at  $17;  12,500  yards  cloth,  at  $1; 
5,000  yards  silk,  at  $2. 

Prepare  a  report  proving  or  disproving  the  claim. 

(N.  y.,  June,  1909;  N.  Y.,  Jan.,  1917.) 


CHAPTER  XV 

DEPRECIATION 

Nature  of  Depreciation — Depreciation  may  be  defined  as  the  shrinkage 
in  value  of  an  asset  through  use,  the  passing  of  time,  decay,  or  obsoles- 
cence.^ Repairing  property  tends  to  decrease  the  rate  of  depreciation  but 
cannot  prevent  the  loss.^  Depreciation  must  not  be  confused  with  loss  in 
efficiency/  The  efficiency  of  most  machines  is  almost  constant  up  to  the 
period  in  which  they  are  scrapped,  yet  the  depreciation  accumulates  during 
the  entire  life  of  the  machine.* 

Depreciation  is  a  cost  of  doing  business,'  the  charge  being  made  to  the 
department  of  the  business  using  the  related  asset.  The  statement  that  the 
reserve  for  depreciation  is  a  division  of  surplus"  therefore  seems  incorrect. 

Causes  of  Depreciation — The  following  table  summarizes  the  causes  of 
depreciation  on  tangible  property.' 

Causes  of  Depreciation. 
I.  Physical  Depreciation 

(a)  Wear  and  tear  from  operation 

(b)  Action  of  time  and  the  elements 
II.  Functional  Depreciation 

(a)  Inadequacy  or  supersession 

(b)  Obsolescence 

III.  Contingent  Depreciation 

(a)  Accidents 

(b)  Diseases 

(c)  Depletion 

Depreciation  on  intangible  property  is  caused  either  by  the  expiration  of 
time  or  by  the  abandonment  of  the  assets.* 

Inadequacy — Inadequacy  indicates  inability  fully  to  perform  the  in- 
tended function."  It  is  caused  by  (a)  changes  in  original  financial  policy, 
(b)  motives  of  engineering  economy,  and  (c)  unforeseen  developments.'* 
Losses  occasioned  by  inadequacy  should  be  provided  for  under  the  head  of 
a  contingent  reserve,  and  not  included  in  the  estimate  of  depreciation, 
except  in  those  cases  when  the  loss  can  be  intelligently  foreseen." 

Obsolescence — Obsolescence,  or  the  shrinkage  in  value  due  to  the  fact 
that  property  has  been  rendered  out  of  date  by  new  inventions  and  im- 
provements," is  usually  too  uncertain  to  constitute  an  element  in  the  cost 


Tor  explanation  of  superior  figures  see  page  337. 

287 


288  C.  P.  A.  ACCOUNTING 

of  operations."  Whenever  possible,  a  reserve  for  contingent  losses  due  to 
obsolescence  should  be  created  out  of  surplus."  In  industries  where  obso- 
lescence can  be  foretold  with  reasonable  certainty,  however,  obsolescence 
should  be  included  in  the  depreciation  charge." 

Depletion — Depletion,  or  the  "giving  out"  of  assets,  must  be  differen- 
tiated from  depreciation,  or  the  "wearing  out"  of  assets."  However,  the 
fact  that  a  mine  is  subject  to  depletion  frequently  has  a  vital  effect  on  the 
depreciation  of  the  mine  equipment,  which  is  of  little  value  when  the  ore 
is  exhausted,  i.  e.,  mine  buildings  having  a  longer  estimated  life  than  the 
ore  body  depreciate  at  the  same  rate  as  the  ore  body,  but,  if  the  life  of  the 
buildings  is  less  than  that  of  the  ore  body,  the  buildings  depreciate  at  their 
normal  rate." 

Actual  and  Theoretical  Depreciation — The  term  "actual  depreciation" 
signifies  the  decrease  in  the  selling  value  of  an  asset  from  its  state  when 
new  to  its  used  condition."  Theoretical  depreciation  is  the  loss  due  to 
wearing  out  of  the  asset  prorated  more  or  less  uniformly  over  the  life  of 
the  asset."  The  use  of  theoretical  depreciation  in  accounting  records  is 
justified  by  the  fact  that  going  concerns  usually  replace,  instead  of  sell, 
their  fixed  assets.** 

Appreciation  and  Depreciation — Since  fixed  assets  normally  are  not  sold, 
fluctuations  in  the  selling  prices  of  fixed  assets  are  not  recognized  on  the 
books  of  account  and  appreciation  can  not  therefore  be  used  to  offset 
depreciation."  In  fact,  if  appreciation  is  booked,  the  charge  for  deprecia- 
tion will  be  automatically  increased." 

Booking  of  Depreciation — When  the  periodic  charge  to  the  depreciation 
expense  account  is  made,  the  credit  can  be  carried  directly  either  to  the 
assets  or  to  a  reserve  for  depreciation  account  f^  the  latter  procedure  is  the 
better,  in  order  that  the  asset  may  be  shown  at  its  original  cost  figtu'es." 
The  recording  of  depreciation  in  connection  with  the  sale  of  a  depreciated 
asset  is  shown  on  page  — . 

Although  there  are  some  accountants  who  advocate  showing  the  de- 
preciation reserves  on  the  right  side  of  the  balance  sheet  either  under  the 
caption  "appropriated  surplus""  or  under  the  caption  "reserves,""'  the 
majority  deducts  the  reserves  from  the  related  assets"  in  order  to  show 
the  net  book  value  of  the  assets. 

Depreciation  Fund — Depreciation  or  replacement  funds  are,  like  all 
funds,  created  by  investing  in  marketable  securities  an  amount  equal  to 
the  charge  for  depreciation.''*  The  entry  for  the  establishing  of  a  deprecia- 
tion fund  consists  of  a  debit  to  the  depreciation  fund  account  and  a  credit 
to  the  cash  accoimt."  When  a  depreciating  asset  is  replaced,  the  fund 
securities  are  sold,  the  entry  being  a  debit  to  the  cash  account  and  a  credit 
to  the  depreciation  fund  account,  the  money  being  used  to  replace  the 
asset.^  It  is  vitally  important  to  remember  that  there  is  no  connection 
between  the  depreciation  fund  and  the  depreciation  reserve."  The  former 
is  the  means  of  accumulating  current  assets  to  replace  a  depreciated  asset,^ 
while  the  latter  is  the  means  of  writing  off  a  depreciating  asset  into  operat- 
ing expense."    Whether  a  depreciation  fvmd  is  created  depends  upon  the 


DEPRECIATION  289 

iinaneial  policy  of  the  business/*  but  a  depreciation  reserve  must  be  created 
unless  the  credit  is  made  directly  to  the  related  asset  account.^  A  de- 
preciation fund  is  especially  serviceable  in  the  case  of  expensive  and  short- 
lived assets.'" 

Depreciation  Rates — An  auditor  would  do  well  to  equip  himself  with  the 
tables  for  depreciation  rates  prepared  by  the  United  States  Government, 
by  the  various  state  commissions,  and  by  large  private  appraisal  firms."' 
The  C.  P.  A.  candidate  should  be  familiar  with  the  general  range  of  the 
rates  in  these  tables  but  should  not  attempt  to  memorize  the  tables. 

Plant  Ledger — As  practically  every  kind  of  fixed  asset  has  an  individual 
depreciation  rate,  the  details  of  the  plant  and  machinery  controlling  ac- 
count are  carried  in  a  subsidiary  ledger,  called  the  plant  ledger."  This 
ledger  usually  has  a  separate  account  for  each  plant  unit."  These  plant 
ledger  accounts  may  be  ruled  to  show  the  year  of  installation,  estimated 
life,  first  cost  including  installation  costs,  and  annual  depreciation  charges 
for  a  number  of  years.*"  As  these  accounts  usually  provide  for  the  sub- 
traction of  the  annual  depreciation  charges  from  previous  book  values  of 
the  assets,  the  balance  of  the  plant  ledger  usually  corresponds  with  the 
dilt'erence  between  the  cost  figures  of  the  plant  and  equipment  less  the 
reserve  for  depreciation.*^ 

Calculation  of  Depreciation — The  determination  of  depreciation  rates 
is  an  engineering  problem,  but  an  accountant  should  understand  how  and 
when  to  use  the  various  methods  of  determining  depreciation  charges." 
These  methods  may  be  broadly  classified,  though  with  some  overlapping, 
under  the  following  heads :" 

Methods  of  Calculating  Depreciation. 

I.  Proportion  Methods 

(a)  Straight  Line 

(b)  Working  Hours 

(c)  Composite  Life 

(d)  Service  Output 

II.  Variable  Percentage  Methods 

(a)  Fixed  Percentage  on  Diminishing  Value 

(b)  Sum  of  Year  Digits  (sometimes  known  as  "Sum  of  Expected 

Life-Periods") 

III.  Compound  Interest  Methods 

(a)  Sinking  Fund 

(b)  Annuity 

IV.  Miscellaneous  Methods 

(a)  Maintenance 

(b)  Replacement 

(c)  Fifty  Per  Cent 

(d)  Appraisal 

(e)  Insurance 

(f )  Gross  Earnings 


290 


C.  P.  A.  ACCOUNTING 


Straight  Line  Method — The  straight  line  method  of  calculating  deprecia- 
tion, so  called  because  its  gi-aphical  representation  is  a  straight  line," 
figures  the  periodic  charge  by  dividing  the  difference  between  the  initial 
and  scrap  value  of  the  asset  by  the  number  of  periods  in  its  service  life.'"* 
Assuming  an  asset  costing  $100,  service  life  3  years,  scrap  value  $10,  the 
annual  charge  would  be  $30  (1/3  X  ($100  — $10)).  Where  the  output  is 
uniform  and  obsolescence  and  inadequacy  are  unimportant,  and  where  the 
cost  of  repairs  is  uniformly  prorated  over  the  life  of  the  asset,  the  straight 
line  method  gives  satisfactory  results.**  The  straight  line  method  has  the 
advantage  of  simplicity."  It  approximates  the  actual  depreciation  about 
as  closely  as  the  more  elaborate  methods,**  but  it  makes  no  provision  for  the 
increased  maintenance  charges  toward  the  end  of  the  life  of  the  asset." 
The  method  is  rigid  and  does  not  satisfactorily  allow  for  overtime  and 
other  fluctuations  in  wear  and  tear.'"  The  method  is  especially  adapted  to 
the  calculation  of  depreciation  on  short-lived  assets." 

Working  Hours  Method — The  working  hours  method  of  calculating  de- 
preciation figures  the  periodic  charge  by  prorating  the  difference  between 
the  initial  and  scrap  value  of  the  asset  between  periods,  according  to  the 
ratio  of  actual  hours  the  asset  works  dui'ing  each  period."  Assuming  an 
asset  costing  $100,  service  life  9,000  working  hours,  scrap  value  $10,  the 
charge  for  a  period  in  which  the  asset  works  3,000  hours  would  be  $30 
(3,000/9,000  X  ($100  — $10)).  Where  the  asset  performs  only  a  few 
processes  equally  wearing  in  their  effects,  where  the  rate  of  operation  never 
exceeds  the  asset's  normal  speed,  and  where  the  up-keep  costs  are  equitably 
distributed,  the  working  hours  method  gives  fairly  satisfactory  results." 
The  method  makes  the  periodic  charge  agree  with  the  use  made  of  the 
asset,"  b«t  fails  to  provide  for  the  increase  in  the  maintenance  charges 
toward  the  end  of  the  asset's  life.'" 

Composite  Life  Method — The  composite  life  method  operates  the  same 
as  the  straight  line  method,  the  outstanding  feature  being  that  depreciation 
is  calculated  on  the  plant  as  a  whole,  rather  than  on  the  individual  assets.** 
Assume  two  machines,  one  costing  $5,000  and  having  a  service  life  of  10 
years  and  a  scrap  value  of  $1,000,  and  the  other  costing  $4,000  and  having 
a  service  life  of  5  yeai-s  and^  a  scrap  value  of  $500.  Tlie  composite  life 
depreciation  rate  can  be  calculated  by  the  following  two  methods : 


First  Method. 


Cost 
Value 

Life 
Periods 

Scrap 
Value 

Depre- 
ciation 

Times 
Replaced 

(a) 

Total  Loss 

in  10  Years 

(b) 

Dollar 
Years 

(c) 

S5,000 
4,000 

10 
5 

$1,000 
500 

$4,000 
3,500 

1 
2 

$4,000 
7,000 

$50,000 
40,000 

$9,000 

$11,000 

$90,000 

DEPRECIATION 


291 


(a)  Number  of  times  renewal  of  the  asset  will  be  required  during  the 

longest  life  period. 

(b)  Product  of  depreciation  multiplied  by  times  replaced. 

(c)  Product  of  cost  value  multiplied  by  life  periods  multiplied  by  times 

replaced. 
$11,000 -^  $90,000  =  12.22 -f- per  cent,  rate  of  depreciation. 

It  is  a  noteworthy  fact  that  all  accountants  do  not  agree  with  the' above 
method  of  calculating  the  depreciation  on  a  composite  group  of  assets. 
Some  accountants  calculate  the  "dollar  years"  by  multiplying  the  "depre- 
ciation" by  the  "life  periods"  multiplied  by  the  "times  replaced.""  This 
would  seem  incorrect  as  the  resulting  depreciation  rate  will  be  applied  on 
the  cost  values  and  not  on  the  cost  less  scrap  values  of  the  assets.  That 
12.22  -|-  per  cent  is  the  correct  rate  can  be  seen  from  the  fact  that  its  use 
creates  a  reserve  of  $11,000  within  the  ten  years,  which  will  exactly  offset 
the  loss  due  to  depreciation.  During  the  ten  years  the  cost  of  the  assets 
will  total  $13,000  ($5,000  -f  (2  X  $4,000) ),  while  the  scrap  value  will  total 
$2,000  ($1,000+ (2  X  $500)),  the  difference  of  $11,000  being  the  loss 
due  to  depreciation. 

Second  Method. 


Cost 
Value 

Life 
Periods 

Scrap 
Value 

Cost  Less 
Salvage 

Rate  of 
Deprecia- 
tion 

(a) 

Annual 
Deprecia- 
tion 

(b) 

S5,000 
4,000 

10 

5 

$1,000 
500 

$4,000 
3,500 

10 
20 

$400 
700 

$9,000 

$1,100 

(a)  Quotient  of  100  divided  by  Life  Periods. 

(b)  Product  of  Cost  less  Salvage  multiplied  by  Rate  of  Depreciation. 
$1,100  ^  $9,000  =  12.22  +  per  cent,  rate  of  depreciation. 

Of  the  above  methods,  the  second  is  the  better,  as  it  is  more  simple  and 
direct.  The  composite  life  method  of  caleidating  depreciation  is  subject 
to  all  the  limitations  of  the  straight  line  method  and  has  the  additional 
disadvantage  of  losing  sight  of  the  depreciation  on  individual  assets. 
However,  the  method  is  serviceable  in  estimating  the  cost  of  depreciation 
for  the  plant  as  a  whole,  and  in  furnishing  a  basis  for  the  comparison  of 
two  similar  plants." 

Service  Output  Method — The  service  output  method  of  calculating  de- 
preciation is  the  same  as  the  working  hours  method,  except  that  the  service 
life  is  measured  in  output  instead  of  working  hours."  Assume  a  machine, 
costing  $100,  capable  of  making  9,000  units  of  product,  scrap  value  $10, 
the  charge  for  a  period  in  which  the  machine  produces  3,000  units  would 
be  $30  (3,000/9,000  X  ($100  — $10)).    As  this  method  requires  the  pre- 


292  C.  P.  A.  ACCOUNTING 

determination  of  output,  the  conditions  must  be  uniform  and  the  asset  must 
perform  only  a  few  processes."  The  method  is  f requeuth'  used  in  connec- 
tion wjth  wasting  assets,"  and  is  also  adapted  to  calculate  the  depreciation 
on  machines  which  have  special  functions  and  which  are  used  only  at 
irregular  times." 

Fixed  Percentage  on  Diminishing  Value  Method — The  fixed  percentage 
on  diminishing  value  method  of  calculating  depreciation  estimates  the 
periodic  charge  as  a  fixed  percentage  of  the  appraised  or  book  value  of  the 
asset  as  at  the  time  of  the  last  appraisal.'"'  Letting  "n"  equal  the  number 
of  periods,  "r"  equal  the  periodic  depreciation  rate,  and  "sv"  and  "cv" 
equal  the  scrap  and  cost  values,  respectively,  the  formula  for  this  method, 
which  in  complex  cases  requires  the  use  of  logarithms  to  solve,  is  :** 

r  =1  —  '^  C  V 

For  an  asset  costing  $100,  service  life  3  years,  scrap  value  $10,  the  de- 
preciation rate  by  the  above  formula  is  approximately 


1—  ^ 


10 
loo  =  .536. 


The  periodic  charge  on  such  an  asset  would  be  $53.60  ($100  X  .536)  for 
the  first  period,  and  $24.87  ( ($100 —  $53.60)  X  .536)  for  the  second 
period.  The  chief  advantage  claimed  for  this  method  is  that  the  periodic 
charges  under  it  are  heavy  at  the  beginning  and  decrease  toward  the  end, 
and  thus  tend  to  equalize  the  increase  in  the  up-keep  costs  which  it  is 
claimed  normally  increase  with  the  age  of  the  machine.**  However,  as 
repairs  frequently  do  not  increase  uniformly  during  the  life  of  the  ma- 
chine, but  may  be  incurred  very  irregularly,  this  argument  is  of  doulitful 
value.  The  method  also  involves  complex  calculations,**  is  distasteful  to 
new  concerns  on  account  of  the  heavy  initial  charges,*'  and  is  subject  to 
the  objections  that  it  prorates  the  charges  on  a  time  rather  than  a  service 
or  output  basis,**  and  that  the  life  of  the  asset  is  not  clearly  indicated  by 
the  depreciation  rate.*"  The  method  can  not  logically  be  used  in  connection 
with  an  asset  having  no  residual  value." 

Sum  of  Year  Digits  Method — The  sum  of  year  digits  method,  sometimes 
called  the  sum  of  expected  life  periods  method,  prorates  the  loss  due  to 
depreciation  (the  excess  of  the  cost  over  the  scrap  value)  over  the  periods 
in  the  ratio  of  fractions  whose  numerators  are  in  the  order  of  the  expected 
life  periods  and  whose  denominators  are  the  sum  of  all  the  expected  life 
periods.''  Assume  an  asset  costing  $100,  sei-viee  life  3  years,  and  scrap 
value  $10.  As  the  expected  life  at  the  beginning  of  each  period  would  be 
3,  2,  and  1  periods,  respectively,  making  a  total  of  6,  the  loss  due  to 
depreciation  of  $90  ($100  — $10)  would  be  prorated  3/6,  or  $45,  to  the 
first  period ;  2/6,  or  $30,  to  the  second  period,  etc.  The  sum  of  year  digits 
method  gives  the  same  general  effect  as  the  fixed  percentage  on  diminish- 


DEPRECIATION  *  293 

ing  value  method,"  and  is  subject  to  the  same  objections  except  that  it  is 
easier  to  calculate." 

Sinking  Fund  Method — The  periodic  charge  for  depreciation  under  the 
sinking  fund  method  consists  of  a  constant  amount,  representing  the 
sum  of  money,  which,  placed  at  compound  interest  at  the  end  of  suc- 
cessive periods,  will  accumulate  to  the  amount  of  the  total  depreciation  of 
the  asset  during  its  life  term.'*  Assume  an  asset  costing  $100,  whose  scrap 
value  after  3  yeai's  will  be  $10.  If  interest  is  reckoned  at  5  per  cent,  the 
constant  amount  would  be  $28.55,  calculated  by  the  formula  for  sinking 
fund  installments  given  in  Chapter  IX,  thus: 

R  =  $90  ^  ^^'^^l~'^  =  $28.55 
.05 

As  the  interest  on  the  fund  would  be  debited  to  the  fund  and  credited 
to  income,  the  interest  must  be  added  to  the  periodic  charge  and  credited 
to  the  reserve."    The  entries  for  the  second  year  would  be : 

Entry  No.  1 

Depreciation  Fund $29. 98 

Cash $28.55 

Interest  Earned  ($28.55  X  .05) 1.43 

Entry  No.  2 

Depreciation  Expense 29 .  98 

Reserve  for  Depreciation 29 .  98 

If  desired  the  sinking  fund  method  can  be  used  to  calculate  the  periodic 
depreciation  charge  without  the  use  of  a  fund."  In  this  case  entry  No.  1 
would  be  omitted  but  the  depreciation  charge  would  be  calculated  as 
though  a  fund  were  maintained''  and  entry  No.  2  would  be  made.  In  the 
foregoing  illustration  the  total  depreciation  charge  for  the  first  year  would 
be  $28.55;  for  the  second  year  $28.55  plus  $1.43  ($28.55  X  .05),  or  $29.98; 
for  the  third  year  $28.55  plus  $2.92  (($28.55  +  $29.98)  X  .05),  or  $31.47.'* 

The  chief  objections  to  the  method  are  that  the  periodic  charge  increases 
as  the  asset  advances  in  age,"  and  that  the  assumed  constant  rate  of  in- 
terest may  vary  materially  from  the  current  rates  of  interest,'"  but  it  is 
adapted,  if  the  fund  is  maintained,  to  the  handling  of  the  depreciation  on  a 
large  single  piece  of  property."  The  fund  feature  of  the  sinking  fund 
method  is  especially  advisable  for  terminable  businesses  which  will  retire 
securities  when  the  assets  are  exhausted,  and  for  continuous  busines.ses 
which  do  not  require  funds  for  additions  and  betterments.*^ 
.  The  sinking  fund  method  is  sometimes  called  the  annuity  method,"'  a 
practice  which  is  unfortunate  as  it  ignores  the  difference  between  the  sink- 
ing fund  and  annuity  methods. 

Annuity  Method — The  annuity  method  of  calculating  depreciation  is 
based  on  the  theory  that  interest  is  a  cost  of  production.*"*  The  annuity 
method  determines  the  annual  charge  by  finding  sucli  sum  as,  when  de- 
ducted each  year  from  the  sum  of  the  remaining  investment  and  interest 


294' 


C.  P.  A.  ACCOUNTING 


thereon  at  a  given  rate,  will  reduce  the  investment  to  scrap  value,  and 
in  addition  return  the  full  amount  of  the  interest  on  the  investment  as 
it  stands  from  beginning  to  end  of  the  plant's  life.*°  The  annuity  method 
of  calculating  depreciation  treats  the  investment  in  the  fixed  asset  as  the 
present  worth  of  an  annuity.*'  The  calculation  of  the  periodic  charge 
for  an  asset  costing  $100,  whose  scrap  value  after  3  years  will  be  $10, 
interest  to  be  figured  at  5%,  is  shown  by  the  following  table:" 


Elements  in  Periodic  Charges 

Total 
Charge 

(d) 

Appraised 

Value 

Plus 

Interest 

(e) 

Depre- 

Periods 

Fixed 
Amount 

(a) 

Interest  on 

Fixed 
Amount 

(b) 

Interest  on 
Invest- 
ment 

(c) 

ciated  or 

Appraised 

Value 

(f) 

0 
1 
2 
3 

$ 

28.55 
28.55 
28.55 

$ 

"'i.43 
2.92 

$ 

5.00 
3.57 
2.08 

$ 

33.55 
33.55 
33.55 

$  

105.00 
75.02 
43.55 

$100.00 
71.45 
41.47 
10.00 

$85.65 

$4.35 

$10.65 

$100.65 

(a)  Calculated  as  in  Sinking  Fund 

Method. 

(b)  Calculated  as  in  Sinking  Fund 

Method. 


(c)  5  per  cent  of    (f). 

(d)  Sura  of  (a),  (b),  and  (c). 

(e)  Sum  of  (d)  and  (f). 

(f)  Difference  between  (e)  and  (d). 


The  "total  charge"  in  the  above  table  can  be  computed  by  the  use 
of  the  following  formula,  by  letting  "D"  stand  for  the  total  charge  or 
periodic  depreciation;  "C",  the  cost  of  the  asset;  "S",  the  scrap  value; 
"p",  the  present  value  of  $1  at  given  rate  due  at  scrapping  of  asset; 
and  "P",  the  present  value  of  annuity  of  $1  at  given  rate  for  as  many 
periods  as  life  of  asset.** 

p_   C-(SXp) 


The  total  depreciation  charge  or  "total  charge"  then  would  be: 
100—  (10  X  .86383760) 


D= 


2.72324803 


=  $33.55 


The  annuity  method  of  calculating  depreciation  automatically  .secures 
a  charge  for  interest  on  investment  as  a  part  of  tlie  depreciation  charge, 
a  result  that  should  be  avoided  because,  whether  or  not  interest  is  an 
element  in  cost,  its  inclusion  under  the  title  of  depreciation  is  misleading 
and  indefensible.""     The  real  depreciation  charge  is  the  same  under  the 


DEPRECIATION  295 

annuity  as  under  the  sinking  fund  method."*    The  journal  entry  for  the 
second  period  in  the  foregoing  illustration  would  be  :** 

Depreciation $33 .  55 

Interest  on  Investment $3 .  57 

Reserve  for  Depreciation  (S28 . 55  +  $1 . 43) 29. 98 

The  method  is  subject  to  the  objection  that  the  periodic  charges  in- 
crease with  the  age  of  the  asset.*^  The  application  of  the  mettiod  is 
mathematical  rather  than  practical,*^  but  the  method  is  adapted  for  cal- 
culating depreciation  on  long-term  leases  and  similar  assets.'* 

Maintenance  Method — The  maintenance  method  periodically  charges 
depreciation  with  an  amount  equal  to  the  cost  of  maintaining  the  asset 
during  the  period.*'  The  method  is  faulty  because  it  makes  the  charging 
of  depreciation  a  matter  of  business  convenience,  instead  of  a  necessary 
cost  of  production,  and  because  under  it  the  charge  fluctuates  violently 
and  tends  to  increase  with  the  age  of  the  asset.** 

Replacement  Method — The  replacement  method  makes  no  charge  for 
depreciation  but  in  lieu  of  such  an  expense  item,  it  charges  all  renewals 
and  replacements  to  revenue.*'  This  method  may  prove  a  satisfactory 
way  of  making  good  the  loss  due  to  depreciation  after  the  point  of 
normal  replacements  has  been  reached  in  a  plant  large  enough  for  the 
law  of  averages  to  equalize  the  replacement  charges.*'  The  method  is 
suitable  only  if  the  asset  consists  of  short-lived  units.**  It  disregards 
accrued  depreciation  up  to  the  point  of  normal  replacements.*** 

The  Fifty  Per  Cent  Method — The  fifty  per  cent  method  is  the  same  as 
the  replacement  method  except  that  after  the  assets  have  reached  the  re- 
newal stage,  the  total  depreciation,  which  maj'  or  may  not  be  booked,  is 
held  to  be  fifty  per  cent  of  the  cost  of  the  property,  the  constant  renewal 
of  parts  preventing  its  exceeding  that  percentage.**'  Although  this  method 
is  inadequate  as  a  means  of  prorating  the  depreciation  cost  over  the  prod- 
uct, it  may  serve  fairly  well  as  a  method  of  valuation.*** 

Appraisal  Method — By  the  appraisal  method,  the  periodic  charge  for 
depreciation  is  the  difference  in  value  between  the  appraisals  at  the  begin- 
ning and  end  of  the  period.**^  This  method  is  inadequate,  as  the  physical 
facts  of  depreciation  ai'e  not  usually  discernible  at  such  short  intervals.*** 
The  method  leads  to  inaccuracy  because  appraisals  usually  take  into 
account  market  fluctuations.*"'  The  appraisal  method  is  expensive  on 
account  of  engineers'  fees,**"  but  it  is  useful  as  an  occasional  check  upon 
the  other  methods.**' 

Insurance  Method — The  insurance  method,  so  called  because  of  the 
analogy  between  it  and  the  policy  practiced  by  some  firms  of  carrying  their 
own  fire  insurance,  is  a  means  of  financing  replacements  and  renewals 
rather  than  a  means  of  distributing  the  depreciation  cost  to  the  product.*** 
The  f und«provided  by  this  method  is  neither  designated  for  the  replacement 
of  any  specific  asset  nor  reserved  until  it  can  be  expended  in  replacing  the 
identical  asset  upon  which  it  was  figured,  but  it  is  spent,  wholly  or  partly, 


296  C.  P.  A.  ACCOUNTING 

during  the  same  year  in  which  it  is  created,  in  replacing  other  equipment 
or  purchasing  new  equipment/** 

Gross  Earnings  Method — The  gross  earnings  method  of  calculating  the 
periodic  charges  for  depreciation  estimates  the  expected  gross  earnings 
during  the  composite  life-period  of  the  plant,  and  then  prorates  the  loss 
due  to  depreciation  to  the  periods  on  the  basis  of  the  gross  earnings  for 
each  period  as  compared  with  the  estimate."*  If  the  product  is  standardized 
and  has  a  fairly  constant  selling  price,  gross  earnings  fluctuate  with  out- 
put, and  in  such  cases  the  gross  earnings  method  may  be  satisfactory."' 
The  method  is  sometimes  used  because  it  tends  to  make  the  periodic  net 
profits  equal,  but  this  reason  alone  does  not  justify  its  use.'" 


DEPRECIATION  297 


QUESTIONS 

depreciation 

Theory 

1.  What  is  depreciation ?  (Pa.,  Nov.,  1899* ;  N.  Y.,  June,  1900* ;  N.  Y., 
June,  1902*;  Md.,  Oct.,  1903*;  Pa.,  Nov.,  1904* ;' Wash.,  April,  1906*; 
N.  Y.,  Feb.,  1908*;  Md.,  Jan.,  1909*;  Ohio,  March,  1910*;  N.  Y.,  Jan., 
1911*;  Colo.,  Dec,  1913*;  Wis.,  April,  1914*;  N.  D.,  June,  1914*;  W.  Va., 
Mav,  1917*;  A.  I.  of  A.,  Nov.,  1917*;  Va.,  Nov.,  1918*;  Ind.,  Nov.,  1918*; 
N.  D.,  July,  1919.) 

2.  Give  your  understanding  of  the  term  "depreciation"  and  state 
wherein  it  is  or  is  not  equivalent  to  "wear  and  tear."     (111.,  May,  1909.) 

3.  State  the  theory  of  depreciation.  (Mich.,  Dec.^  1914*;  Ohio,  Nov., 
1915;  Ind.,  Nov.,  1918;  Ohio,  Nov.,  1918.) 

4.  Why  ought  depreciation  to  be  considered?     (N.  D.,  July,  1919.) 

5.  Is  the  necessity  of  a  provision  for  depreciation  commonly  recog- 
nized in  the  accounts  of  American  corporations?     (Ohio,  Oct.,  1919.) 

6.  What  are  your  views  as  to  the  necessity  of  a  provision  for  depre- 
ciation on  fixed  or  capital  assets?  (N.  Y.,  June,  1901*;  Mich.,  June, 
1908*;  Mo.,  Dec,  1914;  Ohio,  Oct.,  1919.*) 

7.  It  is  contended  that  it  is  unnecessary  to  write  off  depreciation  on 
(a)  Freehold  premises;  (b)  plant  and  machinery,  provided  that  they  are 
maintained  in  a  full  state  of  efficiency  out  of  revenue.  Give  briefly  your 
own  views  on  this  subject.     (111.,  May,  1905.) 

8.  From  an  audit  of  a  public  service  corporation  it  is  found  that  no 
depreciation  of  capital  assets  has  been  provided  by  a  charge  against  earn- 
ings, the  officials  believhig  that  appreciation  of  certain  real  estate  offsets 
a  fair  amount  of  depreciation.  Is  this  a  proper  disposal  of  the  matter? 
Give  reasons.  (Mass.,  June,  1912*;  111.,  May,  1916*;  Mass.,  Oct.,  1916; 
Ohio,  Nov.,  1917*;  A.  I.  of  A.,  Nov.,  1918*;  A.  I.  of  A.,  May,  1921.*) 

9.  Do  you  consider  it  good  accounting  practice  to  charge  off  deprecia- 
tion on  machinery  in  years  when  the  operation  of  the  plant  results  in  a 
loss?     Give  reasons.     (N.  Y.,  June,  1919;  N.  C,  Nov.,  1919.*) 

10.  State  the  object  of  the  Depreciation  account.  (N.  Y.,  Dec,  1898; 
Mich.,  July,  1906*;  Va.,  Oct.,  1911.*) 

11.  What  is  the  effect  on  a  business  of  depreciation?  (Pa.,  Nov., 
1900*;  N.  Y.,  Jan.,  1901*;  Pa.,  Nov.,  1904*;  Pa.,  May,  1906;  A.  I.  of  A., 
May,  1920.*) 


2[)H  C.  P.  A.  ACCOUXTIXG 

12.  Define:  Obsolescence.     (Ill,,  Dec,  1918.) 

13.  What  is  the  difference,  if  any,  between  depreciation,  obsolescence 
and  depletion?     (W.  Va.,  May,  1919.) 

14.  A  company  manufacturing'  tin  tags  charges  to  cost  of  manufacture 
(as  depreciation)  one-fourth  of  the  cost  of  the  stamping  machine,  which 
had  been  in  service  about  one  year.  The  life  of  this  machine  was  esti- 
mated to  be  ten  years,  but  owing  to  the  discovery  by  a  competitor  of  a 
new  method  of  stamping,  which,  while  still  imperfect,  promises  to  revolu- 
tionize the  business,  the  stamping  machine  now  in  use  will  probably  be 
obsolete  within  a  period  of  three  years.  What  would  you  say  concerning 
the  propriety  of  the  above  charge  to  prime  cost?     (N.  Y.,  Jan.,  1918.) 

15.  Define  the  difference  between  fluctuation  and  depreciation  in  the 
value  of  assets.     (N.  Y.,  June,  1919.) 

16.  Define  and  distinguish  "depletion"  and  "depreciation"  and  state 
how  you  would  handle  these  through  the  Balance  Sheet  and  the  operating 
statement.     (N.  C,  June,  1920.) 

17.  Name  three  of  the  principal  elements  that  cause  obsolescence. 
(N.  Y.,  June,  1919.) 

18.  Can  you  name  some  other  reason  why  depreciation  should  be  con- 
sidered in  respect  to  machinery  other  than  that  of  wear  and  tear?  (111., 
May,  1901;  Mass.,  June,  1910.) 

19.  Name  the  advantages  or  disadvantages  of  the  following  methods 
of  bringing  on  to  the  books  of  a  company  the  depreciation  on  its  machin- 
ery: (a)  Crediting  Machineiy  account  with  10  per  cent  of  the  balance  of 
the  account  each  year  and  charging  profit  and  loss;  (&)  crediting  a  reserve 
for  machinery  dejireciation  with  10  per  cent  of  the  balance  of  the  account 
each  5'ear  and  charging  profit  and  loss.  How  can  you  combine  the  best 
features  of  both  the  above  methods?     (111.,  May,  1907.) 

20.  A  company  engaged  in  the  manufacturing  business  has  instructed 
its  accountant  to  show  depreciation  as  an  operating  expense  and  a  credit 
to  plant,  to  set  up  a  depreciation  reserve  and  create  a  depreciation  cash 
fund.  Prepare  the  necessai-y  Journal  entries  to  illustrate  the  idea  of  the 
company,  and  discuss  the  question.     (111.,  Nov.,  1904.) 

21.  What  do  you  consider  the  best  way  of  entering  on  the  books  of  a 
manufacturing  company  the  amount  written  off  to  profit  and  loss  for 
depreciation  on  (a)  buildings;  (b)  large  or  fixed  tools;  (c)  small  or 
expense  tools?     (N.  Y.,  Feb.,  1908.) 

22.  A  public  service  corporation  that  regularly  sets  aside  from  its 
profits  a  sufficient  amount  to  provide  for  depreciation  removes  part  of  its 
old  plant  and  replaces  it  with  a  larger  and  more  costly  one.  The  old 
plant  is  sold  for  scrap.  How  should  the  cost  of  the  new  plant  and  the 
proceeds  from  the  sale  of  the  old  plant  be  treated  in  the  accounts  of  the 
company?  Give  reasons.  (N.  Y.,  Feb.,  1909;  Kan.,  May,  1916;  N.  C, 
Nov.,  1919*;  N.  C.  June,  1920.*) 

23.  State  various  ways  of  treating  depreciation  upon  the  books.  (N.  Y., 
June,  1898*;  N.  Y.,  Jan.,  1901*;  N.  Y.,  Jan.,  1904*;  111.,  May,  1904*; 


DEPRECIATION  299 

Mich.,  July,  1006*;  N.  Y.,  Feb.,  190S*;  III.,  May,  1908*;  Mrl.,  Jan.,  1909*; 
III.,  May,  1909*;  N.  Y.,  Feb.,  1910*;  Ohio,  March,  1910*;  Wash.,  May, 
1910*;  Va.,  Oct.,  1911*;  Wash.,  June,  1912*;  Mass.,  June,  1912*;  Ohio, 
Nov.,  1913*;  Wis.,  April,  1914;  Mo.,  Dec,  1914*;  Mo.,  Dec,  1915*;  Kan., 
Dec,  1915*;  Mich.,  Dec,  1916*;  W.  Va.,  May,  1917*;  Ohio,  Nov.,  1918*; 
S.  C,  Sept.,  1919.*) 

24.  What  disposition  would  you  make  of  any  depreciation  reserve 
account  when  constructing  the  Balance  Sheet?  State  your  reason  for 
above  answer.  (111.,  May,  1904*;  Md.,  Jan.,  1909*;  Mass.,  June,  1910*; 
Va.,  Oct.,  1912;  Ohio,  Nov.,  1913*;  Ohio,  Nov.,  1915*;  Cal.,  May,  1916*; 
Ohio,  Nov.,  1917*;  Ind.,  Nov.,  1918.*) 

25.  How  would  you  treat  in  the  Balance  Sheet  of  a  company  you  were 
auditing:  Reserve  for  depreciation  of  capital  assets;  (a)  invested  in  se- 
curities; (b)  not  specially  invested?     (111.,  May,  1914.) 

26.  What,  in  your  opinion,  is  the  proper  treatment  of  the  depreciation 
on  plant  and  equipment  in  arriving  at  the  profit  or  loss  of  a  business  for 
a  specific  fiscal  period?     (Ohio,  Dec,  1908.) 

27.  Are  depreciation  charges  properly  considered  to  be  in  the  nature 
of  expense  items,  or  as  an  application  of  the  profits?  Explain.  (N.  Y., 
Jan.,  1918.) 

28.  In  order  to  facilitate  the  preparation  of  monthly  Profit  and  Loss 
statements  by  a  corporation,  how  would  you  recommend  depreciation  be 
treated  on  its  books  from  month  to  month?     (Wash.,  April,  190B.) 

29.  The  balance  on  Machinery  Depreciation  account  shows  an  increase 
for  the  year  of  the  amount  provided  out  of  income  which  is  computed  at 
the  rate  of  4  per  cent  on  the  balance  of  Machinery  account  at  the  com- 
mencement of  the  year.  The  method  of  keeping  the  Machinery  and  Ma- 
chinery Depreciation  accounts  has  been  in  force  from  the  commencement 
of  operations.  Draft  your  comments  as  auditor  of  these  accounts,  assum- 
ing that  no  item  other  than  those  above  mentioned  call  for  any  comments. 
(A.  I,  of  A.,  May,  1918.) 

30.  The  book  value  of  the  plant  of  a  corporation  has  been  reduced  to 
merely  a  nominal  sum.  Under  this  condition,  state  (a)  whether,  periodi- 
cally, a  reservation  should  be  made  of  an  amount  estimated  to  cover  depre- 
ciation; (6)  the  reasons  supporting  your  answer.     (Mass.,  June,  1912.) 

31.  How  should  expenditures  for  repairs  or  replacements  be  treated 
in  so  far  as  they  relate  to  the  question  of  depreciation?  (Mich.,  June, 
1913.) 

32.  Corporation  "X"  makes  a  practice  of  charging  to  expenses  and 
carrying  to  Depreciation  Reserve  account  every  half  year  a  certain  per- 
centage of  the  book  value  of  its  plant  and  machinery.  What,  in  your 
opinion,  is  the  correct  method  of  dealing  in  this  case  with  repairs  and  re- 
newals, i.e.,  should  the  latter  be  charged  to  Profit  and  Loss  or  can  they 
be  properly  charged  to  Depi'eeiation  Reserve  account?  Give  reasons  for 
your  answer,     (Wash.,  Sept.,  1907.) 


300  C.  P.  A.  ACCOUNTING 

33.  What  is  the  effect  on  a  business  of  reserve  for  depreciation  ?  (Pa., 
May,  1906.) 

34.  Explain  the  theory  of  a  depreciation  reserve.  As  auditor  of  a 
manufacturing  concern,  what  would  be  your  duties  in  respect  thereto ;  and 
what,  if  any,  charges  would  you  permit  to  be  made  to  the  account?  (111., 
May,  1912.) 

35.  What  are  the  two  principal  purposes  of  a  Reserve  for  Deprecia- 
tion account?     (111.,  May,  1912.*) 

36.  You  are  asked  by  a  client  to  discuss  with  him  the  question  of 
reserves  for  depreciation  and  depletion  of  his  various  capital  assets. 
State  your  position  on  this  subject  and  enumerate  the  considerations  you 
would  advance  in  support  thereof.  Would  you,  or  would  you  not,  be 
guided  by  the  rules  laid  down  by  the  intei'nal  revenue  authorities  in 
deciding  upon  the  rates  to  be  used?     (A.  I.  of  A.,  May,  1918.) 

37.  Explain  the  relationship  between  a  sinking  fund  and  an  allowance 
for  depreciation.  It  is  claimed  that  in  municipal  enterprises  the  require- 
ments that  rates  must  be  high  enough  to  provide  both  for  a  sinking  fund 
to  pay  off  the  bonds  and  also  for  a  reserve  for  depreciation  with  which  to 
replace  the  plant  results  in  a  double  charge  to  consumers.  Criticize  or 
explain  this  theory.     (A.  I.  of  A.,  June,  1917.) 

38.  Under  what  circumstances  should  deduction  be  made  for  deprecia- 
tion?    (N.  Y.,  Dec,  1897.) 

39.  What  classes  of  property,  if  any,  in  your  opinion,  are  exempt 
from  depreciation?  Give  reasons.  (N.  Y.,  Dec.,  1897*;  Mich.,  Dec, 
1906.) 

40.  If  asked  to  give  advice  concerning  the  proper  rates  per  cent  to  be 
adopted  in  providing  for  the  account  for  depreciation  on  buildings,  ma- 
chinery, tools,  etc.,  what  could  vou  recommend?  (N.  Y.,  Jan.,  1901*; 
N.  Y.,  Jan.,  1907;  N.  Y.,  Jan.,  1908.*) 

41.  Name  a  fair  allowance  for  depreciation  per  annum  on  the  follow- 
ing plant  assets:  (a)  Real  estate,  including  fences,  sidewalks,  tracts,  etc; 
(b)  building  and  building  fixtures — fireproof  construction;  (c)  factoi*y 
equipment,  including  benches,  cupboards,  etc.;  (d)  machinery,  both  iron 
and  woodworking;  (e)  fixed  tools,  both  iron  and  woodworking;  (/)  loose 
tools,  both  iron  and  woodworking;  (g)  power  plant;  (h)  electric  wiring 
and  apparatus,  including  dynamos,  motors,  etc;  (i)  blower  system;  (j) 
office  furniture  and  fixtures,  including  typewriters,  adding  machines, 
graphophones,  multigraphs,  etc.;  {k)  horses  and  mules;  (l)  wagons,  har- 
ness and  trappings;  (m)  patterns,  iron  and  wood,  and  drawings.  (Mich., 
June,  1908.) 

42.  A  machine  costing  $81  is  estimated  to  have  a  life  of  four  years, 
with  a  residual  value  of  $16.  Prepare  a  statement  showing  the  annual 
charge  for  depreciation  according  to  each  of  the  following  methods:  (o) 
Straight  line;   (b)  constant  percentage  of  diminishing  value;   (c)  annuity 


DEPRECIATION  301 

inelhod.  (For  eonvenieiice  iu  aiitlmietical  calculation  assume  the  rate  of 
interest  to  be  10  per  cent.)  Discuss  the  significance  of  each  of  the  meth- 
ods.    (A.  I.  of  A.,  June,  1917.) 

43.  Explain  six  different  ways  of  apportioning  depreciation  charges 
from  year  to  year  and  point  out  clearly  their  distinguishing  features. 
(N.  Y.,  June,  1913.) 

44.  A  manufacturing  concern  owning  lands,  buildings,  engines,  boilers, 
and  machinerj',  has  been  in  operation  for  ten  years,  but  has  made  no 
allowance  for  depreciation,  current  repairs  having  been  charged  to  Oper- 
ating Expenses  and  new  machinery  bought  having  been  charged  to  Plant 
account.  You  are  given  carte  blanche  in  the  matter  of  arranging  for  such 
depreciation  at  the  time  of  the  audit  and  also  in  the  matter  of  providing 
for  it  in  the  future. 

State  the  steps  you  would  take  and  the  entries  you  would  direct  for 
both  cases,  using  your  own  figures.  (Fla.,  July,  1909.) 

45.  An  individual  buys  a  fleet  of  ships.  He  then  forms  a  corporation 
to  take  them  over  at  double  the  sum  paid  by  him,  payable  one-half  in 
debenture  bonds  of  the  company  and  one-half  in  its  capital  stock.  A  sink- 
ing fund  is  to  be  provided  for  the  gradual  retirement  of  the  debenture 
bonds.  A  public  accountant  is  called  in  at  the  end  of  five  years  to  make 
up  the  accounts.  He  insists  on  creating  a  depreciation  fund  based  on 
the  full  consideration  paid  by  the  corporation.  The  directors  argue  that 
the  depreciation  fund  should  be  based  on  the  amount  of  debenture  bonds 
issued,  on  the  theorj'  that  the  capital  stock  issued  to  the  vendor  was  in  the 
nature  of  a  bonus  and  did  not  represent  any  real  value.  State  your  views 
regarding  the  two  propositions.     (N.  Y.,  June,  1904.) 

46.  What  do  you  consider  to  be  the  proper  basis  for  providing  for 
depreciation  or  exhaustion  of  the  following  classes  of  property:  (o) 
buildings;  (6)  machinery;  (c)  small  tools;  (d)  freight  cars;  (e)  motor 
trucks;  (/)  patterns;  (g)  patents;  (h)  mine  equipment;  (i)  ore  mine; 
(j)  special  machinery  for  the  manufacture  of  war  munitions.  (111.,  Dec, 
1918.) 

47.  Give  rates  of  depreciation  commonly  used  for  the  following  classes 
of  property,  stating  the  method  of  applying  them;  it  being  understood 
that  ordinary  repairs  and  maintenance  are  charged  against  earnings:  (a) 
buildings,  brock  mill  construction;  (6)  machinery,  large  machinery  such 
as  used  in  automobile  manufacturing;  (c)  steam  boilers,  stationary;  (d) 
ofiRce  furniture.     (Mass.,  Oct.,  1914.) 

48.  Mention  four  items  of  assets  subject  to  depreciation  in  the  Balance 
Sheet,  and  state  the  annual  rate  that  you  would  recommend  to  be  charged. 
(N.  Y.,  June,  1902.) 

49.  Name  the  elements  essential  to  the  proper  calculation  of  a  depre- 
ciation charge.  (N.  Y.,  June,  1911*;  N.  Y.,  June,  1913*;  Mo.,  Dec, 
1914*;  Mass.,  Oct.,  1915*;  A.  I.  of  A,,  May,  1919;  S.  C,  Sept.,  1919*; 
Wis.,  Nov.,  1919.*) 


302  C.  P.  A.  ACCOUNTING 

50.  List  the  various  kinds  of  plant  assets  and  indicate  the  various  per 
cent  of  allowances  for  depreciation,  making  si;ch  explanations  as  are 
neeessarj\     (Mich.,  June,  1914.) 

51.  Name  some  reasons  why  it  is  important  to  keep  distinct  the 
various  items  of  cost  in  the  construction  of  a  building  containing  boilers, 
engines,  shafting  and  heating  plant.  In  the  erection  of  the  building  itself, 
why  should  the  cost  of  the  foundations  be  kept  distinct  from  the  balance 
of  the  building?     (EL,  May,  1904.) 

52.  Give  some  general  principles  which  will  guide  you  in  determining 
whether  too  much  or  too  little  provision  has  been  made  for  depreciation 
of  buildings,  machinery',  tools,  goodwill,  patents,  franchises.  Would  a  flat 
rate  cover  all  these  assets  satisfactorily?  (A.  I.  of  A.,  Nov.,  1918;  W.  Va., 
May,  1919.) 

53.  Where  a  plant  is  purchased  as  a  whole  without  valuation  of  its 
different  parts,  how  would  you  provide  for  depreciation?  (Pa.,  May, 
1902*;  Pa.,  Nov.,  1904.) 

54.  In  the  event  of  a  difference  of  opinion  between  auditor  and  direc- 
tors concerning  the  rate  of  depreciation  on  plant  and  machinery  as  would 
involve  an  important  alteration  in  the  proposed  rate  of  dividend,  how 
can  the  matter  be  settled  to  the  satisfaction  of  both  parties?  (N.  Y., 
June,  1899.) 

55.  Upon  the  audit  of  the  partnership  accounts  of  a  manufacturing 
business,  the  following  condition  is  revealed :  Depreciation  or  discount 
from  the  value  t)f  a  certain  class  of  the  inventory  instead  of  being  30 
per  cent  as  in  prior  years  is  shown  as  10  per  cent.  What  would  you 
deduce  from  these  facts  and  what  would  you  feel  called  upon  to  do  in 
this  instance?     (Kan.,  May,  1916.) 

56.  What  is  your  understanding  of  the  relationship  between  a  sinking 
fund  and  an  allowance  for  depreciation?     (Ind.,  Nov.,  1917.) 

57.  State  three  important  methods  of  calculating  depreciation,  briefly 
explaining  each.  (Fla.,  April,  1907*;  Wash.,  June,  1912*;  Mass.,  Oct., 
1914;  Del.,  June,  1915;  Mass.,  Oct.,  1915*;  N.  Y.,  Jan.,  1916*;  Mich., 
Dec,  1916* ;  Ohio,  Nov.,  1917* ;  Mich.,  June,  1919* ;  N.  D.,  July,  1919* ; 
Ohio,  Oct.,  1919*:  Md.,  Oct.,  1919;  A.  I.  of  A.,  May,  1920.*) 

58.  In  what  eases  should  depreciation  be  calculated  as  a  percentage  to 
cost?  In  what  cases  as  a  percentage  to  the  net  value  (cost  less  deprecia- 
tion reserve)  ?    Give  the  reasons  in  each  case.     (111.,  May,  1915.) 

59.  The  following  are  the  methods  usually  adopted  for  the  writing  off 
of  depreciation:  (a)  A  fixed  proportion  of  original  value;  (6)  a  fixed 
percentage  of  reduced  balances  throwing  the  greater  part  of  the  loss  on 
the  first  few  years;  (c)  a  sinking  fund.  Illustrate  the  three  methods  to 
reduce  $200  to  $100  in  ten  years.     (Md.,  Oct.,  1903.) 

60.  Why  is  an  excessive  allowance  for  depreciation  objectionable?  (&) 
On  what  grounds  is  this  usually  condoned?     (Ohio,  Nov.,  1917.) 


DEPRECIATION  303 

61.  What  is  the  effect  of  an  excessive  charge  for  depreciation?  (Ohio, 
Nov.,  1917*;  Ohio,  Nov.,  1918*;  Ohio,  Oct.,  1919.) 

62.  State  what  verification  you  would  make  of  the  reserve  for  deprecia- 
tion.    (N.  C,  Aug.,  1917.) 

63.  To  what  extent  should  an  auditor  hold  himself  responsible  for  the 
con-ectness  of  depreciation?  (N.  Y.,  Dec,  1896;  N.  Y.,  June,  1899*; 
Va.,  Nov:,  1910*;  Mich.,  June,  1912*;  Mich.,  June,  1913*;  Wash.,  Nov., 
1913*;  111.,  May,  1914.) 

64.  State  how  you  would  determine  that  the  amount  of  depreciation 
charged  off  annually  was  correct.  (Wash.,  May,  1910*;  Mich.,  June, 
1912*;  N.  C,  Nov.,  1918;  N.  C,  June,  1919*;  N.  C,  Sept.,  1919.*) 

65.  Discuss  concisel}'  your  conception  of  an  auditor's  duties  in  con- 
nection with  an  annual  audit,  where  a  difference  of  opinion  exists  between 
the  auditor  and  the  client  as  to  the  sufficiency  of  depreciation  or  depletion 
charges.     (Ohio,  Oct.,  1919.) 

66.  In  an  audit  of  the  books  of  the  "A"  Company  you  find  that  the 
fixed  depreciable  asset  accounts,  their  reserve  for  depreciation  accounts 
and  the  operating  expense  accounts  have  been  improperly  kept.  Charges 
which  should  have  been  made  against  the  reserve  for  depreciation  accounts 
have  been  made,  in  some  cases,  to  the  operating  expense  account  and,  in 
other  cases,  to  the  asset  accounts.  Scrapped  assets  have  been  allowed  to 
remain  in  the  asset  accounts  at  original  cost.  The  balances  in  the  reserve 
for  depreciation  accounts  exceed  the  balances  of  the  depreciable  ass^^t 
accounts.  Outline  the  procedure  which  you  would  follow  in  analyzing 
these  accounts  and  state  all  the  effects  upon  the  net  profits  for  the  years 
under  review.     (Wis.,  Nov.,  1919.) 

67.  What  are  the  duties  of  an  auditor  when  he  finds  no  charges  made 
against  maintenance  or  other  accounts  for  depreciation  of  plant?  Should 
he  be  concerned  with  the  condition  in  this  respect  which  obtains  through- 
out the  period  prior  to  the  one  to  be  covered  by  his  audit?  How  should 
he  report  to  his  client  having  regard  for  the  possibility  of  his  report  being 
used  for  purposes  of  obtaining  loans,  obtaining  additional  capital  in  the 
business,  or  selling  some  part  of  the  existing  capital  interests?  (N.  Y., 
Feb.,  1909.) 

68.  In  the  preparation  of  a  Balance  Sheet  explain  the  basis  upon  which 
you  would  ascertain  that  reserve  for  depreciation  was  properly  valued. 
(Wis.,  April,  1914.) 

69.  What  are  the  principal  conditions  which  would  influence  you  in 
fixing  the  rate  of  depreciation  on  buildings,  machinery  and  tools,  fixtures 
and  patterns?  State  your  reasons  pertaining  to  each  class  of  property. 
(N.  Y.,  June,  1918.) 


304  C.  P.  A.  ACCOUNTING 


PROBLEMS 

DEPRECIATION 

1.  A  factory  consists  of  two  blocks  of  buildings,  "A"  and  "B."  On 
the  1st  of  January,  1907,  "A"  contains  engine  and  boiler  which  cost  $4,000, 
and  machinery  costing  $13,000;  "B"  contains  machinery  costing  $7,000. 
The  following  are  purchases  of  machinery :  October  1,  1907,  "A,"  $1,000 ; 
July  1,1908,  "A,"  $750;  "B,"  $1,500;  April  1,  1909,  "A,"  $600;  "B,» 
$900 ;  October  1,  1909,  "B,"  $250. 

On  January  1,  1908,  machinery  (costing  January  1,  1907,  $1,000)  is 
sold  from  "A"  for  $625,  and  on  July  1,  1908,  machinery  (costing  $1,300 
January  1,  1907)  is  sold  from  "B"  for  $1,000. 

The  accounts  are  made  up  to  December  31  each  year.  On  December 
31,  1909,  the  whole  premises  and  contents  are  destroyed  by  fire  and  the 
fire  insurance  company  agrees  to  pay  upon  the  following  basis :  engine  and 
boiler,  cost  price,  less  depreciation  8  per  cent  per  annum  upon  that  sum ; 
machinery  in  "A,"  cost  less  depreciation  at  10  per  cent  per  annum  and 
upon  diminishing  value;  machinery  in  "B,"  cost,  less  depreciation  at  7^2 
])er  cent  per  annum  upon  diminishing  value. 

Prepare  Ledger  accounts  showing  how  much  is  recoverable  upon  this 
basis.  (111.,  May,  1910.) 

2,  In  your  examination  of  the  Aiatomobile  Delivery  Truck  account  of 
a  company,  you  find  the  following  entries: 

Debits 

January  1,  1914,  Trucks  1,  2,  3,  4,  at  $1,200 $4,800 

July  1,  1914,  Truck  5 1,500 

August  1,  1914,  Truck  6 1,500 

Credits 

August  1,  1914,  Truck  2 $900 

September  1,  1914,  Truck  4 750 

Balance,  September  1,  1914,  $6,150. 

The  Reserve  for  Depreciation  for  Automobile  Delivery  Truck  account 
stood  credited  on  January  1, 1914,  with  $1,800. 

Upon  analyzing  the  transactions  represented  by  these  items,  you  find 
the  following  facts: 

(a)  Truck  5,  purchased  July  1  replaced  Truck  1.  The  portion  of  the 
reserve  for  depreciation  accumulated  on  January  1  for  Truck  1  amounted 
to  $900.     Truck  5  was  purchased  on  open  account. 

(b)  Truck  2  was  traded  in  for  $850  on  the  purchase  of  Truck  6 
costing  $1,500.     The  difference  was  paid  in  cash.     The  reserve  which  had 


DEPRECIATION 


305 


been  accumulated  for  depreciation  on  Truck  2  on  January  1  amounted  to 
$300. 

(c)  Truck  4  was  totally  destroyed  by  an  accident  September  1.  The 
reserve  for  depreciation  on  this  truck  amounted  on  January  1  to  $300 
and  it  was  insured  for  $750. 

Assume  the  rate  of  depreciation  to  be  25  per  cent  per  year. 

Give  Journal  entries  which  would  properly  record  the  above  facts  and 
show  the  balances  of  all  accounts  affected,  as  of  September  1,  1914. 

(Wis.,  April,  1915.) 

3.  From  the  data  given  below,  state  clearly  and  explain  at  least  three 
different  methods  of  arriving  at  the  amount  to  charge  annually  for  the 
depreciation  of  any  one  or  all  of  the  following  items: 


Items 


Value 


Estimated 
Life 


Scrap 
Value 


Buildings . 
Machinery 

Tools 

Patterns. . 


$50,000 

20,000 

5,000 

10.000 


50  years 

20  years 

5  years 

3  vears 


$1,000 

2,000 

100 

100 


(Wis.,  April,  1915.) 

4.  A  corporation  has  been  accustomed  to  charge  the  purchase  of  ma- 
chinery to  the  Machinery  account,  and  each  year  to  charge  the  Manufac- 
turing account  and  to  credit  a  Reserve  for  Depreciation  account  with  an 
amount  which  will  offset  the  cost  of  the  machinery  by  the  time  it  is  esti- 
mated that  it  will  be  advisable  to  scrap  the  machines.  During  the  period 
that  you  have  been  employed  to  audit  the  account,  you  find  that  the 
corporation  has  sold  two  machines  for  $500  each,  and  this  amount  has 
been  credited  to  the  Machinery  account.  One  of  them  cost  $1,000  and  the 
amount  reserved  for  depreciation  on  this  machine  is  $600.  The  other  cost 
$1,500  and  the  amount  reserved  for  depreciation  on  this  machine  is  $850. 
Make  the  adjusting  entries  to  correct  the  books.     (Mass.,  June,  1913.) 

5.  An  engine  installed  in  a  factory  January  1,  1914,  at  a  cost  of 
$1,000  is  replaced  by  one  of  larger  capacity  December  31,  1917,  costing 
(second  hand)  $2,800.  The  discarded  machine  was  sold  for  $900.  The 
cost  of  making  the  change  was  $200.  It  has  been  the  practice  of  the 
company  to  charge  off  10  per  cent  depreciation  annually  (on  the  diminish- 
ing basis)  carrying  the  credit  to  a  Depreciation  Reserve  account.  Make 
the  necessary  Journal  entries.     (111.,  Dec,  1918.) 

6.  A  machine  costing  $12,000  was  estimated  to  have  a  life  of  twelve 
years  with  a  residual  value  of  $1,500.  At  the  close  of  each  year  a  charge 
of  $875  was  made  to  depreciation  and  a  like  amount  credited  to  "reserve" 
for  depreciation.  Just  prior  to  closing  the  books  at  the  end  of  the 
twelfth  year  the  machine  was  discarded  and  sold  for  $2,000  (cash)  and  a 
similar  machine  was  bought,  costing  $16,000.  Show  the  Journal  entries 
you  would  make  to  close  the  books  at  the  end  of  the  twelve  years,  in  order 


306 


C.  P.  A.  ACCOUNTING 


to  close  the  transactions  and  to  make  necessary  adjustmenfs.     (A.  I.  of  A., 
June,  1917*  J  Ind.,  Nov.,  1918.) 

7.     The  "A"  Manufacturing  Company  has  four  general  types  of  de- 
preciable assets. 


Buildings 

Machinery  "A". . 
Machinery  "B" . . 
Office  Equipment 


Rate 


2% 

20% 
10% 


Cost 


$51,000 

11,000 

12,000 

4.100 


Scrap 
Value 


$1,000 

1,000 

2,000 

100 


The  directors  desire  to  keep  but  one  Reserve  for  Depreciation  account 
and  request  you  to  determine  the  composite  rate  which  may  be  used  in 
determining  the  annual  depreciation  charge. 

Determine  the  composite  rate  as  requested,  tabulate  the  necessary  facts 
used  in  determining  it,  and  comment  upon  the  practicability  of  such  a 
plan.  (Wis.,  Nov.,  1919.) 

8.  A  manufacturing  concern  has  annually  for  the  past  six  years  made 
provision  at  the  rate  of  10  per  cent  per  annum  for  depreciation  of  its 
plant  and  machinery,  crediting  the  amount  of  such  depreciation  to  a 
suitable  Reserve  account.  During  the  year  an  engine  which  cost  originally 
$5,000  was  replaced  by  an  improved  engine  costing  $6,800.  The  cost  of 
the  new  engine  was  charged  to  Machinery  account  at  time  of  purchase. 
$300  was  realized  from  the  salvage  of  the  old  engine,  this  amount  being 
credited  to  "Scrap  Sales,"  when  received,  and  later  closed  to  Profit  and 
Loss. 

Draft  the  adjustment  entries  which  j'ou  consider  necessary  and  explain 
the  principle  upon  which  these  entries  are  based. 

(Ohio,  Nov.,  1918.) 

9.  "A,"  "B,"  and  "C"  form  a  partnership  January  1,  1917.  "A" 
invests  $65,000;  "B,"  $45,000;  and  "C,"  $40,000.  Profits  are  to  be  deter- 
mined semiannually  and  are  to  be  shared  in  the  ratio  of  the  original 
investments.  No  interest  to  be  calculated  on  Partners'  Capital  accounts. 
At  the  end  of  six  months  the  Ledger  contains  the  following  balances: 


Accounts  Receivable $100,000 

Accounts  Payable 32,400 

Insurance 6,000 

Interest  Paid 200 

Interest  Received 500 

Notes  Receivable 20,000 

Notes  Payable 20,000 

Purchases 600,000 

Purchase  Allowances 1,000 

Purchase  Discounts 6,000 

Purchase  Returns 3.000 


Sales $592,600 

Sales  Returns 2,000 

Sales  Allowances 500 

Salaries  and  Wages 30,000 

Rent 5,000 

Securities  Owned 5,000 

Miscellaneous  Expense. . .  11,800 

Cash 25.000 

"A"  Capital  Account 65,000 

"B"  Capital  Account. . . .  45,000 

"C"  Capital  Account. . . .  40,009 


DEPRECIATION  307 

The  inventory  of  stock  on  June  30  amounted  to  $110,000.  Of  the  ac- 
counts receivable,  it  is  estimated  that  2  per  cent  are  uncollectible.  During 
the  six  months  the  firm  discounted  notes  receivable  amounting  to  $10,000 
of  Avhich  a  note  of  $1,000  will  not  be  due  until  August  1. 

The  insurance  premiums  paid  were  for  insurance  covering  a  three-year 
period,  expiring  January  1,  1920. 

The  following  minor  bills  are  outstanding:  telephone  and  telegraph, 
$20;  electric  light,  $130;  drayage,  $100.  The  estimated  taxes  for  the 
year  1917  are  $6,800. 

The  notes  payable  are  represented  by  two  notes  of  $10,000  each,  for 
four  months  at  6  per  cent  interest,  dated  May  1  and  June  1,  respectively. 
The  first  note  was  discounted ;  interest  on  the  second  note  was  payable  at 
maturity. 

On  July  1,  1917,  "A'-  withdraws  from  the  firm,  his  interest  being  pur- 
chased by  "B,"  "C,"  and  "D"  in  such  proportions  that  the  capital  of  all 
pax'tners  shall  be  equal.  It  is  agreed  by  all  parties  that  the  value  of  the 
goodwill  is  $15,000,  which  has  been  created  during  the  period  of  the 
partnership,  and  which  amount  it  is  decided  should  be  set  up  on  the  books. 

(a)  Construct  trial  balance  as  at  July  1;  (b)  prepare  Journal  entries 
indicating  the  necessary  adjustments;  (c)  prepare  a  Profit  and  Income 
Statement  for  six  months  ending  June  30;  and  (d)  prepare  a  Balance 
Sheet  as  of  July  1  after  the  withdrawal  of  "A"  and  the  entrance  of  "D." 

(111.,  Dec,  1917.) 
10.  (a)  The  X  Y  Z  company  established  for  ten  years  has  a  machinery 
and  equipment  account  which  has  been  increased  from  year  to  year  as 
new  equipment  purchases  have  been  made.  It  appears  also  that  certain 
renewals  and  repairs  have  been  charged  to  this  account.  Each  year  a 
credit  has  been  made  to  the  account  for  depreciation,  offset  by  correspond- 
ing debit  to  profit  and  loss  account,  the  ratio  of  depreciation  being  ade- 
quate. The  company  now  disposes  of  a  part  of  its  plant  at  a  price  equal 
to  what  was  paid  for  it  seven  years  previously  and  credits  the  entire  amount 
to  machinery  and  equipment  account.  What  adjustments,  if  any,  are 
needed  to  correct  the  account? 

(b)  The  company  also  has  several  delivery  trucks  charged  to  truck 
account  at  cost,  against  which  it  has  set  up  depreciation  at  end  of  each 
year  by  credit  to  a  separate  reserve  for  depreciation  of  trucks,  debiting  the 
amount  to  profit  and  loss  account.  A  truck  was  purchased  January  1, 
1918,  for  $4,000.  Depreciation  has  been  provided  at  20  per  cent  per 
annum.  On  December  31,  1919,  the  truck  is  wrecked  by  collision.  $1,000 
is  obtained  from  the  insurance  company  and  $250  obtained  from  salvage. 
What  entries  are  needed  to  adjust  the  ledger  accounts? 

(A.  I.  of  A.,  Nov.,  1920.) 


308 


C.  P.  A.  ACCOUNTING 


]1.   (a)  Determine  the  average  life  of  the  following  fixed  assets  belong- 
ing to  the  Western  Hardware  ComiDany: 


Assets 


Buildings . 
Machinery 

Tools 

Patterns.  . 


Cost 


$100,000 
70,000 
20,000 
10,000 


Estimated 
Scrap  Value 


J.3o .  000 

25.000 

5.000 


Estimated 
Life  in  Years 


20 
15 
10 


(b)  After  determining  the  average  life  of  the  fixed  assets,  state  the 
amount  of  annual  depreciation  by  the  straight-line  method. 

A.  I.  of  A.,  May,  1921.) 

12.  A  company  has  acquired  machinery,  which  cost  $100,000,  which  it 
expects  to  be  able  to  u;;e  for  10  years.  The  scrap  value  at  the  end  of  that 
time  is  esiimaied  at  $25,000.  A  bond  issue  of  $75,000  due  in  10  years, 
bearing  6  per  cent  inteiest  and  secured  by  a  mortgage  on  the  machinery, 
was  floated  at  98  soon  after  the  purchase  of  the  machinery.  The  trust 
indenture  requires  that  at  the  end  of  each  year,  before  the  payment  of  divi- 
dends, a  smu  shall  be  set  aside  and  charged  against  earnings  sufficient  to 
prove  a  sinking  fund  on  a  5  per  cent  basis  for  the  redemption  of  the  bonds 
at  maturity. 

The  president  of  the  company  is  in  favor  of  providing  a  reserve  for 
depreciation  on  the  machinery  by  the  sinking-fund  method,  using  5  per 
cent  as  a  basis,  although  he  does  not  advocate  creating  a  replacement  fund 
for  the  machinery  as  well  as  a  sinking  fund  for  bonds. 

(a)  Compute  the  amount  of  the  annual  contribution  to  sinking  fund 
for  the  redemption  of  the  bonds. 

(b)  Set  up  a  table  showing  the  accumulation  of  the  fund,  on  the  as- 
sumption that  it  earned  exactly  5  per  cent. 

Also  indicate  the  annual  entries  for  the  sinking  fund  and  for  the  sink- 
ing-fund reserve.     (1.05)'"  =  1.62889463. 

(A.  I.  of  A.,  May,  1921.) 


CHAPTER  XVI 
MANUFACTURING  ACCOUNTS 

Nature  and  Function  of  Cost  Accounting — Cost  accounting  is  that 
specialized  application  of  the  principles  of  accounting  that  results  in  the 
collection  of  the  data  used  to  determine  the  cost  of  producing  a  unit  of 
product  in  a  factory/  It  is  possible  to  obtain  from  the  general  books 
certain  ideas  as  to  the  manufacturing  costs,  but  it  is  quite  impossible  to 
obtain  from  the  general  accounts  the  cost  of  a  particular  unit  of  product 
in  case  more  than  one  kind  of  article  is  turned  out. 

Cost  finding,  the  method  used  in  determining  in  advance  what  the  cost 
of  an  article  should  be  under  existing  conditions,  must  not  be  confused 
with  cost  accounting,  the  method  of  determining  costs  while  the  article  is 
being  produced." 

The  purposes  of  cost  accounting  are:  (a)  to  determine  the  cost,  or  the 
profit  on  each  unit  of  product,  (b)  to  determine  the  profitable  lines  for 
manufacture,  (c)  to  secure  accurate  and  perpetual  inventories,  and  (d) 
to  secure  information  necessary  to  lay  down  wise  managerial  policies.' 

Components  of  Cost  Data — The  ordinary  manufacturing  concern  natu- 
rally divides  itself  into  three  parts :  a  fabricating  section,  an  administrative 
section,  and  a  selling  section.  Each  of  these  sections  has  its  own  charges 
to  contribute  to  the  final  cost  of  the  product  placed  in  the  hands  of  the  pur- 
chaser. The  fabricating  section,  however,  has  three  kinds  of  charges  quite 
different  in  nature,  namely,  material,  direct  labor,  and  overhead.*  Upon  the 
addition  of  each  of  these  various  classes  of  expenses  to  the  value  charged 
against  the  product,  a  new  kind  of  cost  is  secured,  as  shown  in  the  follow- 
ing chart" : 


.Prime  Cost 

Factory  Cost 

Selling  Cost 

Total  Cost. 


The  outlays  witli  wliich  cost  accounting  is  concerned  include  the  charges 
necessary  to  put  the  product  through  the  factoi'y."  The  analysis  of  factory 
expenses  demands  an  understanding  of  the  terms  "direct"  and  "indirect" 


*For  explanation  of  superior  figures  see  page 

309 


337. 


310  C.  P.  A.  ACCOUNTING 

as  applied  to  labor.  The  cost  of  direct,  or  productive,  labor  is  the  charge 
for  services  rendered  directly  at  the  tool-point  in  the  fabricating  process.' 
The  cost  of  indirect  labor  is  the  outlay  necessary  to  the  conduct  of  plant, 
but  is  not  directly  chargeable  to  certain  specific  units  of  product,  as  the 
salaries  of  foremen,  of  timekeepers,  and  of  repairmen.* 

The  usually  accepted  cost  of  raw  materials  is  the  invoice  price  plus  in- 
freight.  This  cost,  added  to  the  cost  of  direct  labor,  makes  up  the  prime 
cost.*  Manufacturing  expense,  or  factory  overhead,  includes  such  charges 
as  taxes,  insurance,  maintenance  and  depreciation  of  factory  buildings  and 
machinery,  power,  heat,  light,  superintendence,  and  other  indirect  labor, 
and  miscellaneous  supplies.** 

Relation  of  Cost  to  General  Accounting — General  accounting  is  the  rec- 
ord of  facts  involving  the  economic  relationship  of  the  manufacturer  to  the 
world  at  large,  while  cost  accounting  is  the  record  of  facts  involving  the 
internal  relationship  of  the  various  cost  elements  and  production  depart- 
ments. Greneral  accounting  is  fiscal  in  its  nature,  while  cost  accounting  is 
statistical.  General  accounting  shows  the  net  profit  made  on  the  business 
as  a  whole,  while  cost  accounting  shows  the  net  profit  made  on  each  oper- 
ating department,  job,  contract,  or  class  of  product." 

Cost  accounting  is  not  a  system  per  se  to  be  set  aside  and  distinguished, 
as  we  distinguish  single  entry  and  double  entry  as  bookkeeping  "systems" ; 
it  is  rather  the  application  of  double  entry  principles  for  the  purpose  of 
determining  unit  cost  in  manufacturing." 

The  manufacturing  accounts  are  controlled  by  certain  accounts  in  the 
financial  books."  This  control  may  be  accomplished  by  either  of  two  gen- 
eral methods.  Under  the  first,  a  general  ledger  account,  termed  "factory 
ledger,"  is  debited  with  all  charges  against  manufacturing  operation  and 
credited  with  all  products  coming  from  the  factory."  A  similar  account  in 
the  factory  ledger  is  used,  bearing  the  name  "general  ledger,"  the  entries 
to  which  are  contra  to  those  in  the  factory  ledger  account  in  the  general 
books."  The  second  method  is  to  eaiTy  in  the  general  ledger  several  ac- 
counts by  means  of  which  the  cost  accounts  are  controlled.  Such  general 
ledger  accounts  would  be  "productive  labor,"  "raw  materials,"  "f actorj-  ex- 
pense," "goods  in  process,"  and  "cost  of  sales."" 

Wherever  practicable,  the  factories  are  divided  into  departments,  and 
departmental  accounts  are  set  up."  The  advantage  of  having  separate  de- 
partmental accounts  for  rent,  depreciation,  power,  etc.,  can  readily  be  seen 
from  the  fact  that  one  department  may  have  little  machinery  and  a  large 
amount  of  floor  space,  while  another  department  may  have  very  little  floor 
space  and  very  costly  machinery. 

Comparison  of  Cost  and  Non-Cost  Systems — In  order  to  illustrate  the 
difference  in  the  operation  of  the  accounts  under  cost  and  non-cost  systems, 
journal  entries  covering  the  \arious  classes  of  transactions  under  the  two 
systems  will  be  tabulated  and  posted  to  the  accounts  given  in  a  starting 
balance  sheet,  which  is  the  same  under  both  systems,  and  then  trial  balances 
and  financial  statements  will  be  drawn  up  for  the  accounts  vinder  each 
system. 


MANUFACTURING  ACCOUNTS 


311 


JOHN  KAY  MANUFACTURING  COMPANY 
INITIAL  BALANCE  SHEET 


Cash $3,000.00 

Accounts  Receivable 2,000. 00 

Raw  Materials 6,000. 00 

Finished  Goods 7,000.00 

Goods  in  Process 3 ,  000 .  00 

Plant  and  Equipment 75,000. 00 


$96,000.00 


Liabilities  and  Capital 

Accounts  Payable $4 , 000. 00 

Bonds  Payable 30,000. 00 

Capital  Stock 50,000. 00 

Surplus 12,000.00 


$96,000.00 


JOURNAL  ENTRIES 
Under  Non-Cost  System :  Under  Cost  System : 

When  purchases  of  raw  materials  are  made: 


Raw  Materials.  $50,000.00 
Accounts 
Payable. . . 


$50,000.00 


Raw  Materials.  $50,000.00 
Accounts 

Payable. . .  $50,000.00 


(No  entry) 


Productive 

Labor .... 

Cash 


When  raw  materials  are  requisitioned: 
Goods   in  Pro- 
cess     $45,000.00 

Raw     Mate- 
rials   $45,000.00 

When  productive  labor  is  paid: 
Goods  in  Pro- 
cess   $30,000.00 

$30,000.00  Cash $30,000.00 


$30,000.00 


Factory   Ex- 
pense    $10,000.00 

Cash $10,000.00 


When  factory  expense  is  incurred: 
Factory   Ex- 


pense   $10,000.00 

Cash $10,000.00 


When  factory  expense  is  prorated  over  jobs: 
(Not  done)                                                    Goods  in  Pro- 
cess   $10,000.00 

Factory   Ex- 
pense   $10,000.00 

When  operating  expenses  are  incurred: 
Salesmen's 


Salesmen's 

Salaries $2,500.00 

Advertising....  1,000.00 

Office  Expense .  3,000.00 

Legal  Expense.  500.00 

Cash 


Salaries $2,500.00 

Advertising. . .  .  1,000.00 

Office  Expense .  3,000.00 

Legal  Expense.  500.00 

Cash 


Interest    Ex- 
pense      $2,000.00 


$7,000.00 

When  non-operating  expenses  are  incurred: 
Interest  Ex- 


Cash. 
(No  entry) 


pense $2,000.00 

Cash 


$2,000.00 
When  work  in  process  is  completed: 

Finished  Goods  $80,000.00 
Goods  in  Pro- 
cess   


$7,000.00 


$2,000.00 


$80,000.00 


312 


C.  P.  A.  ACCOUNTING 


JOURNAL  ENTRIES— Continued 
Under  Non-Cost  System:  Under  Cost  System: 

When  sales  are  made: 


Accounts     Re- 
ceivable. . .  $125,000.00 
Sales $125,000.00 


Accounts     Re- 
ceivable... $125,000.00 
Sales $125,000.00 

When  non-operating  profits  are  made: 


Cash $1,000.00 

Interest 
Earned...  $1,000.00 


Accounts  Pay- 
able    $45,000.00 

Discount    on 

Purchases.  $2,000.00 

Cash 43,000.00 


Cash $1,000.00 

Interest 
Earned...  $1,000.00 

When  creditors  are  paid: 

Accounts   Pay- 
able    $45,000.00 

Discount    on 

Purchases . 

Cash 


$2,000.00 
43,000.00 


When  customers  settle  their  accounts: 


Cash $105,000.00 

Accounts  Re- 
ceivable. . .  $105,000.00 


(No  entry) 


Cash $105,000.00 

Accounts  Re- 
ceivable. . .  $105,000.00 

When  cost  of  sales  for  period  is  determined: 

Cost  of  Sales. .  .   $75,000.00 
Finished 
Goods $75,000.00 


TRIAL  BALANCES 


Cash 

Accounts  Receivable. . , 

Raw  Materials 

Goods  in  Process 

Finished  Goods 

Plant  and  Equipment. . 

Productive  Labor 

Factory  Expense 

Cost  of  Sales 

Salesmen's  Salaries.  .  . . 

Advertising 

OflBce  Expense 

Legal  Expense 

Interest  Expense 

Accounts  Payable 

Bonds  Payable 

Capital  Stock 

Surplus 

Sales 

Interest  Earned 

Discount  on  Purchases . 


Non-Cost 


$17,000.00 

22,000.00 

56,000.00 

3,000.00 

7,000.00 

75,000.00 

,30,000.00 

10,000.00 

nil 
2,500.00 
1,000.00 
3,000.00 
600.00 
2,000.00 


$9,000.00 

30,000.00 

50,000.00 

12,000.00 

125,000.00 

1,000.00 

2,000.00 


$229.000. 00 $229,000. 00  $229,000.00 

J 


Cost 


$17,000.00 

22,000.00 

11,000.00 

8,000.00 

12,000.00 

75,000.00 

nil 

nil 

75,000.00 
2,500.00 
1,000.00 
3,000.00 
500.00 
2,000.00 


$9,000.00 

30,000.00 

50,000.00 

12,000.00 

125,000.00 

1,000.00 

2,000.00 


$229,000.00 


MANUFACTURING  ACCOUNTS 


313 


From  the  above  it  can  be  seen  that  the  non-cost  trial  balance  can  be 
turned  into  the  cost  system  trial  balance  by  the  following  journal  entry : 

Goods  in  Process $5,000.00 

Finished  Goods 5,000.00 

Cost  of  Sales 75,000.00 

Raw  Materials $45,000.00 

Productive  Labor 30,000.00 

Factory  Expense 10,000.00 

The  above  credit  it^ms,  totaling  $85,000,  represent  the  total  manufactur- 
ing cost  charged  to  goods  in  process.  Of  this,  $80,000  represents  the  pro- 
duction costs  of  goods  in  process  completed  during  the  period  and  trans- 
ferred to  the  finished  goods  account,  and  the  remaining  $5,000  represents 
the  amount  added  to  the  inventory  of  the  goods  in  pi'ocess.  Of  the  $80,000 
charged  to  the  finished  goods  account,  $75,000  represents  the  production 
cost  of  the  goods  sold  that  was  transferred  from  the  finished  goods  account 
to  the  cost  of  sales  account,  and  the  remainder  represents  the  amount 
added  to  the  finished  goods  inventory. 

Before  a  financial  statement  can  be  made  from  the  non-cost  trial  bal- 
ance, the  inventories  must  be  ascertained.  The  inventories  are :  raw  ma- 
terials $11,000,  goods  in  process  $8,000,  and  finished  goods  $12,000.  It 
will  be  noted  that  these  inventories  are  the  balances  of  the  respective 
accounts  in  the  cost  system  trial  balance." 

The  balance  sheet  is  the  same  for  both  systems  of  accounting,  so  the 
following  statement  will  be  the  balance  sheet  that  would  be  obtained  from 
both  trial  balances : 

JOHN  KAY  MANUFACTURING  COMPANY 
FINAL  BALANCE  SHEET 


Assets 

Cash $17,000.00 

Accounts  Receivable 22 ,  000 .  00 

Raw  Materials 11,000.00 

Goods  in  Process 8,000.00 

Finished  Goods 12 ,  000 .  00 

Plant  and  Equipment ....  75 ,  000 .  00 

$145,000.00 


Liabilities  and  Capital 

Accounts  Payable $9 ,  000 .  00 

Bonds  Payable 30,000. 00 

Capital  Stock 50,000.00 

Surplus 56,000.00 


$145,000.00 


As  cost  accounting  is  merely  an  addition  to  financial  accounting,  which 
by  internal  adjustments  shows  the  cost  of  the  goods  manufactured,  the 
profit  and  loss  statements  on  page  314  show  that  the  statements  obtained 
from  the  cost  and  non-cost  trial  balances  are  identical,  after  the  gross 
profit  on  sales  has  been  obtained. 

Kinds  of  Cost  Systems — The  particular  cost  system  established,  the 
method  of  control,  and  the  accounts  used  must  depend  upon  the  type  of 
business.  The  collection  of  costs  is  dependent  upon  the  manner  in  which 
articles  proceed  through  the  processes  of  manufacture.     Where  a  prac- 


314 


C.  P.  A.  ACCOUNTING 


tically  steady  flow  of  material  is  put  into  operation  and  poes  uninter- 
ruptedly through  the  plant  to  turn  out  a  uniform  product,  one  lot  of 
material  following  another  without  relation  to  a  particular  order  or  article, 
the  "product  or  process  system"  of  calculating  costs  is  used;"  and  where 
unlike  orders  are  put  through  the  plant,  each  having  its  own  special  list  of 
material  and  method  of  processing,  costs  are  collected  upon  each  order  or 
"job,"  under  the  "special  order  system."^" 

JOHN  KAY  MANUFACTURING  COMPANY 
PROFIT  AND  LOSS  STATEMENT  (DATE) 


Under  Non-Cost  System 

Under  Cost  System 

Sales 

$125  ,000 

$125  ,000 

Cost  of  Sales: 
Raw  Materials: 

Initial    Inventory    and 
Purchases 

$56,000 
11,000 

Final  Inventory 

$45,000 

30,000 
10,000 

Productive  Labor 

Factory  Expense 

$8  ,000 
3,000 

Goods  in  Process : 

Final  Inventory 

$85,000 

Initial  Inventory 

5,000 

Finished  Goods: 
Final  Inventory 

$12 ,000 
7,000 

$80,000 

Initial  Inventory 

5,000 

75,000 

75  ,000 

Gross  Profit  on  Sales 

$50,000 

$50  ,000 

Operating  Expenses : 
Selhng  Expenses : 
Salesmen's  Salaries 

$2,500 
1,000 

$2,500 
1,000 

Advertising 

$3,500 

$3,500 

General  Expenses: 

Office  Expense 

$3,000 
500 

$3,000 
500 

Legal  Expense 

3,500 

7,000 

3,500 

7,000 

Net  Profit  on  Sales 

$43,000 

$43  ,000 

Non-operating  Items: 
Income : 
Discount  on  Purchases... 

$2,000 
1,000 

$2  ,000 
1,000 

Interest  Earned 

$3,000 
2,000 

1,000 

$3,000 
2,000 

Expense: 

Interest  Expense 

1  000 

Net  Profit  for  Period 

$44  ,000 
44,000 

$44 ,000 
44,000 

Appropriation  of  Profits : 
Surplus 

MANUFACTURING  ACCOUNTS  315 

There  are  also  two  general  methods  of  applying  costs  over  the  product, 
known  as  the  "productive  labor  method"  and  the  "process  method."** 
Under  the  first  name,  the  costs  of  labor  (figured  by  hours  or  by  wages) 
and  of  material  are  charged  against  the  product  or  order  direct,  and  to 
this  total  is  added  a  pro  rata  share  of  the  indirect  expenses,  determined 
by  the  amount,  or  the  time  of  the  productive  labor  used  to  produce  that 
product  or  order,  the  grand  total  being  manufacturing  eost.*^  Under  the 
'•'process  method,"  all  charges  are  made  against  the  various  processes  or 
operations,  and  the  total  cost  so  ascertained  is  then  distributed  over  the 
product  on  a  convenient  basis — weight,  number,  or  measure,  according  to 
the  nature  of  the  product/' 

A  specialized  form  of  the  "process  method"  of  distributing  expenses, 
applicable  where  the  greater  part  of  the  processing  is  done  by  machines, 
is  called  the  "machine  cost  method."^*  All  indirect  expenses  having  been 
calculated  for  a  given  period,  each  element  of  the  total  is  charged  against 
various  machines  or  groups  of  machines,  according  to  the  floor  space  or 
some  other  equitable  basis,  so  that  the  grand  total  of  manufacturing  ex- 
pense for  the  period  is  charged  to  all  the  machines  in  the  plant.^'  Usually 
the  unit  of  measurement  taken  is  the  "machine  hour";  that  is,  the  total 
charge  against  a  machine  is  divided  by  the  number  of  hours  of  running 
time  of  the  factory  over  the  period  considered.^'  Product  is  then  charged 
and  the  machine  credited  with  the  machine's  hourly  burden  rate  multi- 
plied by  the  number  of  hours  the  machine  works.  If  the  machine  fails 
to  run  full  time,  it  will  have  a  debit  standing  against  it  at  the  end  of  the 
period."  This  debit  is  due  principally  to  idle  time,  and  is  ordinarily 
absorbed  by  a  "supplementary  rate"  added  to  the  product,  sufficient  in 
size  to  cover  the  deficiency.^ 

Combinations  of  the  two  systems  of  collecting  costs  and  the  two  methods 
of  distributing  them  produce  the  four  general  types  of  cost  accounting 
systems:  (a)  special  order  system,  distributing  indirect  expense  by  the 
productive  labor  method;  (b)  special  order  system,  distributing  by  the 
process  or  machine  cost  method;  (c)  product  system,  distributing  by  the 
productive  labor  method;  (d)  product  system,  distributing  by  the  process 
or  machine  cost  method." 

One  or  the  other  of  these  types  can  be  used  in  any  factory.  The  choice 
between  the  special  order  and  the  product  systems  depends  on  the  nature 
of  the  product,  uniformity  being  required  for  the  successful  operation  of 
the  product  system."  The  method  of  distributing  factory  burden  should 
be  chosen  according  to  the  importance  of  labor  and  machinery  in  produc- 
tion. Factories,  where  labor  is  the  predominating  element,  should  use  the 
direct  labor  method  as  the  basis  of  prorating  burden,  while  those  where 
expensive  and  complicated  machinery  predominate  should  use  the  process 
or  machinery  cost  method. 

Control  of  Material — When  material  is  received,  checked,  and  delivered 
into  storage,  recoi'd  of  the  transaction  is  made  both  in  the  financial  books 
and  the  cost  records."'  In  the  financial  books,  the  entry  is  usually  made  in 
the  voucher  register,  debiting  raw  materials  and  crediting  vouchers  pay- 


316  C.  P.  A.  ACCOUNTING 

able."  In  the  cost  records,  the  individual  items  are  entered  in  a  stores 
ledger,  wherein  an  account  is  kept  with  each  item  of  stock.''  The  stores 
ledger  thus  carries  the  particulars  of  the  materials  account  carried  in  the 
general  ledger.'* 

The  effect  of  the  issue  of  materials  from  the  store  room  to  be  put  into 
production  is  a  debit  to  the  goods  in  process  account  and  a  credit  to  the 
materials  account.'*  If  any  of  the  material  so  issued  is  later  returned  to 
stock,  a  reverse  entry  is  used  to  cover  the  transaction.'* 

The  charge  for  materials,  which  is  to  be  entered  against  a  specified  unit 
of  product,  is  obtained  by  the  use  of  a  lot  number  or  job  number.  The 
requisition  ticket,  bill  of  material,  or  other  paper,  which  authorizes  the 
removal  of  material  from  store  rooms,  contains  a  space  in  which  is  placed 
the  number  against  which  they  are  to  be  charged."  The  various  amounts 
chargeable  against  a  certain  job  are  later  assembled  on  a  cost  sheet.'*  The 
individual  records  of  raw  materials  show  the  balances  which  should  be  on 
hand."  The  balance  of  tlie  raw  materials  account  shows  the  money  value  of 
these  materials,"  which  the  stores  records  usually  carry  both  by  number  and 
value.*^  The  stores  records  should  be  compared  with  physical  counts  made 
independently  of  the  materials  records.*^  In  this  way  there  are  three 
sources  from  which  a  careful  scrutiny  of  materials  may  be  made,  viz.,  the 
control  account  in  the  financial  books,  the  stores  ledger  accounts,  and  the 
physical  inventory. 

Control  of  Labor  Costs — The  purposes  served  by  a  proper  accounting 
for  labor  are  three :  (a)  the  measuring  of  the  relative  efficiency  of  labor 
as  to  production  value;  (b)  the  discovery  of  means  of  increasing  produc- 
tion and  lowering  costs;  and  (c)  the  determination  of  a  basis  for  distribut- 
ing overhead  expenses. 

The  charges  for  labor  are  secured  originally  from  a  time-card  or  time- 
ticket  which  shows  the  operator,  the  operation,  the  time  worked,  and  the 
job  or  lot  to  which  the  cost  is  chargeable,  also  whether  the  labor  is  to  be 
classed  as  direct  or  indirect.*^ 

Wage  Systems — The  more  commonly  employed  plans  of  paying  wages 
are  as  follows: 

(a)  Day  Rate. — A  flat  rate  per  day  regardless  of  output.** 

(b)  Piece  Work. — A  flat  rate  per  unit  of  output.** 

(c)  Differential  Rate. — A  rate  per  unit  of  output,  which  increases  when 
output  is  increased.** 

(e)  Premium  Plan. — A  flat  rate  per  hour  plus  extra  pay  for  time  saved; 
the  time  saved  equals  difference  between  standardized  and  actual  time.*' 

(f )  Bonus  Plan. — A  rate  per  hour  which  increases  when  output  is  in- 
creased.** 

(g)  Stint  Plan. — A  flat  day  rate,  with  privilege  of  going  home  when  the 
assigned  day's  work  is  completed.** 

The  above  mentioned  plans  contain  the  main  points  of  divergence,  but 
there  is  a  large  number  of  combinations  of  systems.  For  instance,  Santa 
Fe  efficiency  plan*"  has  a  standardized  output  which  is  regarded  as  100 


MANUFACTURING  ACCOUNTS 


317 


per  cent  efficiency,  all  other  outputs  being  sealed  as  percentages  of 
efficiency.  The  workmen  are  paid  by  the  hour,  the  hourly  rate  consisting 
of  a  guaranteed  flat  amount  plus  an  additional  amount  which  varies 
according  to  the  efficiency  of  the  workmen.  Thus  the  Santa  Fe  plan  com- 
bines the  day  rate  plan  and  a  rather  complicated  bonus  plan. 

Control  of  Manufacturing  Expense — As  it  is  not  practicable  to  ascertain 
the  exact  amount  of  the  manufacturing  expenses  that  have  occurred  during 
the  time  a  given  job  or  order  is  in  process,  some  sort  of  estimate  is  made 
to  cover  the  job.  Distribution  of  expenses  over  product  may  be  made 
upon  any  number  of  the  following  bases:  (a)  cost  of  productive  labor," 
(b)  cost  of  direct  materials,"'  (c)  prime  cost,"  (d)  hours  of  labor  spent,** 
(e)  units  of  product,"  and  (f )  machine  hours."'  The  use  of  each  of  these 
methods  is  indicated  in  the  following  table : 


Last 

Year 

Factory 

This 
Year 
Job 

No.  6 

Cost  of  Job  under  Various  Methods 

OF  PROBATINa  BuHDEN 

• 

Labor 
Cost 

Material 
Cost 

Prime 

Cost 

Labor 
Hours 

Units  of 
Product 

Machine 
Hours 

Material  Cost. .  . . 

Labor  Cost 

Burden  Cost 

Labor  Hours 

$10,000 

$16,000 

$14,000 

56,000 

42,000 

10,000 

$1.00 
|$1 . 60 

1    (?) 
6 
4 

1 

$1.00 

1.60 

il . 40a 

$1.00 
il.60 
1.406 

$1.00 
1.60 
1.40c 

$1.00 
1.60 
1.50d 

$1 .  00 
1.60 
1.40e 

$1.00 
1.60 
1.33/ 

Machine  Hours .  . 

Pieces  Made 

$4.00 

$4.00 

$4.00 

$4.10 

$4.00 

$3.93 

a.  (14,000  ^  16,000)  X  $1.60 
h.  (14,000  ^  10,000)  X  $1.00 
c.   (14,000  ^  26,000)  X  $2.60 


d.  ($14,000-^56,000)  X6 

e.  ($14,000^10,000)  XI 
/.   ($14,000  -^  42,000)  X  4 


By-Products — A  by-product  is  a  secondary  commodity  of  value  inci- 
dental to  the  manufacture  of  the  primary  product  of  a  factory."  Where 
no  direct  la])or  is  applied,  after  the  separation  from  the  main  product,  the 
by-product  is  classed  as  waste,  or  scrap,  or  offal,  and  is  disposed  of  at  the 
best  going  price  obtainable,  the  amount  realized  therefor  being  credited 
to  the  material  account  involved,  if  both  the  primary  and  secondary  ma- 
terial are  clearly  defined.  When,  however,  the  practice  is  to  accumulate 
varying  percentages  of  waste  of  a  given  kind  and  grade,  such  as  borings, 
turnings,  etc.,  and  no  economic  purpose  would  be  served  by  the  application 
to  material  costs  of  such  .salvage  prices  obtained,  then  the  credit  is  passed 
to  the  overhead  accounts  of  the  departments  contributing  such  secondary 
commodities.  In  the  latter  case,  statistics  are  compiled  and  the  volume 
of  waste  for  given  periods  is  compared  with  the  volume  of  raw  material 
put  into  process. 


318  C.  P.  A.  ACCOUNTING 

Where  direct  labor  is  applied,  the  secondary  commodity  is  called  a  by- 
product. No  fixed  rule  for  all  classes  of  production  can  be  formulated  as 
to  the  basis  for  levying  charges  and  passing  credits  for  by-products,  but 
each  case  must  be  governed  by  physical  conditions.  Where  such  by- 
product is  of  comparatively  small  consequence,  the  cost  involved  can  be 
based  on  percentages  secured  by  careful  tests.  On  the  other  hand,  where 
the  by-product  is  an  important  feature  and  where  the  raw  material  in- 
volved is  subject  to  speculative  market  conditions,  the  by-product  portions 
thereof  will  in  like  manner  usually  reflect  market  conditions.  In  such 
cases,  it  is  customary  to  use  current  quotations  for  the  scrap,  as  such,  in 
crediting  such  scrap  or  by-product  material,  to  the  primary  raw  material 
account.  This  method  is  fair  to  both  the  primary  and  secondary  product 
departments,  where  each  has  its  own  problems  to  contend  with  and  its  own 
profits  to  earn. 

The  cost  of  direct  labor,  materials,  and  manufacturing  overhead  used  in 
producing  by-products,  is  debited  to  the  by-products  account,  and  the  cost 
of  the  by-products  sold  is  credited  to  the  by-products  account"*  and  debited 
to  the  sales  of  by-products  account."  The  income  from  the  sales  of  by- 
products is  credited  to  the  sales  of  by-products  account"  and  the  balance 
of  this  account  is  show^n  in  the  profit  and  loss  statement  either  as  non- 
operating  income'*  or  as  an  item  to  be  shown  separately  and  then  added 
to  the  ordinary  sales.  The  latter  method  seems  preferable  as  the  sales  of 
by-products  appear  to  be  operating  items. 

Inventories — The  rules  given  in  Chapter  XIII  for  the  valuation  and 
verification  of  merchandise  inventories  should  be  used  in  the  valuation  and 
verification  of  the  inventories  of  the  raw  materials,  goods  in  process,  and 
finished  goods.  The  inventories  should  be  valued  at  cost  or  market,  which- 
ever is  the  lower,  cost  meaning  for  the  raw  materials  the  invoice  price  plus 
freight-in,  and  other  expenses  incurred  in  placing  the  goods  in  the  store 
room,"^  and  for  goods  in  process  and  finished  goods,  the  cost  of  the  ma- 
terials and  labor  expended  thereon  plus  a  proper  share  of  the  manufactur- 
ing overhead."  The  dispute  as  to  Avhether  interest  is  an  element  of  manu- 
facturing overhead  does  not  extend  to  the  inclusion  of  interest  in  the  price 
of  inventories  of  goods  in  process  and  finished  goods,  as  the  advocates  of 
including  interest  as  a  cost  reduce  the  inventory  valuations  to  prices  ex- 
clusive of  the  interest  element. 

In  cases  when  a  cost  system  is  maintained,  the  auditor  should  leave  the 
inventory  prices  at  the  cost  figures,  as  any  excess  of  the  cost  over  the 
market  values  should  be  shown  in  a  reserve  for  inventory  fluctuations, 
which  reserve  should  be  deducted  on  the  balance  sheet  from  the  inventories 
valued  at  cost.** 

Perpetual  inventories  should  be  tested,  and,  if  any  material  discrepancies 
are  found,  they  should  be  investigated."'  The  adequacy  of  the  cost  of  the 
system  should  be  examined,  and,  if  the  system  is  faulty,  the  methods  of 
calculating  the  prices  quoted  on  the  inventories  should  be  carefully 
scrutinized." 


MANUFACTURING  ACCOUNTS  319 


QUESTIONS 


manufacturixg  accounts 

Genkral 

1.  What  is  understood  by  "cost"  or  "factory"  accounting'?  (N.  Y., 
Dec,  1S98;  Wash.,  Sept.,  1907;  Va.,  Nov.,  1910.) 

2.  State  your  opinif)n  as  to  the  relative  advantages  and  disadvantages 
of  a  modern  system  of  cost  accounting  for  a  manufacturing  business. 
(Mass.,  June,  1913.) 

3.  How  many  functions  are  contained  in  the  business  of  a  manufac- 
turer?    (Minn.,  Oct.,  1916.) 

4.  What  are  the  main  purposes  in  keeping  cost  accounts?  (Mass.,  Oct., 
1915*;  Cal.,  Nov.,  1916;  Cal.,  June,  1917.) 

5.  Name  the  principal  elements  of  manufacturing  cost.  (N.  Y.,  Dec, 
1898*;  Pa.,  Nov.,  1899*;  N.  Y.,  June,  1901*;  Pa.,  Nov.,  1903*;  Pa.,  Nov., 
1904*;  Mich.,  July,  1906*;  Wash..  Sej-t.,  1907*;  Ohio,  March,  1910*;  Va., 
Oct.,  1911*;  Cal.,"  Nov.,  1916*;  Cal,  June,  1917*;  Ohio,  Oct.,  1919.) 

6.  In  determining  the  cost, of  manufactured  goods,  what  are  the  ele- 
ments of  indirect  ex]ienditures  which  should  be  included?  Give  your 
reasons.      (N.  Y.,   Jan.,  1920.) 

7.  What  information  would  be  required  by  a  manufacturer  in  order 
to  determine  the  price  at  which  he  can  afford  to  sell  an  article?  (Mass., 
Oct.,  1915*;  Minn.,  Oct.,  1916;  Cal.,  June,  1917.*) 

8.  State  wherein  manufacturing  or  factory  costs  differ  from  commer- 
cial or  selling  costs.  (Pa.,  May,  1900*;  N.  Y.,  June,  1908*;  Va.,  Nov., 
1910.) 

9.  Define  the  following  terms  as  used  in  factory  accounting:  Store- 
keeper, general  charges,  writing  off.     (N.  Y.,  Jan.,  1901.)* 

10.  Define:  Storeroom.     (Mich.,  June,  1908.) 

11.  Define:  Stores.     (N.  Y.,  Jan.,  1901;  Mich.,  June,  1908.) 

12.  Define :  Cost  of  production.  (N.  Y.,  Jan.,  1901 ;  Mich.,  June,  190S ; 
Fla.,  July,  1909.) 

13.  Define:  Shop  cost.     (Mich.,  June,  1908;  Fla.,  July,  1909.) 

14.  Define:  By-product.     (N.  Y.,  Jan.,  1911.) 

15.  What  constitutes  selling  cost?  (Pa.,  Nov.,  1904;  Mich.,  July, 
1906;  Va.,  Oct.,  1911.) 


320  C.  P.  A.  ACCOUNTING 

16.  Define:  Prime  east.  (N.  Y.,  Jan.,  1901;  Mich.,  June,  1908;  Fla., 
July,  1909;  Cal.,  Nov.,  191G.) 

17.  Define:  Warehouse.     (Mich.,  June,  1908.) 

18.  Name  and  define  five  accounts  peculiar  to  a  manufacturing  cor- 
poration.    (Cal.,  Nov.,  1916.) 

19.  What  books  and  records  are  essential  to  the  use  of  the  double-entry 
system  in  a  manufacturing  business?  Give  list  and  description.  (Mich., 
July,  1909.) 

20.  What  accounts  are  generally  opened  to  ascertain  the  cost  of  the 
details  included  in  operating  expenses  of  manufacturing?  (Pa.,  May, 
1902;  Pa.,  Nov.,  1904.*) 

21.  Define  cost  accounts  of  a  manufacturing  business  and  state  what 
information  they  furnish.     (N.  Y.,  June,  1908.) 

22.  Define:  Departmental  accounts.     (Wash.,  June,  1915.) 

23.  In  what  manner,  if  any,  will  cost  accounts  differ  from  results  ob- 
tained in  general  books?     Explain  fully.     (Pa.,  Nov.,  1904.) 

24.  State  generally  how  the  books  of  a  firm  doing  a  manufacturing 
business  would  differ  from  those  kept  by  a  trading  concern  as  to  (a) 
books  of  record;  (b)  Ledger  accounts.    "(N.  Y.,  Jan.,  1902.) 

25.  Name  any  three  important  accounts  carried  in  the  financial  books 
of  a  manufacturing  company,  which  are  directly  connected  with  the  con- 
trol of  a  complete  cost  system  and  briefly  explain  each  account.  (Mass., 
Oct.,  1914.) 

26.  Select  some  manufacturing  business  with  which  you  are  familiar 
and  illustrate  the  accounts  you  would  use  in  a  cost  system,  and  how  you 
would  prove  their  correctness  at  the  end  of  the  fiscal  period.  (Mass., 
June,  1912.) 

27.  How  would  you  classify  the  accounts  in  preparing  a  statement  of 
a  manufacturing  business?     (N.  Y.,  June,  1901;  Mich.,  July,  1906.) 

28.  A  company  engaged  in  the  manufacture  and  sale  of  products,  de- 
sires a  separation  of  their  expenses  under  proper  divisional  or  department 
heads.  Illustrating,  make  you  own  selection  of  some  manufacturing  busi- 
ness, and  prepare  classification  of  accounts.  What  ledger  headings  would 
you  use?     (111.,  Nov.,  1904.) 

29.  What  is  shown  in  the  Cost  Books?     (N.  Y.,  Dec,  1898.) 

30.  Mention  four  items  of  information  in  addition  to  those  usually 
shown  in  the  books  of  a  mercantile  business,  which  should  appear  in  a 
set  of  books  for  keeping  the  accounts  of  a  factory.  Give  reasons  for  your 
answer.     (N.  Y.,  Jan.,  1901.) 

31.  What  is  a  Cost  Book?     (Pa.,  Nov.,  1899.) 

32.  Show  the  method  and  the  advantages  in  cost  accounting  of  the 
process  of  articulating  the  General  Ledger,  Factory  Ledger,  and  Stores 
Ledger  by  summary  accounts.  (N.  Y.,  Jan.,  1911;  Wis.,  April,  1914*  j 
Mo.,  Dec.,  1914*;  Mass.,  Oct.,  1917.*) 


MANUFACTURING  ACCOUNTS  321 

33.  What  value  has  a  Cost  Book  in  connection  with  the  general  books? 
(Pa.,  Nov.,  1903;  Wash.,  May,  1911.) 

34.  (a)  What  are  the  resi^ective  functions  of  the  Cost  Ledger  and  the 
General  Ledger?  (b)  How  should  the  said  ledgers  be  harmonized  or 
reconciled  with  each  other?     (Wash.,  March,  1909.) 

35.  Define:  Cost  Sheet.     (N.  Y.,  Jan.,  1919.) 

36.  State  the  aim  and  object  of  a  Cost  Sheet.  Give  an  example  from 
a  business  of  your  own  choosing.    (111.,  Nov.,  1908.) 

37.  State  what  is  meant  by  a  Cost  Sheet,  showing  its  advantages  and 
how  it  is  made  up.  Give  a  form  of  Cost  Sheet  for  some  manufacturing 
business  with  which  you  are  familiar.  (N.  Y.,  Dec,  1897;  Pa.,  May, 
1900*;  Wash.,  May,  1903;  W.  Va.,  May,  1917.) 

38.  Design  a  combined  Production  Order  and  Summary  of  Cost  form 
for  any  business  with  which  you  are  familiar.     (Md.,  Dec,  1917.) 

39.  (a)  Differentiate  between  waste  and  shrinkage  as  found  in  manu- 
facturing plants.  In  cost  accounting:  (h)  state  three  methods  of  treating 
waste;  (c)  state  two  methods  of  treating  shrinkage;  (d)  state  three  meth- 
ods of  treating  defective  goods.     (Wis.,  May,  1916.) 

40.  Discuss  the  treatment  of  idle  time  in  Cost  accounts.  (Wis.,  May, 
1916*;  A.  L  of  A.,  June,  1917;  Wis.,  May,  1919.) 

41.  What  do  you  understand  is  meant  by  the  "machine  rate"  system 
of  cost  accounting?  How  is  it  operated,  and  in  what  particulars  does  it 
differ  from  any  other  system  of  accounting?     (Wash.,  May,  1911.) 

42.  Name  some  of  the  general  methods  of  applying  costs  in  a  factory. 
Describe  briefly.     (Ind.,  Nov.,  1917.) 

43.  Are  working  or  job  orders  covering  a  company's  expenditures  on 
its  own  account  of  any  value  to  the  auditor  when  examining  the  general 
books?     (111.,  May,  1904.) 

44.  Discuss  the  peculiar  problems  encountered  in  factory  cost  account- 
ing, briefly  outlining  the  methods  of  treating  with  same.  (Kan.,  Dec, 
1915;  Mo.,  Dec,  1915.) 

45.  (a)  Explain  the  difference  between  cost  accounting  and  cost  finding. 
(5)  Which  is  preferable,  and  why?     (Wis.,  April,  1914.) 

46.  The  United  States  Government,  through  the  Federal  Trade  Com- 
mission, is  investigating  many  lines  of  business  and  is  "fixing  costs." 
Select  some  business  with  which  you  are  familiar  and  state  what  infor- 
mation you  would  require  to  assist  in  fixing  costs  in  fairness  to  all  con- 
cerned.    (Mass.,  Oct.,  1917.) 

47.  (a)  Give  a  general  definition  of  costs,  (b)  What  are  the  four 
requirements  of  cost  finding?  (c)  What  is  understood  by  "cost  plus" 
contract?  (d)  What  is  the  Government  ruling  as  to  "defective  work"  as 
applied  to  cost  plus  contract?  (e)  How  may  information  be  compiled  to 
support  direct  labor  costs?     (Iowa,  Dec,  1918.) 

48.  Describe  a  cost  system  with  which  you  are  familiar,  covering  de- 


322  C.  P.  A.  ACCOUNTING 

tails,  and  give  your  views  as  to  its  correctness,  or  othenvise,  and  its  bearing 
upon  the  general  books.     (Pa.,  May,  190G*;  Md.,  Jan.,  1909.) 

49.  Mention  the  principles  that  are  involved  in  cost  accounting  and 
state  what  should  be  accomplished  by  properly  applj'ing  and  executing 
them.     (N.  Y.,  June,  1909;  Wash.,  May,  1910**;  Mich.^,  Dec,  1914.*) 

50.  Outline  a  system  of  cost  accounts  with  which  you  are  acquainted. 
Classify  the  accounts  in  their  sequential  order.  (Wash.,  April,  1906; 
Fla.,  July,  1909.*) 

51.  In  designing  a  cost  system  for  a  manufacturing  business,  what 
fundamental  principles  are  necessary  to  maintain?     (N.  C,  June,  1916.) 

52.  What  do  you  consider  the  important  features  of  a  modern  adequate 
system  of  accounting  for  a  manufacturing  concern?     (III.,  May,  1908.) 

^3.  Would  you  recommend  that  monthly  cost  department  statements 
of  a  manufacturing  concern  be  incorporated  in  the  financial  books  or 
otherwise?     Give  reasons  for  your  answer,     (Mass.,  June,  1913.) 

54.  Prepare  a  chart  of  accounts  of  a  manufacturing  business  making 
and  selling  three  distinct  classes  of  goods.  Using  this  chart,  explain  how 
profits  are  traced  from  group  to  group  until  they  reach  the  Surplus 
account.     (Mich.,  June,  1912.) 

55.  Lay  up  a  complete  administrative  chart  for  a  large  manufacturing 
concern  to  show  the  different  departments  and  departmental  heads,  from 
the  board  of  directors  down,  so  as  to  place  responsibility,  indicate  con- 
trol, and  to  show  to  whom  reports  should  be  made.     (Mich.,  June,  1912.) 

56.  State  as  concisely  as  possible  a  proper  system  of  factory  cost 
accounts.     (N.  Y.,  June,  1906.) 

57.  Describe  briefly  at  least  four  different  types  of  cost  accounting 
systems  and  give  a  sufficient  illustration  of  each  clearly  to  identify  the 
types.     (Wis.,  May,  1919.) 

58.  You  are  engaged  to  install  a  complete  factory  cost  and  accounting 
system  in  a  large  manufacturing  plant.  Describe  the  various  steps  in  the 
handling  of  such  a  proposition  and  show  by  charts  the  accounts  (prop- 
erly gi-ouped,  etc.)  of  the  departments  (productive  and  nonproductive), 
logically  arranged,  and  give  a  list  of  the  various  forms,  etc.,  that  would 
be  required  to  record  the  factory  operations  intelligently  to  handle  them 
from  an  accounting  viewpoint  as  an  integral  part  of  the  accounting  system. 
(Mich.,  July,  1909.) 

59.  In  making  an  examination  of  the  accounts  of  a  manufacturing 
company,  you  criticize  the  method  of  handling  invoices  for  incoming  goods 
and  payments  therefor  and  are  then  requested  by  the  management  to 
outline  a  plan  which  will  overcome  your  objections  and  provide  the  com- 
pany with  a  proper  internal  check.  Submit  to  them  your  recommenda- 
tions in  the  matter,  dealing  successively  with  all  steps  from  the  time  the 
need  for  the  goods  is  apparent  to  the  time  of  paying  the  invoice,  explain- 
ing fully  the  different  forms  or  advices  that  should  be  prepared,  their 
origin  and  disposition,  how  the  accounting  department  would  be  satisfied 


MANUFACTURING  ACCOUNTS 


323 


as  to  the  correctness  of  the  invoice,  and  how  the  officers  would  be  satisfied 
as  to  the  propriety  of  the  payiuent  before  signing  the  check.  (111.,  May, 
1915.) 

60.  Making  your  own  selection  ot  a  manufacturing  industry  conducted 
on  a  large  scale,  outline  a  complete  cost  system,  showing  clearly  the  prin- 
cipal productive  departments  _and  briefly  describing  their  functions.  On 
what  basis,  if  at  all,  would  you  distribute  to  each  of  the  departments  you 
have  named  the  following  elements  of  overhead:  (a)  Purchasing  depart- 
ment expense;  (6)  accounting  department  expense;  (c)  executive  salaries; 
{d)  heat  and  light;  (e)  power;  (/)  inward  freight  and  handling  charges; 
(g)  advertising?     (Pa.,  Nov.,  1919.) 

61.  A  manufacturing  concern  has  been  operating  for  a  period  of  nine 
months,  but  owing  to  incomplete  development  of  the  plant  the  production 
during  that  period  was  greatly  below  the  capacity  and  the  cost  of  pro- 
duction consequently  abnormal.  The  directors  are  anxious  to  obtain  a 
statement  not  only  showing  the  result  of  operations  for  the  nine  months, 
but  one  which  would  be  fairly  indicative  of  what  the  results  would  have 
been  had  conditions  been  normal.  Assuming  that  the  actual  time  lost  on 
account  of  the  frequent  stoppages  amounted  in  the  aggregate  to  four 
months,  would  the  auditor  be  justified  in  furnishing  the  latter  statement 
as  Avell  as  the  former,  and  if  so,  how  would  you  proceed  to  show  the 
desired  results  from  the  following  items? 


Manufacturing  materials.  .  $39,865.69 

Freight 5,489.22 

Productive  Wages 8,827 .84 

Non-productive  Labor ....  4,441 .73 

Salaries 6,877.29 


Taxes  and  Interest $1,398 .  59 

General  Expenses 6,537 .  14 

Sales 42.363.33 

Finished  goods  at  cost ....  7,346.45 


(Mich.,  June,  1910*;  Mich.,  Dec.,  1913.) 

62.  A  manufacturer  produces  100  different  kinds  of  goods  for  sale 
and  for  each  kind  he  has  a  formula  or  unit  cost.  Detailed  records  are 
kept  of  sales,  returns  and  allowances,  name  and  quantity  of  each  kind 
of  goods  sold.  He  wishes  to  know  at  the  end  of  the  fiscal  year  the  net 
sales  of  each  kind  and  the  profit  thereon,  (a)  How  would  you  ascertain 
the  information  desired?  (6)  How  would  you  prove  the  profits  thus 
ascertained?     (Mass.,  Oct.,  1915.) 

63.  Discuss  the  practicability,  or  otherwise,  of  installing  a  uniform 
cost  system  in  a  given  line  of  manufacturing,  as,  for  instance,  furniture 
manufacturing.  Outline  the  basic  principles  of  a  cost  system  for  furni- 
ture manufacturers  which  you  believe  could  be  used  by  all  manufacturing 
plants,  whether  large  or  small,  or  Avhether  the  furniture  manufactured 
was  diversified  or  confined  largely  to  one  line  of  furniture,  as,  for  instance, 
tables.     Submit  your  answer  in  report  form.     (Pa.,  Nov.,  1919.) 

64.  State  what  you  consider  to  be  the  essential  principles  of  cost  ac- 
counting. Give  your  opinion  as  to  the  manner  in  which  a  system  of  cost 
accounting  may  effect  great  savings  to  a  business,  either  directly  or  indi- 


324  C.  P.  A.  ACCOUNTING 

rectly.  A  financier  is  interested  in  several  businesses  of  different  natures. 
He  instructs  you  to  install  a  uniform  system  of  cost  accounting  for  all 
the  businesses.  State  what  you  would  do  first  in  such  a  case,  (111.,  May, 
1916.) 

65.  What  are  specification  costs?  What  are  their  special  advantages 
and  disadvantages?     (A.  I.  of  A.,  Nov.,  1917;  111.,  Dec.,  1918.*) 

66.  Explain  the  various  ways  of  determining  the  cost  of  manufactured 
articles.     (Mich.,  June,  1913.) 

67.  What  different  methods  have  come  under  your  observation  of  as- 
certaining the  cost  of  articles  manufactured?  Explain  each  method  fully, 
stating  which,  in  your  opinion,  is  preferable,  giving  reasons.  (Mich.,  Dec, 
1906.) 

68.  Explain  the  manner  of  arriving  at  the  cost  of  mechanism,  the 
parts  of  which  are  made  in  various  departments  and  brought  together  in 
an  assembling  room,  and  also  discuss  the  various  headings  of  costs  that 
you  would  expect  to  deal  with  and  manner  of  arriving  at  same.  (111., 
May,  1904.) 

69.  Of  what  value  in  auditing  is  a  unit  of  production?  (Mass.,  June, 
1912.) 

70.  In  auditing  the  cost  accounts  of  a  corporation,  what  ultimate  object 
has  the  auditor  in  ascertaining  the  correctness  of  the  cost  of  production? 
(N.  Y.,  Jan.,  1917.) 

71.  In  auditing  the  accounts  of  a  manufacturing  firm,  what  salient 
features  of  the  Cost  Ledger  should  receive  attention?    (N.  Y.,  June,  1913.) 

72.  In  auditing  the  accounts  of  a  manufacturing  contractor,  what 
manipulations  of  the  cost  accounts  should  the  auditor  anticipate,  to  guard 
against  inflating  profits?     (N.  Y.,  June,  1913.) 

73.  What  common  expedient  is  resorted  to  by  manufacturing  contract- 
ors to  hide  their  losses  in  the  Cost  Ledger?     (N.  Y.,  June,  1913.) 

Inventories 

74.  What  is  the  auditor's  responsibility  as  to  the  correctness  of  inven- 
tories of  raw  material,  goods  in  process  and  finished  goods  as  to  quantities, 
prices,  and  amounts?  (N.  Y.,  Jan.,  1902*;  Mich.,  June,  1912*;  Ohio, 
Nov.,  1915*;  Ohio,  Oct.,  1919.) 

75.  In  the  audit  of  a  manufacturer's  books,  what  advantage  would  there 
be  in  preparing  a  cost  statement  of  the  product?  (N.  C,  Nov.,  1918*; 
N.  C,  June,  1919;  N.  C,  Sept.,  1919.) 

76.  To  what  extent  are  you  justified  in  accepting  the  certificates  of 
officials  of  a  corporation  as  to  raw  materials  and  finished  product  on 
hand?     (Ind.,  Nov.,  1917.) 

77.  What  points  in  connection  with  the  inventories  of  a  manufacturing 
company  would  you  consider  as  essential  to  be  brought  out  in  estimating 
the  correct  profits  of  the  companv?  Answer  fully.  (Md..  Dec..  1917*: 
Md.,  Oct.,  1919.)  ^  J       K       ,  ,  , 


MANUFACTURING  ACCOUNTS  325 

78.  What  are  the  duties  of  an  auditor  as  to  examination  of  inventories 
of  finished  product,  product  in  process,  and  materials  and  supplies  which 
have  been  taken  and  appraised  by  representatives  of  the  client  in  ease  he 
is  not  permitted  to  make  tests  for  the  purpose  of  satisfying  himself  as  to 
the  integrity  of  the  quantities  shown  ?  How  should  he  cover  such  a  situa- 
tion in  his  report?     (N.  Y.,  Feb.,  1909.) 

79.  What  are  the  special  points  to  which  an  auditor's  attention  should 
be  directed  in  examining  and  verifying  an  inventory  consisting  of  raw 
stock,  supplies,  goods  in  process,  and  manufactured  goods?  (N.  Y.,  June, 
1902.) 

80.  Write  out  in  detail  the  general  instruction  for  taking  the  inventory 
of  raw  materials,  work  in  process,  and  finished  goods  of  a  small  company 
manufacturing  automobiles.  Show  the  general  divisions  that  the  inven- 
tory requires  and  provide  against  errors  in  recording  the  items.  (N.  Y., 
June,  1908;  Mich.,  June,  1910.*) 

81.  What  principle  in  the  theory  of  cost  accounting  procedure  is  in- 
volved in  determining  the  cost  of  inventories  of  materials  and  supplies 
used  or  consumed  in  manufacturing  and  maintenance  purposes,  that  may 
not  be  necessary  in  determining  the  cost  of  merchandise  purchased  for 
resale?     (N.  C,  Nov.,  1918;  N.  C,  Sept.,  1919.) 

82.  State  the  general  principles  to  be  adopted  in  valuing  the  inventory 
of  a  concern  engaged  in  selling  various  articles,  some  of  which  it  manu- 
factures and  others  of  which  it  buys  complete.     (A.  I.  of  A,,  Nov.,  1917.) 

83.  Describe  the  procedure  and  value  of  a  storeroom  audit.  (Cal., 
Nov.,  1916.) 

84.  How  is  the  position  of  auditor  affected  if  the  system  of  the  concern 
under  the  audit  is  defective  as  to  cost  methods?     (A.  I.  of  A.,  May,  1918.) 

85.  When  the  cost  of  making  a  product  is  known,  also  the  overhead 
and  other  expenses  and  the  amount  of  the  sales,  what  other  information 
would  you  require  to  determine  the  profits  and  losses  in  the  sale  of  the 
product?     (N.  C,  June,  1919.) 

86.  As  an  auditor,  in  what  respect  would  you  be  concerned  with  the 
methods  of  cost  accounting  employed  by  the  concern  under  audit  and 
what  steps  would  you  take  to  verify  the  correctness  of  the  accounts 
affected  thereby?     (Ohio,  Nov.,  1918.) 

87.  Discuss  fully  the  proper  principles  to  be  observed  in  the  valuation 
of  inventories  of  partially  used  supplies  on  a  certified  Balance  Sheet. 
(Pa.,  Nov.,  1919.) 

88.  To  what  extent  do  you  deem  it  necessary  to  verify  the  following, 
in  order  that  you  may  certify  to  the  correctness  of  a  Profit  and  Loss 
statement:  (a)  Materials  and  supplies;  (b)  goods  in  process,  and  (c) 
finished  goods?     (Mass.,  June,  1912.) 

89.  In  a  manufacturing  concern  which  you  are  requested  to  audit,  you 
find  what  appears  to  be  a  careful  book  inventory  of  raw  materials,  sup- 
plies, and  work  in  process  maintained,  but  no  physical  inventory   has 


326  C.  P.  A.  ACCOUNTING 

been  made  for  some  years.  Would  you  consider  this  a  satisfactory  state 
of  affairs?  And  if  so,  on  what  safeguards  would  you  insist  to  insure 
constant  accuracy  in  the  records?     (A.  I.  of  A.,  May,  1920.) 

90.  You  have  been  retained  by  a  manufacturing  company  as  a  con- 
sulting accountant  and  are  requested  to  advise  the  officers  what  steps 
to  take  in  order  to  determine  the  cause  of  an  apparent  deficiency  in  the 
inventoiy  of  factory  material  and  work  in  process.  How  Avould  you  pro- 
ceed to  inform  yourself  of  the  circumstances  and  what  would  you  suggest 
as  a  possible  remedy?     (A.  I.  of  A.,  May,  1920.) 

91.  You  are  auditing  the  accounts  of  the  LaPorte  Crane  Company  and 
ascertain  that  the  inventory  at  December  31,  1915,  is  carried  in  the  fol- 
lowing accounts: 

Supplies  in  Store  Room $  75,000 

Work  in  Process — Labor  and  Material  only 3(X),000 

Manufacturing  Overhead 350,000 

Selling  Overhead 175,000 

You  also  ascertain  the  following  facts : 

The  actual  inventory  taken  at  December  31,  1915,  amounted  to  $50,000, 
The  work  in  Process — Labor  and  Material  only,  at  December  31,  1914, 
was  valued  at  $200,000,  while  the  manufacturing  overhead  at  that  date  was 
$150,000  and  the  selling  overhead  $200,000.  What  steps  would  you  take 
to  determine  the  accuracy  of  the  inventories  at  the  date  of  your  examina- 
tion? (111.,  May,  1916.) 


Raw  Materials 

92.  Explain  the  meaning  and  use  of  the  Material  and  Supplies  ac- 
counts.    (La.,  May,  1913.) 

93.  Briefly  outline  the  principles  of  cost  accounting  Avith  reference  to 
accounting  for  materials  and  supplies  in  the  factory.     (Md.,  Oct.,  1919.) 

94.  A  concern  making  baskets  buys  wood  lots,  stumpage,  logs  and  lum- 
ber.    How  would  you  keep  the  accounts?     (Mass.,  June,  1912.) 

95.  A  company  issues  an  order  for  material.  From  that  point  on, 
recite  the  accounting  operations  relating  to  its  purchase  which  a  well- 
organized  concern  would  employ  to  safeguard  its  interests  until  the  bill  is 
paid.     (HI.,  May,  1914.) 

96.  Describe  the  method  of  maintaining  a  check  upon  the  materials 
of  a  manufacturing  plant  and  of  insuring  the  proper  charging  out  to  jobs 
of  the  materials  used  on  them.     (Wash.,  May,  1911.) 

97.  Discuss  various  methods  of  pricing  commodities  withdrawn  from 
storerooms.     (Wis.,  May,  1916.) 

98.  What,  on  the  books  of  a  company,  would  be  the  proper  method  of 
treating  cash  payments  on  expenditures  for  mateiia!  for  manufacturing 
purposes?     (Ohio,  March,  1910.) 

99.  If  you  should  contract  for  an  audit  which  provided  for  an  exami- 


MANUFACTURING  ACCOUNTS  327 

nation  of  all  vouchers,  what  would  you  accept  as  proper  vouchers  for  raw 
material  on  hand?     (Va.,  Nov.,  1918.) 

100.  Give  a  brief  outline  of  the  procedure  you  would  adopt  to  verify 
the  inventory  of  raw  material.  (N.  Y.,  Dec,  1897*;  Ohio,  Dec,  1908*; 
Ohio,  March,  1910*;  Mass.,  June,  1910*;  Mich.,  June,  1912*;  111.,  May, 
1914*;  Kan.,  Dec,  1915;  Mo.,  Dec,  1915;  Ind.,  Nov.,  1917*;  N.  C,  June, 
1920.*) 

101.  What  test  should  be  made  of  the  prime  cost  of  manufactured 
goods  to  guard  against  loss  of  raw  material  through  theft  by  employees? 
(N.  Y.,  June,  1897;  N.  Y.,  June,  1908.) 

102.  On  what  basis  should  the  raw  material  be  valued  in  the  prepara- 
tion of  a  Balance  Sheet?  (N.  Y.,  Dec,  1896;  N.  Y.,  June,  1898*;  N.  Y., 
Jan.,  1900*;  Pa.,  May,  1900*;  N.  Y.,  Jan.,  1902*;  N.  Y.,  Jan.,  1906*; 
N.  Y.,  June,  1908* ;  Ohio,  March,  1910 ;  Wis.,  April,  1914* ;  Mass.,  Oct., 
1914*;  Wis.,  May,  1916*;  Wis.,  April,  1918*;  S.  C,  Sept.,  1919*;  N.  C, 
Nov.,  1919.*) 

103.  A  manufacturer  stated  that  he  wished  to  have  the  market  value  of 
raw  materials  purchased  considerably  below  the  market  recorded  on  his 
books  for  insurance  purposes.  What  would  have  been  your  reply ?  (Wis., 
April,  1918.) 

104.  In  closing  the  books  of  a  eomj'/any  at  the  end  of  its  first  fiscal 
year,  how  would  you  treat  pig  iron  on  hand  costing  $20  per  ton,  the 
market  value  being  $18?     (Mass.,  Oct.,  1917.) 

105.  At  the  end  of  the  fiscal  year  a  concern  inventories  its  raw  material 
at  cost.  Do  you  approve  of  this  method?  State  your  reason  fully.  (N.  Y., 
June,  1911.) 

106.  A  manufacturing  concern  buys  raw  materials  in  advance  for  its 
needs.  These  are  liable  to  fluctuations.  At  what  prices  should  they  be 
inventoried?    Why?     (R.  I.,  Dec,  1907.) 

107.  Discuss  fully  the  proper  principles  to  be  observed  in  the  valuation 
on  a  certified  Balance  Sheet  of  inventories  of  raw  materials  in  excess  of 
nonnal  requirements.     (Pa.,  Nov.,  1919.) 

108.  A  manufacturing  company  purchases  a  large  stock  of  material 
during  the  year  at  low  piiees,  but  at  time  of  annual  inventory,  values  had 
abnormallj'  increased.  How  in  your  opinion  should  inventory  and  loss 
and  gain  be  shown  on  the  books?     (Mich.,  Dec,  1916.) 

109.  In  a  Balance  Sheet  audit,  how  would  you  verify  as  to  quantities 
and  amounts  the  inventory  items  of  i-epair  and  replacement  parts  for 
the  concern's  product?     (A.  I.  of  A.,  May,  1919.) 

110.  In  auditing  the  accounts  of  a  manufacturing  company,  what 
means  should  the  auditor  adopt  to  satisfy  himself  as  to  the  inventoiy  of 
miscellaneous  supplies?     (Ohio,  Dec,  1908.) 

111.  In  what  manner  would  you  treat  "scrap"  as  to  cost  in  manufac- 
turing?   Give  reasons.     (Pa.,  Nov.,  1904.) 

112.  The  Chicago  Silversmith  Company  has  a  revenue  of  $10,000  from 


328  C.  P.  A.  ACCOUNTING 

the  sale  of  silver  and  copper  scrap.  The  sales  of  the  year  under  review 
amounted  to  $300,000  and  the  material  cost  was  $100,000.  How  should 
this  scrap  revenue  be  treated:  (1)  as  a  sale;  (2)  as  a  deduction  from 
material  cost;  (3)  as  a  deduction  from  overhead,  which  was  applied  on 
the  basis  of  labor  cost;  (4)  as  a  miscellaneous  revenue;  or  (5)  as  a  credit 
to  surplus?     (111.,  May,  1917.) 

113.  A  manufacturing  company  imports  its  raw  materials  and  pur- 
chases exchange  on  Europe  for  use  in  payment  therefor.  How  should 
the  exchange  account  be  treated  with  respect  to  the  cost  of  production? 
(N.  Y.,  Jan.,  1920.) 

114.  A  manufacturing  corporation  is  obliged  to  carry  at  least  six 
months'  supply  of  a  special  kind  of  raw  material,  and  because  such  ma- 
terial cannot  be  purchased  every  day  they  are  obliged  to  buy  when  the 
material  is  in  the  market;  consequently  the  prices  fluctuate  considerably. 
When  purchasing  such  material  the  company  pays  spot  cash  for  it.  What 
method  would  you  adopt  to  arrive  at  the  cost  of  the  raw  material  con- 
sumed monthly  in  manufacturing?     (Pa.,  Nov.,  1911*;  N.  Y.,  Jan.,  1919.) 

Goods  in  Process 

115.  Define :  Work  in  process.  (N.  Y.,  Jan.,  1911 ;  Wash.,  Nov.,  1913* ; 
Kan.,  May,  1916.*) 

116.  State  how  you  would  verify  the  Stock  in  Process.  (N.  Y.,  June, 
1906;  Wash.,  Sept.,  1907*;  Ohio,  Dec.,  1908*;  Mich.,  June,  1912*;  Kan., 
May,  1916*;  Cal.,  May,  1916*;  N.  C,  Aug.,  1917*;  111.,  May,  1918;  A.  I. 
of  A.,  May,  1919*;  N.  C,  June,  1920.*) 

117.  If  you  should  contract  for  an  audit  which  provided  for  an  ex- 
amination of  all  vouchers,  what  would  you  accept  as  proper  vouchers  for 
goods  in  process?     (Va.,  Nov.,  1918.) 

118.  How  should  the  partly  manufactured  goods  be  valued  for  use  in  a 
Balance  Sheet?  (N.  Y.,  Dec.,  1896*;  N.  Y..  June,  1898*;  N.  Y.,  Jan., 
1900*;  Pa.,  May,  1900;  N.  Y.,  Jan.,  1906*;  Wash.,  Sept.,  1907*;  N.  Y., 
Jime,  1908* ;  Ohio,  March,  1910* ;  V7is.,  April,  1914* :  Mass.,  Oct.,  1914* ; 
Del.,  June,  1915*;  Ohio,  Nov.,  1918*;  S.  C,  Sept.,  1919.*) 

119.  Discuss  fully  the  proper  principles  to  be  observed  in  the  valuation 
on  a  certified  Balance  Sheet  of  inventories  of  work  in  process  which  has 
produced  salable  by-products.     (Pa.,  Nov.,  1919.) 

120.  To  what  extent  or  i;nder  what  circumstances  could  you  certify  to 
work  in  process  in  the  absence  of  a  cost  system?     (Cal.,  June,  1917.) 

121.  In  auditing  the  books  of  a  manufacturing  concern,  you  learn  that 
the  goods  in  process  have  been  inventoried  at  material  and  labor  value 
only.  State  how  to  detennine  the  amount  of  manufacturing  expense  that 
should  have  been  added  properly  to  complete  the  inventory.  (Mich.,  June, 
1^10.) 

122.  In  auditing  the  accounts  of  a  manufacturing  company,  how 
would  you  determine  the  accuracy  of  the  overhead  included  in  the  inventory 
of  work  in  process?     (111.,  May,  1917.) 


MANUFACTURING  ACCOUNTS  329 

Finished  Goods 

123.  Define:  Finished  product.  (Wash.,  June,  1915;  Wash.,  July, 
1917.) 

124.  State  how  you  would  verify  the  finished  products.  (N.  Y.,  Dec., 
1897* ;  N.  Y.,  June,  1900* ;  Ohio,  Dec,  1908* ;  Ohio,  March,  1910* ;  Mass., 
June,  1910*;  Mich.,  June,  1912*;  111.,  May,  1914*;  Wash.,  June,  1915*; 
Kan.,  May,  1916*;  N.  C,  June,  1916;  N.  C,  Aug.,  1917*;  Ind.,  Nov., 
1917*;  N.  C,  June,  1920.*) 

125.  If  you  should  contract  for  an  audit  which  provided  for  an  exam- 
ination of  all  vouchers,  what  would  you  accept  as  proper  vouchers  for 
finished  product?     (Va.,  Nov.,  1918.) 

126.  How  should  the  completely  manufactured  goods  be  valued  for 
use  in  a  Balance  Sheet?  (N.  Y.,  Dec,  1896*;  N.  Y.,  June,  1898*;  N.  Y., 
Jan.,  1900*;  Pa.,  May,  1900;  N.  Y.,  Jan.,  1906*;  N.  Y.,  June,  1908*; 
111.,  Nov.,  1908*;  Ohio,  March,  1910*;  Wis.,  April,  1914*;  Mass.,  Oct., 
1914*;  Cal.,  May,  1916*;  Ohio,  Nov.,  1918*;  S.  C,  Sept.,  1919*;  Pa., 
Nov.,  1919.*) 

127.  How  can  an  auditor  satisfy  himself  that  the  finished  product  is 
listed  at  proper  valuations  in  a  statement  of  condition?  (Mass.,  April, 
1911.) 

128.  Are  there  any  circumstances  where  a  manufactory  might  in- 
ventory finished  goods  at  selling  price  instead  of  at  cost?  (111.,  Nov., 
1903.) 

129.  If  called  on  to  verify  the  Merchandise  account  of  a  manufacturing 
concern,  what  steps  would  you  take  to  make  tfie  necessary  investigation? 
(N.  Y.,  June,  1911.) 

130.  In  case  of  an  audit  of  the  books  of  a  manufacturer,  how  would 
you  verify  the  figures,  furnished  by  the  superintendent,  of  the  quantity 
produced?     (N.  C,  Nov.,  1918;  N.  C,  Nov.,  1919.*) 

131.  In  the  preparation  of  a  Manufacturing  and  Trading  account  and 
a  Balance  Sheet,  state  on  what  basis  the  following  asset  should  be  valued : 
Manufactured  product.     Give  fully  your  reason.     (N.  Y.,  June,  1908.) 

132.  At  what  price  would  you  carry  recently  manufactured  goods  of  a 
factory,  assuming  the  market  price  to  have  dropped  below  cost?  (Cal., 
May,  1916.) 

133.  The  physical  inventory  of  the  manufactured  stock  of  a  factory, 
taken  December  31,  1908,  is  considerably  less  than  the  balance  shown  by 
the  perpetual  inventory  carried  on  the  financial  books.  When  the  previous 
inventory  was  taken,  six  months  before,  the  difference  was  very  slight.  You 
are  asked  Avhat  steps  should  be  taken  (a)  to  ascertain  the  cause  of  the 
difference,  and  (b)  to  prevent  a  recurrence  of  the  difference.  (Wash., 
May,  1909;  Wash.,  May,  1911;  Mass.,  June,  1913.*) 


330  C.  P.  A.  ACCOUNTING 


"Wages 

134.  Briefly  outline  the  principles  of  cost  accounting  with  reference 
to  reporting  and  analysis  of  labor.     (Md.,  Oct.,  1919.) 

135.  Given  a  plant  emplojang  25  yard  laborers  at  $1.15  i^er  day;  70 
mechanics  at  $2.50  to  $3.25  jier  day;  40  helpers  in  machine  shop  at  $1.50 
per  day;  30  molders  at  $2.75  to  $4  per  day;  10  helpers  and  20  laborers 
in  foundry  at  $1.50  and  $1.25  per  day;  25  patternmakers,  $3  to  $4 
per  day;  10  helpei-s  in  pattern  shop  at  $1.50  per  day;  20  foremen  and 
under-bosses,  in  all  three  departments. 

Devise  a  timekeeping  system  for  these  various  classes  of  labor  and  give 
your  reasons  in  full  for  the  action  you  take. 

(Pa.,  May,  1906.) 

136.  Describe  in  detail  the  principles  underlying  the  following  systems 
of  wages,  and  their  effect  on  the  efficiency  of  labor  in  a  large  plant :  day 
rate,  differential  piece  rate,  premium  plan,  bonus  plan,  efficiency  system. 
(Mich.,  July,  1909*;  Mich.,  June,  1914.) 

137.  Define:  Nonproductive  labor.     (Wash.,  Nov.,  1913.) 

138.  What  classes  of  salaries  and  wages  should  be  charged  directly 
against  the  cost  of  manufacture?    Give  reasons.     (N.  Y.,  Dec.,  1898.) 

139.  Outline  in  order  the  various  steps  you  would  take  in  handling 
labor  tickets.     (Mich.,  June,  1919.) 

Overhead 

140.  What  is  usually  meant  bv  burden?  (111.,  May,  1912*;  Ohio,  Oct., 
1919.) 

141.  Define:  Indirect  charges.     (Mich.,  June,  1908;  Fla.,  July,  1909.) 

142.  Define:   Nonproductive  cost.     (N.  Y.,  Jan.,  1919.) 

143.  What  do  you  understand  by  overhead  charges?  (Fla.,  Juh', 
1909*;  111.,  May,  1912*;  Mich.,  June,' 1913;  111.,  May,  1915*;  Ohio,  Oct., 
1919.*) 

144.  What  do  you  mean  by  accrued  manufacturing  expense?  (Mich., 
June,  19]  9.) 

145.  In  cost  accounting,  what  is  meant  by  "on  cost"?  (Mich.,  June, 
1908*;  111.,  May,  1912;  Wash.,  June,  1915.) 

146.  Upon  what  basis  do  you  consider  "on  cost"  should  be  calculated? 
Give  reasons.     (Wash.,  June,  1915.) 

147.  What  items  can  properly  be  considered  as  part  of  "overhead"? 
(111.,  May,  1915*;  111.,  May,  19i7.) 

148.  Define  the  following  and  give  a  list  of  expenses  which  would 
properly  come  under  each  heading:  (a)  Shop  overhead;  (6)  general  over- 
head.    (A.  I.  of  A.,  May,  1920.) 


MANUFACTURING  ACCOUNTS  331 

149.  In  foniiulating  a  Profit  and  Loss  account  for  a  manufacturing 
concern,  in  which  factory  rent  is  an  element,  under  what  classification 
would  you  allocate  it  in  order  to  be  economically  sound?  (Wash.,  June, 
1915.) 

150.  Is  depreciation  an  element  in  the  cost  of  production,  or  should 
provision  therefor  be  made  out  of  net  profits?  Give  your  reasons.  (111., 
May,  1904*;  N.  Y.,  Feb.,  1910*  ;-Va.,  Oct.,  1912*;  Cal.,  Nov.,  1910*; 
Ohio,  Nov.,  1916*;  Cal.,  June,  1917*;  Ohio,  Nov.,  1917;  Ohio,  Oct.,  1919.*) 

151.  Is  interest  on  capital  invested  in  factory  of  machinery  generally 
regarded  as  a  proper  charge  to  cost  of  manufacturing?  Explain.  (Mich., 
June,  1913*;  Mo.,  Dec.,  1914*;  N.  Y.,  Jan.,  1917*;  111.,  May,  1917*;  Cal., 
June,  1917*;  Pa.,  Nov.,  1917*;  Ohio,  Nov.,  1918;  Iowa,  Dec,  1918; 
A.  I.  of  A.,  Nov.,  1919.*) 

152.  (a)  A  manufacturing  concern  owning  its  own  building  included 
interest  and  rent  amounting  to  $35,000  in  its  distribution  of  burden  during 
the  year,  crediting  this  amount  to  income  accounts.  Assuming  that  the 
production  of  the  factory  was  worth  $200,000  at  factory  cost,  and  the 
final  inventoiy  of  finished  goods  amounted  to  $50,000,  what  changes,  if 
any,  would  you  make  in  the  valuation  of  the  inventory? 

(b)  If  the  $35,000  had  been  actually  expended  for  interest  and  rent 
during  the  year,  what  difference,  if  any,  would  that  make  in  your  pro- 
cedure?   Give  reasons  for  the  position  you  take. 

(Wis.,  May,  1916.) 

153.  Explain  what  is  meant  by  distribution  of  overhead.  (N.  Y.,  Jan., 
1916.) 

154.  Briefly  outline  the  princij^les  of  cost  accounting,  with  reference  to 
distribution  of  "overhead"  expense.     (Md.,  Oct.,  1919.) 

155.  Overhead  factory  expense  is  computed  on  the  hour  basis  in  a 
paint  shop  with  skilled  labor.  How  would  such  application  affect  costs? 
(Wash.,  June,  1915.) 

156.  Show  how  the  appropriateness  of  each  system  of  distributing 
overhead  burdens  in  cost  accounting  may  be  affected  by  the  nature  of  the 
business  in  which  it  is  employed.     (A.  I.  of  A.,  June,  1917.) 

157.  Two  manufacturing  companies  pay  rent  for  their  factories,  though 
one  company  pays  twice  as  much  rent  as  the  other;  in  all  other  respects 
the  cost  of  the  production  is  the  same.  What  effect  should  such  rents  have 
on  the  unit  cost?     (N.  Y.,  June,  1917.) 

158.  A  manufacturing  corporation  carries  on  its  books  an  unapplied 
balance  of  overhead  expense,  which  it  adds  to  the  inventory  when  closing 
the  accounts.    Is  this  correct  in  principle?    Explain.     (N.  Y.,  Jime,  1917.) 

159.  Name  the  various  methods  of  disti'ibuting  "Factory  Expense"  or 
"Factory  Burden,"  so  as  to  apportion  same  to  the  cost  of  the  article  or 
articles  manufactured,  stating  advantaucs  of  each  in  variou.s  kinds  of 
business.  (111.,  May,  1907;  Md.,  Jan^,  1909*;  111.,  May,  1909*;  Va., 
Nov.,  1910*;  Mass.,  April,  1911*;  111.,  May,  1912;  Mich.,  June,  1912*; 


332  C.  P.  A.  ACCOUNTING 

N.  Y.,  June,  1913*;  Mich.,  Dec.,  1913*;  Wis.,  April,  1914*;  III.,  May, 
1914*;  111.,  May,  1915*;  N.  Y.,  Jan.,  1916*;  Mass.,  Oct.,  1916*;  Cal., 
Nov.,  1916* ;  Cal.,  June,  1917* ;  A.  I.  of  A.,  June,  1917* ;  N.  D,,  Aug., 
1917*;  Mass.,  Oct.,  1917*;  Md.,  Dec,  1917*;  A.  I.  of  A.,  May,  1918*;  Ga., 
May,  1919*;  Mich.,  June,  1919*;  Ohio,  Oct.,  1919*;  A.  I.  of  A.,  May, 
1920*;  N.  C,  June,  1920.*) 

160.  Name  and  explain  the  various  methods  bj'  which  cost  accounts  may 
be  handled,  bringing  out  cleariy,  among  other  items,  the  difference  between 
the  specific  order  plan  and  the  process  plan.     (Mich.,  June,  1919.) 

161.  Discuss  at  least  two  theories  of  accounting  for  unearned  burden 
in  a  cost  system.     (Wis.,  Nov.,  1919.) 

162.  Design  a  form  for  recording  the  distribution  of  the  various  items 
of  esthnated  factory  burden  over  three  operating  departments.  Indicate 
the  basis  of  distribution  used  for  each  item,  and  explain  and  illustrate 
three  methods  of  charging  this  departmental  burden  to  the  work  which 
passes  through  each  department.     (Wis.,  May,  1919.) 

163.  A  mill  usually  shuts  down  three  weeks  during  the  year,  for 
general  renovation.  State  (a)  how  you  would  provide,  in  the  accounts, 
through  the  cost  system,  for  this  contingency;  (6)  how  you  would  treat 
the  fixed  charges  during  such  period.     (Mass.,  June,  1912.) 

164.  Is  rent  of  factory  a  proper  charge  under  the  caption  of  Cost  of 
Production?     Explain.     (N.  Y.,  June,  1917.) 

165.  In  formulating  a  Profit  and  Loss  statement  for  manufacturing 
concern  in  which  factory  rent  is  an  element,  under  what  classification 
would  you  allocate  it  in  order  to  be  economically  sound?  (N.  Y.,  June, 
1913.) 

166.  Overhead  factoi-y  expense  is  computed  on  the  hour  basis  in  a 
paint  shop  with  unskilled  labor  and  in  a  machine  shop  with  skilled  labor. 
How  would  such  application  affect  cost?     (N.  Y.,  Jmie,  1913.) 


MANUFACTURING  AOCOUNTft 


333 


PROBLEMS 


MANUFACTURING    ACCOUNTS 


1.  The  Federal  Manufacturing  Company  commenced  business  on  Janu- 
ary 1,  1917,  with  a  paid  up  capital  of  $2,000,000.  It  has  a  system  of  cost 
accounts  which  are  controlled  by  the  general  books. 

The  trial  balance  of  the  company  at  December  31,  1917,  was  as  follows : 


Debits 

Credits 

Cash 

$30,000 

130,000 

25,000 

150,000 

Accounts  Receivable 

Notes  Receivable 

Raw  Material 

Overhead  Burden 

Work  in  Process 

100,000 

300,000 

70,000 

1,369,750 

Finished  Goods 

Dividends  Paid 

Plant  and  Machinery 

Profit  and  Loss 

$23,250 

Interest  on  Plant  Investment 

60,000 

Accounts  Payable    

41,000 

Notes  Payable 

500 

Reserve  for  Depreciation 

50,000 

Capital  Stock 

2,000,000 

$2,174,750 

$2,174,750 

The  general  books  of  the  company  show  charges  and  credit  to  Over- 
head account  as  shown  on  the  following  page. 

On  making  an  examination  of  the  accounts,  you  find  that  the  pur- 
chases of  raw  material  during  the  year  amounted  to  $500,000;  that 
the  cost  of  direct  labor  was  $375,000;  and  that  the  sales  amounted  to 
$723,250. 

An  analysis  of  the  orders  in  process  discloses  the  following  charges: 
Materials,  $25,000;  Direct  Labor,  $37,500;  Burden  (100  per  cent  direct 
labor),  $37,500;  and  Total,  $100,000. 

The  number  of  units  completed  and  delivered  to  the  warehouse  was 
100,000  and  of  this  number  70,000  units  were  sold. 

You  find  that  a  dividend  of  3^4  per  cent  was  declared  during  the  fiscal 
year  and  that  no  entry  was  made  on  the  books. 


334  C.  P.  A.  ACCOUNTING 

Overhead  Account 

Factory  Executive  Salaries  (one-third) $15,000 

Indirect  Labor 30,000 

Cost  Department  Salaries 10,000 

Superintendents'  Salaries 10,000 

Repairs  of  Machinery  and  Buildings 25,000 

Power 5,000 

Factory  Supplies  and  Expenses 5,000 

Interest  on  Plant  and  Equipment 00,000 

Salesmen's  Salaries 20,000 

Salesmen's  Expenses " 10,000 

Advertising 30,000 

Freight  outbound 10,000 

Shipping  Department,  Labor  and  Expenses 15,000 

Officers'  Salaries  (Executive,  two-thirds) 30,000 

Office  Salaries  (clerks) 15,000 

Office  Expenses 5,000 

Cash  Discount  on  Sales 15,000 

Interest  on  Notes  Payable 10,000 

Allowances  to  Customers 10,000 

Bad  Debts 5,000 

Depreciation  of  Plant  and  Machinery 50,000 

Total $385,000 

Credits: 

Cash  Discounts  on  Purchases 10,000 

Burden  applied  to  cost  orders  in  process  during  the 

year  (equal  to  100  per  cent  of  direct  labor) $375,000 


You  are  asked  to  prepare  a  Balance  Sheet  and  a  Profit  and  Loss  state- 
ment; also  a  statement  showing  the  cost  and  net  profit  per  unit. 

Submit  your  working  sheet.  (Ohio,  Nov.,  1918.) 

2.  The  Ohio  Manufacturing  Company  commenced  business  on  January 
1,  1918,  with  a  paid-in-cash  capital  of  $100,000. 

The  transactions  for  the  year  1918  were  as  follows :  purchases  on  credit, 
land,  $5,000;  buildings,  $20,000;  machinery  and  equipment,  $30,000;  raw 
material,  $100,000;  factory  supplies  and  expenses,  $10,200;  and  office 
expenses,  $3,000. 

The  cash  payments  for  the  year  included,  factory  productive  labor, 
$40,000;  factory  non-productive  "labor,  $20,000;  officers'  salaries,  $10,000; 
other  office  salaries,  $8,000 ;  and  salaries  and  expenses  of  salesmen,  $10,000. 

Inventories  at  December  31,  1918,  were:  raw  material,  $20,000;  factory 
supplies,  $1,000;  and  work  in  process  amounting  to  $30,000,  two-thirds  of 
which  amount  was  for  materials  and  one-third  for  productive  labor. 

The  open  accounts  receivable  amounted  to  $20,000,  after  charging  off 
$1,000  for  bad  debts;  and  the  accounts  payable  amounted  to  $18,200. 

The  units  completed  during  the  year  amounted  to  10,000,  of  which 
8,000  were  sold  at  $20  each.  Provide  10  per  cent  for  depreciation  on 
maehinei-y  and  3  per  cent  on  buildings. 

You  are  required  to  prepare  a  Balance  Sheet  and  a  Profit  and  Loss 
statement  as  of  December  31,  1918. 
(Ohio,  Oct.,  1919.)     (Note:     Prorate  burden  according  to  direct  labor.) 


MANUFACTURING  ACCOUNTS  335 

3.     (a)     From  the  following  data,  explain  and  illustrate  four  methods 
of  distributing  the  indirect  expenses  of  a  factory  to  production : 


Materials  Used 

Productive  Wages 

Productive  Labor  Hours. 
Indirect  Expenses 


Department  A 


$10,000 

$3,200 

8,000 

$4,000 


Department  B 


$5,000 

$2,500 

5,000 

$2,500 


Department  C 


$5,000 
$3,500 
10,000 
$2,800 


The  factory  is  supposed  to  run  2,400  hours  a  year. 

(h)  Apply  the  results  obtained  in  (a)  to  the  facts  given  below  for 
job  No.  10,  in  order  to  show  the  different  total  job  costs  obtained  by  each 
of  the  methods.  Assume  (he  material  and  labor  (value  and  time)  charge- 
able to  job  No.  10,  to  be  as  follows: 


Department  A 

Department  B 

Department  C 

Total 

Material 

$1.00 
1.60 
4 

$2.00 
1.50 
3 

$1.00 
1.05 
3 

$4.00 

Labor  Value 

4.15 

Labor  Hours 

10 

(Wis.,  April,  1915.) 

4.  The  boolis  of  Factory  "A,"  the  product  of  which  is  charged  to  the 
main  office  of  the  X,  Y,  Z  Company,  at  factory  cost,  shows  the  following 
facts  January  1,  1910 : 

Cash  (imprest  fund),  $500;  raw  materials,  $17,688.51;  wages  unpaid 
and  distributed,  $2,348.67;  goods  in  process,  at  prime  cost,  $62,258.61,  plus 
$11,352.75  for  factory  expenses,  and  $9,007.50  for  management  charges; 
finished  goods,  $45,290.20. 

The  invoices  for  purchases  of  raw  material  for  the  year  amounted  to 
$78,375.65;  wages  paid,  $133,041.27;  management  charges,  $53,695;  fac- 
tory expenses,  $36,967.08.  The  cash  receipts  for  one  year's  rent  of  loft 
were  $1,200  and  for  11  months'  sale  of  power,  $330,  the  twelfth  month 
being  unpaid. 

The  raw  materials  consumed  for  the  period  amounted  to  $64,188.33; 
management  charges  distributed,  $55,761.90;  factory  expenses  distributed 
to  cost  amounted  to  $43,033.23.  There  was  also  a  loss  on  machinery  re- 
placements of  $107.50. 

The  finished-product  output  for  the  year  amounted  to  $324,583.43,  in- 
cluding all  costs.    The  transfers  to  the  main  office  were  $338,297.90. 

At  the  close  of  the  period,  December  31, 1910,  there  remained  unpaid  and 
undistributed  to  Goods  in  Process  the  regular  factory  payroll  for  three 
days,  amounting  to  $2,857.93,  and  also  1,500  hours  of  operatives'  overtime 
at  an  average  rate  of  45  cents  per  hour,  payable  on  a  basis  of  2^4  hours' 
overtime  as  the  equivalent  of  314  hours'  regular  time. 

Raise  all  the  factory  Ledger  accounts  affected  and  show  final  trial 
balance.  (N.  Y.,  June,  1911;  N.  Y.,  Jan.,  1920.) 


336  C.  P.  A.  ACCOUNTING 

5.  A  company  of  bicycle  manufacturers  makes  up  its  accounts  Decem- 
ber 31,  1907,  for  the  year.  The  following  are  the  debits  to  the  profit  and 
loss  account: 

Raw  Material  on  hand  January  1, 1907 $12,500.00 

Finished  Machines  on  hand  January  1,   1907,   1,600 

Wheels  at  $30 48,000.00 

Purchases  of  Material 62,500.00 

Labor,  productive 82,500.00 

Manufacturing  Expenses:  Coal,  repairs,  paint,  varnish, 
superintendents'    salaries,    unproductive   labor    and 

sundry  other  ejcpenses 23,000.00 

Agents'  Commissions 90,000.00 

Branch  Expense:  Rents,  salaries  and  miscellaneous 40,000.00 

Selling  Expense:  Travelers'  expenses  and  salaries,  dis- 
counts, rebates  and  miscellaneous 30,000.00 

Bad  Debts 8,000.00 

Depreciation  on  Machinery  and  Plant 5,500.00 

The  sales  for  the  year  1907  were  6,000  wheels,  yielding  $540,000;  the 
raw  material  on  December  31,  1907,  taken  at  cost,  were  $4,000,  and  the 
finished  wheels  in  stock  ready  for  sale  numbered  800.  Prepai-e  an  account 
from  the  above  showing: 

(a)  Number  of  wheels  manufactured ; 

(b)  The  cost  per  wheel; 

(e)  The  gross  manufacturing  profit; 

(d)  The  final  net  result,  including  in  the  profit  and  loss  account  the 
stock  of  finished  wheels  on  hand  December  31,  1907,  at  their  cost  as  shown 
by  the  accounts. 

(Mich.,  July,  1909.) 


APPENDIX 

The  Appendix  contains  a  "KEY  TO  REFERENCES," 
and  "REFERENCES  FOR  CHAPTERS."  In  each  chapter 
of  the  text  there  are  superior  figures  after  certain  words. 
Corresponding  figures  will  be  found  under  the  same  chapters 
enumerated  under  the  heading;  "REFERENCES  FOR 
CHAPTERS,"  on  pages  345  to  367    inclusive. 

Immediately  following  the  figures  are  the  Key  Letters,  and 
following  these  are  the  pages  to  which  reference  is  made  in 
the  books  designated  by  the  Key  Letters,  in  the  KEY  TO 
REFERENCES,  on  pages  339  to  344  inclusive. 

To  utilize  the  Appendix,  proceed  as  follows: 
In  Chapter  I,  page  2,  the  first  superior  figure  is  9  (after 
recommends).  Turning  to  the  Contents,  page  xiii,  we  find 
that  References  for  Chapter  I  are  on  page  345.  On  that 
page,  the  number  9  refers  to  A50.  Turning  to  the  KEY  TO 
REFERENCES,  we  find  the  Key  Letter  "A"  on  page  339. 
Opposite  this  Key  Letter  is  the  name  of  the  book,  on  page 
50  of  which  you  will  find  the  reference  desired. 


337 


APPENDIX 

KEY  TO  REFERENCES 

A.  Montgomery,  R.  H.,  Auditing  Theory  and  Practice   (Text  Edition), 

1915,  The  Ronald  Press  Company,  New  York. 

B.  Bennett,  R.  J.,  Corporation  Accounting,  1917,  The  Ronald  Press  Com- 

pany, New  York. 

C.  Castenholz,  W.  B.,  Auditing  Procedure,  1918,  La  Salle  Extension  Uni- 

versity, Chicago,  111. 

D.  Dickenson,  A.  L.,  Accounting  Practice  and  Procedure,  1913,  The  Ron- 

ald Press  Company,  New  York. 

E.  Esquerre,  P.  J.,  The  Applied  Theory  of  Accounts,  1914,  The  Ronald 

Press  Company,  New  York. 

F.  Bentley,  H.  C,  The  Science  of  Accounts,  1911,  The  Ronald  Press  Com- 

pany, New  York. 

G.  Oilman,  S.,  Principles  of  Accounting,  1916,  La  Salle  Extension  Uni- 

versity, Chicago,  111. 
H.  Hatfield,  H.  R.,  Modern  Accounting,  Its  Principles  and  Some  of  Its 

Problems,  1909,  D.  Appleton  and  Company,  New  York. 
I.    Hodge  (A.  C.)  and  McKinsey  (J.  O.),  Principles  of  Accounting,  1920, 

The  University  of  Chicago  Press,  Chicago,  111. 
J.    Klein,    J.   J.,   Elements   of    Accounting   Theory    and    Practice,    1913, 

D.  Appleton  and  Company,  New  York. 
K.  Kester,  R.  B.,  Accounting  Theory  and  Practice,  Vol.  2,   1918,   The 

Ronald  Press  Company,  New  York. 
L.    Greendlinger,  L.,  Financial  and  Business  Statements,  1917,  Alexander 

Hamilton  Institute,  New  York. 
M.  Madden,  J.  T.,  Accounting  Practice  and  Auditing,  1917,  Alexander 

Hamilton  Institute,  New  York. 
N.  Willis,  H.  P.,  The  Principles  of  Accounting,  1911,  La  Salle  Extension 

University,  Chicago,  111. 
0.  Chase,  W.  A.,  Auditing  and  Cost  Accounting,  1911,  La  Salle  Extension 

University,  Chicago,  111. 
P.  Paton  (W.  A.)  and  Stevenson  (R.  A.),  Principles  of  Accounting,  1918, 

Tlie  Macmillan  Company,  New  York. 
Q.  Day,   C.  M.,  Accounting  Practice,  1908,  D.  Appleton  and  Company, 

New  York. 


340  C.  P.  A.  ACCOUNTING 

R.  Racine,  S.  F.,  Accounting  Principles,  1913,  The  Western  Institute  of 
Accountancy,  Connnerce  and  Finance,  Seattle,  Wash. 

S.  Scovell,  C.  H.,  Cost  Accounting  and  Burden  Ajiplication,  1916,  D. 
Appleton  and  Company,  New  York. 

T.  Sprague  (C.  E.)  and  Perrine  (L.  L.),  The  Accountancy  of  Invest- 
ments, 1914,  The  Ronald  Press  Company,  New  York. 

U.  Wildman,  J.  R.,  Principles  of  Auditing,  1916,  The  William  G.  Hewitt 
Press,  Brooklyn,  N.  Y. 

V.  Walton,  S.,  Auditing,  1911,  Alexander  Hamilton  Institute,  New  Y'^ork. 

W.  Wildman,  J.  R.,  Principles  of  Accounting,  1913,  The  William  G. 
Hewitt  Press,  Brooklyn,  N.  Y. 

X,  Hardeastle,  J.,  Accounts  of  Executors  and  Trustees,  1903,  New  York 
University  School  of  Commerce,  Accounts  and  Finance,  New  York. 

Y.  Rcnn,  G.  B.,  Renn's  Piactical  Auditing,  1905,  George  B.  Renn,  Chi- 
cago, III. 

Z.  Spear,  R  H.,  Scientific  Auditing,  1912,  Commercial  World  Publishing 
Company,  Detroit,  Mich. 

AA.  Dicksee,  L.  R.,  Auditing,  1905  (Authorized  American  Edition),  Rob- 
ert H.  Montgomery,  New  Y^ork. 

BB.  Sprague,  C.  E.,  The  Philosophy  of  Accounts,  1907,  The  Ronald  Press 
Company,  New  York. 

CC.  Cox,  H.  C,  Classitied  C.  P.  A.  Problems  and  Solutions— 1915  (1916 
Edition),  The  Ronald  Press  Companj',  New  York. 

DD.  How  to  Audit,  1919,  The  McArdle  Press,  Inc.,  New  Y^ork  City. 

EE.  Saliei-s,  E.  A.,  Accounts  in  Theory  and  Practice,  Principles,  1920, 
McGraw-Hill  Book  Company,  New  York. 

FF.  1914  C.  P.  A.  Problems  and  Solutions,  Vol.  1,  1914,  The  Ronald  Press 
Company,  New  York. 

GG.  1914  C.  P.  A.  Problems  and  Solutions,  Vol.  2,  1914,  The  Ronald  Press 
Company,  New  York. 

HH.  C.  P.  A.  Problems  and  Solutions— 1915,  Vol.  1,  1915,  The  Ronald 
Press  Company,  New  York. 

II.  C.  P.  A.  Problems  and  Solutions— 1915,  Vol.  2,  1916,  The  Ronald 
Press  Company,  New  York. 

JJ.  Cole,  W.  M.,  Accounting  and  Auditing,  1910,  Cree  Publishing  Com- 
pany, Minneapolis. 

KK.  Greendlinger,  L.,  Accountancy  Problems  Avith  Solutions,  Vol.  1,  1910, 
Business  Book  Bureau,  New  York  City. 

LL.  Greendlinger,  L.,  Accountancy  Problems  with  Solutions,  Vol.  2,  1911, 
Key  Publishing  Company,  New  York. 

MM.  Saliers,  E.  A.,  Principles  of  Depreciation,  1915,  The  Ronald  Press 
Company,  New  York. 


KEY  TO  REFERENCES  341 

NN.  Cole,  W.  M.,  Accounts,  Their  Construction  and  Interpretation,  1908, 

Houghton,  Mifflin  and  Company,  Boston. 
00.  Nicholson.  J.  L.,  Cost  Accounting,  Theory  and  Practice,  1913,  The 

Ronald  Press  Company,  New  York. 
PP.    Nicholson  (J.  L.)  and  Rohrbach  (J.  F.  D.),  Cost  Accounting,  1919, 

The  Ronald  Press  Company,  New  York. 
QQ.  Baugh,  F.  H.,  Principles  and  Practice  of  Cost  Accounting,   1915, 

F.  H.  Baugh,  Baltimore,  Md. 
RR.  Kester,  R.  B.,  Accounting  Theory  and  Practice,  Vol.  1,  1917,  The 

Ronald  Press  Company,  New  York. 
SS.   Mitchell,   T.  W.,  Accounting  Principles,  1917,  Alexander  Hamilton 

Institute,  New  York. 
TT.   Klein,  J.  J.,  Bookkeeping  and  Accounting,  1917,  D.  Appleton  and 

Company,  New  York. 
UU.  Miner  (G.  W.)  and  Elwell  (F.  H.),  Principles  of  Bookkeeping,  1912, 

Ginn  and  Company,  Boston. 
W.  Rowe,  H.  M.,  Rowe's  Bookkeeping  and  Accountancy,  1910,  The  H.  M. 

Rowe  Company,  Baltimore,  Md. 
WW.  Baker,  J.  W.,  20th  Century  Bookkeping  and  Accounting,  1912,  South- 
western Publishing  Company,  Cincinnati,  0. 
XX.  McKinsey,  J.  0.,  Bookkeeping  and  Accounting,  1920,  South-Western 

Publishing  Company,  Cincinnati,  O. 
YY.   Finney,  H.  A.,  Introduction  to  Actuarial  Science,  1920,   American 

Institute  of  Accountants,  New  York. 
ZZ.    Rittenhouse    (C.  F.)    and  Clapp   (P.  F.),  Accounting  Theory  and 

Practice,  Unit  I,  1918,  McGraw-Hill  Book   Company,   Inc.,  New 

York. 

AAA.  MacP^arland  (G.  A.)  and  Rossheim  (I.  D.),  A  First  Year  in 
Bookkeeping  and  Accounting,  1913,  D.  Appleton  and  Com- 
pany, New  York. 

15BB.  Approved  Methods  for  the  Preparation  of  Balance  Sheet  State- 
ments, 1917,  Government  Printing  Office,  Washington,  D.  C. 

CCC.  Webner,  F.  E.,  Factory  Accounting,  1918,  La  Salle  Extension 
University,  Chicago,  111. 

DDD.  Sherwood,  J.  F.,  Public  Accounting  and  Auditing,  1920,  South- 
western Publishing  Company,  Cincinnati,  0. 

EEE.  Gilman,  S.  W.,  Cost  Accounts,  1911,  Alexander  Hamilton  Insti- 
tute, New  York. 

FFF.  Rittenhouse  (C.  F.)  and  Clapp  (P.  F.),  Accounting  Theory  and 
Practice,  Unit  II,  1918,  McGraw-Hill  Book  Company,  Inc., 
New  York. 

G(JG.  Reynolds  (W.  B.)  and  Thoiiilon  (F.  W.),  Dudes  of  the  Junior 
Accountant,  1918-1919,  The  Endowment  Fund  of  (he  American 
Institute  of  Accountants,  New  York. 


342 


C.   P.   A.   ACCOUNTING 


HHH,  *  Greeley,  H.  D.,  Business  Accounting,  Vol.  I,  Theory  of  Accounts, 

1920,  The  Ronald  Press  Company,  New  York. 
III.  Cox,  H.  C,  Business  Accounting,  Vol.  IV,  Advanced  and  Ana- 

lytical Accounting,   1920,   The   Ronald  Press   Company,   New 

York. 
J  J  J.        Rittenhouse  (C.  F.)  and  Greeley  (H.  D.),  Business  Accounting, 

Vol.  V,  Illustrative  Accounting  Problems,   1920,   The  Ronald 

Press  Company,  New  York. 
KKK.     1917  Year  Book  of  the  American  Institute  of  Accountants,  1917, 

American  Institute  of  Accountants,  New  York. 
LLL.        Goodyear,  S.  H.,  American  Bookkeeping  Series,  Unit  7D,  1915, 

Goodyear-Marshall  Publishing  Company,  Cedar  Rapids,  Iowa. 
MMM.     Bennett,  R.  J.,  Key  to  Bookkeeping  and  Accounting  Exercises, 

Part  I,  1912,  American  Book  Company,  New  York. 
NNN.      Joi-dan  (J.  P.)  and  Harris  (G.  L.),  Cost  Accounting  Principles 

and  Practice,  1920,  The  Ronald  Press  Company,  New  York. 
000.       Tipson,  F.  S.,  The  Theory  of  Accounts,  1902,  Isaac  Mendoza  Book 

Company,  New  York. 
PPP.       Tipson,  F.  S.,  Auditing,  1904,  Fiederick  S.  Tipson,  New  York. 

QQQ.  Bennett,  G.  E.,  Business  Accounting,  Vol.  II,  Constructive  Ac- 
counting, A  Manual  of  System  Building,  1920,  The  Ronald  Press 
Company,  New  York. 

RRR.  Bennett  (R.  J.)  and  Morton  (F.  W.),  C.P.A.  Questions  and 
Answers,  1914,  International  Accountants'  Society,  Detroit, 
Mich. 

SSS.  Wildman,  J.  R.,  Elementary  Accounting  Problems,  1914,  The 
William  G.  Hewitt  Press,  Brooklyn,  N.  Y. 

TTT.  Mcintosh,  R.  J.,  Reference  Book  of  Accounts  for  Manufacturing 
and  Mercantile  Companies,  1914,  R.  J.  Mcintosh  and  Com- 
pany, Toledo,  Ohio. 

UUU.  Eggleston,  D.  C,  Business  Accounting,  Vol.  Ill,  Cost  Account- 
ing, 1920,  The  Ronald  Press  Company,  New  York. 

VVV,  Greendlinger  (L.)  and  Schultze  (J.  W.),  Modern  Business,  Vol. 
VI,  Accounting  Practice,  1914,  Alexander  Hamilton  Institute, 
New  York. 

AVWW.  Bennett,  G.  E.,  Accounting  Principles  and  Practice,  1920,  Biddle, 
New  York. 

XXX.  Vinal,  E.  R.,  Mathematics  for  the  Accountant,  1920,  Biddle,  New 
York. 

YYY.  Russell  (T.  H.)  and  Jackson  (W.  J.),  Bookkeeping,  Accounting, 
and  Auditing,  1911,  National  Institute  of  Business. 

ZZZ.  Goodyear,  L.  E.,  Principles — Rules  and  Definitions  for  Bookkeep- 
ing, 1916,  Goodyear-Marshall  Publishing  Company,  Cedar 
Rapids,  Iowa. 


KEY  TO  REFERENCES 


343 


A  AAA.        Journal  of  Accountancy,  Volume  1,  The  Accountancy  Publish- 
ing Company,  New  York. 

BBBB.         Journal  of  Accountancy,  Volume  2,  The  Accountancy  Publish- 
ing Company,  New  York. 

CCCC.         Journal  of  Accountancy,  Volume  3,  The  Accountancy  Publish- 
ing Company,  New  York. 

DDDD.         Journal  of  Accountancy,  Volume  4,  The  Accountancy  Publish- 
ing Company,  New  Y^'ork. 

EEEE.         Journal  of  Accountancy,  Volume  5,  The  Accountancy  Publish- 
ing Company,  New  York. 

FFFF.         Journal  of  Accountancy,  Volume  6,  The  Accountancy  Publish- 
ing Companjf,  New  York, 

GGGGr.         Journal  of  Accountancy,  Volume  7,  The  Accountancy  Publish- 
ing Company,  New  Y'ork. 

IIHHH.       Journal  of  Accountancy,  Volume  8,  The  Accountancy  Publish- 
ing Company,  New  York. 

nil.  Journal  of  Accountancy,  Volume  9,  The  Accountancy  Publish- 

ing Company,  New  York. 

JJJJ.  Journal  of  Accountancy,  Volume  10,  The  Accountancy  Publish- 

ing Company,  New  York. 

KKKK.       Journal  of  Accountancy,  Volume  11,  The  Accountancy  Publish- 
ing Company,  New  Y''ork. 

LLLL.  Journal  of  Accountancy,  Volume  12,  The  Accountancy  Publish- 

ing Company,  New  York. 

MMMM.       Journal  of  Accountancy,  Volume  13,  The  Accountancy  Publish- 
ing Company,  New  York. 

NNNN.         Journal  of  Accountancy,  Volume  14,  The  Ronald  Press  Com- 
pany, New  York. 

0000.         Journal  of  Accountancy,  Volume  15,  The  Ronald  Press  Com- 
pany, New  York. 

PPPP.  Journal  of  Accountancy,  Volume  16,  The  Ronald  Press  Com- 

pany, New  York. 

QQQQ.         Journal  of  Accountancy,  Volume  17,  The  Ronald  Press  Com- 
pany, New  York. 

RRRR.         Journal  of  Accountancy,  Volume  18,  The  Ronald  Press  Com- 
pany, New  Y'ork. 

SSSS.  Journal  of  Accountancy,  Volume  19,  The  Ronald  Press  Com- 

pany, New  York. 

TTTT.  Journal  of  Accountancy,  Volume  20,  The  Ronald  Press  Com- 

pany, New  York. 

UUUU.         Journal  of  Accountancy,  Volume  21,  The  Ronald  Press  Com- 
pany, New  York. 

WW.         Journal  of  Accountancj^,  Volume  22,  The  Ronald  Press  Com 
pany.  New  York. 


344  C.  P.  A.  ACCOUNTING 

WWWW.  Journal  of  Accountancy,  Volume  23,  The  Ronald  Press  Com- 
pany, New  York. 

XXXX.  Journal  of  Accountancy,  Volume  24,  The  Ronald  Press  Com- 
pany, New  York. 

YYYY.  Journal  of  Accountancy,  Volume  25,  The  Ronald  Press  Com- 
pany, New  York. 

ZZZZ.  Journal  of  Accountancy,  Volume  26,  The  Ronald  Press  Com- 

pany, New  York. 

AAAAA.  Journal  of  Accountancy,  Volume  27,  The  Ronald  Press  Com- 
pany, New  York. 

BBBBB.  Journal  of  Accountancy,  Volume  28,  The  Ronald  Press  Com- 
pany, New  York. 

CCCCC.  Journal  of  Accountancy,  Volume  29,  The  Ronald  Press  Com- 
pany, New  York. 

DDDDD.  Journal  of  Accountancy,  ^'olume  30,  The  Ronald  Press  Com- 
pany, New  York. 

EEEEE.  Income  Tax  Rulings,  Bulletin  No.  32,  1920—1920  Govern- 
ment Printing  Office,  Washington,  D.  C. 

FFFFF.  Baugh  (F.  H.)  and  Schmeisser  (W.  C),  Theory  and  Practice 
of  Estate  Accounting,  1910,  M.  Curlander,  Baltimore. 

GGGGG.  Administration,  Volume  1,  The  Ronald  Press  Company,  New 
York. 

HHHHH.  Journal  of  Accountancy,  Volume  31,  The  Ronald  Press  Com- 
pany, New  York,  and  The  Journal  of  Accountancy,  Incorpor- 
ated, New  York. 


REFERENCES  FOR  CHAPTERS 


345 


REFERENCES  FOR  CHAPTER  I. 


1. 

J17;    N4;    Rl;    VVl; 

SS8; 

34. 

V206. 

WW5;  AAAl. 

35. 

M322;  AA276. 

2. 

ZZZ201. 

36. 

V171. 

3. 

E54;  J68-9;  Rl;  V17; 

W43; 

37. 

V171. 

AAA5. 

38. 

A  33;  V172-3. 

4. 

P31;  R2;  V23;  GG4;  KK187; 

39. 

C328;  M263;  U162-3;  V173 

AAA22;  ZZZ182. 

Y19. 

5. 

Wl;  CC390;  WW5;  DDD242; 

40. 

V173;  AA261. 

ZZZ165. 

41. 

M263;  V173. 

6. 

R17;  HH113;  RR258; 

SS8. 

42. 

A440;  M272;  V183. 

7. 

J352. 

43. 

A440;  M272;  V183. 

8. 

U2. 

44. 

A442;  M272;  V185. 

9. 

A50 

45. 

A438-9;  M272;  VI 84. 

10. 

A12;    J323;    05-6;    R8;    V8; 

46. 

V185. 

Y15;  ZIO;  AA22;  JJ398-9- 

47. 

A439-40. 

LL219;     DDD6;     00088; 

48. 

A439. 

PPP5;  QQQ6. 

49. 

V183. 

11. 

V8. 

50. 

V183. 

12. 

All. 

51. 

V184. 

13. 

K79;  W287;  DDD8. 

52. 

V186. 

14. 

J323-4. 

53. 

V186. 

15. 

A16;  DDD7. 

54. 

V186. 

16. 

Y15. 

55. 

A53;  O70;  FF283;  III381. 

17. 

VI5. 

56. 

J332;  O70;  P648;  RR501. 

18. 

A17;  V12;  Y15. 

57. 

A53;  AA54. 

19. 

068-9;  V13;  RR212; 

DDD7. 

58. 

A53;  021;  RR50L 

20. 

V12-3. 

59. 

A. 53;  R209;  AA54. 

21. 

A17. 

60. 

A53. 

22. 

V14;  DDD114. 

61. 

A53;  D25;  R210. 

23. 

A12;  AA23. 

62. 

A54;  N189;  064;  AA54. 

24. 

A13. 

63. 

A54. 

25. 

VlO-1. 

64. 

A  56-7. 

26. 

CC46. 

65. 

A56-7;  Y142;  AA37. 

27. 

A7;  AA276-7;  CC44. 

66. 

A57;  V142;  AA37. 

28. 

AA277. 

67. 

A55;  II90. 

29. 

AA274. 

68. 

II9I. 

30. 

AA272-3. 

69. 

A  55;  HH56;  II90. 

31. 

A6. 

70. 

A55;  II91. 

32. 

V205. 

71. 

A55-6. 

33. 

A439-40. 

72. 

A56. 

146 

C.  P. 

A.  ACCOUNTING 

73. 

A5(j. 

77.    A57;  J351. 

74. 

A56; 

Ky.S;  N71. 

78.    A57;    J334;     TIOS;    00181 

75. 

A  55; 

E9U:  N71. 

II36;  DUD98;  0000281. 

7G. 

A56. 

REFERENCES  FOR  CHAPTER  II 


1. 

R2;  V23;  JJ30-1. 

34. 

K556. 

2. 

G3;  HI;  RR15. 

35. 

K556-7. 

3. 

G4-11;  H3-5;  RR15. 

36. 

R165. 

4. 

F16;  J175;R17;  W46 

;NN21: 

37. 

K556. 

RR5G;  YV3;  DDD241. 

38. 

K559;  R226;  WWW164. 

5. 

RR57. 

39. 

K559;R226. 

6. 

RR69;  HHH176. 

40. 

K559. 

7. 

RR62-3. 

41. 

RR267;  XX98;  00034. 

8. 

D14;      E131;      R20; 

W51 ; 

42. 

RR267-8;  XX98. 

FP149;      RR248; 

SS103; 

43. 

RR268;  WW139;  00035. 

VY5;  WW9;  VYV112. 

44. 

RR268. 

9. 

R22;    W51;    FF149; 

SS103 ; 

45. 

P27;  P62. 

DDD248. 

46. 

K568-9. 

10. 

E131. 

47. 

1344-5;  XX417-9. 

11. 

E131;  R22;  EE18;  FF149. 

48. 

K565-6. 

12. 

E131;  RR248 

49. 

K567;  FF178-80. 

13. 

R22;    EE19;    FF80; 
WW9;  AAA22. 

LL212; 

50. 

F20;  J7-8;  P64;  SS84; 
AAA23;  ZZZ191. 

14. 

R55. 

51. 

V23. 

15. 

FF149. 

52. 

P60;  V29. 

16. 

R52. 

53. 

E209;  J9;  KK119;  NN33. 

17. 

R52. 

54. 

P60-1 ;  V29. 

18. 

R54;  W64;  CC405-6; 

55. 

RR234;  WW67. 

DDD253;  ZZZ204. 

56. 

A366;  V214;  AA39. 

19. 

R55. 

57. 

A  366;  C22;  V214. 

20. 

R54;  SS106. 

58. 

A366;  C22. 

21. 

E126;    R220;   GGIO; 

SS106; 

59. 

A298-9;  M293. 

ZZZ177. 

60. 

A308;  C192;  AA30. 

22. 

J31. 

61. 

A  301;  V216. 

23. 

H5;  W50. 

62. 

A301;  PPP7:  BBBBB226 

24. 

W50. 

CCCCC70. 

25. 

E73;  W51. 

63. 

A308;  V216. 

26. 

RR414;  SS106. 

64. 

A  308-9. 

27. 

E129;  R220;  RR420. 

65. 

A  301. 

28. 

R224;  JJ178;  RR422; 

SS115. 

66. 

A  303. 

29. 

RR423;  SS12L 

67. 

A  301. 

30. 

RR414. 

68. 

A303. 

31. 

A301;  U107. 

69. 

M294;  U89. 

32. 

A302-3;  C20;  JJ177. 

70. 

A310. 

33. 

A310-n. 

71. 

A307. 

REFERENCES   FOR   CHAPTERS 


347 


72. 

AA32. 

90. 

WW269. 

73. 

E396;  H35;  J5;  K62;  N165; 

91. 

K36;  SS257. 

P152;  V23;  FF282;   GG8; 

92. 

K32;  V137. 

JJ140;  SS152. 

93. 

K34. 

74. 

J160;    P153;    V12;    FF282; 

94. 

K37. 

JJ146;  KK187;  NN37. 

95. 

K37. 

75. 

J160;  NN39;  SS154;  AAA36. 

96. 

K47-8;  R259;  II209; 

76. 

SS154;  SS157. 

FFF127. 

77. 

J161;  FF146;  GG8;  NN53. 

97. 

K48;  R259;  II209;  FFF127-9, 

78. 

E116;  G82-3;  K590-1;  R227; 

98. 

K41. 

EE139-40;   SSlll-12; 

99. 

K41. 

VVV137. 

100. 

K42. 

79. 

G82-3;  R227;  EE139-40; 

101. 

K42. 

SSlll-2;  VVV137. 

102. 

V136;  ZZZ207. 

80. 

R27. 

103. 

HH55;  PPP16;  BBBBB226. 

81. 

P96;  R27-8. 

104. 

HH55;  PPP16. 

82. 

E113;   R31;    JJ204;   NN392; 

105. 

J338;   V213;   AA34;   HH56; 

SS260-1;  QQQ251; 

PPP16. 

VVV140. 

106. 

HH55-7;  PPP15-6. 

83. 

R31 ;  NN392. 

107. 

HH56-7;  PPP16. 

84. 

K28;  W41;  JJ205;  DDD254; 

108. 

A366;  D29;  HH56;  PPP16. 

ZZZ207. 

109. 

KK95. 

85. 

K29. 

110. 

GGllO;  III413. 

86. 

K29. 

111. 

KK96. 

87. 

D26;  K32;  R259-61. 

112. 

III413. 

88. 

En8-20;  K34-5;  P94-6. 

113. 

CC392. 

89. 

E119;  K35;  P95. 

REFERENCES  FOR  CHAPTER  III 


1. 

F55;    H41-3;    K65-6;    V144; 
AA208;     FF72-3;     SS244; 

10. 

AVWW286-7. 

11. 

2. 

F50;     N166;     0266;     W286; 

CC392;    HH44;    SS168-9; 

12. 

ZZZ169. 

3. 

K63;  R246. 

13. 

4. 

G141 ;  H39 ;  J331 ;  K63 ;  P202 ; 
R246;  JJ173;  KK188; 
SS244. 

14. 

5. 

RR142. 

15. 

6. 

N173;  R248;  W286;  KK188. 

16. 

7. 

T.33-4;   K63-,   P202-3;    W286; 

17. 

WWW309. 

18. 

8. 

K75;  RR240. 

19. 

9. 

F55;  H42;  J263;  K65; 

AA207-8. 

20. 

H54;     J169;     R250;     W289; 

RR241. 
D34-5;   E407-8;   F254-5; 

HHH126. 
A249-50;  B339-40;  K75-6; 

RR240. 
D34;  D38. 
1325;  SS227;  WW153; 

XX394;  III286-7;  JJJ205; 

TTT226;  VVV238. 
J169;  III287. 
F52-3;  K76;  SS226. 
F52;  JJJ52-3. 
A250;  FFFl  09-10. 
1325;   WW153;   XX394; 

ITI286-7;  JJJ205;  VVV238. 
SS227;  TTT226;  WWW314. 


348 


C.  P.  A.  ACCOUNTING 


21. 

E410;     H48;     O105;     R104; 

52. 

LL212;  RR29. 

FF75-6. 

53. 

W55;  CC395;  DDD247; 

22. 

E410;  H48;  O105;  R104. 

QQQQ385. 

23. 

E410;  H48;  O105;  R104. 

54. 

K344;  R155;  CC394;  FF284; 

24. 

K96-7;  R86;  V217. 

GG202;  HH166. 

25. 

C31;  K226;  R71;  AA212. 

55. 

J165-6;  WWIO;  HHH66. 

26. 

D31;  G147;  K97-8;  HHH125; 

56. 

L180;  R85. 

WWW308. 

57. 

L180. 

27. 

K97-8;  V152. 

58. 

L180. 

28. 

K85-6. 

59. 

L180;  CCC59;  ZZZ170. 

29. 

K79;  AV287;  DDD8. 

60. 

E299;  K249. 

30. 

P205;  W295;  SS231. 

61. 

E300;  K249-50. 

31. 

K693;  RR411. 

62. 

G133;  PI  55;  JJ281-2. 

32. 

K693;  W289-91;  RR411-2. 

63. 

AAA142. 

33. 

J231;  CC65-6;  HH22; 

64. 

FF79;  HHfi3;  WW146. 

II248-9;  KK48-9;  LL35; 

65. 

C76-77;  K396;  FF79. 

NNllO;  RR36;  SS55; 

66. 

C59;  F56;  1310. 

XX471;  JJJ94-5; 

67. 

E306;   K343;   L186-7;  P320; 

WWW26-7. 

Hn217-8. 

34. 

B340-3;  F273;  1354;  P588; 

68. 

E366. 

JJ330. 

69. 

G200. 

35. 

CC66;  HH23;  II250;  SR5G. 

70. 

A210;  K403;  III36. 

36. 

CC66-7;  II250-1;  SS57. 

71. 

J30;  RR323. 

37. 

E136. 

72. 

J31;  HH166;  RR324. 

38. 

E136. 

73. 

HH47. 

39. 

LL212. 

74. 

C141;    D146-7;    F58;    K352; 

40. 

r22;  G157-8;  H81;  J163; 

rF153;  HH47;  LL211. 

K71;  N169;  O103-4;  Pill; 

75. 

C19-20;  J30;  RR323. 

W53;  FF280;  ZZZ168. 

76. 

II16. 

41. 

G158;  J165;  K71;  W53. 

77. 

D147-8;  F58;  FF154;  HH48; 

42. 

DD54-6;  BBB25;  DDD40. 

II16;  LL211. 

43. 

r22;  G157;  H81;  J163;  N169; 

78. 

C140;  FF154;  HH47. 

01 03;    Pill;    R99;    W53; 

79. 

D148. 

rF281;  GG263;  II53; 

80. 

G200-2. 

JJ316;  WW9;  CCC59; 

81. 

C19;  G200-2;  FF154;  RR325. 

ZZZ168. 

82. 

0141. 

44. 

FF283. 

83. 

C141-2. 

45. 

GG107. 

84. 

A178-9;   C142;  XXXX228-9; 

46. 

K118. 

BBBBB234. 

47. 

TTH166. 

85. 

018;  Z26;  DD14;  HH255; 

48. 

LL212. 

JJ435-6;  DDD67. 

49. 

G158;  1310;  J165;  M92; 

86. 

0142. 

N160;  W54;  CCC59; 

87. 

0142. 

QQQQ384. 

88. 

F34;  J254;  N41;  W18;  EE23; 

50. 

W52;  ZZZ192. 

LL212;  DDD245;  HHH22; 

51. 

F23;   J103;   LI  79;   N170; 

00015. 

W55;  FF281-.  KK185; 

89. 

F79;  K475;  GG263. 

WWIO;  CCC60. 

90. 

0310-3. 

REFERENCES   FOR  CHAPTERS 


34=) 


91. 

H179;  NN222. 

100. 

P23. 

!»2. 

HUG;  H208. 

101. 

M5. 

!)3. 

WIS. 

102. 

B359;   F254-5;    G143;   J169; 

94. 

ZZZ179-80. 

K76;  L191;  W289;  CC39; 

95. 

L8();  ZZZ18(). 

DD65;  LL69;  SS227; 

96. 

J258;  ZZZ180. 

ZZ153;  FFF68;  HHH103; 

97. 

W20. 

III287;  JJJ134;  SSS82. 

98. 

W20. 

103. 

P23-4. 

99. 

Q312;  Y81;  rF192; 

HH286: 

104. 

P23. 

JJ330;  NNllO; 

JJJ193. 

REFERENCES  FOR  CHAPTER  IV 


1. 

ZZ4;  DDD249;  ZZZ188. 

31. 

F80;  K87. 

2. 

W6-7. 

32. 

F26;  M22;  L49;  N176;  R85; 

3. 

L48;  W225;  W304;  UU367. 

CC403;    KK197;    00015; 

4. 

W58. 

PPP65. 

5. 

D63;  F82. 

33. 

FGO;  F80;  K87;  L49;  W231; 

6. 

K478. 

III296. 

7. 

A207;  NN226. 

34. 

UU317. 

8. 

K388;  II17;  SS23-5. 

35. 

K403;  GG271;  000113. 

9. 

D67;  K390;  SS32. 

36. 

K87;    R84;    CC393;    HH45; 

10. 

W7;  SS32. 

IT103;  KK197. 

11. 

W6. 

37. 

HH45;  JJ312;  NN226; 

12. 

F26;  K482;  R71 ;  W209; 

DDD248;  00041;  ZZZ185. 

FF280. 

38. 

HH45;  NN226;  DDD248; 

13. 

A205;  F26;  FF280. 

00041;  ZZZ185. 

14. 

K405;  HH45;  11X34. 

39. 

00041. 

15. 

HH45. 

40. 

L74-5;  HH45. 

16. 

D217;  G216. 

41. 

K478. 

17. 

D107;  K399;  V151;  AA170. 

42. 

H276-7;  J212-4;  W2;  EE193. 

18. 

E229;  K404. 

43. 

A274-5;  B343-9;  D64-5; 

19. 

H224;  K399-400. 

E433-7;  F265;  G236-7; 

20. 

A209. 

L192-4;  P195;  RR47-8. 

21. 

A209;  K400. 

44. 

J223-5;  M96-8;  R239-43. 

22. 

K400. 

45. 

1324;  DD57;  EE196;  RR48; 

23. 

D106;  K401;  AA171-2. 

XX289;   WW217;  FFF66; 

24. 

K402;  AA171-2. 

JJJ238;  VVV63;  YYYY70. 

25. 

K402. 

46. 

HH12;  UU416;  TTTT466. 

26. 

K402. 

47. 

J227;  EE196;  RR48;  FFF67. 

27. 

A210;  A214-15;  A331;  K403. 

48. 

K479. 

28. 

A213. 

49. 

D64-5;  K485-6. 

29. 

M22;    R85;    CC403;    HH45; 

50. 

BBB24. 

00015;  VVV422. 

51. 

K489. 

30. 

F80;  K87;  M21;  CC392; 

52. 

RR509. 

HH45;    UU317;    DDD245; 

53. 

G119;   RR.388-9;   SS170; 

00015. 

WW212. 

350 


C.  P.  A.  ACCOUNTINa 


54.  G122-3;  P216-7;  II42-3. 

55.  RR388-90. 

56.  G122-3;  J347;  P216-7;  II42-3. 


57.  RR388-9. 

58.  E402-4;  U5G-7;  GG144. 


REFERENCES  FOR  CHAPTER  V 


1. 

E20;  F31;  AV23;  GG82; 

27. 

HH196;  KKl;  RR271; 

28. 

AAA113;  FFFl;  HHH18. 

29. 

2. 

E20. 

3. 

G218;  P255;  FF307. 

30. 

4. 

HH85;  KK214. 

31. 

5. 

HH85. 

32. 

6. 

E20;  RR275. 

33. 

7. 

E21-2;  P256;  W25-6;  KK216; 
LL240-1;  RR275; 

34. 

VVV217-8. 

35. 

8. 

E21;  P256;  RR277;  VVV218; 

36. 

ZZZ201. 

37. 

9. 

P256;  KK213;  RR277; 

38. 

VVV218. 

39. 

10. 

E22;  P256;  RR276. 

40. 

11. 

KK213;  RR276;  VVV218. 

41. 

12. 

J86;  GG186;  RR20. 

42. 

13. 

GG186;  RR20-1. 

14. 

E21;  P256-7;  RR275. 

43. 

15. 

E21;  F31;  G218;  P256;  W25; 

44. 

GG84;  HH197;  LL235; 

45. 

RR21;  AAA114. 

46. 

IG. 

GG84;  HH197;  RR20. 

47. 

17. 

E26-7. 

48. 

18. 

E27. 

49. 

19. 

E27;  GG85. 

50. 

20. 

E27. 

51. 

21. 

E20;    W25;    GG83;    HH199; 

52. 

RR272. 

53. 

22. 

GG83. 

54. 

23. 

A186-90;   E22;   G276;  J88-9; 

55. 

M44-6;  Y117;  FF151-2; 

56. 

KK4;  NN354;  RR274; 

57. 

UU128;  WW98;  XX254; 

AAA113-4;  FFFl; 

58. 

VVY222;   XXXX135. 

59. 

24. 

E24. 

60. 

25. 

E25;  FFF2;  VVV229; 
XXXX140. 

61. 

26. 

E25;  FFF2;  HHH348; 
XXXX143. 

62. 

E25. 

E25;  M75. 

E25;  G219;  J94;  GG137; 

RR280;  FFF9;  XXXX139. 
E25;  RR280-1;  FFF2. 
E25. 

E380;  F23;  HIO;  Ml. 
E381. 
E381-2;  F187-8;  G214;  NN55; 

RR80. 
M47-8;  R21. 
M47-8. 

VV22;  WWIOI. 
E382;  M6;  ZZll. 
A385. 
E382. 
RR48. 
FF206;  HH36;  II233;  KK57; 

RR30. 
F261;  KK76. 
HH218-9;  FFF68. 
M74;  P257;  RR347. 
E382;  M74-5-,  RR 353-4. 
M80. 

E25;  M77;  RR353;  TTTT321. 
E24:  M75;  FF137. 
NN353. 
M47-8. 
M48. 
J92. 

J93;  H317-8. 
G278;  H320;  M49. 
J105;  RR358;  FFF4. 
G280;   H320;   J106;    EEIOO; 

II141;  RR286. 
G278;  H319-20-,  J106-7;  M53. 
H319;  M53. 

G280;  P260;  EEIOO;  RR286. 
G279;  H319;  J106;  M51; 

P261 ;  EEIOO. 
G279;  H319;  M51;  P261; 

EEIOO;  RR362;FFF5. 


REFERENCES  TOR  CHAPTERS 


351 


63.  G280;  EEIOO;  RR286; 

FFF4;  JJJ187. 

64.  JJJ186-7. 

65.  G280;  H318;  M52;  FFF4. 

66.  A24;  M250;  U5. 

67.  A25;  M250. 

68.  A185;  J336-7;  UG3. 

69.  M270. 


70.  E23. 

71.  P257;  W28;  FF307;  RR276. 

72.  LL240. 

73.  W29;LL268-9. 

74.  W29;   LL241;   KR276. 

75.  P257;  W29;LL240-1;RR276. 

76.  LL240. 


REFERENCES  FOR  CHAPTER  VI 


8. 

9. 
10. 
11. 
12. 
13. 
14. 
15. 
16. 
17. 
18. 


E25;  J94;  M53;  RR280. 
J94;  M58;  HHH344. 
J94;  M58. 
J94;  M54;  KK27;  RR281; 

HHH345-6. 
M59;   HHH344. 
J95-6 ;  M50-7 ;  KK27 ;  RR281 ; 

XXXX138. 
M56-7;  11*^1282-3;  HHH346; 

XXXX138. 
M56;  RR283;  XXXX138. 
M56;  XXXX138. 
M59;  HHH344. 
M58-9 ;  RR280. 
M61-3. 
J97;   LLlll. 
LLlll. 

H323;   JJ378;   KK44. 
J97-8;  HHH349;  JJJ29. 
G281;  M64. 
H323;  J98;  M66-7;  P264; 

EE95. 


19. 

KK45. 

20. 

R243;  KK155;  RR349; 

FFFll;   QQQQ462; 

RRRR69. 

21. 

M65;  RR374. 

22. 

M65;  RRRR69. 

23. 

G283;  H324-7;  M65; 

RRRR69. 

24. 

P264. 

25. 

E25;  M46;  RR372;  FFF2. 

26. 

M73. 

27. 

M74. 

28. 

J90;  LL212;  RRRR68; 

XXXX145. 

29. 

P265;  RR371. 

30. 

E20;  RR280-1;  FFF2. 

31. 

M97;  LL212;  XXXX144. 

32. 

KK155;  RR370. 

33. 

RR374. 

34. 

x\I61-3;  KK145;  KK160. 

REFERENCES  FOR  CHAPTER  VII 


1.  E22;  EE91;  HH196;  RR393; 

FFFll. 

2.  A188;  E22-3. 

3.  KK9;  RR394. 

4.  RR394. 

5.  M70;  GGlfiS;  RFi395. 

6.  RR395. 

7.  M78;  RR395. 
S.  RR395. 

9.  RK395. 


10.  M78;  RR395. 

11.  GG168. 

12.  RR395. 

13.  A480;  R71;  AA322. 

14.  A480;  A482;  AA322. 

15.  FF48. 

16.  M76;    P267;    V160;    EE91; 

FF309;  KK7;  RR396; 
FFF12. 

17.  M77;    M1G4;    P2fi7-8;   V160; 

RR396. 


352 


C.  V.  A.  ACCOUNTING 


18. 

M77-8;  V160-1. 

,30. 

B192-4;    J140;    M197-9; 

19. 

CC356-7-,  HH259. 

P277-8;  EE164;  RR459; 

20. 

G284;    H327-9;    M78;    V162; 

FFF94;  JJJ57;  TTT210-1; 

RK395;  JJJ36;  KRR62-3. 

VVV247-51. 

21. 

JlOl. 

31. 

B194;    J140;    M199;    P278; 

22. 

JlOl. 

p]E164-5;  RR460;  FFF94; 

23. 
24. 
25. 
26. 

J102. 

H328-9;  RR399. 

H328;  RR398. 

RR4()0;  FFF94;  JJJ59. 

32. 
33. 

JJJ58;  VVV247-51. 
M82. 
K652;  M82-3;  V162; 

LL169-70;   NN353; 

RR399-400;   HHH356. 

27. 

B193;  RR400. 

34. 

K654;  LL168-9;  NN353; 

28. 

M70;  GG168;  RR395. 

FFF13;  JJJ40. 

29. 

R 145-6. 

35. 

K654;  LL168-9;  JJJ40. 

REFERENCES  FOR  CHAPTER  VIII 


1.  \V29;  FF306;  KK211. 

2.  B2;  FF63. 

3.  Kl;  KK214;  LL239; 

DDD198. 

4.  LL239. 

o.    B2-3;  E28;  K2;  W19-20; 

W29-30;  KK214;  DDD198; 
FFF72-3. 

6.  K3. 

7.  K2-3;  EE148. 

8.  BIO;  J128;  K3;  EE147-8. 

9.  B4;  K2;  FFF73. 

10.  K2. 

11.  B13;  C131;  J115;  K3; 

GG186;   LL234-5;    RR439; 
FFF73-4;  AAAA2V23I. 

12.  B14;  GG186-7;  RR440; 

FFF74;  AAAAA232. 

13.  B5;  F33;  J115-6;  W30; 

KK211;  LL230;  RR441-2; 
VV193. 

14.  B5;  P270;  KK211;  RR442-3; 

VV193. 

15.  B6;   F33-4;   J136-7;   W31 ; 

FF63;  RR441-2. 

16.  B6. 

17.  E39. 

18.  B6;  E39. 

19.  U62. 

20.  E41;  GG87. 


21. 

22. 
23. 
24. 


25. 
26. 
27. 

28. 
29. 
30. 
31. 
32. 
33. 
34. 
35. 
36. 
37. 
38. 
39. 
40. 
41. 
42. 
43. 
44. 
45. 
46. 


Bll;  E41;   F43;  GG87; 

HH201.  ^ 
BIO;  F43-4'! 
r44;  HH89. 
H45;  E41;  GG87;  HH89; 

HH201;  II71;  KK213; 

LL237. 
GG87;  HH201. 
E:41;  GG87;  HH202;  LL232. 
Br2;    J118;    GG88;    HH202; 

KK213;FFF75. 
B12;  B36-7;  GG88;   HH202. 
B33;  KK213. 
11143. 

B13;  F32;  EE151. 
B113;  E41-2;  F175. 
E32;  K299. 
F175;  G310. 
HH87;LL237. 
E40;  II70. 
E40. 

E40;  II70. 
E40. 

B113;  D127;  E40;  J132. 
E40-1;  FF55. 
E41. 
A138. 
II70. 

B47;  E46. 
B47;  M178. 


REFERENCES  FOR  CHAPTERS 


353 


47. 

B48;  J119;  K24;  YlOl. 

87. 

48. 

B48;  E44;  K24;  YlOl; 
KK207. 

88. 

49. 

B48. 

50. 

B48;  E44;  J119;  K24;  YlOl; 

KK207. 

89. 

51. 

A179-80;  C142;  U62-3; 

90. 

XXXX228-9;  BBBBB234. 

91. 

52. 

A179;  C144. 

53. 

K3. 

92. 

54. 

E42. 

55. 

E42-3;  G296;  U62;  W15; 
RR444;  VV194. 

93. 

5G. 

IvK212. 

94. 

57. 

F33. 

95. 

58. 

B8;  J118;  HH87. 

59. 

E33-4. 

96. 

60. 

F36. 

97. 

01. 

E29-30;  HH87. 

98. 

62. 

B16. 

99. 

63. 

E29;  F34;G220;  J118; 

100. 

RR450. 

101. 

G4. 

G221;  J118;  RR450. 

102. 

65. 

DDD195;  BBBBB249. 

66. 

B17;  KIO. 

103. 

67. 

B17-8;  M16. 

104. 

68. 

B17;  M15;  DDD195; 

105. 

BBBBB247. 

106. 

69. 

B91;.BBBBB391-2. 

107. 

70. 

B91;  M16;  CCCCC232. 

108. 

71. 

B91;  M16;  CCCCC232. 

72. 

M16;  BBBBB252. 

109. 

73. 

BBBBB390. 

74. 

Ml 5-6;  HHH422;  III200; 
BBBBB252. 

75. 

B90;  DDD197;  III201;- 
BBBBB253. 

76. 

B89-90;  III201. 

110. 

77. 

BBBBB389. 

78. 

UUUU297;  BBBBB390. 

111. 

79. 

DDD197;  BBBBB253. 

80. 

DDD197;  BBBBB253. 

81. 

BBBBB389. 

112. 

82. 

KIO;  BBBBB250. 

113. 

83. 

BBBBB248. 

84. 

KIO;  BBBBB388. 

85. 

D128-9. 

86. 

J129. 

B19;  E329. 

B21;    G303;    J129;    KlO-11 ; 

L173;  V117;  HH91; 

VV194;  DDD202;  FFF76; 

HHH371;  III201. 
B19. 

CC174;  II188 
E30;  J118;  K4;  HH90; 

KK186. 
E30-1;   J118;  K5;  HH91; 

KK186;  RR451. 
B26;  K6;  CC174;  II138; 

DDD201. 
B26;  K7. 
B25;  K5;  HH91;  KK186; 

rFF75;DDD200. 
K7. 

K7-8;  DDD201. 
B27;  K7;  DDD201. 
K9;  DDD201-2. 
K8. 

K8;  DDD202. 
B27;  KIO;  DDD202;  FFF76; 

00067;  ZZZ207. 
K12;DDD202. 
K13;  DDD202. 
DDD202. 

G310;  H176;  K15-6. 
II149;  VVVV233. 
B359;   J130;   M201;   TT326; 

RRR88. 
C132;     E329;     F37;     G304; 

K384;  M188-9;  P285; 

CC39;  DD56;  GG222; 

KK80;  LL105;  WW215; 

FFFSO;  III193;  JJJ298; 

SSS98;    TTT27;   VVV269. 
B22;  C132;  F173;  L173; 

W197. 
A133-4;    B369;    C132;    F87; 

DD54;  LL83;  RRR109; 

SSS120;  TTT26;  VVV279. 
B369. 
D132;     K248;     P290;     Y81; 

WW215;  BBB25;  DDD41; 

FFF81;  HHH406;  III209; 

JJJ178;  TTTT236; 

DDDDD225. 


354 


C.  P.  A.  ACCOUNTING 


114. 

B61;  E46-7;  F40;  J120; 

151. 

P293. 

152. 

115. 

B62;  r41;  F46. 

153. 

116. 

B62;  E47. 

154. 

117. 

E47;  F46. 

155. 

118. 

E47;  F46. 

156. 

119. 

B64;  E47;  F41-2;  J120; 

157. 

M181. 

158. 

120. 

F46;  P293;  V114;  Z41; 

159. 

XXXX231. 

160. 

121. 

Z41;  XXXX231. 

161. 

322. 

B72;  F47. 

123. 

B73;  E48;  F47;  J120;  K22; 

162. 

M180;  P292;LL237; 

163. 

NN400;  VVV264. 

164. 

124. 

B72;  E48;  P291;  Z41; 

165. 

00031;  PPP74. 

166. 

125. 

B105;  M189;  RR453;  11X192; 

167. 

SSS94-5. 

168. 

126. 

M190;  RRR77. 

169. 

127. 

E31;  W33;  YYY171. 

170. 

128. 

A82-3;  F171-2;  H147;  J153; 

171. 

TT326;  YYYY263. 

172. 

129. 

LL105;  JJJ154;  RRR133. 

130. 

B369. 

173. 

131. 

E45;  RR446. 

174. 

132. 

E45. 

133. 

P290-1;RR446. 

134. 

E48;  M183;  P293. 

175. 

135. 

E53;  M183. 

176. 

136. 

E53;  K23. 

177. 

137. 

B68;  E51;  M179;  VVV262. 

178. 

138. 

B66-7;  F47. 

139. 

B67. 

MO. 

B108;    G309;   J124;   EE158; 
JJ215-6;  RR456;  III194. 

179. 

Ml. 

B57;  J119-20;  M181. 

M2. 

M181. 

180. 

143. 

M180;  JJ218;  NN399; 

VVV265. 

181. 

144. 

B57-8. 

145. 

P291. 

146. 

B54-5;  Jl  19-20;  M181; 

182. 

RR447. 

183. 

147. 

B54. 

184. 

M8. 

B54;  J124;  M181;  RR447. 

185. 

M9. 

B55. 

150. 

1^55. 

186. 

E32. 

J124;  K12;  III210. 

K12. 

GG168. 

B175;  SSS96. 

H222-3;  K19;  III211. 

K20;  R168;  III211. 

K19. 

AA194. 

K19-20;  III211. 

J122;  K19-20;  DDD204; 

III211. 
B117-8. 

J130;  V113;  DDD204. 
G307-8;RR458. 
B164;  J152-3;  M202. 
V113. 
RR455. 
RR455. 
L166;R102. 
E332;  K379. 

B113;  J133;  L166;  TTTT393. 
D128;  1339;  YYYY264; 

AAAAA322. 
B115;  G303;  K379;  JJ294. 
A194;    A375;    B115;    G303; 

H156;  K379;  R167; 

TTTT394. 
A194;  C114;  E331;  R167. 
E332;  FF93;  III203. 
F175;   G312;   H177;  LI  74. 
B120;    F175;    6312;    H176; 

J126;  L174;  W197;  II145; 

NN405;  III203;  TTT206. 
B120-1;    F176;   G312;    J128; 

H177;  LI  75;  AV197; 

NN405;  III203;  TTT207. 
B120-2;  F176;  G312;  H178-9; 

L175-6;  W197-8;  III203. 
B123;  G312-3;  P287;  W198; 

CC380;  IIH42;  III205; 

YYY175. 
B124-5. 

K16;  SSS118-9. 
F175;  J127. 
B125;  LL24;  111209; 

YYY176-7. 
K17-8. 


187. 
188. 
189. 
190. 
191. 
192. 
193. 
194. 
195. 

196. 
197. 
198. 
199. 


200. 

201. 
202. 
203. 
204. 
205. 

206. 

207. 

208. 
209. 
210. 

211. 
212. 
213. 


REFERENCES  FOR  CHAPTERS 

214 

215. 


355 


B125;  K16-17;  III209. 

E40;  K381. 

B473 ;  E33 ;  K381. 

E33;  M205. 

H160 ;  K381-2. 

D129;  H159-60;  K381-2.  216. 

A192;  C132;  M318-9;  Q42.         217. 

Q42. 

A388;  C131;  M319;  V113-4;      218. 

BBB21. 
A388;  CC161.  219. 

V114. 

V114.  220. 

A193;  C132;  CC161;  DD41; 

JJ436;  DDD204;  221. 

XXXX229.  222. 

A388;    C132;    V113;    DD41;      223. 

DDD204;  XXXX229.  224. 

C132. 

DD42;  BBB21;  DDD204.  225. 

A192-3;  C133.  226. 

A193.  227. 

A193;  DD42;  BBB21;  228. 

DDD204.  229. 

CC162.  230. 

A345;   E316-7;   H76;   K330; 

FF289. 
K330.  231. 

K330;  TTT26. 
B93;   F178;   AA191;    GG50; 

III17;  DDDDD314.  232. 

A345;  C66-7.  233. 

FrF135.  234. 

B357;  CC18;  RRR259; 

NNNN174. 


WWW296;  YYYY71; 

DDDDD314. 
B129;  C38;  D51 ;  DI)54; 

BBB25;  DDD230;  III288; 

JJJ297. 
C38;  E262;  K241. 
D34;  W294;  DD24;  BBBIO; 

VVV40. 
C38;  D38;  K259;  L141; 

DD24. 
A99;    P577;    W294;    DD24; 

BBBIO;  VVV40. 
C39;  D117;  G189;  K242; 

L143;  0121;  VVV41. 
A102. 

A131;  L143. 
K243. 
C39;   D117-8;  K243;   W281; 

DD25;  DDD75. 
E283. 

C40;  K260;L239;R92. 
W230. 
D84. 

A323;  DD25;  DDD75. 
C38;   M302;   Q40;   CC42; 

DD25;   FF217;   BBBIO; 

DDD74;  GGG53;  PPP9. 
C38-9;    Q40;    CC42;    DD24; 

HH162;   BBBIO;   DDD74; 

GGG53;PPP9. 
C38;  DD25;  BBBIO;  DDD74. 
C38. 
C40;  M303;  Q25;  DD25; 

BBBlO-11;  DDD75. 


REFERENCES  FOR  CHAPTER  IX 


1.  R58;  RR320. 

2.  AAA153. 

3.  F133;    R58;    W251;    NN25; 

NN159;  AAA53. 

4.  AAA53. 

5.  R58;  RR320-1. 

6.  W254;  RR551. 

7.  P355;  RR551. 

8.  F133. 


9.  NN25;  RR551. 

10.  P350;RR543-4. 

11.  F133;FF78;RR252;  WW94. 

12.  RR253;  WW94. 

13.  C62;  F133. 

14.  RR223-4. 

15.  F137;  H97;  L152;  P189; 

V103;  JJ68;  RR118. 

16.  F137;  P189;  RR119. 


356 


C.  P.  A.  ACCOUNTING 


17. 
18. 
19. 
20. 
21. 
22. 
23. 

24. 
25. 
26. 
27. 

28. 

29. 

30. 
31. 

32. 

33. 
34. 


35. 


36. 
37. 

38. 
39. 
40. 
41. 
42. 
43. 
44. 
45. 
46. 
47. 
48: 
49. 
50. 


51. 
52. 


B256;  W254;  III142. 

W130;  WW151;  XX381. 

L152. 

G231;  P189;  AA59;  RR255. 

W252. 

G231;  1325;  L62;  RR255. 

B357;  P:E197;  FF256; 

VV244;  AAA178;  RRR99. 
W251;  RR199. 
RR198. 

RR199;  UU119;  AAA56. 
W252;  RR199. 
M135;  RR550;  YYY89; 

VVVV316;  BBBBB397-8. 
RR549;  YYY89;  VVVV316; 

BBBBB397. 
GG49-50. 
C175. 
A384;     V97;     XXXX229-30; 

YYYY151;  AAAAA75. 
A 324-5. 
S98;  JJ356;   NN279;   0033; 

PP139;  EEE374; 

UUU241-2. 
A95;  D206;  K472;  L233;  W9; 

II13;  BBB14;  NNN443; 

CCCCC155. 
S97-8. 
K470;    L234;    S116;    CC202; 

II13;  0041;  CCCCC156. 
K470;  0041. 
K470. 

K470;  CC202;  II13. 
K471;  CC202;  II13-4;  0041. 
S104;  S116;  S118. 
S123-4. 
S123. 

L235;  OO40. 
PP140. 

PP140;  NNN434. 
PP140;  NNN434. 
P349. 
K272;  CC391;  NN166; 

SS285;  YY13;  DDD244; 

III146;  RRR47;  SSSS40G. 
T120;  CC401;  SS281-2;  YY3. 
P352;  T121;  NN420;  SS282; 

YY5;  XXX48;  SSSS407. 


53.  B286;    H271;     P361;    Tr22; 

CClll;  NN42G;  QQ58; 
SS288;  YY15;  III147-8; 
XXX61;   SS8408; 
YYYY77;  AAAAA245. 

54.  B287;  G341;  H271:  P372; 

T122;  CClll;  NN428; 
YY21;  III152;  RRR52; 
XXX76;  PPPP361; 
SSSS413;  AAAAA247-8. 

55.  P359;  T121;  NN421;  QQ56; 

SS285;  YY12;XXX52. 

56.  K272;    P367;    T122;    LL65; 

NN427;  QQ56;  SS289; 
YY31;  III149-50;  XXX66; 
YYYY77;  AAAAA245; 
CCCCC314. 


57. 

P375;   T122;  NN428;  YY40. 

58. 

H95-6;  K273-4;  T122;  LL64; 

NN168;    YY46;    VVV320; 

AAAAA246. 

59. 

T122;  YY49-50. 

60. 

T108;  NN200. 

61. 

T108. 

62. 

T104-6. 

63. 

T106. 

64. 

A130;  K269. 

65. 

K269. 

66. 

K270. 

67. 

K270;    P383;    T89;    CC119; 

LL65;  NN171;  YY48; 

JJJ150;  BBBB45; 

VVVV321. 

68. 

K270;    P381;    T90;    NN172: 

YY52;  DDDDD217. 

69. 

T95;   Nm79;   YY52-3; 

XXX107-8. 

70. 

T96. 

71. 

SSSS419. 

72. 

YY54-5. 

73. 

NN201;  YY56-7. 

74. 

YY28. 

75. 

TlOO;  YY58-60;  0000338-9; 

YVVY322;  DDDDD216. 

76. 

YY29;  CCCCC470-2. 

77. 

YY24-5. 

78. 

P363. 

79. 

YY36. 

REFERENCES  FOR  CHAPTERS 


357 


80.  P368. 

81.  T63;  YY37;  XXX70-, 

ZZZnd;  SSSS406. 

82.  yY38;  XXX70. 

83.  T64;  XXX70. 

84.  T62;  YY18;  III147;  XXX68. 


85.  YY19;  XXX69. 

86.  T63;  XXX68. 

87.  SSSS419. 

88.  YY34;  XXX70. 

89.  T63;  XXX70;  DDDDD215. 


REFERENCES  FOR  CHAPTER  X 


1.  A181;    D149;    G329;    K411; 

R171. 

2.  K411. 

3.  C103;  M17;  HHG4. 

4.  E373;  F54;  G329;  K411 ; 

WW9. 

5.  B331;   p]408;  W182. 

6.  C103;    DISO;    F53-4;    G334; 

R173;  K411;  nH84; 
RR252-3. 

7.  B434;  G186;  Dl)];);  BBB15-, 

DDD135. 

8.  E252;  K322;   W278;   III259. 

9.  B332. 

10.  C33;K228. 

11.  C33. 

12.  L223. 

13.  E373. 

14.  K415;  M17;  P311; 

XXXX476. 

15.  K413. 

16.  K413. 

17.  K415. 

18.  K415-6. 

19.  C99;  F161;  R166. 

20.  F140;  K305;  R117;  R182. 

21.  K416. 

22.  E374;    G221 ;    K416;    P310; 

FF83;  XXXX476. 

23.  B325;    C116;    F151;    G342; 

R183. 

24.  B326;    D151;    E377;    F151; 

G342;   L208;   0147;  R183; 
V119;    KK189;    0000298. 

25.  B326;    G342:    L209;     0147; 

R184;  V119;  KK189; 
0000298. 

26.  K418. 


27. 

28. 

29. 
30. 
31. 
32. 
33. 
34. 
35. 

30. 
37. 
38. 
39. 

40. 
41. 
42. 
43. 
44. 
45. 
46. 
47. 
48. 
49. 
50. 


51. 
52. 
53. 

54, 


55, 
56, 


A143;     C119;    F151;    G342; 

K419;  R184;  II60. 
D152;  G342;  K419;  L201); 

R184. 
B326;  L210. 
K420. 

G343;  K420. 
K420;  L210;  R184. 
G221;  K417;  HH45;  HH64. 
G330. 
A 197;  C114;  G334;  M18; 

W174;  HH64. 
A196;  L219;  M18. 
E369;  G330;  W56. 
K461;  CC113;  EE187. 
K463;  L219;  W185;  EE187; 

CC114. 
B289;  L220;  V108;  GG260. 
B336;  G335;  V108. 
E292;  G335. 
E292. 
E292. 
R194. 

B324;  HH64;  DDD252. 
B289;  L220;  V108;  GG260. 
E369;  G330;  L220. 
B324;  E293;  G335-6. 
B204;    B279;     B324;    K448; 

R190;  V104;   PIH64; 

ITI143;PPP106;ZZZ201. 
B280;  G336-7;  K448;  R190. 
B280;  B289;  L218. 
B280;  E294;  R190. 
B324-5;  D148;   E376;  L218; 

L220;   FFFF395; 

KKKK263. 
M307. 
A128;  M307. 


358 


C.  P.  A.  ACCOUNTING 


57.  A 128;  M307. 

58.  A128;  B294;  B302. 

59.  B203;  B302;  K458;  HH65. 

60.  A128;    B302;    0141;    P327; 

HH65. 

61.  A128;  P327;  JJJ154. 

62.  K458;  G211;  H192;  DDD73. 

63.  B292. 

64.  B292;    K4G1;    L222;    R195; 

DDD73;  III158;  SSSS392; 
WWWW203;  XXXX478. 

65.  B297;  DDD73. 

66.  K460;  FF115;  SSSS390; 

WAVWW202;  XXXX478. 

67.  B299. 

68.  K462;  L218;  DDD73;  III160. 


69. 

70. 
71. 
72. 
73. 


74. 
75. 
76. 
77. 

78. 


L218;   II86;  DDD73; 

III159-60. 
K4G2;  DDD73;  III161. 
II86;  III159. 
B299;  D148;  L218. 
F179;   K459;   P306;   CC113; 

EE187;   II86;   DDD72; 

III157. 
B299;  111157;  PPP106. 
B300-1. 

K461;  PPP106. 
K462. 
B308;    F179;     K463;     L219; 

P308;  W181;  CC114; 

EE187;  II86;  NX126; 

DDD74;  III161;  PPP106. 


REFERENCES  FOR  CHAPTER  XI 


1.  E34;  P323;  W114;  W169; 

ZZZ170. 

2.  B208;  E37;  V99;  W170; 

II103;  NN242;  DDD180; 
IIII276. 

3.  B217;   E37;   W170;  II103-4; 

Kia96;  DDD180. 

4.  B217;  B245;  DDD181. 

5.  B217;  B245;  E35;  P325; 

NN242.    ^ 

6.  B248-9;   E35;  P325;  NN243. 

7.  E36. 

8.  E36. 

9.  E36-7. 

10.  B209. 

11.  B220;  E37;  DDD181. 

12.  B218;  E38;  DDD181. 

13.  B218;  E38. 

14.  B217;  E37;  P325;  W170; 

III 03-4;  KK196;  DDD180. 

15.  B216-7;  DDD182. 

16.  B211;  U149. 

17.  B231;  m48;  DDD182. 

18.  B213;  K360;  DDD182. 

19.  B214;    B313;    P325;    CC404; 

DDD182. 

20.  B215;  E37-8;  U148;  DDD182. 

21.  B213;E38;  P325;DDD182. 


22. 

P327. 

23. 

RRR87. 

24. 

B242;  G211;  K369;  V99; 

DDD184;  SSS186; 

VVV273. 

25. 

R157. 

26. 

H191. 

27. 

E339;  G211. 

28. 

K370. 

29. 

K370;  0142;  FF78. 

30. 

B241-2;  K364;  R157;  III140. 

31. 

B242;  K364;  R157;  III139. 

32. 

K364-5;  R157. 

33. 

B244;  DDD184. 

34. 

B268-9;  E341-2;  K365; 

NN199. 

35. 

NN199. 

36. 

C59;  D35;  F91;  G205;  CCIO; 

FF93;  II82;  FFF87; 

JJJ155. 

37. 

B96;  D45;  G207;  FF93; 

LL69;  FFF87;  YYYY68. 

38. 

P577. 

39. 

B268;  DDD185. 

40. 

J134. 

41. 

B268;    F91;    CC117;    II287; 

DDD185;  DDDDD313. 

42. 

J134;  TTTT235. 

REFERENCES  FOR   CHAPTERS 


359 


43. 

B247. 

63. 

44. 

B249. 

64. 

45. 

B250-1. 

65. 

46. 

B252. 

66. 

47. 

B254;  K366;  U150;  W173. 

67. 

48. 

B259-60. 

68. 

49. 

B261. 

69. 

50. 

B261. 

51. 

B262;  E37;  W170;   II103-4; 
KK196;  DDD180. 

70. 

52. 

B262. 

71. 

53. 

B269;  D134;  K367;,DDD185. 

72. 

54. 

B269;  D134;  DDD185. 

73. 

55. 

A374-5;   B270;   D134;   E305; 
R166-7;  NN199-200; 
DDD185. 

74. 

5(3. 

V102;  DDD185. 

75. 

57. 

B270;  P407;  FF123;  NN199; 
DDD186. 

58. 

B231;  F182. 

76. 

59. 

B232-4;  F182-3. 

60. 

B237;  B253;  F183;  U150. 

77. 

61. 

A 158;  A388;  C122-3;  M316; 

V98-9;   DD40;   BBB20; 

78. 

DDD188. 

79. 

62. 

Q42. 

A388. 

A388;  C123. 

M316. 

A388. 

A388;  C123;  U149. 

C124;  U149-50. 

C124;  U150;  DD40;  HH152; 

JJ435;  BBB20;  DDD188. 
A389;  M316;  DD40;  BBB20; 

DDD188. 
V99. 
D118. 

T152;  W113. 
M302;  C38;  Q40;  FF217; 

DD25;  BBBIO;  DDD74; 

PPP9. 
C38;  Q40;  CC42;  DD24; 

FF122;  GG204;  HH161; 

BBBIO;  DDD74;  PPP9. 
A323;  DD25;  BBBll; 

DDD75. 
C38;  DD25;  JJ435;  BBBIO; 

DDD74. 
M303;  Q25. 
DD25;  BBBll;  DDD75. 


REFERENCES  FOR  CHAPTER  XII 


1. 

E30;  E355;  J136;  K428; 

14. 

L212;  EE183;  RR461. 

15. 

2. 

L213;  P301-2. 

16. 

3. 

B142;  P302;  K436;  III72. 

17. 

4. 

B153;  K436;  L211;  V160. 

18. 

5. 

B151;  E361;  K439;  L211 ; 

P303;  V157. 

19. 

0. 

B152;  K438;  III72. 

20. 

7. 

B144;  K438;  L216;  P303; 

21. 

V156;  III72. 

22. 

8. 

B145-6;  III86-7. 

23. 

9. 

A230-1;  F45;  K433;  L213; 

24. 

V156. 

25. 

10. 

L105;   P233;   FF160;   11157; 
YYY211. 

26. 

11. 

K446. 

27. 

12. 

D74;  K406. 

28. 

13. 

E358-9;  G184;   HH69;  1161. 

29. 

K379;  CC235. 

D70;  H208. 

D73. 

K433;  L216. 

B133;   F45;  J118;  V156; 

K429;  L212;  RR461. 
K431;  V157. 
K432. 

E361;  J137;  V159. 
B135;  K432;  11163. 
B136;  K432. 

B136;  E356-7;  K434;   III66. 
E357. 
F174;  L173;   I1I69; 

VVVV66;  XXXX410. 
K434;  L212-3;  ITI66. 
B142;  K442;L212;  JJ213. 
K349;  L213;  AAAAA76. 


360 


C.  P.  A.  ACCOUNTING 


30. 

B153;  E362;  K349;  JJ437. 

53. 

31. 

E362;  K349. 

54. 

32. 

B142-3;  E362;  G199;  K442; 

55. 

JJ213;  KK77;  RR462; 

56. 

III82. 

33. 

B143;    E362;    F181;    G199; 

57. 

K442;P301;V159;GG329; 

58. 

RR463;  III83. 

34. 

L212. 

59. 

35. 

RR460-1. 

60. 

36. 

B142;  G326;  J136-7;  P301-2; 

61. 

JJ213;  III82-3. 

62. 

37. 

B153;  K436;  L211;  V160; 

63. 

III92. 

04. 

38. 

B153;L211. 

65. 

39. 

III92. 

06. 

40. 

B151;   EE184;   QQll ;   II 191. 

U7. 

41. 

B144;  K438;  P303;  III85. 

42. 

B152-3;  III92. 

C8. 

43. 

B144-5;   B152-3;  0328; 

69. 

III85-6;  III92;  YYY179. 

70. 

44. 

B145;  III86. 

73. 

45. 

B149;  K434;III87. 

72. 

46. 

B149;  III89. 

73. 

47. 

B150;  III90-1. 

74. 

48. 

B150;  III91. 

75. 

49. 

A198;    C133;    R156;    W196: 
CC21;  DD42;  GG198; 

76. 

HH47;  JJ438;  BBB21 ; 

77. 

DDD204;  JJJ241; 

78. 

000151. 

79. 

50. 

B76-8;  B138-41;  r48;  V150; 

III80-1;  ZZZ181. 

80. 

51. 

Q25;  JJ423. 

52. 

AA48. 

025;  AA48-9;  JJ423. 

Z42;  AA49;  JJ437. 

B145-6. 

B334;  F180-1;  J136;  L195; 

P299 ;  VV167. 
B335;  L207;  III97-8. 
1279;  J118;  K407;  P294; 

HH45;  SS106;  00019. 
G302;  L196. 
L196. 

B334;  G221;  K407. 
G303;  L207;  JJ223-4;  III96. 
B335-6;  G312;  K408;  AA47. 
L196;  JJ302;  III96. 
L208. 

K408;  L196;  III95. 
A194;  A375;  B15;  G303; 

K379;  R167. 
1280;  JJ224. 
K426;  GG211. 
K408;  JJ302. 
P550;  DD57;  III292. 
K426;  1 1 1292. 
K426. 

I35G;  K426-7;  P198;  III306. 
C133. 
DD42;   GG320;    BBB21; 

DDD209. 
Q42. 
SS106. 
EE220;  JJ330;  LL19;  III94; 

SSS186. 
B462;  D47:  F53;  6.301; 

DD56;  BBB25;DDD41; 

RRRIOO. 


REFERENCES  FOR  CHAPTER  XIII 


1.  E165;  CC399;  DDD250; 

ZZZ194. 

2.  E167;  R56-7;  RR88. 

3.  F118;  G35. 

4.  F117-8;  R57;  GG12;  118. 

5.  F117;  V126;  GG12;  HH115; 

118;  RR88-91. 

6.  G172. 

7.  G175. 


8.  A54-5;  G175;  1 190. 

9.  II91. 

10.  A55;  G175;  II90. 

n.  A55;  II90-1. 

12.  C193;  J341. 

13.  A338;  m04. 

14.  A338-9;  C156. 

15.  U104-5;  AA90;  FF294. 

16.  A 151;  AA174-5. 


REFERENCES  FOR  CHAPTERS 


361 


17. 

C156;  029;  FF294. 

57. 

J173;  J181;  Y28;  EE278; 

18. 

U105. 

ZZZ206. 

19. 

A301. 

58. 

A455;  W213;  EE278; 

20. 

A301;  U106. 

DDD99. 

21. 

A301. 

59. 

A455;  L66;   W213;  RR65-6; 

22. 

D147;  FF234. 

DDD99;  ITT396; 

23. 

A151-2. 

WWWW190. 

24. 

C91;  L310;  ]{70;  DD21; 

60. 

Flll-2;  C20;  K230;  R67; 

RR381;   BBB17;  DDD102. 

V57;  W124-5;  CC296; 

25. 

A56. 

NN297. 

26. 

A56. 

61. 

K230;  F112;  W124-5; 

27. 

A315;  C150. 

CC296;  NN297. 

28. 

P91;  RR343. 

62. 

K230;  NN297;  VV.142. 

29. 

A301. 

63. 

K230. 

30. 

A303. 

64. 

C26;  K230. 

31. 

C152-3. 

65. 

D104-5;  K230. 

32. 

RR344. 

66. 

W204-5. 

33. 

RR344. 

67. 

J213;  WW217. 

34. 

C261. 

68. 

WW191. 

35. 

FF94;  XX403;  QQQ182. 

69. 

146;  XX418. 

36. 

XX403. 

70. 

K230. 

37. 

DDDDD74. 

71. 

C149;  V127;  W201-2. 

38. 

EEEEE12. 

72. 

F92;   J213;  VV85;   WW194. 

39. 

R68;  RR343. 

73. 

L55. 

40. 

A210;  A214-5;  A332;  K403; 

74. 

RR243;  VV87;  WW192. 

V130;  AA172;  III36; 

75. 

F92;  J213;  L70;  FF180; 

VVVV401. 

XX417. 

41. 

V130;  AA172. 

76. 

A357. 

42. 

A332;  C71-2;  D102-3;  K403; 

77. 

C70;L71;  ^YW193. 

RR343;  VVVV401. 

78. 

A456-7;  C7();  L172;  0119; 

43. 

RR382. 

II89;  SS181. 

44. 

V128;  RR341. 

79. 

F88;  WWII;  111307; 

45. 

K530. 

NNNN174. 

46. 

R214;  V128;  RR341. 

80. 

C172;  G252. 

47. 

L49;  M298;  R68;  W167; 

81. 

1324;  J172;  RR243;  VV142; 

RR341;  QQQ178. 

WW217;  XX289;  JJJ49. 

48. 

F217;  L52;  NN142. 

82. 

E88;  N71. 

49. 

RR141;  JJJ99. 

83. 

GG54;  PPP97. 

50. 

RR425. 

84. 

C189. 

51. 

RR141. 

85. 

HH56-7. 

52. 

M298;  R68;  L49;  032; 

86. 

W201. 

RR482;  QQQ180. 

87. 

L69;  V126;  VV142;  XX289. 

53. 

RR483. 

88. 

L71-2. 

54. 

RR483. 

89. 

JJJ226. 

55. 

L50;    M298;    R214;    RR484; 

90. 

1324;  V126;  W201;  IT45; 

QQQ181. 

RR243;    VV142;    WW217; 

56. 

R67-8;  KK112. 

XX289;  JJJ49. 

362 


C.  P.  A.  ACCOUNTING 


91. 

W202. 

113. 

A90;  C27;  CC292;  DD22; 

92. 

1324;  W205;  RR243;  VV142; 

BBB13;  DDD103. 

WW217;  XX289;  JJJ49. 

114. 

A91;  C27;  C36;  DD17;  II49; 

93. 

U106-7. 

II155;  JJ430;  BBB13; 

94. 

A36r)-,  C173. 

DDD103;  000138; 

95. 

D108;  K353-4. 

PPPIO;  XXXX236. 

96. 

CC315. 

115. 

A91;  C27;  CC292;  GG331; 

97. 

CC315. 

1148;  II155. 

98. 

G34;  CC398;  VV53; 

116. 

A91;  C26-7;  CC292-3;  DD21; 

DDD249;  ZZZ190. 

FF227;  GG318;  II49; 

99. 

DUO;  G171;  K240;  U108. 

II160;  BBB12;  DDD103. 

100. 

D109;   E171;    F116;   G171-2; 

117. 

A91;  C28;  CC293;  DD23; 

K238-9;    U108;     GG180-1; 

FF228;   BBB13;   DDD104. 

DDD245. 

118. 

A84;  A99;  C27;  UllO;  DD22; 

101. 

A395;  U108;  W120;  GG181; 

GG50;  II154;  VV53; 

DDD98. 

BBB12;  DDD103;  PPP65. 

102. 

A395;  DUO;  UIOS. 

119. 

C32-3;  L156-7;  W124. 

103. 

DUO. 

120. 

GG191;  HHH205;  III272-3. 

104. 

A88;  C25;  CC290;  DD20; 

121. 

J327;   R72;    GG63;   HH165; 

GG331;  1148;  BBBll; 

000146. 

DDDIOI. 

122. 

CC302;  HH231. 

105. 

A88;  C25;  Z37;  CC290; 

123. 

E155;  VI 34;  RR299; 

DD17;  FF228;  GG50; 

ZZZ205. 

II160;  JJ430;  BBBll; 

124. 

A227;  L51;  AA189;  CC296; 

DDDlOl;  PPPIO; 

DDD82;  PPP28. 

LLLL356. 

125. 

H288;L50;  W202;  DD56; 

106. 

A88;   C25;  L308;   Q41 ;  Z37; 

T.L97. 

CC290;  DD20;   GG318; 

126. 

W207;  LL97. 

II48;  BBRll;  DDDlOl ; 

127. 

A  325-6;  DDD83. 

PPnO;  LLLL356. 

128. 

Y135-6. 

107. 

A88-9;  C25;  L308;  Q41 ;  Z37; 
CC290 ;  DD20 ;  GG318 ; 

129. 

V134-5;  AA59. 

II48;   BBBll:    DDDlOl; 

130. 

P72;    S35;   GG231;    KK113; 

PPPIO;   LLLL356. 

LL12;     VV142;     WW217; 

108. 

A89;  C25;  C36;  CC291; 

AAA157;  EEE272; 

DD20;  FF228;  GG181; 

RRR123. 

II155:  BBBll;  DDD102. 

131. 

B348;  E436;  F59;  G250; 

109. 

A89;   C2f);   CC291-2;   DD21; 

T325;  J172;  L62;  R242; 

FF227;  III 55;  BBBr2; 

U186;  W230;  CC19;  DD57; 

DDD102;  PPPf.5. 

IvK87;  QQ6;  RR255; 

110. 

A89;    C26;     CC292;    DD21; 

IJU386;  XX289;  ZZ113; 

FF227;   II49;  II160; 

III308;  JJJ49;  NNN314; 

BBB12;  DDD102. 

SSS55;  TTT225; 

111. 

A89-90;  C20;  CC292:  DD21 ; 

WWW181 ;  YYYY70. 

BBB12;DDD102. 

132. 

V135. 

112. 

A 90;  C27;  Z37;  CC292; 

133. 

J172;  P72;  GG232;  1 145; 

DD22;  JJ430;  BBB13; 

KK113;LL12;  VV142; 

DDD103. 

WW217;  AAA157. 

REFERENCES   FOR  CHAPTERS 


363 


134.  B367;  EE197;  HH12; 

RRR124. 

135.  B348;  E436;  F59;  G250; 

1342;  R242;  U186;  W232; 
CC18;DD57;  RR255; 
UU386;  XX289;  ZZ114; 
III308;  JJJ49;  NNN314; 
SSS55;  TTT225; 
WWW185;  YYYY70; 
CCCCC148. 

136.  A317. 

137.  A317-8. 


138.  P654;  JJ289-90;  NN342; 

SS144-6. 

139.  RR308-9. 

140.  P654. 

141.  K414-5;  WWW231. 

142.  C116;  R176. 

143.  F135;  DD54;  KK46;  SS388; 

BBB25;  DDD40;  111288; 
WWW231;  DDDDD224. 

144.  JJJ240. 

145.  CC260;  RRR276. 

146.  SS388. 

147.  F136 ;  AA189. 


REFERENCES  FOR  CHAPTER  XIV 


1. 

A363;L78;  W263;RR117. 

25. 

2. 

E309;  F128;  K571;  W155-6; 
W264-5. 

3. 

A363;  E309-10;  W155-6; 

W265. 

26. 

4. 

K580;  QQQQ465. 

27. 

5. 

C60;  W264;  VV95. 

28. 

6. 

C60;  F129;  1311;  W264; 
WWr24;  XX372;  QQQ344. 

29. 

7. 

C60;   1311;   W264;  WW125; 
XX373. 

30. 

8. 

C60. 

9. 

H259;  P308;  R178-9; 
KK161;  EEEE198. 

31. 

10. 

R179;  W268;  XXXX475. 

11. 

H260;  XXXX475. 

12. 

P309;  R179. 

32. 

13. 

L223-4;  R192;  W180; 

KK161. 

33. 

14. 

C50-1;  C62;U46;  CC265; 

34. 

II322-3. 

35. 

15. 

C50-1;  C61. 

36. 

16. 

C51;  DD31;  BBB16. 

37. 

17. 

C61-2;  CC266;II322. 

38. 

18. 

A362;  DD48. 

30. 

19. 

C61;  Q41;  GCt318. 

20. 

CC266;  II322. 

40. 

21. 

K575;  U132;  W266-7. 

41. 

22. 

K575. 

42. 

23. 

K576. 

43. 

24. 

K578. 

44. 

K579;  NN93;  111273, 

JJJ195;  WWW253; 

QQQQ466;  AAAAA229; 

CCCCC311. 
K579. 

1263;  K579;  JJJ195. 
K579. 
R72;  G063;  HH165; 

000146. 
W262;  CC312;  GG184-5; 

FFF14;  JJJ118; 

RRRR150;  VVVV229. 
CC312;  GG184-5;  FFF14; 

JJJ118;   RRRR150; 

UUUU298;   VVVV229; 

YYYY154. 
RRRR150;  UUUU298; 

VVVV231;  YYYY354. 
S164;  NNN277. 
W269. 
A163-4. 

AAA178;  TTT30. 
C98. 

A161 ;  C98. 
A75;  DD17;  BBB9; 

DDD84-5. 
A 354;  J313;  U66. 
E105;  F202;  J313. 
A354;  F202. 
V141-2. 
E104. 


^1)4 


C.  P.  A.  ACCOUNTING 


45. 

A359;  C168. 

57. 

YYYY71;  AAAAA230. 

46. 

FF228;  11161;  ZZZZ143. 

58. 

C313;  CC18;  ZZ16;  JJJ106 

47. 

C166. 

NNNN174. 

48. 

A354. 

59. 

W234;  FFF135;  RKR132. 

49. 

C167;  II161. 

60. 

CCCCC74. 

50. 

C170;  GG318. 

61. 

1341;  W230;  XX4]5; 

51. 

A359-60;  GG318. 

NNN314;  WWWISI. 

52. 

O20;  Z40;  II162;  XXXX220. 

6i 

AA63. 

53. 

C314;  F129;  S76;  DD57; 

63. 

CC361. 

GG231;  JJ356;  KK45; 

64. 

A102;  A364. 

NN278;  0026;  PP17; 

65. 

C316;  W224. 

QQ14;  RR255;  VV201; 

66. 

YYYY71 ;  ZZZZ225. 

WW276;  AAA177; 

67. 

W223. 

EEE269;  NNN233-5; 

68. 

RR109. 

TTT29;UUU111. 

69. 

J17;  UU36-7;  ZZ39. 

54. 

B348;    E436;    G231;    W235; 

70. 

J176-8;  SS-243;  RRR122-3. 

CC339;  HH40;  SSS55; 

71. 

1326-8;  RR 235-7. 

WWW184-5. 

72. 

AVW156-7. 

55. 

CCCCC23. 

73. 

RR234;  WW67. 

5G. 

PP524;  NNN256; 

74. 

RR234. 

AAAAA230. 

75. 

PPP8. 

REFERENCES  FOR  CHAPTER  XV 


1.  R114;  R127-8;  W236;  CC218; 

HH43;  SS297;  DDD143; 
CCCCIOI. 

2.  G346;  Hr21;  MM62; 

UU319. 

3.  K113;  MM61;  SS298. 

4.  K115;  MM63. 

5.  G347;  K137;  CC218;  JJ295; 

MM124;  NN209; 
WWW224;  DDDDD315. 

6.  DD30. 

7.  A404;    E371;    G350;    K121; 

CC220-1;  QQ47;  SS305-6; 
UU319;  AAA136;  DDD146; 
WWW223;  TTTT165-6. 

8.  K121;  SS306. 

9.  K123;  MM29. 

10.  K124;  EE69. 

11.  G352;  K125. 

12.  P106;  V65;  EE69. 


13. 
14. 
15. 
16. 
17. 
18. 
19. 
20. 
21. 

22. 
23. 


24. 

25. 
26. 


A225;  E372. 

A225;  G352;  TTTT150. 

K129. 

K311. 

G183. 

K104. 

K115;  MM63. 

MM63-4. 

G349;  R117;  MM25; 

HHH326. 
R118. 
H126;  CC218;  EE74;  Hn43; 

AAA13S-9;  DDD151-2; 

PPP119. 
K187;  N90;  R176;  DD30; 

AAA138-9;  DDD151. 
E408;  HH232. 
B359;   L191;   W289;    CC260; 

HH284;  LL134;  00325; 

SS233;    FFr60:   RRRIOO; 

SSS56. 


REFERFCNCES  FOR  CHAPTERS 


365 


27.  IVm);    G14fi;    H126-7;    134; 

J218;  014:^;   H25();  CCfj?; 
DDSf);  EETfr.   FF 197 
aG221;  HHl.l;  I J 233 
KK32;  lAMU;  KIt253 
SS388;  UU384;  VV152; 
W\V215;  XX230;  BBB25; 
1)1)1)40;  EEP:2G8; 
FFF109;   IIHH134; 
lH28fi;  JJJ52;  TTT2G; 
1)DDDD224. 

28.  R120. 

29.  N89;  P497;  MM55;  DDD153. 

30.  P499;  MM5f). 

31.  NN113. 

32.  MM50-1;  DDD152. 

33.  MM 50. 

34.  MM57;  DDD152. 

35.  H126;  CC218;  EE74;  HH43; 

AAA138-9;  DDD151-2; 
PPP119. 

36.  R120. 

37.  A421;A424;H142;  0129-30; 

PP146. 

38.  G353-4;  R225;  MM39; 

PP391. 

39.  G354;  K192;  MM40;  ITI180; 

TTTT169. 

40.  G355;  K193;  MM42; 

TTTT169. 

41.  G354;  MM46-7;  TTTT169. 

42.  K146. 

43.  K150;   K152;   K157;    K161; 

K169. 

44.  G358 ;  K152 ;  P511 ;  HHH322. 

45.  D167;    H128;    J186;    K152 

O130;  P511;  R119;  W243 
CC219;  GG179;  II200 
MM134;  QQ49;  DDD148 
HHH322;  III171;  JJJ168 
VVV409;  WWW227; 
EEEE192;  HHHH407; 
DDDDD310. 

46.  K175. 

47.  H128;  J187;  P512;  MM123; 

DDD148;  HHH322; 
111171;  VVV410. 


48. 

49. 
50. 
51. 
52. 
53. 
54. 
55. 
56. 

57. 

58. 
59. 


60. 
61. 
62. 
63. 


64. 


65. 


66. 

67. 
68. 
69. 
70. 
71. 


72. 
73. 


P512;      MM123;      DDD149; 

111111322. 
G363;  J187;  V67;  DDD151. 
1II171. 

H128;  DDD151;  VVV410. 
G363;  K154;  QQ48;  III172. 
K177. 

G363;   III172. 
G363. 
K155;  CC230;  MM23; 

III177. 
G368;  CC229-30;  JJJ169. 
K199. 
G364;    K150;    CC220; 

DDD150-1;  III172; 

CCCCC71. 
K178. 
G364. 
CCCCC71. 
A406;  D167-8;  J188;  K157; 

O130;  R119;  CC219; 

GG179;  II200-1;  JJ271; 

MM139 ;  QQ49-50 ;  DDD149 ; 

HHH322;  I1I173;  PPP118; 

VVV410;  EEEE192; 

HHHH407. 
H129;  J 188;  K157;  MM141; 

NN430;  VVV411; 

WWW228;  XXX85; 

MMMM247;  XXXX319; 

DDDDD310. 
J189;    G361;    K179;    R121; 

W243;    HHH323;    III174-, 

PPP118-9;  VVV411; 

WWW227-8;  EEEE192. 
G360;  K179;  W243;  JJ272; 

DDD149;  VVV411. 
H131;  DDD149. 
K179. 

DDD149;  VVV412. 
HHH324. 
G362;  K159;  CC229;  JJ272; 

SS322;  JJJ169;  SSS.S487; 

DDDDD311. 
K179;  JJ272. 
K179. 


366 


C.  P.  A.  ACCOUNTING 


74. 

P515;  W244;  AA180; 

90. 

K182;  MM129;   SS316. 

JJ268-9;  MM145-6;  QQ50; 

91. 

YY61. 

SS312-3;  YY63;  HHH324; 

92. 

G364-5;  O130;  IIII353. 

III175;  EEEE193. 

93. 

G365. 

75. 

G366-7;  P516-7;  HHH325. 

94. 

H134;  DDD151. 

76. 

K161;  W243;  MM129; 

95. 

G369;  K169. 

SS312;  III177. 

96, 

K182. 

77. 

K161. 

97. 

K169;  III167. 

78. 

P513;    SS313. 

98. 

K183. 

79. 

D168-9;   G367;  K181; 

99. 

III167. 

MM146;    SS.313;   III176. 

100. 

K183. 

80. 

MM130. 

101. 

K170;  K183. 

81. 

A407. 

102. 

K183. 

82. 

P517. 

103. 

K170;  0131;  R119;  HH119; 

83. 

D167;  CC219;  GG180;  II201; 

QQ49;   SS325;  III166. 

DDD149. 

104. 

K183. 

84. 

G364;K163;  XXXX322. 

105. 

K184;  NN321;  SS325;  III166. 

85. 

K163;  MM152;  QQ57;  YY61; 

106. 

G362. 

VVV412;  HHHH407; 

107. 

R121. 

EEEE193. 

108. 

K184. 

88. 

DDDDD312. 

109. 

K170;  MM77. 

87. 

K165. 

110. 

K185. 

88. 

SS315-6;  YY62;  XXXX320; 

111. 

K184. 

DDDDD312. 

112. 

P510. 

89. 

K181. 

REFERENCES  FOR  CHAPTER  XVI 


1.  F222;  HH48;  YV202; 

EEE219;  ZZZ177. 

2.  HH48. 

3.  D195;  H293-5;  J317;  II194; 

VV209-10;  WW235;  UUU5. 

4.  F225;   H305;  J295;   SIO; 

NN270;  0022;  YY200; 
WW236;  CCCl-2;  DDD99; 
EEE274;  NNN20;  TTT176; 
UUU16. 

5.  H296;  J295;   CC199;  II201; 

PP19-20. 

6.  PP17. 

7.  F225-6;  P615;  W203;  0024; 

EEE274. 

8.  F226;    P616;    W203;    0025; 

EEE274;  NNN22. 


9.    G246;  H296;  J215;  Q90; 
W203;  NN270;  0032; 
PP19;  000113;  RRR33; 
TTT176;  UUU16;  YY V344. 
JO.    H301;  0026;  PP17;  YY201; 
WW239;  EEE371-2. 

11.  NNN3;  ZZZ177. 

12.  0021;  PP5. 

13.  C177;  EEE240;  EEE249. 

14.  00127-8;   PP308-9. 

15.  GG46;    00123-4;    PP315. 

16.  GG203;    PP306;    YY224. 

17.  EEE222;  EEE236;  NNN64. 

18.  VV251;  WW282. 

19.  0046;  PP29-30;  TTT189; 

ruui2. 

20.  F228-9;  0046;  PP24-5; 

WW235;  TTT186-7; 
UUU13. 


REFERENCES  FOR  CHAPTERS 


367 


21. 

F224;  0046. 

48. 

22. 

0046. 

23. 

0046. 

24. 

E183-4;   0047. 

49. 

25. 

H301;  P617;  0061-2; 

50. 

VV219-20. 

51. 

26. 

H300;  VV220;  \YW275. 

27. 

H302. 

28. 

EE261;    0066-7;    VV217. 

29. 

0047-8. 

30. 

F249-50;  PP29. 

31. 

R223;  EEE320;  EEE334. 

32. 

G176;  II34;  VV236. 

33. 

H311;  J309;  R223-4;  GG273; 
PP246;  QQ17;  EEE334. 

52. 

54. 

J311;  R223;  VV225; 
EEE386. 

53. 

35. 

G176;  S31;  P122;  GG273; 
II34;  EEE320. 

36. 

PP86. 

37. 

H312;  PP83;  VV255; 

NNN135;  TTT178;  UUU34. 

54. 

38. 

J312;  EEE222;  EEE235-6. 

39. 

E177;  H311. 

40. 

GG203;  WW250. 

41. 

E177;  GG273;  PP79;  QQ19; 
VV256. 

42. 

C25;    G172;    S31-2;    GG273; 

r  jT 

TTT177;  DDDDD70. 

oD. 

43. 

S49-50;  WW262. 

56 

44. 

CC389;  0071-2;  PP99; 
VV212;  WW237;QQQ305; 
TTT179;   UUU196-7; 
KKKK423-4. 

tJyJt 

45. 

CC389;  0072-4;  PPIOO; 
VV212;  WW237;  QQQ305; 

TTT179;  UUU197-8; 

57. 

KKKK424-5. 

58. 

46. 

CC389;  0074-5;  PP103; 

59. 

VV212;  CCC295;  QQQ305; 

60. 

TTT179;   UUU200-1; 

61. 

KKKK427. 

47. 

CC389;  0075;  PP104; 

62. 

VV212;  CCC295; 

63. 

EEE297-8;  QQQ305; 

64. 

TTT180;UUU202; 

65. 

KKKK425-6. 

66. 

CC389;  0075-6;  PP104; 

VV212;  QQQ305;  TTT180; 

KKKK426. 
0077-8;  PP105. 
IIII328. 
E182;  F231;  H297;  J316; 

0193;  S62-3;  CC205-6; 

EE258;  FF285;  HH50; 

JJ352;  0055-6;  PP169-71; 

VV214;  WW239;  CCC305; 

EEE373;  NNN224; 

TTT183-4;  UUU272-5; 

GGGG365-6. 
11297;  J315-6;  EE258-9; 

FF285;  UUU281. 
F232;  H297;  J316;  S63; 

CC205-6;    EE259;    HH50; 

0058;  PP166-9; 

VV214-5;   EEE373; 

TTT185;  UUU281; 

GGGG365. 
F231-2;    H297;    0193;    S64; 

CC205-6;  EE258;HH50; 

JJ353;    0056-7;    PP171-3 

VV^214;  WW240;  CCC310; 

EEE373;NNN223; 

TTT184-5;  UUU275-7; 

GGGG366;  GGGG367. 
H297;  0192-3;  HH50; 

EEE372-3;   GGGG364-5. 
E183;  F232-3;  H297;   0192; 

S65-6;  CC205-6;  EE260; 

HH50;  JJ358;  0059-67; 

PP173-7;  VV215;  WW241; 

EEE373-4;  NNN224; 

TTT185;  UUU277-9; 

GGGG367. 
FF283;  LL214. 
F216. 

F216-7;  L52. 
F217;  L52. 
F240;  L53;  WW285; 

NNN345. 
L157;   CC204;   GG205. 
CC203-4;  GG205;  DDD99. 
C33. 

A92;  HHHH413. 
A92;  C37. 


UNIVERSITY  OF  CALIFORNIA  LIBRARY 

Los  Angeles 
This  book  is  DUE  on  the  last  date  stamped  below. 


Form  L9-32m-8,'58(5876s4)444 


Library' 
Graduate  Sch  )0l  of  Business  Adrainistratiofl 
University  of  California 
,  .      Los  Angeles  24,  California 


A     000  177  614 


SOUTHERN  BRANGrt; 

UNIVIRSIIY  Of  CALIFORNIA 

.  LIBRARY, 
llJDS  ANGELES,  CALIF.